You are on page 1of 382

MATEMTICAS I

Expresiones Numricas y Algebraicas.


Polinomios. Lgica, Conjuntos y Relaciones.
Aplicaciones, Funciones Numricas y sus
Grcas. Nmeros Reales. Ecuaciones e
Inecuaciones de Primer Grado. Matrices,
Determinantes y Sistemas Lineales.
Trigonometra I.

CURSO DE MATEMTICAS BSICAS PARA


USO DE PROFESORES Y ESTUDIANTES

Danilo Gortaire Jtiva

Ediciones Nacionales Unidas, EDINUN

Quito, Ecuador. 2007


Resumen

El libro que el lector tiene en sus manos, MATEMTICAS I, representa un Curso de Matemticas
Bsicas para uso de Profesores y Estudiantesy viene a ser la primera parte de una propuesta moderna y
seria de texto en 3 tomos para usarlo en el proceso de enseanza - aprendizaje de la matemtica en los 3 ltimos
aos de educacin media. Tiene como finalidad fundamental la de convertirse en una gua de ayuda para
profesores y alumnos , tanto de los conceptos tericos y prcticos sobre los temas: expresiones numricas y
algebraicas (repaso del material de 10mo de bsico); polinomios; lgica y conjuntos; relaciones, funciones y sus gr
cas; nmeros reales; desigualdades, ecuaciones e inecuaciones de primer grado (algebraicas, racionales,
irracionales, con valor absoluto, etc.); matrices, determinantes y sistemas de ecuaciones lineales; trigonometra I;
as como de los mtodos y tcnicas de resolucin de problemas. Sealemos las particularidades del curso:

El texto viene ilustrado con muchsimos cuadros - resmenes de la teora, ejemplos y ejercicios propuestos,
grcas matemticas y fotos a todo color.

La segunda innovacin es en su exposicin. Cada captulo inicia con sus unidades de competencia e
indicadores de logro para el estudioso, la introduccin al tema y
nalmente su desarrollo a 3 niveles. El curso est diseado para trabajar en las distintas especialidades o
niveles: bsico, intermedio y superior.

Para tal efecto, para la comprensin del material del nivel bsico, no se exige la lectura de los materiales
de los niveles intermedio y superior; para la comprensin del material del nivel medio, no se exige la
lectura del material del nivel superior. El texto presentado con una barra vertical (de color rojo) en el
margen exterior, corresponde al nivel superior; el texto presentado con una barra vertical (de color azul)
corresponde al nivel intermedio; la parte restante del texto corresponde al nivel bsico.

Las particularidades anteriores no son, ni deben ser una camisa de fuerza para el profesor, ni para
el alumno, sino que de acuerdo a la disponibilidad de tiempo (carga horaria semanal), especialidad u otras
caractersticas del curso y alumnado, se pueden hacer variaciones o ampliaciones de los contenidos del curso.
Adems una exposicin de este tipo, es decir, a 3 niveles, permitir que muchos estudiantes amplen y desarrollen
sus conocimientos matemticos y curiosidades, e investiguen otros tpicos, que pueden no pertenecer a su
especialidad. Esto permitir en parte una nivelacin de conocimientos matemticos.

El curso complementa la bibliografa existente sobre estos temas y cuestiones pertenecientes a la matemtica
bsica, tan necesarias en la enseanza - aprendizaje de la matemtica a nivel de colegio y preuniversitario.
Entre otros propsitos del curso, se tiene el de capacitar al lector, profesor o estudiante, en el planteo, manejo,
comprensin, clculo y aplicacin adecuados de los mtodos de anlisis de ecuaciones, desigualdades, sistemas
lineales, etc., basados en la utilizacin de transformaciones lgicas y numricas equivalentes, as como el uso
apropiado de las aplicaciones y de los conceptos fundamentales del conjunto de los nmeros reales y de las
relaciones y funciones.

El texto, para una lectura ms amena, va acompaado de muchas ilustraciones matemticas, fotos de clebres
matemticos con sus respectivas minibiografas y otras motivaciones. Posee 2 anexos (breve cronologa de las
matemticas, estructuras matemticas, resumen de frmulas bsicas, glosario de trminos matemticos).
ndice general

Prefacio 1

1. PRELIMINARES ARITMTICOS 8

1.1. Nmeros Naturales y sus Operaciones . . . . . . . . . . . . . . . . . . . . . . . . . . . . . . . . . 9

1.1.1. Operaciones con los naturales . . . . . . . . . . . . . . . . . . . . . . . . . . . . . . . . . . 10

1.1.2. Mximo comn divisor (m.c.d.) y Mnimo comn mltiplo (m.c.m.) . . . . . . . . . . . . 16

1.2. Nmeros Enteros y sus Operaciones . . . . . . . . . . . . . . . . . . . . . . . . . . . . . . . . . . 20

1.3. Nmeros Racionales y sus Operaciones . . . . . . . . . . . . . . . . . . . . . . . . . . . . . . . . 22

1.3.1. Fracciones. Fracciones propias, impropias y mixtas . . . . . . . . . . . . . . . . . . . . . . 22

1.3.2. Operaciones con fracciones. Fracciones decimales . . . . . . . . . . . . . . . . . . . . . . . 25

1.4. Introduccin a los Nmeros Reales y sus Operaciones . . . . . . . . . . . . . . . . . . . . . . . . . 31

1.4.1. Nmeros irracionales y sus operaciones . . . . . . . . . . . . . . . . . . . . . . . . . . . . 31

1.4.2. Intervalos numricos. Valor absoluto de un nmero real . . . . . . . . . . . . . . . . . . . 36

1.5. Introduccin a los Nmeros Complejos . . . . . . . . . . . . . . . . . . . . . . . . . . . . . . . . 41

1.5.1. Operaciones con los Nmeros Complejos. Representacin Geomtrica . . . . . . . . . . . . 44

1.5.2. Nmeros Complejos Conjugados. Operaciones . . . . . . . . . . . . . . . . . . . . . . . . 48

1.5.3. Potenciacin de Nmeros Complejos . . . . . . . . . . . . . . . . . . . . . . . . . . . . . . 49

2. EXPRESIONES RACIONALES E IRRACIONALES. POLINOMIOS 55

2.1. Expresiones Algebraicas . . . . . . . . . . . . . . . . . . . . . . . . . . . . . . . . . . . . . . . . . 56

2.1.1. Tipos de expresiones algebraicas . . . . . . . . . . . . . . . . . . . . . . . . . . . . . . . . 56

2.1.2. Conjunto de valores admisibles c:v:a: de una expresin algebraica . . . . . . . . . . . . . . 56

2.2. Transformacin y Simplicacin de Expresiones Racionales . . . . . . . . . . . . . . . . . . . . . 58

2.2.1. Monomios y polinomios . . . . . . . . . . . . . . . . . . . . . . . . . . . . . . . . . . . . . 58

2.2.2. Productos y cocientes notables, factoreo . . . . . . . . . . . . . . . . . . . . . . . . . . . 60

2
NDICE GENERAL 3

2.2.3. Descomposicin de polinomios en factores . . . . . . . . . . . . . . . . . . . . . . . . . . . 65

2.2.4. Expresiones racionales fraccionarias. Operaciones con fracciones y su simplicacin . . . 74

2.3. Estudio Posterior de los Polinomios de una Variable . . . . . . . . . . . . . . . . . . . . . . . . . 82

2.3.1. Operaciones con polinomios y su descomposicin en factores . . . . . . . . . . . . . . . . . 85

2.3.2. Propiedades de la divisibilidad de polinomios . . . . . . . . . . . . . . . . . . . . . . . . . 88

2.4. Transformacin y Simplicacin de Expresiones Irracionales . . . . . . . . . . . . . . . . . . . . . 98

3. NOCIONES DE LGICA, CONJUNTOS Y RELACIONES 103

3.1. Nociones de Lgica . . . . . . . . . . . . . . . . . . . . . . . . . . . . . . . . . . . . . . . . . . . . 104

3.1.1. Proposiciones y conectivos. Tablas de verdad. lgebra de proposiciones . . . . . . . . . . 104

3.1.2. Leyes del lgebra de proposiciones . . . . . . . . . . . . . . . . . . . . . . . . . . . . . . . 106

3.1.3. Juicio y razonamiento. Paradojas . . . . . . . . . . . . . . . . . . . . . . . . . . . . . . . . 106

3.1.4. Funcin proposicin o predicado. Cuanticadores . . . . . . . . . . . . . . . . . . . . . . . 108

3.1.5. Retculos. lgebra de Boole . . . . . . . . . . . . . . . . . . . . . . . . . . . . . . . . . . . 119

3.2. Nociones sobre la Teora de Conjuntos . . . . . . . . . . . . . . . . . . . . . . . . . . . . . . . . . 125

3.2.1. Conjuntos y sus operaciones . . . . . . . . . . . . . . . . . . . . . . . . . . . . . . . . . . . 125

3.3. Aplicaciones, Correspondencias y Relaciones entre Conjuntos . . . . . . . . . . . . . . . . . . . . 136

3.3.1. Tipos de aplicaciones y operaciones con aplicaciones . . . . . . . . . . . . . . . . . . . . . 138

3.3.2. Relaciones de equivalencia y de orden . . . . . . . . . . . . . . . . . . . . . . . . . . . . . 144

4. APLICACIONES, FUNCIONES NUMRICAS Y SUS GRFICAS 156

4.1. Concepto de Funcin . . . . . . . . . . . . . . . . . . . . . . . . . . . . . . . . . . . . . . . . . . . 157

4.1.1. Clasicacin simple de las aplicaciones . . . . . . . . . . . . . . . . . . . . . . . . . . . . 158

4.2. Construccin de Grcas de Funciones . . . . . . . . . . . . . . . . . . . . . . . . . . . . . . . . 169

4.2.1. Construccin de Rectas y Parbolas . . . . . . . . . . . . . . . . . . . . . . . . . . . . . . 171

4.3. Prcticas con las Grcas de Algunas Funciones y Relaciones . . . . . . . . . . . . . . . . . . . . 182

5. NMEROS REALES 189

5.1. El Cuerpo de los Nmeros Reales . . . . . . . . . . . . . . . . . . . . . . . . . . . . . . . . . . . . 190

5.1.1. Axiomas y propiedades generales del conjunto de los nmeros reales . . . . . . . . . . . . 190

5.2. Mtodo de Induccin Completa . . . . . . . . . . . . . . . . . . . . . . . . . . . . . . . . . . . . . 202

6. IGUALDADES Y DESIGUALDADES NUMRICAS 209


4 NDICE GENERAL

6.1. Conceptos Bsicos . . . . . . . . . . . . . . . . . . . . . . . . . . . . . . . . . . . . . . . . . . . . 210

6.1.1. Deniciones y propiedades elementales de las desigualdades . . . . . . . . . . . . . . . . . 210

6.1.2. Leyes de los signos . . . . . . . . . . . . . . . . . . . . . . . . . . . . . . . . . . . . . . . . 212

6.2. Suma y Diferencia de Desigualdades Numricas . . . . . . . . . . . . . . . . . . . . . . . . . . . 214

6.3. Producto de Desigualdades Numricas . . . . . . . . . . . . . . . . . . . . . . . . . . . . . . . . . 216

6.3.1. Potenciacin y Radicacin de Desigualdades Numricas . . . . . . . . . . . . . . . . . . . 219

6.3.2. Ejemplos de algunas desigualdades numricas clsicas . . . . . . . . . . . . . . . . . . . . 221

7. ECUACIONES E INECUACIONES 223

7.1. Conceptos Generales . . . . . . . . . . . . . . . . . . . . . . . . . . . . . . . . . . . . . . . . . . . 224

7.2. Equivalencia entre Desigualdades . . . . . . . . . . . . . . . . . . . . . . . . . . . . . . . . . . . . 227

7.3. Ecuacin Lineal o de Primer Grado . . . . . . . . . . . . . . . . . . . . . . . . . . . . . . . . . . . 231

7.4. Problemas de Planteo con Ecuaciones de Primer Grado . . . . . . . . . . . . . . . . . . . . . . . 234

7.5. Inecuaciones Lineales . . . . . . . . . . . . . . . . . . . . . . . . . . . . . . . . . . . . . . . . . . . 245

7.6. Mtodo de los Intervalos . . . . . . . . . . . . . . . . . . . . . . . . . . . . . . . . . . . . . . . . . 249

7.7. Solucin de Algunas Ecuaciones e Inecuaciones Irracionales . . . . . . . . . . . . . . . . . . . . . 253

7.8. Inecuaciones con dos Incgnitas y su Interpretacin Geomtrica . . . . . . . . . . . . . . . . . . . 256

7.9. Conjuncin y Disyuncin de Desigualdades Numricas . . . . . . . . . . . . . . . . . . . . . . . . 258

7.9.1. Simplicaciones y Observaciones . . . . . . . . . . . . . . . . . . . . . . . . . . . . . . . . 261

7.10. Construccin de Desigualdades . . . . . . . . . . . . . . . . . . . . . . . . . . . . . . . . . . . . . 264

7.10.1. Propiedades de las Construcciones de Desigualdades e Igualdades . . . . . . . . . . . . . . 267

7.10.2. Construccin de Ecuaciones y su Equivalencia . . . . . . . . . . . . . . . . . . . . . . . . 270

7.11. Inecuaciones Algebraicas con una Incgnita . . . . . . . . . . . . . . . . . . . . . . . . . . . . . . 271

7.11.1. Solucin de Conjunciones de desigualdades . . . . . . . . . . . . . . . . . . . . . . . . . . 272

7.11.2. Solucin de Disyunciones de Desigualdades . . . . . . . . . . . . . . . . . . . . . . . . . . 273

7.11.3. Solucin de Inecuaciones Racionales . . . . . . . . . . . . . . . . . . . . . . . . . . . . . . 275

7.12. Introduccin a las Inecuaciones Irracionales . . . . . . . . . . . . . . . . . . . . . . . . . . . . . . 276

7.12.1. Solucin de algunas Inecuaciones Irracionales (con un solo radical de ndice par) . . . . . 276

7.13. Ecuaciones e Inecuaciones con Valor Absoluto . . . . . . . . . . . . . . . . . . . . . . . . . . . . 278

7.13.1. Solucin de Ecuaciones y Inecuaciones con Valor Absoluto . . . . . . . . . . . . . . . . . . 279

7.13.2. Desigualdades ms Generales con dos Variables y Valor Absoluto . . . . . . . . . . . . . . 282


NDICE GENERAL 5

8. MATRICES, DETERMINANTES Y SISTEMAS LINEALES 288

8.1. Elementos de la Teora de Matrices . . . . . . . . . . . . . . . . . . . . . . . . . . . . . . . . . . . 289

8.1.1. Tipos de matrices . . . . . . . . . . . . . . . . . . . . . . . . . . . . . . . . . . . . . . . . 289

8.1.2. Operaciones con matrices . . . . . . . . . . . . . . . . . . . . . . . . . . . . . . . . . . . . 291

8.2. Sistemas de Ecuaciones Lineales . . . . . . . . . . . . . . . . . . . . . . . . . . . . . . . . . . . . 295

8.2.1. Sistemas de ecuaciones lineales m n. Mtodo de reduccin de Gauss. . . . . . . . . . . . 295

8.2.2. Sistemas de ecuaciones lineales n n. Mtodo de los determinantes de Cramer. . . . . . . 299

8.3. Aplicacin: Descomposicin en Fracciones Parciales . . . . . . . . . . . . . . . . . . . . . . . . . 308

8.3.1. Fracciones racionales y su descomposicin en fracciones parciales . . . . . . . . . . . . . . 310

9. TRIGONOMETRA I 319

9.1. Introduccin y Funciones Trigonomtricas . . . . . . . . . . . . . . . . . . . . . . . . . . . . . . . 320

9.1.1. Concepto de ngulo y su medicin . . . . . . . . . . . . . . . . . . . . . . . . . . . . . . . 320

9.1.2. Denicin de las funciones trigonomtricas . . . . . . . . . . . . . . . . . . . . . . . . . . 323

9.1.3. Signos de las funciones trigonomtricas . . . . . . . . . . . . . . . . . . . . . . . . . . . . 325

9.1.4. Variacin de las funciones trigonomtricas para 2 [0; 2 [ . . . . . . . . . . . . . . . . . . 325

9.1.5. Construccin de un ngulo conociendo el valor dado de cierta funcin trigonomtrica . . . 327

9.2. Funciones Trigonomtricas de Ciertos ngulos Notables. Grcas . . . . . . . . . . . . . . . . . . 329

9.2.1. Dependencia entre las funciones trigonomtricas de cierto ngulo . . . . . . . . . . . . . 330

9.2.2. Identidades y ejemplos varios . . . . . . . . . . . . . . . . . . . . . . . . . . . . . . . . . . 332

9.2.3. Funciones trigonomtricas de argumento negativo y su reduccin . . . . . . . . . . . . . . 335

9.2.4. Frmulas de reduccin. Funciones trigonomtricas y sus grcas . . . . . . . . . . . . . . 337

A. RESUMEN DE ALGUNAS FRMULAS 353

A.1. lgebra . . . . . . . . . . . . . . . . . . . . . . . . . . . . . . . . . . . . . . . . . . . . . . . . . . 353

A.2. Geometra Analtica . . . . . . . . . . . . . . . . . . . . . . . . . . . . . . . . . . . . . . . . . . . 355

A.3. Trigonometra . . . . . . . . . . . . . . . . . . . . . . . . . . . . . . . . . . . . . . . . . . . . . . . 355

B. ESTRUCTURAS. SMBOLOS. CRONOLOGA. GLOSARIO 358

B.1. Estructuras Algebraicas . . . . . . . . . . . . . . . . . . . . . . . . . . . . . . . . . . . . . . . . . 358

B.2. Algunos Smbolos Matemticos . . . . . . . . . . . . . . . . . . . . . . . . . . . . . . . . . . . . . 362

B.3. Principales Jalones en la Historia de la Matemtica . . . . . . . . . . . . . . . . . . . . . . . . . 364

B.4. Glosario de Trminos Matemticos Elementales . . . . . . . . . . . . . . . . . . . . . . . . . . . 365


PREFACIO

Instantia est mater doctrinae. Repetitio est mater studiorum. (Nam) Sine doctrina vita est quasi mortis imago.

El olvido de las matemticas perjudica a todo el conocimiento, ya que el que

las ignora no puede conocer las otras ciencias ni las cosas de este mundo.

Roger Bacon

La matemtica moderna est construida sobre la base de la lgica y teora de conjuntos, concepto de funcin, nmeros
reales y otras estructuras. Han cambiado sus contenidos y forma de ensearla, han aparecido nuevos cursos, necesidades
y aplicaciones y se han creado nuevos programas de matemticas aplicados a otras especialidades: administrativas, infor-
mticas, nancieras, estadsticas, fsicas, del medio ambiente, electrnicas, econmicas, etc. Un curso para bachillerato
debe ensear temas fundamentales que abran las puertas al estudio posterior de la matemtica superior y no las cosas
tan trilladas y cansadas como la manipulacin y simplicacin de identidades escritas en mil formas.

Un curso de matemticas no puede ser una obra verdaderamente original, el lector encontrar temas ya tratados en
obras magnficas, es muy semejante a la interpretacin de una obra musical, que como siempre tiene y tendr muchos
intrpretes, para cada uno de los cuales es importante uno u otro lado de la interpretacin musical. El curso reune experiencias
con mis estudiantes en el proceso de enseanza - aprendizaje de la materia para tratar de resolver el dilema de saber si el
profesor es el bueno y los estudiantes son los malos o viceversa; o llegar a ese equilibrio que se asemeja a una curva
normal Gaussiana, que nos dice que en condiciones de estabilidad natural hay una minora de estudiantes mal aprovechados y
otra con conocimientos sobresalientes y una mayora de estudiantes con buenos conocimientos, pero dnde estn esas
fronteras, y como conseguirlas? Quizs no hay cierta lgica secuencial con los temas, pero es preferible comenzar con lo ms
simple (la axiomtica de los nmeros reales est muy lejos de ser simple por su contenido y para su asimilacin como posible
1er captulo). En la escuela aprend que el rea del crculo es A = r2 y solamente en la universidad conoc su demostracin.
He aqu la moraleja de que los profesores de matemticas debemos escribir libros de matemticas no tanto para estudiantes de
matemticas, sino ms para los no matemticos, es decir, que para no quejarnos de lo poco que saben de matemticas nuestros
estudiantes es necesario popularizar esta ciencia hacia los necesitados.

0
Motivacin y Metodologa. La matemtica (del griego { de que signica conocimiento,
ciencia y actualmente, tambin demostracin) como expresin de la mente humana, es la ciencia de los fundamentos

6
NDICE GENERAL 7

que trata las estructuras, formas, magnitudes y relaciones numricas de conguraciones del pensamiento (teniendo o
no en cuenta su signicado real), reeja la voluntad y el deseo de perfeccin. Organiza los hechos y las cosas de un
orden general, desarrolla el espritu constructivo, llega a la verdad y a la originalidad del ser. Sus elementos bsicos
son: lgica e intuicin, generalidad y particularidad. Esas fuerzas opuestas y la lucha por su sntesis constituyen la
vida, la utilidad y el supremo valor de esta ciencia. Es un rgano indispensable de la cultura, porque logra armonizarse
libremente a la norma, a la regla, a la ley. Tiene una consecuencia didctica: la enseanza tiene que encararse con un
doble criterio, instrumental y formativo, y desde un campo ms amplio que el estrictamente matemtico. Para mejorar su
enseanza en todos los niveles escolares, se ha considerado: Qu la matemtica es ms que una asignatura de enseanza,
es una disciplina cultural en su ms extenso sentido. Qu es un mtodo de investigacin, cuerpo de conocimientos,
principios y conceptos. Qu es una ciencia bsica, es decir, un sistema de conocimientos que permite comprender los
valores fundamentales que se relacionan con los conceptos. Qu se la debe organizar y ensear con el n de ofrecer
a los estudiantes experiencias vitales, que le sirvan para resolver problemas y comprender la naturaleza. Esto incluye
otras habilidades intelectuales especcas, como interpretar datos y sacar conclusiones. Qu esta ciencia contribuye
a desarrollar capacidades para la adquisicin del saber o para la aprehensin del saber como acto de esfuerzo y de
conquista. La metodologa del curso se desarrolla en forma ordenada y sistemtica hasta llegar al ncleo central que
son los nmeros reales, funciones, ecuaciones e inecuaciones de variados tipos. Sus captulos corresponden al programa,
secuencia y motivacin de estudios del bachillerato. Se precisan los conceptos, propiedades y teoremas como criterio
necesario para el conocimiento matemtico. Los ejemplos y ejercicios (practico lo que aprend) son para asimilar los
conceptos expuestos y para ayudar al estudiante a incrementar su creatividad y clculo mental.

Uso del texto. El texto presenta 9 captulos principales y 2 anexos (ver ndice) con sus respectivos contenidos, 3
niveles de exposicin (sin barra, con barra azul y con barra roja), correspondientes a distintas especialidades y que de
acuerdo a las necesidades pueden ser tanto includas como excludas en el estudio y lectura. Los errores en el libro, as
como la exposicin de temas son responsabilidad del autor y agradecera su indicacin para futuras correcciones.

Agradecimientos 1 A quienes conozco 24, 23 y 17 aos: Svetlana Vasilievna, Mara Augusta Danilovna y Danilo
Danilovich, esposa, hija e hijo que me brindaron su comprensin y tiempo del hogar en la ejecucin de este trabajo.
Al Ing. Vicente Velsquez Guzmn, Gerente general de Ediciones Nacionales Unidas, EDINUN por su amable in-
vitacin para trabajar en la Editorial, por su inmenso apoyo, cuidado y gran inters puesto en la produccin de esta obra.
Al Lcdo. Juan Carlos Tqueres por su valiossima orientacin pedaggica.

Danilo Gortaire Jtiva Quito, Ecuador. Febrero, 2007.

Figura 1: El gran matemtico griego, Euclides (Alejandra, siglo III a. C.), una de las mentes matemticas ms brillantes
de todos los tiempos. Sus estudios geomtricos se han mantenido inclumes durante varios miles de aos y se siguen
estudiando en el bachillerato. Con el descubrimiento de las geometras no eucldeas (intentos de demostracin y negacin
del V Postulado de las paralelas) y la teora general de la relatividad que sostiene que el espacio universal no es del tipo
eucldeo sino curvo, la geometra euclideana dej de ser considerada como verdad absoluta para el espacio. Ahora se
le identica a Euclides con el smbolo de la enseanza de la matemtica a la antigua y con su respectivo estancamiento.
Captulo 1

PRELIMINARES ARITMTICOS

Los pueblos marchan al trmino de su grandeza

con el mismo paso que camina su educacin.

Simn Bolvar

El progreso y el perfeccionamiento de la Matemtica

estn ntimamente ligados a la prosperidad del Estado.

Napolen I

Unidad de competencia: Utilizar los conceptos de la aritmtica bsica, identicando y operando correctamente
con los conjuntos numricos y sus elementos con la nalidad de aplicarlos en la simplicacin de expresiones aritmticas
y algebraicas tanto racionales como irracionales.

Indicadores de logro:

| Identicar conjuntos numricos (naturales, enteros, racionales, irracionales, etc.) y operar con ellos correctamente.

| Reconocer los diferentes criterios de divisibilidad y aplicarlos a la descomposicin en factores primos.

| Comprender los algoritmos para el m.c.m. y m.c.d.

| Aplicar las propiedades fundamentales de las fracciones y sus distintas conversiones.

| Conocer, manipular y simplicar adecuadamente las distintas expresiones numricas racionales e irracionales.

| Operar con facilidad con los radicales.

| Interpretar, analizar e integrar los conceptos de intervalos numricos y valor absoluto.

| Conocer los conceptos bsicos y operaciones con los nmeros complejos.

Cierta persona se bebe un barril de sidra en 14 das, y conjuntamente con su esposa el mismo barril se lo toman en 10
das. Se necesita saber en cuntos das se lo beber su esposa estando sola.

En un da caluroso 6 labradores se tomaron un tonel de limonada en 8 horas. Se necesita saber, Cuntos labradores
se bebern ese mismo tonel en 3 horas?

8
1.1. NMEROS NATURALES Y SUS OPERACIONES 9

1.1. Nmeros Naturales y sus Operaciones

Los nmeros naturales N, aparecen como resultado de contar los objetos o elementos pertenecientes a ciertos
conjuntos nitos, es decir, al compararlos uno a uno con ciertos representantes o smbolos. Cada nmero natural
es el representante de la cantidad de elementos que puede tener un conjunto. Los naturales, tabulados en forma
creciente, tienen la forma N = f1; 2; 3; 4; 5; 6; 7; : : : ; n; : : :g y se denominan serie natural. De entre 2 naturales,
mayor se denomina a aquel que se encuentra posteriormente en la serie natural, as, por ejemplo, 7 es mayor
que 4. Dados 2 naturales a y b, a > b signica decir que a es mayor que b; y a < b signica que a es menor que
b:

La serie de nmeros naturales N posee las propiedades:

El menor nmero natural es el 1:


Despus de cada nmero natural, en la serie natural, le sigue un nico natural llamado siguiente.
La serie natural es innita.

En el sistema decimal de numeracin, un nmero cualquiera, por ejemplo el 3147, suele representarse en la
forma 3147 = 3 103 + 1 102 + 4 10 + 7, y decimos que tiene: 7 unidades, 4 decenas, 1 centena y 3 unidades de
mil. Para un nmero grande como 15 041 706 400 000, que se lee como 15 trillones, 41 billones (o millares), 706
millones, 400 mil; se tendra 0 unidades, 0 decenas, 0 centenas, 0 unidades de mil, 0 decenas de mil, 4 centenas
de mil, 6 unidades de milln, 0 decenas de milln, 7 centenas de milln, 1 unidad de billn, 4 decenas de billn,
0 centenas de billn, 5 unidades de trilln y 1 decena de trilln. Generalizando, cualquier natural n puede
representarse en la forma:

n = ak 10k + ak 1 10
k 1
+ + a1 10 + a0 ; (1)
donde k es natural y ai son las cifras del sistema decimal de numeracin, es decir, las cifras 0, 1, 2, 3, 4, 5, 6,
7, 8, 9, (i = 0; 1; 2; :::; k):

Con frecuencia y por comodidad, un nmero natural de 2 cifras lo representaremos mediante ab, uno de 3
cifras mediante abc, uno de 4 cifras con abcd, etc.

Ejemplos:

1. 3 es mayor que 1 se representa mediante 3 > 1. 7 es menor que 16 se representa mediante 7 < 16:

2. Utilizando la frmula (1), representar los nmeros naturales: a) 25, b) 308, c) 79061, d) 5432.

a) 25 = 2 101 + 5, es decir, a1 = 2, a0 = 5; k = 1. En este caso tenemos 2 decenas y 5 unidades.


b) 308 = 3 102 + 0 101 + 8, y a2 = 3, a1 = 0, a0 = 8; k = 2, con 3 centenas, 0 decenas y 8 unidades.
c) 79061 = 7 104 + 9 103 + 0 102 + 6 101 + 1, es decir, a4 = 7, a3 = 9, a2 = 0, a1 = 6, a0 = 1; k = 4.
En este caso tenemos 7 decenas de mil, 9 unidades de mil, 0 centenas, 6 decenas y 1 unidad.
d ) 5432 = 5 103 + 4 102 + 3 101 + 2, es decir, a3 = 5, a2 = 4, a1 = 3, a0 = 2; k = 3. En este caso
tenemos 5 unidades de mil, 4 centenas, 3 decenas y 2 unidades.

Practico lo que aprend:

1. Interpretar las relaciones: a) 3 < 12, b) 23 > 15, c) 300 < 1524, d) 578 < 579, e) 17 < 2512, f ) 300 > 299.

2. Utilizando la frmula (1), representar los nmeros naturales: a) 432, b) 2005, c) 1524, d) 578579, e)
172512, f ) 300299, g) 666137, h) 670554.
10 CAPTULO 1. PRELIMINARES ARITMTICOS

Observaciones: Aparte del sistema decimal (base 10) de numeracin, existen otros sistemas de numeracin:
binario (usa las cifras 0 y1), ternario (usa el 0, 1 y 2), sistema de base 8 (usa las cifras del 0 al 7), sistema de
base 16 (usa las 10 cifras del 0 al 9 y las 6 primeras letras del alfabeto), etc. As, por ejemplo, en el sistema
binario tendremos las equivalencias: 010 = 02 , 110 = 12 , 210 = 102 , 310 = 112 , 410 = 1002 , etc.

1.1.1. Operaciones con los naturales

Dados 2 nmeros naturales o componentes, se denomina operacin aritmtica a la bsqueda de un 3er


nmero natural (resultado de la operacin). En aritmtica se analizan 4 operaciones: suma (adicin), resta
(diferencia), multiplicacin (producto), divisin (cociente). Representaremos con a, b, c, etc. a nmeros naturales
cualesquiera.

Adicin. La operacin de suma de naturales es un concepto matemtico primitivo y formalmente no se


dene, pero consiste en que dados varios naturales llamados sumandos, se encuentra un natural llamado su
suma. Si a es el nmero de elementos del conjunto A y b es el nmero de elementos del conjunto B, entonces
mediante la suma de a y b se entiende al nmero c, que es el nmero de elementos de la unin de A y B. La
unin de los conjuntos A y B es un nuevo conjunto que contiene a todos los elementos de estos 2 conjuntos,
pero si en los conjuntos hay elementos idnticos, entonces stos entran en la unin una sola vez. La operacin
se representa mediante a + b = c.

La suma de varios naturales siempre existe y su resultado es nico, la adicin de naturales es una operacin
cerrada ya que no sale del conjunto N.

Diferencia. Es la operacin aritmtica mediante la cual, dada la suma de 2 componentes y uno de ellos,
se encuentra el otro. De acuerdo a esto, la suma de los 2 componentes se llama minuendo, el conocido se llama
sustraendo, y el buscado se denomina diferencia. La operacin de la diferencia de 2 nmeros a y b es inversa
a la de la adicin y se la representa mediante a b. Por denicin, a b = c siempre y cuando c + b = a.
Esquemticamente tendremos:
a b = |{z}
c
| {z }
m inuendo sustraendo diferencia

La diferencia de naturales no es una operacin cerrada, por ejemplo, (7 10) 2


= N. La diferencia c de
naturales, tambin es un natural, siempre y cuando a > b, pero es nica.

Producto. Se denomina producto de naturales a la suma de componentes iguales. Cada una de estas
componentes se llama multiplicando, al nmero de estas componentes se llama multiplicador, y a la suma
producto. Por denicin, tendremos

a+a+ + a = c, es decir a b
|{z} = c
|{z}
| {z }
b veces multiplicando multiplicador pro ducto

La operacin del producto es una operacin cerrada en N y el resultado es nico.

Divisin. Es la operacin aritmtica, mediante la cual, dados el producto de 2 factores y uno de ellos, se
encuentra el otro. Segn esto, el producto se llama dividendo, el factor conocido se llama divisor y el factor
buscado se denomina cociente. Por denicin, tendremos

a b = |{z}
c , siempre y cuando c b = a
| {z }
dividendo divisor co ciente

a
La divisin del natural a para el natural b, tambin se representa con , a b, a : b o a=b. La operacin de la
b
divisin no es una operacin cerrada en N, por ejemplo, (8 3) 2 = N. Si esta operacin es realizable (es exacta!)
en N, entonces su resultado es nico, y en este caso a se denomina mltiplo de b, y b se denomina divisor de a:
1.1. NMEROS NATURALES Y SUS OPERACIONES 11

Divisin con resto. Divisin con resto es la operacin aritmtica mediante la cual dados 2 naturales a y
b, se encuentran los naturales c y r, para los que se cumple la relacin

a = b c + r con r < b; (4)

donde a es el dividendo, b es el divisor, c es el cociente y r es el resto de la divisin. Si c = 0, entonces se


dice que a es divisible por b, o que a es mltiplo de b, o b es un divisor de a. La operacin de la divisin
3
con resto es una operacin cerrada en N, por ejemplo, (8 5 = 1 + , pues 8 = 5 1 + 3), y el resultado
5
es nico. Mediante la divisin comn y corriente con escalera podemos determinar el cociente y resto de
3 6 4 2 1 2 5
- 2 5 1 4 5 6
1 1 4
- 1 0 0
una divisin con resto: 1 4 2 36421 es el dividendo, 25 es el divisor, 1456 es
- 1 2 5
1 7 1
- 1 5 0
2 1
el cociente y 21 es el resto. Utilizando la relacin o frmula (4), tendremos las representaciones equivalentes:
36421 21
36421
| {z } = |{z}
25 1456
|{z } + |{z}
21 o = 1456 + :
25 25
dividendo divisor co ciente resto

Practico lo que aprend:

Utilizando el mtodo de la divisin con escalera, la frmula (4) y el ejemplo anterior, hallar las representa-
ciones de las siguientes divisiones con resto: a) 234 15, b) 348 25, c) 2731 23, d) 8904 236,

e) 25034 505, f ) 7700 1209, g) 8546 2312, h) 8434 6013, i) 6554 1524, j) 80034 456

Leyes de las operaciones con los nmeros naturales

Las leyes fundamentales de las operaciones con los naturales se consideran sin demostracin y mas bien como
axiomas:

1. Leyes conmutativas: a + b = b + a; a b=b a


El orden de los sumandos no altera la suma. El orden de los factores no altera el producto.

2. Leyes asociativas: a + b + c = (a + b) + c = a + (b + c) ; a b c = (a b) c = a (b c)
La suma y el producto de naturales puede ser realizado por partes o por asociacin.

3. Leyes distributivas del producto con respecto a la suma: (a+b) c = a c+b c, c (a+b) = a c+b c

4. Leyes de monotona: Si a > b, entonces a + c > b + c y a c > b c para cualquier natural c:

Orden en las operaciones y uso de los parntesis

En una expresin numrica, primero se ejecutan las operaciones que estn entre parntesis; en el interior de
cualesquiera parntesis primero se ejecutan los productos y divisiones cerradas, para luego las sumas y diferencias
cerradas. Para expresiones numricas largas o complejas se utilizan fundamentalmente los parntesis ( ), luego
los corchetes [ ] y nalmente las llaves f g, en ese orden para la escritura y de adentro hacia afuera. As, por
12 CAPTULO 1. PRELIMINARES ARITMTICOS

ejemplo, si se desea encontrar el valor de la expresin 2 f[28 13 + (927 187) 31] 6 120g, que tambin
puede escribirse en la forma 2 ((28 13 + (927 187) 31) 6 120) o 2 ((28 13 + (927 187) 31) 6 120),
entonces el orden de las operaciones ser:
00 1 1
BB C C
2|{z} @@|28{z13} + (927 187)|{z}31A 6 120A = 7528
|{z}| {z } ra |{z} |{z}
7ma 1ra 3
4 ta 2 da 5 ta 6 ta

Recuerda sobre las


leyes con naturales:
Ley: Suma Producto Suma, producto
a+b=b+a a b=b a
Conmutativa:
5+3=3+5 7 3=3 7
a + (b + c) = (a + b) + c a (b c) = (a b) c
Asociativa:
3 + (5 + 7) = (3 + 5) + 7 5 (2 3) = (5 2) 3
c (a + b) = a c + b c
Distributiva:
3 (2 + 5) = 3 2 + 3 5
Si a > b ) a + c > b + c Si a > b ) a c > b c
Monotona:
7>4)7+2>4+2 6>4)6 2>4 2

Practico lo que aprend:

Indicar el orden en las operaciones y hallar el resultado de:

1. a) 3 ((58 14 + (27 17) 5) 2 80), b) (18 12 + (3 (18 12) + (93 87) 15) 3 80) 2

2. a) 400 f[2 (80 14) + (126 87) 6] 2 9g 3, b) f[9 (48 14) + (90 28) 6] 3 10g 2

3. a) 61 ((58 16 + (67 47) 4) 2 11) 2, b) (16 (10 2) + (3 (12 8) + (27 17) 3) 2 6) 2

Criterios de divisibilidad

En muchsimos casos, sin efectuar la divisin del nmero natural a para el natural b, se puede determinar
si el resto de esta divisin es 0 o no, es decir, si a es mltiplo de b (b es divisor de a) o no. La respuesta a esta
cuestin viene dada mediante los siguientes teoremas (criterios de divisibilidad ):

1) Divisibilidad de una suma: Si cada uno de los sumandos es divisible para cierto natural, entonces la
suma tambin es divisible para este natural.

As, por ejemplo, sabemos que cada uno de los sumandos 16 + 100 + 28 es divisible para 4, entonces tambin
lo ser su suma 144 (16 + 100 + 28 = 144). Al contrario, si los sumandos no son divisibles para cierto natural,
esto no signica que su suma tampoco lo sea: 37 + 19 es divisible para 4 (pues 37 + 19 = 56), aunque 37 y 19
no son mltiplos de 4.

2) Divisibilidad de un producto: Si en una multiplicacin de nmeros naturales, por lo menos uno de


los factores es divisible para cierto natural, entonces el producto tambin es divisible para ese nmero.

As, por ejemplo, sin realizar la multiplicacin 16 18 50 podemos armar que el producto anterior es divisible
por 5, pues 50 es un mltiplo de 5.

3) Divisibilidad por 2: Un natural es divisible por 2, si y solamente si, su ltima cifra es divisible por 2.

As, por ejemplo, los nmeros: 128, 3070, 17 12 13, 4006, 1230 son mltiplos de 2.
1.1. NMEROS NATURALES Y SUS OPERACIONES 13

4) Divisibilidad por 3: Un natural es divisible por 3, si y solamente si, la suma de sus cifras es divisible
por 3. As, por ejemplo, los nmeros: 102 (la suma de sus cifras es 1 + 0 + 2 = 3), 3060 (la suma de sus cifras
es 3 + 0 + 6 + 0 = 9), 6006 (la suma de sus cifras es 6 + 0 + 0 + 6 = 12), 7029 (la suma de sus cifras es
7 + 0 + 2 + 9 = 18) son mltiplos de 3. Si un nmero posee muchas cifras, entonces su divisibilidad por 3
se comprueba recursivamente, buscando parejas de cifras cuyas sumas son mltiplos de 3: para 672300926127
tenemos que el 6, el 72, el 300, el 9, el 261 y el 27 son mltiplos de 3 y por tanto, todo el nmero es divisible
por 3.

La demostracin de la divisibilidad por 3 la efectuamos para el nmero abcd de 4 cifras: abcd = 1000a+100b+
10c + d = (999a + a) + (99b + b) + (9c + c) + d = (999a + 99b + 9c) + (a + b + c + d). El nmero 999a + 99b + 9c
es divisible por 3 por ser 999, 99 y 9 mltiplos de 3, y la suma (999a + 99b + 9c) + (a + b + c + d) ser divisible
para 3, siempre y cuando la suma de cifras a + b + c + d sea un mltiplo de 3:

5) Divisibilidad por 4 (25): Un nmero natural de no menos de 3 cifras es divisible para 4, si y solamente
si, el nmero que forman sus 2 ltimas cifras es divisible para 4 (para 25) o termina en 2 ceros. As, por ejemplo,
los nmeros: 48, 100, 3060, 71028, 7029 son mltiplos de 4 porque sus 2 ltimas cifras son nmeros divisibles
para 4. Los nmeros 100, 725, 1900, 12375 son mltiplos de 25.

| Para reconocer si un ao cualquiera es bisiesto es necesario y suciente que sus 2 ltimas cifras sean ceros
o que formen un nmero mltiplo de 4. Los aos 2008, 2012, 2016, 2020, etc. sern bisiestos.

La demostracin de la divisibilidad por 4, sin perder la generalidad, la hacemos para el nmero abcde de
5 cifras: Tenemos que abcde = 10000a + 1000b + 100c + 10d + e y como 10000, 1000, 100 son divisibles por 4,
entonces la suma 10000a + 1000b + 100c es divisible para 4. Esto signica que si el nmero de 2 cifras 10d + e
es divisible para 4, entonces todo abcde es divisible por 4, y en el caso contrario abcde no sera divisible para 4:

6) Divisibilidad por 5: Un nmero natural es divisible para 5, si y solamente si, su ltima cifra es 0 o 5.
As, por ejemplo, los nmeros: 45, 100, 3065, 71020, 70295 son mltiplos de 5.

7) Divisibilidad por 6: Un nmero natural es divisible para 6, si y solamente si, es par y la suma de
sus cifras nos da un nmero mltiplo de 3, pues debe ser al mismo tiempo divisible para 2 y para 3. As, por
ejemplo, los nmeros: 18, 102, 30684, 7020, 370200 son mltiplos de 6.

8) Divisibilidad por 7 (11 y 13): Un natural es divisible por 7 (11 y 13), si y solamente si, la diferencia
entre el nmero formado por sus 3 ltimas cifras y el nmero formado por las cifras restantes (o al revs) nos da
un nmero mltiplo de 7 (11 y 13). As, por ejemplo: 5019 (19 5 = 14) es divisible por 7, 193200 (200 193 = 7)
es divisible por 7, 72100 (100 72 = 28) es divisible por 7; 21428 (428 21 = 407) es divisiblle por 11, 1815
(815 1 = 814) es divisiblle por 11; 1014 (14 1 = 13) es divisible por 13, 21112 (112 21 = 91) es divisible
por 13.

| Adems, un natural es divisible por 11 siempre y cuando la diferencia de las cifras de lugar impar y las
cifras de lugar par nos da un mltiplo de 11 o 0. Por ejemplo, los nmeros: 1111 ((1 + 1) (1 + 1) = 0), 9152
((9 + 5) (1 + 2) = 11), 182908 ((1 + 2 + 0) (8 + 9 + 8) = 22), 132 ((1 + 2) (3) = 0) son mltiplos de 11.

9) Divisibilidad por 8 (125): Un nmero natural es divisible para 8 (125), si y solamente si, sus 3 ltimas
cifras son ceros o el nmero formado por estas 3 cifras forman un nmero mltiplo de 8 (125). As, por ejemplo:
5000, 1024, 7008, 4120 son divisibles por 8; 5000, 7500, 432125 son divisibles para 125.

10) Divisibilidad por 9: Un nmero natural es divisible para 9, si y solamente si, la suma de sus cifras
es un mltiplo de 9. As, por ejemplo, los nmeros: 702 (la suma de sus cifras es 7 + 0 + 2 = 9), 3060 (la suma
de sus cifras es 3 + 0 + 6 + 0 = 9), 6606 (la suma de sus cifras es 6 + 6 + 0 + 6 = 18), 187029 (la suma de sus
cifras es 1 + 8 + 7 + 0 + 2 + 9 = 27) son mltiplos de 9.

Observacin: Si la suma de las cifras de un nmero no es un mltiplo de 9, entonces el nmero tampoco


14 CAPTULO 1. PRELIMINARES ARITMTICOS

es mltiplo de 9, pero el resto de la divisin del nmero para 9, es el mismo que resulta de la suma de sus cifras
continuada hasta obtener una sola cifra. Por ejemplo, 2784 no es divisible por 9 ya que 2 + 7 + 8 + 4 = 21 no
es mltiplo de 9, pero el resto de la divisin es 3 (ya que 2784 = 9 309 + 3), que coincide con 2 + 7 + 8 + 4 =
21 ! 2 + 1 = 3. Sin perder generalidad, hacemos la demostracin de este corolario para un nmero de 4 cifras
que no es divisible por 9: sea abcd = 1000a + 100b + 10c + d = 999a + 99b + 9c + a + b + c + d = 9 ef g + r,
donde el nmero de 3 cifras ef g es el cociente, r es el resto y la suma de las cifras debe coincidir con el resto
siempre y cuando la suma a + b + c + d se la reduzca a una sola cifra. Este corolario lleva el nombre de regla
del resto de la divisin por 9.

11) Divisibilidad por 10 (100, 1000, etc.): Un natural es divisible para 10 (100, 1000, etc.), si y
solamente si, termina en un 0 (2 ceros, 3 ceros, etc.). Por ejemplo, 517000, 53000 son divisibles por 10, 100 y
1000:

Observacin: Es evidente que estos criterios de divisibilidad funcionan tambin para los nmeros enteros
negativos representados por el conjunto Z = f 1; 2; 3; : : :g :

Recuerda sobre la divisibilidad:


Divisibilidad por: Criterio: Ejemplos:
2 Todo nmero par. 48, 102, 7614
3 La suma de sus cifras es un nmero mltiplo de 3. 105, 40512, 93
4 Las 2 ltimas cifras son ceros o nmero mltiplo de 4. 2008, 724, 1300
5 El nmero termina en 5 o 0. 970, 195, 4755
6 El nmero es par y tambin divisible para 3. 102, 594, 1236

Pruebas de las operaciones aritmticas Valindonos de la observacin del criterio de divisibilidad por 9
o regla del resto de la divisin por 9, podemos enunciar las siguientes pruebas:

Prueba de la suma: Se aplica la regla del resto de la divisin por 9 a todos los sumandos y a la suma total.
Se aplica esta misma regla a los restos y a la suma total de restos. Si la cifra resto de la suma total coincide con
la cifra resto de la suma total de restos, entonces la suma es correcta:

4 3 8 ! 15 ! 6
3 2 5 ! 10 ! 1
+ 7 6 ! 13 ! 4
1 2 4 2 ! 9 ! 9
8 ! 8 ! 8
1 10 2 0 8 9 28 ! 10 ! 1

Prueba de la resta: Se suman los restos de la diferencia y del sustraendo para que nos den el del minuendo:

6 15 5 2 8
- 4 1 9 ! 5 ! 5
1 0 9 ! 10 ! 1
6

Prueba de la multiplicacin: Se verica el mismo producto, pero con los restos de los factores:
1.1. NMEROS NATURALES Y SUS OPERACIONES 15

4 3 8 ! 15 ! 6
7 9 ! 16 ! 7
3 9 4 2 42 ! 6
3 0 6 6
3 4 6 0 2 ! 15 ! 6

Prueba de la divisin: Se multiplica el resto del cociente por el del divisor; al producto se le suma el del
residuo y nos ha de dar el resto del dividendo:

9 27 48726 327 ! 12 ! 3
1602 149 ! 14 ! 5
2946 15 ! 6
3 +3
9

Practico lo que aprend:

1. Con los criterios de divisibilidad, analizar la multiplicidad de los nmeros:

a) 120, 440, 2970, 90000, 5544, 67200, 7560, 14008, 1200, 4800, 2900, 120000
b) 3600, 1400, 1860, 25000, 100000, 40200, 25544, 8000, 144000, 87560

2. Realizar las siguientes operaciones aritmticas y realizar sus respectivas pruebas:

a) 123 + 435 + 589, 753 + 1425 + 6005, 923 + 835 + 89, 1553 + 2735 + 509 + 5789, 993 + 1485 + 229
b) 7123 589, 87835 8589, 25085 1729, 47800 38589, 17835 9995, 97835 33219
c) 47835 889; 412005 109; 4005 289; 42205 589; 835 589; 1835 229; 1235 549
d ) 4783 129, 4105 209, 4005 589, 49925 789, 5800 779, 4123 879, 92005 4109

Descomposicin en factores primos

Si un nmero posee slo 2 divisores (el 1 y el mismo), entonces este se denomina primo; si posee ms de 2
divisores, entonces se denomina compuesto. Por ejemplo, el nmero 17 es primo y 17 = 1 17; el nmero 35 es
compuesto y 35 = 5 7; el 360 es compuesto y 360 = 2 2 2 3 3 5 = 23 32 5:

Observaciones: El nmero 1 no se considera ni primo, ni compuesto. El nmero 2 = 1 2 es el nico


primo par. Los nmeros primos son elementos ms simples, ladrillos o tomos con los que se construye toda
la aritmtica y la teora de nmeros.

| Representaremos con P = fa 2 N : a es divisible slo por 1 y por ag al conjunto de todos los nmeros pri-
mos, es decir, P = f2; 3; 5; 7; 11; 13; 17; 19; 23; : : :g. El 1 no se considera primo. P es un conjunto con innito
nmero de elementos. La siguiente tabla se denomina criba de Eratstenes, all estn sin tachar los nmeros
primos (en negrilla) que se obtienen por eliminacin de mltiplos, as, por ejemplo, se toma el 1er nmero primo
2, luego se eliminan todos los nmeros cada 2 celdas, es decir todos los pares; despus tomamos el 2do nmero
primo, el 3, y eliminamos todos los nmeros cada 3 celdas, es decir los mltiplos de 3; y as sucesivamente:

1/ 2 3 4/ 5 6/ 7 8/ 9/ 1/0/
11 1/2/ 13 1/4/ 1/5/ 1/6/ 17 1/8/ 19 2/0/

9/1/ 9/2/ 9/3/ 9/4/ 9/5/ 9/6/ 97 9/8/ 9/9/ 1/0/0/


16 CAPTULO 1. PRELIMINARES ARITMTICOS

Teorema fundamental de la aritmtica. Todo nmero natural puede descomponerse en forma nica en
producto de factores primos con sus respectivas multiplicidades (potencias).

Este importantsimo teorema nos dice que cualquier natural equivale al producto de varios nmeros primos
elevados a sus respectivas potencias, y que esta descomposicin es nica. As, por ejemplo, la descomposicin en
factores de los nmeros 360 es 360 = 23 32 5; de 1260 es 1260 = 22 32 5 7; de 103950 es 103 950 = 2 33 52 7 11,
las mismas que se obtienen utilizando la escalera de descomposicin:

103950 2
360 2 1260 2 51975 3
180 2 630 2 17325 3
90 2 315 3 5775 3
3 2 2 2
45 3 ) 360 = 2 3 5, 105 3 ) 1260 = 2 3 5 7, 1925 5 ) 103 950 = 2 33 52 7 11
15 3 35 5 385 5
5 5 7 7 77 7
1 1 11 11
1

Si en la descomposicin en factores primos de un natural aparece un factor que se repite ms de una vez,
entonces este factor tiene la forma de potencia bm = b| b {z }b, donde b es la base y m 2 N es el exponente.
m veces

1.1.2. Mximo comn divisor (m.c.d.) y Mnimo comn mltiplo (m.c.m.)

Mximo comn divisor. Se denomina m.c.d. de los nmeros naturales n1 y n2 , que se los considera com-
puestos, a aquel mximo nmero contenido como divisor tanto en n1 como en n2 . Para calcular el m:c:d: (n1 ; n2 )
se descomponen estos dos nmeros en sus factores primos y se toman los factores comunes a ambos con el menor
exponente. As, para hallar el m:c:d: (120; 180) se procede a construir las respectivas descomposiciones

120 2 180 2
60 2 90 2
30 2 45 3
) 120 = 23 3 5, ) 180 = 22 32 5,
15 3 15 3
5 5 5 5
1 1

y luego se toma los factores comunes con el menor exponente, entonces m:c:d: (120; 180) = 22 3 5 = 60.
Los nmeros 22 = 4, 3 y 5 se denominan divisores comunes de 120 y 180, y 60 = 22 3 5 es el m.c.d.

Anlogamente, el m:c:d: de los naturales compuestos n1 , n2 y n3 viene a ser el mximo nmero contenido
como divisor de n1 , n2 y n3 . Para calcular el m:c:d: (n1 ; n2 ; n3 ) se descomponen estos 3 nmeros en sus factores
primos y se toman los factores comunes con el menor exponente. Para hallar el m:c:d: (120; 180; 200) se procede
a construir las respectivas descomposiciones

120 2 180 2 200 2


60 2 90 2 100 2
30 2 45 3 50 2
) 120 = 23 3 5, ) 180 = 22 32 5, ) 200 = 23 52 ,
15 3 15 3 25 5
5 5 5 5 5 5
1 1 1
1.1. NMEROS NATURALES Y SUS OPERACIONES 17

luego se toma los factores comunes con el menor exponente, entonces m:c:d: (120; 180; 200) = 22 5 = 20.
Los nmeros 22 = 4 y 5 se denominan divisores comunes de 120, 180 y 200, y 20 = 22 5 es el m.c.d.

| Igualmente se dene y se calcula el m:c:d: (n1 ; n2 ; n3 ; : : : nk ) de k nmeros naturales. Si para 2 nmeros


(o ms) naturales m y n se tiene que m:c:d: (m; n) = 1, entonces los nmeros m y n se denominan mutuamente
primos. As, los nmeros 72 = 23 32 y 35 = 5 7 son mutuamente primos, pues m:c:d: (72; 35) = 1:

Mnimo comn mltiplo. Se denomina m.c.m. de los nmeros naturales compuestos n1 y n2 al nmero ms
pequeo que contiene (que es mltiplo) tanto a n1 como a n2 . Para calcular el m:c:m: (n1 ; n2 ) se descomponen
estos 2 nmeros en sus factores primos y se toman los factores comunes y no comunes a ambos con el mayor
exponente. Para hallar el m:c:m: (120; 180) se procede a construir las respectivas descomposiciones

120 2 180 2
60 2 90 2
30 2 45 3
) 120 = 23 3 5, ) 180 = 22 32 5,
15 3 15 3
5 5 5 5
1 1

y luego se toman los factores comunes y no comunes con el mayor exponente, entonces m:c:m: (120; 180) =
2 32 5 = 360. El nmero 360 = 23 32 5 es el nmero ms pequeo que contiene, o es mltiplo de 120 y 180.
3

Los nmeros 720, 1440, 2160, etc. tambin son mltiplos de 120 y 180, pero ninguno de ellos es el menor, como
lo es 360.

Anlogamente, el m.c.m. de los nmeros naturales compuestos n1 , n2 y n3 viene a ser el nmero ms


pequeo, mltiplo de n1 , n2 y n3 . Para calcular el m:c:m: (n1 ; n2 ; n3 ) se descomponen estos 3 nmeros en
sus factores primos y se toman los factores comunes y no comunes con el mayor exponente. Para hallar el
m:c:m: (120; 150; 280) se procede a construir las respectivas descomposiciones

120 2 280 2
150 2
60 2 140 2
75 3
30 2 70 2
) 120 = 23 3 5, 25 5 ) 150 = 2 3 52 , ) 280 = 23 5 7,
15 3 35 5
5 5
5 5 7 7
1
1 1

y luego se toman los factores comunes y no comunes con el mayor exponente: m:c:m: (120; 150; 280) =
23 3 52 7 = 4200. El 4200 es el mltiplo ms pequeo que contiene a 120 (lo contiene 35 veces), a 150 (28
veces) y a 280 (15 veces). Los nmeros 8400, 12600, etc. tambin son mltiplos de 120, 150 y 280, pero el
nmero ms pequeo que los contiene es el m:c:m: (120; 150; 280) = 4200.

Observaciones:

| De forma anloga se dene y se calcula el m:c:m: (n1 ; n2 ; n3 ; : : : nk ) de k nmeros naturales.

| Si los nmeros naturales m y n son mutuamente primos, es decir el m:c:d: (m; n) = 1, entonces el
m:c:m: (m; n) = m n. As, para m = 60 = 22 3 5 y n = 77 = 7 11 se tiene que el m:c:d: (60; 77) = 1
por ser dos nmeros mutuamente primos, siendo el m:c:m: (60; 77) = 60 77 = 4620:

| Para 2 naturales m y n se cumple la relacin m:c:d: (m; n) m:c:m: (m; n) = m n; anlogamente para
3 o ms nmeros m, n, k, . . . se cumple la relacin m:c:d: (m; n; k; : : :) m:c:m: (m; n; k; : : :) = m n k : : :
Por ejemplo, para m = 60 = 22 3 5 y n = 450 = 2 32 52 se tiene que m:c:d: (60; 450) = 2 3 5 = 30 y
18 CAPTULO 1. PRELIMINARES ARITMTICOS

m:c:m: (60; 450) = 22 32 52 = 900, cumplindose que m:c:d: (60; 450) m:c:m: (60; 450) = 30 900 = 27000 =
m n = 60 450:

Recuerda sobre el mnimo comn Recuerda sobre el mximo comn


mltiplo o m:c:m: (m; n) : divisor o m:c:d: (m; n) :
Se toman los factores comunes y no comunes Se toman slo los factores comunes con el
con el mayor exponente: m:c:m: (60; 24) = menor exponente: m:c:d: (60; 24) =
m:c:m: 22 3 5; 23 3 = 23 3 5 = 120; m:c:d: 22 3 5; 23 3 = 22 3 = 12;
m:cm: (24; 80; 45) = m:c:m: 23 3; 24 5; 32 5 m:c:d: (12; 72; 120) = m:c:d: 22 3; 23 32 ; 23 3 5
= 24 32 5 = 720 = 22 3 = 12

Practico lo que aprend:

1. Hallar los valores de cada una de las siguientes expresiones:

a) m:c:d: (120; 100), m:c:d: (80; 150), m:c:d: (1200; 800); m:c:d: (480; 360), m:c:d: (120; 100)
b) m:c:m: (12; 10), m:c:m: (80; 150), m:c:m: (1200; 800), m:c:m: (480; 360), m:c:m: (160; 100)
c) m:c:d: (120; 100; 80), m:c:d: (120; 80; 150), m:c:d: (100; 120; 40), m:c:d: (240; 480; 360)
d ) m:c:m: (48; 100; 60), m:c:m: (120; 80; 90), m:c:m: (64; 80; 60), m:c:m: (1200; 1000; 800) :

2. Comprobar que m:c:d: (m; n; k; : : :) m:c:m: (m; n; k; : : :) = m n k : : : para los grupos de nmeros:

a) (12; 10), (40; 64), (120; 100), (80; 150), (160; 800), (480; 360), (120; 100), (80; 900) :
b) (12; 36; 60), (64; 80; 40), (40; 60; 80), (84; 72; 120), (120; 100; 800), (80; 600; 800) :

| Ahora, conociendo que la operacin cerrada y fundamental en los naturales N es la suma de los mismos,
podemos introducir y construir el producto de naturales como una suma abreviada, as:

a b=a+a+ +a=b+b+ + b:
| {z } | {z }
b veces a veces

En calidad de ejemplo, podemos ver que 3 5 = 3 + 3 + 3 + 3 + 3 = 5 + 5 + 5:


| {z } | {z }
5 veces 3 veces

bn , donde b es la base y n el exponente,


| As mismo, podemos considerar a una potencia con naturales |{z}
Potencia
como un producto abreviado, as, por denicin: bn = |b b {z }b. De esta ltima relacin se desprenden las
n veces
siguientes 3 reglas fundamentales para las potencias con naturales:

a) bm b n = b| b {z }b b| b {z }b = b| b {z }b = b
m+n
,
m veces n veces m+n veces
n
b) (bm ) = b|m bm{z bm} = b| b {z mn
}b = b ,
n veces m n veces
n
c) (b c) = |b c b {z
c b }c = bn cn
n veces

Las mismas que nos dicen que: a) para multiplicar potencias con las mismas bases, se pone la base y se suman
los exponentes; b) para elevar una potencia a otra potencia, se pone la base y se multiplican los exponentes; c)
la potencia de un producto es igual al producto de sus potencias:
1.1. NMEROS NATURALES Y SUS OPERACIONES 19

Recuerda las leyes con las potencias naturales: Ejemplos:


n
Denicin de potencia: b = |b b {z }b 23 = |2 {z
2 2}
n veces 3 veces
Producto de potencias: bm bn = bm+n 32 35 = 32+5 = 37
n 3
Potencia de otra potencia: (bm ) = bm n 24 = 24 3 = 212
n 4
Potencia de un producto: (b c) = bn cn (3 2) = 34 24

Observaciones: Se suele representar mediante N0 = f0; 1; 2; 3; 4; 5; 6; 7; : : : ; n; : : :g a la serie natural que


empieza con el 0. Si cierto conjunto carece de elementos, entonces se dice que el conjunto es vaco y la cantidad
de elementos de este conjunto se denomina cero (nmero 0). El 0 es menor que cualquier natural y la serie o
conjunto de nmeros 0, 1, 2, 3, 4, 5, 6, 7, : : :, n, : : : se denominan enteros no negativos.

Si a es un natural y a 6= 0, entonces las operaciones con el cero se denen de la siguiente manera:

Adicin: a + 0 = 0 + a; 0 + 0 = 0:

Diferencia: a 0 = a; a a = 0; 0 0 = 0:

Producto: a 0 = 0 a = 0; 0 0 = 0:

Divisin: 0 a = 0. La divisin para 0 no es posible o realizable. La expresin 0 0 es indeterminada (por


ejemplo, 0 0 podra ser igual a 9, o a 2, o a cualquier otro valor) y no est denida con un valor nico.

Ejemplos:

1. Simplicar las siguientes operaciones:


3
a) (2 3) 2 34 = 23 33 2 34 = 23+1 33+4 = 24 37
4
3 2 2 2 4 4
b) 24 53 32 74 = 24 59 36 78 = 24 518 312 716 = 216 348 572 764

3 4 4
c) 24 32 55 23 52 = 24 36 515 23 52 = 216 324 560 23 52 = 219 324 562

2. Representar las siguientes expresiones en potencias de 2, 3 y 5:


3 9 6 2
a) 83 122 202 = 89 126 202 = 23 22 3 22 5 = 227 212 36 24 52 = 243 36 52
3 4 4 36 24
b) 1203 272 = 1209 276 = 12036 2724 = 23 3 5 33 = 2108 3108 536
3 2 4 3 3 18 18 24
c) 2402 323 542 = 2406 326 548 = 24 3 5 25 2 33 = 2186 390 518

3. Hallar los valores de a, b, x, y, z para que se cumplan las igualdades:


3 2 6
a) 162 812 = 2a 3b ) 166 34 = 24 38 = 224 38 = 2a 3b , a = 24, b = 8
2 3
b) 182 123 = 2a+b 3b ) 32 2 22 3 = 28 37 = 2a+b 3b , a + b = 8, b = 7 ) a = 1
c) 802 123 = 22x y 3y+z 52z ) 802 123 = 28 52 26 33 = 214 33 52 = 22x y
3y+z 52z , 2x y = 14,
z + y = 3, 2z = 2 ) z = 1, y = 2, x = 8:

Practico lo que aprend:

1. Simplicar cada una de las siguientes expresiones:


3 3 3 3 3
a) (2 3) 2 34 , 22 5 2 34 , 5 32 22 32 , 2 52 3 22 32 , 52 32 54 34
20 CAPTULO 1. PRELIMINARES ARITMTICOS

3 2 2 2 3 2 3 2 3
b) 32 52 2 24 , 52 52 32 32 24 , 22 52 32 24 35
3 4 3 2 4 3 3 4 2
c) 24 32 25 23 34 , 23 32 55 23 53 , 54 112 52 53 34 113

2. Representar las siguientes expresiones en potencias de 2, 3 y 5:


2 2 2 2 2
a) 162 123 102 , 82 122 1002 ; 122 163 252 , 252 183 272 , 1002 813 52
2 2 2 2
b) 123 243 202 , 102 83 243 202 , 1002 183 243 2002 , 103 123 184 502
3 2 4 3 3 2 4 3 3 2 4 3
c) 1202 163 1082 , 24 1202 163 122 , 36 122 163 182

3. Hallar los valores de a, b, x, y, z para que se cumplan cada una de las igualdades:
4 4 4 4
a) 123 92 = 2a 3b , 242 182 = 2a 3b , 82 62 = 2a 3b , 62 242 = 2a 3b
2
b) 542 123 = 2a+b 3b , 1083 242 = 22a+b 3b , 1922 93 = 24a b
3b , 162 93 = 22a b
32b
c) 1602 123 = 22x y
3y+z 52z , 802 122 = 2x y
3y+z 52z , 302 1204 = 22x+2y 3y z
52z

1.2. Nmeros Enteros y sus Operaciones

El conjunto de los nmeros enteros Z aparece con la necesidad de ampliar el conjunto de naturales intro-
duciendo la operacin de la resta en N, donde slo estaba denida la operacin cerrada de la suma de naturales,
el producto de naturales como suma abreviada, y la potenciacin con naturales como producto abreviado.

Para introducir el conjunto de los enteros es necesario tener un conjunto simtrico con respecto a la diferencia
en N y un elemento neutro (centro de simetra), denominado cero o 0. En este caso los nmeros enteros tendrn
la forma: 8 9
>
< >
=
Z = : : : ; n; : : : ; 2; 1; 0
|{z} ; 1; 2; : : : ; n; : : : ;
:|
> {z }
elem ento neutro
| {z }>;
N N+

donde N+ = f1; 2; : : : ; n; : : :g son los enteros positivos representados con Z+ , N = f 1; 2; : : : ; n; : : :g son


los enteros negativos representados con Z , y el 0 es el elemento neutro.

De esta manera Z = Z [ f0g [ Z+ = f: : : ; n; : : : ; 2; 1; 0; 1; 2; : : : ; n; : : :g, que signica que todos los


nmeros enteros son la unin de los enteros negativos, el cero y los enteros positivos, donde son cerradas las opera-
ciones de adicin y de diferencia de enteros. Para denir la operacin del producto de enteros es necesario primero
introducir los convencionalismos a b = a a a = (a b) y a ( b) = b b b = (a b), y la
| {z } | {z }
b veces a veces
(+) (+) = (+)
(
(+) ( ) = ( ) n an ; si n es natural par
ley de los signos: , ( a) = ( a) ( a) ( a) =
( ) (+) = ( ) | {z } an ; si n es natural impar
n veces
( ) ( ) = (+)

a 1
| La ley de signos para la divisin de enteros es la misma que para el producto, pues = a , b 6= 0:
b b
3
| Las operaciones de divisin de enteros (por ej. 2
= Z), as como la potenciacin con exponente entero
5
a
negativo (por ej. 3 2 2
= Z) no son operaciones cerradas en Z; a0 = 1 con a 6= 0. Las operaciones con a 6= 0,
0
0
y 00 no son realizables y no estn denidas.
0
| Para las operaciones con enteros positivos y negativos, se opera de igual manera que con los natu-
rales, pero es conveniente primero calcular el signo del resultado. As, por ejemplo, en la expresin 2
1.2. NMEROS ENTEROS Y SUS OPERACIONES 21

3 2 3
3
32 82 ( 6) , primero aplicamos la ley de los signos y calculamos el signo: ( ) ( ) (+) ( ) = ( ).
3 2 3
3
Luego simplicamos el resultado: 2 32 82 (6) = 2 318 216 29 39 = 226 327 :

Reglas operacionales con los nmeros enteros

Presentamos las reglas para operar (sumar, restar, multiplicar y dividir) con los nmeros enteros Z:

Reglas operacionales en Z:
1) a + (b + c) = a + b + c 2) (a + b) + c = (a + c) + b = a + (b + c)
3) a (b + c) = (a b) c 4) (a + b) c = (a c) + b = a + (b c)
5) a (b c) = (a + c) b = (a b) + c 6) (a b) c = (a c) b = a (b + c)
7) a (b c) = (a b) c 8) (a b) c = (a c) b = a (b c)
9) a (b c) = (a b) c 10) (a b) c = (a c) b = a (b c)
11) (a + b) c = (a c) + (b c) 12) (a b) c = (a c) (b c)

a a a
Para 11), el error frecuente es: a (b + c) 6= (a b) + (a c), 6= , (a b) c 6= (a c) (b c)
b+c b c
a a a
Para 12), el error frecuente es: a (b c) 6= (a b) (a c), 6=
b c b c

Leyes para los cambios en las sumas, diferencias, productos y cocientes

Pasamos a analizar los cambios de los resultados en las operaciones aritmticas cerradas en dependencia de
los cambios de sus componentes:

Para la suma 1) Si a + b = c, entonces (a d) + b = c d. Si a b = c, entonces (a d) b=c d


y diferencia: 2) Si a b = c, entonces a (b d) = c d. Si a b = c, entonces (a d) (b d) = c
a c
1) Si a b = c, entonces (a d) b = c d y b=
Para el d d
producto: b
2) Si a b = c, entonces (a d) =c
d
a c
1) Si a b = c, entonces (a d) b=c d y b=
d d
Para el cociente c b
2) Si a b = c, entonces a (bd) = y a =c d
sin resto: d d
a b
3) Si a b = c, entonces (ad) (bd) = c y =c
d d

Para el cociente a b r
1) Si a = b c + r, entonces a d = (b d) c + r d y = c+
con resto: d d d

Ejemplos:

1. Simplicar las siguientes operaciones:


3 4
a) (2 3) 2 ( 3) = 23 33 2 34 = 23+1 33+4 = 24 37
3 2
4 2 2
b) 23 52 3 3 = +23 56 33 38 = 23 512 36 316 = 23 36+16 512 = 23 322 512
3 4
5 4
c) 24 32 ( 5) 23 52 = + 24 36 515 23 52 = 216 324 560 23 52 = 219 324 562
22 CAPTULO 1. PRELIMINARES ARITMTICOS
3
3 5 4
d) 42 ( 3) = 260 312

2. Representar las siguientes expresiones en potencias de 2, 3 y 5:


3 9 6 2
3
a) ( 8) 122 202 = 89 126 202 = 23 22 3 22 5 = 227 212 36 24 52 = 243 36 52
3 4
2 4 36 24
b) 1203 ( 27) = 1209 276 = 12036 2724 = 23 3 5 33 = 2108 3108 536

3 2 4 3 3 18 18 24
c) 2402 323 542 = 2406 326 548 = 24 3 5 25 2 33 = 2186 390 518

Practico lo que aprend:

1. Simplicar cada una de las siguientes expresiones:


3 4 3 3 3 3
a) ( 2) 2 ( 3) , 22 ( 5) 34 , 5 32 ( 5) 32 , 52 3 22 32 , 52 32 54 34
3 2 2 2 3 2 3 2 3
b) 32 52 2 24 , 52 53 3 32 24 , 52 32 24 35
3 5 3 3 4 3 3 4 2 3
c) 24 32 25 23 34 , 23 32 55 23 5 , 54 72 5 53 34 ( 7)

2. Representar cada una de las siguientes expresiones en potencias de 2, 3 y 5:


2 2 2 2 3
a) 163 122 102 , 83 122 1002 , 124 163 252 , 253 183 273 , 1004 813 53
3 2 2 2
b) 122 243 203 , 102 84 243 202 , 1003 182 243 2003 , 103 125 184 503
3 4 5 2 3 2 2 3 3
c) 1203 2704 , 1002 202 , 123 2705 180 , 1203 2704 123 642

1.3. Nmeros Racionales y sus Operaciones

Pasamos a estudiar los nmeros racionales Q, es decir, aquellos nmeros que pueden ser representados
m
mediante fracciones , con m y n enteros, m 6= 0. El nombre de nmeros racionales se debe a la relacin o
n
razn entre dos enteros a y b 6= 0, representada mediante a b o a=b, y el smbolo Q proviene de la palabra
a
latina quotient (cociente o cuociente) que es el resultado de la razn: = q .
b
|{z} |{z}
co ciente
razn

1.3.1. Fracciones. Fracciones propias, impropias y mixtas

m 3 1
Se denomina fraccin comn al nmero representado con la razn , con m; n 2 N. As, por ejemplo, , ,
n 5 4
8
, etc. son fracciones comunes. El nmero m se denomina numerador de la fraccin y n su denominador. En
3
m
particular, puede ser n = 1 y la fraccin toma la forma = m lo que signica que cualquier nmero natural
1
(entero) m puede representarse como fraccin de denominador 1, es decir, cualquier nmero natural (entero)
m
tambin es una fraccin, pero con denominador 1. La representacin equivale a poner m : n o m n:
n
m m
Observacin: La fraccin con m 2 Z y n 2 N se denomina fraccin negativa y es la misma que .
n n
m
Las fracciones comunes se dividen en fracciones propias e impropias. La fraccin , sin tomar en cuenta su
n
signo, se denomina propia si m < n, e impropia si m n. Toda fraccin impropia puede transformarse en suma
de un natural (entero) y fraccin propia, o de un entero si m es mltiplo de n.
1.3. NMEROS RACIONALES Y SUS OPERACIONES 23

| Es comn representar la suma de un nmero natural (o entero) y una fraccin propia (es decir, una fraccin
1 7
impropia) como fraccin especial, denominada fraccin mixta. As, por ejemplo, 2 + = =
3 3
|{z}
fraccin im propia
1 2 17 2 1 85 1
2 , 3 = = 3 , 7+ = = 7
3
|{z} 5 | {z5} 5
| {z } 12 12
|{z} 12
|{z}
fraccin m ixta fraccin im propia fraccin m ixta fraccin im propia fraccin m ixta

Recuerda sobre
las fracciones:
Fraccin comn: Fraccin propia: Fraccin impropia: Fraccin mixta:
a a b
Relacin entre Fraccin comn Fraccin comn Fraccin del tipo a :
b b c
2 naturales: con a < b : con a b :

21 2 8 12 1 7 23 8 3 28 21 17 1 3 2 1
; ; ; ; ; ; ; ; ; 1 ; 2 ; 5 ; 8
7 3 5 17 3 15 25 21 3 11 7 15 3 15 7 6
De fraccin mixta a comn,
Se divide el numerador
se multiplica el denomina-
para el denominador
dor por la parte entera y se
con resto:
suma con el numerador:

7 2 3+1 1 3 5 2+3 13
= =2+ 2 = =
3 3 3 5 5 5

Ejemplos:

Reducir las siguientes fracciones impropias a suma de un entero y una fraccin propia:

16 47 93
1. a) , b) , c) :
4 15 10
Solucin: El primer mtodo consiste en descomponer el numerador en un mltiplo del denominador ms
su resto
16 47 45 + 2 45 2 2 93 90 + 3 90 3 3
a) = 4. b) = = + = 3 + . c) = = = 9 :
4 15 15 15 15 15 10 10 10 10 10
104 147 112
2. a) , b) , c) :
3 13 11
Solucin: El segundo mtodo consiste en dividir con escalera el numerador para el denominador y
obtener el cociente y resto
104 3 147 13 112 11
104 2 147 4 112 2
a) 14 34 ) = 34 + , b) 17 11 ) = 11 , c) 110 10 ) = 10 + :
3 3 13 13 11 11
2 4 2

Practico lo que aprend:

Reducir las siguientes fracciones impropias a suma de un entero y una fraccin propia:

102 49 87 179 287 77 109 97 1709


1. a) , b) , c) , d) , e) , f) , g) , h) , i)
5 17 15 25 100 3 13 3 15
119 157 102 387 309 37 517 117 89
2. a) , b) , c) , d) , e) , f) , g) , h) , i)
23 14 13 19 102 4 12 11 3
24 CAPTULO 1. PRELIMINARES ARITMTICOS

Fracciones equivalentes. Propiedad fundamental de las fracciones. Simplicacin de fracciones

a c a c 3 6
Dos fraccionesy son equivalentes (iguales, = ), si y solamente si, ad = bc. Por ejemplo, = ya
b d b d 5 10
8 24 a a m
que 3 10 = 5 6, y = ya que 8 21 = 7 24. De la igualdad entre fracciones se sigue que = ,
7 21 b b m
pues a b m = b a m, es decir, una fraccin no se altera si multiplicamos tanto numerador como denominador
por un mismo nmero natural (o entero distinto de 0), esta es la propiedad fundamental de las fracciones.
a
Simplicar una fraccin signica cambiar por otra fraccin con menor numerador y denominador, por
b
12 3 4 3/ 4 4 a
ejemplo, = = = . Es evidente que una fraccin podr ser simplicada siempre y cuando
45 3 15 3/ 15 15 b
el numerador a y el denominador b no sean mutuamente primos, es decir, cuando a y b contienen uno o varios
30 3 2 5 3/ 2 5/ 2
factores comunes. Por ejemplo, = = = es una fraccin simplicable o reducible; las
45 9 5 3/ 3 5/ 3
7 13 2
fracciones , , son fracciones irreducibles ( m:c:d: (numerador; denominador) = 1), pues 7 y 6, 13 y 15,
6 15 3
2 y 3 son naturales mutuamente primos, es decir, m:c:d: (7; 6) = 1, m:c:d: (13; 15) = 1 y m:c:d: (2; 3) = 1.
a
| El objetivo fundamental de simplicar o reducir una fraccin es convertirla en una fraccin irreducible
b
a0
, ms simple y donde m:c:d: (a0 ; b0 ) = 1.
b0
Practico lo que aprend:

Simplicar las siguientes fracciones:

30 60 150 90 1344 5040 362 880


1. a) , b) , c) , d) , e) , f) , g)
18 216 216 450 10 584 40 320 360
1200 2700 4500 180 36 450 90 39 916 800
2. a) , b) , c) , d) , e) , f) , g)
1000 3240 576 4500 900 450 2520

Comn denominador de fracciones o c:d:

a c
Si se tienen 2 (o ms) fracciones , , se denomina comn denominador de las mismas al m:c:m: (b; d) (o
b d
m:c:m: (b; d; : : :)) o mnimo comn mltiplo de sus denominadores. La funcin del comn denominador es la de
a c
transformar las fracciones , (o ms) en nuevas fracciones equivalentes con denominadores iguales, y de esta
b d
manera hacerlas comparables, es decir, averiguar cual de las 2 (o ms) fracciones es mayor o cual es menor. Por
ejemplo, el c:d: para 5=6 y 17=20 ser el m:c:m: (6; 20) = m:c:m: 2 3; 22 5 = 60 = c:d: Como 60 6 = 10 y
5 5 10 50 17 17 3 51 17 5
60 20 = 3, tendremos que = = y = = . Se concluye que > :
6 6 10 60 20 20 3 60 20 6
a c
| Si las fracciones y (o ms) poseen denominadores mutuamente primos, o sea m:c:d: (b; d) = 1
b d
(m:c:d: (b; d; : : :) = 1), entonces el comn denominador ser el producto de sus denominadores, es decir b d (o
b d ). As, por ejemplo, para el c:d: de 2=5, 3=7 se tiene que el m:c:m: (5; 7) = 35 = c:d: y como 35 5 = 7
2 2 7 14 3 3 5 15 3 2
y 35 7 = 5 se tendr que = = y = = . Adems, > :
5 5 7 35 7 7 5 35 7 5
Ejemplos:

Reducir las siguientes fracciones a comn denominador:

1 1 3
1. , ,
4 2 10
Solucin: Para 1=4, 1=2, 3=10, el m:c:m: (4; 2; 10) = m:c:m: 22 ; 2; 2 5 = 22 5 = 20, entonces 20 4 = 5,
1 1 5 5 1 1 10 10 3 3 2 6
20 2 = 10, 20 10 = 2. Por tanto, = = , = = , = = . Las fracciones
4 4 5 20 2 2 10 20 10 10 2 20
1.3. NMEROS RACIONALES Y SUS OPERACIONES 25

1=4, 1=2, 3=10 reducidas a comn denominador son: 5=20, 10=20 y 6=20, respectivamente. Adems, se
1 3 1
tiene que > > :
2 10 4
16 5
2. , 3,
5 10
Solucin: Para 16=5, 3, 5=10 tenemos que el m:c:m: (5; 1; 10) = m:c:m: (5; 1; 2 5) = 2 5 = 10, entonces
16 16 2 32 3 3 10 30 5 5 1 5
10 5 = 2, 10 1 = 10, 10 10 = 1. Por tanto, = = ,3= = = , = = .
5 5 2 10 1 1 10 10 10 10 1 10
Las fracciones 16=5, 3, 5=10 reducidas a comn denominador son: 32=10, 30=10 y 5=10, respectivamente.
16 5
Se comprueba que >3> :
5 10
4 12 1
3. , , 1
5 7 10
1
Solucin: Para 4=5, 12=7, 1 10 tenemos que el m:c:m: (5; 7; 10) = m:c:m: (5; 7; 2 5) = 2 5 7 = 70,
4 4 14 56 12 12 10 120
entonces 70 5 = 14, 70 7 = 10, 70 10 = 7. Por tanto, = = , = = ,
5 5 14 70 7 7 10 70
1 11 11 7 77 1
1 = = = . Las fracciones 4=5, 12=7, 1 10 reducidas a comn denominador son: 56=70,
10 10 10 7 70
12 1 4
120=70 y 77=70, respectivamente. Adems se comprueba que >1 > :
7 10 5

Recuerda sobre el comn denominador o c:d: Ejemplo


a c 5 17
Reducir las fracciones , , etc. a c:d: signica hallar Para , , el m:c:m: (6; 20) =
b d 6 20
el m:c:m: (denominadores) = m:c:m: (b; d; : : :) = c:d: m:c:m: 3 2; 22 5 = 60 = c:d,
y convertirlas a fracciones equivalentes y comparables, entonces 60 6 = 10 y 60 20 = 3 :
5 5 10 50 17 17 3 51
= = , = =
con idnticos denominadores. 6 6 10 60 20 20 3 60
Adems, se comprueba que 5=6 < 17=20:

Practico lo que aprend:

Reducir cada grupo de fracciones a comn denominador y ponerlas en orden creciente:

5 4 5 1 4 3 1 5 17 1 3 7 9 7 5 4
1. a) , ; b) , , , ; c) , , ; d) , , , ; e) , ,
2 3 12 5 3 10 15 30 60 25 40 20 80 12 16 15
3 5 7 7 3 4 7 13 8 11 27 19 5 7 11
2. a) , , ; b) , 4, ; c) , , ; d) , , , ; e) , ,
4 2 10 5 10 15 30 60 25 40 100 80 12 16 15
5 5 7 8 7 7 5 7 5 11 33 17 5 7 7
3. a) , , ; b) , 2, ; c) , , ; d) , , , ; e) , ,
12 8 24 15 30 30 24 60 36 48 100 60 18 12 16

1.3.2. Operaciones con fracciones. Fracciones decimales

A continuacin presentamos las operaciones fundamentales con fracciones.

Suma de fracciones: Para sumar fracciones comunes se presentan 2 casos: a) Si los denominadores de
las fracciones son iguales, entonces se suman los numeradores y se mantiene el mismo denominador, es decir,
a c a+c
+ = . b) Si los denominadores de las fracciones son distintos, entonces las fracciones se reducen a comn
b b b
3 5 3+5 8 1 3 1 2+3 5
denominador y luego se aplica el caso a). As, por ejemplo, + = = = 2; + = = .
4 4 4 4 2 4 4 4
Diferencia de fracciones: Para restar fracciones comunes se distinguen dos casos: a) Si los denominadores
de las fracciones son iguales, entonces se restan los numeradores y se mantiene el mismo denominador, es decir,
a c a c
= . b) Si los denominadores de las fracciones son distintos, entonces las fracciones se reducen a comn
b b b
3 5 3 5 2 1 3 1 3 1 2 1
denominador y luego se aplica el caso a). As, por ejemplo, = = = ; = = :
4 4 4 4 2 4 2 4 4
26 CAPTULO 1. PRELIMINARES ARITMTICOS

Producto de fracciones: Para multiplicar fracciones comunes se hallan por separado los productos de
a c a c
los numeradores y denominadores y con estos se halla la fraccin producto, es decir, = . As, por
b d b d
3 2 3 2 6 3
ejemplo, = = = :
4 5 4 5 20 10
a c
Divisin de fracciones: Para dividir fracciones comunes (encontrar su cociente), se aplica la regla =
b d
a d 3 2 3 5 15
, es decir, al divisor a=b se lo multiplica por la fraccin d=c, inversa de c=d. Por ejemplo, = = .
b c 4 5 4 2 8
a c a=b
Cabe notar que , tambin se representa en la forma y entonces, el resultado o cociente se lo obtiene
b d c=d
a=b a d
con la regla producto de extremos a y d dividido para el producto de los medios b y c, es decir, = :
c=d b c
a n a
Potencia de una fraccin: La expresin se denomina potencia de la fraccin y se la calcula
n
b b
a n a
con la regla = n , se supone que a; b; n 2 Z y b 6= 0. En la potenciacin de fracciones se cumplen las
b b
a 0 am 1 a0 an b m
relaciones: = 1, am an = n = am n , 1 an = n = n = a n , m =
b a a a b a n
Observacin: Las operaciones anteriores sern correctas en dependencia de los valores numricos reales
1
que tomen las bases y las potencias. Las operaciones aa y para a = 0 no estn denidas.
a

Recuerda sobre las operaciones con fracciones: Ejemplos:


Suma:
a c a+c a c m+n 2 4 2+4 2 1 2 2+1 3
+ = , + = + = , + =
b b b b d c:d: (b; d) 3 3 3 3 2 6
Diferencia:
a c a c a c m n 2 4 2 4 2 1 2 2+1 3
= , = = , =
b b b b d c:d: (b; d) 3 3 3 3 2 6
Producto:
a c a c a c a c 2 4 2 4 2 5 2 5 5
= 2 , = = 2 , = =
b b b b d b d 3 3 3 3 4 3 4 3 2
Divisin:
a c a b a a c a d 2 4 2 3/ 2 5 2 4 8
= = , = = , = =
b b b c c b d b c 3 3 3/ 4 3 4 3 5 15
Potencias:
2
a n an am 1 an b m 2 22 25 1 23 5 2
= n , n = am n , n = a n , m = = , = 25 3
, =3 2
, =
b b a a b a n 3 32 23 32 52 2 3

Ejemplos:

Realizar cada una de las siguientes operaciones con fracciones:

7 3 4 2 7 3 4 3 4 3
1. a) + , b) , c) , d) , e) 4
5 5 9 9 5 5 5 5 5 2
7 3 7+3 10 4 2 4 2 2 7 3 7 3 21
Solucin: a) + = = = 2, b) = = , c) = = ,
5 5 5 5 9 9 9 9 5 5 5 5 25
4 3 4 5/ 4 5/ 4 4 3 4 11 4 2 8
d) = = = , e) 4 = = =
5 5 5/ 3 5/ 3 3 5 2 5 2 5 11 55
7 1 7 3 3 2 4 3 5 3
2. a) + , b) , c) , d) , e) 2
6 5 9 4 5 7 5 2 6 4
7 1 7 5+1 6 41 7 3 7 4 3 9 1 3 2 3 2 6
Solucin: a) + = = , b) = = , c) = = ,
6 5 30 30 9 4 36 36 5 7 5 7 35
4 3 4 2 4 2 8 5 3 5 11 5 4 5 4 10
d) = = = , e) 2 = = = = :
5 2 5 3 5 3 15 6 4 6 4 6 11 6 11 33
1.3. NMEROS RACIONALES Y SUS OPERACIONES 27
2 2 3 2 2
3 1 7 3 3 1 4 1 5 3
3. a) + , b) , c) , d) , e) 1
2 5 9 2 5 3 5 2 6 2
2 2
3 1 9 1 9 5+1 4 49 7 3 7 9 7 4 9 9 53
Solucin: a) + = + = = , b) = = = ,
2 5 4 5 20 20 9 2 9 4 36 36
3 2 2 2
3 1 3 1 3 1 1 4 1 4 1 4 4 16 5 3 5 5
c) = = = , d) = = = , e) 1 = =
5 3 5 27 5 27 45 5 2 5 4 5 1 5 6 2 6 2
5 25 5 4 2
= = :
6 4 6 25 15

Practico lo que aprend:

Realizar cada una de las siguientes operaciones con fracciones:

7 3 4 6 6 2 4 3 4 3 5 2 2 3 32 1 12
1. a) + , b) , c) 1 , d) , e) 5 , f) +1 , g) 3
4 4 7 7 5 5 9 9 7 2 6 3 5 4 22 3 5
5 2 7 9 3 1 1 5 7 1 1 4 2 2
2. a) + 1, b) + 1, c) , d) , e) 3 , f) +
6 5 8 5 7 4 6 2 8 4 6 3 5 5
7 9 1 3 1 4 1 5 2 7 1 4 5 1 6
3. a) , b) 2 , c) 1 , d) 1 3 , e) 2 2
8 4 5 7 2 5 6 2 3 8 4 5 4 3 5
2 3 3 2 3 3 2 3 2
3 1 1 3 3 3 1 4 1 5 2
4. a) 1 , b) 1 , c) 1 , d) 1 , e) 1
4 2 3 2 2 5 3 5 2 6 3

Fracciones decimales y sus operaciones

a
Una fraccin ordinaria se denomina fraccin decimal, si mediante divisin (a b), se obtiene un nmero
b
compuesto de una parte entera y una parte de decimales, es decir, de la forma A:BCD : : :, donde A es la
parte entera, B son las dcimas, C son las centsimas, D son las milsimas, etc. En este caso, tendremos que
a B C D
= A:BCD : : : = A + + + + . As, por ejemplo, para denominadores que son mltiplos de 10,
b 10 100 1000
3 47 48 3 23
se obtiene fcilmente que = 0;3, = 0;47, = 0;048, 2 = = 2;3.
10 100 1000 10 10
A la inversa, por ejemplo, si se tiene
236 200 + 30 + 6 2 3 6
75;236 = 75 = 75 + = 75 + + + ;
1000 1000 10 100 1000
entonces en 75;236 se contienen 5 unidades, 7 decenas (75 enteros); 2 dcimas, 3 centsimas y 6 milsimas. Cabe
notar que 75;236 = 75;23600 : : : y por tanto, cualquier cantidad de ceros a la derecha de la ltima cifra decimal
signicativa, no alteran el valor del nmero decimal.

Operaciones aritmticas con las fracciones decimales. Fracciones peridicas Las operaciones de
suma, diferencia, producto y divisin con fracciones decimales nitas se reducen a las ya conocidas reglas
aritmticas, donde para operar se respetan las diferentes posiciones de las cifras (unidades, decenas, centenas,
etc.; dcimas, centsimas, milsimas, etc.) para obtener los resultados. As, por ejemplo:

13.50 15.025 2.12 2.58 22.1 13 0.221 13 257.6 112


+ 5.86 - 0.740 0.15 0.03 -13 1.7 -13 0.017 -224 2.3
19.36 14.285 1060 0.0774 91 91 336
2120 -91 -91 -336
0.3180 0 0 0
28 CAPTULO 1. PRELIMINARES ARITMTICOS

| Para multiplicar una fraccin decimal por 10, 100, 1000, etc., se recorre la coma o el punto a la derecha 1
puesto, 2 puestos, 3 puestos, etc., respectivamente. As, por ejemplo: 2;67 10 = 26;7, 32;0587 1000 = 32058;7.

| Para dividir una fraccin decimal para 10, 100, 1000, etc., se recorre la coma o el punto a la izquierda 1
puesto, 2 puestos, 3 puestos, etc., respectivamente. Por ejemplo: 26;7 10 = 2;67, 2;1 100 = 0;021.

| Para dividir una fraccin decimal para otra (o un entero por otro), se recurre al mtodo de la escalera.
El cociente o resultado as obtenido puede ser: exacto (es decir, con resto nulo); o puede conducir a una fraccin
decimal innita (con un nmero innito de cifras despus de la coma o punto decimal), pero peridica (es decir,
con un grupo de cifras que se repiten). Analicemos estos casos con ejemplos:

24.12 6.03 2.00000 3 169.00000 33


- 24.12 4
|{z} - 18 0;6666 - 165 5;121212
| {z } | {z }
co ciente fraccin p eridica fraccin p eridica
0
|{z} 20 40
resto
- 18 - 33
20 70
- 18 - 66
20 40
- 18 - 33
20 70
.. ..
. .

Toda fraccin decimal con innito nmero de cifras decimales despus de la coma o punto se denomina
fraccin decimal innita. No se excluyen los nmeros con innito nmero de ceros. As, por ejemplo, 3;75 =
a
3;75000 : : : tambin es una fraccin decimal innita. De aqu se desprende que cualquier fraccin comn puede
b
ser representada como fraccin decimal innita.

Si como resultado de la divisin de 2 nmeros decimales nitos (es decir, enteros, o decimales con nmero
nito de cifras despus del punto o coma) obtenemos un nmero decimal con innito nmero de cifras despus
de la coma o punto, pero con un grupo de cifras que se repiten sucesivamente, entonces tenemos un nmero
denominado, fraccin decimal peridica. El grupo de cifras que se repiten sucesivamente se denomina periodo,
y se lo representa con su grupo de cifras con una barra encima, o con la letra T . As, por ejemplo, 0;33,
1;412, 0:201, 27;1230123, 0;0804321, son fracciones decimales (innitas) peridicas, con periodos: T = 3,
T = 12, T = 201, T = 123, T = 4321, respectivamente. Cabe notar que los periodos T = 0 y T = 9 son
impropios y que, por ejemplo, las fracciones decimales con estos periodos obedecen a las reglas de transformacin:
7;10 = 7;100 : : : = 7;1, 3;040 = 3;0400 : : : = 3;04; 0;29 = 0;299 : : : = 0;3, 6;19 = 6;199 : : : = 6;2.

| Si en una fraccin decimal (innita) peridica, el periodo T empieza enseguida despus de la coma o punto
decimal, entonces la fraccin es peridica pura (por ejemplo, 0:12, 18:4, 123:321), y si entre el punto o coma
hay otras cifras, entonces la fraccin es peridica mixta (por ejemplo, 0;815, 12;74, 49;123321).

Conversin de una fraccin decimal (innita) peridica a fraccin comn. Antes de describir
la regla de conversin, analicemos los siguientes ejemplos, donde se notar claramente que primero se debe
transformar la fraccin decimal peridica a fraccin decimal peridica pura.

Ejemplos:

Convertir las siguientes fracciones decimales peridicas a fracciones comunes:

1. a) 4:3, T = 3; b) 2:273, T = 273; c) 0;254, T = 54; d) 3;69, T = 9.


1.3. NMEROS RACIONALES Y SUS OPERACIONES 29

39 13
Solucin: a) Sea x = 4:3 ) 10x = 43:3 ) 10x x = 43:3 4:3 = 39 ) 9x = 39 ) x == .
9 3
b) Sea x = 2:273 ) 1000x = 2273:273 ) 1000x x= 2273:273 + 2:273 ) 999x = 2271, por tanto
2271 757
x= = .
999 333
252 14
c) Sea y = 0;254 ) 10y = 2:54 ) 1000y 10y = 254:54 2:54 ) 990y = 252 ) y = = .
990 55
333
d) Sea y = 3;69 ) 100y 10y = 369:9 36:9 ) 90y = 333 ) y = = 3;7. Lo que signica que
90
3;69 = 3;699 : : : = 3;7 = 3;700 : : :

2. a) 2;456, T = 6, b) 3;301212, T = 12, c) 0;0249, T = 9, d) 4;5312, T = 312


6 2
245:6 245 + 245 +
Solucin: a) Sea 2;456 = = 9 = 3 = 737 .
100 100 100 300
12 4
330:12 330 + 330 +
b) 3;301212 = = 99 = 33 = 5447 .
100 100 100 1650
9
24:9 24 +
c) 0;0249 = = 9 = 25 = 1 . Lo que signica que 0;0249 = 0;02499 : : : = 0;025 =
1000 1000 1000 40
0;02500 : : :
312 104
45:312 45 + 45 +
d) 4;5312 = = 999 = 333 = 15 089 .
10 10 10 3330

Recuerda los pasos de conversin de fraccio-


Ejemplo: Ejemplo:
nes decimales peridicas a fracciones comunes:
Se convierte la fraccin decimal peridica dada 126:15 30:102
1) 12;615 = = 0;30102 =
a fraccin decimal peridica pura. 10 100
Se descompone la fraccin en parte entera ms 126 + 0:15 30 + 0:102
2) = =
parte decimal peridica pura. 10 100
Se representa a la parte decimal peridica en la 15 102
T 126 + 30 +
3) forma , donde el denominador tiene tan- 99 = 999 =
9:::9 10 100
tos nueves como cifras tenga el periodo T .
5 34
126 + 30 +
4) Se realizan las simplicaciones del caso. 33 = 4163 333 = 2506
10 330 100 8325

Practico lo que aprend:

Convertir los siguientes expresiones decimales a fracciones comunes:

1. a) 0;1215 , b) 1;218, c) 3;030123, d) 1:0153, e) 6;0405, f ) 3:012, g) 3;006, h) 0;030


15 5
3012:15 3012 + 3012 + 99 401
Se tiene, por ejemplo, 0;301215 = = 99 = 33 = .
10000 10000 10000 330 000
3:12 1;015 0;012
2. a) 2;456, b) 0;54, c) 0;09, d) 24;53, e) 10;3504, f )
, g) , h)
2;454 0:15 3:18
Simplicar las expresiones y dejar el resultado en forma de fraccin comn nica:
0 12
4 3 1 2
B 2
1 3 4 C 1 2 1 1
3. a) 1:6 + 0:12 2:3
, b) @ A 4 , c) 6:3 + 0:15 3:3
1 3 1
8:12
3
Hallar los valores de x, y, z:
30 CAPTULO 1. PRELIMINARES ARITMTICOS

4 2
3 3
50 2 3
(16)
4
(54)
3 ( 2) (32) 162 33
x y z
4. a) 3 = 2 3 5 , b) 1 4 3 = 2x 3y 7z
4 2 (70) ( 8) (54) (49 2) 3
81 ( 100) (33 )
! 1 ! 1
3 2 2 3 2 2
( 2) 27 (8) (81) ( 2) 162 (16) (27)
5. a) 3 3 = 2x 3y , b) 3 3 = 2x 3y
( 4) (72) ( 4) (108)
Simplicar las expresiones, dejando los resultados en potencias de 2 y de 3:
0 1 2
3 4 1 2 3 1 1 2
3
1 1 3
2
2 3 24 B 6 8 12 3 (8) C
6. a) 2 , b) 512 @ A
2 1 3 1 1 2 3
3 3 6 6 4

1
Porcentajes Entre las fracciones decimales, la ms utilizada en la prctica corresponde a 0;01 = , la
100
misma que se denomina porcentaje y se representa mediante 1 %. Como 1 % = 0;01, entonces 2 % = 0;02,
8 % = 0;08, 10 % = 0;1, 15 % = 0;15, 120 % = 1;2, 350 % = 3;5, etc. Muchos clculos estadsticos, nancieros,
econmicos, agrcolas, etc. se los hace usando porcentajes. As, por ejemplo, para calcular el 35 % del contenido
tomado de agua de un tonel de 500 litros, tendremos: 0;35 500 = 175 litros, lo que signica que el 35 % de 500
es 175.

Ejemplos:

1. Un obrero mecnico debe producir 80 piezas diarias. Al acabar su da de trabajo result que cumpli con
el 120 % de su tarea del da. Cuntas piezas elabor?
Solucin: Tenemos que 120 % = 1;2, entonces 80 1;2 = 96 piezas fueron elaboradas en ese da, es decir,
hubo un incremento de 16 piezas. En calidad de comprobacin podemos decir que si 80 piezas representan
al 100 %, entonces el incremento de 16 piezas corresponder (por regla de 3: 80 es a 100, 16 a cunto
16 100
ser?) a = 20, es decir, a un 20 % de incremento en la produccin de ese da.
80
2. Un estudiante debe preparar 80 preguntas en una semana de lunes a domingo. Hasta el domingo result
que prepar 60 preguntas. Qu porcentaje de la tarea cumpli hasta el domingo?
60
Solucin: Tenemos que la tarea cumplida puede ser representada con la fraccin , que expresada en
80
60
porcentaje ser = 0;75 = 75 %. Por tanto, al estudiante le queda todava por contestar al 25 % de
80
preguntas, es decir, 0;25 80 = 20 preguntas.

3. Un tractor removi en 9 horas 120 toneladas de tierra, que representan al 75 % de la obra total. Cuntas
toneladas de tierra faltaran para completar la obra y en cuntas horas ms?
Solucin: Tenemos que el trabajo cumplido es 75 % x = 120, donde x es el valor de la obra total, es
120
decir, 0;75 x = 120 ) x = = 160 toneladas. Por tanto, faltaran 40 toneladas de tierra todava por
0;75
160 9
remover y (por regla de 3: 120 es a 9, 160 a cunto ser?) de la relacin = 12 se desprende que
120
faltan todava 3 horas de trabajo.

4. Se sabe que el impuesto al valor agregado o i.v.a. es del 12 %. Cierta persona, por la compra de un terno
paga $120 includo el impuesto. Cul es el valor bruto del terno?
Solucin: Supongamos que el valor bruto del terno es x, entonces el i.v.a. ser de 12 % x y tendremos que
120
su suma ser x+12 % x = $120 ) x+0;12x = 120 ) (1 + 0;12) x = 120 ) x = = 107;14. Por tanto,
1;12
el valor bruto del terno, sin impuesto, es de $107;14. Como comprobacin tenemos: 107;14+12 % 107;14 =
107;14 + 0;12 107;14 = 120.
1.4. INTRODUCCIN A LOS NMEROS REALES Y SUS OPERACIONES 31

1.4. Introduccin a los Nmeros Reales y sus Operaciones

Pasamos a introducir los nmeros reales representados mediante R y compuestos por la unin de los nmeros
racionales Q e irracionales Q0 , es decir, R = Q [ Q0 .

Anteriormente expusimos sobre los nmeros racionales Q, sus operaciones y propiedades. Ahora expondremos
sobre los nmeros irracionales Q0 , los radicales y sus propiedades.

1.4.1. Nmeros irracionales y sus operaciones

Se denominan nmeros irracionales al conjunto Q0 de todos aquellos nmeros, imposibles de representarse


mediante una fraccin o quebrado n a=b, o acuya expresin decimal
o es innita y sin periodo. Los irracionales los
0
representaremos mediante Q = x : x 6= ; a; b 2 Z; b 6= 0 . Si un nmero escrito en forma decimal, contiene
b
un innito nmero de decimales, pero sin periodo, entonces el nmero siempre es irracional y por lo tanto no
a
puede ser escrito en forma de fraccin , con a, b 2 Z, b 6= 0. Ejemplos de nmeros irracionales son =
b r p
p 4 5 p
3;1415926 : : :, e = 2;718281 : : :, 2 = 1;41421356 : : :, 7 , 3 5 , etc. Cabe sealar, que no es nada fcil
3
mostrar que cierto nmero, por lo general raro, es irracional, adems, en virtud de que todo nmero irracional
posee una expresin decimal innita irregular, entonces no p es posible su total escritura en forma decimal,
p p 13 4
13
sino slo simblica. Expresiones del tipo 17 3, 3 5, 2
, etc. son realmente smbolos y su verdadero
3
valor, apreciable para nosotros, slo es posible en forma aproximada y con ayuda de nmeros racionales en
15707963
expresin decimal nita o fraccin. As, por ejemplo, los muy conocidos nmeros 3;1415926 = ' ,
5000000
35355339 p
1;41421356 = ' 2 son ejemplos de aproximaciones racionales de esos nmeros irracionales.
25000000
| No se debe confundir el valor de un nmero irracional con su aproximacin racional, decimal, as: =
15707963 p p 35355339
3;1415926 : : :, pero ' 3;1415926 = ; y 2 = 1;41421356 : : :,pero 2 ' 1;41421356 = .
5000000 25000000
| Un nmero irracional, expresado mediante radicales (los otros nmeros irracionales , e, etc. no!) viene
p
representado mediante expresiones del tipo n a (a es la cantidad subradical y n 2 N es el ndice) o sus combi-
naciones nitas de sumas, restas, multiplicaciones, cocientes, potencias enteras o fraccionarias.

Nmeros reales y la recta numrica

Los nmeros racionales e irracionales forman los nmeros reales y cada real corresponde a un nico punto
!
de la recta numrica de origen de coordenadas O, direccin positiva Ox y escala de medicin igual a 1 o valor
de la longitud del segmento [0; 1] :

| La recta o eje numrico real no posee vacos o huecos, pues la unin de todos los nmeros racionales
Q e irracionales Q forman el continuo de puntos reales, y cada punto corresponde a un nico nmero real o
representante y viceversa.

| Todos los nmeros reales son comparables: a) El nmero real a es mayor que el real b, y se escribe a > b,
si la diferencia a b es un nmero positivo (mayor que 0). b) El nmero real a es menor que el real b, y se
escribe a < b, si la diferencia a b es un nmero negativo (menor que 0). c) El nmero real a es mayor o igual
que el real b, y se escribe a b, si la diferencia a b es un nmero positivo o nulo (mayor o igual que 0). d) El
nmero real a es menor o igual que el real b, y se escribe a b, si la diferencia a b es un nmero negativo o
nulo (menor o igual que 0). Los smbolos >, < se denominan smbolos de desigualdades estrictas, y , son
los smbolos de desigualdades no estrictas.
32 CAPTULO 1. PRELIMINARES ARITMTICOS

2
Para comparar los nmeros 2=3 y 0;67 formamos la diferencia 2=3 0;67 y hallamos su valor: 0;67 =
3
2 67 2 100 67 3 1 2
= = < 0, que nos dice que es negativa y por tanto < 0;67.
3 100 300 300 3

Figura 1.1: El eje numrico real compuesto de puntos que representan a todo el continuo de los nmeros
racionales e irracionales.

Proporciones Sean a; b; c; d nmeros reales, distintos de 0 para los que se cumple la igualdad a b = c d.
a c
Esta igualdad entre dos razones, se denomina proporcin y se la representa mediante = . a y c son los
b d
antecedentes, b y d son los consecuentes. Los valores a y d se denominan extremos y los valores b y c medios
a c
de la proporcin = . As, por ejemplo, entre los nmeros 3, 5 y 6, 10 podemos armar la igualdad de dos
b d
razones o proporcin: 35 = 106 .
a c
| En una proporcin = , cualquiera de sus trminos (cuarta proporcional) puede ser obtenido a partir de
b d
x c b c a c b c a c a d
los 3 trminos restantes. As, por ejemplo, si = ) x = ; si = ) x = ; si = ) x = ;
b d d b x a x d c
a x a d a c
si = ) x = . As mismo, la media proporcional de la proporcin = vienen a ser dos medios o
b d b b d
a x x c
extremos iguales: = ) x2 = a d o = ) x2 = b c.
x d b x

Propiedades de las proporciones I: Representacin: Ejemplos:


El producto de los extremos es igual a c 4 8
= ) ad = bc = ) 4 10 = 5 8
al producto de los medios b d 5 10
a c d c 4 8 10 8
Los extremos pueden ser intercambiados = ) = = ) =
b d b a 5 10 5 4
a c d c 4 8 4 5
Los medios pueden ser intercambiados = ) = = ) =
b d b a 5 10 8 10

Propiedades de las proporciones II: Representacin: Ejemplos:


En una proporcin pueden ser invertidas a c b d 4 8 5 10
= ) = = ) =
las razones. b d a c 5 10 4 8
En una proporcin pueden ser sumados o a c a b c d 4 8 4 5 8 10
= ) = = ) =
restados los denominadores de las razones b d b d 5 10 5 10
En una serie de razones iguales, la suma
a c e 4 8 2
= = = ) = = = )
b d f 5 10 2;5
de los antecedentes es a la suma de los
consecuentes como uno cualquiera de los a+c+e+ a 4+8+2+ 4
= =
antecedentes es a su consecuente b+d+f + b 5 + 10 + 2;5 + 5

a c e g
| Si se tiene una cadena de proporciones = = = = , entonces cada de stas tiene un valor jo
b d f h
1.4. INTRODUCCIN A LOS NMEROS REALES Y SUS OPERACIONES 33
a c e g
(cociente), es decir,
= = = = , y de aqu se deduce que a = b , c = d , e = f , g = h . Ahora,
b d f h
sumando miembro a miembro estas ltimas igualdades, tendremos que a + c + e + g = b + d + f + h ,
entonces
a+c+e+g a
a + c + e + g = (b + d + f + h) ) = = , etc.
b+d+f +h b

Operaciones con radicales

Sean a 0 y n un natural mayor que 1, entonces existe y en forma nica un nmero no negativo x, tal se
que cumple la igualdad xn = a. El nmero x se denomina raz aritmtica de ndice n del nmero no negativo
p
a y se representa con n a. El nmero a se denomina cantidad subradical y el natural n > 1 ndice del radical.
p p p p
As, por ejemplo, 2 4 = 2, 3 27 = 3, 4 16 = 2. En denitiva, se tiene ( n a = x , xn = a; a 0; x 0).

Cuando a < 0 y n es un natural impar, el nmero nico y no positivo x que cumple con la relacin
p p
n
a = x , xn = a, se denomina raz de ndice impar de un nmero negativo. As, por ejemplo, 3 8 = 2,
p
7
p
1 = 1, 5 32 = 2.
p
| Cuando a < 0 y n es un natural par, la expresin n a no tiene sentido real y la igualdad xn = a no se
p
cumple para ningn nmero real x. En este caso, el valor n a se denomina imaginario puro. Por ejemplo, los
p p p p p p p p
valores 2 1, 4 8, 2 9 = 3 2 1, 4 16 = 2 4 1, 6 1, 6 32, etc. son imaginarios puros.

| En la siguiente tabla tenemos un resumen de las operaciones bsicas de radicacin. Generalmente se


considera que la cantidad subradical no es negativa y que los ndices pueden ser naturales, enteros o racionales.

Recuerda las opera-


Regla: Ejemplos:
ciones con radicales:
p q 1
1
p 1 Expresin con expo-
2
3 = 3 2 , 5 47 = 74 5 ,
m
a=a m
p 1
q
2=5
nente fraccionario. 0;2
0;5 = (0;5) 0;2 , 5=2 23 = 32
p3
p p p p p
p p
p m Separar los factores 6 = 3 2 3 3, 5 3;2 = 5 1;6 5 2,
m
ab = m
a b q p 1=3 q p p p
1=3 9 1=3 3 0;25
del subradical. 2 = 3 2, 12 = 0;25 3 0;25 4
p p p p
p p Elevar a m el factor
2
3 2 5 = 32 5, 2 0;3 3 =
0;3
20;3 3
a m b = m am b p3
p
3
p
1=3
p
1=3
externo e introducirlo. 4 5 = 43 5, 2 3= 21=3 3
r p2
r p
2 3 3 3 0;25 0;25
3
r p = p , = p ,
a m
a Separar numerador y de- 4 2
4 0;451 3
0;451
m
p
= m r p3
r p
2=3
b b nominador del subradical 2 2 2=3 3 3
3
= p , = 2=3p
1;7 3
1;7 1;5 1;5
p 4 p3 4 p 2
p n p n Elevar la cantidad sub- 3
2 = 24 = 2 3 , 5
7 = 72=5
( m a) = m an = a m p 2 p
0;4 2 p 3
radical a la potencia n. 0;4
3 = 32 = 2 0;4 , 2=3 5 = 59=2
p p p 1=3 p p p
p p p Multiplicar los
3 2
5 = 3 2 5, 6
3= 25
n m
a= mn
a p3 1=2
p p p p
0;2 0;3
p
ndices. 7 = 3=2 7, 4 = 0;06 5
p p p p
p
mn
p
n Simplicar ndice
3 2
45 2 = 45 ,
3 15
512 = 54
5

akm = ak p p p4
p2
con exponente. 18
1;511;2 = 3 1;510;2 , 3 6= 3 3

Practico lo que aprend:

1. Extraer del radical aquellos factores posibles:


p p p p p p p p
1) a) 120, b) 4 80, c) 3 32, d) 3 512, e) 3 5000, f ) 2 2352, g) 4 3888, h) 2 18 000
p p q p 3 q p 5 q p 7
3 4
2) a) 1080, b) 405, c) 450 4 17 , d) 108 2 3 , d) 243 1 + 2
34 CAPTULO 1. PRELIMINARES ARITMTICOS

2. Introducir al interior del radical los factores externos y luego simplicar:


r r p r
p 3 5 3p 2p 3p3 1p4 25 9 3 4 25
1) a) 4 3, b) 3 , c) 8, d) 6 e) 4, f ) 6, g) , h)
12 2 3 2 3 3 16 5 54
r r
p p 2 9 3 16 1p 1p 1 p
2) a) 2 2 2, b) 4 , c) 3 , d) 3 125, e) 4 128, f ) p 8 81
3 4 2 27 5 2 3
3. Simplicar las expresiones:
p p p p p p p p p p p p
1) a) 2 5 48 + 3 40 12 2 15 27, b) 176 2 275 + 1584 891, c) 2 3 0;125 + 4 0;0016
q p q p r q
pp p
3
p pp 2 3 4
p 2p
3
p 4
p 3
p p p p
8
p 2
3
3 4 4 3 8
2) a) 5 5 5 , b) 16 8 2 32 2 2 4 4, c) 3 9 27 3 312 3 3

p p
4. Simplicar las expresiones y dejar el resultado slo en funcin de 2y 3:

3
p p p p p 2 p p 1
p p p 2
1) a) 2 2 2 3 3 2 + 3 12 + 2 27 , b) 2 2 2 3 3 3 2+ 12 2 27
4
p p 1
p p p p p p p p
2) a) 3 3 2 5 3 2 3 2 + 2 12 2 27 , b) 43 2 2 4 3 5 2 + 2 12 3 27
3 4
1 2 2 3 2 1 4 3
3) a) 27 3 8 3 32 5 81 4 , b) 100 2 64 3 0;250;5 16 0;75

h 1 4 5 1 1
i 23 3 1
2
3 1 1
1
2
3
2
2 2
4) a) 3 3 5 3 2 4 16 5 2 4 3 2 , b) 9 2 16 4 162 64 4 72 2

5. Expresar las siguientes cantidades con radicales de un mismo ndice:


r r r r r r r
p3
p p 6
p
4
p p 5
p3 3 4 4 5 6 2 1 3 1 4 1 5 1
1) a) 3, 2, 5, 7; b) 5, 3, 2; c) , , ; d) , , ,
9 3 3 2 3 4 5
p p
2) Reduciendo a comn ndice, averiguar cual de los siguientes nmeros esqel mayor: a)
q 2 o 3 4;
p p p p p p p p
7
p 3
pp p
4 5 p
4 3
b) 8 10 o 4 3; c) 5 5 o 6 6; d) 4 2 o 5 3; e) 6 24 o 48; f ) 25 o 28

Racionalizacin El proceso algebraico mediante el cual se consigue eliminar los radicales del denominador
P
de cierta fraccin , pero sin alterar su valor, se denomina racionalizacin del denominador. Anlogamente
Q
se dene la racionalizacin del numerador. Este proceso siempre es factible (pero no siempre es fcil!) si se
encuentra el as denominado factor racionalizante.

p
3
p p p
3
2 2 2 32 239 239 32
Al racionalizar p 3
, tendremos: p
3
= p
3
p
3
= p
3
= , donde p
3
es el factor racional-
3 3 3 p 3 2
p 3 3 3p p 32 p
5 5 5 5
3 3 3 22 3 22 3 22 354 22
izante. Para p , tenemos: p = p
5
p5
= p5
= = , donde p
5
es el factor
258 258 2 23 22 2 25 2 2 4 22
racionalizante.
p p p p p p
1 1 5 2 3 5 2 3 5 2 3
Para racionalizar p p , tendremos: p p p p = p 2 p 2 , donde p p es
5+2 3 5+2 3 5 2 3 5 2 3 5 2 3
p p p p p p
5 2 3 5 2 3 2 3 5
el factor racionalizante. En denitiva nos da = = .
5 4 3 7 7
p p p p
1 1 3 2+2 3 3 2+2 3
Para racionalizar p p , tendremos p p p p = p 2 p 2 que nos da
3 2 2 3 3 2 2 3 3 2+2 3 3 2 2 3
p p p p
3 2+2 3 3 2+2 3
= .
9 2 4 3 6
Para expresiones con 3 trminos o radicales en el denominador,
p la p
racionalizacin
p se procede
p p porppartes
1 1 2 3 2 + 5 2 3 2+ 5
y agrupando, as: p p p = p p p p p p = p p 2 p 2,
2 3 2 5 2 3 2 5 2 3 2 + 5 2 3 2 5
1.4. INTRODUCCIN A LOS NMEROS REALES Y SUS OPERACIONES 35
p p p p p p p p p
2 3
2+ 5 2 3 2+ 5 2 3 2+ 5
que nos da p 2 p p p 2 = p = p , que por no estar
2 3 2 2 3 2+ 2 25 12 4 6 + 2 25 4 6 11
todava racionalizada, tendremos
p p p p p p p p p p p
2 3 2+ 5 2 3 2+ 5 4 6 + 11 2 3 2+ 5 4 6 + 11
p = p p = p 2 ;
4 6 11 4 6 11 4 6 + 11 4 6 112
p p p p p p p p p
22 3 11 2 + 11 5 + 4 2 6 8 3 6 4 5 6
que despus de simplicar toma la forma p 2 o lo que
4 6 112
p p p p p p p p p p
22 3 11 2 + 11 5 + 8 3 24 2 4 30 35 2 30 3 11 5 + 4 30
es lo mismo = .
16 6 121 25
p p
1 1 1 5+ 2 3
Para racionalizar p p , tendremos p p = p p p p =
5+ 2+3 5+ 2+3 5+ 2 +3 5+ 2 3
p p p p p p
5+ 2 3 5+ 2 3 5+ 2 3
p p 2 que desarrollando nos da p 2 p p p 2 = p =
5+ 2 32 5 2 5 2+ 2 9 5 2 10 + 2 9
p p p
5+ 2 3 2 10 + 2
p p , es decir,
2 10 2 2 10 + 2
p p p p p p p p p p p p p
5+ 2 3 2 10+2 2 2 2 5+6 10 2 2 10+2 5 10 6 12 2 6 5+6 10 6
p 2 = = :
2 10 22 4 10 4 36

Recuerda las reglas para racionalizar:

p
m
p
m
p
m
p
m
A A Bm 1 A Bm 1 A Bm 1 A Bm 1
p =
1. m p
m p
m
p
= m = p
m
=
B Bm B
B Bm 1 B Bm 1
| {z }
factor racionalizante
p p p p
2. A partir de la identidad b2 a2 = (b a) (b + a), se tiene que b a = b+ a b a y la
racionalizacin:
p p p p
1 1 b a b a
p p =p p p p =
b a b a b a b a
| {z } | {z } | {z }
denom inador irracional factor racionalizante fraccin racionalizada

p
3 p p
3
p
3
p
3
3. Con ayuda de la identidad b a= b 3
a b2 ab + a2 , tenemos la racionalizacin:
p
3
p
3
p
3
p
3
p
3
p
3
1 1 b2 ab + a2 b2 ab + a2
p p = p p p p p =
3
b 3
a 3
b 3
a 3 b2 3 3
ab + a2 |
b a
{z }
| {z } | {z }
fraccin irracional factor racionalizante fraccin racionalizada

Con la identidad bn an = (b a) bn 1
abn 2
+ a2 bn 3
, n 2 N y n par o impar, + an 2
b an 1

1
es posible racionalizar los denominadores de fracciones irracionales del tipo p
p p , donde debemos
b q
a
tomar un n que sea igual al mnimo comn mltiplo de p y q, es decir, n = m:c:m: (p; q) :

Practico lo que aprend:

Racionalizar y simplicar las expresiones:

r
1 2 2 3 1 4 2 1 1
1. a) p
5
, b) p , c) p
4
, d) p , e) p , f) , g) p , h) p p
3 3 5 2 2 3 233 3 3 2 2 2 3 5
1 1 3 1 3 1
2. a) p p , b) p p , c) p p , d) p p , e) p , f) p p
3 2 3 2 2 5+2 2 5 3 4 2 332+2 233 332
36 CAPTULO 1. PRELIMINARES ARITMTICOS

1 1 1 1
3. a) p p , b) p p , c) p p , d) qp p
qp
p
x y x+y x y+1 2+ x y x y+ x+ y

1 1 1 1 1
4. a) p p , a) p p , b) p p , c) p , d) p
1 3 2 2 1 2 3+3 2 233+332 a b+2 c a b+23c
p p p
1 1 2 2 3 2
5. a) p p , b) p p , c) p p , d) p
3
p
3 a+2 2 3 a+2 2 3 2 2 1 3 2 2 1

1 1 1 1
6. a) p p , b) p p p , c) p p
3
, d) p
3
p
3
p
3
3 2 2 1 3 3 2 2+ 5 3 2 1 3 2 2 5
r p r p
p p A+ A2 B A A2 B
7. Demostrar la frmula de radicales dobles: A B=
2 2

8. Utilizando la frmula anterior, mostrar la validez de las siguientes igualdades:

p p q p
p p p p p p
a) a) 10 4 6 = 6 2, b) 3 2 2 = 2 1, c) 17 4 9 + 4 5 = 5 2:
p p p p p p p p p p p p
b) a) 2 3, b) 3 + 2 2, c) 5 + 24, d) 5 2 6, e) 9 + 17, f ) 3 5:

p p p p
9. La diferencia 57 40 2 40 2 + 57 es un nmero entero. Hallar este nmero.
p
p
3
p 1 5 p
3
p p
3
p
10. Usando alguna transformacin, comprobar que: a) 2 + 5 = + , b) 5 2 + 7 5 2 7 = 2:
2 2

1.4.2. Intervalos numricos. Valor absoluto de un nmero real

Intervalos numricos Pasamos a describir algunos subconjuntos numricos de R, representables en la recta


numrica mediante puntos, intervalos, segmentos, semirectas, etc. (ver tabla resumen). Empezamos con-
siderando que R =] 1; +1[ y que cualquier nmero real a cumple con 1 < a < +1:

Propiedades de las desigualdades numricas Para cualesquiera nmeros reales a, b, c, d se cumplen las
siguientes propiedades:

Recuerda las leyes sobre


Ejemplos: Observaciones:
las desigualdades en R:
Reciprocidad de
1) Si a > b, entonces b < a: Si 5 > 2 ) 2 < 5:
la desigualdad.
Si a > b y Si 7 > 5 y 5 > 3 Transitividad de
2)
b > c, entonces a > c: ) 7 > 3: una desigualdad.
Si a > b, entonces Si 7 > 5 Suma de trminos
3)
a + c > b + c: ) 7 + 3 > 5 + 3: a una desigualdad
Si a > b y c > 0, Si 7 > 5 y c = 2 > 0 Producto de desigualda-
4)
entonces ac > bc: ) 7 2 > 5 2: des del mismo sentido.
Si a > b y c < 0, Si 7 > 5 y c = 2 < 0 Producto de desigualda-
5)
entonces ac < bc: ) 7 2 < 5 2: des de sentidos contrarios
1.4. INTRODUCCIN A LOS NMEROS REALES Y SUS OPERACIONES 37

Leyes: Ejemplos: Observaciones:


Si a > b y c > d, Si 7 > 5 y 4 > 2 Suma de desigualdades
6)
entonces a + c > b + d: ) 7 + 4 > 5 + 2: del mismo sentido.
Si a, b, c y d son > 0, siendo Si 6, 5, 4, 2 son > 0, con 6 > 5 Producto de desigualda-
7)
a > b, c > d ) ac > bd: y 4 > 2 ) 6 5 > 4 2: des del mismo sentido
Si a > b y c < d, Si 6 > 3 y 5 < 7 Diferencia de desigualda-
8)
entonces a c > b d: ) 6 5 > 3 7: des de sentidos contrarios
Si a > b > 0, entonces Si 9 > 4 > 0, entonces Inversa de una
9) 1 1 1 1
< : < : desigualdad.
a b 9 4
Si a > b > 0, entonces para Si 9 > 4 > 0, entonces Potencia natural de
10)
cualquier n 2 N : an > bn : 93 > 43 : una desigualdad.

Valor absoluto Se denomina valor absoluto (o mdulo) del nmero real a al mismo nmero a, si a 0,
y a su opuesto a, si a < 0. El valor absoluto o mdulo de a se representa mediante jaj. De acuerdo a esta
denicin tendremos que (
a, si a 0
jaj = :
a, si a < 0

Tendremos, por ejemplo, que: j 4j = ( 4) = 4, j7;2j = 7;2, j0j = 0, j 2j = 2, j 5j = ( 5) =


p p p p p p p p p
5 , 2 3 = 2 3 =3 2, 2 3+ 2+1 = 2 3+ 2+1 =2 3 2 1,
( (
a b, si a b 0 a b, si a b
ja bj = =
(a b) , si a b < 0 (a b) , si a < b

| El valor de jaj es siempre positivo o nulo y puede ser interpretado geometricamente como la distancia del
punto a al origen O en la recta numerica. Esta distancia la representaremos con d (a; 0) = jaj. As, por ejemplo,
d ( 3; 0) = j 3j = 3, d (5;32; 0) = j5;32j = 5;32.

| La distancia entre 2 puntos a y b del eje real se dene como el valor d (a; b) = ja bj. As, por ejemplo,
d ( 3; 0) = j 3 0j = 3, d (5; 2) = j5 2j = 3, d ( 7; 2) = j 7 2j = 9, d ( 1; 5) = j 1 + 5j = 4. Es evidente
que d (a; b) = ja bj = d (b; a)

| Para todo nmero real a se cumple la relacin evidente jaj a jaj.

| Para demostrar las desigualdades triangulares ja + bj jaj + jbj y d (a; b) d (a; c) + d (c; b) (que es
la misma que ja bj ja cj + jc bj), nos apoyamos en que para todo real se cumple jaj a jaj y
jbj b jbj, las mismas que al sumarlas miembro a miembro nos dan: jaj jbj a + b jaj + jbj, es decir,
(jaj + jbj) a+b jaj + jbj, que signica que ja + bj jaj + jbj. Si en esta ltima desigualdad ponemos
a = x z y b = z y, entonces tendremos jx z/ + z/ yj jx zj + jz yj, l.q.q.d.

| La igualdad (ecuacin) del tipo jx 3j = 2 equivale a d (x; 3) = 2 y pide hallar todos los valores de x cuyas
distancias al punto 3 sean iguales a 2. Tendremos las equivalencias jx 3j = 2 ) x 3 = 2 ) (x 3 = 2)
o (x 3 = 2), de donde x = 5 o x = 1 son los puntos buscados. As mismo, la igualdad (ecuacin) del tipo
5 5
j2x + 5j = 6 equivale a 2 x = 6 o d x; = 3 y pide hallar todos los valores de x cuyas distancias
2 2
5 5 5 5
al punto sean iguales a 3. Tendremos las equivalencias x = 3 ) x+ = 3 ) x+ =3 o
2 2 2 2
5 1 11
x + = 3 , de donde x = o x = 2 son los puntos buscados. Las ecuaciones j3 xj = 2 y j 2x 5j = 6
2 2
tienen idnticas soluciones que las 2 ecuaciones anteriores, pues j3 xj = jx 3j y j 2x 5j = j2x + 5j.

| Igualdades (ecuaciones) como j2x 5j = 4 no poseen solucin porque j2x 5j 0 y el lado derecho es
38 CAPTULO 1. PRELIMINARES ARITMTICOS

negativo. As mismo jx + 3j = 1, jxj + j3x 1j = 3, etc. son ejemplos de ecuaciones sin solucin por tener el
lado izquierdo positivo o nulo y el derecho negativo.

Tabla resumen de Intervalos numricos.

Recuerda las propiedades


Descripcin: Ejemplos:
del valor absoluto (v. ab.):
p p
No negatividad del valor j 5;2j = 5;2, 5 2 = 5 2
1) jaj 0 p p
absoluto. j0j = 0, 3 11 = 11 3
p p
Simetra del valor j2;7j = j 2;7j, 5 2 = 2 5
2) jaj = j aj p p
absoluto. j 0;4j = j0;4j, 1 2 = 2 1
V. ab. del producto es igual j 5 (2)j = j 5j j2j = 10,
3) jabj = jaj jbj
al producto de los v. ab. j 4 ( 2)j = j 4j j 2j = 8
5 j5j 5
= = ,
a jaj V. ab. del cociente es igual 2 j 2j 2
4) =
b jbj al cociente de los v. ab. 42 j 42j
= = 21
2 j 2j
2 2
2
jaj = a2 Propiedad del cuadrado j 4j = 42 , j1;3j = 1;32 ,
5) p 2 p q
2
a = jaj y de la raz. 32 = j3j, ( 2) = j 2j
d (a; 0) = jaj 0 Distancia no negativa. d ( 7; 0) = j 7j = 7, d (4; 0) = j4j = 4
d (a; b) = ja bj = d (b; a) Simetra de la distancia. d ( 2; 3) = j 2 3j = j 5j = d (3; 2)
6)
ja + bj jaj + jbj Desigualdad triangular. j 2 + 5j j 2j + j5j, j1 5j j1j + j 5j
d (a; b) d (a; c) + d (c; b) Desigualdad triangular. d (3; 2) d (3; 5) + d (5; 2) :

| La desigualdad (inecuacin) del tipo jx 3j < 2 equivale a d (x; 3) < 2 y nos pide hallar todos los
1.4. INTRODUCCIN A LOS NMEROS REALES Y SUS OPERACIONES 39

valores de x cuyas distancias al punto 3 sean menores que 2. Tendremos las equivalencias jx 3j < 2 ) 2 <
x 3 < 2 ) (x 3 > 2) y (x 3 < 2), de donde x > 1 y x < 5 son los valores buscados, que equivalen a
1 < x < 5 o x 2]1; 5[. En forma anloga, la desigualdad (inecuacin) del tipo jx 3j 2 tendra la solucin
1 x 5 o x 2 [1; 5], pero la desigualdad (inecuacin) del tipo jx 3j 2 no tendra solucin. As mismo,
5 5
la desigualdad (inecuacin) del tipo j2x + 5j < 6 equivale a 2 x < 6 o d x; < 3 y nos pide
2 2
5
hallar todos los valores de x cuyas distancias al punto sean menores que 3. Tendremos las equivalencias
2
5 5 5 5 11 1
x < 3 ) 3 < x + < 3 ) x + > 3 y x + < 3 , de donde x > y x = son los
2 2 2 2 2 2
11 1 11 1
valores buscados, que equivalen a < x < o x 2] ; [. En forma anloga, la desigualdad (inecuacin)
2 2 2 2
5 11 1 11 1
del tipo x 3 tendra la solucin x ox2[ ; ], pero la desigualdad (inecuacin) del
2 2 2 2 2
5
tipo x 3 no tendra solucin.
2

Interpretacin geomtrica del valor absoluto: d (a; 0) = jaj, d (a; b) = d (b; a) = ja bj, etc. La
desigualdad ja + bj jaj + jbj hace referencia a que en un tringulo un lado siempre es menor o igual a
la suma de los otros dos y por eso se denomina desigualdad triangular.

| La desigualdad (inecuacin) del tipo jx 3j > 2 equivale a d (x; 3) > 2 y nos pide hallar todos los valores
de x cuyas distancias al punto 3 sean mayores que 2. Tendremos las equivalencias jx 3j > 2 ) x 3 < 2 o
x 3 > 2, de donde x < 1 y x > 5 son los valores buscados, que equivalen a x 2] 1; 1[[]5; +1[. En forma
anloga, la desigualdad (inecuacin) del tipo jx 3j 2 tendra la solucin x 1 o x 5 o x 2] 1; 1][[5; +1[,
pero la desigualdad (inecuacin) del tipo jx 3j 2 tendra por solucin a todos los reales. As mismo, la
5 5
desigualdad (inecuacin) del tipo j2x + 5j > 6 equivale a 2 x > 6 o d x; > 3 y nos pide
2 2
5
hallar todos los valores de x cuyas distancias al punto sean mayores que 3. Tendremos las equivalencias
2
40 CAPTULO 1. PRELIMINARES ARITMTICOS

5 5 5 11 1
x > 3 ) x+ < 3 o x+ > 3, de donde x <y x > son los valores buscados, que equivalen
2 2 2 2 2
11 1 5
a x 2] 1 [[] ; +1[. En forma anloga, la desigualdad (inecuacin) del tipo x 3 tendra
2 2 2
11 1 5
la solucin x 2] 1 ] [ [ ; +1[, pero la desigualdad (inecuacin) del tipo x 3 tendra por
2 2 2
solucin a todos los reales.

Recuerda la solucin de
Ecuacin o
una ecuacin o inecuacin Ejemplos:
inecuacin:
con valor absoluto:
jx aj = b, b 0 x a= b jx 3j = 4 ) x 3 = 4 ) x = 1; 7
jx aj = b, b<0 ; jx 3j = 4 ) ;
jx aj < b, b 0 b+a<x<b+a jx 3j < 4 ) 4 < x 3 < 4 ) x 2] 1; 7[
jx aj b, b 0 b+a x b+a jx 3j 4 ) 4 x 3 4 ) x 2 [ 1; 7]
jx 3j > 4 ) (x 3 < 4) [ (x 3 > 4)
jx aj > b, b 0 (x < a b) [ (x > a + b) o (x 2] 1; 1[[]7; +1[)
jx aj b, b 0 (x a b) [ (x a + b) jx 3j 4 ) 4 x 3 4 )
( 1 x 7) o (x 2] 1; 1][[7; +1[)
j2x 3j < 4 ) 2 < x 3=2 < 2 )
jcx aj < b, b 0 a=c b=c < x < a=c + b=c x 2] 1=2; 7=2[
jcx aj > b, b 0 (x < a=c b=c) [ (x > a=c + b=c) j2x 3j > 4 ) (x < 3=2 2) [ (x > 3=2 + 2)
o (x 2] 1; 1=2[[]7=2; +1[)

Partes entera y fraccionaria de un nmero Sea x un nmero real. Se denomina parte entera de x al
mayor entero no superior a x, la representaremos mediante [x]. As, por ejemplo, [ 5;2] = 6, [2;999] = 2,
[ 0;85] = 1, [0;93] = 0, [2;35] = 2, [10] = 10. Se denomina parte fraccionaria de x a la diferencia entre
el nmero x y su parte entera, la representaremos mediante fxg, es decir, fxg = x [x]. As, por ejemplo,
f 5;2g = 5;2 ( 6) = 0;8, f2;999g = 2;999 2 = 0;999, f 0;85g = 0;85 ( 1) = 0;15, [0;93] = 0,
f2;35g = 2;35 2 = 0;35, f10g = 10 10 = 0.

| Las potencias del tipo b con b 2 R+ y 2 Q0 lo dejamos para el captulo de nmeros reales.

Practico lo que aprend:

1. Calcular el valor absoluto de las siguientes expresiones:


p p p p p p p p
a) j 3;2j, b) 5;4 + 5 , c) 3 + 5 , d) 4 5 , e) 2:4 5 , f) 1 + 5 7 , g) 5 2+ 3 ,
p p p p p p p p p p
h) 2 5 2 3 , i) 8 + 12 , j) 3 5 7 , k) 2 3 5 + 7 , l) 5 8+2 5 7.

2. Resolver las ecuaciones con valor absoluto e interpretarlas geomtricamente en R:


a) jxj = 3, b) j2xj = 5, c) j 3xj = 6, d) j2xj = 8, e) j3xj = 5, f ) jx 1j = 2, g) jx 2j = 4, h)
x p p x p p
2 = 8, i) 2x 3 = 3, j) 3 = 9, k) j 5 + 2xj = 1, l) 2 2x 3 = 2 + 3
2 3
3. Resolver las inecuaciones con valor absoluto e interpretarlas geomtricamente en R:
x
a) jxj < 2, b) 1, c) j 3xj 9, d) j2xj < 8, e) jxj > 3, f ) jx 1j 2, g) jx 4j < 4,
4
x x 2x x 2x
h) 1 7, i) j2x 1j < 1, j) 3 5, k) 1 + < 1, l) 3 3, m) 1 + > 1,
2 3 3 2 5
p p p p p p p p
n) 2 x 1 < 2 1, o) 1 3x + 2 2, p) 3 x 3 < 3, q) 3 2x 1 2 3,
p p p p p p p x 5 p
r) 2 2x 2 2 2, s) 2 3x 1 > 2 1, t) 2 x + 2 2, u) 3 > 3.
2
1.5. INTRODUCCIN A LOS NMEROS COMPLEJOS 41

1.5. Introduccin a los Nmeros Complejos

Los nmeros complejos aparecen gracias al problema de que en R no es posible resolver cualquier ecuacin
cuadrtica con coecientes reales, por ejemplo, x2 + 1 = 0, pues el cuadrado de ningn real x nos dar 1.

Se trata de ampliar los reales R hasta obtener un nuevo sistema de nmeros, tal que la ecuacin anterior
tenga solucin. Utilizaremos los puntos del plano como material de construccin de este nuevo sistema de
nmeros. As como los nmeros reales estn representados por puntos de una lnea recta (correspondencia
biunvoca entre todos los puntos de la recta y el conjunto de todos los nmeros reales), que es tan habitual que
no hacemos distincin alguna entre un nmero real x y su punto del eje correspondiente O (x).

Ahora deniremos un sistema de nmeros que sean representados por todos los puntos del plano R2 . Ten-
dremos que sumar o multiplicar los puntos del plano, lo que nos da derecho a elegir la denicin de estas
operaciones con los puntos, preocupndonos de que el nuevo sistema de nmeros posea las propiedades que
son el motivo de su creacin. Estas deniciones nos demostrarn que ningunas otras deniciones de las opera-
ciones, incluso las que a primera vista parecen ms naturales, nos conduciran al objetivo, que consiste en la
construccin de una ampliacin del sistema de nmeros reales R. Se demostrar que en esta construccin, la
sustitucin de los puntos del plano por otro material, no nos conducira a un sistema de nmeros diferente, por
sus propiedades algebraicas, del sistema de nmeros complejos C que construiremos a continuacin.

Supongamos que en el plano R2 se ha elegido un sistema rectangular de coordenadas xOy. Convengamos en


designar los puntos del plano con las letras z1 , z2 , z3 , etc. y en representar con la notacin (x1 ; y1 ) el punto z1 de
abscisa x1 y ordenada y1 , es decir, que como en la geometra analtica, escribiremos z1 = (x1 ; y1 ) y z2 = (x2 ; y2 ),
llamaremos suma z1 + z2 de estos puntos al punto que tiene la abscisa x1 + x2 y ordenada y1 + y2 , o sea,

z1 + z2 = (x1 ; y1 ) + (x2 ; y2 ) = (x1 + x2 ; y1 + y2 ) (1)

Llamaremos producto de los puntos z1 = (x1 ; y1 ) y z2 = (x2 ; y2 ) al punto de abscisa x1 x2 y1 y2 y ordenada


x1 y2 + y1 x2 , es decir

z1 z2 = (x1 ; y1 ) (x2 ; y2 ) = (x1 ; y1 )(x2 ; y2 ) = (x1 x2 y1 y2 ; x1 y2 + y1 x2 ) (2)

De este modo, hemos denido 2 operaciones algebraicas en el conjunto de todos los puntos del plano R2 .
Demostremos que estas operaciones poseen todas los propiedades que ellas mismas tienen en el sistema de
nmeros reales o en el sistema de nmeros racionales: son conmutativas y asociativas, estn ligadas por la ley
distributiva y para ellas existen las operaciones inversas, la diferencia y la ley de la divisin (excluyendo la
divisin por cero).

Las leyes conmutativas y asociativas de la suma son evidentes (o ms exactamente, se deducen de las
propiedades correspondientes de la suma de los nmeros reales), puesto que al sumar los puntos en el plano,
se suman por separado sus abscisas y sus ordenadas. La conmutatividad del producto se basa en que en la
denicin del producto, los puntos z1 y z2 gozan de simetra:

z1 + z2 = (x1 + x2 ; y1 + y2 ) = (x2 + x1 ; y2 + y1 ) = z2 + z1

z1 z2 = (x1 ; y1 )(x2 ; y2 ) = (x1 x2 y1 y2 ; x1 y2 + y1 x2 ) =


= (x2 x1 y2 y1 ; y2 x1 + x2 y1 ) = z2 z1 ;

adems, es fcil comprobar que

z1 + z2 + z3 = (z1 + z2 ) + z3 = z1 + (z2 + z3 ) ; z1 z2 z3 = (z1 z2 ) z3 = z1 (z2 z3 )


42 CAPTULO 1. PRELIMINARES ARITMTICOS

Pasamos a analizar la cuestin de las operaciones inversas. Si se han dado los puntos z1 = (x1 ; y1 ) y
z2 = (x2 ; y2 ), su diferencia ser un punto z = (x; y) tal que se cumpla z2 + z = z1 :

(x2 ; y2 ) + (x; y) = (x1 ; y1 ):

De aqu, en virtud de (2), se deduce que

x2 + x = x1 ; y2 + y = y1 :

Por lo tanto, la diferencia de los puntos z1 = (x1 ; y1 ) y z2 = (x2 ; y2 ), es el punto

z1 z2 = (x1 x2 ; y1 y2 ) (4)

y esta diferencia queda denida unvocamente. En particular, el origen de coordenadas (0; 0) sirve de cero, y
el punto opuesto al punto z1 = (x1 ; y1 ) ser el punto

z1 = ( x1 ; y1 ): (5)

Supongamos ahora que se dan los puntos z1 = (x1 ; y1 ) y z2 = (x2 ; y2 ), y que el punto z2 es diferente de cero, o
sea, que al menos una de las coordenadas x2 o y2 no es igual a cero y, por consiguiente, x22 + y22 6= 0: El cociente
de la divisin de z1 por z2 tiene que ser un punto z = (x; y) tal que z2 z = z1 , o sea, (x2 ; y2 )(x; y) = (x1 ; y1 ).
De aqu, en virtud de (3), se tiene que,

x2 x y2 y = x1 , y2 x x2 y = y1 :

resolviendo este sistema de ecuaciones, obtenemos que:


x1 x2 + y1 y2 y1 x2 x1 y2
x= , y= :
x22 + y22 x22 + y22
z1
Por lo tanto, para z2 6= 0, el cociente existe y se determina unvocamente:
z2

z1 x1 x2 + y1 y2 y1 x2 x1 y2
= ; : (6)
z2 x22 + y22 x22 + y22

Poniendo aqu z2 = z1 , obtenemos que en nuestra multiplicacin de los puntos la unidad es el punto (1; 0),
situado en el eje de abscisas a la distancia 1 del origen de coordenadas O a la derecha. Poniendo luego en (6)
z1 = 1 = (1; 0), obtenemos que, para z2 6= 0, el punto inverso a z2 es:

x2 y2
z2 1 = ; : (7)
x22 + y22 x22 + y22
1.5. INTRODUCCIN A LOS NMEROS COMPLEJOS 43

Por lo tanto, se ha construdo un sistema de nmeros representados por puntos del plano R2 , donde las opera-
ciones con esos puntos (nmeros) quedan denidas por las frmulas (2) y (3). ste conjunto de puntos (nmeros)
se denomina sistema de nmeros complejos C.

Ahora demostremos que este sistema C representa una ampliacin del sistema de nmeros reales R. Con
este n, veamos los puntos situados en el eje de abscisas, o sea, los puntos de la forma (x; 0); poniendo en
correspondencia al punto (x; 0), el nmero real x, obtenemos evidentemente una correspondencia biunvoca
entre el conjunto considerado de puntos y el conjunto de todos los nmeros reales. La aplicacin de las frmulas
(2) y (3) a estos puntos proporciona las igualdades

(x1 ; 0) + (x2 ; 0) = (x1 + x2 ; 0);


(x1 ; 0) (x2 ; 0) = (x1 ; 0)(x2 ; 0) = (x1 x2 ; 0);

o sea, los puntos (x; 0) se suman y se multiplican entre s, igual que los nmeros reales correspondientes. Por
lo tanto, el conjunto de puntos situados en el eje de abscisas Ox, considerado como una parte del sistema de
nmeros complejos, no se diferencia en nada por sus propiedades algebricas del sistema de nmeros reales,
representado ordinariamente por puntos de una recta. Esto nos permite no hacer a continuacin ninguna
distincin entre el punto (x; 0) y el nmero real x, o sea, que tendremos que (x; 0) = x. En particular, el cero
(0; 0) y la unidad (1; 0) del sistema de nmeros complejos resultan ser los nmeros reales ordinarios 0 y 1.

Tenemos que mostrar ahora que entre los nmeros complejos est contenida una raz de la ecuacin cuadrti-
ca (1), es decir, un nmero cuyo cuadrado sea igual al nmero real 1. Este ser, por ejemplo, el punto (0; 1),
o sea, el punto situado en el eje de ordenadas Oy a la distancia 1 del origen de coordenadas O, hacia arriba.
En efecto, aplicando la regla (3), obtendremos que:

(0; 1) (0; 1) = (0; 1)(0; 1) = ( 1; 0) = 1:

Designemos este punto mediante la letra i (de imaginario), de modo que i2 = 1:

Finalmente, demostremos que para los nmeros complejos introducidos se puede obtener su expresin ordi-
naria. Para esto, hallemos primero el producto del nmero real y por el punto i :

y i = (y; 0) (0; 1) = (y; 0)(0; 1) = (0; y);

por consiguiente, ste es el punto que tiene la ordenada y y est situado en el eje de ordenadas, adems todos
los puntos del eje de ordenadas se representan en forma de productos de stos. Si ahora (x; y) es un punto
arbitrario, en virtud de la igualdad (x; y) = (x; 0) + (0; y), se tiene denitivamente que: (x; y) = x + yi, o sea,
que verdaderamente llegamos a la expresin ordinaria de los nmeros complejos. La expresin (x; y) = x + yi se
denomina forma algebrica de un nmero complejo, por supuesto, en la expresin x + yi, la suma y el producto
se deben entender en el sentido de las operaciones denidas en el sistema de nmeros complejos C ya construdo.

Observacin: De la teora constructiva del sistema de nmeros complejos C, anteriormente expuesta, se


desprende que no existe una relacin de orden estricta entre los nmeros complejos z1 y z2 . No es posible
poner para z1 y z2 una relacin del tipo z1 > z2 o z1 < z2 , donde z1 = (x1 ; y1 ) y z2 = (x2 ; y2 ). No es posible,
por ejemplo, poner 3 2i > 2 + 5i o 3 2i < 2 + 5i.

La construccin expuesta del sistema de nmeros complejos nos lleva a la siguiente pregunta: se puede
obtener una suma y producto de puntos del espacio de tres dimensiones, de modo que el conjunto de stos forme
un sistema de nmeros complejos o hipercomplejos parecido al sistema de nmeros reales? Esta cuestin sale
fuera del contexto del presente curso, pero solamente sealaremos que la respuesta es negativa.
44 CAPTULO 1. PRELIMINARES ARITMTICOS

Recuerda las
operaciones Forma de par ordenado: Forma algebraica:
con complejos
Suma y (x1 ; y 1 ) (x2 ; y 2 ) = (x1 x2 ; y 1 y 2 ) (a + bi) (c + di) = (a c) + (b d) i
diferencia ( 3; 5) + (4; 1) = (1; 4) ; 3 + 5i + 4 i = 1 + 4i;
z = z1 z2 ( 1; 4) (3; 1) = ( 4; 5) 1 + 4i (3 i) = 4 + 5i
(x1 +y 1 i) (x2 +y 2 i) =
(x1 ; y 1 ) (x2 ; y 2 ) = (x1 x2 y 1 y2 ; x1 y2 +y 1 x2 )
Producto = (x1 x2 y 1 y2 ) + (x1 y2 +y 1 x2 ) i
( 3; 5) (4; 1) = ( 12 + 5; 3 + 20) = ( 7; 23)
z = z1 z2 ( 3 + 5i) (4 i) = 7 + 23i;
( 1; 4) (3; 1) = ( 3 + 4; 12 + 1) = (1; 13)
( 1 + 4i) (3 i) = 1 + 13i
x1 x2 +y1 y2 x1 +y1 i x1 x2 +y1 y2 y1 x2 x1 y2
Divisin (x1 ; y 1 ) (x2 ; y 2 ) = x22 +y22
; y1 xx22 +yx21 y2 x2 +y2 i = x22 +y22 + x22 +y22 i
2 2
(cociente) 3+5i ( 3+5i)(4+i) 17+17i
z1 ( 3; 5) (4; 1) = ( 1; 1) ; 4 i = (4 i)(4+i) = 16+1 = 1+i
z= 7 11 1+4i ( 1+4i)(2+i) 6+7i 6 7
z2 ( 1; 4) (3; 1) = 10 ; 10 2 i = (2 i)(2+i) = 4+1 = 5+5i

La suma de nmeros complejos denida anteriormente coincide en su esencia con la suma de vectores en el
plano que parten del origen de coordenadas O, resulta natural la siguiente pregunta: es posible denir para
ciertos valores de n el producto de vectores del espacio vectorial real de n dimensiones Rn , de modo que ste
sea, con respecto a la multiplicacin y a la adicin ordinaria de los vectores, un sistema numrico que contenga
al sistema de nmeros reales? Se puede demostrar que esto no se puede hacer si se quiere que se cumplan todas
las propiedades de las operaciones que tienen lugar en el sistema de nmeros racionales Q, reales R y complejos
C. En el espacio de cuatro dimensiones R4 esta construccin es posible si se prescinde de la conmutatividad de
la multiplicacin; el sistema de nmeros obtenidos se denomina sistema de cuaterniones. Tambin es posible una
construccin anloga en el espacio de ocho dimensiones, resultando el llamado sistema de nmeros de Cayley.
Desde luego, en este caso no hay que prescindir solamente de la conmutatividad del producto, sino tambin de
su asociatividad, sustituyendo esta ltima por otra operacin menos rigurosa.

1.5.1. Operaciones con los Nmeros Complejos. Representacin Geomtrica

El nmero complejo i lo llamaremos unidad imaginaria, y a los nmeros de la forma yi, nmeros imaginarios
puros, pudiendo sealarlos como los puntos del plano (que estn en el eje de ordenadas Oy) que los representan.
En la expresin del nmero complejo z en la forma z = x + yi, el nmero x se denomina parte real de z, y el
nmero y, parte imaginaria de z: Re z = x, Im z = y.

El plano R2 , cuyos puntos se han identicado con los nmeros complejos, se llamar plano complejo C R2 .
El eje de abscisas Ox de este plano se llama eje real, puesto que sus puntos representan a los nmeros reales;
respectivamente, el eje de ordenadas Oy del plano complejo se llamar eje imaginario.

La suma, resta, multiplicacin y divisin de los nmeros complejos expresados en la forma algebrica x + yi,
como se deduce de las frmulas (2), (3), (4) y (6), del prrafo anterior, se efectan del modo siguiente:

(x1 + y1 i) + (x2 + y2 i) = (x1 + x2 ) + (y1 + y2 ) i; (a)

(x1 + y1 i) (x2 + y2 i) = (x1 x2 ) + (y1 y2 ) i; (b)

(x1 + y1 i) (x2 + y2 i) = (x1 x2 y1 y2 ) + (x1 y2 + y1 x2 ) i; (c)


x1 + y1 i x1 x2 + y1 y2 y1 x2 y2 x1
= + i: (d)
x2 + y2 i x22 + y22 x22 + y22
Al sumar (restar) los nmeros complejos, se suman (restan) por separado sus partes reales y sus partes imag-
inarias. Al multiplicar nmeros complejos, se hallan los productos que dan por separado sus partes reales y
1.5. INTRODUCCIN A LOS NMEROS COMPLEJOS 45

sus partes imaginarias y se las simplica. Al dividir nmeros complejos, primero multiplicamos numerador y
denominador por la expresin conjugada del denominador y luego se aplica la anterior regla para el producto.

Estas reglas y frmulas se pueden deducir, sin memorizarlas, operando directamente. As, por ejemplo, para
la divisin tendremos que

x1 + y1 i (x1 + y1 i) (x2 y2 i) (x1 x2 + y1 y2 ) + (y1 x2 x1 y2 )i


= =
x2 + y2 i (x2 + y2 i) (x2 y2 i) x22 + y22
x1 x2 + y1 y2 y1 x2 x1 y2
= 2 2 + i;
x2 + y2 x22 + y22

que coincide con la frmula (d) anterior.

Ejemplos:

1. (2 + 5i) + (1 + 7i) = (2 + 1) + (5 + 7) i = 3 + 12i


3 4 3 4 3 3 4 4 21 32
2. i + i = + i+ i = i
5 3 2 5 5 2 3 5 10 15
3. (3 9i) (7 + 8i) = (3 7) + (9 8) i = 4 17i
3 4 3 4 3 3 4 4 3 32
4. + i i = + + i= + i
4 5 5 3 4 5 5 3 20 15
5. (1 + 2i) (3 i) = [1 (3) + 2( 1)i2 ] + [1( 1) + 2 (3)]i = 5 5i
1 3 3 9 7i 3 3 9 7i 21 2 27 5
6. (3 7i) + i = + i 7i i= + i i = i
2 4 2 4 2 4 2 4 2 4 4 4
2 2 i 2i 2i
7. = = = = 2i
i i i i2 ( 1)
2
1+i (1 + i)(1 + i) 1 + i + i + (i) 1+i+i 1
8. = = 2 = =i
1 i (1 i)(1 + i) 12 (i) 1 ( 1)
2
2+i (2 + i)(3 2i) 6 4i + 3i 2 (i) 10 i 10 i
9. = = 2 = =
3 + 2i (3 + 2i)(3 2i) 32 (2i) 9+4 13 13
2 2
2 3i 2 3i 2 3i (2 3i) 4 12i + (3i) 5 12
10. = = 2 = = i
2 + 3i 2 + 3i 2 3i 4 (3i) 4+9 13 13

Representacin geomtrica y forma trigonomtrica de un nmero complejo1

La representacin de los nmeros complejos por puntos del plano conducen al deseo natural de obtener una
interpretacin geomtrica de las operaciones ya denidas para los nmeros complejos. La representacin es fcil
si consideramos a cada nmero complejo como punto de R2 , o como vector bidimensional. Para la suma: dados
los nmeros (puntos, vectores) z1 = x1 + y1 i y z2 = x2 + y2 i, unamos con segmentos el origen de coordenadas
con los puntos (x1 ; y1 ) y (x2 ; y2 ) correspondientes a dichos nmeros, y sobre estos segmentos, como lados,
tracemos un paralelogramo (ver gura). Es evidente que el cuarto vrtice de este paralelogramo ser el punto
(x1 + x2 ; y1 + y2 ). Por lo tanto, la suma de nmeros complejos se efecta geomtricamente por la regla del
paralelogramo, o sea por la regla de la suma de vectores que parten del origen de coordenadas.

El nmero opuesto al nmero z1 = x1 + y1 i, es el punto del plano complejo que es simtrico al punto z1
con respecto del origen de coordenadas O (ver gura), es decir, el nmero z1 = x1 y1 i. De aqu se puede
obtener sin dicultad la interpretacin geomtrica de la resta.
1 Para la comprensin de estas subsecciones se recomienda la lectura de los inicios del captulo IX: Trigonometra I.
46 CAPTULO 1. PRELIMINARES ARITMTICOS

La interpretacin geomtrica de la multiplicacin y divisin de los nmeros complejos quedar clara solamente
despus de que introduzcamos una nueva expresin para los nmeros complejos, la interpretacin trigonomtrica.
Para la expresin del nmero complejo z en la forma z = x+yi, utilizamos las coordenadas cartesianas del punto
correspondiente a este nmero. Sin embargo, la posicin del punto en el plano queda tambin determinada, si
se conocen sus coordenadas polares: la distancia r del origen de coordenadas al punto y el ngulo que forma
la direccin positiva del eje de abscisas con la direccin que va desde el origen de coordenadas hacia este punto
(ver gura).

El nmero r es real y no negativo, siendo adems igual a cero solamente para el punto 0. Para un nmero z
situado en el eje real, a sea, para un nmero real, el nmero r es el valor absoluto de z; por esto, a veces, para
cualquier nmero complejo z, a r tambin se le llama valor absoluto o mdulo del nmero z, representndose
mediante jzj.

Figura 1.2: Coordenadas polares para la forma trigonomtrica de un nmero complejo z:

El ngulo se llamar argumento del nmero z y se designar con la notacin: arg z: El ngulo puede
tomar cualesquiera valores reales, tanto positivos como negativos, teniendo que medirse los ngulos positivos en
direccin contraria al del movimiento de las agujas del reloj; sin embargo, si los ngulos se diferencian entre s
en 2 , o en un nmero multiplo de 2 , sus puntos correspondientes del plano coinciden.

De este modo, el argumento de un nmero complejo z tiene innitos valores, que se diferencian entre s en
nmeros enteros multiplos de 2 , por consiguiente, de la igualdad de dos nmeros complejos, representados por
sus mdulos y sus argumentos, solamente se puede hacer la conclusin de que sus argumentos se diferencian
1.5. INTRODUCCIN A LOS NMEROS COMPLEJOS 47

en un nmero entero multiplo de 2 , mientras que sus mdulos son iguales. Solamente para el nmero 0 el
argumento es indenido, sin embargo, este nmero queda completamente determinado por la igualdad: j0j = 0:

El argumento del nmero complejo es una generalizacin natural del signo del nmero real. En efecto, el
argumento de un nmero real positivo es igual a cero, el argumento de un nmero real negativo es igual a ; en
el eje real, del origen de coordenadas parten solamente dos direcciones, las cuales se pueden distinguir por los
smbolos: + y . En el plano complejo hay innitas direcciones que parten del punto 0, diferencindose por el
ngulo que forman con la direccin positiva del eje real.

Las coordenadas cartesianas y polares de un punto estn ligadas por las relaciones (ver guras):

x = r cos ; y = r sin ; (1)


que se cumplen independientemente de la posicin del punto en el plano.

De aqu que
p
r=+ x2 + y 2 (2)
Apliquemos las frmulas (1) a un nmero complejo arbitrario z = x + yi = r cos + (r sin )i, o sea,

z = r(cos + i sin ): (3)

Recprocamente supongamos que el nmero z = x+yi, se expresa en la forma z = r0 (cos +i sin ), donde r0
p
y 0 son unos nmeros reales, siendo r0 0. Entonces, r0 cos = x, r0 sin = y, de donde r0 = + x2 + y 2 ,
y en virtud de (2), r0 = jzj. De aqu, aplicando (1), obtenemos: cos 0 = cos , sin 0 = sin , o sea, 0 = arg z.
Por lo tanto, todo nmero complejo z se expresa unvocamente en la forma (3), donde r = jzj, = arg z (por
supuesto, el argumento est denido salvo un sumando, multiplo de 2 ). Esta expresin del nmero z se
llama forma trigonomtrica.

Los nmeros
19 19 p h i
z1 = 5(cos + i sin ), z2 = cos i sin , z3 = 6 cos + i sin
4 4 3 3 7 7
estn dados en forma trigonomtrica, aqu,
p 19
jz1 j = 5; jz2 j = 1; jz3 j = 6; arg z1 = ; arg z2 = ; arg z3 = ;
4 3 7
13
(o bien, arg z2 = ; arg z3 = ).
3 7
48 CAPTULO 1. PRELIMINARES ARITMTICOS

Los 3 nmeros complejos


4 4 3
z1 = ( 2)(cos
+ i sin ), z2 = 3(cos i sin ), z3 = 2(cos + i sin )
5 5 3 3 3 4
no estn dados en forma trigonomtrica, a pesar de que estas expresiones se parezcan a la expresin del tipo
(3). Estos nmeros complejos se expresan en forma trigonomtrica del modo siguiente:
6 6 2 2 p p p
z10 = 2(cos + i sin ), z20 = 3(cos + i sin ), z30 = 3 cos arctan 2 + i sin arctan 2
5 5 3 3
0
La forma trigonomtrica del nmero z3 es algo engorrosa, como casi siempre ocurre al pasar de la expresin
ordinaria del nmero complejo a la forma trigonomtrica y viceversa. Como se sabe, salvo algunos casos, dados
los valores numricos del seno y del coseno, resulta posible hallar el valor exacto del ngulo, o dado el ngulo,
resulta imposible hallar los valores exactos de su seno y coseno.

1.5.2. Nmeros Complejos Conjugados. Operaciones

Sea el nmemro complejo z = x + yi. El nmero x yi, que se diferencia de z solamente en el signo de
la parte imaginaria, se llama nmero conjugado de z y se designa mediante z. Recordemos, que al estudiar
la divisin de los nmeros complejos recurramos implcitamente a los nmeros conjugados, a pesar de que no
habamos introducido este concepto.

Es evidente que el nmero conjugado de z es z, es decir, se puede hablar de pares de nmeros conjugados.
Los nmeros reales, y solamente stos, son conjugados consigo mismo.

Geomtricamente, los nmeros conjugados son puntos simtricos entre s con respecto al eje real (ver gura).

De aqu se deducen las igualdades


jzj = jzj ; arg z = arg z: (1)

La suma y el producto de nmeros complejos conjugados son nmeros reales. En efecto,


2
z + z = 2x = 2 Re z; zz = x2 + y 2 = jzj (2)

La ltima igualdad muestra que el nmero zz es, incluso, positivo para z 6= 0.

La igualdad (x1 y1 i) + (x2 y2 i) = (x1 + x2 ) (y1 + y2 )i muestra que el conjugado de la suma de dos
nmeros es igual a la suma de los nmeros conjugados:

z1 + z2 = z 1 + z 2 : (3)

Anlogamente de la igualdad (x1 y1 i) + (x2 y2 i) = (x1 x2 y1 y2 ) (x1 y2 + y1 x2 )i resulta que el conjugado


del producto es igual al producto de los nmeros conjugados:

z1 z2 = z1 z2 (4)

Una comprobacin directa muestra que se verican tambin las igualdades

z1 z2 = z 1 z2 ; (5)
z1 z1
= : (6)
z2 z2

Queda demostrada la siguiente proposicin: si el nmero z se expresa de cierto modo por los nmeros
complejos z1 , z2 ,..., zn mediante la suma, el producto, la resta y la divisin, entonces, al sustituir en esta
expresin todos los nmeros zk por sus conjugados, se obtiene el nmero conjugado de z; en particular, si el
nmero z es real, ste no se altera al sustitur todos los nmeros complejos zk por sus conjugados.
1.5. INTRODUCCIN A LOS NMEROS COMPLEJOS 49

Recuerda las caractersticas


de un nmero complejo:
Caractersticas: Expresiones: Ejemplos:
Nmero complejo z = x + iy o z = (x; y) ; x; y 2 R z = 1 + 4i o z = (1; 4)
Parte real Re z = x Re (1 + 4i) = 1
Parte imaginaria Im z = y Im (1 + 4i) = 4
Complejo conjugado z = x + iy ) z = x iy z = 1 4i o z = (1; 4)
Imaginario puro z = iy o z = (0; y) ; y 6= 0
z = 7i o z = (0; 7)
Real puro z = x o z = (x; 0) = x
z = 5 o z = (5; 0)
Unidad imagi- i = (0; 1), i2 = (0; 1) (0; 1) =
p i2 = 1, i3 = i, i4 = 1
naria = ( 1; 0) ) i = 1
( 3; 0) = 3 (1; 0) = 3
Unidad real 1 = (1; 0), (x; 0) = x (1; 0) = x
p
Forma trigonomtrica z = r (cos + i sin ) z = 1 + i = 2 (cos =4 + i sin =4)
p p p
Mdulo de z r = jzj = jx + iyj = + x2 + y 2 r = jzj = j1 + ij = + 12 +12 = 2
Argumento de z arg z = + 2 k; k 2 Z arg z = =4 + 2 k; k 2 Z

1.5.3. Potenciacin de Nmeros Complejos

Pasamos a analizar los problemas de la elevacin de los nmeros complejos a una potencia. Para elevar el
nmero z = x + yi a una potencia entera y positiva n, es suciente aplicar la frmula del binomio de Newton
a la expresin x + yi, es decir calcular (x + yi)n = (x + yi)(x + yi) : : : (x + yi) (esta frmula funciona tambin
| {z }
n veces
para los nmeros complejos, puesto que su demostracin se basa solamente en la ley distributiva). Para la
potenciacin de nmeros complejos hay que tomar en cuenta las igualdades: i2 = 1, i3 = i, i4 = 1, etc. En
general
i4k = 1, i4k+1 = i, i4k+2 = 1, i4k+3 = i; k 2 Z

Si el nmero z est dado en forma trigonomtrica, entonces, siendo n un entero positivo, de la frmula (4)
del prrafo anterior resulta la frmula siguiente, llamada frmula de De Moivre:
n
[r (cos + i sin )] = rn (cos n + i sin n ) ; (1)

o sea, que al elevar un nmero complejo a una potencia, se eleva el mdulo a esta potencia y se multiplica
el argumento por el exponente de la potencia. La frmula (1) es vlida tambin para los exponentes enteros
negativos. En efecto, en virtud de la igualdad z n = (z 1 )n , es suciente aplicar la frmula de De Moivre al
nmero z 1 , cuya frmula trigonomtrica viene dada por la frmula (10) del prrafo anterior.

Ejemplos:

1
1. i127 = i124 i3 = i, i37 = i36 i = i, i122 = i120 i2 = 1, i2157 = i2156 i = i, i 453 = i452 i 1
= i,
1 1 1 1
i 8751 = i8752 i = i, i 400 = i400 = 1, i 402 = i400 i2 = ( 1) = 1

2. (2 + 5i)3 = 23 + 3 22 5i + 3 2 52 i2 + 53 i3 = 8 + 60i 150 125i = 142 65i


4
3. (a 2bi) = a4 + 16b4 + 32iab3 8ia3 b 24a2 b2 = a4 + 16b4 24a2 b2 + 8 4ab3 a3 b i

(2 + 5i)4 (2 + 5i)4 (3 + 5i)3 (2 + 5i)4 (3 + 5i)3 282 + 166 730i 141 83 365
4. 3
= 3 3
= 3
= = + i
(3 5i) (3 5i) (3 + 5i) (9 + 25) 39 304 19 652 19 652
hp i4 p
5. 2(cos + i sin ) = ( 2)4 (cos + i sin ) = 4
4 4
p 6 p p 6
6. 8(cos 12 + i sin 12 ) = 512 14 14 i 2 + 14 + 41 i 6 = 512i
50 CAPTULO 1. PRELIMINARES ARITMTICOS
h i 5 1
7. 2(cos + i sin ) = p 5
6 6 3+i
h i 3 1 7 7
8. 3(cos + i sin ) = (cos + i sin ):
5 5 27 5 5
9. Demostrar las relaciones:
a) z1 + z2 = z1 + z2 ; b) z1 z2 = z1 z2 ; c) (z n ) = z n ; n 2 N:
a) Demostracin: z1 = (x1 ; y1 ) ; z2 = (x2 ; y2 ) :

z1 + z2 = (x1 + x2 ; y1 + y2 ) = (x1 + x2 ; y1 y2 ) =
= (x1 ; y1 ) + (x2 ; y2 ) = z1 + z2 :

b) Demostracin: z1 z2 = (x1 x2 y1 y2 ; x1 y2 + x2 y1 ) =
(x1 x2 y1 y2 ; x1 y2 x2 y1 ) = (x1 ; y1 ) (x2 ; y2 ) = z1 z2 :
c) Demostracin: Utilizando la frmula de De Moivre, tenemos:
z = (r cos ; r sin ) ; z = (r cos ( ) ; r sin ( ))
n
(z) = (rn cos ( n ) ; rn sin ( n )) = (rn cos n ; rn sin n ) =
= rn cos n ; rn sin n = (z n ) :

10. Realizar las siguientes operaciones:


p 6
2 4 3 i
a) (2 i) (2 + i) (3 2i) + 7; b) (1 + i) ; c) 2 + 2

Solucin: Antes de pasar a las operaciones con complejos expresados en la forma algebraica, tomemos en
cuenta que: i2 = 1, i3 = i2 i = i, i4 = i3 i = i2 = 1, etc.
2 2
a) (2 i) (2 + i) (3 2i) + 7 = (2 i) (2 + i) + 4 + 2i =
= (2 + i) ((2 i) (2 + i) + 2) = (2 + i) (4 + 1 + 2) = 14 + 7i
4
b) (1 + i) = 1 + 4i + 6i2 + 4i3 + i4 = 1 + 4i 6 4i + 1 = 4
p 6 p p p
3
c) 2 + i
2 = 27
64 + i 5464 3 135
64 i 6064 3 + 45
64 + i 6643 1
64 = 1

11. Hallar los cocientes:


p
1 3
1 1 2 +i
a) ; b) ; c) p2
i 1+i 1
i 23
2
Solucin:
z1 z1 z2 z1 z2
a) Utilizando la conocida frmula de multiplicar el denominador por la conjugada = = 2 ,
z2 z2 z2 jz2 j
1 1 ( i) i
tendremos = = 2 = i.
i i ( i) jij
1 (1 i) 1 i 1 i 1 i
b) = = 2 = =
1+i (1 + i) (1 i) j1 + ij 2 2 2
0 p 10 p 1
p 1 i 3 CB 1 i 3 C p
1 i 3 B 1 3
@ + A@ + A
+i p
+ 2 2 2 2 1 3
c) 2 p 2 = p 2 = 2 2 = +i
1 i 3 1 i 3 1 2 2
2 2 2 2

12. Representar los siguientes nmeros complejos en su forma trigonomtrica:


p
a) 3; b) i; c) 1 + i; d) 1+i 3
a) Solucin: j 3j = 3, = , 3 = 3 (cos + i sin ) :
1.5. INTRODUCCIN A LOS NMEROS COMPLEJOS 51

b) Solucin: j ij = 1, = , i = cos + i sin


2 2 2
p p
c) Solucin: j1 + ij = 2; = 4; 1+i= 2 cos + i sin
4 4
p 2 p 2 2
d) Solucin: 1 + i 3 = 2, = , 1 + i 3 = 2 cos + i sin
3 3 3
13. Calcular las expresiones:
12
p 30 p p 20 1 i
a) 1+i 3 ; b) 2 i 2 ; c) ;
1+i
11
1+i 41 p 7
d) p ; e) (2 + 2i) ; f) 3 i
3 13
a) Solucin: r
p p 2 p
1+i 3 = 12 + 3 = 2; arg 1 + i 3 =
3
p 30 30 30
1+i 3 = 230 cos + i sin = 230
3 3
b) Solucin:
p p
2 i 2 = 2 cos + i sin ;
4 4
p p 20 20 20
2 i 2 = 220 cos + i sin = 220
4 4
c) Solucin: p
1 i 2 cos 4 + i sin 4
= p = cos + i sin ;
1+i 2 cos 4 + i sin 4
2 2

12 12
1 i
= cos + i sin =
1+i 2 2
12 12
= cos + i sin =1
2 2

d) Solucin: p
1+i 2 cos 4 + i sin 4 1 7 7
p = p =p cos + i sin ;
3 13 2 3 cos( 3 ) + i sin( 3) 6 12 12
11
1+i 77 177
p + i sin
= p = cos
3 13 6 12 65 12
p p !
1 5 5 1 3 1 3+1
= p cos + i sin = 5 p +i p :
65 6 12 12 6 4 3 3

e) Solucin:
p 41 41
(2 + 2i)41 = 8 cos + i sin =
4 4
p 41 41 41
= 8 cos + i sin = 820 (2 + 2i)
4 4

f ) Solucin:
p 7 7
5 5
3 i = + i sin 27 cos=
6 6
35 35
= 27 cos + i sin =
6 6
p !
3 1 p
= 27 cos + i sin =27
+i = 26 3+i
6 6 2 2
52 CAPTULO 1. PRELIMINARES ARITMTICOS

Practico lo que aprend:

Simplicar las expresiones y presentarlas como un nico nmero complejo:

1. a) 3 (2 i) (2 + i) 4 (3 2i) (3 + 2i), b) 5 (2 3i) (2 + 3i) 3 (2 5i) (2 + 5i)

2 3 3 2 3 2 3 2
2. a) (4 i) (4 + i) (2 2i) (2 + 2i) , b) (2 3i) (2 + 3i) (2 5i) (2 + 5i)

2 3 2 3 4 3 4 2
3. a) i3 (1 3i) (1 + 3i) i3 (1 5i) (1 + 5i) , b) (1 i) (1 + i) (2 i) (2 + i)

4 2 4 2 4 4 3 3
4. a) (2 i) (2 + i) (3 i) (3 + 5i) , b) i3 (1 3i) (1 + 3i) i3 (1 5i) (1 + 5i)
2 2 2 2
2 3i 4 3i 1 2i 5 i 1 2i 3 i 3 2i 1 i
5. a) , , b) , c) :
2 + 3i 4 + 3i 1 + 2i 5+i 1 + 2i 3+i 3 + 2i 1+i
2 2 2 2
2 5i 4 7i 1 i 6 i 5 2i 4 i 9 2i 7 i
6. a) , b) , c) , d) :
2 + 5i 4 + 7i 1+i 6+i 5 + 2i 4+i 9 + 2i 7+i

7. a) i2 2i3 + 3i4 4i5 + 5i6 , b) i2 + 2i3 + 3i4 + 4i5 + 5i6 , c) i2 3i3 + 5i4 7i5 + 9i6

8. a) i2k 2i3n + 3i4m 4i5l + 5i6j , b) i2k 3i3l + 5i4m 7i5n + 9i6j ; si j = 2, k = 3, l = 5, m = 7, n = 4.

Representar los siguientes nmeros complejos en forma trigonomtrica y gracarlos:


2 2 2 2
2 3i 4 3i 1 2i 5 i 1 2i 3 i 3 2i 1 i
9. a) , b) , c) , d) , e) , f) , g) , h) ,
2 + 3i 4 + 3i 1 + 2i 5+i 1 + 2i 3+i 3 + 2i 1+i
2 2 2 2
1 3i 2 3i 3 2i 1 i 2 2i 2 i 3 2i 1 i
i) , j) , k) , l) , m) , n) , o) , p) :
2 + 3i 4 + 3i 1 + 2i 5+i 1 + 2i 3+i 1 + 2i 2+i
Representar en forma trigonomtrica los siguientes nmeros:

10. a) z = cos 30o + i sin 30o , b) z = 1 + cos 40o + i sin 40o , c) z = cos + i sin , d) z = sin i cos ,
e) z = tan i, 0 < , 6 = =2:

11. Sea jzj = 5, Dnde se encuentran los puntos que representan los nmeros complejos:

a) 4z, b) 2 z, c) 1 + 3z, d) z=2, c) 1 2z, d) 3 + z=2, e) 2 3z?

12. Demostrar las relaciones:


z1 z1
a) z1 z2 = z1 z2 , b) = , c) P (z) = P (z), donde P (z) = a0 + a1 z + a2 z 2 + + an z n , y a0 ,
z2 z2
a1 , a2 , , an son nmeros (coecientes) reales.

13. Realizar las siguientes operaciones:


2
p 6 2 + 4i (3 2i) i x + iy
a) 1 + i 3 , b) , c) 2 , d) , x2 + y 2 6= 0
3 + 5i (4 + i) + i x iy

14. Hallar la parte real e imaginaria de los siguientes nmeros complejos:


p !3 2 5 3 2
1 i 3 i5 + 2 (1 + i) (1 i) + (1 + i)
a) , b) , c) 3 , d) 2 3
2 2 i15 + 1 (1 i) (1 + i) + (1 i)

15. Hallar el mdulo y argumento de los siguientes nmeros complejos:


3 8 p 6
a) ( 4 + 6i) , b) (1 + i) 1 i 3 , c) 1 + cos + i sin , d) 1 + cos i sin
7 7
1.5. INTRODUCCIN A LOS NMEROS COMPLEJOS 53

Figura 1.3: Clasicacin de los conjuntos numricos.

Recuerda estas expresiones para re-


Conjuntos numricos: Ejemplos:
presentar a los conjuntos numricos:
Conjunto de n- 1, 12, 100, 79, 101,
N = f1; 2; 3; : : : ; n; : : :g
meros naturales: 1745, 8001123
Conjunto ampliado 0, 1, 23, 100, 110,
N0 = f0; 1; 2; 3; : : : ; n; : : :g
de los naturales: 1745, 58903, 19
Conjunto de los 1200, 342, 7,
Z = f: : : ; n; : : : ; 2; 1; 0; 1; 2; : : : ; n; : : :g
nmeros enteros: 0, 7, 342, 19
2 8
Conjunto de n- p , 3 , 4;072, 7,
Q= x:x= ; p; q 2 Z; q 6= 0 3 7
meros racionales: q 0, 12;025, 4;312, 18,
p3
p p
Conjunto de nme- p 15, 8 2=17, 3
Q0 = x : x 6= ; p; q 2 Z; q 6= 0 3 p
ros irracionales: q , p , e3 , 2
2 7 5
97;3, 0;015, 0,
Conjunto de n- p
R = Q [ Q0 2
4
5 9
meros reales: , p , 0:7
6 7
Conjunto de n- p 5:12 8i, 3 2i, i
C = z : z = x + iy; x; y 2 R; i = 1 p5 i
p
meros complejos: 5=3, 3 + 2:3 , 1 7i
54 CAPTULO 1. PRELIMINARES ARITMTICOS

Figura 1.4: Algunos de los matemticos ms famosos colocados en el eje del tiempo.

Figura 1.5: Izq. El conocido matemtico y gemetra lie Cartan (Dolomieu, 1869 - Pars, 1951). Trabajos sobre
variedades diferenciales, topologa algebraica, grupos de Lie y geometra. Creador al mismo tiempo que Henri Poincar
(Nancy, 1854 - Pars, 1912) del clculo diferencial exterior. Autor de la teora de espinores. Der. Uno de los ms famosos
matemticos franceses contemporneos, miembro - fundador del grupo Bourbaki, Henri Cartan (Nancy, 1904
-), hijo de lie Cartan (1869-1951). Sus trabajos estn relacionados con la teora de funciones de variable real
y de varias variables complejas en espacios analticos, algebra clsica y de Lie y topologa algebraica. Junto
con el bourbakista polaco Samuel Eilenberg (Varsovia, 1913 - ) hizo importantsimas contribuciones al lgebra
homolgica.
Captulo 2

EXPRESIONES RACIONALES E
IRRACIONALES. POLINOMIOS

De buena gana ardera hasta morir como Faetn, si ste fuera el precio por

alcanzar el sol y conocer su forma, su tamao y la sustancia de que est hecho.

Eudoxo

Ptolomeo le pregunt una vez a Euclides si haba algn camino ms corto

para el conocimiento de la geometra que por el estudio de los Elementos, a

lo que Euclides respondi que no haba ningn camino real a la geometra.

Proclo Didoco

Unidad de competencia: Transformar y simplicar expresiones racionales, as como, interpretar, analizar e


integrar los conceptos bsicos sobre los polinomios en una variable, sus propiedades, operaciones, races y conversin a
productos. .

Indicadores de logro:

| Reconocer y aplicar las distintas reglas de los productos y cocientes notables para simplicar y determinar el valor
de una expresin algebraica.

| Identicar las principales caractersticas y propiedades de los polinomios para aplicarlas en el factoreo, simplicacin
y bsqueda de races.

| Manejar el producto y divisin sintticas.

| Conocer y utilizar el teorema del resto y sus aplicaciones.

Suponiendo que una partida normal de ajedrez, como trmino medio tiene 40 jugadas, entonces el nmero posible de
5 35
partidas llegara a (20 20) (30 30) 2 10116

Es verdad que el producto de 2 nmeros que terminan en 76, tambin termina en 76 ?

55
56 CAPTULO 2. EXPRESIONES RACIONALES E IRRACIONALES. POLINOMIOS

2.1. Expresiones Algebraicas

Pasamos a estudiar las distintas expresiones algebraicas (monomios, polinomios, etc.) y sus operaciones.

2.1.1. Tipos de expresiones algebraicas

Con nmeros y variables (letras), as como con la ayuda de los smbolos de suma, diferencia, produc-
to, divisin (+; ; o ; o =), elevacin a potencia racional, radicacin y el uso adecuado de parntesis
(fllavesg ; [corchetes] ; (parntesis)) se forman las denominadas expresiones algebraicas. As, por ejemplo,

z 2a2 5a + 3 a2 xy 3
1) 5a2 b 5ab2 (b a) , 2) 3x 2y + , 3) , 4) + x3
4 2a 1 y z
4 p p 5 p
1 2 3d
5) , 6) x4 3
2x y, 7) 3x c , 8) 3c2=5 d5=3 ,
a b x

son expresiones algebraicas enteras, racionales e irracionales.

Si una expresin algebraica no posee divisiones entre variables, ni radicacin aplicada a las variables, ni
potencias fraccionarias con las variables, entonces tenemos una expresin algebraica entera (ejemplos 1), 2) y
5)). Si la expresin algebraica aparte de las 4 operaciones aritmticas entre nmeros y variables posee potencias
naturales y divisin entre variables (ejemplos 3), 4), y 5)), entonces se denomina expresin algebraica frac-
cionaria. Las expresiones algebraicas enteras y las fraccionarias forman las expresiones algebraicas racionales
(ejemplos 1), 2), 3), 4), 5) y 7)). Si por el contrario, se utilizan la radicacin y la potenciacin fraccionaria con
las variables, entonces tendremos expresiones algebraicas irracionales (ejemplos 6) y 8)).

2.1.2. Conjunto de valores admisibles c:v:a: de una expresin algebraica

Los valores de las variables para los que las expresiones algebraicas poseen sentido, se denominan valores
admisibles. El conjunto de estos los valores de las variables, se llama conjunto de valores admisibles o c:v:a:
p x2
de la expresin algebraica. As, por ejemplo, la expresin algebraica entera a3 5b + , y en general todas
7
las enteras, tiene sentido para cualesquiera valores que tomen las variables a, b y x, siendo el c:v:a: para las
2
a 3c b
3 variables todos los nmeros reales. La expresin algebraica fraccionaria p , y en
b 1 d a 2a
general todas las fraccionarias, tiene sentido para aquellos valores de las variables a, b, c y d que no anulan los
denominadores, siendo el c:v:a: para las 4 variables todos los nmeros reales con las condiciones a 6= 0, b 6= 1,
p b2=5
d 6= a. La expresin algebraica irracional 2b a , y en general todas las irracionales, no tienen sentido
c
para aquellos valores de las variables a, b, y c que convierten las expresiones subradicales de ndice par o bajo
exponente fraccionario en valores negativos, siendo el c:v:a: para las 3 variables todos los nmeros reales que
cumplen con las condiciones 2b a 0, b 0, c 6= 0.

| Si a las variables de una expresin algebraica les asignamos valores admisibles, entonces se obtiene una
expresin numrica cuyo valor se denomina valor de la expresin algebraica. As, por ejemplo, si R (a; b; c) =
p b2=5
2b a y le asignamos los valores admisibles a = 2, b = 3, c = 1, y a = 1, b = 1, c = 1, entonces
c
p 32=5 p 12=5
R (2; 3; 1) = 2 3 2 = 2 + 32=5 ; y R (1; 1; 1) = 2 1 1 = 1 1 = 0, que son los valores
1 1
buscados respectivos de la expresin dada.

Practico lo que aprend:

Clasicar las siguientes expresiones algebraicas:


2.1. EXPRESIONES ALGEBRAICAS 57
p
z3 3b2 b + 5 a2 y 3 z 3 y z
1. a) 2a2 b3 3ab2 b a3 , b) 5x2 2y + , c) , d) + 2y 3 , e) 2 4y 5=3
3 2b 1 y z y z

4
1 2 2a p p 3 p 3x y + 2
2. a) , b) 2a2 + b 2c, c) 2y 3z , d) 3a3=5 2b2=3 , e) :
x z y 2x 1
Determinar los conjuntos de valores admisibles c:v:a: de las siguientes expresiones:

p 2
2ab2 b z3 b2 5b + 1 a2 x 2 2z y 1 2 3a
3. a) c3 , b) 2x2 y+ , c) 2
, d) + 2x3 , e) 4, f ) ,
x c b 4 a x y+z a b c
3
p p 3a p x 2y + 1 3a + 5 1 2 1
g) 2a4 + 2b c, h) 2y , i) 3a3=5 2b1=4 , j) , k) , l) p p
3
+ :
b 3x + 2 x (2x 1) a b c
Hallar los valores de las siguientes expresiones para los valores dados:

z3
4. a) R (a; b; c) = 2ab2 b c3 , si a = 1, b = 2, c = 1; b) R (x; y; z) = 2x2 y +
, si x = 2, y = 2, z = 1;
3 p
2 2
b 5b + 1 a x 2 y 2z
c) R (b) = , si b = 2; d) R (a; x) = + 2x3 , si a = 2, x = 1; e) R (y; z) = 4, si
b 4 a x y+z
2
1 2 3a p
y = 2, z = 3; f ) R (a; b; c) = , si a = 1, b = 1, c = 3; g) G (a; b; c) = 2a4 + 2b c,
a b c
p 3 1 p p 2=3
si a = 1, b = 3, c = 2; h) G (a; y) = 2y 3a , si a = , y = 2; i) G (a; b) = 3a3=5 2b , si
3
x 2y + 1 1 3a + 5
a = 32, b = 8; j) G (x; y) = , si x = , y = 1; k) G (a; x) = , si a = 1, x = 1;
3x + 2 3 x (2x 1)
1 2 1
l) G (a; b; c) = p p
3
+ , si a = 1, b = 8, c = 1:
a b c

Observaciones:

| La sustitucin de una expresin analtica por otra idnticamente igual a ella en cierto conjunto, lle-
va el nombre de transformacin idntica de la expresin dada en ese conjunto. As, por ejemplo, x2 4 es
(x 2) (x + 2) x2 + 1
identicamente igual a (x 2) (x + 2) o a en todo R, pero es identicamente igual a
(x2 + 1)
p
(x 2) (x + 2) (x 5) (x 2) (x + 2) x + 3
slo en Rn f5g o a p slo en ] 3; +1[:
x 5 x+3
Al realizar transformaciones idnticas de una expresin es posible la variacin de su campo de denicin o
conjunto de valores admisibles c:v:a: Por ejemplo, reduciendo los trminos semejantes al simplicar la expresin
p p
x2 3x 5 + x x, ampliamos su campo de denicin: la expresin dada slo est denida para x > 0,
mientras que el polinomio obtenido despus de la simplicacin x2 + 3x 5 est denido para cualesquiera
valores de x 2 R. Las 2 expresiones anteriores son idnticamente iguales slo en el conjunto [0; +1[.

| El campo de denicin de la expresin puede as mismo variar despus de la simplicacin de una fraccin.
x3 1
As, la fraccin algebraica est denida para x 6= 1 y x 6= 2. Despus de simplicar por (x 1),
(x 1) (x + 2)
x2 + x + 1
obtenemos la fraccin denida para x 6= 2. Las 2 expresiones anteriores son idnticamente iguales
x+2
en el comjunto Rn f 2; 1g.

| La variacin o cambio del c:v:a: de una expresin algebraica es tambin posible como resultado de otras
transformaciones (elevacin a potencia, radicacin, etc.), por lo que, despus de efectuar la transformacin de la
expresin dada, siempre hay que saber responder a la pregunta en qu conjunto esta es idntica a la obtenida.

| Una expresin algebraica lleva el nombre de racional si ella puede ser obtenida slo mediante operaciones
de sumar, multiplicar, restar, dividir y elevacin a una potencia entera.
58 CAPTULO 2. EXPRESIONES RACIONALES E IRRACIONALES. POLINOMIOS

Figura 2.1: Izq. El clebre escritor, lsofo, astrnomo, fsico y matemtico francs Jean le Rond DAlembert (Pars,
1717 - 1783), una de las guras principales de la Ilustracin. Desarroll algunas teoras en el clculo clsico, dinmica
(principio de D Alembert), ecuaciones diferenciales en derivadas parciales y Astronoma. Colabor con Diderot en la
redaccin de la Enciclopedia. Der. El famoso matemtico alemn Richard Dedekind (Brunswick, 1831 - 1916). Su
aportacin fundamental es el concepto de nmero real (cortaduras de Dedekind y nmeros irracionales), del continuum
y del conjunto innito. Gran especialista en teora de nmeros. Alumno de Carl Gauss (1777 - 1855) y Lejeune Dirichlet
(1805 - 1859).

2.2. Transformacin y Simplicacin de Expresiones Racionales

Pasamos a describir brevemente los conceptos fundamentales sobre las expresiones racionales y polinomios,
sus productos y cocientes notables y sus respectivas operaciones de simplicacin y factoreo.

2.2.1. Monomios y polinomios

Las expresiones algebraicas racionales enteras trabajan con monomios y polinomios de una o varias variables.
Se denomina monomio a tal expresin que contiene slo nmeros o coecientes y potencias naturales de los
p
productos de las variables. As, por ejemplo, 3b 7;4b5 , 3a3 bc2 , 13ac2 1;414ab2 c3 , 3;1416b5 x2 y 3 , 35 x2 z ( y)
p xy
son monomios, y a 2b, 3 ab3 , no son monomios racionales enteros (el primero por ser una diferencia, el
2az
segundo por ser irracional y el tercero por ser un cociente entre variables).

| La suma de todas las potencias de las variables nos da el grado del monomio y los monomios son semejantes
p
cuando coinciden los grados de las respectivas variables, as, por ejemplo, 3a3 bc2 y 3;7a3 bc2 son semejantes,
12x2 y y 4=5xy 2 no son semejantes.

| Con los monomios semejantes son posibles las operaciones de suma y diferencia, el producto es posible
con cualesquiera monomios, la divisin es posible siempre y cuando el cociente sea otro monomio racional y
entero, y la potenciacin con exponente natural. As, por ejemplo:

p p 11
1. 12a3 x2 y + 2a3 x2 y = 2 12 a3 x2 y, 2=3bx2 y 13 1=4bx2 y 13 = 2 13
12 bx y

2. 3=5a2 by 2 3a2 by 2 = 18=5a2 by 2 , ab3 x2 yz 3 2=3ab3 x2 yz 3 = 31 ab3 x2 yz 3

3. 5ab3 x2 3=5a2 bcxy 2 = 3a3 b4 cx3 y 2 , 3=2uv 2 8=9au2 v 3 = 43 au3 v 5

3=5a2 bcxy 2 3=2b3 c2 x4 z 3 27 2 2


4. = 3bxy, = 8 b xz
1=5a2 cy 4=9bc2 x3 z
3 4
5. 3=2b2 xy 3 = 27=8b6 x3 y 9 , 0;2a3 bx2 y 2 z 3 = 0;001 6a12 b4 x8 y 8 z 12

Se denomina polinomio a una suma de monomios ya sean de varias variables o de una sola variable o
p
indeterminada. Ejemplos de polinomios son: 2a b + c, 7x + 2, 2=3x2 y, 5 7mn4 , 3x4 + x2 + 1, 3a2 y
2.2. TRANSFORMACIN Y SIMPLIFICACIN DE EXPRESIONES RACIONALES 59
p
7bxy + 5b2 y 3 , 3x2 ax + b=2, etc. La suma de monomios puede estar escrita en forma estndar, es decir,
coeciente por variables y en orden alfabtico. Si un polinomio est escrito en la forma estandar, entonces
este puede ser ordenado en forma creciente o decreciente de acuerdo a cierta variable. As, por ejemplo, el
p
polinomio 3a2 xy 3 +1/4ax2 y+ 2xy 2 est ordenado de acuerdo a la variable a y cada monomio est escrito en
forma estandar, sin embargo, por agrupacin, podramos ordenarlo de acuerdo a la variable x, lo que nos da
p p
1=4ax2 y + 2y 2 3a2 y 3 x; u ordenarlo de acuerdo a la variable y, para obtener 3a2 xy 3 + 2xy 2 + 1=4ax2 y.

| Con los polinomios son posibles las operaciones de suma, diferencia, producto, divisin (con cociente
racional y entero) y potenciacin con exponente natural, pero siempre es necesario ordenar y reducir trminos
semejantes. As, por ejemplo:

1. Suma: 3a2 xy 3 + 14 ax2 y + 3a2 xy 3 + x2 y = 6a2 xy 3 + 14 ax2 y + x2 y, que est ordenado de acuerdo a
la variable a.

2. Diferencia: 3a2 xy 3 + 14 ax2 y 3a2 xy 3 + x2 y = 1 2


4 ax y x2 y = 1
4a 1 x2 y, que se lo considera
ordenado de acuerdo a a, x o y.

3. Producto: 3a2 xy 3 + 41 ax2 y 3a2 xy 3 + x2 y =


2
3a2 xy 3 3a2 xy 3 x2 y + 3a2 xy 3 14 ax2 y + 41 ax2 y x2 y = 9a4 x2 y 6 3 3 3 4
4a x y 3a2 x3 y 4 + 41 ax4 y 2 ,
que se lo considera ordenado de acuerdo a la variable a.

4. Divisin: Si x2 2y (ax + y) = ax3 + x2 y 2axy 2y 2 , ordenada de acuerdo a x, la dividimos por


ax3 + x2 y 2axy 2y 2
ax + y: ax3 + x2 y 2axy 2y 2 (ax + y) = , aplicando la regla de la escalera,
ax + y
tendremos:
ax3 +x2 y 2axy 2y 2 ax +y
ax3 x2 y x2 2y
0 0 2axy 2y 2 ,
+2axy +2y 2
0 0
ax3 + x2 y 2axy 2y 2
de donde = x2 2y que nos dice que el numerador es mltiplo del denominador
ax + y
o que ste es divisor del numerador, pues es una divisin sin resto.
2
5. Potenciacin: 3a2 xy 3 + x2 y = 3a2 xy 3 + x2 y 3a2 xy 3 + x2 y =
2
3a2 xy 3 3a2 xy 3 + 3a2 xy 3 x2 y + x2 y 3a2 xy 3 + x2 y =
x4 y 2 6a2 x3 y 4 + 9a4 x2 y 6 , que est ordenado de acuerdo a x.

Practico lo que aprend:

1. Dados los polinomios P = x2 3xy + y 2 , R = 2x + y 2 , T = 2x3 + y 4 + 2xy 2 6x2 y 3xy 3 + x2 y 2 ,


Q = 2x2 xy + y 2 , S = x2 2y 2 , U = 2x4 2y 4 + 2xy 3 x3 y 3x2 y 2 . Hallar los resultados de las
operaciones: a) P + Q, b) 2P 3Q, c) P R, d) T R, e) P Q, f ) RS, g) U S, h) R2 , i) T U ,
j) 2U + 3T , k) P 2 3Q2 , l) T P , m) U Q, n) P S QR, o) P RS QRS, p) 3P QS 2RS 2

2. Dados los polinomios P = 2x2 xy + y 2 , R = x + 3y 2 , T = 2x3 + 3y 4 + xy 2 x2 y 3xy 3 + 6x2 y 2 ,


Q = 2x2 3xy + y 2 , S = 3x2 2y 2 , U = 6x4 2y 4 + 6xy 3 9x3 y x2 y 2 . Hallar los resultados de las
operaciones: a) P + Q, b) 2P 3Q, c) P R, d) T R, e) P Q, f ) RS, g) U S, h) R2 , i) T U ,
j) 2U 3T , k) P 2 2Q2 , l) T P , m) U Q, l) T P , m) U Q, n) P S QR, o) RS QRS.
60 CAPTULO 2. EXPRESIONES RACIONALES E IRRACIONALES. POLINOMIOS

2.2.2. Productos y cocientes notables, factoreo

Pasamos a analizar las principales identidades entre expresiones algebraicas (monomios, polinomios, frac-
ciones en varias variables, etc.) que vienen representadas en forma de productos o de cocientes.

Una identidad (igualdad que se cumple para cualesquiera valores de sus variables o indeterminadas pertenecientes
a cierto conjunto de valores admisibles o c:v:a:) expresada en forma de producto, se denomina producto no-
table. Si la misma identidad est escrita en forma de divisin (o cociente, posible en cierto conjunto de valores
admisibles o c:v:a:) o razn, entonces se denomina cociente notable. As, por ejemplo:

a2 b2 a2 b2
a2 b2 = (a b) (a + b); = a + b; =a b
| {z } a b a+b
pro ducto notable
| {z } | {z }
co ciente notable co ciente notable

| El conjunto de valores admisibles o c:v:a: es aquel conjunto, generalmente numrico, en el cual cierta
2 2
expresin algebraica tiene sentido. As, por ejemplo, la expresin aa bb no tiene sentido si a = b, pues aparece
p
la divisin por 0 y la indeterminacin del tipo 00 . Anlogamente, si trabajamos con reales, la expresin 2a b
carece de sentido a menos que se considere que 2a b 0, es decir, 2a b sea positiva o nula, o lo que
es lo mismo que b 2a, caso contrario tendremos la raz cuadrada de una expresin subradical negativa
(anlogamente se razona para cualquier raz con ndice par, pues las expresiones con radicales de ndice impar
poseen un c:v:a: =] 1; +1[ ). As mismo, la expresin a3 2b tiene sentido, siempre y cuando a 6= 0 y b 6= 0,
o lo que es lo mismo ab 6= 0.

| De lo anterior se desprende que es suciente estudiar y aprenderse las principales identidades y reglas
correspondientes slo a los productos notables, siendo los cocientes notables sus frmulas derivadas factibles en
su c.v.a.

| Cuando no se respeta el conjunto de valores admisibles o c:v:a: de una expresin algebraica, pueden
aparecer paradojas, o demostraciones interesantes, as, por ejemplo:

Si a = b = 1 ) a = b ) ab = b2 ) ab a2 = b2 a2 ) a (b a) = (b a) (b + a)
) a = b + a ) 1 = 1 + 1 ) 1 = 2:

La falacia anterior aparece en el antepenltimo paso, es decir, cuando simplicamos ambos miembros de la
igualdad por b a, que equivale a dividir por cero, pues b a = 1 1 = 0:

Recuerda los principales productos notables aplicados al factoreo de expresiones algebraicas:

1. Diferencia de cuadrados: La expresin x2 a2 = (x a) (x + a), se denomina diferencia de cuadrados.


Su generalizacin corresponde a x2n a2n = (xn an ) (xn + an ), n 2 N.
As, por ejemplo:

a) a2 16 = (a 4) (a + 4)
4 2
b) 16x 1 = 4x 1 4x2 + 1 = (2x 1) (2x + 1) 4x2 + 1
c) 9x6 4y 4 = 3x3 2y 2 3x3 + 2y 2
4
d ) 81 16y 4 z 8 = 34 2yz 2 = 32 22 y 2 z 4 32 + 22 y 2 z 4 = 3 2yz 2 3 + 2yz 2 9 + 4y 2 z 4
4 4
e) 16x16 y 4 81z 8 = 2x4 y 3z 2 = 2x4 y 3z 2 2x4 y + 3z 2 +4x8 y 2 + 9z 4 :

2. Suma o diferencia de cubos: La expresin x3 a3 = (x a) x2 ax + a , se denomina suma o


diferencia de cubos. Su generalizacin viene a ser x3n a3n = (xn an ) x2n an xn + a2n , n 2 N.
As, por ejemplo:
2.2. TRANSFORMACIN Y SIMPLIFICACIN DE EXPRESIONES RACIONALES 61

a) a3 8 = (a 2) a2 + 2a + 4
b) a3 + 8 = (a + 2) a2 2a + 4
6 2
c) 27x + 1 = 3x + 1 9x4 3x2 + 1
d ) 8x6 27y 9 = 2x2 3y 3 4x4 + 6x2 y 3 + 9y 6
3 3
e) 64x6 y 6 z 12 = 4x2 y2 z4 = 4x2 y2 z4 16x4 + 4x2 y 2 z 4 + y 4 z 8 =
2x + yz 2 2x yz 2 4x2 2xyz 2 + y 2 z 4 4x2 + 2xyz 2 + y 2 z 4

3. Las frmulas 1. y 2. anteriores son casos particulares de la suma o diferencia de potencias:

a) xn an = (x a) xn 1
+ axn 2
+ a2 xn 3
+ + an 2
x + an 1
, cualquier n
b) xn an = (x + a) xn 1
axn 2
+ a2 xn 3
+ an 2
x an 1
, n es par
c) xn + an = (x + a) xn 1
axn 2
+ a2 xn 3
an 2
x + an 1
, n es impar

As, por ejemplo:

a) xn 1 = (x 1) xn 1
+ xn 2
+ xn 3
+ +x+1
b) a4 16 = a4 24 = (a 2) a3 + 2a2 + 4a + 2 = (a + 2) a3 2a2 + 4a 2
c) x4 a4 = (x + a) x3 ax2 + a2 x a3 = (x a) x3 + ax2 + a2 x + a3 = (x + a) (x a) x2 + a2
5
d ) 32x5 + 1 = (2x) + 1 = (2x + 1) 24 x4 23 x3 + 22 x2 2x + 1
e) x7 + 1 = (x + 1) x6 x5 + x4 x3 + x2 x+1
4 8 12 2 4 3 4 2
f ) 16x y 81z = 2xy 3z = 2xy 3z 3 8x3 y 6 + 12x2 y 4 z 3 + 18xy 2 z 6 + 27z 9 + =
3z 3 + 2xy 2 8x3 y 6 12x2 y 4 z 3 + 18xy 2 z 6 27z 9 + = 3z 3 + 2xy 2 2xy 2 3z 3 9z 6 + 4x2 y 4

4. Descomposicin del trinomio Ax2 + Bx + C o Ax2n + Bxn + C: Partiendo de que x2 + (a + b) x +


ab = (x + a) (x + b), y en general de que x2n + (a + b) xn + ab = (xn + a) (xn + b), se concluye que se
necesitan encontrar dos nmeros que multiplicados nos den ab y sumados a + b. As mismo, se tiene que
pqx2 + (pb + aq) x + ab = (px + a) (qx + b).
As, por ejemplo:

a) x2 + 9x + 20 = (x + 5) (x + 4), pues 4 + 5 = 9 y 4 5 = 20
b) x2 5x 14 = (x 7) (x + 2), pues 2 7= 5 y 2 ( 7) = 14
2
c) x 11x + 30 = (x 6) (x 5), pues 6 5= 11 y ( 6) ( 5) = 30
(6x 9) (6x 10)
d ) As mismo, 6x2 19x + 15 = = (2x 3) (3x 5), pues 9 10 = 19 y ( 9)
3 2
( 10) = 6 15
(3x + 15) (3x 1)
e) 3x2 + 14x 5= = (x + 5) (3x 1), pues 15 1 = 14 y 15 ( 1) = 15
3
(2x + 6) (2x + 1)
f ) 2x2 + 7x + 3 = = (x + 3) (2x + 1), pues 6 + 1 = 7 y 6 1 = 2 3
2
g) Anlogamente tendremos: x4 + 3x2 + 2 = x2 + 2 x2 + 1 , pues 2 + 1 = 3 y 2 1 = 2
h) x4 5x2 + 4 = x2 4 x2 1 = (x 2) (x + 2) (x 1) (x + 1)
i ) x6 + x3 2 = x3 + 2 x3 1 = x3 + 2 (x 1) x2 + x + 1
j ) x6 + 7x3 8 = x3 + 8 x3 1 = (x + 2) x2 2x + 4 x + x2 + 1 (x 1)
6x4
3 6x4 + 2
k ) 6x8 x4 1= = 2x4 1 3x4 + 1 , pues 3+2= 1y 3 2 = 6 ( 1)
3 2
2x4 2 2x4 + 1
l ) 2x8 x4 1= = x4 1 2x4 + 1 = (x 1) (x + 1) x2 + 1 2x4 + 1 , pues
2
2+1= 1 y 2 1 = 2 ( 1).
62 CAPTULO 2. EXPRESIONES RACIONALES E IRRACIONALES. POLINOMIOS

5. Agrupacin de trminos o factor comn: Una expresin algebraica puede contener tanto coecientes
como trminos comunes, los mismos que pueden presentarse con diferentes potencias. Una expresin de
este tipo puede descomponerse en factores tomando el o los factores comunes con la menor potencia o
agrupando los trminos semejantes y tomando sus factores comunes.
As, por ejemplo:

a) 2ab2 4bc = 2b (ab 2c)


b) 4x2 y 3 z 4 8x3 y 2 z 4 + 12x2 y 3 z 3 = 4x2 y 2 z 3 (yz 2xz + 3y)
c) ax + ay bx by = (ax + ay) (bx + by) = a (x + y) b (x + y) = (x + y) (a b)
2 2 2 2
d ) 6x + 2xy 3xy y = 6x + 2xy 3xy + y = 2x (3x + y) y (3x + y) = (2x y) (3x + y)
e) c + d + ac + ad bc bd = (ac bc + c) + (d + ad bd) = c (a b + 1) + d (1 + a b)
= (a b + 1) (c + d)

6. Cuadrado de una expresin: Para elevar al cuadrado una expresin con sumas y/o diferencias de
trminos, se suman los cuadrados de todos y cada uno de sus trminos y los dobles productos de cada
2
trmino por los restantes, pero slo yendo hacia adelante. (a b) es un caso particular del binomio de
Newton.
2
a) (x a) = x2 2ax + a2
2
b) (a + b c) = a2 + b2 + c2 + 2ab 2ac 2bc
2
(a + b + c + + y + z) = a2 + b2 + c2 + + y 2 + z 2 + 2a (b + c + + y + z) +
c)
+2b (c + d + + y + z) + 2c (d + e + + y + z) + + 2x (y + z) + 2yz =
2 2 2 2 2
=a +b +c + + y + z + 2ab + 2ac + + 2az + + 2xy + 2yz

As, por ejemplo:


2 2 2
a) (3x + 2y) = (3x) + 2 3 2xy + (2y) = 9x2 + 12xy + 4y 2
2 2
b) (2x y 2) = (2x) + y 2 + 22 + 2 (2x) ( y) + 2 (2x) ( 2) + 2 ( y) ( 2) = 4x2 + y 2 + 4 4xy 8x + 4y
3 2
c) x2 + y 3 2z t = x4 + y 6 + 4z 2 + t6 + 2x2 y 3 4x2 z 2x2 t3 4y 3 z 2y 3 t3 + 4zt3
p 2 p 2 p p p
d) 2x y 3 + 3 = 2x4 + y 6 + 3 2 2x2 y 3 + 2 6x2 2 3y 3
p p 2 p p p
e) x3 2xy 3 + 3z 2 = x6 + 2x2 y 6 + 3z 4 2 2x4 y 3 + 2 3x3 z 2 2 6xy 3 z 2
n
7. Binomio de Newton: La potencia n-sima (n natural) del binomio a b, es decir, el producto (a b) =
(a b) (a b) (a b), se denomina binomio de Newton y se lo desarrolla con las frmulas:
| {z }
n veces

n n n n (n 1) n n (n 1) (n 2)
(a + b) = an + a 1
b+ a 2 2
b + an 3 3
b + + bn
1! 2! 3!
n n n n (n 1) n n (n 1) (n 2) n
(a b) = an a 1
b+ a 2 2
b an 3 3
b + + ( 1) bn
1! 2! 3!
n n(n 1) n(n 1)(n 2)
Los coecientes 1, 1! , 2! , 3! , , n(n 1) k!(n k+1) , , 1, donde 0 k n, se denominan
coecientes del binomio y pueden ser calculados siguiendo la ley explcita de su formacin de acuerdo a
las frmulas anteriores ( n(n 1) k!(n k+1) es el coeciente general del binomio; k! = 1 2 3 : : : (k 1) k es el
valor de k factorial, as, por ejemplo, 4! = 4 3 2 1 = 24, 3! = 3 2 1 = 6, 1! = 1, 0! = 1, etc.), o utilizando
el tringulo de Pascal:
0
(a + b) = 1 ! 1
1
(a + b) = 1a + 1b ! 1 1
2
(a + b) = 1a2 +2ab + 1b2 ! 1 2 1
3
(a + b) = 1a3 +3a2 b + 3ab2 +1b3 ! 1 3 3 1
4
(a + b) = 1a4 +4a3 b + 6a2 b2 +4ab3 + 1b4 ! 1 4 6 4 1

n n n
(a + b) = an + 1! a b + n(n2! 1) an
1 1 2 2
b + + bn ! 1 n
1!
n(n 1)
2!
n(n 1)(n 2)
3!
n(n 1) (n k+1)
k! 1
2.2. TRANSFORMACIN Y SIMPLIFICACIN DE EXPRESIONES RACIONALES 63

As, por ejemplo:


5
a) (a b) = 1a5 5a4 b + 10a3 b2 10a2 b3 + 5ab4 + 1b5
6
b) (x + b) = 1x6 + 6x5 b1 + 15x4 b2 + 20x3 b3 + 15x2 b4 + 6xb5 + 1b6 = x6 + 6bx5 +
+15b2 x4 + 20b3 x3 + 15b4 x2 + 6b5 x + b6
3 3 2 1 2 3
c) (x 2y) = 1 (x) 3 (x) (2y) + 3 (x) (2y) 1 (2y) = x3 6x2 y + 12xy 2 8y 3
3 3 2 1 2 3
d ) 2x + y 2 = 1 (2x) + 3 (2x) y2 + 3 (2x) y 2 + 1 y2 = 8x3 + 12x2 y 2 + 6xy 4 + y 6
4 4 3 1 2 2 1 3 4
e) 3a2 b c4 = 1 3a2 b 4 3a2 b c4 + 6 3a2 b c4 4 3a2 b c4 + 1 c4 = 81a8 b4
108a6 b3 c4 + 54a4 b2 c8 12a2 bc12 + c16
p 5 5 4 p 2 1 3 p 2 2
f ) 2xy 2 z 3 + 3a2 = 1 2xy 2 z 3 + 5 2xy 2 z 3 3a + 10 2xy 2 z 3 3a +
2 p 3 1 p 4 p 5 p
10 2xy 2 z 3 3a2 + 5 2xy 2 z 3 3a2 + 1 3a2 = 32x5 y 10 z 15 + 80 3a2 x4 y 8 z 12
p p
+240a4 x3 y 6 z 9 + 120 3a6 x2 y 4 z 6 + 90a8 xy 2 z 3 + 9 3a10 :

Observaciones: Los casos de factoreo de expresiones algebraicas, generalmente se reducen a tres, a saber:
1) Suma o diferencia de potencias, An B n ; 2) Descomposicin del trinomio, Ax2n + Bxn + C; 3) Factor
comn o agrupacin.

| La descomposicin en factores puede ser una combinacin de casos de factoreo, as por ejemplo, para
descomponer en factores 2a2 b3 x4 20a2 b3 x2 + 18a2 b3 , tendremos la secuencia:

2a2 b3 x4 20a2 b3 x2 + 18a2 b3 = 2a2 b3 x4 10x2 + 9 = 2a2 b3 x2 9 x2 1 =


2 3
= 2a b (x + 3) (x 3) (x + 1) (x 1) ;

donde intervienen 3 casos de factoreo: factor comn, descomposicin del trinomio y diferencia de potencias.

| Existe la descomposicin factorial mixta que utiliza la completacin de un cuadrado y la diferencia de


cuadrados. Ilustramos este caso con ejemplos:

2
a. x4 +x2 +1 = x4 + x2 + 1 + x2 x2 = x4 +2x2 +1 x2 = x2 + 1 x2 = x2 + x + 1 x2 x+1
| {z } | {z }
com pletacin del cuadrado diferencia de cuadrados

2 2
b. x4 + x2 y 2 + y 4 = x4 + x2 y 2 + y 4 + x2 y 2 x2 y 2 = x2 + y x2 y 2 = x2 + xy + y 2 x2 xy + y 2
2 p p
c. a4 + b4 = a4 + b4 + 2a2 b2 2a2 b2 = a2 + b2 2a2 b2 = a2 + 2ab + b2 a2 2ab + b2 .
| {z } | {z }
com pletacin del cuadrado diferencia de cuadrados
Como se podr ver en este ejemplo, nos hemos salido del campo de los nmeros racionales para entrar al
p
campo de los reales (irracionales), donde est 2.
2 p p 3=2 2
d. x8 + b6 = x8 + b6 + 2b3 x4 2b3 x4 = x4 + b3 2b3 x4 = x4 + 2b3=2 x2 + b2 x4 2b x + b2 .
Aqu tambin nos hemos salido del campo de los nmeros racionales para entrar al campo de los reales
p
(irracionales), donde estn 2 y b3=2 .

| De los anteriores ejemplos se desprende que una descomposicin en factores est condicionada al campo al
cual pertenecen las variables y coecientes de la expresin algebraica. La descomposicin factorial es ms amplia
mientras mayor sea el campo al cual pertenezcan las variables y coecientes de la expresin algebraica a descom-
ponerse, y como tenemos la inclusin Q R C, entonces en el campo de los nmeros complejos, cualquier
expresin algebraica tiene la posibilidad de ser descompuesta en factores. As, por ejemplo, a4 +4 = a4 +4+4a2
2
4a2 = a2 + 2 4a2 = a2 2a + 2 a2 + 2a + 2 en el campo de los nmeros racionales Q; y a4 + 4 = a4
p p p p p p p p
( 4) = a2 4 a2 + 4 = a2 2i a2 + 2i = a 2 i a+ 2 i a 2i a + 2i en el
4 4 2 2 2 2 2 2
p 2
p
campo complejo C. Anlogamente, a +1 = a +1+2a 2a = a + 1 2a = a 2a + 1 a + 2a + 1
64 CAPTULO 2. EXPRESIONES RACIONALES E IRRACIONALES. POLINOMIOS
p p p p
en el campo real R; y a4 + 1 = a4 ( 1) = a2 i a2 + i = a i a+ i a i a+ i en el
campo complejo C. As mismo, en C se tiene que x4 1 = x2 1 x2 + 1 = (x 1) (x + 1) (x i) (x + i) :

| La descomposicin factorial en C conduce a un producto de factores lineales o de primer grado y es siempre


posible, mientras que en los otros campos se vuelve irreductible o irreducible hasta cierto paso. As, por ejemplo
p p p p p p
x4 4 = x2 2 x2 + 2 = x 2 x + 2 x2 + 2 = x 2 x + 2 x + 2i x 2i
| {z } | {z } | {z }
reduccin en Q reduccin en R reduccin en C

Recuerda la identidad,
producto notable o Frmulas
caso de factoreo
Diferencia de cuadrados: a2 b2 = (a b) (a + b) ;
a2n b2n = (an bn ) (an + bn )
Diferencia de cubos: a3 b3 = (a b) a2 + ab + b2
Suma de cubos: a3 + b3 = (a + b) a2 ab + b2
Suma o diferencia
xn an = (x a) xn 1 + axn 2 + + an 2 x + an 1 , cualquier n;
de potencias n n n 1 n 2 2 n 3
x a = (x + a) x ax +a x + an 2 x an 1 ,
n es par;
xn + an = (x + a) xn 1 axn 2 + a2 xn 3 an 2 x + an 1 ,
n es impar.
Descomposicin pqx2 + (pb + aq) x + ab = (px + a) (qx + b) ;
del trinomio pqx2n + (pb + aq) xn + ab = (pxn + a) (qxn + b)
2
(a b) = a2 2ab + b2
Cuadrado de 2
(a + b + c + + z) = a2 + b2 + + z 2 + 2ab + 2ac + 2az+
una expresin
+ + 2xz + 2yz
n
(a + b) = an 1! n n 1
a b + n(n2! 1) an 2 b2 n(n 1)(n
3!
2) n 3 3
a b +
Binomio de n
+ b
Newton
Coeciente general: n(n 1)(n k!
2) (n k+1)
,k n
Agrupacin de trmi- Ax + Ay + + Az = A (x + y + + z)
nos o factor comn ax ay + bx by = (a + b) (x y)

Practico lo que aprend:

Descomponer las siguientes expresiones en el mayor nmero de factores posibles. Analizar los
posibles campos Q, R, C de descomposicin:

1. Utilizar la diferencia de cuadrados:

a) a) x2 25, b) 81x4 1, c) 16x6 9y 4 , d) 100 16y 8 z 4 , e) 16x16 y 4 81z 8 , f ) 16x16 y 4 z8


b) a) x8 16, b) 81y 4 16, c) x6 y 4 , d) 81 16x8 y 12 z 4 , e) 16z 16 81z 8 , f ) x40 z 8 , g) 16x16 625
2 2 2 2 4 4
c) a) (x y) (2x + y) , b) (2a + b c) (3a b + c) , c) (x y) (x + y 1)

2. Utilizar la suma o diferencia de cubos:

a) a) 8x3 1, b) 27x6 1, c) x6 + 8a3 , d) 81 y 9 z 6 , e) x15 y 3 81, f ) x6 + 64y 6 , g) 1 + z 12


b) a) 8x3 + 27, b) x6 1, c) x6 + a12 b6 , d) 1 y 12 z 6 , e) 64x12 z 6 , f ) x6 y 6 , g) 1 z 15
3 3 3 3 6 6
c) a) (x y) + (x + 2y) , b) (2x y 1) 8 (3x + y + 1) , c) (2a b) (a + 2b)

3. Utilizar la suma o diferencia de potencias:


2.2. TRANSFORMACIN Y SIMPLIFICACIN DE EXPRESIONES RACIONALES 65

a) a) y n 1, b) y m z m 1, c) x6 + z 6 , d) 1 y 12 z 6 , e) 1 y 4 z 8 , f ) 32 y 10 z 5 , g) 1 + 64z 12

b) a) x4 16, b) y 8 z 16 1, c) x12 + z 6 , d) y 12 z 18 , e) y 16 z 8 , f ) 32z 5 y 10 , g) 1 + 64z 12


5 5 6 6
c) a) x4 y 8 z 12 1, b) y 9 z 18 1, c) (a b) + (2a + b) , d) (x y + 1) + (2x + y 1) , e) y 36 z 72

4. Utilizar la descomposicin del trinomio:

a) a) x2 + 11x + 30, b) x2 4x 21, c) x2 8x + 15, d) 2x2 3x + 1, e) 6x2 + 7x + 2, f ) 6x2 7x + 2

b) a) 15x2 + 11x + 2, b) 12x2 2x 2, c) 6x4 + x2 2, d) 2x4 + x2 1, e) 3x8 + 5x4 + 2, f ) 6x6 x3 2

c) a) x4 5x2 + 4, b) x4 5x2 36, c) x4 x2 12, d) 9x4 13x2 + 4, e) 8x4 6x2 + 1, f ) 8x4 6x2 + 1,
g) 2x6 3x3 + 1, h) 2x8 x4 1, i) 8x6 + 7x3 1, j) 16x8 17x4 + 1, k) 8x6 + 215x3 27:

5. Utilizar la agrupacin de trminos o factor comn:

a) a) ax ay+2bx 2by, b) 2ax 4ay+bx 2by, c) a+b+ax ay+bx by, d) 2a+2b+2ax ay+2bx by,
e) ax ay bx + az + by bz, f ) b 2a + 4ax 6ay 2bx + 3by, g) ad bd + cd ae + be ce

b) a) a + b c + ax ay + bx by cx + cy, b) 2a + 2b 2x + y + 2ax ay + 2bx by 2, c)


2a+6b+au 3av +3bu 9bv, d) a+b 2u+v +22au av +2bu bv 1, e) a2 x2 a2 y 2 4b2 x2 +4b2 y 2 ,
f ) a2 x a2 y b2 x a2 z + b2 y + b2 z, g) 4a2 9b2 + 4a2 x 4a2 y 9b2 x + 9b2 y:

6. Utilizar el cuadrado de una expresin:

2 2 2 2 2 2
a) a) (3x 5) , b) ( 1 + 2y) , c) ( 3x 2y) , d) (1 a b) , e) (c 2a + b) , f ) ( 2a + bc d)
p 2 p 2 p p 2 2
b) a) c2 2a + b , b) a 2b2 + 3 , c) a 2b2 + 3 c , d) ( a + b c 2d) ,
2 2 p p 2
e) a2 + b ac 2bd , f ) ( x + 2y 3z + 4u) , g) 2x 2 3y 2 3z + 2xy :

7. Utilizar el Binomio de Newton:

3 4 4 5 6 7 5
a) a) (x 3) , b) ( 1 + 2y) , c) ( 2x 3y) , d) (a 2b) , e) (2a + b) , f ) (bc d) , g) a2 c + b3
p 5 p 6 p 4 p p 8 6
b) a) 2b3 c2 + 3 c , b) c 2c2 d , c) 3a2 + 2 3bc3 , d) 2x + 3 , e) (a b + c) :

8. Completar los cuadrados y factorar:

4 4
a) a) x4 + x2 z 2 + z 4 , b) x4 + x2 y 4 + y 8 , c) a8 + a4 y 3 + y 6 , d) x8 + y 8 , e) (2a b) + (a + b) ,
4 4 4 4
f ) (x 2y) + (a + 2b) , g) (a b + c) + (a c) , h) a8 + a4 y 8 + y 16
4 4
b) a) x6 + 3x3 z 3 + 4z 6 , b) 4x8 + 3x4 y + y 2 , c) 4a6 + 3a3 z + z 2 , d) x8 + 256a8 , e) 16 (x 2y) + (x + y) ,
4 4 4 4 8
f ) 81 (x y) + 16 (2x + y) , g) (a b + 3) + 81 (a 2) , h) (x y) + 256y 8

2.2.3. Descomposicin de polinomios en factores

Antes de analizar las propiedades de los polinomios en una forma ms amplia, estudiaremos mecnicamente
un potente mtodo de descomposicin en factores de polinomios en una o varias variables. Dejaremos en claro
que este mtodo no funciona y no es aplicable para todo polinomio.
66 CAPTULO 2. EXPRESIONES RACIONALES E IRRACIONALES. POLINOMIOS

Figura 2.2: Izq. El brillante matemtico sovitico, especialista en ecuaciones diferenciales, oscilaciones y clculo varia-
cional, topologa (teora de la dimensin, homotopa) , grupos continuos y optimizacin dinmica (principio de los
mximos de Pontryagin), totalmente ciego desde los 14 aos, Lev Pontryagin (Mosc,1908 - Mosc, 1988). Der. El
nio prodigio hngaro, Janos von Neumann (Budapest, 1903 - Washington, 1957). Especialista en lgica, teora de
conjuntos, anlisis funcional, mecnica cuntica y en las ms variadas aplicaciones de las matemticas: teora de juegos,
computacin. Se le considera el padre de los ordenadores digitales modernos. Colabor en el Proyecto Manhattan, que
desarroll la bomba atmica norteamericana.

Regla de Ru ni o esquema de Horner

Para resolver muchos problemas algebraicos suele ser preciso representar un polinomio de una o varias
variables en forma de producto de dos o ms polinomios de menor grado o bien en forma del producto del
polinomio por un monomio que contenga no menos de una variable. No obstante, no cada polinomio permite
realizar la descomposicin en factores sobre el campo de nmeros reales R. Por ejemplo, los polinomios x + 3,
x2 + 6x + 10, x2 x + 1, x2 + x + 1, etc. no pueden ser descompuestos en factores de menor grado con
coecientes pertenecientes a R (se ver ms tarde que en C, todos los polinomios se pueden descomponer en
factores (polinomios) de mximo primer grado). Semejantes polinomios reciben el nombre de no reducibles o
irreducibles. Se considera que la descomposicin de polinomios en factores est determinada o nalizada si los
polinomios obtenidos ya no son reducibles en cierto campo numrico Q, R o C.

Durante la descomposicin de polinomios en factores se hace uso de diversos procedimientos: mediante


factor comn o agrupacin, empleando las frmulas de productos notables o multiplicacin abreviada, etc.
Examinemos, mediante varios ejemplos y aplicaciones, el mtodo de descomposicin de Ru ni o esquema de
Horner, el mismo que ser fundamentado ms adelante:

| Analizando la divisin del polinomio P2 (x) = a0 x2 + a1 x + a2 , a0 6= 0 para el polinomio T (x) = x ,


vemos que tendremos un polinomio cociente q (x) = b0 x + b1 y un resto r, donde q tendr grado uno y el resto
r ser una constante (de grado cero):

P2 (x) r a0 x2 + a1 x + a2 r
= q (x) + ) = b 0 x + b1 + ) a0 x2 + a1 x + a2 = (b0 x + b1 ) (x )+r
T (x) T (x) x x

Abriendo la ltima relacin, tendremos que a0 x2 + a1 x + a2 = b0 x2 ( b0 b1 ) x b1 + r. Ahora, para


encontrar b0 , b1 y r igualamos los coecientes de iguales potencias de ambos lados (mtodo de los coecientes
indeterminados). Tendremos las igualdades:
8 8 8 8
>
< a0 = b0 >
< b0 = a0 >
< b0 = a0 >
< b0 = a0
a1 = ( b0 b1 ) ) b1 = b0 + a1 ) b1 = a0 + a1 ) b1 = a0 + a1
>
: >
: >
: >
:
a2 = b1 + r r = b1 + a2 r = ( a0 + a1 ) + a2 r = a0 2 + a1 + a2

De la ltima igualdad r = a0 2 + a1 + a2 se ve que el resto r de la divisin P=T es igual al valor del polinomio
a0 x2 + a1 x + a2 para x = , es decir a0 2 + a1 + a2 . Se concluye que q (x) = b0 x + b1 = a0 x + a0 + a1 y
2.2. TRANSFORMACIN Y SIMPLIFICACIN DE EXPRESIONES RACIONALES 67

r = a0 2 + a1 + a2 = P2 ( ). Es evidente que si la divisin P=T es exacta, entonces r = a0 2 + a1 + a2 =


P2 ( ) = 0 y T sera un divisor de P , o lo que es lo mismo P sera un mltiplo de T . Otra forma de calcular el
resto r de la divisin P=T es poniendo en la igualdad a0 x2 + a1 x + a2 = (b0 x + b1 ) (x ) + r el valor x = ,
2
entonces r = a0 + a1 + a2 = P2 ( ).

Por ejemplo, para hallar el cociente q y el resto r de dividir P2 (x) = 2x2 3x + 4 para T (x) = x + 2 =
2
x ( 2), tendremos q (x) = 2x + ( 2) 2 3 = 2x 7 y r = P2 ( 2) = 2 ( 2) 3 ( 2) + 4 = 18, pues
2
2x 3x + 4 = (2x 7) (x + 2) + 18.

El proceso anterior podemos esquematizarlo (esquema de Horner o mtodo de Ru ni ) as:

2 3 4 2
2
a0 a1 a2 2 ( 2) ( 3) ( 2) + 2 ( 2)
2
a0 a1 + a0 2 2 3 + 2 ( 2) 4 + ( 3) ( 2) + 2 ( 2)
, | {z }
a0 a1 + a0 a2 + a1 + a0 2 resto r
| {z }
resto r # # #
2x 7; r = 18

| Analizando ahora la divisin del polinomio P3 (x) = a0 x3 + a1 x2 + a2 x + a3 , a0 6= 0 para el polinomio


T (x) = x , vemos que al hacer la divisin, tendremos un polinomio cociente q (x) = b0 x2 + b1 x + b2 y un
resto r, donde q tendr grado dos y el resto r ser una constante:

P3 (x) r a0 x3 + a1 x2 + a2 x + a3 r
= q (x) + ) = b0 x2 + b1 x + b2 +
T (x) T (x) x x
) a0 x3 + a1 x2 + a2 x + a3 = b0 x2 + b1 x + b2 (x )+r

Abriendo la ltima relacin, tendremos que a0 x3 +a1 x2 +a2 x+a3 = b0 x3 ( b0 b1 ) x2 ( b1 b2 ) x b2 +r.


Ahora, para encontrar b0 , b1 , b2 y r igualamos los coecientes de iguales potencias de ambos lados (mtodo de
los coecientes indeterminados). Tendremos las igualdades:
8 8 8
>
> a0 = b0 >
> b0 = a0 >
> b0 = a0
>
< a >
< b = b +a >
< b = a +a
1 = ( b 0 b1 ) 1 0 1 1 0 1
) )
>
> a2 = ( b 1 b2 ) >
> b 2 = b 1 + a2 >
> b 2 = ( a0 + a1 ) + a2
>
: >
: >
:
a3 = b2 + r r = b2 + a3 r = ( ( a0 + a1 ) + a2 ) + a3
8
>
> b0 = a0
>
< b = a +a
1 0 1
)
>
> b 2 = ( a0 + a1 ) + a2
>
:
r = a0 3 + a1 2 + a2 + a3

De la ltima igualdad r = a0 3 + a1 2 + a2 + a3 se ve que el resto r de la divisin P=T es igual al valor


del polinomio a0 x3 + a1 x2 + a2 x + a3 para x = , es decir a0 3 + a1 2 + a2 + a3 . Se concluye que q (x) =
b0 x2 + b1 x + b2 = a0 x2 + ( a0 + a1 ) x + ( a0 + a1 ) + a2 y r = a0 3 + a1 2 + a2 + a3 = P3 ( ). Es evidente
que si la divisin P=T es exacta, entonces r = a0 3 + a1 2 + a2 + a3 = P3 ( ) = 0 y claro T es un divisor de
P , o P es un mltiplo de T . Otra forma de calcular el resto r de la divisin P=T es poniendo en la igualdad
a0 x3 +a1 x2 +a2 x+a3 = b0 x2 + b1 x + b2 (x )+r el valor x = , entonces r = a0 3 +a1 2 +a2 +a3 = P3 ( ).

Por ejemplo, para hallar el cociente q y el resto r de dividir P3 (x) = 5x3 3x + 2 para T (x) = x 1,
3
tendremos q (x) = 5x2 + ((1) 5 + 0) x + (1 ((1) 5 + 0) 3) = 5x2 + 5x + 2 y r = P3 (1) = 5 (1) 3 (1) + 2 = 4,
3 2
pues 5x 3x + 2 = 5x + 5x + 2 (x 1) + 4.

El proceso anterior podemos esquematizarlo (esquema de Horner o mtodo de Ru ni ) as:


68 CAPTULO 2. EXPRESIONES RACIONALES E IRRACIONALES. POLINOMIOS

5 0 3 2 1
a0 a1 a2 a3 5 (1) 5 (1) 2 (1)
a0 a1 + a0 2 a2 + a1 2 + a0 3 5 5 2 4
2
, |{z}
a0 a1 + a0 a2 + a1 + a0 a3 + a2 + a1 2 + a0 3 resto r
| {z } # # # #
resto r
5x2 +5x +2; r=4

| Generalizando el proceso anterior para la divisin del polinomio P3 (x) = a0 xn +a1 xn 1 + +an 1 x+an ,
a0 6= 0 para el polinomio T (x) = x , veremos que al hacer la divisin, tendremos un polinomio cociente
n 1 n 2
q (x) = b0 x + b1 x + + bn 2 x + bn 1 , b0 6= 0 y un resto r, donde q tendr grado n 1 y el resto r ser
una constante:
P3 (x) r a0 xn + a1 xn 1 + + an 1x + an r
= q (x) + ) = b0 xn 1
+ b1 xn 2
+ + bn 2x + bn 1 +
T (x) T (x) x x
n n 1 n 1 n 2
) a0 x + a1 x + + an 1x + an = b0 x + b1 x + + bn 2x + bn 1 (x )+r

Abriendo la ltima relacin, tendremos que los coecientes buscados b0 , b1 , b2 , etc. se hallarn por igualacin
y recursivamente (mtodo de los coecientes indeterminados):
8 8
>
> a0 = b0 >
> b0 = a0
>
> >
>
>
> a1 = b1 b0 >
> b1 = a0 + b0
>
> >
>
< a2 = b2 b1 < b2 = a2 + b1
)
>
> >
>
>
> >
>
>
> a = b b >
> bn 1 = an 1 + bn 2
>
> n 1 n 1 n 2 >
>
: :
an = r bn 1 r = an + bn 1

y el valor del resto r puede ser calculado de la relacin

a0 xn + a1 xn 1
+ + an 1x + an = b0 xn 1
+ b1 xn 2
+ + bn 2x + bn 1 (x )+r

mediante la sustitucin x = , es decir,


n n 1
r = a0 + a1 + + an 1 + an = Pn ( ) :

Recuerda las siguientes reglas sobre la divisibilidad de polinomios y sus races:

| El proceso anterior se simplica representndolo con una tabla o escalera de clculo, denominado esquema
de Horner o mtodo de Ru ni.

| El anterior resultado se denomina Teorema de Bezout, teniendo adems como consecuencia que el
polinomio Pn (x) se divide para el binomio T (x) = x exactamente, si y solamente si, el valor del poli-
nomio Pn (x) para x = es igual a cero, es decir, Pn ( ) = 0 o lo que quiere decir que x = es una raz
del polinomio Pn (x). Todos estos resultados conducen a un nuevo mtodo de descomposicin en factores o
factoreo de polinomios (mtodo de Ru ni ), pues si r = 0, entonces a0 xn + a1 xn 1 + + an 1 x + an =
n 1 n 2
b0 x + b1 x + + bn 2 x + bn 1 (x ):

| Por simple sustitucin se demuestra que al dividir el polinomio Pn (x) para el binomio T (x) = x
se obtiene un resto r igual al valor del polinomio Pn (x) para x = = , es decir, r = Pn ( = ). As mismo, si
r = Pn ( = ) = 0, entonces x = = es una raz del polinomio Pn (x), o lo que es lo mismo Pn (x) es divisible
exactamente para el binomio T (x) = x , es decir T es un divisor de Pn .

| Se demuestra fcilmente que si x = con 2 Z, 6= 0 es una raz del polinomio Pn (x) = a0 xn +a1 xn 1 +
+ an 1 x + an de coecientes enteros, entonces el trmino independiente an es divisible exactamente para
esta raz . Adems, races enteras del polinomio Pn (x) = a0 xn + a1 xn 1 + + an 1 x + an de coecientes
enteros, pueden ser solamente los divisores del trmino independiente an .
2.2. TRANSFORMACIN Y SIMPLIFICACIN DE EXPRESIONES RACIONALES 69

| As mismo, para que el nmero racional x = = , donde y son enteros mutuamente primos, sea raz
del polinomio Pn (x) = a0 xn + a1 xn 1 + + an 1 x + an de coecientes enteros, es necesario que sea divisor
del trmino independiente an y que sea divisor del coeciente a0 . Esta condicin es solamente necesaria, pero
no suciente.

Ejemplos:

1. Descomponer el polinomio P = P (a; b) = a2 2a3 b + b2 en factores.


Solucin: Unamos los sumandos extremos en un grupo y los medios en otro, y en el segundo grupo
saquemos de los parntesis el factor comn. Obtenemos:
P (a; b) = a2 + b2 2ab a2 + b2 = a2 + b2 (1 2ab).

2. Descomponer en factores P = P (a) = a3 7a2 + 7a + 1


Solucin: Representemos los trminos 2do y 3ro del polinomio prejado de la forma siguiente: 7a2 =
3a2 4a2 ; 7a = 12a 5a. Entonces escribimos P (a) = a3 3a2 4a2 + 12a 5a + 15.
Agrupamos los sumandos pares y en cada grupo sacamos de los parntesis los factores comunes:
P (a) = a3 3a2 4a2 12a (5a 15) = a2 (a 3) 4a (a 3) 5 (a 3) = (a 3) a2 4a 5 .
Queda por descomponer en factores el polinomio a2 4a 5. Empleando la frmula de descomposicin
en factores de un trinomio cuadrtico ax2 + bx + c = a (x x1 ) (x x2 ) , obtenemos: a2 4a 5 =
(a a1 ) (a a2 ) = (a + 1) (a 5). As pues, P (a) = (a 3) (a + 1) (a 5).

3. Descomponer en factores P = P (a; b; c) = ab (a + b) bc (b + c) + ac (a c).


Solucin: Aprovechemos que la expresin en los primeros parntesis es la suma de las expresiones
contenidas en los parntesis segundo y tercero: a + b = (b + c) + (a c). Entonces
P (a; b; c) = ab ((b + c) + (a c)) bc (b + c) + ac (a c) = ab (b + c) + ab (a c) bc (b + c) + ac (a c).
A continuacin, efectuamos la agrupacin de los trminos y sacamos de los parntesis el factor comn.
Obtenemos:
P (a; b; c) = (ab (b + c) bc (b + c)) + (ab (a c) + ac (a c)) = (b + c) (ab bc) + (a c) (ab + ac) =
= (b + c) b (a c) + (a c) a (b + c) = (a c) (b + c) (a + b).

4. Descompongamos en factores P = P (a) = a3 5a2 a+5


Solucin: Realicemos la agrupacin y, despus, saquemos de los parntesis el factor comn:
P (a) = a3 5a2 (a 5) = a2 (a 5) (a 5) = (a 5) a2 1 . Empleando seguidamente la
frmula p2 q 2 = (p q) (p + q) obtenemos f (a) = (a 5)(a 1)(a + 1)

5. Descompongamos en factores P = P (a; b) = 4a2 12ab + 5b2


Solucin: Completamos el binomio 4a2 12ab hasta el cuadrado perfecto. Obtenemos: (2a)2 2(2a)(3b) +
(3b)2 .
Entonces P (a; b) = (4a2 12ab + 9b2 ) 9b2 + 5b2 = (2a 3b)2 (2b)2 =
= (2a 3b 2b)(2a 3b + 2b) = (2a 5b)(2a b).

6. Descompongamos en factores P (a) = a4 10a2 + 169


Solucin: Como a4 + 169 = (a2 )2 + 132 , completando el cuadrado perfecto, obtenemos
P (a) = (a4 + 26a2 + 169) 26a2 10a2 = (a2 + 13)2 (6a)2 = (a2 6a + 13)(a2 + 6a + 13).

7. Descomponer en factores P = P (a; b) = a6 + a4 + a2 b2 + b4 b6


Solucin: Como a6 b6 = (a3 )2 (b3 )2 = (a3 b3 )( a3 + b3 = (a b)(a2 + ab + b2 ) y a4 + a2 b2 + b4
= (a4 + 2a2 b2 + b4 ) a2 b2 = (a2 + b2 )2 (ab)2 = (a2 + ab + b2 )(a2 ab + b2 ), entonces P (a; b) =
(a2 + ab + b2 )(a2 ab + b2 )((a b)(a + b) + 1) = (a2 + ab + b2 )(a2 ab + b2 )(a2 b2 + 1):
70 CAPTULO 2. EXPRESIONES RACIONALES E IRRACIONALES. POLINOMIOS

8. Descomponer en factores P = P (a) = a3 + 9a2 + 27a + 19


Solucin: Es fcil ver que para completar el cuadrado perfecto a la expresin P (a) le falta 8. Por ello,
podemos escribir
3
P (a) = a3 + 9a2 + 27a + 27 8 = (a + 3) 23 = (a + 3 2) (a + 3)2 + 2(a + 3) + 4 = (a + 1)(a2 +
8a + 19).

.
9. Demostrar que si a 2 N y P = P (a) = a4 + 6a3 + 11a2 + 6a, entonces P (a) ..24:
Solucin: representamos 6a3 y 11a2 en forma de suma de los trminos semejantes: 6a3 = a3 + 5a3 y
11a2 = 5a2 + 6a2 . Entonces
P (a) = a4 + a3 + 5a3 + 5a2 + 6a2 + 6a = (a4 + a3 ) + 5a3 + 5a2 + 6a2 + 6a =
a3 (a + 1)+5a2 (a + 1)+6a (a + 1) = a (a + 1) a2 + 5a + 6 = a (a + 1) (a + 2) (a + 3). Pero de 4 nmeros
naturales sucesivos, por lo menos uno de ellos se divide por 3, as como 2 nmeros son pares, es decir, uno
de ellos se divide tambin por 4, por lo tanto, el producto de estos cuatro nmeros se divide por 3 2 4. As
.
pues, P (a) ..24.

.
10. Demostrar que si P = P (a) = a2 a2 + 14 + 49, donde a es un nmero impar, entonces P (a) ..64.
2
Solucin: Notemos que P (a) = a2 a2 + 14 + 49 = a2 + 7 . Como a es impar, a = 2n 1, donde n 2 N,
entonces,
2 2
P (a) = P (2n 1) = ((2n 1)2 + 7)2 = 4n2 = 16 n2 n + 2 . La expresin obtenida se
4n + 8
. .
divide por 16. Por esta razn, para demostrar que P (a) ..64 es suciente demostrar que (n2 n + 2)2 ..4.
2
Analicemos 2 posibles casos: 1) n es un nmero par: (n2 n + 2)2 = ((2k) 2k + 2)2 = 4(k 2 k + 1)2 , y
2
entonces la expresin obtenida es divisible por 4. 2) n es un nmero impar: (n2 n + 2)2 = ((2k 1)
2
2 (2k 1) + 2)2 = 4k 2 8k + 5 , y entonces la expresin obtenida no es divisible por 4.
.
Observaciones: Recordemos que el signo .. signica se divide por (sin resto).
.
| Si n es par, n2 tambin lo es y, por lo tanto, n2 n + 2 es par, o sea, n2 n + 2 ..4, lo que signica
.
que P (a) ..64.
| Si n es impar, n2 tambin lo es, pero entonces n2 n es par y n2 n + 2, tambin. As, pues, en
.
semejante caso P (a) ..64.

11. El polinomio Pn (x) = xn n


se divide exactamente por el binomio x , n 2 N, pues r = Pn ( ) =
n n
= 0:

12. El polinomio Pn (x) = xn n


se divide exactamente por el binomio x + con n par (n = 2m), pues
2m 2m
r = P2m ( ) = ( ) = 0:

13. El polinomio Pn (x) = xn + n se divide exactamente por el binomio x + , n impar (n = 2m + 1), pues
2m+1
r = P2m+1 ( ) = ( ) + 2m+1 = 0:

14. Demostrar que el nmero 20n + 16n 3n 1 con n par (n = 2m) se divide por 19:
n n n
Solucin: Tenemos 20 + 16 3 1 = 202m 1 + 162m 32m =
(20m 1) (20m + 1) + (16m + 3m ) (16m 3m ), donde en el 1er sumando, el 1er factor se divide exacta-
mente, para m 2 N, por el nmero (20 1), es decir por 19. En el 2do sumando, el 1er factor se divide
exactamente, para m impar (m = 2k + 1, k 2 N) por el nmero (16 + 3) = 19; y cuando m es par (m = 2k,
k 2 N) el 2do factor equivale a 16k 3k 16k + 3k que al factorarlo sucesivamente, tarde o temprano
aparecer el factor (16 + 3) = 19, quedando demostrada la proposicin.
2.2. TRANSFORMACIN Y SIMPLIFICACIN DE EXPRESIONES RACIONALES 71

15. Hallar el cociente y el resto de dividir P3 (x) = 2x3 x2 + 4x 3 para: a) T1 (x) = x 3, b) T2 (x) = x + 3,
c) T3 (x) = 2x 3, d) T3 (x) = 2x + 3:
3 2
Solucin: a) = 3 ) r = P3 ( ) = 2 (3) (3) + 4 (3) 3 = 54, q (x) = 2x2 + 5x + 19. b) = 3)
3 2
r = P3 ( ) = 2 ( 3) ( 3) + 4 ( 3) 3 = 78, q (x) = 2x2 7x + 25:
2 1 4 3 3 2 1 4 3 3
2 (3) 5 (3) 19 (3) 2 ( 3) 7 ( 3) 25 ( 3)
2 5 19 54
|{z} 2 7 25 78
, |{z}
resto resto
# # # # # #
2x2 +5x +19 2x2 7x +25
3 2
c) = = 3=2 ) r = P3 ( = ) = 2 (3=2) (3=2) + 4 (3=2) 3 = 15=2, q (x) = x2 + x + 7=2, pues
3
x2 + x + 7=2 (2x 3) + 15=2 = 2x3 x2 + 4x 3. d) = = 3=2 ) r = P3 ( = ) = 2 ( 3=2)
2
( 3=2) + 4 ( 3=2) 3 = 18, q (x) = x2 2x + 5, pues x2 2x + 5 (2x + 3) 18 = 2x3 x2 + 4x 3:
2 1 4 3 3=2 2 1 4 3 3=2
2 (3=2) 2 (3=2) 7 (3=2) 2 ( 3=2) 4 ( 3=2) 10 ( 3=2)
2 2 7 15=2 2 4 10 18
|{z} |{z}
resto , resto
# # # # # #
( 2) ( 2) ( 2) ( 2) ( 2) ( 2)
x2 +x +7=2 x2 2x +5

16. Descomponer en factores y hallar las races de los polinomios: a) P3 (x) = x3 + 3x2 4x 12, b)
P4 (x) = x4 + 4x3 x2 4x, c) P3 (x) = 6x3 + 13x2 + x 2
Solucin: a) Para P3 (x) = x3 + 3x2 4x 12 tenemos que 12 es divisible por 1, 2, 3, 4, 6, 12,
por tanto, las races enteras estn entre estos nmeros. Utilizando el esquema Horner-Ru ni, probamos,
por ejemplo, con x = 1, luego con x = 2, etc. y si el resto r es igual a 0, entonces obtenemos las races
requeridas:
1 3 4 12 1 1 3 4 12 2
1 ( 1) 2 ( 1) 6 ( 1) 1 (2) 5 (2) 6 (2)
1 2 6 6 1 5 6 0
|{z}
|{z} ,
resto resto
# # # # # #
x2 +2x 6 x2 +5x +6
Se tiene que x = 1 no es raz y x = 2 es raz. Como el cociente es q (x) = x2 + 5x + 6 = (x + 3) (x + 2),
entonces tendremos P3 (x) = x3 + 3x2 4x 12 = (x + 3) (x + 2) (x 2), siendo sus 3 races x = 3,
x = 2, x = 2:
b) Para P4 (x) = x4 + 4x3 x2 4x = x x3 + 4x2 x 4 tenemos que 4 es divisible por 1, 2, 4,
por tanto, las races enteras estn entre estos nmeros. Utilizando el esquema Horner-Ru ni, probamos,
por ejemplo, con x = 1, luego con x = 2, etc. y si el resto r es igual a 0, entonces obtenemos las races
requeridas:
1 4 1 4 1
1 ( 1) 3 ( 1) 4 ( 1)
1 3 4 0
|{z}
resto
# # #
x2 +3x 4
Se tiene que x = 1 es raz. Como el cociente es q (x) = x2 + 3x 4 = (x + 4) (x 1), entonces tendremos
P4 (x) = x4 + 4x3 x2 4x = x (x + 4) (x 1) (x + 1), siendo sus 4 races x = 0, x = 1, x = 1, x = 4:
72 CAPTULO 2. EXPRESIONES RACIONALES E IRRACIONALES. POLINOMIOS

c) Para P3 (x) = 6x3 + 13x2 + x 2 tenemos que a0 = 2 es divisible por 1, 2; y an = 6 es divisible


por 1, 2, 3, 6, por tanto, las races racionales estn entre los nmeros racionales: 1, 2, 3, 6,
1=2, 1=3, 2=3, 1=6. Utilizando el esquema Horner-Ru ni, probamos, por ejemplo, con x = 1=2,
luego con x = 1=2, etc. y si el resto r es igual a 0, entonces obtenemos las races requeridas:
6 13 1 2 1=2 6 13 1 2 1=2
6 (1=2) 16 (1=2) 9 (1=2) 6 ( 1=2) 10 ( 1=2) 4 ( 1=2)
6 16 9 5=2 6 10 4 0
|{z} , |{z}
resto resto
# # # # # #
6x2 +16x 9 6x2 +10x 4
Se tiene que x = 1=2 no es raz y x = 1=2 si es raz. Como el cociente es q (x) = 6x2 + 10x 4 =
2 (x + 2) (3x 1), entonces tendremos que P4 (x) = 6x3 + 13x2 + x 2 = 2 (x + 2) (3x 1) (x 1=2) =
(x + 2) (3x 1) (2x 1), siendo sus 3 races x = 1=3, x = 1, x = 1=2, x = 2:

17. Descomponer en factores las expresiones: a) R (a; b; x) = 3ab + ax 2a2 b2 + x2 , b) S (a; b; x) =


x3 abx + 2a2 b + ax2 2a2 x bx2 , c) T (a; b; x) = x3 + a2 b a2 x bx2 :
Solucin: a) Analizamos la expresin R como un polinomio en x: R (a; b; x) = R2 (x) = x2 + ax
2a2 3ab + b2 = x2 + ax (2a b) (a b) y utilizamos la regla de Ru ni evaluando R2 (x) para
x = a b.
1 a 2a2 + 3ab b2 a b 1 a b 2a2 ab 2a2 b a
1 (a b) (2a b) (a b) 1 (a) (2a b) (a) 2ab (a)
1 2a b 0
|{z} , 1 2a b 2ab 0
|{z}
resto resto
# # # # #
x 2a b x2 + (2a b) x 2ab
Se tiene que x = a b es una raz. Como el cociente es q (x) = x + 2a b, entonces tendremos que
R2 (x) = x2 + ax 2a2 3ab + b2 = (x a + b) (x + 2a b), siendo sus 2 races x = a b, x = 2a + b:
b) Analizamos a S como polinomio en x: S (a; b; x) = S3 (x) = x3 + (a b) x2 2a2 + ab x + 2a2 b y
evaluamos a S3 (x) con Ru ni-Horner para x = a, x = b, x = 2a, x = 2b, etc., que son los divisores
de 2a2 b.
Se tiene que x = a es una raz. Como el cociente es q (x) = x2 + (2a b) x 2ab = (x + 2a) (x b),
entonces tendremos que S3 (x) = x3 + (a b) x2 2a2 + ab x + 2a2 b = (x + 2a) (x b) (x a), siendo
sus 3 races x = 2a, x = b, x = a:
c) Analizamos a T como polinomio en x: T (a; b; x) = T3 (x) = x3 bx2 a2 x + a2 b y evaluamos con la
regla de Ru ni a T3 (x) para x = a, x = b, x = a2 , etc., que son divisores de a2 b :
1 b a2 a2 b b
1 (b) 0 (b) a2 (b)
1 0 a2 0
|{z}
resto
# # #
x2 +0 a2
Se tiene que x = b es una raz. Como el cociente es q (x) = x2 a2 = (x a) (x + a), entonces tendremos
que T3 (x) = x3 bx2 a2 x + a2 b = (x a) (x + a) (x b), siendo sus 3 races x = a, x = a, x = b:

Practico lo que aprend:

Descomponer en factores los siguientes polinomios:

1. a) a4 1, b) a6 1, c) a6 + 1, d) a4 18a2 + 81, e) a12 2a6 + 1, f ) a5 + a3 a2 1:


2.2. TRANSFORMACIN Y SIMPLIFICACIN DE EXPRESIONES RACIONALES 73
2
2. a) a4 + 2a3 2a 1, b) 4b2 c2 b2 + c2 a2 , c) a4 a2 b2 + b4 , d) a4 + 4a2 5

3. a) 4a4 + 5a2 + 1, b) c4 (1 + ab) c2 + ab, c) a4 + 324, c) a8 + a4 + 1, d) a5 + a3 a2 1

4. a) 2a4 + a3 + 4a2 + a + 2, b) a4 + 3a3 + 4a2 12, c) a2 + a + 3 a2 + a + 4 12,

5. a) 2a2 b + 4ab2 a2 c + ac2 4b2 c + 2bc2 4abc, b) (ab + ac + bc) (a + b + c) abc

2 2 2 2 5
6. a) a (b 2c) + b (a 2c) 2c (a + b) + 8abc, b) a3 a2 7 36a, c) (a + b) a5 + b5

7. a) a2 b2 (b a) + b2 c2 (c b) + a2 c2 (a c), b) 8a3 (b + c) b3 (2a + c) c3 (2a b)

3
8. a) (a + b + c) a3 + b3 + c3 , b) a4 + 9, c) a4 + b4 , d) a3 + 5a2 + 3a 9, e) a4 + a2 + 1

9. a) a (a + 1) (a + 2) (a + 3) + 1, b) (a + 1) (a + 3) (a + 5) (a + 7) + 15, c) a10 + a5 + 1

2 2
10. a) 2 a2 + 2a 1 + 5 a2 2a 1 a2 + 1 + 2 a2 + 1 , b) (a b) c3 (a c) b3 + (b c) a3

3 3 3 3 3 3 p
11. a) (a b) + (b c) (a c) , b) a2 + b2 b2 + c2 a2 c2 , c) a4 + a2 + 2a + 2:

12. a) a4 + 2a3 b 3a2 b2 4ab3 b4 , b) a2 b2 + ab2 + a2 c + b2 c + bc2 + 3abc

13. a) a4 + b4 + c4 2a2 b2 2a2 c2 2b2 c2 , b) a5 + a4 + a3 + a2 + a + 1

14. a) a4 + 2a3 + 3a2 + 2a + 1, b) a4 2a3 b 8a2 b2 6ab3 b4

.
15. Demostrar que si a 2 N, entonces a5 5a3 + 4a ..120:

.
16. Demostrar que si a es un nmero mutuamente primo con 6, entonces a2 1 ..24:

.
17. Demostrar que si a 2 N, entonces 2a3 + 3a2 + a ..6:

18. Con qu valores de a 2 N la expresin a4 + 4 es un nmero primo?

a a2 a3
19. Demostrar que si a es nmero par, entonces + + es un nmero entero.
12 8 24

a5 a4 7a3 5a2 a
20. Demostrar que si a 2 N, entonces + + + + es un nmero entero.
120 12 24 12 5
21. Descomponer los siguientes polinomios en producto de por lo menos 2 factores:

1) a) 5mx + 3ny 5my 3ny, b) 5x + xy + 5y + y 2 , c) ax bx + by + cy cx ay, d) 3a5 6a4 b + 3a3 b2 ,


e) 36x2 y 2 100, f ) 4p2 q 4 81z 2 , g) x5 y 10 32, h) 9x4 37x2 + 4, i) 32y 10 z 15
2 2 3
2) a) (m + 2n) 4 (3m n) , b) a4 c2 + 9y 2 6a2 y, c) 125a3 343b3 , d) (m n) + 1,
3 3 3
e) (2a b) (3b a) , f ) a4 x4 16y 4 , g) 8 (a + b) 1, f ) a8 z 4 1

3) a) 216x6 +37x3 8, b) x4 10x2 +9, c) x6 +7x3 8, d) 6x2 +7x 3, e) 10x2 13x 3, f ) x8 10x4 +9
3 4 4
4) a) x2 +ax+(a b) b, b) (2x 1) +x6 , c) x4 +16, d) (x 1) +16 (x + 3) , e) x4 +x2 +1, f ) x5 +x+1
3 3 3 3
5) a) x2 + y 2 + z2 x2 y 2 + z 2 , b) x3 + y 3 + z 3 (x + y + z) , c) x3 + y 3 + z 3 3xyz,
4 2 4 4
d) 4x4 12x2 y 2 + 9y 1, e) (x b) (2x + b) , f ) a16 b8 c24 1
74 CAPTULO 2. EXPRESIONES RACIONALES E IRRACIONALES. POLINOMIOS

Figura 2.3: Izq. Henri Lebesgue (Beauvais, Oise, 1875 - Pars, 1941), el clebre matemtico y pedagogo francs, creador
de la medida (generalizacin de longitud, rea, volumen) de Lebesgue para conjuntos ms generales y de la integral
ms general de Lebesgue, que es una ampliacin y generalizacin de la integral de Riemann (Breselenz, Hannover,
1626 - Selasca, Italia, 1866) para funciones ms generales y ms complicadas. Resolvi varias cuestiones de topologa
(pavimentacin de supercies), teora de la medida y teora moderna de funciones. En sus obras (Lecciones sobre la
integracin y el anlisis de las funciones primitivas (1904) y Lecciones sobre las series trigonomtricas (1906)), se ampla
el estudio de las series trigonomtricas, el anlisis de Fourier y la teora de funciones. Der. La notable mecnica
(estudio fsico matemtico sobre la rotacin de un slido alrededor de un punto) y matemtica rusa Sofa Kovalvskaya
(Mosc, 1850 - Estocolmo, 1891), especialista en ecuaciones en derivadas parciales (tesis: Zur Theorie der partiellen
Dierentialgleichungen) . Al no ser aceptada en la universidad de Berln (1871), por ser mujer, fue discpula particular
del famoso matemtico alemn y admirador suyo, Karl Weierstrass (Ostenfelde, Baviera, 1815 - Berln, 1897).

2.2.4. Expresiones racionales fraccionarias. Operaciones con fracciones y su sim-


plicacin

P
Toda expresin algebraica en forma de fraccin , donde P y Q son expresiones racionales enteras (poli-
Q
nomios) y Q con toda seguridad contiene variables, se denomina fraccin racional. As, ejemplos de fracciones
racionales seran:

2x 1 2ax b + 2 b=a + 2c=b


1. a) 2
, b) p , c) p
4x + 1=3 (a + 1=3) x 2=3 a 3 (b=a + 2c=b)
b+2
2x3 8 2ab2 bz=x2 + 2cx3 =y 2
2. a) 2 , b) c , c) .
x + 2a 1=3 (ax2 y 3 4) (x=y 2y=x)
ay 2 + (xy 1)
b c

P P R
| La propiedad fundamental de las fracciones racionales viene dada por la identidad = , donde
Q Q R
Q 6= 0, R 6= 0 y R es una expresin racional entera. Esto signica que el numerador y denominador de una
P
fraccin racional puede ser multiplicado o dividido por la misma constante, monomio o polinomio R 6= 0, sin
Q
que la fraccin altere. As, por ejemplo:

2x 1=2 6 (2x 1=2) 12x 3 b=a + 2c=b ab ( b=a + 2c=b) b2 2ac


a) = = =, b) = =
x2 + 1=3 6 ( x2 + 1=3) 6x2 + 2 x=a 2y ab ( x=a 2y) bx + 2aby
a a
2ax2 y
2=a 1=b + 3=c 2abc (2=a 1=b + 3=c) 4bc 2ac + 6ab 2
2x y 2x2 y
a) = = , b) x = =
abcx2 abcx2 a2 b2 c2 x2 2 1 x
1=abc 2abc 1=abc 2a 2ax2 y 1
2 2 2a
2.2. TRANSFORMACIN Y SIMPLIFICACIN DE EXPRESIONES RACIONALES 75

a2
x3 y + 2ax2 y

| Utilizando la propiedad fundamental de las fracciones racionales tendremos que se cumplen las siguientes
propiedades:
P P P P P P P P P
a) = , b) = , c) = , d) = =
Q Q Q Q Q Q Q Q Q
Estas propiedades se reducen a decir que una fraccin racional no se altera si cambiamos de signo numerador
y denominador; o si cambiamos de signo la fraccin y su numerador; o si cambiamos de signo la fraccin y su
denominador; o si cambiamos de signo 2 factores que estn o bien en el numerador, o bien en el denominador,
o bien un factor en el numerador y el otro en el denominador. En general un nmero par de cambios de signo
en una fraccin no alteran su valor. As, por ejemplo:

x 3 x+3 x+3 x 3 ax ax ax
a) 2
= 2 = = , b) = =
x +a x a x2 + a x2 a ( x + 1) (2 x) (x 1) (2 x) ( x + 1) (x 2)
ax
=
(x 1) (x 2)
a
(cx a) (b 3x) (cx a) (3x b) (cx a) (b 3x) (cx a) (3x b) 3
a) = = = , b) 2x =
( x + c) (a x) (x c) (a x) (x c) (x a) (x c) (x a) xy
2b 1
b
a a a
+3 3 +3
2x = 2x = 2x :
xy xy xy
2b + 1 2b + 1 2b 1
b b b

Simplicacin de fracciones

P
Simplicar una fraccin signica dividir tanto el numerador P como el denominador Q para un factor
Q
P
comn a ambos. Para que la fraccin pueda ser simplicada es necesario factorar tanto numerador como
Q
P
denominador, y si resulta que tanto P como Q poseen factores comunes, entonces la fraccin puede ser sim-
Q
plicada, caso contrario la fraccin no es reducible. Es evidente que la transformacin en factores del numerador
P y denominador Q se la debe hacer en el mismo campo numrico Q, R o C, y tomar muy en cuenta el c:v:a:
para transformaciones idnticas.

As, por ejemplo:

2x2 + 5x 3 (2x 1) (x + 3) 2x 1 8x3 1


1. a) 2
= = , x 6= 1, 3 en el campo Q; b) =
x + 4x + 3 (x + 1) (x + 3) x+1 4x2 1
(2x 1) 4x2 + 2x + 1 4x2 + 2x + 1
= , x 6= 1=2, 1=2 en el campo Q;
(2x 1) (2x + 1) 2x + 1
ab + ax bx b2 (a b) (x + b) a b
c) = = , a 6= b, x 6= b en el campo Q:
ab + ax + bx + b2 (a + b) (x + b) a+b

x2 3xy x (x 3y) x (x 3y) x x3 1


2. a) 2 2
= 2 2
= = , x 6= 3y, 3y en el campo Q; b) 4 =
9y x (x 9y ) (x 3y) (x + 3y) x + 3y x 1
(x 1) x2 + x + 1 x2 + x + 1
= , x 6= 1, 1 en el campo Q;
(x 1) (x + 1) (x2 + 1) (x + 1) (x2 + 1)
p p
x2 2 x 2 x+ 2 1 p p
c) 4 = p p = 2 , x 6= 2, 2 en el campo R.
x 4 x 2
2 x + 2 (x + 2) (x + 1)
76 CAPTULO 2. EXPRESIONES RACIONALES E IRRACIONALES. POLINOMIOS

| Una fraccin racional de varias variables a, b, . . . , x, y, etc. puede representarse mediante P = P (a; b; : : : ; x; y),
Q = Q (a; b; : : : ; x; y), R = R (a; b; : : : ; x; y), etc. que nos indica que la fraccin racional contiene tanto en el
numerador como en el denominador coecientes y algunas variables.

Ejemplos:

a4 10a2 + 169
1. Simplicar la fraccin P (a) = .
a2 + 6a + 13
Solucin: Factorando el numerador, tenemos: a4 10a2 + 169 = a2 + 6a + 13 a2 6a + 13 .
a2 + 6a + 13 a2 6a + 13
Por lo tanto, P (a) = = a2 6a + 13. Como a2 + 6a + 13 no se reduce a cero
a2 + 6a + 13
2
para ningn valor real de a (pues a2 + 6a + 13 = (a + 3) + 4 > 0), entonces P (a) = a2 6a + 13 para
todos los valores reales de a.

2a2 + ab b2
2. Simplicar la fraccin P (a; b) = .
a+b
Solucin: Representando ab como la suma de los trminos semejantes 2ab ab, obtenemos

2a2 + ab b2 = 2a2 + 2ab ab b2 = 2a (a + b) b (a + b) = (a + b) (2a b).


(a + b) (2a b)
Entonces P (a; b) = = 2a b. La simplicacin por a + b se ha realizado partiendo de la
a+b
condicin de que a + b 6= 0. De modo que P (a; b) = 2a b, si a 6= b.

a4 10a2 + 169
3. Simplicar la fraccin P (a) = .
a2 + 6a + 13
Solucin: Factorando el numerador, obtenemos que: a4 10a2 + 169 = a2 + 6a + 13 a2 6a + 13 .
a2 + 6a + 13 a2 6a + 13
Por lo tanto, P (a) = = a2 6a + 13. Como a2 + 6a + 13 no se reduce a cero
a2 + 6a + 13
2
para ningn valor real de a (pues a2 + 6a + 13 = (a + 3) + 4 > 0), entonces P (a) = a2 6a + 13 para
todos los valores reales de a.

Reduccin de fracciones racionales a comn denominador

Se denomina comn denominador c:d: de varias fracciones racionales a una expresin racional entera di-
visible para cada uno de los denominadores de las fracciones dadas, es decir, el comn denominador c:d: viene
a ser el mnimo comn mltiplo m:c:m: de todos los denominadores de las fracciones dadas.

| Para encontrar el comn denominador c:d: de varias fracciones, hallamos el m:c:m: de los denominadores,
es decir, factoramos todos y cada uno de los denominadores y tomamos los factores comunes y no comunes con
el mayor exponente.

Ejemplos:

3a c 2a + c
1. Hallar el comn denominador c:d: de las fracciones y , y reducirlas a comn denominador.
a+b a b
3a c 2a + c
Solucin: El comn denominador c:d: de las fracciones y es (a + b) (a + b) o a2 b2 y su
a+b a b
3a c (3a c) (a b) 2a + c (2a + c) (a + b)
reduccin a comn denominador nos da = y = :
a+b a2 b2 a b a2 b2

3 1 x 1
2. Hallar el c:d: de las fracciones , y , y reducirlas a comn denominador.
x+3 x 3 x
2.2. TRANSFORMACIN Y SIMPLIFICACIN DE EXPRESIONES RACIONALES 77

3 1 x 1
Solucin: El comn denominador c:d: de las fracciones
, y es x (x + 3) (x 3) o
x+3 x 3 x
3 3x (x 3) 1 x (x + 3)
x x2 9 y su reduccin a comn denominador nos da = , = y
x+3 x (x2 9) x 3 x (x2 9)
x 1 (x 1) x2 9
= :
x x (x2 9)
1 1 1
3. Hallar el comn denominador c:d: de las fracciones , y , y reducirlas a comn denominador.
c+a c b c
1 1 1
Solucin: El comn denominador c:d: de las fracciones , y es c (c + a) (c b) y su
c+a c b c
1 c (c b) 1 c (c + b) 1
reduccin a comn denominador nos da = , = y =
c+a c (c + a) (c b) c b c (c + a) (c b) c
(c + a) (c b)
:
c (c + a) (c b)
3 1 x
4. Hallar el c:d: de las fracciones 2, y , y reducirlas a comn denominador.
(x + 3) x2 9 x2 + 2x 3
3x 1
Solucin: El comn denominador c:d: de las fracciones 2, y
que despus de fac-
(x + 3) 9 + 2x 3 x2 x2
3 1 1 x x
torar sus denominadores equivalen a 2, = y = ,
(x + 3) x2 9 (x 3) (x + 3) x2 + 2x 3 (x + 3) (x 1)
2 3 3 (x 3) (x 1)
es (x + 3) (x 3) (x 1) y su reduccin a comn denominador nos da 2 = 2 ,
(x + 3) (x + 3) (x 3) (x 1)
1 (x + 3) (x 1) x x (x + 3) (x 3)
= 2 y = 2 :
x2 9 (x + 3) (x 3) (x 1) x2 + 2x 3 (x + 3) (x 3) (x 1)

Practico lo que aprend:

Simplicar las fracciones e indicar el c:v:a: en el que es posible tal simplicacin as como el
campo numrico en el que se efecta tal descomposicin en factores:

x2 + x 6 8x3 + 1 ax ab bx + b2 a2 + ac 2c2 x3 9x
1. a) 2
; b) 2
; c) 2
; d) 2 2
; e) ;
x + 7x + 12 4x 1 ax ab + bx b a 3ac + 2c 3x + 4x3 + x4
2

a3 + a2 a 1 4b 2a + ax 2bx a2 4ac + 4c2 a2 + b2 + 1 + 2a 2b 2ab


f) 3 2
; g) ; h) ; i)
a a a+1 ax 4b 2a + 2bx a 2c a b+1
2
(x 3) 27x3 + 1 (b + a) (b + x) a2 c4 x3 16x a3 27
2. a) ; b) ; c) ; d) ; e) ; f ) ;
x2 + 7x + 12 9x2 1 ax ab + bx b2 a2 + ac2 2c4 x4 2x3 3x2 2a2 a 15
2 2 2 2 2
ab ax bx + x a ac 6c a + b + 1 2a 2b + 2ab
g) ; h) 2 ; i) :
2ab ax 2bx + x2 (a + 2c) a+b 1
Hallar el comn denominador c:d: de las fracciones y reducirlas a comn denominador c:d.
a c a+c c b a c b 1 x 2 2
3. a) y ; b) 2 , 2 y ; c) 2 , 2 y ; d) 2 y 2 ; e) 2,
a+b a b (a + b) (a b) a b b a a b (x + 1) x 1 (x 1)
1 x x 2 1 x 1 1 a b
y 2 ; f) , y 2 ; g) , y 2 ; h) , y
x+1 x 1 x 1 x+1 x + 3x + 2 x 2 x+3 x +x 6 x+2 x+4
a2 a 1 a2 a 1 b 1 2 1
2
; i) 2
, y 3 2
; j) , y 2 2
; k) , y 2 :
x + 6x + 8 x x + 2 x + 2x x y x+y x y x y x + 2y x + xy 2y 2
a c a+c 2 1 a x y z 1 2
4. a) y ; b) , y 2 2
; c) 2 , 2 y ; d) 2 y 2
;
a b a+b a b a b (a + 3) a 9 a+3 3 (x 2) 2x 8
2 1 x x 2 1 x 1 1
e) 2, 2x 6
y 2
x 9
; f)
x 1
,
x + 2
y 2
x + x 2
; g)
3 (x 4)
,
2x + 6
y 2
x x 12
;
2 (x + 3)
a b a2 a 1 a2 3 1 3
h) , y 2 ; i) , y 3 ; k) , y :
x+2 3x + 3 x + 3x + 2 3x x + 2 x + 2x2
2 x y 2x + 4y x2 + xy 2y 2
78 CAPTULO 2. EXPRESIONES RACIONALES E IRRACIONALES. POLINOMIOS

Operaciones con fracciones racionales

P1 P2 P1 P2 Pn
Para las fracciones y (y en general para un nmero nito n de fracciones , , , ) estn
Q1 Q2 Q1 Q2 Qn
denidas las operaciones de suma o diferencia, producto, divisin (cociente) y potenciacin con enteros.

| Las operaciones con fracciones algebraicas racionales son muy semejantes a las operaciones con fracciones
numricas que ya fueron analizadas en el captulo anterior.

| Por analoga con los nmeros enteros, para hallar el mnimo comn mltiplo (comn denominador o c:v:
de los denominadores Q1 , Q2 , : : :) m:c:m: (Q1 ; Q2 ; : : :) = c:d: (Q1 ; Q2 ; : : :), primero se descomponen en factores
todos y cada uno de los denominadores Q1 , Q2 , : : : y luego se toman los factores comunes y no comunes con el
mayor exponente, siendo esta expresin la buscada.
P1 P2
Recuerda las operaciones con las fracciones , :
Q1 Q2

P1 P2
Suma y diferencia: Diferenciamos 2 casos. El caso cuando las fracciones y poseen iguales denomi-
Q1 Q2
nadores, es decir, cuando Q1 = Q2 , entonces tendremos

P1 P2 P1 + P2 P1 P2 P1 P2
+ = y =
Q1 Q1 Q1 Q1 Q1 Q1

As, por ejemplo:

2x 1 3x + 2 (2x 1) + (3x + 2) 5x + 1
a) + = =
x+3 x+3 x+3 x+3
2x 1 3x + 2 (2x 1) (3x + 2) x 3 x+3
b) = = = = 1
x+3 x+3 x+3 x+3 x+3
x3 y3 x3 + y 3 x2 xy + y 2 (x + y)
c) + = = = x2 xy + y 2
x+y x+y x+y x+y
x a b x a+b
d) + =
x a x a x a x a
a x a x a x a x 1
e) 2 2 2
= 2 2 = 2 = 2 =
(x a) x 2ax + a (x a) (x a) (x a) (a x) a x
P1 P2
Y el caso cuando las fracciones y poseen distintos denominadores, es decir, cuando Q1 6= Q2 ,
Q1 Q2
entonces tendremos
P1 P2 P1 R1 P2 R2 P1 R1 P2 R2
= = ;
Q1 Q2 c:d: (Q1 ; Q2 ) m:c:m: (Q1 ; Q2 )
donde R1 = c:d: (Q1 ; Q2 ) Q1 y R2 = c:d: (Q1 ; Q2 ) Q2 (posteriormente, de ser posible, P1 R1 P2 R2
y la fraccin resultante deben ser simplicados). Anlogamente se dene una suma algebraica con
ms de 2 fracciones. Tenemos, por ejemplo:
2x y 3x + y (2x y) (x 3) + (3x + y) (x + 3) 5x2 + 3x + 6y
a) + = =
x+3 x 3 (x + 3) (x 3) x2 9
2x 1 2x 1 2x (x 1) 1 (x + 1) 2x2 3x 1
b) 2 = 2 = 2 = 2
(x + 1) x2 1 (x + 1) (x 1) (x + 1) (x + 1) (x 1) (x + 1) (x 1)
2x 6x 2x 6x 2x (x + 3) 6x 2x2
c) + = = =
x 5 (x + 3) (5 x) x 5 (x + 3) (x 5) (x + 3) (x 5) (x + 3) (x 5)
x 1 1 x 1 1 (x 1) (x + 1) + 1 x2 1
d) + = + 2 = = 2 =
x2 x3 +x2 x2 x (x + 1) 2
x (x + 1) x (x + 1) x+1
2
a b c a (x + a) b (x + a) + c
e) 2 + 3 = 3
x+a (x + a) (x + a) (x + a)
2.2. TRANSFORMACIN Y SIMPLIFICACIN DE EXPRESIONES RACIONALES 79
a x x a x x
f) 2 + 2 = 2 2 + 2 =
(b a) b2 2ab + a2 a 2ab + b 2
(a b) (b a) (a b)
a x x a
= 2 2 + 2 = 2:
(a b) (b a) (b a) (a b)

P1 P2 P1 P2 P1 P2
Producto: Para multiplicar las fracciones y tenemos la regla = , es decir, se multiplican
Q1 Q2 Q1 Q2 Q1 Q2
los numeradores y denominadores entre s. La fraccin resultante, si es posible, debe ser simplicada. As,
por ejemplo:

5x 5 4x + 4 5 (x 1) 4 (x + 1) 5x2 5
a) = = 2
8x + 24 3x + 9 8 (x + 3) 3 (x + 3) 6 (x + 3)
x+3 x x+3 x x (x + 3) x
b) = = = 2
x + 2 x2 + 5x + 6 x + 2 (x + 2) (x + 3) (x + 2) (x + 2) (x + 3) (x + 2)
2
y+1 3x x2 + 4x + 4 (y + 1) 3x (x + 2) x+2
c) = =
3x + 6 y 2 2y 3 x2 3 (x + 2) (y 3) (y + 1) x2 x (y 3)
2
x3 x2 4abx + 4a2 b2 x3 (x 2ab)
d) 2
= = x (x 2ab) :
x 2ab x x 2ab x2
P1 P2
Divisin: Para efectuar la divisin de la fraccin para y hallar su cociente, tenemos la regla
Q1 Q2
P1
P1 P2 Q1 P1 Q2
= = ,
Q1 Q2 P2 P2 Q1
Q2
es decir, se multiplican los extremos y medios por separado y se forma la fraccin resultante. La fraccin
resultante o cociente, si es posible, debe ser simplicada.
As, por ejemplo:

x 1 x+1 (x 1) 5 (x + 3) 5x 5
a) = =
x+3 5x + 15 (x + 3) (x + 1) x+1
2
x+3 x x+3 (x + 2) (x + 3) (x + 3)
b) = =
2x + 4 x2 + 5x + 6 2 (x + 2) x 2x
x+2 x (x + 2) 3 (y 3) (y + 1) (x + 2) (y 3)
c) = =
6y + 6 3y 2 6y 9 6 (y + 1) x 2x
2
4x3 2x2 4x3 (x 2ab)
d) = = 2x (x 2ab) :
x 2ab x2 2
4abx + 4a b 2 x 2ab 2x2
P
Potenciacin: Para elevar la fraccin a la potencia n (n es natural o entero) es necesario elevar a la n, en
Q
forma separada, tanto el numerador como el denominador, es decir, se tiene
n n n
P P P P Pn P Q
= = n; = ; P 6= 0; Q 6= 0:
Q Q Q Q Q Q P
| {z }
n veces

As, por ejemplo:


3 3
3a2 bx3 3a2 bx3 9a6 b3 x9 9a3 b3 x9
a) = 3 = =
2ay 2 (2ay 2 ) 8a3 y 6 8y 6
3 2 3 2
3a2 bx3 9ab2 x2 3a2 bx3 4ay 3 27a6 b3 x9 16a2 y 6 2a6 x5
b) = = =
2c2 y 2 4ay 3 2c2 y 2 9ab2 x2 8c6 y 6 81a2 b4 x4 3bc6
2 2 2 2
x+3 6x + 12 x+3 6 (x + 2) 3 x2 (x + 2)
c) = = =
2x2 + 4x x2 + 5x + 6 2x (x + 2) (x + 3) (x + 2) x (x + 2) 9
80 CAPTULO 2. EXPRESIONES RACIONALES E IRRACIONALES. POLINOMIOS
1 1 1
x+2 x (x + 2) 3 (y 3) (y + 1) (x + 2) (y 3) x
d) = = = (x+2)(y 3)
3y + 3 3y 2 6y 9 3 (y + 1) x x

| Para la simplicacin de expresiones algebraicas en las que intervienen operaciones mixtas con fracciones,
tales como sumas, diferencias, productos, divisiones y potenciacin entera, es conveniente calcular correctamente
los comunes denominadores y aplicar las reglas de descomposicin en factores o factoreo.

Ejemplos:

2 2
1 2a 1 (a 3) + 12a
1. Simplicar P (a) = 2
+ 2 + 2 .
a + 3a + 2 a + 4a + 3 a + 5a + 6 2
Solucin: Realizando las operaciones indicadas, obtenemos:
2
a + 3 + 2a (a + 2) + a + 1 a2 6a + 9 + 12a
P (a) = =
(a + 1) (a + 2) (a + 3) 2
2 2 2
2a2 + 6a + 4 a2 + 6a + 9 4 a2 + 3a + 2 (a + 3)
= 2 2 =2
(a + 1) (a + 2) (a + 3) 2 (a2 + 3a + 2) (a + 3) 2

As, pues, P (a) = 2 si a 6= 1, a 6= 2, a 6= 3.


a2 b2 c2
2. Simplicar P (a; b; c) = + + .
(a b) (a c) (b c) (b a) (c a) (c b)
Solucin: Reduciendo todas las fracciones al mnimo comn denominador, obtenemos:

a2 (b c) b2 (a c) + c2 (a b)
P (a; b; c) = :
(a b) (b c) (a c)

Advirtiendo que b c = (a c) (a b), transformamos el numerador de la siguiente forma: a2 (b c)


b2 (a c) + c2 (a b) =
= a2 (a c) a2 (a b) b2 (a c) + c2 (a b) = (a c) a2 b2 + (a b) c2 a2 =
= (a c) (a b) (a + b c a) = (a b) (b c) (a c).
As, pues , P (a; b; c) = 1 si a 6= b, a 6= c, b 6= c.

3. Demostremos que si a + b + c = 0, entonces a3 + b3 + c3 = 3abc.


3
Solucin: Como a + b + c = 0, a = b c. Entonces, a3 + b3 + c3 = ( b c) + b3 + c3 =
3
= (b + c) + b3 + c3 = b3 + 3b2 c + 3bc2 + c3 + b3 + c3 = 3b2 c + 3bc (b + c). Pero b + c = a. De
modo que a3 + b3 + c3 = 3bc ( a) = 3abc.

4. Demostraremos que si a + b + a = 0, donde a 6= 0, b 6= 0, c 6= 0, entonces


a b b c c a c a b
+ + + + = 9:
c a b a b b c c a

Solucin: Consideremos el producto del primer factor por la primera fraccin del segundo factor:
a b b c c a c b c c a c
+ + =1+ + =
c a b a b a b a b

b2 bc + ac a2 c c (a b) a2 b2 c
1+ =1+ =
ab a b ab a b
(a b) (c (a + b)) c c (c a b)
1+ =1+ .
ab a b ab
2c2
Segn el planteamiento a + b = c, por ello, para el producto que consideramos, obtenemos 1 + .
ab
2.2. TRANSFORMACIN Y SIMPLIFICACIN DE EXPRESIONES RACIONALES 81

De forma anloga el producto del primer factor por la segunda fraccin del segundo factor es igual a
2a2 2b2
1+ , en tanto que por la tercera fraccin, 1 + . Sumando los resultados obtenidos
bc ca

2c2 2a2 2b2 c2 a2 b2 2 c3 + a3 + b3


1+ +1+ +1+ =3+2 + + =3+
ab bc ca ab bc ac abc

2 a3 + b3 + c3 2 3abc
Como a3 + b3 + c3 = 3abc, entonces 3 + = 3+ = 9, que es lo que se deseaba
abc abc
demostrar.

Practico lo que aprend:

Operaciones con fracciones:

1. Reducir a un comn denominador los siguientes grupos de fracciones:


1 1 1 1 1 1 1 1 1
a) 2
, 2 2
, 2 ; b) 2 , 3 2
, 2 2
; c) 2 , , 2 ;
xy + x x y 2x 2xy (a + b) a a b a b a + a 2 a2 4a + 3 a 1
1 1 1 1 1 1 1 1 1
d) , , 2 ; e) 2 , , ; f) , 2 ,
a2 4 a2 + 3a + 2 a + 2a b 2b 3 b2 b 2 2b2 2 y2 4y + 4 y 4 ay 2a
2. Simplicar las expresiones:

a+2 3a2 + 6a a3 2a b2 4b 45 b2 12b 45 b2 4 b + 11


a) 1) , 2)
3 a2+ 4a + 4 a4 4a2 + 4 b2 14b 15 b2 6b 27 b2 121 b + 2
7 12a3 + 24a2 a2 + 2a 16a2 49 2a2 a 1 (x b) (x c) (x a) (x c)
b) 1) 12 , 2) + +
14a2 7a 2a 1 2 2
4a + a 14 2a + 5a + 2 (a b) (a c) (b a) (b c)
(x a) (x b) 6x2 5x 6 12x3 + 20x2 21x 36
, 3)
(c a) (c b) 6x2 + x 12 12x3 28x2 + 3x + 18

3. Demostrar las siguientes relaciones:


2 2
a2 (a + x) (a + y) a+2 a 1 a+1 4 2 3 1
a) + 2 2
= 1, b) + + = 2, c) +
xy x xy xy y a+1 a 2 a+2 4 a2
2 a x+1 x+3
3 1 48 m2 nr mn2 r mnr2
= 2 2
, d) + + =0
1 x 3 x (x 9) (x 1) (m n) (m r) (n r) (n m) (r m) (r n)
Simplicar las siguientes expresiones:
x2 4 x 2 x2 9 x 3 x2 9 x+3
4. a) , b) 3 , c) 3
x3 + 8 2x2 4x + 8 x + 27 3x2 9x + 27 x 27 3x2 + 6x + 18
x2 4 x+2 x2 4 x2 5x + 6
5. a) , b) 2
x3 8 2x2 + 4x + 8 x + 4x + 4 x2 x 6
0 ! 1
x2 4 x2 5x + 6 2 x2 4 x 2 1

6(x2 4x+4) @ A
2 3
x + 8 2x2 4x + 8
6. a) x + 4x + 4 x2 x 6 , b) 0 1
x2 4 x 2 x2 9 x+3
@ A(x 2)2
x3 + 8 2x2 4x + 8 x3 27 3x2 + 9x + 27
0 1
2 x+2 x2 4
(x2
+4x+4) @ A
2x2 + 4x + 8 x3 8
7. 1
x2 4 x2 5x + 6
x2 + 4x + 4 x2 x 6
1
2x2 4x + 8 x3 + 8
x 2 x2 4

1 2 1 x2 + x 6
8. +
x2 +x 2x2 x 3 4x 6 2x2 + 2x
82 CAPTULO 2. EXPRESIONES RACIONALES E IRRACIONALES. POLINOMIOS
0 1 !
(x + 3) (x 2) 1 2 1
@ A +
2
2x + 2x 2
x + x 2x 2 x 3 4x 6
9. 0 1 0 1
x2 4 x2 5x + 6
@ A @ A
x2 + 4x + 4 x2 x 6
0 1 0 1
(x + 3) (x 2) 1 2 1
B 2 C +
10. B
(2x + 2x) C B 2
@ x +2x 2x
2 x 3 4x 6 C
A
@ x2 4 x2 5x + 6 A x 4 x+2
x2 + 4x + 4 x2 x 6 x3 8 2x2 + 4x + 8
5a2 a 4 a6 a4 + a2 1 a4 + a2 2 a4 a2 12 2a4 + 7a2 + 6
11. a) , b) , c) , d) , e)
a3 1 a3 + a2 + a + 1 a6 + 8 a4 + 8a2 + 15 3a4 + 3a2 6
5a4 + 5a2 3a2 3b a4 + a2 b2 + b4 1 1 2a 4a3 8a7
12. a) 4 2
, b) 6 6
, c)
a + 3a + 2 a b 1 a 1+a 1 + a2 1 + a4 1 + a8
a a2 + a 1 a2 a 1 2a3
13. + + 3
a2 1 a3 a + a 1 a + a2 + a + 1
2 a4 1
1 1
b a a b + b2 + c2 a2
14. a) +a b +b a , b) a b + c 1+
a+b a b a+b a b 1 1 2bc
a b+c
1 1 1
15. + +
(a b)(a c) (b c)(b a) (c a)(c b)

a3 b ab3 + b3 c bc3 + c3 a ca3 (a2 b2 )3 + (b2 c2 )3 + (c2 a2 )3


16. a) , b)
a2 b ab2 + b2 c bc2 + c2 a ca2 (a b)3 + (b c)3 + (c a)3
17. Demostrar las identidades:
b c c a a b 2 2 2
a) + + = + + ,
(a b)(a c) (b c)(b a) (c a)(c b) a b b c c a
(d b)(d c) (d c)(d a) (d a)(d b)
b) a2 + b2 + c2 = d2
(a b)(a c) (b c)(b a) (c a)(c b)

18. Demostrar que si a; b; c 2 R, de la igualdad (a b)2 +(b c)2 +(c a)2 = (a+b 2c)2 +(b+c 2a)2 +(c+a 2b)2
se deduce que a = b = c.

19. Demostrar que si a 2 R, entonces (a 1)(a 3)(a 4)(a 6) + 10 es un nmero positivo.

a5 + b5 + c5 a3 + b3 + c3 a2 + b2 + c2
20. Demostrar que si a + b + c = 0, entonces =
5 3 2
l m n a b c l2 m2 n2
21. Demostrar que si + + =1y + + = 0, entonces 2 + 2 + 2 = 1:
a b c l m n a b c
a b c a b
22. Demostrar que si + + = 0, donde a 6= b, a 6= c, b 6= c, entonces + +
b c c a a b (b c)2 (c a)2
c
= 0:
(a b)2

(a3 + b3 + c3 )(a4 + b4 + c4 )
23. Demostrar que si a+b+c = 0, entonces a5 (b2 +c2 )+b5 (a2 +c2 )+(c5 b2 +a2 ) = :
(b c)2

2.3. Estudio Posterior de los Polinomios de una Variable

Es interesante saber que en varios sistemas numricos como son los racionales Q, los reales R y complejos
C, mantenindose dentro de los lmites de cada sistema, es posible efectuar no solo la suma y producto, sino
tambin la resta de sus elementos. Esto no siempre es posible hacerlo con los enteros o reales positivos (Z+ ; R+ ).
2.3. ESTUDIO POSTERIOR DE LOS POLINOMIOS DE UNA VARIABLE 83

Todo sistema de nmeros reales (complejos) que contiene la suma, la diferencia y el producto de 2 cualesquiera
de sus elementos, se denomina anillo numrico 1 . Son anillos numricos los sistemas numricos de todos los
enteros, racionales, reales y complejos. No son anillos numricos los sistemas numricos positivos o negativos de
los enteros, racionales o reales.

Existen otros ejemplos ms interesantes de anillos (nmeros, polinomios, funciones, matrices, etc.):

Ejemplos:

1. Los enteros pares: la suma, diferencia y producto de 2 enteros pares nos da un nuevo entero par.

2. Los enteros divisibles por n 2 N. Por ejemplo, los mltiplos de 2 (los pares); los enteros mltiplos de 3;
etc.
p
3. El conjunto de nmeros del tipo a + b 2 con a, b 2 Q. A este anillo pertenecen, por ejemplo, todos los
p
racionales Q (cuando b = 0); el mismo nmero 2 (cuando a = 0 y b = 1); etc.

4. El conjunto de los polinomios de primer grado }1 = fP1 (x) = a1 x + a0 ; a1 6= 0g, el conjunto de los
polinomios de segundo grado }2 = P2 (x) = a2 x2 + a1 x + a0 ; a2 6= 0 , el conjunto de los polinomios
de tercer grado }3 = P3 (x) = a3 x3 + a2 x2 + a1 x + a0 ; a3 6= 0 , etc., donde x es la indeterminada o
variable y a0 , a1 , etc. son coecientes racionales, reales o complejos. Ejemplos de polinomios son: P1 (x) =
p
4x 3=2, P2 (x) = x2 + 32 x 5, P3 (x) = 35 x3 + 8x2 5; P1 (x) = 2x 1, P2 (x) = 2 x2 + x 7=2,
p q
P3 (x) = 35 x3 +2 2x2 ; P1 (x) = 3x i=2, P2 (x) = (2 i) x2 + 23 x 5i, P3 (x) = 35 x3 +8x2 5ix+2 7i;

5. El conjunto general de los polinomios en la indeterminada x con coecientes de un campo o cuerpo dado
(Q; R; C) e incluso de un anillo numrico dado:

} = Pn (x) = an xn + an 1x
n 1
+ an 2x
n 2
+ : : : + a1 x + a0 ;

donde n 2 N es el grado del polinomio; a0 , a1 , : : :, an 1 , an son los coecientes del polinomio, nmeros
pertenecientes a cierto cuerpo (campo) o anillo numrico dado.
Para cierto valor x = x0 , el valor o nmero P (x0 ) se denomina valor del polinomio en el punto x0 . La
relacin funcional

y = Pn (x) = an xn + an 1x
n 1
+ an 2x
n 2
+ : : : + a1 x + a0 ;

segn la cual a cada valor x (x 2 R; C) le corresponde un nico valor de y (y 2 R; C) dado por la frmula
anterior, se denomina funcin polinomial. Si los coecientes del polinomio y su variable x son reales,
entonces tenemos una funcin polinomial real.

Denicin. Un conjunto < se denomina anillo, si se han denido en l 2 operaciones, llamadas adicin
o suma y multiplicacin, siendo ambas conmutativas y asociativas, y ligadas por la ley distributiva, poseyendo
adems la suma la operacin inversa, llamada diferencia (resta). Para un anillo numrico o un sistema de
polinomios (representamos con a, b, c, etc. a los elementos, nmeros o polinomios del anillo) se cumplen los
axiomas:

1) La adicin es conmutativa : a + b = b + c
2) La adicin es asociativa: a + (b + c) = (a + b) + c
3) La multiplicacin es asociativa: a (bc) = (ab) c
4) La adicin y multiplicacin estn asociadas por la ley distributiva: (a + b) c = ac + bc
1 Para estudios posteriores consultar el curso [6] de la bibliografa.
84 CAPTULO 2. EXPRESIONES RACIONALES E IRRACIONALES. POLINOMIOS

| La multiplicacin de los nmeros posee la propiedad: si el producto de 2 nmeros es igual a cero,


al menos uno de los factores es igual a cero. Esto no es cierto para cualquier anillo, puede suceder que
a 6= 0, b 6= 0, y sin embargo ab = 0; los elementos a, b con esta propiedad se denominan divisores de cero. Entre
los anillos numricos no se pueden hallar ejemplos de anillos con divisores de cero. Tampoco contienen divisores
de cero los anillos de polinomios con coecientes numricos. Presentamos algunos anillos con divisores de cero.

Ejemplos:

1. En el anillo de matrices Mn (R) de orden n = 2 (matrices o tablas cuadradas 2 2 compuestas de 2 las


y de 2 columnas) y de elementos reales, tenemos el ejemplo
! ! !
1 0 0 0 0 0
= ;
0 0 0 1 0 0
| {z }| {z } | {z }
a6=0 b6=0 ab=0

por tanto, las matrices a y b de arriba son divisores de cero y a b = 0:

2. En el anillo de funciones tenemos el ejemplo


( (
0, si x 0 3x, si x 0
f (x) = ; g (x) = y f (x) g (x) = 0;
2x, si x > 0 0, si x > 0 | {z }
| {z } | {z } ab=0
a6=0 b6=0

por tanto, las dos funciones f y g son divisores de cero y f g = 0.

| Para el anillo } de los polinomios tenemos que se cumplen las propiedades2 :

1. El anillo (}; +; ) es conmutativo unitario. El elemento neutro (con unidad) es el polinomio P (x) = 1
para el producto y P (x) = 0 para la suma.

2. El anillo (}; +; ) es ntegro (sin divisores de cero). Se tiene que

P (x) Q (x) = 0 , (P (x) = 0) _ (Q (x) = 0) ;

es decir, o P es cero, o Q es cero, o ambos.

3. Si para el escalar a 2 R (C) se cumple para el polinomio Pn (x) la relacin Pn (a) = 0, entonces el valor
x = a, se denomina raz del polinomio Pn . As, por ejemplo, si P3 (x) = x3 + 2x + 4, entonces el valor
3
x = 2 es una de sus races, pues P3 (2) = (2) + 2 (2) + 4 = 0. As mismo, para P2 (x) = 2x2 + 5x 3,
2
los valores x = 1=2 y x = 3 son sus 2 races, pues P2 (1=2) = 2 (1=2) + 5 (1=2) 3 = 0 y P2 ( 3) =
2
2 ( 3) + 5 ( 3) 3 = 0. Para P2 (x) = x2 + 2x + 2, los valores x = 1 + i y x = 1 i son sus 2 races,
2 2
pues P2 ( 1 + i) = ( 1 + i) + 2 ( 1 + i) + 2 = 0 y P2 ( 1 i) = ( 1 i) + 2 ( 1 i) + 2 = 0:
Tenemos los siguientes teoremas:

a) El valor x = a es raz (o cero) de Pn si y slo si (x a) es divisor de Pn (x), es decir, al dividir


Pn (x)
Pn (x) por (x a) obtenemos como resto r = 0, o lo que es lo mismo = Qn 1 (x), o Pn (x) =
x a
(x a) Qn 1 (x). As, por ejemplo, el valor x = 1 es una de las 3 races del polinomio P3 (x) = 2x3
3 2 Pn (x) 2x3 3x2 + x + 6
3x2 +x+6, pues P3 ( 1) = 2 ( 1) 3 ( 1) +( 1)+6 = 0 y = = 2x2 5x+
x a x ( 1)
6 = Qn 1 (x) = Q2 (x). As mismo, x = 2 es una de las 4 races del polinomio P4 (x) = 3x4 21x 6,
4 Pn (x) 3x4 21x 6
pues P4 (2) = 3 (2) 21 (2) 6 = 0 y = = 3x3 + 6x2 + 12x + 3 = Qn 1 (x) =
x a x 2
2 Ver apndice al nal del libro.
2.3. ESTUDIO POSTERIOR DE LOS POLINOMIOS DE UNA VARIABLE 85

Q3 (x). Los valores x = 1 i, x = 1+i son 2 de las 3 races del polinomio P3 (x) = x3 +x2 4x+6, pues
P3 (1 i) = (1 i)3 +(1 i)2 4(1 i)+6 = 0 y P3 (1+i) = (1+i)3 +(1+i)2 4(1+i)+6 = 0, cumplindose
Pn (x) x3 + x2 4x + 6 x3 + x2 4x + 6
que = = = x + 3 = Qn 1 (x) = Q1 (x) :
(x a1 ) (x a2 ) (x 1 + i) (x 1 i) x2 2x + 2
b) Todo polinomio Pn (x) no nulo y de grado n posee un mximo de n races (se incluye la multiplicidad
o repeticin de las races). As, por ejemplo, P4 (x) = x4 13x2 + 36 posee 4 races simples: x = 3,
x = 3, x = 2, x = 2. El polinomio P3 (x) = x3 + 2x2 4x 8 = (x + 2) (x + 2) (x 2) posee
2 races mltiples o con repeticin y una raz simple: x = 2, x = 2 y x = 2. El polinomio
P4 (x) = x4 + 2x2 + 1 posee 2 parejas de races mltiples o con repeticin: x = i, x = i y x = i,
x = i.
c) Teorema fundamental del lgebra: Todo polinomio P (z) de cualesquiera coecientes numricos
(de R o C), cuyo grado n 2 N no sea menor a la unidad, tiene por lo menos una raz, generalmente
compleja.
d ) Teorema de D Alembert: Todo polinomio no constante en el cuerpo o campo C posee n races,
generalmente complejas, incluyendo su multiplicidad. Adems:
1) Todo polinomio Pn (x) en el cuerpo C puede descomponerse en un producto de n factores lineales
o de primer grado: Pn (x) = an (x 1 ) (x 2 ) : : : (x n) ; i 2 C.
3 2
As, por ejemplo: P3 (x) = x + 2x 4x 8 = (x + 2) (x + 2) (x 2); P4 (x) = x4 13x2 + 36 =
(x 2) (x 3) (x + 3) (x + 2); P3 (x) = x3 + x2 4x + 6 = (x + 3) (x 1 + i) (x 1 i)
2) Sea el polinomio Pn (x) = xn + a1 xn 1 + a2 xn 2 + : : : + an 1 x + an y i sus races, incluyendo
su grado de multiplicidad (repeticin) en el cuerpo C, entonces se cumplen las frmulas de
Vieta:
8
>
> a1 = ( 1 + 2 + : : : + n )
>
>
>
> a2 = 1 2 + 1 3 + ::: + 1 n + 2 3 + ::: n 1 n
>
>
< a = (
3 1 2 3 + 1 2 4 + : : : + n 2 n 1 n)
>
>
>
>
>
> an 1 = ( 1)
n 1
( 1 2 n 1+ 1 2 n 2 n + ::: + 2 3 n)
>
>
: n
an = ( 1) 1 2 n

As, por ejemplo: para P3 (x) = x3 +2x2 4x 8 = (x + 2) (x + 2) (x 2) se tiene ( 1 + 2 + 3 ) =


( 2 2 + 2) = 2 = a1 , 1 2 + 1 3 + 2 3 = ( 2) ( 2) + ( 2) (2) + ( 2) (2) = 4 = a2 y
( 1 2 3 ) = ( 2) ( 2) (2) = 8 = a3 . As mismo, si P2 (x) = 2x2 + 5x + 2 = 2 x2 + 52 x + 1 =
(2x + 1) (x + 2), entonces ( 1 + 2 ) = ( 1=2 2) = 5=2 = a1 , 1 2 = ( 1=2) ( 2) = 1 =
a2
3) Si el nmero complejo = + i es una raz del polinomio Pn (x) (es decir Pn ( ) = 0) de
coecientes reales ( a0 , a1 , . . . , an 2 R), entonces el complejo conjugado = + i = i
tambin es una raz de Pn (x) (es decir, Pn ( ) = 0). Adems, el polinomio Pn (x) con coecientes
reales posee siempre races complejas conjugadas dos a dos.
As, por ejemplo, P2 (x) = x2 2x + 2 tiene por raz el valor = 1 + i, pues P2 (1 + i) =
2
(1 + i) 2 (1 + i) + 2 = 0, entonces la conjugada de , es decir, = 1 + i = 1 i, tambin es
2
raz, pues P2 (1 i) = (1 i) 2 (1 i) + 2 = 0:

Ahora presentamos un estudio ms detallado de los polinomios, sus operaciones, y nuevamente revisamos
algunas ecuaciones algebraicas.

2.3.1. Operaciones con polinomios y su descomposicin en factores

Se denomina polinomio de una variable con coecientes numricos a la suma nita


86 CAPTULO 2. EXPRESIONES RACIONALES E IRRACIONALES. POLINOMIOS

Pn = Pn (x) = an xn + an 1x
n 1
+ :::a2 x2 + a1 x + a0 ; (1)

donde x es la variable o indeterminada; a0 , a1 , a2 ,:::,an son los coecientes (siendo 0 xk = 0 y 1 xk = xk ) y


n 2 N es el grado del polinomio.

En particular, todo nmero es un polinomio de grado cero (P0 = const:), denominado constante. La variable
x, tambin se considera como un polinomio de grado uno.

La representacin (1) se denomina desarrollo del polinomio Pn segn las potencias crecientes o decrecientes
de la variable x:

La representacin (1) es nica, lo que signica que la igualdad

a0 + a1 x + a2 x2 + ::: = b0 + b1 x + b2 x2 + :::

(donde x es la variable y ai , bj son los coecientes) se cumple, si y solamente si a0 = b0 , a1 = b1 , a2 = b2 , :::


As mismo, la unicidad de la representacin (1) equivale a decir que a0 + a1 x + a2 x2 + ::: = 0, (a0 , a1 , a2 ,:::
son coecientes numricos), si y solamente, si a0 = a1 = a2 = ::: = 0. Aqu se debe entender la igualdad a cero
del polinomio en si y no de su valor en cierto punto de la indeterminada o variable x.

La representacin (1) consta de trminos. Si la representacin (1) consta de un solo trmino, entonces
tendremos un monomio (axk ; k 6= 0) de grado k, si consta de dos trminos, un binomio, etc.

Para los polinomios estn denidas las operaciones de suma, diferencia y producto con sus respectivas
propiedades:

Pn Qm = a0 + a1 x + a2 x2 + ::: + an xn b0 + b1 x + b2 x2 + ::: + bm xm
Pn Qm = a0 + a1 x + a2 x2 + ::: + an xn b0 + b1 x + b2 x2 + ::: + bm xm

Sea el polinomio P 6= 0; su grado deg P es la potencia ms alta de entre todos sus trminos en la repre-
sentacin (1). En nuestro caso deg P = n, siendo su coeciente an 6= 0. El coeciente a0 se denomina coeciente
independiente.

El polinomio nulo, 0 no tiene trmino mayor y su grado no est denido (a veces se considera que deg 0 = 1,
dndole al smbolo 1 propiedades denidas).

Propiedades del grado de un polinomio

1. deg(P Q) max (n; m).


El grado de la suma o diferencia de dos polinomios Pn y Qm no es mayor que el mayor de estos.
As, por ejemplo, deg 2x3 x + 1 + 2x3 + x 2 = 2, deg 2x3 x+1 2x3 + x2 2 = 1:

2. deg Pn Qm = deg Pn + deg Qm = n + m:


El grado del producto de dos polinomios Pn y Qm es igual a la suma de los grados de estos.
As, por ejemplo, deg 2x3 x+1 2x4 + x2 1 = 7:

Si P 6= 0, entonces el desarrollo de un polinomio como suma de trminos lo podemos expresar en forma


descendente:
Pn = an xn + an 1 xn 1 + ::: + a0 ;

donde an xn es el trmino mayor, an 6= 0 y n es el grado de P:


2.3. ESTUDIO POSTERIOR DE LOS POLINOMIOS DE UNA VARIABLE 87

El coeciente del trmino mayor del producto de dos polinomios Pn y Qm resulta ser el producto an bm de
los respectivos coecientes de los trminos mayores an y bm :

El valor del polinomio (1) en el punto x = c viene determinado por el nmero

Pn (c) = a0 + a1 c + a2 c2 + ::: + an cn

De la misma manera se dene el valor de un polinomio en un punto que es otro polinomio (composicin de
polinomios o polinomio de otro polinomio). As, por ejemplo, si P2 (x) = 2x2 x 1, entonces: Pn (3x2 ) =
2 2
2 3x2 3x2 1 = 18x4 3x2 1, Pn ( x + 3) = 2 ( x + 3) ( x + 3) 1 = 2x2 11x + 14, Pn (2x 5) =
2 2
2 (2x 5) (2x 5) 1 = 8x 42x + 54 tambin son polinomios de grados 2n = 4, n = 2 y n = 2,
respectivamente. Como la representacin (1) es nica, entonces el valor de Pn (c) est determinado unvocamente.

Para cualquier par de polinomios P y Q se tiene

(P Q)(c) = P (c) Q(c); (P Q)(c) = P (c)Q(c)

El concepto de valor de un polinomio P en un punto dado x = c, unvocamente determina una funcin


(funcin polinmica) que asigna a cada punto x = c el valor del polinomio P (c) en ese punto. Esta funcin
tambin se denomina polinomio y se la representa mediante Pn (x) o y = Pn (x), en este caso un nmero
cualquiera corresponde a una funcin constante, y polinomios distintos corresponden a funciones distintas.

Si para el polinomio Pn (x) se cumple que Pn (c) = 0, entonces el valor x = c, se denomina raz del polinomio
Pn : Si Pn es un polinomio con coecientes reales (ai 2 R; i = 0; : : : ; n) y x = c es una raz imaginaria del
polinomio Pn , entonces el valor x = c (el conjugado de c), tambin es una raz de Pn : As, por ejemplo,
p es muy
1 3
fcil comprobar que el polinomio de segundo grado P2 (x) = x2 + x + 1 tiene por races c = y su
p 2 2
1 3
conjugada c = + . Lo mismo sucede con el polinomio Q2 (x) = 2x2 2x + 5 que tiene por races a
2 2
1 3 1 3
c = + i y su conjugada c = i:
2 2 2 2
El anterior resultado es una consecuencia de uno de los adelantos ms grandiosos de toda la matemtica, el
as llamado teorema fundamental del lgebra, que dice: todo polinomio con cualesquiera coecientes numricos
(reales o complejos), cuyo grado no sea menor a la unidad, tiene por lo menos una raz, generalmente compleja.

Otra consecuencia de este famoso teorema es el teorema: todo polinomio Pn (x) de grado n 1, de cua-
lesquiera coecientes numricos, tiene n races, incluyendo sus races mltiples.

Si P y T son dos polinomios, siendo T 6= 0, entonces existen los polinomios q y r, tales que:

1. P = T q + r;

2. o bien r = 0, o bien deg r < deg T:

Las 2 condiciones anteriores determinan al par de polinomios q y r unvocamente. El polinomio q se denomina


cociente y el polinomio r residuo de dividir P por T:

Es evidente que si los coecientes de los polinomios P y T son reales (por ej. racionales), entonces los
coecientes de q y r, tambin son del mismo tipo. Dados los polinomios P y T , la bsqueda de los polinomios
q y r se la hace utilizando el algoritmo de la divisin con resto (mtodo de la escalera).

Vamos a decir que el polinomio P es divisible por el polinomio T , o lo que es lo mismo, que el polinomio T
divide al polinomio P (lo que representaremos mediante P j T ), si existe un polinomio q, tal que P = T q. Es
evidente que cuando el polinomio P es divisible por el polinomio T , el resto es nulo, es decir r = 0. As, por
88 CAPTULO 2. EXPRESIONES RACIONALES E IRRACIONALES. POLINOMIOS

ejemplo, el polinomio P3 (x) = 2x3 3x2 + 5x 2 es divisible por el polinomio T2 (x) = x2 x + 2, pues se
tiene que 8
>
< P3 = T2 q1
q1 (x) = 2x 1
>
:
r=0

2.3.2. Propiedades de la divisibilidad de polinomios

Para indicar que cierto polinomio P es divisible por otro h utilizaremos la simbologa P j h :

1) Si P j h y T j h, entonces (P T ) j h:
2) Si P j T , entonces para cualquier polinomio h, se tiene hP j T:
3) El polinomio nulo, 0 es divisible por cualquier polinomio no nulo.
4) Si P j T y T j P , entonces P = aT , siendo a una constante, a 6= 0:
5) Si P j T , P 6= 0, entonces deg P deg T:

| Un caso particular muy importante es cuando se analiza la divisivilidad del polinomio Pn (x) por el
binomio x c, donde c es un nmero cualquiera:

Si Pn (x) = a0 xn + a1 xn 1
+ : : : + an 1x + an dividimos por T1 (x) = x c, entonces

Pn = T1 qn 1 + r ) Pn (x) = (x c) qn 1 (x) + r

Ahora, colocando en la igualdad anterior el valor x = c, tendremos que

Pn (c) = (c c) qn 1 (c) + r ) Pn (c) = r

Llegamos a los siguientes resultados:

Teorema (Bezout). El resto r de dividir el polinomio Pn (x) por el binomio T1 (x) = x c es igual a
Pn (c), es decir r = Pn (c) :

Teorema (Descartes). El polinomio Pn (x) es divisible por el binomio T1 (x) = x c, si y solamente, si


c es raz del polinomio Pn (x), es decir, cuando r = Pn (c) = 0:

Ejemplo:

Sea el polinomio P4 (x) = 2x4 5x3 x2 5x 3 que al dividirlo por el binomio T1 (x) = x + 2 nos da
como resto
4 3 2
r = P4 ( 2) = 2 ( 2) 5 ( 2) ( 2) 5 ( 2) 3 = 75
1
Al dividirlo por T1 (x) = x , tendremos como resto
3
4 3 2
1 1 1 1 1 400
r = P4 =2 5 5 3=
3 3 3 3 3 81
Al dividirlo por T1 (x) = x 3, nos da como resto
4 3 2
r = P4 (3) = 2 (3) 5 (3) (3) 5 (3) 3 = 0;

lo que nos indica que el valor x = 3 es una raz de P4 (x) :


1
Al dividirlo por T1 (x) = x + , tendremos como resto
2
4 3 2
1 1 1 1 1
r = P4 =2 5 5 3 = 0;
2 2 2 2 2
2.3. ESTUDIO POSTERIOR DE LOS POLINOMIOS DE UNA VARIABLE 89

1
lo que signica que el valor x = es otra raz de P4 (x) :
2
| La divisin con resto del polinomio Pn (x) = a0 xn + a1 xn 1 + ::: + an por el binomio T1 (x) = x c puede
ser realizada con ayuda del esquema de Horner o regla de Ru ni, este tiene la ventaja de darnos, aparte del
resto r, tambin el cociente qn 1 de dicha divisin:

a0 a1 a2 ::: an 1 an c
b0 b1 b2 ::: bn 1 bn
# # # ::: # #
a0 a0 c + a1 b1 c + a2 ::: bn 2 c + an 1 bn 1 c + an

En el esquema anterior tenemos en la primera la los n coecientes del polinomio dado Pn (en orden de potencias
descendentes) y el coeciente c de T1 . En la segunda la tenemos los n 1 coecientes del polinomio cociente
qn 1 (tambin en orden de potencias descendentes) y el coeciente bn . En la tercera la tenemos los smbolos
de igual (=) para indicar como se calculan los coecientes b0 , b1 ,. . . , bn a travs de sus respectivas relaciones
dadas en la cuarta la.

Los valores b0 ; b1 ; :::; bn 1; representan los coecientes del polinomio cociente qn 1, siendo bn el resto, es
decir
Pn = T1 qn 1 + r = (x c)(b0 xn 1
+ b1 xn 2
+ ::: + bn 1) + bn

Ilustramos el esquema de Horner mediante ejemplos.

Ejemplos:

1. Dados los polinomios P4 (x) = 5x4 3x3 x + 3 y T1 (x) = x 2, hallar el polinomio cociente q3 (x) y el
resto r de dividir P4 por T1 :
Solucin:
5 3 0 1 3 2
b0 b1 b2 b3 b4
# # # # #
5 5 (2) + ( 3) (7) (2) + (0) (14) (2) + ( 1) (27) (2) + (3)
Segn el esquema de Horner tendremos que

b0 = 5; b1 = 5(2) + ( 3)= 7; b2 = (7) (2) + (0) = 14; b3 = (14) (2) + ( 1) = 27; b4 = r = (27) (2) + (3) = 57;

por lo tanto, el cociente y el resto sern:

q3 (x) = 5x3 + 7x2 + 14x + 27;


r = 57

2. Dados los polinomios P5 (x) = 4x5 5x + 3 y T1 (x) = x + 1, hallar el polinomio cociente q4 (x) y el resto
r de dividir P5 por T1 :
Solucin:
4 0 0 0 5 3 1
4 ( 1) 4 ( 1) 4 ( 1) 4 ( 1) 1
4 4 4 4 1 4

El cociente y el resto sern: q4 (x) = 4x4 4x3 + 4x2 4x 1, r = 4

3. Dados los polinomios P5 (x) = 4x5 x3 2x2 + 5x 3 y T1 (x) = 2x + 3, hallar el polinomio cociente
q4 (x) y el resto r de dividir P5 por T1 :
90 CAPTULO 2. EXPRESIONES RACIONALES E IRRACIONALES. POLINOMIOS

Solucin:
3
4 0 1 2 5 3 2
3 3 3 3 3
4 2 +0 6 2 1 8 2 2 14 2 +5 26 2 3
4 6 8 14 26 42

Observacin: Ntese que en este caso, el divisor tiene la forma T10 = ax + b y que para aplicar el esquema
b
de Horner vamos a dividir el binomio T10 = ax + b por a, obteniendo como nuevo divisor T1 = x + . Siendo as,
a
b
el resto r ser igual a r = Pn y el polinomio cociente tendr los coecientes correspondientes a la ltima
a
la del esquema de Horner, pero tambin divididos por a. Para nuestro ejemplo tendremos

b0 = 42 = 2; b1 = 6
2 = 3; b2 = 82 = 4; b3 = 14
2 = 7; b4 = 26
2 = 13; r = 42;

por lo tanto, el cociente y el resto sern: q4 (x) = 2x4 3x3 + 4x2 7x + 13, r = 42.

Recuerda sobre los polinomios:

| Todo polinomio Pn 6= 0 puede ser expresado como producto de factores lineales con coecientes complejos,
es decir representado en la forma Pn (x) = a0 (x 1 )(x 2 ):::(x n ), donde, a0 es el coeciente del trmino
mayor de Pn ; 1 , 2 , :::, n son sus races (no necesariamente distintas) y cualquiera de las races de Pn coincide
con uno de los valores de i : La representacin anterior es nica (excluyendo el orden de los factores lineales
x i ).

Si en la representacin en factores lineales anterior unimos los factores lineales iguales, entonces tendremos el
k1 k2 km
desarrollo en factores: Pn (x) = a0 (x 1 ) (x 2 ) :::(x m) , donde 1 , 2 , :::, m son races distintas
del polinomio Pn :

Todo polinomio Pn 6= 0 con coecientes reales (ai 2 R; i = 0; 1; : : : ; n) puede ser desarrollado como producto
de factores lineales y/o de segundo grado con coecientes reales:
k1 k2 km
Pn (x) = a0 (x 1 ) (x 2 ) :::(x m) (x2 + p1 x + q1 )l1 :::(x2 + ps x + qs )ls

(puede ser m = 0 o s = 0), donde p2i 4qi < 0; i = 1, 2, : : :, s. La expresin 4 = p2i 4qi , denominada
discriminante del polinomio cuadrtico x2 +pi x+qi proviene de la bsqueda de races de la ecuacin x2 +pi x+qi =
0:
pi 2 pi 2
x2 + pi x + qi = 0 ) x2 + pi x = qi ) x2 + pi x + = qi )
| {z 2 } 2
com pletam iento del cuadrado
r r
pi 2 pi 2 pi pi 2 pi pi 2
x+ = qi ) x + = qi ) x = qi )
2 2 2 2 2 2
r p
pi p2i 4qi pi p2i 4qi
x = )x= ;
2 4 2
p p
pi p2i 4qi pi + p2i 4qi
de donde se obtienen 2 races x1 = y x2 = y se concluye que si el discrim-
2 2
2
inante 4 = pi 4qi 0, entonces las anteriores 2 races son reales, y si 4 = p2i 4qi < 0, las 2 races son
imaginarias y conjugadas. De lo anterior tenemos la descomposicin en factores:
p ! p !
p p 2 4q p + p 2 4q
i i i i
x2 + pi x + qi = x i
x i
= (x x1 ) (x x2 ) :
2 2

Continuando con las caractersticas del polinomio general Pn , tenemos adems que:

1. a0 es el coeciente del trmino mayor del polinomio Pn


2.3. ESTUDIO POSTERIOR DE LOS POLINOMIOS DE UNA VARIABLE 91

2. 1; 2; : : : ; m son todas las races reales del polinomio Pn , m n:

3. Cada uno de los polinomios x2 + pi x + qi con pi , qi 2 R, no es factorable como producto de factores


lineales con coecientes reales. Nos adelantamos diciendo que si para cualquier polinomio cuadrtico
P2 (x) = x2 + pi x + qi se tiene que p2i 4qi < 0 (es decir, su discrimante p2i 4qi es negativo), entonces
cada polinomio x2 + pi x + qi posee 2 races complejas desiguales (conjugadas) que tambin son races del
polinomio Pn y en este caso x2 + pi x + qi no puede ser representado como producto de factores lineales
(de primer grado) con coecientes reales.

Ejemplos:

Realizar las operaciones indicadas:

1. P = (3 2x + 5x2 + 4x4 ) + (x x2 x3 ):

Solucin: Agrupando tendremos P = 3+( 2x+x)+(5x2 x2 ) x3 +4x4 = P4 (x) = 3 x+4x2 x3 +4x4

2. P = (2 5x + 3x2 + x3 )(3 + x + 2x3 + x4 ):

Solucin: Los trminos de grado k se obtienen al multiplicar los trminos de grado cero con los de grado
k, etc.

3
P = 2 3+(2 1 5 3)x + (2 0 5 1 + 3 3)x2 +(2 2 5 0 + 3 1 + 1 3)x +
4 5 6
+(2 1 5 2 + 3 0 + 1 1)x +( 5 1 + 3 2 + 1 0)x + + (3 1 + 1 2)x +1 1x7

P7 = 6 13x + 4x2 + 10x3 7x4 + x5 + 5x6 + x7

3. P = (x a)(xn 1
+ axn 2
+ a2 xn 3
+ ::: + an 2
x + an 1
)

Solucin: P = xn + (a a) xn 1
+ a2 a2 xn 2
+ + an 1
an 1
x an = xn an

Observacin: Es evidente que el producto de polinomios, a veces, resulta ms conveniente cuando


utilizamos el esquema de la escalera. En sta colocamos en calidad de multiplicando al polinomio con ms
trminos, y en calidad de multiplicador al polinomio con menos trminos.

4. Hallar el producto de los polinomios P4 = 2x4 x2 + x 1 y Q2 = x2 x + 1:

Solucin: Tenemos
2x4 0 x2 +x 1
x2 x +1
2x6 0 x4 +x3 x2
2x5 0 +x3 x2 +x
2x4 0 x2 +x 1
6 5
2x 2x +x4 2x3 3x2 2x 1

El polinomio producto ser: P4 Q2 = T6 = 2x6 2x5 + x4 + 2x3 + 2x 1.

5. Sean P = 2x5 x4 + 23x2 15x + 21, T = x2 + 2x 3. Dividir con resto r el polinomio P por el polinomio
T:
92 CAPTULO 2. EXPRESIONES RACIONALES E IRRACIONALES. POLINOMIOS

Solucin:
2x5 x4 0 +23x2 15x +21 x2 +2x 3
2x5 4x4 6x 3
2x3 5x2 +16x 24
5x4 +6x3 +23x2
+5x4 +10x3 15x2
+16x3 +8x2 15x
16x3 32x2 +48x
24x2 +33x +21
+24x2 +48x 72
81x 51

El polinomio cociente P (con resto r) y el polinomio residuo, seran:

Q3 = 2x3 5x2 + 16x 24, r1 = 81x 51


P5 r1
La anterior divisin equivale a la representacin: = Q3 + o P5 = T2 Q3 + r1 , es decir
T2 T2
P5 = x2 + 2x 3 (2x3 5x2 + 16x 24) + (81x 51):
| {z }| {z } | {z }
T2 Q3 r1

6. Hallar el cociente y resto de dividir P = 2x5 3x4 11x3 + 12x + 5 para T = x 3.


Solucin:
2 3 11 0 12 5 3
6 9 6 18 18
2 3 2 6 6 13

La anterior divisin equivale a la representacin P5 = T1 (2x4 + 3x3 2x2 6x 6) 13 y entonces


P5 r
= Q4 + o P5 = T1 Q4 + r;
T1 T1
es decir,
P5 = (x 3)(2x4 + 3x3 2x2 6x 6) 13:
| {z } | {z } |{z}
T2 Q3 r

7. Si Pn (x) = (1 + x)n + (1 x)n y T2 (x) = x2 1, entonces hallar el cociente y resto de dividir P para T .
Solucin: En este problema no nos sirve ni utilizar el esquema de la escalera, ni el esquema de Horner.
Nos conviene ms utilizar el teorema del residuo de Bezout:
Tenemos que
deg T = 2, Pn = T2 qn 2 + (a + bx);
| {z }
r1

por lo tanto, para resolver el problema, debemos hallar a y b. Esto es posible evaluando al polinomio
Pn (x) en los puntos x = 1 y x = 1
(
Pn (1) = T2 (1)qn 2 (1) + a + b;
;
Pn ( 1) = T2 ( 1)qn 2 ( 1) + a b

es decir, obtenemos el sistema lineal:


( (
2n = 0 q(1) + a + b a + b = 2n
) ;
2n = 0 q( 1) + a b a b = 2n

de donde
a = 2n ; b = 0; r = a + bx = 2n ; Pn = T2 qn 2 + 2n
2.3. ESTUDIO POSTERIOR DE LOS POLINOMIOS DE UNA VARIABLE 93

Por lo tanto:
Pn 2n (1 + x)n + (1 x)n 2n
qn 2 = = ;
T2 x2 1
(1 + x)n + (1 x)n 2n
Pn = T2 + 2n
x2 1

8. Cierto polinomio P al dividirlo por x2 + x + 1 y por x 1 nos da por resto 2x 3 y 5, respectivamente.


Hallar el resto de dividir el polinomio P por x3 1:
Solucin: Las condiciones del problema nos conducen a las 2 relaciones:

P = (x2 + x + 1)q1 + 2x 3; P = (x 1)q2 + 5

El resto buscado r (deg r < 3) tendr la forma r = a + bx + cx2 . Entonces

P = (x3 1)q + r = (x 1)(x2 + x + 1)q + a + bx + cx2 :

Transformando esta expresin, mediante 2 mtodos, obtendremos:


8
>
> P = (x2 + x + 1)(x 1)q + (x2 + x + 1)c + (b c)x + a c
>
< = (x2 + x + 1)[:::] + (b c)x + a c;
>
> P = (x 1)(x2 + x + 1)q + (x 1)(cx + b + c) + a + b + c
>
:
= (x 1)[:::] + a + b + c

En otras palabras, hemos dividido el polinomio P por x2 + x + 1 y por x 1, respectivamente. Las


expresiones entre corchetes no son necesarias y no las ponemos. Las anteriores igualdades nos indican que
(b c)x+a c y a+b+c son los restos de dividir el polinomio P por x2 +x+1 y por x 1, respectivamente.
Por tanto, de acuerdo a las condiciones del problema, tendremos que
(
(b c)x + a c = 2x 3
;
a+b+c=5

que por el mtodo de los coecientes indeterminados, nos da:


8 8
>
< b c = 2; >
< a = 1;
a c = 3; ) b = 4;
>
: >
:
a + b + c = 5; c=2

El resto buscado r ser r = cx2 + bx + a = 2x2 + 4x 1:


Otra solucin sera partiendo de las condiciones del problema
8
> 2
< P = (x + x + 1)q1 + 2x 3;
P = (x 1)q2 + 5;
>
:
P = (x3 1)q + a + bx + cx2
p
2 1 3
donde representando con "j = "1;2 a las races del polinomio x + x + 1, es decir, "1;2 = i ,y
2 2
tomando en cuenta que se cumplen las relaciones: "21 = "2 , "22 = "1 , "31;2 = 1. Entonces

P ("j ) = 0 q1 ("j ) + 2"j 3 = 2"j 3 y P (1) = 0 q2 (1) + 5 = 5

Por otro lado, de la 3ra condicin del problema, tendremos para P :

P ("j ) = 0 q("j ) + a + b"j + c"j y P (1) = 0 q(1) + a + b + c;

es decir 8
> 2
< 1) a + b"1 + c"1 = 2"1 3;
2) a + b"2 + c"22 = 2"2 3;
>
:
3) a + b + c = 5:
94 CAPTULO 2. EXPRESIONES RACIONALES E IRRACIONALES. POLINOMIOS

Para resolver el sistema lineal anterior y hallar a, b, c, primero sumamos las 3 ecuaciones, y obtenemos
3a = 3, de donde a = 1. Luego b + c = 5 a = 6, es decir b = 6 c. Colocamos el valor de b en la 1ra
ecuacin: 1 + (6 c)"1 + c"2 = 2"1 3, es decir,
p p
c("2 "1 ) = 2 4"1 , o i 3c = 2i 3. De donde c = 2 y b = 4. Por lo tanto

r = cx2 + bx + a = 2x2 + 4x 1:

9. Calcular P (c), si P = 3x6 + 5x5 + 2x4 + 6x3 3x2 + 7x + 5 y c = 2:


Solucin: En este caso es ms cmodo utilizar el esquema de Horner (por el teorema de Bezout P (c) = r),
es decir dividir P por el binomio x c = x + 2:

3 5 2 6 3 7 5 2
6 2 8 4 2 10
3 1 4 2 1 5 5
|{z}
r=P( 2)

Por lo tanto P ( 2) = 5. Es evidente que el cociente (con resto) de dividir P6 = 3x6 + 5x5 + 2x4 + 6x3
3x2 + 7x + 5 por T1 = x + 2 es igual a q5 = 3x5 x4 + 4x3 2x2 + x + 5, siendo el resto r = P ( 2) = 5.

10. Hallar P (c), si P = (x 1)10 + (x + 1)4 y c = 2:


Solucin: Para este caso, el esquema de Horner no es aplicable, pues P no est desarrollado en potencias
de x. Haciendo el reemplazo directo, tendremos: P (2) = (2 1)10 + (2 + 1)4 = 1 + 34 = 82:

11. Hallar P (c), si P = x5 + ax4 (a2 + b3 )x2 ab3 x + 2a2 b2 y c = b:


Solucin: Aqu hacemos el reemplazo directo:

5 4 2
P (b) = (b) + a (b) (a2 + b3 ) (b) ab3 (b) + 2a2 b2 =
= b5 + ab4 a2 b2 b5 ab4 + 2a2 b2 = a2 b2 :

12. Descomponer en factores el polinomio P = 3x4 23x2 36.


Solucin: Para hallar las races y factores del polinomio P hacemos x2 = y, entonces

3y 2 23y 36 = 0 ) (3y + 4) (y 9) = 0
r
2 4 4 2i p
) y1 = x = ) x1;2 = = p ; y2 = x2 = 9 ) x3;4 = 9= 3
3 3 3
Como el coeciente de la mayor potencia es 3, entonces la descomposicin en factores lineales sera:

2i 2i
P4 = 3(x 3)(x + 3)(x p )(x + p )
3 3

13. Descomponer x4 6x3 + 9x2 16 en factores lineales reales.


Solucin: Tenemos

P = x4 6x3 + 9x2 16 = (x2 3x)2 16 = (x2 3x 4)(x2 3x + 4)

Los factores cuadrticos tienen por discriminantes: 41 = 25 y 42 = 7, respectivamente. Por lo tanto,


el polinomio P puede descomponerse en 3 factores, 2 factores lineales reales y uno cuadrtico:

P = x4 6x3 + 9x2 16 = (x + 1)(x 4)(x2 3x + 4)


2.3. ESTUDIO POSTERIOR DE LOS POLINOMIOS DE UNA VARIABLE 95

14. Hallar los valores de a y b para que el polinomio P sea divisible por el polinomio T

P = (x + 1)n + axn + b, T = x2 1:

Solucin: Tenemos T = (x 1)(x + 1). Para que el polinomio P se divida por T , necesitamos que
P j (x 1) y P j (x + 1). Segn el teorema de Descartes, esto equivale a P (1) = P ( 1) = 0, es decir
2n + a + b = a( 1)n + b = 0. Para n par, esto es imposible. Para n impar tendremos a = b = 2n 1 .

15. Hallar los valores reales de a y b para que el polinomio

P = a(x5 2x4 2x3 + 7x2 + 5x 1) + b(x3 3x2 + 6x 2) 2

sea divisible por T = x2 4x + 5.


p
Solucin: Las races de T son x1;2 = 2 4 5 = 2 i y por tanto T = (x 2 i)(x 2 + i). Para
que P sea divisible por T es necesario y suciente que x1;2 sean races de P . Como los coecientes de P
__
son reales y x2 = x1 , entonces es suciente que P (x1 ) = 0 (entonces y P (x2 ) = 0). Para calcular P (x1 )
utilizamos el esquema de Horner para cada uno de los polinomios dados entre parntesis:
1 2 2 7 5 1 2+i 1 3 6 2 2+i
2+i 2i 1 8+i 3+i 3 + 4i , 2+i 3+i 5 + 5i
1 i 3 + 2i 1+i 2+i 2 + 4i 1 1+i 3+i 3 + 5i
Tenemos que P (2 + i) = a(2 + 4i) + b(3 + 5i) 2 = 0, de donde
( (
2a + 3b = 2 a = 5;
) , pues a; b 2 R:
4a + 5b = 0 b = 4:

Figura 2.4: Izq. El gran matemtico, fsico y genial astrnomo francs, Pierre Simn Laplace (Beaumont en Auge,
1749 - Pars, 1827), especialista en probabilidades y estadstica, ecuaciones y mtodos de la fsica - matemtica y sus
aplicaciones. Demostr la estabilidad del sistema solar y explic su origen. Fue ministro del Interior durante el Consulado
de Napolen. Der. El famoso lsofo, jurista, historiador, bibliotecario, cientco e inventor (prensas, molinos de viento,
bombas de agua, relojes, calculadoras, lmparas, submarino), gelogo, matemtico y lgico, el alemn Gottfried Leibniz
(Leipzig, 1646 - Hannover, 1716). Uno de los creadores e inventores del Clculo diferencial e integral junto con Newton
(1642 - 1727), fundador de la topologa, creador del sistema binario de numeracin. Su losofa se basa en el concepto
de mnada, una fuerza o entidad innitesimal, espiritual, indestructible y dinmica, sincronizadas desde su creacin por
Dios, lo que explica la armona del universo.

Practico lo que aprend:

Factorar como diferencia de cuadrados, cubos, etc., o desarrollar las potencias y luego simpli-
car:

1. a) (x2 2x + 1)2 (x2 + 2x 1)2 , b) (2x3 2x + 1)2 (2x3 + 2x 1)2


96 CAPTULO 2. EXPRESIONES RACIONALES E IRRACIONALES. POLINOMIOS

2. a) (x2 2x + 1)3 (x2 + 2x 1)3 , b) (3x3 2x + 1)3 (3x3 + 2x 1)3

3. a) (x3 1)5 (x3 + 2)5 , b) (x2 2x)3 (x2 + 3x)3 , c) (x2 x + 1)4 (x2 + x + 1)4
Simplicar las expresiones:

4. a) (2x3 x2 + 3)(2x3 + x2 3), b) (3x4 + x2 x + 1)(3x4 x2 + x 1)

5. a) (x4 + x3 x 1)(x6 + x3 + x + 1), b) (2x3 x2 + x 2)(2x3 + x2 x + 2)2)

6. (x3 x 1)(x3 + x + 1) (x2 2x + 1)(x2 + 2x 1) + (x2 + x 1)(x2 x + 1)


Dados los polinomios P , Q. Hallar el cociente y el resto de dividir P por Q:

7. a) P = 3x5 + 5x3 + 3x2 + x 9, Q = x2 x + 3; b) P = x12 1, Q = x + 1

8. a) P = x6 2x5 + 5x3 5x2 + 3, Q = x3 2x + 2; b) P = x15 + 1, Q = x 1

9. a) P = x3 + x2 x + 1, Q = 3x2 2x + 1; b) P = x4 + x3 , Q = x i

10. a) P = x20 + x10 + 1, Q = x3 1; b) P = x3 + 5x 4, Q = 2x 4

11. a) P = 4x5 x4 3x2 4, Q = 2x + 3; b) P = 2x5 + 3x2 4, Q = 2x 3

12. Cierto polinomio P al dividirlo por x a, x b, x c nos da como resto a, b, c, respectivamente. Hallar
el resto de dividir P por (x a)(x b)(x c):
Dado el polinomio P , hallar el valor de P (c) :

13. P = 3x4 x2 5x + 2, c = 3; b) P = x4 x2 , c = 1 i:

14. P = 2x5 x3 x + 6, c = 2; b) P = x4 x2 + 2, c = 3i

15. P = x4 + 2x2 x + 4, c = 2 + i; b) P = x8 x4 , c = 1 + i:
Realizar las siguientes operaciones con polinomios y fracciones racionales:

16. Realizando las operaciones indicadas y luego igualando los coecientes de la parte izquierda con los de la
derecha (mtodo de los coecientes indeterminados), hallar los valores de A, B, C, D, E de tal manera
que se cumplan las siguientes igualdades:

a) x4 + x3 x2 + 1 = (x + 1) x3 + Ax2 + Bx + C
b) x5 + x2 x 1 = x2 1 x3 + Ax2 + Bx + C
2x2 2x 6 A B C
c) 3 2
= + +
x + 2x x 2 x+1 x 1 x+2
x2 1 A B C
d) = + +
x3 + 5x2 + 3x 9 x 1 x + 3 (x + 3)2
2x + 1 A B Cx + D
e) = + + 2
x4 1 x+1 x 1 x +1

17. Hallar el cociente Q (x) y expresar los polinomios P (x) y T (x) en la forma

P (x) R (x)
= Q (x) + , deg R (x) < deg T (x)
T (x) T (x)

a) P (x) = 2x3 2x + 3, T (x) = 2x + 1; b) P (x) = 4x3 + 3x2 x + 1, T (x) = 2x 3; c) P (x) =


x4 + 2x3 3x2 + 5x 3, T (x) = x2 x + 1; d) P (x) = 6x5 x3 + 4x2 + 6x + 1, T (x) = 3x3 + x2 x 1:

18. Utilizando el esquema de Horner, hallar el cociente y el resto de dividir P (x) por T (x) :
a) P (x) = 2x3 + x2 3x + 2, T (x) = x 2; b) P (x) = 4x3 + 3x2 x + 1, T (x) = 2x 3; c)
P (x) = 3x3 3x2 +x 3, T (x) = 3x+2; e) P (x) = 4 (x + 5) (x + 6) (x + 10) (x + 12) 3x2 , T (x) = x+1;
f ) P (x) = x12 3x4 + x + 1, T (x) = 4x 2:
2.3. ESTUDIO POSTERIOR DE LOS POLINOMIOS DE UNA VARIABLE 97

Figura 2.5: Izq. El matemtico espaol Julio Rey Pastor (Logroo, 1888 - Buenos Aires,1962), discpulo del gran
matemtico, gometra y padagogo Felix Klein (Dsseldorf, 1849 - Gotinga, 1925), fundador del Instituto de Matemtica
de Buenos Aires. Introdujo y divulg en Espaa y en Argentina las matemticas modernas. Conocido especialista en
teora de las funciones. Der. El conocido matemtico alemn Herman Weyl (Elmschorn, Hamburgo, 1885 - Princeton,
1955), gran especialista en series trigonomtricas, variable compleja, ecuaciones diferenciales e integrales y poderosas
aplicaciones de los grupos continuos a la geometra y fsica. Hizo muchas contribuciones a la geometra diferencial y al
estudio de las supercies de Riemann. Estudioso de la losofa de las matemticas (intuicionismo), lgica y de la teora
de la relatividad. Fue el primer lector sobre lecciones sobre la teora general de la relatividad. Fracas en su deseo de
construr una teora nica o unicada del campo.

19. Con el esquema de Horner averiguar si P (x) es divisible por T (x). Hallar el cociente respectivo:
a) P (x) = x3 + 2x2 x 2, T (x) = x2 1; b) P (x) = x3 + 2x2 x 2, T (x) = x2 + 3x + 2;
c) P (x) = x4 5x2 + 4, T (x) = x2 x 2; d) P (x) = x4 5x2 + 4, T (x) = x2 3x + 2; e)
3 2
P (x) = 6x4 + 4x2 + x3 + x 2, T (x) = 6x2 + x 2; f ) P (x) = x2 + x 2 3 x2 + x 2 + 2x3 2,
2 2
T (x) = x 1; g) P (x) = xn + 32, T (x) = x + 2, n =?; h) P (x) = 2x4 + x x3 1 + 98 ,
T (x) = 2x + 1:

20. Hallar los valores de a y b para que el polinomio P (x) sea divisible por T (x):
a) P (x) = x3 2ax2 + 3x b, T (x) = x2 + x 2; b) P (x) = x4 + ax3 2bx + 3, T (x) = x2 x 2; c)
2
P (x) = x3 +ax2 +(2b 1) x 1, T (x) = x2 +x 6; d) P (x) = x4 10x2 + 16 b x4 10x2 + 18 +2a,
2 2 2 2 2
T (x) = x 2; e) P (x) = x 5x + 6 5 x 5x + b + 6a, T (x) = x 1:

21. Con el esquema de Horner - Ru ni, descomponer en factores e indicar las races de los polinomios:
a) P (x) = x3 3x2 4x + 12, b) P (x) = 4x3 16x2 x + 4, c) P (x) = x5 3x4 x3 + 3x2 , d)
P (x) = 6x3 10x2 8x + 8, e) P (x) = x4 + 5x3 + 2x2 20x 24, f ) P (x) = 12x3 4x2 3x + 1

22. Usando el esquema de Horner-Ru ni, hallar las races racionales de las ecuaciones :
a) 2x3 11x2 + 17x + 6 = 0, b) 3x3 2x2 27x + 18 = 0, c) 2x5 x4 32x + 16 = 0, d) 6x3
17x2 31x + 12 = 0, e) 24x3 118x2 + 92x 16 = 0, f ) 720 + 144x 125x2 25x3 + 5x4 + x5 = 0, g)
36x4 204x3 +349x2 170x+25 = 0, h) 6x4 5x3 175x2 +125x+625 = 0, i) 12x3 44x2 + 51x 18 = 0:

23. Hallar las condiciones para que el polinomio P sea divisible por Q:
a) P = x3m x3n+1 + x3p+2 , Q = x2 x + 1; m, n, p 2 N: b) P = x3m + x3n+1 + x3p+2 , Q = x4 + x2 + 1;
m, n, p 2 N: c) P = x2n + xn + 1, Q = x2 + x + 1; n 2 N: d) P = (x + 1)n xn 1, Q = x2 + x + 1;
n 2 N: e) P = (x + 1)n + xn + 1, Q = x2 + x + 1; n 2 N: f ) P = x30 + ax17 + b, Q = x3 1:

24. Si P (xn ) es divisible por x 1. Demostrar que P (x) es divisible por x 1:


98 CAPTULO 2. EXPRESIONES RACIONALES E IRRACIONALES. POLINOMIOS

2.4. Transformacin y Simplicacin de Expresiones Irracionales

Aquellas expresiones algebraicas en las que hay extraccin de races de por lo menos una variable o bien la
elevacin de sta a potencia racional fracccionaria, llevan el nombre de irracionales con relacin a dicha variable.

Recordemos la denicin de raz aritmtica. Si a 0 y n 2 N, n > 1, slo hay un nmero no negativo x tal
que se cumple la igualdad xn = a. Este nmero x se denomina raz aritmtica de ndice n (o raz n-sima) y es
p
la potencia del nmero no negativo a, el mismo que se designa mediante n a:
p p
De lo dicho se desprende que la igualdad 2 49 = 7 es cierta, en tanto que las igualdades 2 49 = 7 o bien
p2
49 = 7 son falsas de acuerdo al concepto de raz aritmtica de ndice n:
p
Si n es un nmero impar mayor que 1 y a < 0, por n ase entiende semejante nmero negativo x, tal que
xn = a.
p p p
| Si n, k, m 2 N, a 0 y b 0, entonces: n ab = n a n b. Esta propiedad se propaga al producto de
p p p p
cualquier nmero de factores, por ejemplo, 3 8;27;125 = 3 8 3 27 3 125 = 2 3 5 = 30:
r p
a n
a
| Si n 2 N, a 0 y b > 0, entonces: n = p n
.
b b
Recuerda las siguientes reglas de la radicacin:

r p
p
n
p p a n
jaj
Si a < 0 y b < 0 las 2 propiedades anteriores toman la forma: ab = n jaj n jbj, n
= p
b n
jbj
p p
n
p
5
p p5
( n a)k = ak , por ejemplo, ( a2 )3 = 5 (a2 )3 = a6
pn k
p p pp p
a = nk a, por ejemplo, 4 3 a = 12 a
p
mn
pn
p6
p3
p
15 p
amk = ak , por ejemplo, a4 = a2 , a3 = 5 a:

Observaciones: Si los ndices de las races son nmeros impares, entonces las propiedades anteriores tambin
se cumplen para a < 0, b < 0 y para ab < 0. Recordemos otra importante propiedad de qlapraz aritmtica: si n es
p
2k 2 2 p2
un nmero par, es decir, n = 2k, tiene lugar la identidad 2k
a = jaj, por ejemplo, 2
3 2 = j 3 2j =
p2
2 3:

Recuerda las siguientes deniciones y propiedades con los radicales:

Deniciones de potencias con exponentes racionales:

1) Si a es diferente de 0, entonces a0 = 1:
m p
2) Si a 0, a n = n am (n, m son nmeros naturales, n 2).
3) Si a > 0, m 2 Z, a m = 1=am :

Propiedades fundamentales de las potencias con exponentes arbitrarios racionales:

1. ar :as = ar+s

2. (ar )s = ars

3. (ab)r = ar br
a r ar
4. =
b br
2.4. TRANSFORMACIN Y SIMPLIFICACIN DE EXPRESIONES IRRACIONALES 99

ar
5. = ar s ,
as
donde a > 0, b > 0, r y s son nmeros racionales arbitrarios.

Ejemplos:

p p p p p p
1. Simplicar la expresin A = ( 2 32 + 2 45 2
98)( 2 72 2
500 2
8):
Solucin: Primero simpliquemos cada uno de los radicales dados:
p
2
p p p p p p p p p p p
32 = 2 16 2 = 4 2 2, 2
45 = 2 9 5 = 3 2 5, 2 98 = 2 49 2 = 7 2 2, 2
72 = 2
36 2 = 6 2 2,
p
2
p p p p p
500 = 2 100 5 = 10 2 5, 2 8 = 2 4 2 = 2 2 2:
Despus de esto la expresin prejada toma la forma:
p p p p p p p p p p
A = (4 2 2 + 3 2 5 7 2 2)(6 2 2 10 2 5 2 2 2) = (3 2 5 3 2 2)(4 2 2 10 2 5).
Despus, obtenemos:
p p p p
A = 12 2 10 24 150 + 30 2 10 = 42 2 10 174 = 6(7 2 10 29).

2. Simplicar la expresin
q r q r q
p2
p 2
p
2
p 2 2
p
2
p 2 2
p
2
p
2 2 2
A= 2+ 3 2+ 2+ 3 2+ 2+ 2+ 3 2 2 + 2 + 2 3:
Solucin: Multipliquemos primero los factores tercero y cuarto
q q q r q
2
p2
p 2
p
2
p 2
p2
p 2 2
p
2
p
2
2 2+ 3 2 + 2 + 3 = 4 ( 2 + 3) = 4 ( 2 + 2 + 2 3)2 =
2 2 2

r q q
2
p 2
p
2
p
= 4 (2 + 2 2 + 2 3)2 = 2 2 + 2 3.

Multipliquemos el resultado obtenido por el segundo factor:


q p q p q p q p
2 2
p 2 2
p 2 2
p p 2
p
2 2+ 32
2 + 2 + 3 = 4 ( 2 + 2 3)2 = 2 4 (2 + 2 3) = 2 + 2 3. Multipliquemos
2

este resultado por el primer factor:


p p p p q p p
2 2
2 2
3 2 + 3 = 2 4 ( 2 3)2 = 2 4 3 = 1. As, pues, A = 1.
3

q p
8
3. Simplicar la expresin A = (2 7)4 .
q p p
Solucin: De acuerdo con la 5ta propiedad de las races obtenemos A = j2 7j. Pero 2 7 < 0 y,
q p pp
por lo tanto A = (2 7) = 7 2.
p p
4. Simplicar la expresin A = 27 10 2:
Solucin: Est claro que la expresin se simplicar si resulta que bajo el signo de la raz tenemos el
p
cuadrado perfecto de la diferencia entre dos ciertos nmeros. Representemos 10 2 como el producto doble
p p
de 2 nmeros, la suma de cuyos cuadrados sea igual a 27, es decir, 10 2 = 2 2 5.
p p qp p p p
De este modo, A = 2 2 2 5 + 25 = ( 2 5)2 = j 2 5j y como 2 5 < 0, A = 5 2:
p
3
p p
5. Simplicar la expresin A = 9 3 11 2:
Solucin: Razonando como en el anterior ejemplo, escribamos el radicando en forma de un cubo perfecto
de la diferencia entre dos nmeros.
p p p p p p p p p p p p
Tenemos 9 3 = 3 3 + 6 3 = ( 3)3 + 3 3 ( 2)2 y 11 2 = 9 2 + 2 2 = 3( 3)2 2 + ( 2)3 :
qp p p p p p qp p p p
3
De esta forma, A = ( 3) 3 2
3( 3) 2 + 3 3( 2) 2 ( 2) = 3 ( 3
3 2)3 = 3 2.
100 CAPTULO 2. EXPRESIONES RACIONALES E IRRACIONALES. POLINOMIOS

1
6. Liberar de la irracionalidad al denominador de la fraccin A = p
3
:
2 1
Solucin: Multiplicando numerador y denominador de la fraccin por el cuadrado no perfecto de la
p
suma de los nmeros 3 2 y 1, obtenemos :
p p p p
( 3 2)2 + 3 2 + 1 3
4+ 32+1 p
3
p
3
A= p p p = p = 4 + 2 + 1:
(32 3 3
1)(( 2)2 + 2 + 1) 3
( 2)3 13

3
7. Liberar de la irracionalidad al denominador de la fraccin A = p p :
1+ 2 3
p
Solucin: Liberamos de 3 al denominador: multiplicamos numerador y denominador por la conjugada
del denominador
p p p p p p
3(1 + 2 + 3) 3(1 + 2 + 3) 3(1 + 2 + 3)
A= p p p p = p = p :
(1 + 2 3)(1 + 2 + 3) (1 + 2)2 3 2 2

p p p p p
p 3(1 + 2 + 3) 2 3 2+2+ 6
Liberamos de 2 al denominador: A = p p = :
2 2 2 4
p3
p p
3
p
8. Calcular la suma 20 + 392 + 20 392:
p3
p p
3
p
Solucin: Hacemos A = 20 + 392+ 20 392 y elevemos al cubo los 2 miembros de esta igualdad:
p p
3
p p3
p p3
p
(20 + 392) + 3( 20 + 3 392)2 20 392 + 3 20 + 392:
p3
p p p
3
p p3
p p
3
p
( 20 392)2 + (20 392) = A3 . O bien 40 + 3 20 + 392: 20 392 ( 20 + 3 392+
p3
p p3
p p
3
p
20 392) = A3 , donde 20 + 392 + 20 392 = A:
De esta forma, obtenemos:
q p
3
40 + 3A 202 ( 392)2 = A3 ) 40 + 6A = A3 = 0:

Pero A3 6A 40 = (A3 4A2 ) + (4A2 16A) + (10A 40) =


= A2 (A 4) + 4A(A 4) + 10(A 4) = (A 4) (A2 + 4A + 10):
Como A2 + 4A + 10 = (A2 + 4A + 4)p
2
+ 6 = (A + 2)p + 6 6= 0, la igualdad (A 4)(A2 + 4A + 10) = 0 slo
3
p 3
p
se cumple para A = 4. As , pues, 20 + 392 + 20 392 = 4:
p p
9. Transformar f (a) = a2 4a + 4 + a2 + 6a + 9 a una forma sin signos de la raz y sin mdulos.
p p p p
Solucin: Como a2 4a + 4 = (a 2)2 = ja 2j y a2 + 6a + 9 = (a + 3)2 = ja 3j.
Los puntos a1 = 3 y a2 = 2 dividen la recta numrica en los intervalos ] 1; 3[, [ 3; 2[ y [2; +1[.
Analicemos la expresin prejada en cada uno de estos intervalos.
Con a < 3, tenemos: ja 2j = a + 2, ja + 3j = a 3, o sea,

f (a) = a+2 a 3= 2a 1:

Con 3 a < 2, tenemos: ja 2j = a + 2, ja 3j = a + 3 y, entonces, f (a) = a + 2 + a + 3 = 5.


Con a 2, tenemos: ja 2j = a 2, ja + 3j = a + 3, y f (a) = a 2 + a + 3 = 2a + 1. As, pues,
8
>
< 2a 1, si a < 3
f (a) = 5, si 3 a < 2
>
:
2a + 1, si a 2;
2.4. TRANSFORMACIN Y SIMPLIFICACIN DE EXPRESIONES IRRACIONALES 101
r r
a + b2 p a + b2 p
10. Simplicar f (a; b) = +2 a 2
2 a, donde a 0, b > 0.
b b
r p r p p p
( a + b)2 ( a b)2 j a + bj j a bj
Solucin: f (a; b) = = p2
:
b b b
p p p
Como
p a p0, b > 0, a + b > 0, y por lo tanto , j a + bj = a + b. Esto signica que f (a; b) =
a + b j a bj
p :
b
p p
Ahora analizamos dos casos: 1) a b 0 y 2) a b < 0:
p p
p p a+b a+b p
1) Tenemos: j a bj = a b y entonces f (a; b) = p = 2 b:
b
p p p
p p a+b+ a b 2 a
2) Aqu j a bj = ( a b) y entonces f (a; b) = p = p .
b b
8 p p
< 2p b; si a b
) f (a; b) = 2 a p
: p ; si a < b
b
p p p p p
ab2 p
3 3 3
a 3 a 2a 3 b + a2 b2 a2 b 3
11. Simplicar f (a; b) = ( p3
p3
+ p p
3
) : a2
a2 ab 3
a b
Solucin: Simplicamos consecutivamente las fracciones entre parntesis:
p p p p p p p p p p
3
a 3 a 2a 3 b + a2 b2
3 3
a2 ( a2 2 3 ab + b2 )
3 3
a( 2 a 3 b)2 p p p
1) p3
p
3
= p p p3
= p p
3
= 3 a( 3 a 3 b):
a2 ab 3
a( a3
b) 3
a b
p p p p p
3
a2 b
3
ab2 3
ab( a3 3
b) p
A continuacin, 2) p p
3
= p p
3
= 3 ab:
3
a b 3
a b
p p p3
p3
p
3
Ahora, 3) 3
a( a
3
b) + ab = a . 2
p3
p
3
Por n, 4) a2 : a2 = 1:
As, pues, f (a; b) = 1.
r
1 1 a+1
12. Simplicar f (a) = p p +p p : 1+
a+ a+1 a a 1 a 1
Solucin: Eliminamos los radicales en los denominadores de la 1ra y 2da fracciones:
p p p p
1 a a+1 a a+1 p p
1) p p = p p p p = = a+1 a:
a+ a+1 a+ a+1 a a+1 a (a + 1)
p p p p
1 a a+1 a+ a 1 p p
2) p p = p p p p = = a + a 1:
a a 1 a a 1 a+ a 1 a (a 1)
1 1 p p
3) p p +p p = a + 1 + a 1:
a+ a+1 a a 1
r p p
a+1 a 1+ a+1
4) 1 + = p :
a 1 a 1
p p
p p a 1+ a+1 p
5) a+1+ a 1 : p = a 1:
a 1
p
As, pues, f (a) = a 1:

Practico lo que aprend:

1. Hallar los valores de las expresiones:


p p p p
p p p p 5+ 2 5 2
a) 2a2 5ab + 2b2 si a = 6 + 5 y b = 6 5. b) 3a2 + 4ab 3b2 si a = p p yb= p p . c)
p p 5 p 2 p 5 +p2
p a + b 1 x + 1 xy + x a + x + a x
4a3 + 2a2 8a + 7 si a = 21 3 + 1 . d) si a = p yb= p . e) p p
a b+1 xy + 1 xy 1 a+x+ a x
2ab 1 1 1 1 1 1
si x = 2
. f ) 2a 1 + x2 2 x + 1 + x2 2 si x = ab 1 2 ba 1 2 :
1+b 2
102 CAPTULO 2. EXPRESIONES RACIONALES E IRRACIONALES. POLINOMIOS
r r p p
p p A2 B A A+A2 B
2. Utilizando la frmula A B= simplicar las expresiones:
2 2
r p p
p p p p p p p p 3 2+ 3 2 3
a) 7 + 4 3, b) 3 2 2, c) 5+2 6+ 5 2 6 , d) p p p +p p p :
2 2+ 2+ 3 2 2 3
3. Racionalizar el denominador de las fracciones:
pp p
1 1 5+ 3 1 1 1
a) p
4
p , b) p
3
p
3
, c) pp p , d) p
3
p p , e) p p , f ) pp p
5 2 3 2 5 3 4+ 6+ 39
3 4
2 4
4 1 2+ 3
3
4. Comprobar las siguientes igualdades:
q pp p
p p p
4
8 2+1 1 p
3
p p 232
3
20 + 12 3
a) q p pp q
p pp = p , b) 26 + 15 3 2 3 = 1, c) p = p ,
4 4 2 1+ 3 2+ 3
8+ 2 1 8 2 1
p p p p p p
5 2 6 5 + 2 6 49 20 6 6+4 2 6 4 2 p p
3
p
d) p p p p = 1, e) p p p +p p p = 8, f ) 5 2 + 7
27 3 18 + 3 12 8 2+ 6+4 2 2 6 4 2
p
3
p
5 2 7=2

5. Demostrar las identidades e indicar el dominio de su determinacin o c.v.a.:


q p p p p p
a) 4 6a 5 + 2 6 3 2a 2 3a = 6a
2
p p 2 p p 2
4
a+ 4
b 4
a 4
b (16a + 4b) p p
10 a 3 b
b) + p p =1
4a b 2 a+ b
r 2
16 32 2
c) 8a2 + a4 + 24 = a
a4 a2 a
p p
a2 + 2a 3 + (a + 1) a2 9 a+3
d) p =p , si a > 3:
a2 2a 3 + (a 1) a2 9 a 3

6. Simplicar las expresiones:

1 0;25 2
a) 2a + a0;75 a1;5 1 + a 0;5
p
4
p p ! p ! p p
ab ab 1 4 ab 4
ab 1 4
ab ab
b) p + p
4
p4
p
1 ab ab 1 + a3 b3 ab
m+n m+n n m
c) p p p + p p
m n m+n m mn mn + n
p r !
1+a 1 a 1 1
d) p p +p 1
1+a 1 a 1 a 2 1+a a2 a
2 2
m1;5 m0;5 n0;5
3
n0;5 3

e) m + 0;5 +
m m0;5 m0;5 n0;5
p ! 2
1 23a p
f) p p p p a2 + 8a + 16
a 4 a 1 3
a4 3
64a
1 1 3
! 1
(1 a) 4 (1 + a) 4 (1 a) 4 1 1+a 4
g) 3 + (1 a) 2
2 (1 + a) 4 2 1 a
3 p p 3 p p p
(a b) a+ b + 2a a + b b 3 ab a
h) p p +
a a+b b a b
Captulo 3

NOCIONES DE LGICA,
CONJUNTOS Y RELACIONES

La Matemtica es la reina de las ciencias y la teora de nmeros es la reina de la Matemtica.

F. Gauss

La matemtica: el inconmovible Fundamento de todas las Cien-

cias y la generosa Fuente de Benecios para los asuntos humanos.

Isaac Barrow

Unidad de competencia: Reconocer las nociones bsicas de la lgica simblica o cuanticacional como lenguaje
esquemtico para aprender a manejar, identicar y operar con diversas relaciones entre proposiciones, predicados, conjun-
tos, etc., representados por smbolos y frmulas cuyas interpretaciones estn exentas de las ambiguedades y confusiones
del lenguaje ordinario o comn.1 .

Indicadores de logro:

| Identicar e interpretar el signicado de proposiciones enlazadas mediante conectivos o cuanticadores.

| Construir tablas de verdad y calcular valores de verdad de proposiciones o enunciados.

| Manejar ecientemente las leyes lgicas que permitan desarrollar un correcto razonamiento.

| Utilizar adecuadamente el lenguage matemtico para formalizar enunciados de distinto tipo.

| Reconocer, clasicar y generar ejemplos y contraejemplos en el manejo de proposiciones, conectivos, cuanticadores,


tablas de verdad, etc.

| Utilizar objetos, diagramas, grcas y smbolos para representar conjuntos, aplicaciones, relaciones y operaciones
entre ellos.

| Aplicar las leyes del lgebra proposicional en la demostracin de relaciones entre conjuntos.

El juicio que estoy haciendo es falso (Eublides). Por favor ignoren este aviso.
1 Estos temas pueden ser ampliados en el texto [14].

103
104 CAPTULO 3. NOCIONES DE LGICA, CONJUNTOS Y RELACIONES

3.1. Nociones de Lgica

Comenzamos el presente captulo con algunos conceptos bsicos de la lgica elemental y proposicional.

3.1.1. Proposiciones y conectivos. Tablas de verdad. lgebra de proposiciones

Proposicin Una proposicin es un enunciado del que se puede armar sin ambigedad si es verdadero
(V ) o falso2 (F ). Una proposicin puede ser simple o compuesta, las simples se las representa con letras latinas
minsculas (p, q, etc.), las compuestas, con letras maysculas (P = P (p; q; : : :), Q (p; q; : : :), etc. Una proposicin
compuesta depende de otras simples, estando stas enlazadas mediante conectivos proposicionales.

Conectivos proposicionales Entre las proposiciones de un lenguaje, por ejemplo: {Hoy es viernes; Ambato
es una ciudad de Tungurahua; esto es Albania, etc.}, es posible denir los siguientes conectivos proposicionales:
_, ^, , que se leen: o, y y no, respectivamente, denominndose la primera disyuncin ( _); la segunda,
conjuncin ( ^); y la tercera negacin ( ) 3 . La proposicin p _ q (se lee p o q) es la disyuncin entre p y q, es
decir, la que signica que tiene lugar o bien p o bien q o ambas al mismo tiempo. La proposicin p ^ q (se lee p y
q) es la conjuncin entre p y q, es decir, la que signica que tienen lugar p y q al mismo tiempo. La proposicin
p Y q (se lee p o excluyente q) es una disyuncin especial con un o excluyente entre p y q, es decir, signica que
tiene lugar o bien p o bien q pero no ambas al mismo tiempo. La proposicin p o p (se lee no p) signica la
negacin de la proposicin p, es decir, no p.

Mediante el smbolo (o tambin con los 2 puntos :) indicaremos que cierta proposicin (o smbolo) colocada
a su izquierda equivale a otra proposicin (o smbolo) colocada a su derecha. As pues, si p representa la
proposicin Es domingo (p es domingo o p : es domingo ), y q la proposicin esto es Albania (q
esto es Albania o q : esto es Albania), entonces es posible formar las nuevas proposiciones compuestas:
p _ q hoy es domingo o esto es Albania. p ^ q hoy es domingo y esto es Albania. p hoy no es
domingo. q esto no es Albania.

Figura 3.1: Izq. El genio de la lgica y de lo indecidible, el austriaco Kurt Godel (Brno, Imperio Austro-hngaro1906
- Princeton, 1978), y el genio de la relatividad general, el alemn Albert Einstein ( Ulm, Alemania,1879 - Princeton
1955). Der. Norbert Wiener (Columbia, Misuri, 1894 - Estocolmo, 1964), famoso matemtico estadunidense, padre de
los procesos cibernticos y la teora de los autmatas, gran especialista en procesos aleatorios, armnicos, movimiento
browniano y muchsimas aplicaciones (series temporales).

Tablas de verdad Dando a p y q todos los posibles valores, V o F (o 1 = V y 0 = F), se obtienen las
2 En lugar de la V de verdadero, y en lugar de la F de falso, se usan tambin los valores 1 y 0, respectivamente.
3 La negacin se representa mediante p o mediante p. As mismo, p & q y p ^ q representan a una misma conjuncin. La
expresin p Y q es una disyuncin especial, Y es un o excluyente, que signica: o bien p, o bien q, pero no ambos al mismo tiempo.
3.1. NOCIONES DE LGICA 105

siguientes tablas, llamadas tablas de verdad:

La proposicin p_q es falsa nicamente si p y q son falsas simultneamente, en cualquier otro caso es verdad.
Y esto es su valor de verdad.

La proposicin p ^ q nicamente es verdad si lo son p y q simultneamente, en cualquier otro caso es falsa.


Y esto es su valor de verdad.

p q p_q p^q pYq p q p_q p^q pYq


V V V V F 1 1 1 1 0 p p p p
V F V F V o 1 0 1 0 1 , V F o 1 0
F V V F V 0 1 1 0 1 F V 0 1
F F F F F 0 0 0 0 0

Tautologa y contradiccin Se llama tautologa a una proposicin que siempre es verdad; se indica por
T . La proposicin contraria, su negacin, T , se suele indicar por K, siempre es falsa y se llama contradiccin
o antiloga.

p p p_ p T p^ p K p p p_ p T p^ p K
V F V F o 1 0 1 0
F V V F 0 1 1 0

En calidad de ejemplos, comprobar que p ! (p _ q) y (p ! (q ! r)) ! ((p ^ q) ! r) son tautologas (sus


negaciones son contradicciones), y (p _ p) ! (q ^ q) es una contradiccin (su negacin es tautologa).

Implicacin Dadas dos proposiciones p y q, la proposicin p _ q se llama implicacin, se la designa


mediante p ! q, y se lee: p implica qo si p entonces q. La tabla de verdad correspondiente a la implicacin
es:

La proposicin p ! q nicamente es falsa si p es verdadera y q es falsa, en cualquier otro caso es verdadera. Y


esto es su valor de verdad. Si la proposicin P ! Q (simple o compuesta) es verdadera, se dice que constituye
un teorema, siendo P la hiptesis (lo que nos dan), y Q la tesis (lo que hay que demostrar o conclusin). En
este caso se dice tambin que P es una condicin suciente de Q (ya que de P se deduce Q), y que Q es una
condicin necesaria de P .

p q p!q p q p!q
V V V 1 1 1
V F F o 1 0 0
F V V 0 1 1
F F V 0 0 1

Proposiciones equivalentes. Bicondicional Dos proposiciones se dice que son equivalentes, si cada una
de ellas implica la otra, es decir, si: (P ! Q) ^ (Q ! P ), lo que tambin se expresa escribiendo P $ Q con
ayuda del bicondicional $. En el caso de equivalencia, las dos proposiciones P y Q son simultneamente verdad
o mentira, y diremos que: P (Q) es verdadera si y slo si Q (P ) es verdadera. Y esto es su valor de verdad.
Para ser P (Q) verdadera es condicin necesaria y suciente que Q (P ) sea verdadera. Dos proposiciones
simples o compuestas P y Q se dicen logicamente equivalentes si sus tablas de verdad son idnticas, es decir si
sus 2 columnas resultantes coinciden, esto ser representado mediante P Q.

Un ejemplo muy importante de dos proposiciones equivalentes es: (P ! Q) $ ( Q! P ), por lo que


diremos a continuacin.
106 CAPTULO 3. NOCIONES DE LGICA, CONJUNTOS Y RELACIONES

Demostracin por reduccin al absurdo En un teorema: P ! Q, para establecer que la conclusin Q


es cierta, existe un mtodo llamado por reduccin al absurdo, que consiste en suponer que Q es cierta (Q
falsa) y razonar lgicamente hasta llegar a una contradiccin con la hiptesis P . (Con ello se llega a un absurdo,
y se dice que el absurdo ha estado en suponer que la conclusin Q era falsa).

3.1.2. Leyes del lgebra de proposiciones

Sean P = P (p; q; : : :), Q = Q (p; q; : : :), etc. proposiciones simples o compuestas, entonces:

Recuerda las leyes para las proposiciones:


Idempotencia: P _ P $ P; P ^ P $ P
Asociativa: P _ (Q _ R) $ (P _ Q) _ R; P ^ (Q ^ R) $ (P ^ Q) ^ R
Conmutativa: P _ Q $ Q _ P; P ^ Q $ Q ^ P
Simplicacin: P _ (P ^ Q) $ P ; P ^ (P _ Q) $ P
Distributiva: P _ (Q ^ R) $ (P _ Q) ^ (P _ R); P ^ (Q _ R) $ (P ^ Q) _ (P ^ R)
Neutros: P _ K $ P; P ^ T $ P
Absorcin: P _ T $ T; P ^ K $ K
Complemento: P _ ( P) $ T; P ^ ( P) $ K
De Morgan: (P _ Q) $ ( P ) ^ ( Q); (P ^ Q) $ ( P ) _ ( Q)
Doble negacin: ( P) $ P; (K) $ T

Polinomios Booleanos Supongamos ahora que las letras p, q, etc. que antes denotaban enunciados, son
variables o indeterminadas combinadas con los conectivos _, ^, Y, , !, $, etc., entonces obtenemos los as
denominados polinomios booleanos. Ejemplos de estos polinomios seran:

f (p; q) = (p ^ q) Y (q ! p) , Q (p; q; r) = ((p $ r) ^ (q ! p)) q, etc.

Se pueden utilizar los conectivos para obtener combinaciones de polinomios booleanos, y poner, por ejemplo,

f (p; q) ! Q (p; q; r) o f (p; q) _ Q (p; q; r) $ Q (p; q; r) Y f (p; q) , etc.

que vienen a ser nuevos polinomios booleanos. Es evidente que una proposicin compuesta P (p; q; : : :) tambin
es un polinomio booleano y puede ser analizado como tal. Supongamos ahora que cada una de las variables p, q,
r, etc. de cierto polinomio booleano f (p; q; r; : : :) se remplazan respectivamente por los enunciados particulares
denotados mediante p0 , q0 , r0 , etc., entonces la expresin f (p0 ; q0 ; r0 ; : : :) tambin es un enunciado y por lo
mismo tiene un valor de verdad y su respectiva tabla de verdad.

| Por ahora, aceptaremos intuitivamemte que conjunto es una reunin de objetos con cierta propiedad.

3.1.3. Juicio y razonamiento. Paradojas

Se denomina juicio a la forma de pensamiento abstracto en que se arma o se niega algo respecto a los
objetos, los vnculos entre un objeto y sus propiedades o las relaciones entre objetos. si hay correspondencia
entre un juicio y el estado real de las cosas, entonces es un juicio verdadero, caso contrario es un juicio falso.
Razonamiento es la forma de pensamiento mediante la cual, y a base de ciertas reglas de inferencia, de uno o
varios juicios verdaderos se obtiene un nuevo juicio que se inere de aquellos de modo necesario o con determinado
grado de probabilidad. Ejemplos de razonamientos (el primero correcto, el segundo no) seran:
3.1. NOCIONES DE LGICA 107

Todas las ballenas son mamferos cetceos. Todos los metales son cuerpos slidos.
Este animal es una ballena. El mercurio es un metal.
Este animal es mamfero cetceo. El mercurio es un cuerpo slido.

Los 2 juicios sobre la raya se denominan premisas y el tercero, conclusin. Los razonamientos se dividen en
deductivos, inductivos y analogas. Los razonamientos incorrectos que no garantizan la verdad de la conclusin,
ni siquiera cuando sabemos que las hiptesis son verdaderas, se denominan falacia:

Todas las reglas tienen excepciones


Pero esta armacin es una regla.
Por tanto tiene excepciones.
Luego hay reglas que no tienen excepcin.

Las 4 frases anteriores son contradictorias, se parte de una armacin y se demuestra lo contrario. El origen
de la contradiccin proviene de la frase inicial que es una armacin falsa, pues si la declaracin es cierta, se
concluye que es falsa. Al contrario, si es falsa, se concluye que hay reglas que no tienen excepciones y no caemos
en una contradiccin, por lo tanto hay reglas que no tienen excepcin.

Aquel raciocinio que demuestra tanto la veracidad como la falsedad de un juicio, es decir, demuestra tanto
este juicio como su negacin, se denomina paradoja. Se conocen las paradojas del montn, del calvo, de Zenn,
del general y el barbero, del catlogo de todos los catlogos normales, etc. Analizamos las siguientes paradojas:

| Paradoja del alcalde de la ciudad : cada alcalde vive en su ciudad o fuera de ella. Se orden destinar una
ciudad especial para alojar solamente a los alcaldes que no vivan en sus ciudades. se pregunta, dnde debera
el alcalde de esta ciudad especial?. Si quiere vivir en su ciudad no lo podra hacer, por cuanto en ella solo
pueden vivir los alcaldes que no viven en sus ciudades; si no quiere vivir en su ciudad, entonces, igual que todos
los alcaldes que no viven en sus ciudades, debe vivir en la ciudad especial, es decir, en la suya. As pues, no
puede vivir en su ciudad ni fuera de ella!. Esta paradoja es anloga a la del general y el barbero.

| Paradoja del montn: sea un montn de granos de arena. Condicin: la diferencia entre un montn y no
montn no estriba en un grano de arena. Comencemos a tomar de ese montn grano a grano, pero segn la
condicin el montn sigue siendo montn: 120 granos forman un montn; 119, un montn; 25, un montn; ...
; 2, un montn y 1 grano, tambin. Aunque el sentido comn apunta que 1 grano no es un montn. El quid
de la paradoja consiste en que los cambios cuantitativos graduales (disminur de grano en grano) no acarrean
cambios cualitativos. La paradoja del calvo es anloga, es decir, la diferencia entre un calvo y un no calvo no
consiste en un pelo.

| Paradoja de Russell de los conjuntos normales: analizamos la paradoja sobre el catlogo de todos los
catlogos normales. Los catlogos se dividen en 2 gneros o grupos: 1) los que no se mencionan as mismos entre
los catlogos enumerados (normales) y 2) los que forman parte de los catlogos enumerados (anormales). El
bibliotecario recibe la orden de confeccionar un catlogo de todos los catlogos normales y solamente normales.
Debe mencionarlo en su propio catlogo? Si lo menciona, el catlogo confeccionado por l ser anormal, es
decir, no tendra derecho a mencionarlo. Si no lo menciona, uno de los catlogos normales, precisamente el
confeccionado por l, no ser mencionado, aunque debera mencionar todos los catlogos normales. As pues,
no puede mencionar ni dejar de hacerlo el catlogo que confecciona. Cmo se debe proceder en este caso? Es
natural percatarse de que el concepto catlogo normalno tendr un volumen jado mientras no se aclare qu
catlogos deben tomarse en consideracin, por ejemplo, en que biblioteca y en el tiempo en que se hallan. Si la
tarea consiste en confeccionar un catlogo de todos los catlogos normales existentes hasta el 31 de diciembre
de 2005, el volumen del concepto catlogo de todos los catlogos normales ser jado y, al confeccionar su
catlogo, el bibliotecario no deber mencionarlo. Pero, si le plantean de nuevo una tarea anloga despus de
haber confeccionado ya su primer catlogo, tendr que incluirlo tambin. As se solucionar la paradoja.
108 CAPTULO 3. NOCIONES DE LGICA, CONJUNTOS Y RELACIONES

| Paradoja de Russell del conjunto de todos los conjuntos: analizamos el caso en que el concepto de conjunto
de todos los conjuntos es contradictorio. Supongamos que para el conjunto X la expresin o propiedad P (X)
signica que X no se contiene as mismo en calidad de su elemento. Analicemos la clase Y = fX : P (X)g de
conjuntos que satisfacen la propiedad P . Si Y es un conjunto, entonces, o bien es cierto P (Y ), o bien es cierto
P (Y ) = P (Y ). Sin embargo, esta alternativa para Y no es posible. En efecto, P (Y ) no es posible ya que de
la denicin de Y se seguira que Y contiene a Y , es decir, es cierto P (Y ); por otro lado, P (Y ) tampoco
es posible porque signicara que Y contiene a Y , y esto contradice la denicin de Y como clase de aquellos
conjuntos que no se contienen as mismos. Por tanto, Y no es un conjunto.

| Paradoja de la verdad y la falsedad. La presente declaracin es falsa. Pueden considerarse 2 hiptesis:


a) o bien la declaracin es falsa, y como es falsa, lo que se declara es cierto, y la declaracin es cierta; b) o
bien la declaracin es cierta, y lo que declara es cierto, la declaracin es entonces falsa. En este caso no puede
evitarse el crculo vicioso. Si la declaracin es falsa, es cierta, y si es cierta, es falsa. Puede deducirse que esta
declaracin no es ni cierta ni falsa. La paradoja de Parmnides que dice: Todos los cretences son mentirosos
es anloga a la anterior y hay que subdividirla en casos. Se debe tener en cuenta que lo contrario de Todos
los cretences son mentirosos no es Todos los cretences dicen la verdad, sino No todos los cretences son
mentirosos.

| Paradoja de la puerta a la libertad. Cierto sultn encierra a un prisionero en una celda con 2 puertas, la
de la libertad y la de la esclavitud. Existe un solo carcelelero, este o bien dice siempre la verdad, o bien siempre
miente, o bien dice a veces la verdad y a veces miente. Eso si, el carcelero razona todo en trminos lgicos. El
prisionero tiene derecho a hacer una sola pregunta, y slo una al carcelero. Se pregunta, puede encontrar la
puerta de la libertad, cuando ignora si el carcelero miente o dice la verdad?. Se podra decir que, sea cual sea la
pregunta formulada, el prisionero no sabr si la respuesta es verdadera o falsa, y por tanto no podr encontrar la
puerta de la libertad, pero si existe solucin. El prisionero hace la siguiente pregunta: es verdad la siguiente
armacin: si esta puerta es la de la libertad, su respuesta a mi pregunta ser falsa, y si no ser verdad?. El
carcelero razona en trminos lgicos, tanto al mentir como al decir la verdad, su respuesta sealar al prisionero
la puerta a la libertad. Analicemos los casos:

El carcelero: La puerta sealada es la de la libertad: La armacin es: El carcelero responde:


dice la verdad si falsa no
miente si verdadera no
dice la verdad no verdadera si
miente no falsa si

Como se ver, se puede obligar al carcelero a revelar la verdad.

| Un caso parecido es cuando el mismo prisionero est con 2 carceleros, uno que siempre dice la verdad y el
otro que siempre miente. As mismo, el prisionero slo puede hacer a uno de los 2 carceleros una y solamente una
pregunta. El prisionero no sabe cual de los 2 siempre miente y cual siempre dice la verdad. Puede el prisionero
obtener su libertad con toda seguridad? El prisionero debe preguntar a uno de los carceleros: si le dijera a tu
compaero que me seale la puerta de la libertad, qu me respondera?. En los 2 casos, el carcelero seala la
puerta a la esclavitud.

3.1.4. Funcin proposicin o predicado. Cuanticadores

En matemticas son muy usuales ciertas expresiones, enunciados o polinomios booleanos dependientes de
una o varias variables, en los que intervienen las variables x, y, z, etc.. Estas expresiones especiales que se
convierten en proposiciones para ciertos valores jos de sus variables, se denominan funciones proposicin o
predicados. As, por ejemplo: p(x) x2 0, x 2 R; q(y) : 5y y 2 = 4, y 2 N; r(x) g(x) = ;
3.1. NOCIONES DE LGICA 109

x y
r(x; t) 2x 3t = 5, x; t 2 Z; P (x; z) : x3 z < 1, x; z 2 N; Q(x; y; z) 3z, x; y; z 2 Q;
x2 + y 4
x z
R(x; z) 3x, (x; z) 2 f( 3; 2) ; ( 1; 3) ; (5; 9)g, etc., son predicados dependientes de las variables
z
x, y, z, etc., las mismas que pertenecen a ciertos conjuntos (por ejemplo, numricos N, Q, etc.) de denicin:
x 2 D1 , y 2 D2 , z 2 D3 , (x; y) 2 D1;2 , (y; z) 2 D2;3 , etc. Es evidente que las funciones proposicin, polinomios
booleanos o predicados tienen valor de verdad para ciertos valores jos de sus variables, por ejemplo para x = x0 ,
y = y0 , z = z0 , etc. El conjunto de valores de cada una de las variables, de sus parejas, o de sus triadas, etc. para
los que las funciones proposicin o predicados son verdaderas, se denomina conjunto de validez o de valores de
verdad y se lo representa mediante Vx , Vy , Vz , Vx;y , Vx;y;z , etc. As, por ejemplo, si se tiene el predicado p (x) :
x2 +3x 4 y x 2 D1 = f 6; 5; 3; 1; 0; 1=2; 1; 5; 10g, entonces el conjunto de validez o de valores de verdad
es Vx = f 6; 5; 1; 5; 10g, pues las proposiciones (valores de los predicados) p ( 6) :18 4, p ( 5) :10 4,
p (1) :4 4, p (5) :40 4, p (10) :130 4 son verdaderas y las restantes son falsas. Si tenemos
Q (y; z) :(3y 2z) 2 N, N = f1; 2; 3; : : :g y D2;3 = f(y; z)g = f( 2; 0) ; ( 1; 1) ; (0; 2) ; (1; 1) ; (3; 2) ; (5; 7)g,
entonces el conjunto de validez o de valores de verdad es Vy;z = f(1; 1) ; (3; 2) ; (5; 7)g, pues las proposiciones
(valores de los predicados) Q (1; 1) :1 2 N, Q (3; 2) :5 2 N, Q (5; 7) :1 2 N son verdaderas.

Cuanticadores Analizando la proposicin p(x) : x2 0con x 2 D1 = R, vemos que es cierta para todo
nmero real sin excepcin (Vx = D1 = R), lo que se indica mediante (8x 2 R) p(x), y se lee: cualquiera que
sea x de R, se cumple p(x), pues p(x) x2 0 es siempre cierta. El smbolo 8 se denomina cuanticador
universal y se lee para todo o cualquiera que sea.

Ahora, analizando la proposicin q(y) 4 y < 0 sobre Y = f 1; 0; 1=2; 5; 7g, vemos que es cierta, si y
es igual a 5 o a 7, lo que se indica por: (9y 2 Y) q(y), y se lee: existe al menos un elemento de Y, que cumple
q(y). El smbolo 9 se denomina cuanticador existencial y se lee existe un o se encuentra un o hay
un. Adems se escribe 9!, para indicar que existe un elemento, y slo uno, cumpliendo cierta condicin o
2
proposicin, as por ejemplo, para la proposicin R(z) : (3=2 + z) 0 con z 2 D1 = Q, vemos que es
cierta solamente para el valor z = 3=2 2 Q y solo para ese valor: (9!z 2 Q) (R(z)), es decir, existe un nico z
que satisface R (z).

Figura 3.2: Izq. El lsofo, lgico, matemtico, ensayista y pacista britnico, inconforme con la sociedad y a veces con la
matemtica, Bertrand Russell (Trelleck, Inglaterra, 1872 - Pernhyndeudraeth, Gales, 1970). Pretendi, sin xito, reducir
toda las bases matemticas a la lgica. Junto a Alfred N. Whitehead escribi los famosos Principia Mathematica.
Gran divulgador de la ciencia, luchador por los derechos civiles, anticlerical y furibundo pacista. Premio Nobel de
Literatura en 1950. Der. Bernhard Riemann (Breselenz, Hannover, 1826 - Selasca, Italia, 1866), genial matemtico
del siglo XIX. Gran especialista en Anlisis, Variable compleja, Teora de nmeros, Geometra eucldea y no eucldea
(geometra elptica), Topologa y ecuaciones de la fsica matemtica. Muri de tuberculosis.

Equivalencias entre cuanticadores Si E es el dominio de denicin, por ejemplo, de la funcin proposi-


cin p(x) (o P (x; y; : : :), etc.), se tienen las siguientes equivalencias, que vienen a ser las reglas para negar
predicados que contienen cuanticadores:
110 CAPTULO 3. NOCIONES DE LGICA, CONJUNTOS Y RELACIONES

[(8x 2 E) p(x)] $ [(9x 2 E) p(x)] ; [(9x 2 E) p(x)] $ [(8x 2 E) p(x)]

Ejemplos:

1. Escribir las negaciones de las siguientes expresiones, formalizando primero.

a) Estudias o te vas de vacaciones.


Solucin: p : estudias. q : vas de vacaciones. Formalizacin: p _ q. Negacin: (p _ q) p^ q
)No estudias y no te vas de vacaciones.
b) Si no es rectngulo, no es cuadrado.
Solucin: p : es rectngulo. q : es cuadrado. Formalizacin: p ! q. Negacin: (p ! q) p^q
)No es rectngulo y es cuadrado.
c) (8x 2 R) x + 2 1 ^ x 6= 2 _ x 3 > x2
Solucin:
(9x 2 R) h x+2 1 ^ x 6= 2i_ x 3 > x2 ; (9x 2 R) h [p (x) ^ (q (x) _ r (x))]
i ;
(9x 2 R) p (x) _ (q (x) _ r (x)) ; (9x 2 R) p (x) _ q (x) ^ r (x) ;
2
(9x 2 R) x + 2 > 1 _ x = 2 ^ x 3 x
p
d ) (9x 2 N) 2x + 1 > 1 Y x 3
Solucin:
p
(8x 2 N) 2x + 1 > 1 Y x 3 ; (8x 2 N) (p (x) Y q (x)) ;
p
(8x 2 N) (p (x) $ q (x)) ; (8x 2 N) 2x + 1 > 1 $ x 3
e) (8x 2 R) (9y 2 R) [jx yj y ! 2x 3y 6= 7]
Solucin:
(9x 2 R) (8y 2 R) [jx yj y ! 2x 3y 6= 7] ; (9x 2 R) (8y 2 R) (p (x; y) ! q (x; y)) ;
(9x 2 R) (8y 2 R) p (x; y) ^ q (x; y) ; (9x 2 R) (8y 2 R) (jx yj y ^ 2x 3y = 7)

2. Hallar los conjuntos de validez Vp de las siguientes funciones lgicas, si X = f 1; 0; 1; 2; 3; 5g

a) (x 2 X) p (x) : x2 1 x
Solucin: Vp = x 2 X : x2 1 x = f 1; 2; 3; 5g
b) (x 2 X) (p (x) : j2x 3j < 3)
Solucin: Vp = fx 2 X : j2x 3j < 3g = f1; 2g
c) Es verdad o falso que: (8x 2 X = N) (p (x) : 2x 1 < 3)
Solucin: Es falso, pues como contraejemplo tenemos que para x0 = 1 2 X : 2 ( 1) 1= 3 3.
Se debe cumplir para todos!
d ) Es verdad o falso que: (9x 2 X = Z) p (x) : x2 1 6= 3
2
Solucin: Es verdad, pues para x0 = 1 2 Z : ( 1) 1 = 0 6= 3. Se debe cumplir para algunos!

Observaciones:

| Considerando los smbolos ((! y ) o y (), ($ y , o $ y ) obsrvese que P (p; q; : : :) !


Q (p; q; : : :) (P (p; q; : : :) $ Q (p; q; : : :) o P (p; q; : : :) Q (p; q; : : :)) es precisamente una proposicin y que su
tabla de verdad solo puede contener o bien V (1) o bien F (0) en su ltima columna. Pero P (p; q; : : :) )
Q (p; q; : : :) (P (p; q; : : :) , Q (p; q; : : :) o P (p; q; : : :) Q (p; q; : : :)) dene una relacin entre proposiciones
compuestas, a saber, que la proposicin compuesta P (p; q; : : :) ! Q (p; q; : : :) (P (p; q; : : :) $ Q (p; q; : : :) o
P (p; q; : : :) Q (p; q; : : :)) solo contiene V (o 1) en su columna resultante de su tabla de verdad, o sea que
3.1. NOCIONES DE LGICA 111

es una tautologa. Smbolos ), (, ,, generalmente se los utiliza en equivalencias totalmente ciertas como
podran ser en el uso de axiomas o en resultados de teoremas ya demostrados o ya conocidos. As, por ejemplo,
x 2 R ) x2 + 1 > 0; x 2 R, x2 + x = 12 , ((x = 4) _ (x = 3)); (p ! q) ^ (q ! r) ) (p ! r) ya que
(p ! q) ^ (q ! r) $ (p ! r)(es una tautologa;( P (p; q; : : :) , P (p; q; : : :), pues P (p; q; : : :) $ P (p; q; : : :) es
x y=3 x=4
una tautologa; por ejemplo, , , 2x4 + 3 < 0; x 2 R ) ; , etc.
x+y =5 y=1

| Es evidente que en las leyes del lgebra de proposiciones podemos poner los smbolos ), (, , en lugar de
los smbolos !, , $, respectivamente, pues los resultados a obtenerse con proposiciones simples o compuestas
que cumplan directamente con las leyes del lgebra de proposiciones dan como resultado tautologas.

Ejemplos:

1. Dice que hace calor o que no hace calor la proposicin? No es cierto que sea falso que no hace calor.
Solucin: C: no hace calor. C: es falso que C (lo que equivale a C ). C: no es cierto
que C .

2. Cmo se formalizaran los siguientes enunciados?

a) No vi la novela, pero le el peridico


Solucin: Siendo N : ver novela y P : leer el peridico. N ^P
b) Ni vi la novela ni le el peridico.
Solucin: N^ P
c) No es cierto que viese la novela y leyese el peridico.
Solucin: (N ^ P )
d ) Vi la novela, aunque no le el peridico.
Solucin: N ^ P
| Es frecuente el error de interpretar c) como no P y no R(o sea: ni P ni R). De ser as, c) y b)
diran exactamente lo mismo y seran por tanto formulaciones equivalentes de una misma verdad, lo
cual es falso. b) Es una conjuncin de dos negaciones. Un trmino niega tajantemente que se ha visto
la novela, y el otro que se ha ledo el peridico. La conjuncin ser por tanto verdadera slo cuando
efectivamente no se haya visto la novela ni ledo el peridico. c) Es la negacin de una conjuncin. Y
la negacin de una conjuncin slo descarta que ocurran las dos cosas, pero no una de ellas, c) ser
por tanto verdadera cuando: 1) yo haya visto la novela pero no ledo el peridico; 2) ledo la revista
pero no visto la novela; 3) Ni he visto la novela ni ledo el peridico. En este tercer caso la verdad
de c) coincidira con la de b). Pero una coincidencia slo parcial no es suciente fundamento para
armar la equivalencia entre dos proposiciones.

Dicen lo mismo estas proposiciones?:

3. Llueve y o bien nieva o sopla viento.


Solucin: L ^ (N _ V ) (L: llueve; N : nieva; V : hace viento).
La proposicin 3 es una conjuncin cuyo primer trmino arma tajantemente que llueve y cuyo segundo
trmino es una disyuncin que dice que nieva o hace viento. La verdad de 3 exige que llueva y que adems
ocurra uno cualquiera de los otros dos fenmenos (o los dos).

4. O est lloviendo y nevando, o est soplando el viento.


Solucin: Las proposiciones 3 y 4, ni sintctica ni semnticamente son equivalentes. La proposicin 4 es
una disyuncin con un primer trmino formado por una conjuncin, y cuya formalizacin sera (L^N )_V .
La verdad de 4 no exige ningn fenmeno en concreto, sino slo que ocurra al menos una de las alternativas
sin pronunciarse por cul. En un da ventoso y sin lluvia la proposicin 3 sera falsa y la 4 verdadera.
112 CAPTULO 3. NOCIONES DE LGICA, CONJUNTOS Y RELACIONES

5. O bien no es cierto que llueve y nieva, o sopla viento.


Solucin: (L ^ N ) _ V

6. O est lloviendo y nevando, o ninguna de las dos cosas.


Solucin: (L ^ N )_ (L _ N )

7. Cmo se formalizaran los siguientes enunciados?


a) No es el caso que ni llueva ni haga viento. Solucin: ( L^ V )
b) No es cierto que llueva y nieve. Solucin: (L ^ N )
c) No es cierto que llueve pero no nieva. Solucin: (L^ N )
d) Llueve o hace viento. Solucin: L _ V
e) O bien no est lloviendo o est nevando. Solucin: L_N
Formalizar como disyunciones (inclusivas) las siguientes frases. En la primera de ellas hacer
una perfrasis y formularla reejando su sentido exclusivo:

8. O los hombres han nacido iguales o no son libres.


Solucin: I_ L (I: los hombres... iguales; L: ... son libres)
Como disyuncin exclusiva: (I_ L)^ (I^ L)

9. O bien G y P son ambos culpables, o P es inocente (= no culpable).


Solucin: (G ^ P )_ P (G: G culpable; P : P culpable).

10. O G es culpable, o l y P lo son conjuntamente.


Solucin: G _ (G ^ P )

11. O bien P es culpable, o G es inocente, o ambos son culpables.


Solucin: P _ G _ (P ^ G)

12. O H tiene razn, o P y G son o ambos culpables o ambos inocentes; y G es culpable.


Solucin: H _ ((P ^ G) _ ( P^ G)) ^ G (H: H tiene razn).

Practico lo que aprend:

Construir las negaciones de las 12 proposiciones anteriores y formalizarlas.

Formalizacin de enunciados condicionales:

Ejemplos:

1. Son equivalentes algunas de las proposiciones siguientes?


a) Luis se ir si Penlope se queda. b) Si Penlope se queda, entonces Luis se va.
c) Supuesto que Penlope se que de,Luis se ir. d) Luis se ir en caso de que Penlope se quede.
Solucin: Todas ellas son equivalentes : P ) L (P : Penlope se queda; L: Luis se va)

2. Miles de vidas podran salvarse cada ao si la gente utilizara el cinturn de seguridad.


Solucin: S ) C (S: la gente utiliza el cinturn de seguridad; C: se salvan miles de vidas)

3. Democracia signica un modo de vida en el que la libertad y la justicia estn presentes.


Solucin: D ) (L ^ J)

4. No es el caso que, si la luna est hecha de queso, entonces las naves espaciales no pueden alunizar en ella.
Solucin: (L ) A) (L: La luna est hecha de queso; A: las naves pueden alunizar)
3.1. NOCIONES DE LGICA 113

5. Si la reina Elizabeth est furiosa, entonces o el Lobo est desconcertado o Diana no ser coronada reina.
Solucin: E ) L_ D (E: Elizabeth est furiosa; L: el Lobo est desconcertado; D: Diana es coronada)

6. Si los verdaderos amigos tienen todo en comn, entonces t no puedes ser ms rico que tu compaero si
dices que son verdaderos amigos. Platn.
Solucin: C ) (A ) R) (C: los amigos tienen todo en comn; A: dices que son verdaderos amigos;
R: puedes ser ms rico que tu compaero).

Practico lo que aprend:

Construir las negaciones de las 6 proposiciones anteriores y formalizarlas.

Formalizacin de enunciados condicionales:

Ejemplos:

1. 17 es un nmero primo porque slo es divisible por s mismo y por la unidad.


Solucin: C ^ U ) P (C: 17 es divisible por s mismo; U : 17 es divisible por la unidad; P : 17 es primo)

2. Abdal ir a la esta slo si Marta va.


Solucin: A ) M (A: Abdal va a la esta; M : Marta va)
La presencia de la partcula si precediendo a la palabra Marta podra inducir a formalizar este
enunciado como M ) A, lo cual sera errneo. Un condicional, ledo de izquierda a derecha, simboliza
las condiciones sucientes (representadas por el antecedente), pero no las necesarias (representadas por
el consecuente). En este ejemplo, la palabra sloest subrayando precisamente esa necesidad: para que
Juan vaya, es necesario que Marta vaya a la esta; o, si Abdal est, es porque Marta est, pero no a la
inversa.
Tal vez este ejemplo sea ms ilustrativo: Llover slo si hay nubes, cuya formalizacin es L ) N . Las
nubes son desde luego condicin necesaria para que llueva, pero no suciente. De simbolizar ese enunciado
como N ) L, estaramos armando una falsedad, puesto que son muchos los das en que el cielo est
bastante nublado y sin embargo no llueve. Dicho en pocas palabras y a ttulo de regla prctica: la expresin
slo si introduce consecuentes.

3. Frida pertenece al club slo si es mujer.


Solucin: C ) M (C: Frida pertenece al club; M : Frida es mujer)

4. Aniceto entrar en la Universidad Central del Ecuador slo si obtiene buena puntuacin en los exmenes.
Solucin: U ) P (U : entra en la Universidad; P : obtiene buena puntuacin)

5. Proporcinenos los medios y nosotros solucionaremos el asunto. (Churchill a Roosevelt en la II Guerra


Mundial).

Solucin: P ) S
La presencia de la copulativa ypodra inducir a interpretar esta frase como una conjuncin, cuando su
verdadero sentido es el de un condicional: si usted nos proporciona los medios, nosotros solucionaremos
el asunto (la guerra).

6. Obtendrs la licenciatura a condicin de que apruebes el ltimo curso.


Solucin: S ) L

7. A menos que me detenga a almorzar en la carretera, llegar temprano por la tarde.


Solucin: C)P (C: me detengo a almorzar; P : llego temprano por la tarde)
114 CAPTULO 3. NOCIONES DE LGICA, CONJUNTOS Y RELACIONES

8. Nos veremos en el caf, supuesto que no llueva.


Solucin: L)C (L: llueve; C: caf)

9. Una condicin necesaria, aunque quiz no suciente, para que comiencen las negociaciones es que P cese
en sus actos de agresin contra I.
Solucin: N ) A (N : comienzan las negociaciones; A: P comete actos de agresin contra I)

10. Si los que ya son sabios no buscan la sabidura y los ignorantes impenitentes tampoco, entonces los que la
busquen no sern los sabios ni los ignorantes, sino aquellos que reconocen su propia ignorancia y desean
remediarla.
Solucin: ( S^ I) ) ( (S _ I) ^ R) (S: los sabios buscan la sabidura; I: los ignorantes la buscan;
R: los que reconocen su ignorancia ...).

Practico lo que aprend:

Construir las negaciones de las 10 proposiciones anteriores y formalizarlas.

Formalizacin de enunciados:

Ejemplos:

1. Creer en otras mentes es racional si y slo si creer en Dios es racional.


Solucin: M , D (M : creer en otras mentes es racional; D: creer en Dios es racional)
Hay gran diferencia entre las expresiones slo si y si y slo si. La primera indica condiciones
necesarias, mientras que la segunda recoge tanto las condiciones necesarias como las sucientes de un
condicional. Al simbolizar, la primera expresin introduce simplemente el consecuente de una implicacin;
la segunda, en cambio, es la expresin de un bicondicional, cuyo trasunto formalizado es una coimplicacin.

2. Aparicio har el doctorado cuando y solamente cuando obtenga la licenciatura.


Solucin: A , D

3. Un conjunto C es un subconjunto propio de un conjunto D si y slo si no hay ningn miembro de C que


no sea miembro de D, pero hay un miembro de D que no es miembro de C.
Diccionario:
9
>
A : hay un miembro de C que no es miembro de D; =
B : hay un miembro de D que no es miembro de C; Solucin: P , A ^ B:
>
;
P : C es un subconjunto propio de D:

Practico lo que aprend:

Construir las negaciones de las 3 proposiciones anteriores y formalizarlas.

Formalizacin en lenguaje de predicados:

Ejemplos:

1. La tierra es esfrica.
Solucin: E t (E: esfrico; t: tierra)
La tierra es esfrica es una proposicin singular, en cuya formalizacin slo intervienen una variable
predicativa (E) y una constante, que, obviamente, no se cuantica.
3.1. NOCIONES DE LGICA 115

2. Algo es esfrico.
Solucin: 9x Ex
Comprese con el ejemplo anterior. La x no es aqu una constante, sino una variable de objetos sin
determinar que requiere el uso de cuanticador.

3. 2 es primo y par.
Solucin: P s ^ Qs (P : primo; Q: par; s: 2)

4. 2 es un nmero primo y par


Solucin: N d ^ P d ^ Qd

5. Algn nmero es primo y par.


Solucin: 9x (N x ^ P x ^ Qx)

6. Los tigres son felinos.


Solucin: 8x (T x ) F x) (T : tigre; F : felino)

7. Hay serpientes que no son venenosas.


Solucin: 9x (Sx^ V x) (S: serpiente; V : venenosa)
Una proposicin negativa, sea universal o particular, se formaliza no negando la proposicin entera sino
slo el predicado.

8. Existen seres vivos que no pueden nadar.


Solucin: 9x (V x^ N x) (V : ser vivo; N : poder nadar)

9. Los rinocerontes nunca trepan a los rboles.


Solucin: 8x (Rx ) T x) (R: rinoceronte; T : trepa)
Las proposiciones 7 y 8 son ejemplos de proposiciones particulares negativas, y la 9 es una proposicin
universal negativa, que, sumadas a las dos armativas (universal y particular) ya consideradas, completan
el llamado cuadrado lgico de oposicin entre proposiciones categricas.

10. Algunos animales trepan a los rboles.


Solucin: 9x (Ax ^ T x)

11. Los hombres no son siempre pobres.


Solucin: 9x (Hx^ P x) (H. hombre; P : pobre)

12. Las mordeduras de serpiente son a veces mortales.


Solucin: 9x (Sx ^ M x) (S: serpiente; M : mordedura mortal)

13. Algunas serpientes no son venenosas y su mordedura no es mortal.


Solucin: 9x (Sx^ V x^ M x) (V : venenosa)

14. El catarro comn nunca es mortal.


Solucin: 8x (Cx ) M x) (C: catarro comn)

15. Un joven seal con su dedo a la emperatriz.


Solucin: 9x (Jx ^ Ex) (J: joven, E: seal a la emperatriz)

16. Ningn hombre es totalmente optimista.


Solucin: 8x (Hx ) Ox) (H: hombre; O: totalmente optimista)
116 CAPTULO 3. NOCIONES DE LGICA, CONJUNTOS Y RELACIONES

17. Los camellos no valen para animales falderos.


Solucin: 8x (Cx ) F x) (C: camello; F : animal faldero)

18. Cuando aborrezco a un insecto, lo hago apartarse de mi camino.


Solucin: 8x (Ax ) Cx) (A: aborrezco a un insecto; C: lo aparto de mi camino)

19. No slo he fracasado como ser humano, he fracasado tambin como ladrn, puesto que estoy en la crcel.
Solucin: Cy ) Hy ^ Ly (C: estoy en la crcel; H: he fracasado como hombre; L: he fracasado como
ladrn; y: yo). Obsrvese la ausencia de variables de individuo y, por tanto, de cuanticadores!

20. O las teoras de la ciencia gentica son ciertas y las del ocultismo errneas, o las teoras del ocultismo son
ciertas y las de la ciencia gentica errneas.
Solucin: 8x (((M x ) Cx) ^ (Ox ) Ex)) _ ((Ox ) Cx) ^ (M x ) Ex))) (M : t.a ciencia gentica; O:
t.a ocultista; C: cierta; E: errnea)

21. El que duda est acertado.


Solucin: 8x (Dx ) Ax) (D: duda; A: acertado)

22. Slo los directores tienen secretarias.


Solucin: 8x (Sx ) Dx) (S: secretaria; D: director)

23. Ningn camin con ms de 5 aos de servicio merece ser reparado si ha sufrido una avera muy grave.
Solucin: 8x ((Cx ^ Dx) ) (Ax ) Rx)) (C: camin; D: tiene, ms de 5 aos; A: ha sufrido una
avera muy grave; R: es reparado)

24. No todas las leyes de la naturaleza son causales.


Solucin: 9x (Lx^ Cx) (L: ley natural; C: causal)

25. Nadie, salvo el ms fuerte, se alzar con el premio.


Solucin: 8x (P x ) F x) (P : logra el premio; F : es el ms fuerte)

26. No es cierto que todas las proposiciones sean de la forma sujeto-predicado.


Solucin: 8x (P x ) Sx), o tambin: 9x (P x^ Sx) (P : proposicin; S: forma sujeto-predicado)

27. Algunos analgsicos son peligrosos slo si se los toma en cantidades excesivas.
Solucin: 9x (Ax ^ (P x ) Ex)) (A: analgsico; P : peligroso; E: cantidad excesiva).

28. Hay estudiantes que son a la vez inteligentes y muy trabajadores.


Solucin: 9x (Ex ^ (Ix ^ T x)

29. Un profe es buen pedagogo si y slo si est bien formado y es mono.


Solucin: 8x(P x ) (Bx , F x ^ M x)) (P : profe; B: buen pedagogo; F : est bien formado; M : es mono).

Practico lo que aprend:

Construir las negaciones de los 29 predicados (proposiciones) anteriores y formalizarlas.

Formalizacin en lenguaje de predicados relativos:

Ejemplos:

1. Todo peridico apoya a un candidato.


Solucin: 8x (P x ) 9y (Cy ^ Axy)) (P : peridico; C: candidato; Axy: x apoya a y)
3.1. NOCIONES DE LGICA 117

2. Todo candidato es apoyado por un peridico.


Solucin: 8x (Cx ) 9y(P y ^ Axy))
Ntese que la lgica ordinaria prescinde de las variantes retricas del lenguaje y no formaliza de modo
distinto la voz activa y la pasiva.

3. Hay un candidato que no est apoyado por ningn peridico.


Solucin: 9x (Cx ^ 8y (P y ) Axy))

4. Ningn peridico apoya a todos los candidatos.


Solucin: 9x(P x ^ 8y (Cy ) Axy))

5. Todo el mundo conoce a alguien.


Solucin: 8x (P x ) 9y (P y ^ Cxy) (P : persona; Cxy: x conoce a y)

6. Alguien conoce a todo el mundo.


Solucin: 9x (P x ^ 8y (P y ) Cxy)) o tambin: 9x(P x^ 9y(P y^ Cxy)

7. Todo el mundo conoce a todo el mundo.


Solucin: 8x (P x ) 8y (P y ) Cxy))

8. Alguien conoce a alguien.


Solucin: 9x (P x ^ 9y (P y ^ Cxy))

9. No todo el mundo conoce a todo el mundo.


Solucin: 8x (P x ) 8y (P y ) Cxy))

10. Nadie conoce a todo el mundo.


Solucin: 8x (P x ) 9y (P y^ Cxy)) o tambin: 9x(P x ^ 8y(P y ^ Cxy))

11. Jpiter es mayor que la Tierra.


Solucin: M jt (j: Jpiter; t: Tierra; M : mayor que)

12. Algo es mayor que la Tierra.


Solucin: 9x M xt

13. Algo es mayor que algo.


Solucin: 9x9y M xy

14. Todos leen libros.


Solucin: 8x9y (Ay ^ Lxy) (Universo: personas. A: libro; Lxy: x lee y)

15. Todos leen algo.


Solucin: 8x9y Lxy

16. Alguien lee todo.


Solucin: 9x8y Lxy

17. Juan no lee un libro.


Solucin: 9x (Lx^ Ljx)

18. Lucio Gutirrez fue Presidente de Ecuador.


Solucin: P ge (P : presidente; g: Lucio Gutirrez; e: Ecuador.)
118 CAPTULO 3. NOCIONES DE LGICA, CONJUNTOS Y RELACIONES

19. La Torre Pisa no es tan alta como el Empire State Building.


Solucin: Aps (A: es tan alta como; p: Torre Pisa; s: Empire State Building)

20. Imbabura est situada entre Carchi y Pichincha.


Solucin: Sicp (S: estar situada entre)

21. Alguien lee La Celestina.


Solucin: 9x Lxc

22. Todo el mundo quiere a alguien.


Solucin: 8x 9y Qxy

23. Hay quien quiere a todo el mundo.


Solucin: 9x 8y Qxy

24. Hay quien no quiere a nadie.


Solucin: 9x 9y Qxy

25. Adn es el padre de Can.


Solucin: P ac

26. Alguien fue el padre de Can.


Solucin: 9x P xc

27. Slo los malvados y los locos son capaces de torturar a un nio.
Solucin: 8x 8y (N x ^ T yx ) M y _ Ly) (N : nio; M : malvado; L: loco; T xy: x tortura a y)

28. Los hombres buenos cuentan con amigos.


Solucin: 8x ((Hx ^ Bx) ) 9y (Hy ^ Ayx)) (H: hombre; B: bueno; Axy: x es amigo de y)

29. Todo lo que contenga hierro es atrado por el imn.


Solucin: 8x (Hx ) 8y(Iy ) Ayx)) (H: hierro; I: imn; Axy: x atrae a y)

30. Los muertos no cuentan historias.


Solucin: 8x 8y ((M x ^ Hy) ) Cxy) (M : muerto; H: historia; Cxy: x cuenta y)

31. Todo hombre desea algo.


Solucin: 8x (Hx ) 9y Dxy) (H: hombre; Dxy: x desea y)

32. Todo hombre desea a alguna mujer.


Solucin: 8x (Hx ) 9y (M y ^ Dxy))

33. Todo hombre desea a la mujer de otro.


Solucin: 8x (Hx ) 9y9z (M y ^ Hz ^ Eyz ^ Dxy)) (H: hombre; M : mujer; Exy: x es esposa de y;
Dxy: x desea a y)

34. Ernestina tiene un amante al que ella no quiere.


Solucin: 9x (Axe^ Qex) (Axy: x es amante de y; Qxy: x quiere a y)

35. Los que quieren a los que quieren a Ernestina no quieren a Ernestina.
Solucin: 8x 8y (Qye ^ Qxy ) Qxe)
3.1. NOCIONES DE LGICA 119

36. Ernestina no quiere a todos los que la quieren.


Solucin: 8x (Qxe ) Qex)

37. Dios slo ayuda a aquellos que se ayudan a s mismos.


Solucin: 8x (P x ^ Axx ) Adx) (P : persona; Axy: x ayuda a y; d: Dios)

38. Los candidatos honestos siempre son derrotados por los deshonestos.
Solucin: 8x ((Cx ^ Hx ) 9y ((Cy^ Hx ^ Dxy)) (C: candidato; H: honesto; Dxy: x es derrotado
por y)

39. Los barberos afeitan a todos aquellos que no se afeitan a s mismos.


Solucin: 8x (Bx ) 8y ( Ayy ) Axy)) (B: barbero; Axy: x afeita a y)

40. Los barberos que no se afeitan a s mismos son afeitados por alguien que es barbero.
Solucin: 8x (Bx^ Axx ) 9y (By ^ Ayx)).

Practico lo que aprend:

Construir las negaciones de los 40 predicados (proposiciones) anteriores y formalizarlas.

3.1.5. Retculos. lgebra de Boole

Dadas dos leyes de composicin, _ (cup) y ^ (cap) sobre un conjunto C, diremos que la terna (C, _,
^) es un retculo si 8a, b, c 2 C se cumple:

1. Idempotencia: a _ a = a, a ^ a = a

2. Asociativa: a _ (b _ c) = (a _ b) _ c, a ^ (b ^ c) = (a ^ b) ^ c

3. Conmutativa: a _ b = b _ a, a ^ b = b ^ a

4. Simplicacin: a _ (b ^ a) = a, a ^ (b _ a) = a
Retculo distributivo Un retculo (C, _, ^) se llama distributivo si cumple las leyes:

5. Distributivas: a ^ (b _ c) = (a ^ b) _ (a ^ c), a _ (b ^ c) = (a _ b) ^ a _ c

lgebra de Boole Si en un retculo distributivo (C, _, ^) existen los elementos:

6. Neutros: 0 para la _ : a _ 0 = a, 8a 2 C; 1 para la ^ : a ^ 1 = a, 8a 2 C

7. Complementario (de uno dado): 8a 2 C, 9a 2 C, tal que a _ a = 1, a ^ a = 0, entonces se dice que (C,
_, ^) es un lgebra de Boole.

Ejemplos de lgebras de Boole:

Interruptores en serie y en paralelo. Si en un circuito elctrico tenemos dos interruptores en serie, A y B, se


observa que cuando los 2 estn cerrados el circuito tambin lo est, en cuyo caso se enciende la bombilla. En
cualquier otro caso el circuito est abierto, por lo que la bombilla permanece apagada. Si indicamos con 1 el
120 CAPTULO 3. NOCIONES DE LGICA, CONJUNTOS Y RELACIONES

hecho de que el interruptor o el circuito est cerrado, y con un 0 el caso contrario, representando por A ^ B la
A B A^B
1 1 1
conexin de ambos interruptores se tendr la siguiente tabla segn los distintos casos: 1 0 0
0 1 0
0 0 0

Figura 3.3: Puerta AND.

Ahora, si los interruptores A y B estn en paralelo, el circuito est abierto, y la bombilla permanecer
apagada cuando ambos interruptores estn abiertos, en otro caso el circuito estar cerrado y la bombilla se
A B A_B
1 1 1
encender. Indicamos con A _ B la conexin en paralelo con la siguiente tabla de valores: 1 0 1
0 1 1
0 0 0

Figura 3.4: Puerta OR.

Si se considera un interruptor que funcione justamente al revs de los dems, o sea que al levantar la palanca,
A A
entonces quede cerrado. A un interruptor de este tipo le corresponde la tabla: 1 0
0 1

Observacin. El conjunto de todos los interruptores con estas operaciones de conexin: _, ^, tiene estruc-
tura de lgebra de Boole.
3.1. NOCIONES DE LGICA 121

Figura 3.5: Puerta NOT.

Leyes del lgebra de Boole


Idempotencia: A _ A = A; A ^ A = A
Asociativa: A _ (B _ C) = (A _ B) _ C; A ^ (B ^ C) = (A ^ B) ^ C
Conmutativa: A _ B = B _ A; A ^ B = B ^ A
Simplicacin: A _ (A ^ B) = A; A ^ (A _ B) = A
Distributiva: A _ (B ^ C) = (A _ B) ^ (A _ C); A ^ (B _ C) = (A ^ B) _ (A ^ C)
Neutros: A _ 0 = A; A ^ 1 = A
Absorcin: A _ 1 = 1; A ^ 0 = 0
Complemento: A _ A = 1; A ^ A = 0
De Morgan: (A _ B) = A ^ B; (A ^ B) = A _ B
Doble negacin: A = A; 0 = 1

Nota. Las lgebras de las partes de un conjunto, as como la de proposiciones son, tambin, ejemplos de
lgebras de Boole.

Ejemplos:

1. Encontrar la expresin correspondiente al circuito. Ver gura.


y hallar su expresin ms simple utilizando las leyes del lgebra de Boole.
Solucin: La expresin correspondiente al circuito descrito es:
distributiva distributiva
(A ^ B) _ (A ^ B) = ((A ^ B) _ A) ^ ((A ^ B) _ B) =
complemento
= ((A _ A) ^ (B _ A)) ^ ((A _ B) ^ (B _ B) =
idempotente simplicacin simplicacin
= (1 ^ (B _ A)) ^ ((A _ B) ^ B) = (B _ A) ^ B = B
Por tanto, el circuito equivalente ms simple es. Ver gura correspondiente.

2. Sean A, B, C subconjuntos de E. Con A se indica el conjunto complementario de A. Se dene la operacin


" " por: A B = A [ B.

a) Probar que las tres expresiones siguientes son iguales a E:

1) A (B A); 2) (A (B C)) ((A B) (A C); 3) (B A) ((B A) B)

Solucin:
1) A (B A) = A (B [ A) = A [ (B [ A) =
= (A [ B) [ A (B [ A) [ A = B [ (A [ A) = B [ E = E
2) (A (B C)) ((A B) (A C)) = (A [ (B [ C)) ((A [ B) (A [ C)) =
122 CAPTULO 3. NOCIONES DE LGICA, CONJUNTOS Y RELACIONES

Figura 3.6: Ejemplo 1.

Figura 3.7: Circuito equivalente al del ejemplo 1.

= (A [ (B [ C)) [ ((A [ B) [ (A [ C) = (A \ (B [ C)) [ ((A \ B) [ (A [ C)) =


= (A \ (B \ C)) [ ((A \ B) [ (A [ C)) = ((A \ (B \ C)) [ (A \ B)) [ (A [ C) =
= (((A \ (B \ C)) [ A) \ ((A \ (B \ C)) [ B)) [ (A [ C) =
= (A \ ((A [ B) \ ((B \ C) [ B))) [ (A [ C) = ((A \ (A [ B)) \ ((B [ B) \ (C [ B))) [ (A [ C) =
= (A \ (E \ (C [ B))) [ (A [ C) = (A \ (C [ B)) [ (A [ C) = ((A \ C) [ (A \ B)) [ (A [ C) =
= (((A \ C) [ A) \ ((A \ C) [ B)) [ (A [ C) = (A \ ((A [ B) \ (C [ B))) [ (A [ C) =
= ((A \ (A [ B)) \ (C [ B)) [ (A [ C) = (A \ (C [ B)) [ (A [ C)
= ((A \ (C [ B)) [ A) [ C = ((A [ A) \ ((C [ B) [ A)) [ C =
= (E \ (C [ B) [ A)) [ C = ((C [ B) [ A) [ C = (C [ (B [ A)) [ C = (C [ C) [ (B [ A) = E
3) (B A) ((B A) B) = (B [ A) ((B [ A) B) = (B [ A) ((B [ A) [ B) =
= (B [ A) [ ((B \ A) [ B) = (B \ A) [ ((B [ B) \ (A [ B)) == (B \ A) [ (E \ (A [ B)) =
= (B \ A) [ (A [ B) = ((B \ A) [ A) [ B == ((B [ A) \ (A [ A)) [ B =
= (B [ A) [ B = (B [ B) [ A = E
b) Interpretando A, B y C como interruptores, representar los circuitos asociados a las tres expresiones
anteriores.
Solucin:
1) A (B A) = A [ (B [ A). Ver gura respectiva.
2) (A (B C)) ((A B) (A C) = ((A \ (B \ C)) [ (A \ B)) [ (A [ C). Ver gura respectiva.
3) B A B A B = B\A [ B \ A [ B . Ver gura.

3. Hallar las expresiones, en trminos de conmutadores, que corresponden a las redes. Ver guras.

Solucin: A _ ((B _ C) ^ (C _ (B ^ D)):


Solucin: ((C _ (A ^ B)) _ ((A _ (B ^ C) ^ A)) ^ C:
3.1. NOCIONES DE LGICA 123

Figura 3.8: Ejemplo 2.1.

Figura 3.9: Ejemplo 2.2.

Figura 3.10: Ejemplo 2.3.

a)

Figura 3.11: Ejemplo 3a.


124 CAPTULO 3. NOCIONES DE LGICA, CONJUNTOS Y RELACIONES

b)

Figura 3.12: Ejemplo 3b.

4. En una habitacin de 3 estudiantes demcratas, se desea instalar un circuito de interruptores para la luz
de modo que sta se encienda solamente cuando lo desee la mayora, es decir, si 2 o 3 de los estudiantes
pulsan su interruptor. Realizar el montaje.

Solucin: Designemos por A, B y C los interruptores asociados a cada uno de los estudiantes y con-
struyamos la tabla de valores correspondiente a los distintos casos:
A B C M
1 1 1 1 A^B^C 1:a
1 1 0 1 A^B^C 2:a
1 0 1 1 A^B^C 3:a
1 0 0 0
0 1 1 1 A^B^C 5:a
0 1 0 0
0 0 1 0
0 0 0 0
Para que se encienda la luz debe darse una cualquiera de las situaciones indicadas en las las: 1:a , 2:a , 3:a
y 5:a

Por tanto, el montaje pedido se obtendr por la conexin en paralelo (_) de los montajes elementales
correspondientes a estas cuatro las, es decir:

M = (A ^ B ^ C) _ (A ^ B ^ C) _ (A ^ B ^ C) _ (A ^ B ^ C) cuyo esquema es, (ver gura).

Figura 3.13: Ejemplo 4.


3.2. NOCIONES SOBRE LA TEORA DE CONJUNTOS 125

Equivalente a este otro ms simple (A ^ B) _ (((A ^ B) _ (A ^ B)) ^ C). Ver gura respectiva.

Figura 3.14: Circuito equivalente al del ejemplo 4.

3.2. Nociones sobre la Teora de Conjuntos

3.2.1. Conjuntos y sus operaciones

En Matemticas, la palabra conjunto (los trminos: agrupacin, reunin, familia, tribu, conglomerado, etc.
suelen considerarse como sinnimos y son utilizados en dependencia del contexto matemtico analizado) es uno
de los trminos bsicos o primitivos no denidos. Se acepta como vlida la idea intuitiva que de l se tiene. Sin
embargo, en la actualidad existe una axiomtica (por ejemplo, las axiomticas de Zermelo-Fraenkel-Skolem, o
de Von Neumann) de la teora de conjuntos totalmente acabada y aceptada4 .

Para dar una base axiomtica a la teora de conjuntos fundada por G. Cantor, evitando las paradojas, E.
Zermelo y A. Fraenkel propusieron los siguientes axiomas5 :

A1 Axioma de extensionalidad : Dos conjuntos teniendo los mismos elementos son iguales.

A2 Axioma de seleccin y unin: Sean R (x; y) una relacin y E un conjunto. Si para todo elemento y de E la
relacin R (x; y) es colectivizante en x, entonces la relacin (9y 2 E) R (x; y) es colectivizante en x.

A3 Axioma del par : Sean x e y. Existe un conjunto y uno solo cuyos elementos son x e y; se lo nota mediante
fx; yg.

A4 Axioma de las partes: Sea x un conjunto. La relacin y x es colectivizante en y.

A5 Axioma de eleccin: Para toda aplicacin f de un conjunto A en el de las partes de un conjunto B tal
que, para todo elemento a de A, f (a) es no vaco, existe una aplicacin G de A en B tal que para todo
elemento a de A, G (a) pertenece a f (a).

A6 Axioma de regularidad : Para todo conjunto no vaco x, existe un elemento y de x tal que y \ x = ;. Este
axioma tiene por consecuencia que ningn conjunto es elemento de si mismo.

A7 Axioma del innito: Existe un conjunto x tal que el conjunto vaco pertenece a x y que para todo elemento
y de x, y [ fyg es tambin un elemento de x.
4 Lo bsico sobre conjuntos encontramos en [14].
5 Se recomienda una nueva lectura y apreciacin de los presentes axiomas luego de leer todo el presente captulo.
126 CAPTULO 3. NOCIONES DE LGICA, CONJUNTOS Y RELACIONES

Se desprende de estos ejemplos que un axioma no es una proposicin evidente por si misma, sino cierto
enunciado puesto a priori. As, por ejemplo, el axioma de eleccin ha sido rechazado por algunos de los ms
grandes matemticos de principios del siglo XX. Existen corrientes matemticas que no lo aceptan.

Figura 3.15: Izq. El creador de la teora de los conjuntos, de los conjuntos numerables e innitos, de los nmeros
transnitos, el clebre y genial matemtico y lsofo alemn Georg Cantor (San Petersburgo, Rusia, 1845 - Halle, 1918).
Hizo admirables estudios sobre los nmeros algebraicos y trascendentes, series trigonomtricas, ordinales transnitos y
topologa de los nmeros reales. Su teora conjuntista fue incomprendida por muchos colegas matemticos e injustamente
atacada por su profesor, el banquero y matemtico alemn Leopold Kronecker (Liegnitz, 1823 - Berlin, 1891) y otros.
Estos ataques y sus continuas depresiones le condujeron a problemas mentales hasta que muri en un hospital psiquitrico.
Der. El conocido matemtico alemn, Gustav Lejeune Dirichlet (Duren, 1805 - Gotinga, 1859), profesor de Riemann
y sucesor de Gauss en Gotinga. Gran especialista en funciones (introdujo una de las deniciones de funcin), series
numricas y de funciones, teora de nmeros y ecuaciones de la fsica matemtica.

Elementos: Los objetos que integran un conjunto se llaman elementos de ese conjunto. Para indicar que
un objeto aes elemento de un conjunto C, se escribe a 2 C, que se lee a pertenece a C. Si por el contrario,
el objeto a no es elemento del conjunto C, se escribe a 2
= C, leyndose a no pertenece al conjunto C. Si los
objetos a, b, c son de C, escribiremos a, b, c 2 C.

Un conjunto C se podr dar de dos formas:

1. Por extensin (o explcitamente): indicando cada uno de los elementos que lo integran. Por ejemp-
lo: el conjunto de cifras C = f0; 1; 2; 3; 4; 5; 6; 7; 8; 9g; el conjunto de las capitales de provincia V =
fEsmeraldas, Portoviejo, Quito, Guayaquil, Tulcn, . . . , Tena, Nueva Lojag.

2. Por comprensin (o implcitamente): indicando las propiedades caractersticas que cumplen todos sus
elementos, esquemticamente se pone C = fx : x cumplen las propiedades P1 ; P2 ; :::; Pn g. Por ejemplo,
C = fx es natural: 5 < x < 11g = f6; 7; 8; 9; 10g; M = fx es real: x2 < 1g = [ 1; 1]:

Tradicionalmente los conjuntos se designan con letras maysculas y los elementos con minsculas.

D escrip cin del conjunto


z
8 }| 9{
>
< >
=
C
|{z} = x
|{z} :
|{z} P (x)
>
:Elem entos del conjunto | {z } >
;
N ombre del conjunto Tales que Propiedades de los elem entos

Diagramas de Euler - Venn Dado uno o varios conjuntos, se trata de dibujar un recinto plano, cerrado y
de forma arbitraria y suponer que representa, de alguna forma, al conjunto o conjuntos dados. Representacin
que puede ser muy til para visualizar un problema conjustista (sobre la unin, interseccin, diferencia, etc.
entre conjuntos) e incluso para indicar la tcnica a emplear para resolverlo (ver Figura 3.16).
3.2. NOCIONES SOBRE LA TEORA DE CONJUNTOS 127

Subconjuntos o partes. Algunas operaciones entre conjuntos

A continuacin describimos las operaciones fundamentales entre conjuntos y sus diagramas de representacin
o diagramas de Euler-Venn, representados en las dos guras siguientes.

Figura 3.16: Diagrama de Euler - Venn

Subconjuntos o partes Dado un conjunto C y una propiedad P de un cierto elemento x de C, los


elementos de C que poseen dicha propiedad forman un nuevo conjunto S: S = fx 2 C : x verican P g.
As, diremos que S es un subconjunto de C. Anlogamente, dados dos conjuntos A y B, se dice que A es un
subconjunto de B si todos los elementos de A lo son de B. La relacin entre un conjunto y sus subconjuntos se
dice que es la de inclusin (de stos en aqul), escribindose: S C, A B, y leyndose: S est contenido
en C, A est contenido en B.

Si C = fletras del alfabetog = fa; b; c; : : : ; x; y; zg, entonces S = fvocalesg = fa; e; i; o; ug es una parte
o subconjunto de C. Si C = fnmeros naturalesg = f1; 2; 3; : : :g, entonces S = fn : n es mltiplo de 7g =
f7; 14; 21; : : :g es una parte o subconjunto de C:

Conjunto vaco Podemos considerar como conjunto a un conjunto que no tenga ningn elemento. Se lo
llama conjunto vaco y se lo representa con ;. El conjunto vaco ; es siempre una parte o subconjunto de cualquier
conjunto. As, por ejemplo, los conjuntos A = n nmero natural : n2 = 9 y B = fla letra t en la palabra Ecuadorg
son conjuntos vacos porque no existen naturales cuyos cuadrados sean iguales a 9 y porque la palabra Ecuador
no contiene la letra t.

Subconjuntos propios Un subconjunto S de C es propio, si S 6= C, y S 6= ;.

Conjunto de las partes Dado un conjunto C tiene especial inters el conjunto de todas las partes o
subconjuntos de C. Dicho conjunto es aquel cuyos elementos son todos los subconjuntos de C. Se designa por
P(C): P(C) = fS : S Cg. El conjunto vaco ; y todo C tambin son partes de C. Si, por ejemplo,
C = fa; b; cg, entonces el conjunto de todas las partes (o subconjuntos de C) P(C) estara compuesto de: el
vaco (;), los subconjuntos de un solo elemento (fag ; fbg ; fcg), los subconjuntos compuestos de dos elementos
(fa; bg ; fa; cg ; fb; cg), el subconjunto compuesto por tres elementos (fa; b; cg = C). De esta manera se ve que
C posee 8 subconjuntos o partes: P(C) = f;; fag ; fbg ; fcg ; fa; bg ; fa; cg ; fb; cg ; fa; b; cgg. Se puede demostrar
que si el conjunto C posee n elementos, entonces el conjunto de las partes P(C) contendr 2n subconjuntos,
incluyendo al subconjunto vaco ; y a todo el conjunto C. La cuenta de las partes de C se la hace incluyendo
todas las partes compuestas de 0 elementos (el vaco), las partes que contienen un solo elemento, las partes
que contienen 2 elementos, . . . , las partes que contienen n 1 elementos, y nalmente la parte que contiene n
elementos (todo C).

Conjunto universal Todos los conjuntos que intervengan en un cierto problema se considerarn como
subconjuntos de un conjunto U o C, llamado universo de referencia o conjunto universal.

Igualdad de conjuntos Dos conjuntos A y B diremos que son iguales: (A = B) siempre y cuando
(A B) ^ (B A) : En caso contrario escribiremos A 6= B.

Unin de conjuntos Se llama unin de los conjuntos o partes A y B, al nuevo conjunto de todos los
128 CAPTULO 3. NOCIONES DE LGICA, CONJUNTOS Y RELACIONES

elementos que pertenecen al menos a uno de los dos conjuntos. Se denota mediante A [ B: A [ B = fx :
(x 2 A) _ (x 2 B)g:

Interseccin de conjuntos Se llama interseccin de dos conjuntos o partes A y B, al nuevo conjunto


formado por los elementos comunes a A y a B, designndose por A \ B: A \ B = fx : (x 2 A) ^ (x 2 B)g:

Conjuntos disjuntos Se dice que dos conjuntos o partes A, B son disjuntos si A \ B 6= ;. La unin
de una familia nita o innita Ai , i 2 I, de conjuntos o partes disjuntas se suele representar con t Ai , siendo
i2I
evidente que \ Ai = ;, donde I es un conjunto de ndices, nito o innito.
i2I

Conjunto complementario Dado un conjunto C y su subconjunto S. Se llama conjunto complementario


de S en C o respecto de C al conjunto S 0 (se lee complemento de S ) o SC (se lee complemento de S con
respecto a C ), denido mediante: S 0 = SC = fx 2 C : x 2
= Sg:

Operaciones entre conjuntos: Partes o subconjuntos: A; B; C; D U . Inclusin de un conjunto en


otro: B A, A B. Unin de conjuntos: A [ B. Interseccin de conjuntos: A \ B. Diferencia
entre conjuntos: AnB, BnA. Diferencia simtrica entre conjuntos:
A M B = (AnB) [ (BnA) = (A [ B) n (A \ B). Complemento de un conjunto: B C = B 0 = U nB.
Al conjunto universo lo representamos con U y a un elemento cualquiera o representante con x:

Suma de conjuntos Si dos conjuntos A y B son disjuntos, a su unin se le llama conjunto suma de A con
B, anotndose con A + B. As pues, A + B = C , A \ B = ?, A [ B = C.

Diferencia de conjuntos Se llama diferencia de dos conjuntos o partes A y B, en ese orden, al nuevo
conjunto formado por los elementos pertenecientes a A y no pertenecientes a B, designndose por A n B o
A B: A n B = A B = fx : (x 2 A) ^ (x 2 = B)g. Es evidente que la diferencia A n B representa al conjunto
de elementos que estn en A y no petenecen a B, es decir, de los elementos que forman el complemento de B
con respecto a A.
3.2. NOCIONES SOBRE LA TEORA DE CONJUNTOS 129

Diferencia simtrica de conjuntos Se llama diferencia simtrica de dos conjuntos o partes A y B, en ese
orden, al nuevo conjunto formado por los elementos pertenecientes a A y B, pero no pertenecientes a ambos. Se
designa mediante A M B: A M B = (A [ B)n(A \ B) = fx : (x 2 A [ B)^(x 2 = A \ B)g. La diferencia simtrica
entre los conjuntos A y B suele denirse tambin mediante A M B = (A n B)[(B n A) = fx : (x 2 A)Y(x 2 B)g.

Recuerda las operaciones y relaciones entre conjuntos: Descripcin de la operacin o relacin:


Conjunto universo U : A; B; C; : : : U A; B , etc. son subconjuntos de U
Relacin de inclusin o subconjuntos:
A est contenido en B
A B o B A
Conjuntos disjuntos: A \ B = ; No poseen elementos comunes
Complemento: A0 es el conjunto complemento de A,
A0 = AC = fx : (x 2 = A) ^ (x 2 U )g y A [ A0 = U
Igualdad entre conjuntos:
A = B , sis se contienen mutuamente
A = B , (A B) ^ (B A)
Unin de conjuntos: A [ B es el conjunto que contiene
A [ B = fx : (x 2 A) _ (x 2 B)g a todos los elementos de A y de B
Interseccin de conjuntos: A \ B es el conjunto que contiene s-
A \ B = fx : (x 2 A) ^ (x 2 B)g lo los elementos comunes a A y a B
Diferencia entre conjuntos: A n B es el conjunto que contiene s-
A n B = A B = fx : (x 2 A) ^ (x 2 = B)g lo los elementos de A, pero no de B
Diferencia simtrica: A 4 B es el conjunto que contiene
A 4 B = (A [ B) n (A \ B) = los elementos de A y de B pero que
(A n B) [ (B n A) = fx : (x 2 A) Y (x 2 B)g no estn en su interseccin
A t B contiene los elementos no co-
Suma directa de conjuntos: A u B = A t B
munes a A y a B , es decir A \ B = ;

Leyes del lgebra de conjuntos. Particin. Producto cartesiano

En la siguiente tabla presentamos las principales leyes operativas entre conjuntos, donde U representa el
conjunto universo o referencial; ; es el conjunto vaco; A, B, C son subconjuntos de U ; [ representa la unin
de conjuntos; \ la interseccin de conjuntos; A0 = AC es el complemento de A.

Particin de un conjunto Dado un conjunto A y una coleccin de subconjuntos de l: A1 , A2 ,:::, An ,


siendo cada uno distinto del ?, diremos que dicha coleccin es una particin de A (ver gura) si se verica:

1) Ai \ Aj = ?, 8i 6= j; i, j = 1; 2; :::; n. 2) A1 [ A2 [ ::: [ An = A.

Par ordenado Si x e y son dos objetos matemticos, el objeto (x; y) se llama par ordenado de primera
componente x y segunda componente y. Por denicin (x; y) = (x0 ; y 0 ) signica que x = x0 e y = y 0 .

Producto cartesiano Sean A y B, dos conjuntos no vacos, llamaremos producto cartesiano de A por B, al
conjunto de todos los pares ordenados de primera componente en A y segunda componente en B, lo anotaremos
con A B; por tanto: A B = f(x; y) : x 2 A; y 2 Bg.

Anlogamente se dene A A = A2 = f(x; y) : x 2 A; y 2 Ag, A B C = f(x; y; z) : x 2 A; y 2 B; z 2 Cg,


y A1 A2 An = f(x; y; : : : ; t) : x 2 A1 ; y 2 A2 ; : : : ; t 2 An g, A A A = An = f(x; y; : : : ; t) :
x; y; : : : ; t 2 Ag
130 CAPTULO 3. NOCIONES DE LGICA, CONJUNTOS Y RELACIONES

Recuerda las leyes del lgebra de conjuntos


Leyes: Simbologa:
Leyes de idempotencia A [ A = A, A \ A = A
Leyes asociativas A [ (B [ C) = (A [ B) [ C, A \ (B \ C) = (A \ B) \ C
Leyes conmutativas A [ B = B [ A, A \ B = B \ A
Leyes de simplicacin A [ (B \ A) = A, A \ (B [ A) = A
Leyes de absorcin A [ U = U, A \ ? = ?
Leyes distributivas A [ (B \ C) = (A [ B) \ (A [ C), A \ (B [ C) = (A \ B) [ (A \ C)
Leyes de identidad A [ ? = A, A \ U = A
0
Leyes de complemento A [ A0 = U , A \ A0 = ;, (A0 ) = A, U 0 = ;, ;0 = U
Leyes de De Morgan (A [ B)0 = A0 \ B 0 , (A \ B)0 = A0 [ B 0
Dada una identidad en el lgebra de conjuntos, si intercambiamos [ y \,
Ley de dualidad e intercambiamos tambin U y ;, la nueva identidad resultante, se llama
dual de la primera y es igualmente cierta.

Particin del conjunto A: Ai \ Aj = ?, 8i 6= j; i, j = 1; 2; :::; n.


A1 [ A2 [ ::: [ An = A. El conjunto A se descompone en partes disjuntas.

Ejemplos:

1. Dados tres conjuntos A, B y C, tales que: A [ B A[C y A\B A \ C: Qu se puede decir de los
conjuntos B y C?
Solucin: El que A [ B A [ C implica que la parte de B que no est contenida en A, debe estar
contenida en C.
En efecto, si x 2 B pero x 2
= A ) x 2 A [ B, pero por hiptesis

A[B A [ C ) x 2 A [ C; x2
=A)x2C

El que A \ B A \ C implica, tambin, que la parte de B contenida en A est contenida en C.


3.2. NOCIONES SOBRE LA TEORA DE CONJUNTOS 131

En efecto, si x 2 B y x 2 A ) x 2 A \ B A\C C ) x 2 C.
Luego, si x 2 B (tanto si x 2 A como si x 2
= A) ) x 2 C. En consecuencia, podemos decir que B C.

2. Simplicar la expresin (((A [ B) [ C) \ A) \ (((B [ C) \ (B 0 \ C 0 )) [ A)


Solucin: Recordando las leyes fundamentales del lgebra de conjuntos, es fcil seguir el proceso de
simplicacin siguiente:
aso ciativa
#
(((A [ B) [ C) \ A) \ (((B [ C) \ (B 0 \ C 0 )) [ A0 ) =
sim plicacin D e M organ
# #
= ((A [ (B [ C)) \ A) \ (((B [ C) \ (B 0 \ C 0 ) [ A0 ) = A \ (((B [ C) \ (B 0 \ C 0 )) [ A0 ) =
com plem ento elem ento neutro com plem ento
# # #
0 0 0 0
= A \ (((B [ C) \ (B [ C) ) [ A ) = A \ (? [ A ) = A\A = ?

3. Simplicar la expresin (((A [ B)0 \ C 0 ) \ (B [ (A [ B)0 )) [ ((A \ B)0 [ A0 )


Solucin:
conmutativa
#
(((A [ B 0 ) \ C 0 ) \ (B [ (A [ B)0 )) [ ((A \ B)0 [ A0 ) =
D e M organ
#
= ((C 0 \ (A [ B)0 ) \ (B [ (A [ B)0 )) [ ((A0 [ (B 0 )0 ) [ A0 )
aso ciativa y com plem ento
#
= (C 0 \ ((A [ B)0 \ (B [ (A [ B)0 ))) [ (A0 [ B) [ A0 )
aso ciativa y sim plicacin conmutativa y aso ciativa
# #
= (C 0 \(A[B)0 )[(A0 [(B[A0 )) = (C 0 \(A[B)0 )[(A0 [A0 )[B)
D e M organ e idem p otente conmutativa, aso ciativa
# #
0 0 0 0
= (C \ (A \ B )) [ (A [ B) = (((C 0 \ B 0 ) \ A0 ) [ A0 ) [
sim plicacin
#
B = A0 [ B:

4. Demostrar las identidades:

a) (A [ U ) \ (A \ ?) = ?. Encontrar su identidad dual.


Solucin: (A [ U ) \ (A \ ?) = U \ ? = ?. Su dual: (A \ ?) [ (A [ U ) = ? [ U = U
b) ((A [ U ) \ A0 ) [ ((B \ C) [ A) = U . Encontrar su identidad dual.
Solucin:
((A [ U ) \ A0 ) [ ((B \ C) [ A) = (U \ A0 ) [ ((B \ C) [ A) =
= A0 [ ((B \ C) [ A) = A0 [ (A [ (B \ C) = U [ (B \ C) = U
Su dual: [(A \ ?) [ A0 ] \ [(B [ C) \ A] = ?

5. Sea U = f1; 2; 3; 4; 5; 6; 7; 8; 9; a; b; c; d; e; f; gg, el conjunto universal. En l consideramos los subconjuntos:


A = f1; 2; 4; 6; 8; b; c; d; f g; B = f1; 4; 7; a; d; gg; C = f3; 5; 9; a; eg. Encontrar:

a) A + C, A + B
Solucin: Para que tenga sentido hablar de suma de dos conjuntos, necesariamente, han de ser
disjuntos. Efectivamente, A y C son disjuntos (A \ C = ?), luego, por denicin, A + C = A [ C =
f1; 2; 3; 4; 5; 6; 8; 9; a; b; c; d; e; f g:
Sin embargo, A y B no son disjuntos, pues A \ B = f1; 4; dg =
6 ?. Por tanto, carece de sentido hablar
de suma de A y B.
b) A A, A B, B A, A C
Solucin: Por denicin de diferencia de dos conjuntos, se tiene:
A A = fx : x 2 A; x 2
= Ag = ?; A B = fx : x 2 A; x 2
= Bg = f2; 6; 8; b; c; f ]
B A = fx : x 2 B; x 2
= Ag = f7; a; gg; A C = fx : x 2 A; x 2
= Cg = f1; 2; 4; 6; 8; b; c; d; f g
132 CAPTULO 3. NOCIONES DE LGICA, CONJUNTOS Y RELACIONES

6. Es posible calcular los elementos del conjunto E, y de sus subconjuntos A y B, sabiendo que: E A=
f2; 9; 13; 18; 20g; E B = f2; 6; 18; 20g; A B = f1; 5; 6; 14g?
Solucin: Para ayudarnos en la resolucin de este problema, trazaremos el diagrama de Venn (ver gura
3.17). En l se irn colocando los elementos en la forma que nos indique el siguiente razonamiento:
El 2 pertenece a E AyaE B; luego, 2 2 E, 2 2
= A, 2 2
= B. As, lo situaremos en E, pero fuera de A
y de B.
El 9 y el 13 son de E A, pero no de E B. Por tanto: 9; 13 2 E; 9; 13 2
= A y 9; 13 2 B. Los incluiremos,
pues, en B y fuera de A.
Al igual que el 2, el 18 y el 20 son de E A y de E B. As pues, los situaremos tambin fuera de A y
de B.
El 6 solamente pertenece a E B, luego no es de B, pero s es de A.
Los elementos de A B son de A y no de B, por eso van en A y fuera de B y de A \ B, as pues 1, 5, 6,
14 2 A. Segn esto, parece ser que podamos armar que:
E = f1; 2; 5; 6; 9; 13; 14; 18; 20g, A = f1; 5; 6; 14g y B = f9; 13g

Figura 3.17: Ejemplo. Conjunto solucin.

Sin embargo, esto no es as, ya que puede suceder que A \ B 6= ?; por ejemplo, si A \ B = f3; 4; 7; 25g.
Se seguiran cumpliendo las condiciones del enunciado: E A = f2; 9; 13; 18; 20g, E B = f2; 6; 18; 20g,
A B = f1; 5; 6; 14g, y, sin embargo, la composicin de los conjuntos E, A y B, sera completamente
distinta; por ejemplo: el universal E, resultara ser el conjunto f1; 2; 3; 4; 5; 6; 7; 9; 13; 14; 18; 20; 25g. Por
tanto, si en el enunciado del problema no se da A \ B, lo nico que podremos obtener sern los conjuntos
E (A \ B), A (A \ B), B (A \ B). As, en nuestro caso, ser:
E (A \ B) = f1; 2; 5; 6; 9; 13; 14; 18; 20g, A (A \ B) = f1; 5; 6; 14g, B (A \ B) = f9; 13g

7. Demostrar que B A0 = B \ A
?
Solucin: Veamos primero que B A0 B \ A.
Si x 2 B A0 ) x 2 B; x 2
= A0 ) x 2 B; x 2 A ) x 2 A \ B ) B A0 B \ A.
?
Anlogamente, B \ A B A0 .
= A0 ) x 2 B
Si x 2 B \ A ) x 2 B; x 2 A ) x 2 B; x 2 A0 ) B \ A B A0 .
Esta doble contencin nos asegura la validez del enunciado.

8. Demostrar que (A [ B) (A \ B) = (A B)[(B A). Resultado ste que recibe el nombre de diferencia
simtrica entre A y B y designado mediante A 4 B.
Solucin:
?
1) Veamos que (A [ B) (A \ B) (A B) [ (B A).
3.2. NOCIONES SOBRE LA TEORA DE CONJUNTOS 133
8
>
< x2A
En efecto: Si x 2 (A [ B) (A \ B) ) x 2 A [ B; x 2
= A\B ) x 2
= A\B y )
>
:
x2B
8
>
< x2
= A \ B; x 2 A ) x 2 A; x 2
=B)x2A B
) x 2 (A B) [ (B A)
>
:
x2
= A \ B; x 2 B ) x 2 B; x 2
=A)x2B A
La otra contencin a demostrar es:
?
2) (A B) [ (B A) (A [ B) (A \ B).
En efecto: Si x 2 (A B) [ (B A) )
8
>
< x 2 A B ) x 2 A; x 2 =B
) ) x 2 A [ B, x 2
= A \ B ) x 2 (A [ B) (A \ B) )
>
:
x 2 B A ) x 2 B; x 2 =A
) (A B) [ (B A) (A [ B) (A \ B).

9. Dar las posibles particiones del conjunto E = f1; 2; 3g.


Solucin: Una particin de un conjunto E es una familia de subconjuntos de l, fAi gi2I tal que:
1) Ai 6= ?, 8i 2 I.
2) Ai \ Aj = ?, si i 6= j.
3) [ Ai = E.
i2I

Por tanto, una posible particin de E es la formada por los conjuntos: A1 = f1g, A2 = f2g, A3 = f3g:
Otras particiones de E sern las siguientes:

B1 = f1g; B2 = f2; 3g; C1 = f2g; C2 = f1; 3g; D1 = f3g; D2 = f1; 2g:

La particin formada por el subconjunto f1; 2; 3g es la impropia.

10. Sea U = f1; 2; 3; 4; 5; 6; 7; 8; 9; a; b; c; d; e; f; gg el conjunto universal. Demostrar que:

a) Los conjuntos: A1 = f1; 5; 9; a; d; gg, A2 = f3; 6; 8; b; cg, A3 = f2; 7; e; f g, A4 = f4g constituyen


una particin de U .
Solucin:
1) Ai 6= ?; 8i = 1; 2; 3; 4:
2) A1 \ A2 = A1 \ A3 = A1 \ A4 = A2 \ A3 = A2 \ A4 = A3 \ A4 = ?.
3) A1 [A2 [A3 [A4 = f1; 5; 9; a; d; g; 3; 6; 8; b; c; 2; 7; e; f; 4g == f1; 2; 3; 4; 5; 6; 7; 8; 9; a; b; c; d; e; f; gg =
U.
b) Los conjuntos: B1 = fa; b; f; 2; 5; 9g, B2 = f1; 2; 4; 6; 7; b; c; e; gg, B3 = fc; d; g; 3; 4; 8g no forman
una particin de U , pero que, no obstante, s la forman los conjuntos: B1 4B2 , B1 \B2 , B3 (B2 \B3 )
Solucin: B1 ; B2 y B no forman una particin de U , ya que, por ejemplo: B1 \ B2 = f2; bg =
6 ?.
Sin embargo, los conjuntos: B1 4 B2 = (B1 [ B2 ) (B1 \ B2 ) = f1; 4; 5; 6; 7; 9; a; c; e; f; gg,
B1 \ B2 = f2; bg, B3 (B2 \ B3 ) = f3; 8; dg si forman particin, puesto que:
1) B1 4 B2 6= ?; B1 \ B2 6= ?; B3 (B2 \ B3 ) 6= ?:
2) (B1 4 B2 ) \ (B1 \ B2 ) = ?, (B1 4 B2 ) \ (B3 (B2 \ B3 )) = ?, (B1 \ B2 ) \ (B3 (B2 \ B3 )) = ?.
3) ((B1 4 B2 ) [ (B1 \ B2 )) [ (B3 (B2 \ B3 )) = (B1 [ B2 )[
[(B3 (B2 \ B3 )) = f1; 2; 4; 5; 6; 7; 9; a; b; c; e; f; gg [ f3; 8; dg = U:
134 CAPTULO 3. NOCIONES DE LGICA, CONJUNTOS Y RELACIONES

11. Mediante formalizacin y tabla de verdad demostrar la igualdad entre conjuntos: A[(B \ C) = (A [ B)\
(A [ C)
Solucin:
(8x) (x 2 (A [ (B \ C))) $ x 2 ((A [ B) \ (A [ C))
x 2 [A [ (B \ C)] $ x 2 [(A [ B) \ (A [ C)]
[x 2 A _ x 2 (B \ C)] $ [x 2 (A [ B) ^ x 2 (A [ C)]
[x 2 A _ (x 2 B ^ x 2 C)] $ [(x 2 A _ x 2 B) ^ (x 2 A _ x 2 C)]
p : x 2 A. q : x 2 B. r : x 2 C
[p _ (q ^ r)] $ [(p _ q) ^ (p _ r)] :
Para construir la tabla de verdad, representamos mediante 0 el valor de Falso (F ) y el de Verdad (V )
con 1:
p q r [p _ (q ^ r)] $ [(p _ q) ^ (p _ r)]
1 1 1 1 1 1 1 1 1 1 1 1 1 1 1 1
1 1 0 1 1 1 0 0 1 1 1 1 1 1 1 0
1 0 1 1 1 0 0 1 1 1 1 0 1 1 1 1
1 0 0 1 1 0 0 0 1 1 1 0 1 1 1 0
0 1 1 0 1 1 1 1 1 0 1 1 1 0 1 1
0 1 0 0 0 1 0 0 1 0 1 1 0 0 0 0
0 0 1 0 0 0 0 1 1 0 0 0 0 0 1 1
0 0 0 0 0 0 0 0 1 0 0 0 0 0 0 0
Como el resultado de la tabla de verdad es una tautologa, entonces la relacin inicial entre conjuntos es
verdadera.
c
12. Mediante formalizacin y tabla de verdad demostrar la igualdad entre conjuntos: A (B [ C) = (A \ B)[
(A \ C)
Solucin:
c
(8x) (x 2 [A (B [ C) ] $ x 2 [(A \ B) [ (A \ C)])
c
x 2 [A (B [ C) ] $ x 2 [(A \ B) [ (A \ C)]
c
[x 2 A ^ x 2
= (B [ C) ] $ [x 2 (A \ B) _ x 2 (A \ C)]
[x 2 A ^ x 2 (B [ C)] $ [(x 2 A ^ x 2 B) _ (x 2 A ^ x 2 C)]
[x 2 A ^ (x 2 B _ x 2 C)] $ [(x 2 A ^ x 2 B) _ (x 2 A ^ x 2 C)]
p : x 2 A. q : x 2 B. r : x 2 C
[p ^ (q _ r)] $ [(p ^ q) _ (p ^ r)]

Tabla de verdad:
p q r [p ^ (q _ r)] $ [(p ^ q) _ (p ^ r)]
1 1 1 1 1 1 1 1 1 1 1 1 1 1 1 1
1 1 0 1 1 1 1 0 1 1 1 1 1 1 0 0
1 0 1 1 1 0 1 1 1 1 0 0 1 1 1 1
1 0 0 1 0 0 0 0 1 1 0 0 0 1 0 0
0 1 1 0 0 1 1 1 1 0 0 1 0 0 0 1
0 1 0 0 0 1 1 0 1 0 0 1 0 0 0 0
0 0 1 0 0 0 1 1 1 0 0 0 0 0 0 1
0 0 0 0 0 0 0 0 1 0 0 0 0 0 0 0
Como el resultado de la tabla de verdad es una tautologa, entonces la relacin inicial entre conjuntos es
verdadera.
3.2. NOCIONES SOBRE LA TEORA DE CONJUNTOS 135

13. Mediante formalizacin y tabla de verdad demostrar la igualdad entre conjuntos: A4(B C) = (A 4 B)
C

Solucin:

(8x) (x 2 [A 4 (B C)] $ x 2 [(A 4 B) C])

x 2 [A 4 (B C)] $ x 2 [(A 4 B) C]

[x 2 A Y x 2 (B C)] $ [x 2 (A 4 B) C]

[x 2 A Y (x 2 B ^ x 2
= C)] $ [(x 2 A Y x 2 B) ^ x 2
= C]

p : x 2 A. q : x 2 B. r : x 2 C
[p Y (q^ r)] $ [(p Y q) ^ r]

Aparentemente esta es la tabla de verdad de la anterior proposicin compuesta. Indicar donde est el
error!
p q r [p Y (q ^ r)] $ [(p Y q) ^ r]
1 1 1 1 1 1 0 0 1 0 1 0 1 0 0 1
1 1 0 1 0 1 1 1 0 1 1 0 1 0 1 0
1 0 1 1 1 0 0 0 1 0 1 1 0 0 0 1
1 0 0 1 1 0 0 1 0 1 1 1 0 1 1 0
0 1 1 0 0 1 0 0 1 1 0 1 1 0 0 1
0 1 0 0 1 1 1 1 0 1 0 1 1 1 1 0
0 0 1 0 0 0 0 0 1 1 0 0 0 0 0 1
0 0 0 0 0 0 0 1 0 1 0 0 0 0 1 0

14. Mediante formalizacin y tabla de verdad demostrar la igualdad entre conjuntos: (A B) (A C) =


A (B C)

Solucin:

8 (x; y) ((x; y) 2 [(A B) (A C)]) $ (x; y) 2 [A (B C)]

[(x; y) 2 (A B) ^ (x; y) 2
= (A C)] $ [x 2 A ^ y 2 (B C)]

[(x 2 A ^ y 2 B) ^ (x 2
= A_y 2
= B)] $ [x 2 A ^ (y 2 B ^ y 2
= C)]

[x 2 A _ (x 2 B ^ x 2 C)] , [(x 2 A _ x 2 B) ^ (x 2 A _ x 2 C)]

p : x 2 A. q : y 2 B. r : y 2 C

[(p ^ q) ^ ( p_ r)] $ [p ^ (q_ r)]

Como el resultado de la siguiente tabla de verdad es una tautologa, entonces la relacin inicial entre
conjuntos es verdadera.
p q r [(p ^ q) ^ ( p _ r)] $ [p ^ (q _ r)]
1 1 1 1 1 1 0 0 1 0 0 1 1 1 1 1 1 0 1
1 1 0 1 1 1 1 0 1 1 1 0 1 1 1 1 1 1 0
1 0 1 1 0 0 0 0 1 0 0 1 1 1 0 0 0 0 1
1 0 0 1 0 0 0 0 1 1 1 0 1 1 0 0 1 1 0
0 1 1 0 0 1 0 1 0 1 0 1 1 0 0 1 1 0 1
0 1 0 0 0 1 0 1 0 1 1 0 1 0 0 1 1 1 0
0 0 1 0 0 0 0 1 0 1 0 1 1 0 0 0 0 0 1
0 0 0 0 0 0 0 1 0 1 1 0 1 0 0 0 1 1 0
136 CAPTULO 3. NOCIONES DE LGICA, CONJUNTOS Y RELACIONES

3.3. Aplicaciones, Correspondencias y Relaciones entre Conjuntos

Introducimos los importantsimos conceptos de relaciones, leyes y correspondencias entre conjuntos.

Par ordenado Si x e y son dos objetos matemticos, el objeto (x; y) se llama par ordenado de primera
componente x y segunda componente y. Por denicin la relacin (x; y) = (x0 ; y 0 ) equivale a x = x0 e y = y 0 .

Ejemplo:

Demostremos que es posible denir el par ordenado (x; y) como el conjunto ffxg; fx; ygg.

Solucin: La propiedad caracterstica del par ordenado (x; y) es que (x; y) = (z; t) , x = z e y = t.
?
Por tanto, se trata de ver que: ffxg; fx; ygg = ffzg; fz; tgg , x = z e y = t:
(
fxg = fzg y fx; yg = fz; tg (1) o
)) Si ffxg; fx; ygg = ffzg; fz; dgg entonces
fxg = fz; tg y fx; yg = fzg (2)

En el caso (1), fxg = fzg ) x = z, y por tanto fx; yg = fz; tg ) fx; yg = fx; tg ) y = z.

En el caso (2), fxg = fz; tg ) x = z = t, y por tanto fx; yg = fzg ) fx; yg = fxg ) x = y. Luego x = y y
z = t.

() Si x = z y y = t, es evidente que ffxg; fx; ygg = ffzg; fz; tgg.

Observacin. Cabe sealar que por ejemplo, (3; 2) es un par ordenado de primera componente x = 3 y
segunda componente y = 2 y que (3; 2) 6= ( 2; 3); en cambio f3; 2g = f 2; 3g es el conjunto formado por
los elementos 2 y 3, tomados en cualquier orden.

Producto cartesiano Producto cartesiano A B de los conjuntos A por B, es el conjunto formado de


todos los pares ordenados de primera componente en A y segunda componente en B:

A B = f(x; y) : x 2 A; y 2 Bg:

As, por ejemplo, si A = fa; b; cg y B = f 1; 2g, entonces

A B = f(x; y) : x 2 A; y 2 Bg = f(a; 1); (a; 2); (b; 1); (b; 2); (c; 1); (c; 2)g:

Observacin. Es evidente que si los conjuntos A y B son nitos y compuestos de m y n elementos,


respectivamente, entonces el nmero de pares ordenados que componen el producto cartesiano A B ser m n.
En este caso suele decirse que el cardinal (nmero de elementos distintos) de A, B y A B es de m, n y mn,
respectivamente, y representar mediante: #A = m, #B = n y # (A B) = mn.

Correspondencias y relaciones Sean A y B dos conjuntos cualesquiera, no vacos, llamaremos relacin


o correspondencia entre los elementos de A y los elementos de B a todo subconjunto R, no vaco, de A B.
Dado un par (a; b) 2 A B, si (a; b) 2 R diremos que a est relacionado con b por R (anotndolo aRb), o
tambin R hace corresponder al elemento a 2 A el elemento b 2 B(anotndolo b = R(a)). As, por ejemplo,
si A = fa; b; cg y B = f 1; 2g, entonces A B = f(a; 1); (a; 2); (b; 1); (b; 2); (c; 1); (c; 2)g y cualquier
subconjunto R A B es una relacin o correspondencia entre los elementos de A y de B. Podemos tomar en
calidad de relacin o correspondencia R a R1 = f(b; 1)g, R2 = f(a; 1); (c; 1); (c; 2)g, etc. o a todo R.

Aplicaciones Sean A y B dos conjuntos y F A B una relacin entre A y B. Se dice que F dene una
aplicacin f de A en B, si para cada x 2 A existe un y 2 B y uno slo tal que (x; y) 2 F . Lo que se indica por
y = f (x). A y se le llama imagende x por f , y a x se le llama una antiimagen o preimagende y por f . As,
por ejemplo, si A = fa; b; cg y B = f 1; 2g, entonces A B = f(a; 1); (a; 2); (b; 1); (b; 2); (c; 1); (c; 2)g y slo
3.3. APLICACIONES, CORRESPONDENCIAS Y RELACIONES ENTRE CONJUNTOS 137

ciertos subconjuntos F A B son aplicaciones entre los conjuntos A y B. Podemos tomar en calidad de apli-
caciones F a F1 = f(b; 1)g, F2 = f(a; 1); (b; 1); (c; 2)g, etc.; sin embargo F3 = f(a; 1); (b; 2); (c; 1); (c; 2)g
ni todo F = A B ya no son aplicaciones porque hay valores de x que poseen 2 valores distintos de y (o
imgenes). Para F2 = f(a; 1); (b; 1); (c; 2)g tenemos, por ejemplo, que la imagen de x = b es y = 1 y la
preimagen de y = 2 es x = 2, pero la preimagen de y = 1 es o bien x = a o bien x = b, es decir, fa; bg.
f
Es muy usual el empleo del diagrama sagital: f : A ! B o A ! B para indicar el funcionamiento
de una aplicacin de A en B. A es el conjunto de salida o conjunto de denicin de f . B es el conjunto de
llegada de f . Como se pudo observar, una aplicacin es un caso especial de relacin o correspondencia.

Recuerda las componentes de una aplicacin:


El dominio de f : Dom f = fx 2 A : existe f (x)g.
La imagen de f se dene con Im f = fy 2 B : existe x 2 A con f (x) = yg = f (A):
El grafo de la aplicacin f se dene mediante F = f(x; y) 2 A B : y = f (x)g:

Imagen recproca de un subconjunto Sea Y un subconjunto de B, la imagen recproca de Y por f se


indica por f 1 (Y ) y es el subconjunto de A denido por: f 1 (Y ) = fx 2 A : f (x) 2 Y g. As, por ejemplo, si
A = fa; b; cg, B = f 1; 2g y A B = f(a; 1); (a; 2); (b; 1); (b; 2); (c; 1); (c; 2)g, entonces para la aplicacin
F2 = f(a; 1); (b; 1); (c; 2)g, tenemos f 1 ( 1) = fa; bg, f 1 (2) = c, f 1 ( 1; 2) = f 1 (B) = fa; b; cg.

Representacin del producto cartesiano. Relaciones Sean A y B representados por puntos de sendas
rectas, el producto A B se representar en la forma cartesianaindicada en la gura. La relacin R A B
se representar con puntos que representan los elementos de R de entre los de A B. As, por ejemplo, para
A B = f(1; b); (2; a) ; (3; c),(4; b)g se tiene la representacin

Producto Cartesiano.

La relacin R A B tambin puede representarse con su representacin sagital en la cual se disponen en


columnas los elementos de A y los de B y se unen mediante una echa, de origen a 2 A y nal en b 2 B, si y
slo si aRb (o sea b = R(a)). En el caso en que B = A, a los elementos de A se les representar por puntos y si
xRy se unir x con y mediante una echa de origen x y nal en y.

Representacin sagital.
138 CAPTULO 3. NOCIONES DE LGICA, CONJUNTOS Y RELACIONES

Esquema de una relacin en A:

3.3.1. Tipos de aplicaciones y operaciones con aplicaciones

Pasamos a clasicar y a caracterizar las aplicaciones e introducimos algunas operaciones entre ellas.

Aplicacin inyectiva A una aplicacin f de A en B se le llama inyectiva si la igualdad f (x) = f (x0 )


implica que x = x0 , para todos x; x0 2 A. O tambin si a elementos distintos de A les corresponden imgenes
distintas en B, es decir, si x1 6= x2 ) f (x1 ) 6= f (x2 ). As, por ejemplo, si A = fa; b; cg, B = f 1; 2g y
A B = f(a; 1); (a; 2); (b; 1); (b; 2); (c; 1); (c; 2)g, entonces la aplicacin F = f(a; 1); (c; 2)g representada
con f es inyectiva.

Aplicacin sobreyectiva o exhaustiva Se dice que una aplicacin f : A ! B, es exhaustiva, sobreyec-


tiva (o epiyectiva) si para cada y 2 B, existe por lo menos un x 2 A tal que: f (x) = y. As, por ejemplo,
si A = fa; b; cg, B = f 1; 2g y A B = f(a; 1); (a; 2); (b; 1); (b; 2); (c; 1); (c; 2)g, entonces la aplicacin
F = f(a; 1); (b; 1); (c; 2)g representada con f es sobreyectiva.

Aplicacin biyectiva Si una aplicacin es exhaustiva e inyectiva, se dice que es biyectiva. Si para cada
y 2 B, y = f (x) ) x 2 A es nico. As, por ejemplo, si A = fa; b; cg, B = f 1; 0; 2g y A B =
|{z}
Im agen
f(a; 1); (a; 0); (a; 2); (b; 1); (b; 0); (b; 2); (c; 1); (c; 0); (c; 2)g, entonces la aplicacin F = f(a; 1); (b; 0); (c; 2)g
representada con f es biyectiva.

Aplicacin recproca Sea f : A ! B una aplicacin biyectiva. La correspondencia F 1 , de B en A,


denida por (y; x) 2 F 1 si y slo si (x; y) 2 F , dene una aplicacin de B en A, llamada recproca de f . Se
denota por f 1 . As, por ejemplo, si A = fa; b; cg, B = f 1; 0; 2g y

A B = f(a; 1); (a; 0); (a; 2); (b; 1); (b; 0); (b; 2); (c; 1); (c; 0); (c; 2)g;

entonces la aplicacin F = f(a; 1); (b; 0); (c; 2)g representada con f es biyectiva y G = f( 1; a); (0; b); (2; c)g
representada con f 1 es la aplicacin recproca.

Suma, diferencia y producto de aplicaciones Sean A y B conjuntos, f y g aplicaciones de A en


B, (f : A ! B, g : A ! B); las aplicaciones S, R, P y Q de A en B (S; R; P; Q : A ! B) denidas
mediante S(x) = (f + g) (x) = f (x) + g (x), R(x) = (f g) (x) = f (x) g (x), P (x) = (f g) (x) = f (x) g (x),
f f (x)
Q(x) = (x) = , 8x 2 A, se denominan suma, diferencia, producto y cociente (g (x) 6= 0, 8x 2 A) de
g g (x)
f
aplicaciones, respectivamente, y se escriben f + g, f g, f g y , respectivamente.
g
Composicin de aplicaciones Sean A, B y C conjuntos, f una aplicacin de A en B, y g una aplicacin
de B en C (f : A ! B, g : B ! C); a la aplicacin h, de A en C (h : A ! C) denida por h(x) = g(f (x)),
8x 2 A, se le denomina aplicacin compuesta de f y g o aplicacin composicin de f con g y se escribe
h = g f . As, por ejemplo, si A = fa; b; cg, B = f 1; 2g, C = fu; v; wg, F = f(a; 1); (b; 1); (c; 2)g
representada con f : A ! B, G = f( 1; w); (2; u); (2; v)g representada con g : B ! C, entonces la aplicacin
H = f(a; w) ; (b; w) ; (c; u) ; (c; w)g representada con h : A ! C es la compuesta, es decir, h = g f que acta de
la siguiente manera h (x) = g (f (x)), x 2 A.
3.3. APLICACIONES, CORRESPONDENCIAS Y RELACIONES ENTRE CONJUNTOS 139

Ejemplos:

1. Sean los conjuntos A = fa; b; c; d; e; f g y B = f1; 2; 3; 4g. Consideremos los subconjuntos R y S del
producto cartesiano A B, dados por:

R = f(a; 1) ; (b; 2) ; (c; 3) ; (d; 4)g ; S = f(a; 1) ; (b; 4) ; (c; 2) ; (d; 3) ; (e; 2) ; (f; 4)g

Denen R y S aplicaciones entre A y B ?. Justicar la respuesta.


Solucin: Estudiemos primero R. Ver la gura 3.18.Es claro que R no representa una aplicacin entre A

Figura 3.18: Ejemplo 1. Relacin R.

y B, pues los elementos e y f de A no tienen imagen.


En cuanto a S, tenemos. Ver gura 3.19.y s que representa una aplicacin de A en B, ya que todo elemento

Figura 3.19: Ejemplo 1. Relacin S.

de A tiene una, y slo una, imagen en B.

2. Sea A = f1; 2; 3; 4g y B = fa; b; c; d; eg. Decir cules de las correspondencias siguientes denidas mediante
sus grafos son aplicaciones y cuales son inyectivas, exhaustivas o biyectivas:

a) G1 = f(1; a) ; (1; b) ; (2; c) ; (2; d) ; (3; e)g


Solucin: G1 no es aplicacin, ya que los elementos 1 y 2 tienen dos imgenes, y, adems, el elemento
4 no tienen ninguna. Ver gura 3.20.
b) G2 = f(1; a) ; (2; e) ; (3; c) ; (4; d)g
Solucin: G2 es aplicacin, pues cada elemento de A, tienen una, y slo una imagen en B. G2
es inyectiva, ya que a elementos distintos de A les corresponde imgenes distintas en B. G2 no es
exhaustiva, puesto que el elemento b 2 B no tiene antiimagen. Por tanto, tampoco ser biyectiva.
Ver gura 3.21.
c) G3 = f(1; a) ; (2; a) ; (3; a) ; (4; a)g
Solucin: G3 es aplicacion. G3 no es inyectiva, pues 1 6= 2 y no obstante G3 (1) = G3 (2) = a. G3
no es exhaustiva, ya que los elementos b; c; d; e de B no tienen antiimagen. Ver gura 3.22.
140 CAPTULO 3. NOCIONES DE LGICA, CONJUNTOS Y RELACIONES

Figura 3.20: Ejemplo 2. Relacin G1 :

Figura 3.21: Ejemplo 2. Relacin G2 .

Figura 3.22: Ejemplo 2. Relacin G3 .


3.3. APLICACIONES, CORRESPONDENCIAS Y RELACIONES ENTRE CONJUNTOS 141

3. Sean los conjuntos A = fa; b; c; d; e; f g, B = f1; 2; 3; 4; 5; 6g, y consideremos las correspondencias R y S


de A en B dadas por:
R = f(a; 1) ; (b; 4) ; (c; 2) ; (d; 3) ; (e; 2) ; (f; 4)g, S = f(a; 4) ; (b; 3) ; (c; 2) ; (d; 1) ; (e; 5) ; (f; 6)g

a) Dar las antiimagenes del 2 por R y S.


1 1
Solucin: R (2) = fc; eg, S (2) = fcg. Ver gura 3.23.

Figura 3.23: Ejemplo 3. Relaciones S y R.

b) Estudiar R y S, viendo si son inyectivas exhaustivas o biyectivas.


Solucin: R y S son aplicaciones. R no es inyectiva por que c 6= e y no obstante R(c) = R(e) = 2.
R tampoco es exhaustiva, pues los elementos 5 y 6 de B no tienen antiimagen en A. S es inyectiva,
puesto que a elementos distintos de A, les corresponde imagenes distintas en B. S es tambin
exhaustiva, pues todo elemento de B tiene al menos una antiimagen en A. Por tanto, S sera tambin
biyectiva.
1
c) Calcular S .
Solucin: S 1 = f(4; a) ; (3; b) ; (2; c) ; (1; d) ; (5; e) ; (6; f )g. Ver gura 3.24. Es claro que S 1
es
tambin una aplicacin biyectiva.

1
Figura 3.24: Ejemplo 3. Relacin S .

1
d ) Obtener y estudiar R S .
Solucin: Obtengamos R S 1 : R S 1 = f(1; 3) ; (2; 2) ; (3; 4) ; (4; 1) ; (5; 2) ; (6; 4)g. Es una apli-
cacin no inyectiva, pues R S 1 (2) = (R S 1 )(5) y tampoco exhaustiva, ya que los elementos
5 y 6 no tienen antiimagen. Ver gura 3.25.

4. Se considera la aplicacin f : R ! R, denida por f (x) = x2 + x 2. Calcular: f (3), f (x 2), f (3) f (2),
f (f ( 2)), f (x + h), f 1 (10), f 1 (4):
Solucin:
f (3) = 32 + 3 2 = 10, f (x 2) = (x 2)2 + (x 2) 2 = x2 4x + 4 + x 2 2 = x2 3x.
142 CAPTULO 3. NOCIONES DE LGICA, CONJUNTOS Y RELACIONES

1
Figura 3.25: Ejemplo 3. Relacin R S .

f (3) f (2) = 10 f (2) = 10 (22 + 2 2) = 10 4 = 6, f (f ( 2)) = (f ( 2)2 ) + (f ( 2)) 2 =


(( 2)2 ) + ( 2) 2)2 +
+(( 2)2 ) + ( 2) 2) 2 = (4 4)2 + (4 4) 2= 2,
f (x + h) = (x + h)2 + (x + h) 2 = x2 + 2hx + h2 + x + h 2 = x2 + (2h + 1)x + h2 + h 2:
1
f (10) = fx 2 R : f (x) = 10g = x 2 R : x2 + x 2 = 10 :
x2 + x 2 = 10 ) x2 + x 12 = (x + 4) (x 3) = 0 ) x1 = 3, x2 = 4:
1
Luego, f (10) = f3; 4g :
Anlogamente f 1 (4) = x 2 R : x2 + x 4 y x2 + x 6 = (x + 3) (x 2) = 0 ) x1 = 2, x2 = 3.
Por lo tanto, f 1 (4) = f2; 3g :

5. Sea f : A ! B una aplicacin y fXi g = fX1 ; X2 ; : : :g una familia de subconjuntos de A. Comprobar si


son ciertas las igualdades:

a) f ([Xi ) = [f (Xi )
Solucin:
?
) f ([Xi ) [f (Xi ): En efecto, si y 2 f ([Xi ) ) 9x 2 [Xi , tal que f (x) = y. Ahora bien, si x 2
[Xi ) x 2 Xi , para cierto j = 1; 2; : : : ) f (x) 2 f (Xj ), y por tanto f (x) = y 2 [f (Xi )( f (Xj )).
?
) f ([Xi ) [f (Xi ). Si y 2 f ([Xi ) ) y 2 f (Xj ), para cierto j = 1; 2; : : : ) 9x 2 Xj , tal que
f (x) = y.
Pero si x 2 Xj ) x 2 [Xi ( Xj ) ) f (x) 2 f ([Xi ).
b) f (\Xi ) = \f (Xi )
Solucin:
?
) f (\Xi ) \f (Xi ). Si y 2 f (\Xi ) ) 9x 2 \Xi . tal que f (x) = y. Ahora bien, si x 2 \Xi ) x 2
Xi , 8i = 1; 2; : : : ) f (x) = y 2 f (Xi ), 8i = 1; 2; : : : ) f (x) = y 2 \f (Xi ).
?
) f (\Xi ) \f (Xi ). Si y 2 \f (Xi ) ) y 2 f (Xi ), 8i = 1; 2; : : : ) 8i = 1; 2; : : :, 9xi 2 Xi , tal
que f (xi ) = y, pero este xi no tiene por que ser el mismo para todos los Xi ; por lo que en general
y2= f (\Xi ) y en consecuencia \f (Xi ) 6 f (\Xi ).
Nota. y 2 f (\Xi ), si existiese x 2 \Xi (x 2 Xi ; 8i = 1; 2; : : :), tal que f (x) = y.
As, pues, en el caso b) nicamente se puede asegurar la contencin: f (\Xi ) \(f (Xi )):

6. Sea f : A ! B una aplicacin; si Y B, se dene la imagen recproca de Y por f como: f 1 (Y ) =


fx 2 A : f (x) 2 yg. Sea fBi g = fB1 ; B2 ; : : :g una familia de subconjuntos de B, probar que es cierta la
igualdad: f 1 (\Bi ) = \f 1 (Bi ):
Solucin:
3.3. APLICACIONES, CORRESPONDENCIAS Y RELACIONES ENTRE CONJUNTOS 143

?
) En primer lugar veamos que f 1 (\Bi ) \f 1 (Bi ). Para ello tomemos un x 2 f 1 (\Bi ) (arbitrario) )
f (x) 2 \Bi ) f (x) 2 Bi , 8i = 1; 2; : : : ) x 2 f 1 (Bi ), 8i = 1; 2; : : : ) x 2 \f 1 (Bi ) y en consecuencia
f 1 (\Bi ) \f 1 (Bi ):
?
) Recprocamente: f 1 (\Bi ) \f 1 (Bi ). En efecto,sea x 2 \f 1
(Bi ) ) x 2 f 1
(Bi ), 8i = 1; 2; : : : )
f (x) 2 Bi , 8i = 1; 2; : : : ) f (x) 2 \Bi ) x 2 f 1 (\Bi ).

7. Demostrar que si f : A ! B y g : B ! A son aplicaciones tales que (g f )(x) = x, 8x 2 A, entonces f


es inyectiva y g es exhaustiva.
Solucin: Veamos en primer lugar que f es inyectiva. En efecto, si f (x) = f (y), para ciertos x, y 2 A,
entonces g(f (x)) = g(f (y)) ) (g f )(x) = (g f )(y) ) x = y. Por tanto, f es inyectiva.
Para probar la exhaustividad de g, tomemos un elemento arbitrario x 2 A y veamos que existe al menos
un elemento y 2 B tal que g(y) = x. En efecto, por hiptesis 8x 2 A es (g f )(x) = x ) g(f (x)) = x,
por lo que bastar con tomar el elemento y = f (x). Luego g es exhaustiva.

8. Sean tres aplicaciones f : A ! B, g : B ! C y h : C ! D; tales que g f y h g son biyectivas.


Demostrar que f y g son biyectivas tambin.
Solucin: Veamos en primer lugar que f es inyectiva: Dados x, y 2 A si f (x) = f (y), entonces g(f (x)) =
g(f (y)) ) (g f )(x) = (g f )(y) ) x = y (por ser g f biyectiva, y, por tanto, inyectiva).
g : B ! C es tambin inyectiva: En efecto, si dados x, y 2 B es g(x) = g(y) ) h(g(x)) = h(g(y)) )
(h g)(x) = (h g)(y) ) x = y (por ser h g biyectiva, y, por tanto inyectiva).
Veamos que f es exhaustiva: Dado y 2 B arbitrario, es g(y) = z 2 C, pero por ser h = g f : A ! C
biyectiva y en particular exhaustiva, existe x 2 A tal que h(x) = g(f (x)) = z = g(y) ) g(f (x)) = g(y) )
f (x) = y (por ser g inyectiva, ya demostrado). As pus, si 8y 2 B, 9x 2 A : f (x) = y ) f es exhaustiva.
g : B ! C es exhaustiva: En efecto, dado z 2 C, hemos de ver que existe y 2 B tal que g(y) = z. Pues
bien, por ser z 2 C y g f : A ! C biyectiva, existe x 2 A tal que (g f )(x) = z ) g(f (x)) = z. Por o
que bastar con tomar y = f (x). Quedando probada de este modo la exhaustividad de g.

9. Sea f : A ! B una aplicacin, se dice que el subconjunto X de A est saturado por f si y slo si
f 1 (f (X)) = X: Demostrar que f es inyectiva si, y solo si, todo X A est saturado por f .
Solucin:
? 1
)) f inyectiva ) f (f (X)) = X, 8 X A. Supuesta f inyectiva, se trata de probar las dos con-
? ?
1 1
tenciones: f (f (X)) X yf (f (X)) X (Esta ltima es evidente).
Demostremos la primera de ellas: Dado x 2 X es f 1 (f (x)) = fy 2 X : f (y) = f (x)g, pero por ser
f inyectiva el que f (y) = f (x) ) y = x, y en consecuencia f 1 (f (x)) = fxg ) f 1 (f (x)) 2 X,
8x 2 X ) f 1 (f (x)) X.
1 ?
() Si 8X A, es f (f (X)) = X ) f es inyectiva.
Recordemos que f es inyectiva si 8x, y 2 A, x 6= y, entonces f (x) 6= f (y). Pues bien supuestos los
elementos x, y 2 A distintos, consideremos el conjunto X = fxg. Por hiptesis es f 1 (f (fxg)) = fxg )
f 1 (f (x)) = x, es decir, f 1 (f (x)) = fy 2 X : f (y) = f (x)g = fxg por lo que si y 6= x, entonces y 2
= fxg,
y, por tanto, f (y) 6= f (x):

10. Sean las aplicaciones f : X ! Y , g : Y ! Z y h = g f : X ! Z:


Demostrar que:

a) Si h es inyectiva, entonces f es inyectiva, y si, adems, f es exhaustiva, entonces g es inyectiva.


?
Solucin: h inyectiva ) f es inyectiva. En efecto, si dados x1 , x2 2 X es f (x1 ) = f (x2 ) ) g(f (x1 )) =
g(f (x2 )) ) (g f )(x1 ) = (g f )(x2 ) ) h(x1 ) = h(x2 ) ) (por ser h inyectiva) que x1 = x2 .
144 CAPTULO 3. NOCIONES DE LGICA, CONJUNTOS Y RELACIONES

h inyectiva y f exhaustiva ) g inyectiva: en efecto, sean y1 , y2 2 Y tales que g(y1 ) = g(y2 ). Por ser
f exhaustiva existen x1 , x2 2 X tales que f (x1 ) = y1 , f (x2 ) = y2 . Por tanto,
8
>
< g(y1 ) = g(f (x1 )) = (g f )(x1 ) = h(x1 )
k ) h(x1 ) = h(x2 )
>
:
g (y2 ) = g(f (x2 )) = (g f )(x2 ) = h(x2 )
y por ser h inyectiva ) x1 = x2 ) f (x1 ) = f (x2 ) = y1 = y2 .
b) Si h es exhaustiva, entonces g es exhaustiva, y si, adems, g es inyectiva, entonces f es exhaustiva.
?
Solucin: h exhaustiva ) g exhaustiva: sean h : X ! Z y g : Y ! Z:
Dado z 2 Z se trata de ver que 9y 2 Y tal que g(y) = z.
Por ser h exhaustiva, por hiptesis, dado z 2 Z; 9x 2 X : h(x) = z ) (g f )(x) = g(f (x)) = z, por
lo que bastar con tomar y = f (x).
?
h exhaustiva y g inyectiva ) f exhaustiva. Ver gura 3.26.

Figura 3.26: Ejemplo 11. Relacin entre las aplicaciones f , g y h.

Sea y un elemento arbitrario de Y , hemos de probar que 9x 2 X tal que f (x) = y. En efecto, 8y 2 Y
es g(y) 2 Z, pero por ser h exhaustiva ) 9x 2 X tal que h(x) = g(y) ) (g f )(x) = g(y) )
g(f (x)) = g(y), por ser g inyectiva ) f (x) = y.

3.3.2. Relaciones de equivalencia y de orden

Sea A un conjunto y R una relacin entre sus elementos, es decir, el subconjunto R A A. Diremos que
R es una relacin de equivalencia denida en A si:

Es reexiva: 8x 2 A ) xRx.
Es simtrica: si xRy ) yRx; 8x; y 2 A.
Es transitiva: si xRy e yRz ) xRz; 8x; y; z 2 A.

Clases de equivalencia Sea A un conjunto y R una relacin de equivalencia denida en A. Para cada
a 2 A se dene la clase de equivalencia de representante a, mdulo Rcomo el conjunto de todos los elementos
de A relacionados con a. Se designa por a~ siendo, pues: a
~ = fx 2 A : xRag.

Propiedades:
1. a2a ~; 8a 2 A.
2. Si x 2 a
~)x ~=a ~.
3. Si x 2
=a~ )x\a ~=?
4. [ a~=A
a2A
3.3. APLICACIONES, CORRESPONDENCIAS Y RELACIONES ENTRE CONJUNTOS 145

Figura 3.27: Ejemplo de relacin de equivalencia : Sea un plano y sea O un punto del mismo. En el conjunto
f0g, se dene la siguiente relacin: P Q , P , Q y O estn alineados.

Las propiedades 3. y 4. contituyen una particin de A.

Al conjunto formado por todas las clases de equivalencia, se le anota por A R, y se denomina conjunto
cociente de A por la relacin de equivalencia R o tambin conjunto cociente de A mdulo R. As pues,
A R = f~ a : a 2 Ag.

Relacin de orden Dado un conjunto A y una relacin entre sus elementos, A A, diremos
que sta es una relacin de orden si:

Es reexiva: 8x 2 A ) x x.
Es antisimtrica: si x y e y x ) x = y; 8x; y 2 A.
Es transitiva: si x y e y z ) x z; 8x; y; z 2 A.

Cuando entre los elementos de un conjunto A se ha denido una relacin de orden, se dice que A es un
conjunto ordenado. Cuando para todo par a, b de A una de las tres relaciones: a < b, a = b, b < a, es cierta, se
dice que es un orden total.

Sea A, un conjunto ordenado por , y sea X A. Se dice que X est acotado superiormente (inferiormente)
si existe un k 2 A, tal que x k (k x) 8x 2 X. A k se le denomina una cota superior (inferior) de X. Si
un conjunto est acotado superior e inferiormente, se dice que est acotado.

Cardinales. Conjuntos equipotentes

Se dice que dos conjuntos A, B, son equipotentes cuando es posible establecer una biyeccin entre ellos.

Cardinal de un conjunto La relacin entre conjuntos: A relacionado con B si y slo si A es equipotente


a B, es evidentemente de equivalencia.

A cada conjunto, A, se le asocia un nuevo objeto matemtico, card(A) = #A, llamado cardinal de A o
tambin su nmero cardinal, denido por la condicin: Dos conjuntos tienen el mismo cardinal si y slo si
son equipotentes.

Al cardinal (nmero de elementos) del conjunto vaco se le anota por 0, al cardinal del conjunto f?g por el
smbolo 1, al cardinal del conjunto f?; f?gg por 2, ..., etc.

0 = card(?); 1 = card(f?g); 2 = card(f?; f?gg); 3 = card(f?; f?g; f?; f?ggg); : : :

Conjuntos nitos Dado A, se dice que card(A) = #A es el nmero de sus elementos. Si ste es un
nmero natural se dice que A es nito. En caso contrario se dice que es innito. Cierto conjunto se dice
146 CAPTULO 3. NOCIONES DE LGICA, CONJUNTOS Y RELACIONES

tambin innito si este es equipotente con una de sus partes, as, N = f1; 2; 3; : : :g es innito porque es
equipotente con el conjunto de todos los naturales pares f2; 4; 6; : : :g. As, por ejemplo,
n A = fa; b; c; x; yg,
o
B = fvrtices de un polgonog, C = fpeces del ocanog son conjuntos nitos, y Q = x = pq : p 2 Z; q 2 N ,
R, C = frectas que pasan por un punto jo del planog son conjuntos innitos.

Conjuntos numerables Si A es equipotente a una parte de N (o a Q, cuerpo de los nmeros racionales), se


dice que A es numerable y si es equipotente a todo N (a todo Q) se dice que es innito numerable. Si A es nito,
es numerable. As, por ejemplo, el conjunto de todos los naturales pares f2; 4; 6; : : :g es innito numerable, y
B = fvrtices de un polgonog es numerable.

Algunos cardinales

card(A [ B) = card(A) + card(B) card(A \ B) o


#(A [ B) = #(A) + #(B) #(A \ B)
card(A B) = card(A) card(B); card (P (A)) = 2card(A) o
#(A B) = #(A) #(B); # (P (A)) = 2#(A) ;

donde P (A) representa al conjunto de todas las partes (subconjuntos) del conjunto A. As, por ejemplo, si
A = f 2; 3g, B = f 2; 0; 6; 7g, entonces A[B = f 2; 0; 3; 6; 7g, A\B = f 2g, P (A) = f;; f 2g ; f0g ; f 2; 0gg,
A B = f( 2; 2) ; ( 2; 0) ; ( 2; 6) ; ( 2; 7) ; (3; 2) ; (3; 0) ; (3; 6) ; (3; 7)g, entonces facilmente se comprueba que
card(A [ B) = card(f 2; 0; 3; 6; 7g) = 2 + 4 1 = 5, card(A B) = 2 4 = 8, card (P (A)) = 22 = 4.

Nota. Tambin se escribe n(A) o #A para indicar el card(A).

Figura 3.28: Izq. El ms prolco de todos los matemticos, el suizo Leonhard Euler (Basilea,1707 - San Petersburgo,
1783), gran divulgador en Europa de todas las ramas de las matemticas, astronoma y mecnica. Los ltimos aos de
su vida estuvo ciego pero sigui trabajando intensamente con ayuda de su hijo como secretario. Der. Pierre de Fermat
(Beaumont de Lomagne, 1601 - Castres, 1665), jurista, parlamentario, juez y famoso matemtico acionado, uno de los
creadores de las bases de la teora de las probabilidades, geometra analtica y de varias cuestiones del clculo, teora de
nmeros y fsica. Su famosa ecuacin diofntica (solucin) - enigma o conjetura de Fermat: xn + y n = z n ; x; y; z 2 Z+ ,
N 3 n 3 demor aproximadamente 330 aos en convertirse en teorema y demostrarse su insolubilidad en enteros
(Andrew Wiles, 1994).

Ejemplos:

1. Estudiar las siguientes relaciones binarias denidas en el conjunto Z:

a) xRy , x y > 0, (x y 6= 0)
Solucin: R no es reexiva: Ya que 0R0 , 0 0 > 0, lo cual no es cierto.
?
R es simtrica: Si xRy ) yRx. En efecto: xRy ) x y > 0 ) y x > 0 ) yRx.
3.3. APLICACIONES, CORRESPONDENCIAS Y RELACIONES ENTRE CONJUNTOS 147
9
xRy >
=
?
R es transitiva: 8x, y, z 2 Z si: y ) xRz
>
;
yRz
En efecto: xRy ) x y > 0, yRz ) y z > 0 ) (x y) (y z) > 0 ) x (y 2 ) z > 0 )
) x z > 0 (por ser y 2 > 0, 8y 2 Z) ) xRz.
b) xSy , x y > 0:
Solucin: S es reexiva: Puesto que 8x 2 Z es x x = x2 0.
S es simtrica: Si xSy ) x y > 0 ) y x > 0 ) ySx:
S no es transitiva: Ya que , por ejemplo:
4S0 (pues 4 0 0) y 0S7 (pues 0 7 0), y no obstante 4 7= 28 < 0 ) 4 7 0) 4S7
/
( 4 no S7).

2. En el conjunto A = f 2; 1; 0; 1g se dene la siguiente relacin binaria: xRy , x < jyj. Se pide:

a) Grafo que dene dicha relacin.


Solucin:

2R 2 pues 2 < j 2j = 2 ) ( 2; 2) 2 R; 2R 1 pues 2 < j 1j = 1 ) ( 2; 1) 2 R;


2R0 pues 2 < j0j = 0 ) ( 2; 0) 2 R; 2R1 pues 2 < j1j = 1 ) ( 2; 1) 2 R;
1R 2 pues 1 < j 2j = 2 ) ( 1; 2) 2 R; 1R 1 pues 1 < j 1j = 1 ) ( 1; 1) 2 R;
1R0 pues 1 < j0j = 0 ) ( 1; 0) 2 R; 1R1 pues 1 < j1j = 1 ) ( 1; 1) 2 R;
0R 2 pues 0 < j 2j = 2 ) (0; 2) 2 R; 0R 1 pues 0 < j 1j = 1 ) (0; 1) 2 R;
0R1 pues 0 < j1j = 1 ) (0; 1) 2 R; 1R 2 pues 1 < j 2j = 2 ) (1; 2) 2 R:
( )
( 2; 2), ( 2; 1), ( 2; 0), ( 2; 1), ( 1; 2), ( 1; 1),
Por tanto: R =
( 1; 0), ( 1; 1), (0; 2), (0; 1), (0; 1),(1; 2):
b) Representacin grca.
Solucin: Ver gura 3.28.
c) Estudiar sus propiedades
Solucin: La relacin binaria R no es:
Reexiva: pues (0; 0) 2
= R.
Simtrica: ya que (0; 1) 2 R y (1; 0) 2
= R.
Transitiva: puesto que (1; 2) 2 R, ( 2; 1) 2 R y, no obstante, (1; 1) 2
= R:
Antisimtrica: (1; 2) 2 R, ( 2; 1) 2 R y, sin embargo, 1 6= 2:

Figura 3.29: Ejemplo 3. Representacin grca de la relacin binaria.


148 CAPTULO 3. NOCIONES DE LGICA, CONJUNTOS Y RELACIONES

3. En el conjunto A = f 2; 1; 0; 1; 2; 3g se dene la siguiente relacin: xRy , jx yj 1


Obtener el diagrama correspondiente y estudiar sus propiedades.
Solucin:

2R 2 ya que j 2 ( 2)j = j 2 + 2j = 0 1; 2R 1 ya que j 2 ( 1)j = j 1j = 1 1

1R 2 ya que j 1 ( 2)j = j 1 + 2j = 1; 1R 1 ya que j 1 ( 1)j = j 1 + 1j = 0

1R0 ya que j 1 0j = j 1j = 1; 0R 1 ya que j0 ( 1)j = j1j = 1

0R0 ya que j0 0j = j0j = 0; 0R1 ya que j0 1j = j 1j = 1

1R0 ya que j1 0j = j1j = 1; 1R1 ya que j1 1j = j0j = 0

1R2 ya que j1 2j = j 1j = 1; 2R1 ya que j2 1j = j1j = 1

2R2 ya que j2 2j = j0j = 0; 2R3 ya que j2 3j = j 1j = 1

3R2 ya que j3 2j = j1j = 1; 3R3 ya que j3 3j = j0j = 0


( )
( 2; 2), ( 2; 1), ( 1; 2), ( 1; 1),( 1; 0), (0; 1), (0; 0),
R= . Ver gura 3.29.El grafo de R
(0; 1), (1; 0), (1; 1), (1; 2), (2; 1), (2; 2), (2; 3), (3; 2), (3; 3).

Figura 3.30: Ejemplo 4. Grca de la relacin binaria R.

y, por tanto, la relacion es:


Reexiva: Puesto que contiene a todos los elementos de la diagonal principal: ( 2; 2), ( 1; 1) : : :, (3; 3).
Simtrica: Ya que (x; y) 2 R , (y; x) 2 R:
Hay en este diagrama simetra rspecto a la diagonal principal por ejemplo: ( 1; 2) 2 R ) ( 2; 1) 2 R:
No transitiva : Basta con observar que: ( 1; 0) 2 R; (0; 1) 2 R y, no obstante, ( 1; 1) 2
= R:

4. Sea E = f 2; 1; 0; 3; 4g, denimos: aRb , 2a b 3. Cul es su grafo? Qu propiedades cumple esta


relacin?
Solucin:
2R 2 pues 2(2) (2) = 2 3, 2 R 1 pues 2( 2) + 1 = 3 3
2R 0 pues 2( 2) 0= 4 3, 2 R 3 pues 2( 2) 3= 7
2R 4 pues 2( 2) 4= 8, 1R 2 pues 2( 1) ( 2) = 2+2=0 3
1R 1 pues 2( 1) ( 1) = 1 3, 1 R 0 pues 2( 1) 0= 2 3
3.3. APLICACIONES, CORRESPONDENCIAS Y RELACIONES ENTRE CONJUNTOS 149

1R 3 pues 2( 1) 3= 5 3, 1 R 4 pues 2( 1) 4= 6 3

0R 2 pues 2(0) ( 2) = 2 3, 0 R 1 pues 2(0) ( 1) = 1

0 R 0 pues 2(0) 0 = 0, 0 R 3 pues 2(0) 3= 3

0 R 4 pues 2(0) 4= 4, 3 R 3 pues 2(3)


3 = 3, 3 R 4 pues 2(3) 4 = 2
( )
( 2; 2), ( 2; 1), ( 2; 0), ( 2; 3), ( 2; 4), ( 1; 2), ( 1; 1), ( 1; 0),
R= . Ver gura 3.30.
( 1; 3), ( 1; 4), (0; 2), (0; 1), (0; 0), (0; 3), (0; 4), (3; 3), (3; 4).
Esta relacin R es:

No reexiva: ( 4; 4) 2
= R:

No simtrica: (3; 4) 2 R y (4; 3) 2


= R:

Transitiva: Si (a; b) 2 R y (b; c) 2 R, entonces (a; c) 2 R: Basta con observar los distintos casos.

Figura 3.31: Ejemplo 5. Grca de la relacin R:

5. Sea A = f1; 2; 3; 4; 5g. Se dene una relacin R en A A mediante: (a; b)R(c; d) ) a b=c d. Probar
que es de equivalencia, y dar las clases.

Solucin: Veamos que R satisface las propiedades:

Reexiva: 8(a; b) 2 A A, (a; b)R(a; b), ya que a b=a b.

Simtrica: 8(a; b); (c; d) 2 A A; si (a; b)R(c; d) ) a b=c d)c d=a b ) (c; d)R(a; b).

Transitiva: 8(a; b); (c; d); (e; f ) 2 A A; si


9
(a; b)R(c; d) >=
?
y ) (a; b)R(e; f )
>
;
(c; d)R(e; f )

En efecto: (a; b)R(c; d) ) a b=c d y (c; d)R(e; f ) ) c d=e f )a b=c d=e f)

)a b=e f ) (a; b)R(e; f )


]
En cuanto a las clases de equivalencia, digamos que una clase (a; b) estar constituida por todos los pares
150 CAPTULO 3. NOCIONES DE LGICA, CONJUNTOS Y RELACIONES

(x; y) 2 A A tales que a b=x y. As pues, tendremos las siguientes clases:


]
(1; 1) ]
= f(1; 1); (2; 2); (3; 3); (4; 4); (5; 5)g; (1; 2) = f(1; 2); (2; 3); (3; 4); (4; 5)g
]
(1; 3) ]
= f(1; 3); (2; 4); (3; 5)g; (1; 4) = f(1; 4); (2; 5)g; (1; ] 5) = f(1; 5)g
]
(2; ]
1) = f(2; 1); (3; 2); (4; 3); (5; 4)g; (3; 1) = f(3; 1); (4; 2); (5; 3)g
]
(4; 1) = f(4; 1); (5; 2)g; (5; ] 1) = f(5; 1)g

Ntese que la unin de todas ellas es justamente el conjunto A A y que Card (A A) = Card A Card A =
25.

6. En Z Z0 se dene la relacin: (a; b) R (c; d), si y solo si, ad = bc. Demostrar que se trata de una relacin
de equivalencia y hallar el conjunto cociente.
Nota: Z0 = Z f0g:
Solucin: Empecemos probando que esta relacin R cumple las propiedades:
Reexiva: 8(a; b) 2 Z Z0 , (a; b)R(a; b), ya que ab = ba
Simtria: 8(a; b), (c; d) 2 Z Z0 ; si (a; b)R(c; d) ) ad = bc ) cb = da ) (c; d)R(a; b):
Transitiva: 8(a; b), (c; d); (e; f ) 2 Z Z0 ; si
9
(a; b)R(c; d) =>
?
y ) (a; b)R(e; f )
>
;
(c; d)R(e; f )

En efecto: (a; b)R(c; d) ) ad = bc (1) y (c; d)R(e; f ) ) cf = de (2).


Ahora bien:
)
ad = bc ) a=b = c=d a c e a e
) = = ) = ) af = be ) (a; b)R(e; f ):
cf = de ) c=d = e=f b d f b f

O tambin el que ad = bc (1) y cf = de (2) ) adf = bcf y bcf = bde ) adf = bde ) af = be )
(a; b)R(e; f ).
^
El conjunto cociente Z Z0 =R est formado por todas las clases f(a; b) ; 8(a; b) 2 Z Z0 g. Siendo cada
una de ellas el conjunto:
]
(a; b) = f(x; y) 2 Z Z0 : (a; b)R (x; y)g = f(x; y) 2 Z Z0 : ay = bxg =
a x
= (x; y) 2 Z Z0 : =
b y
Es decir, cada clase est constituda por todas las fracciones racionales de igual valor, por lo que cada una
de ellas representar un nmero racional, y el conjunto cociente ser el conjunto de los nmeros racionales
Q.

7. En el conjunto A = f0; 1; 2; 4g se da la siguiente relacin: 8x, y 2 A, xRy , x2 + x = y 2 + y.

a) Demostrar que es de equivalencia


Solucin: Veamos que la relacin R cumple las propiedades:
Reexiva: 8x 2 A, xRx, ya que x2 + x = x2 + x
Simtrica: 8x, y 2 A, si xRy ) x2 + x = y 2 + y ) y 2 + y = x2 + x ) yRx:
9
xRy >
=
?
Transitiva: 8x, y, z 2 A, si y ) xRz
>
;
yRz
En efecto: xRy ) x2 + x = y 2 + y, yRz ) y 2 + y = z 2 + z ) x2 + x = y 2 + y = z 2 + z )
) x2 + x = z 2 + z ) xRz. Por lo tanto, se trata de una relacin de equivalencia.
3.3. APLICACIONES, CORRESPONDENCIAS Y RELACIONES ENTRE CONJUNTOS 151

b) Hallar el conjunto cociente A=R


x; 8x 2 Ag. Puesto que cada clase xe esta constituda por todos los y 2 A tales
Solucin: A=R = fe
que xRy;
es decir,tales que x2 + x = y 2 + y, para determinar explcitamente todas las clases, construyamos la
siguiente tabla de valores:
x x2 + x
0 0
1 2
2 2
4 20

Dicha tabla nos pone de maniesto que hay 3 clases distintas, a saber: e
0 = f0g; e
1 = f1; 2g (1R 2,
2 2 e
ya que 1 + 1 = ( 2) + ( 2)); 4 = f4g :

8. En los naturales N se dene la relacion binaria R: dados x, y 2 N, xRy , x divide a y (abreviadamente


x j y). Es R una relacin orden, es total?
Solucin: La relacin R cumple las propiedades:
Reexiva: 8x 2 N; xRx; puesto que x j x:
?
Antisimtrica: Si xRy; yRx ) x = y. En efecto: xRy ) x j y ) 9n 2 N, tal que y = nx; yRx ) y j x )
9m 2 N, tal que x = my, por tanto y = nx = n(my) ) y = (nm)y ) nm = 1 ) n = m = 1 (n; m 2 N).
Luego x = y:
?
Transitiva: Si xRy; yRz ) xRz. En efecto: xRy ) x j y ) 9n 2 N, tal que y = nx; yRz ) y j z )
9m 2 N, tal que z = my, por tanto z = my = m(ny) = (mn)x ) x j z ) xRz:
As pues, la relacin es de orden. El orden no es total, ya, que, por ejemplo, los elementos 3, 7 2 N son
tales que ni 3 divide a 7, ni 7 divide a 3, es decir, ni 3R7, ni 7R3.

9. Dado el conjunto A = f1; 2; 3; : : : ; 10g, con la relacin de orden : aRb , (a divide a b). Obtener, si es que
existen, los elementos: mximo, mmimo, maximales y minimales.
Solucin: En el conjunto A = f1; 2; 3; :::; 10g:
1R1 puesto que 1 j 1, 1 R 2, puesto que 1 j 2,
.......................................................................................................................................
1 R 10, puesto que 1 j 10, 2 R 2, puesto que 2 j 2, 2 R 4, puesto que 2 j 4,
2 R 6, puesto que 2 j 6, 2 R 8, puesto que 1 j 8, 2 R 10, puesto que 1 j 10,
3 R 3, puesto que 3 j 3, 3 R 6, puesto que 3 j 6, 3 R 9, puesto que 4 j 4,
4 R 8, puesto que 4 j 8, 5 R 5, puesto que 5 j 5, 5 R 10, puesto que 5 j 10,
6 R 6, puesto que 6 j 6, 7 R 7, puesto que 7 j 7, 8 R 8, puesto que 8 j 8,
9 R 9, puesto que 9 j 9, 10 R 10, puesto que 10 j 10:
La representacin cartesiana del grco correspondiente es la gura 3.32.

10. Sea f : A ! B una aplicacin. En A se dene la siguiente relacin Rf : dados a, b 2 A, aRf b , f (a) =
f (b).

a) Probar que esta relacin es de equivalencia.


Solucin: La relacin Rf , satisface las propiedades:
Reexiva: 8a 2 A, aRf a; ya que f (a) = f (a)
Simtrica: 8a, b 2 A, aRf b ) f (a) = f (b) = f (a) ) bRf a:
152 CAPTULO 3. NOCIONES DE LGICA, CONJUNTOS Y RELACIONES

Figura 3.32: Ejemplo 13. Relacin de orden R:

9
aRf b >
=
?
Transitiva: 8a, b, c 2 A, si y ) aRf c
>
;
bRf c
En efecto: aRf b ) f (a) = f (b) y bRf c ) f (b) = f (c) ) f (a) = f (b) = f (c) )
) f (a) = f (c) ) aRf c:

b) Dar el conjunto cociente


a; 8a 2 Ag, donde cada clase e
Solucin: El conjunto cociente A=Rf = fe a viene dada por el subcon-
junto:

e
a = fx 2 A : xRag (= fx 2 A : f (x) = f (a)g)

es decir, por todos los elementos x 2 A, que tiene la misma imagen por f que el elemento a:

11. Sea f : A ! B y g : B ! C dos aplicaciones. Demostrar que si g es inyectiva, entonces Rf = Rg f

Solucin: Dados x; y 2 A empecemos viendo que si xRf y, entonces x (Rg f ) y. En efecto, si x (Rf ) y )
f (x) = f (y) ) g(f (x)) = g(f (y)) ) (g f )(x) = (g f )(y) ) x (Rg f ) y.

Recprocamente: 8x, y 2 A, si x (Rg f ) y, entonces x (Rf ) y. En efecto, si x (Rg f ) y ) (g f )(x) =


(g f )(y) ) gf ((x)) = g(f (y)) ) (por ser g inyectiva) que f (x) = f (y) ) x (Rf ) y:

Por tanto, si g es inyectiva, Rf = Rg f .

12. Sea M y N dos conjuntos nitos tales que card M = card N = n .

a) Probar que todo el conjunto de las aplicaciones de M en N , que suelen indicarse por N M , es un
conjunto nito con nm elementos es decir que el card fAplicaciones: M ! N ; card M = m,
card N = ng = nm = Vn; m . (Variaciones con repeticin de n elementos tomados de m en m).
Solucin: Sean los conjuntos M = fa1; a2 ; ::::am g y N = fb1 ; b2 ; :::::bn g de cardinales m y n respec-
tivamente.
Toda aplicacin de M en N queda determinada al dar las imgenes de los ai para i = 1; 2; :::; m.
3.3. APLICACIONES, CORRESPONDENCIAS Y RELACIONES ENTRE CONJUNTOS 153

Sea f una aplicacin genrica de M en N :


f
M ! N
a1 b1
a2 b2
.. ..
. .
am bn

Como imagen de a1 , f (a1 ), podemos, tomar en principio, cualquiera de los elementos de N , por tanto,
hay n posibilidades (cada una de ellas darn lugar a una aplicacin). Sea, por ejemplo; f (a1 ) = b2 ,
como imagen de a2 , f (a2 ) podemos tomar cualquiera de los n elementos de N , incluido el b2 puesto
que nadie dice que f sea inyectiva. Por tanto, tendremos un total de n n = n2 posibilidades, y en
consecuencia n2 aplicaciones distintas.
Reinterando este proceso hasta llegar al am se incluye que card fAplicaciones: M ! N g = nm .
Nota. El que este nmero de aplicaciones sea nm , es la causa de que algunos autores escriban N M o
B A para indicar el conjunto de las aplicaciones de M en N (A en B). El nmero nm = V rn; m =
variaciones con repeticin de n elementos tomados de m en m.
b) Probemos que: card fAplicaciones inyectivas: M ! N g = n (n 1):::(n m+1) = Vn;m . (Variaciones
ordinarias de n elementos tomados de m en m.)
Solucin: En efecto, sea, f : M ! N una aplicacin inyectiva genrica
f
M ! N
a1 b1
a2 b2
.. ..
. .
am bn

Como imagen de a1 , f (a1 ), podemos tomar, al igual que antes, cualquiera de los n elementos de N ,
luego hay n posibilidades. S , por ejempplo: f (a1 ) = b2 al dar la imagen de a2 , f (a2 ), por ser f
inyectiva, no podemos tomar de nuevo b2 , pues f (a2 ) debe ser distinto de f (a1 ) = b2 por tanto,
nicamente hay n 1 posibilidades.
Si, por ejemplo, tomamos f (a2 ) = b3 , como imagen de a3; f (a3 ), podemos elegir cualquier elemento
de B2 distinto de b1 y b3 , por tanto, habrn n 2 posibilidades.
Repitiendo este razonamiento hasta llegar al elemento am se tiene:
Elementos de M: Posibilidades:
a1 n
a2 n 1
a3 n 2
a4 n 3
.. ..
. .
am n (m 1)
Por tanto, Card fAplicaciones inyectivas: M ! N g = n(n 1)(n 2) : : : (n m + 1). A dicho
nmero se lo llama, tambin, variaciones (ordinarias) de n elementos tomados de m en m, y se lo
designa mediante Vn;m .

13. Probar que si A es un conjunto nito tal que card A = n, entonces el


card fAplicaciones biyectivas : A ! Ag = card fPermutaciones de Ag = n (n 1)::::;2 1 = n!
Solucin: Sea A = fx1; x2 ; :::; xn g, toda autoaplicacin de A queda denida al dar las imgenes de xi ,
8i = 1, 2,:::, n. Sea f : A ! A una aplicacin de este tipo, es decir:
154 CAPTULO 3. NOCIONES DE LGICA, CONJUNTOS Y RELACIONES

f
A ! A
x1 x1
x2 x2
.. ..
. .
xn xn
Como imagen de x1 podemos tomar en principio cualquiera de los n elementos de A. Habr, pues, un total
de n posibilidades. Fijado f (x1 ), por ejemplo: f (x1 ) = x3 , la imagen de x2 , f (x2 ) por ser f biyectiva y,
por tanto, inyectiva, podr ser cualquier elemento de A a excepcin del x3 ; en consecuencia habrn n 1
posibilidades. Fijado f (x2 ), por ejemplo f (x2 ) = x5 y repitiendo el mismo razonamiento, se observa que
al dar la imagen del x3 hay n 2 posibilidades, y as sucesivamente hasta llegar al xn , para el cual habrn
n (n 1) = 1 posibilidad. Por tanto, card fPermutaciones de Ag = n (n 1)::::(n 2) 1 = n!

Practico lo que aprend:

Utilizando demostraciones de pertenencia y/o contencin de la parte izquierda en la parte


derecha y viceversa, o usando formalizaciones que conduzcan a una tabla de verdad con o sin
tautologa, demostrar las relaciones entre conjuntos:

1. Demostrar la equivalencia de las siguientes tres relaciones: A B, A \ B = A, A [ B = B, es decir,


demostrar que del cumplimiento de cualquiera de ellas, se sigue que las dos otras son correctas.

2. Demostrar que: A B = A \ B 0 :

3. Demostrar las inclusiones


a) (B C) (B A) A C; b) A C (A B) [ (B C) ;
c) (A \ C) [ (B \ D) (A [ B) \ (C [ D)

4. Demostrar las igualdades:


a) A (B C) = (A B) [ (A \ C) ; b) (A B) C = (A C) (B C) ;
c) (A [ B) C = (A C) [ (B C) ; d) (A \ B) C = (A C) \ (B C) ;
e) (A B) [ (B C) [ (C A) [ (A \ B \ C) = A [ B [ C;
f ) (A B) \ C = (A \ C) (B \ C) = (A \ C) B:

5. Se concluye o no que de A B = C, A = B [ C ?

6. Se concluye o no que de A = B [ C, A B = C ?

7. Son o no verdaderas las igualdades


a) A (B [ C) = (A B) C; b) A [ (B C) = (A [ B) C;
c) (A B) [ C = (A [ C) B?
Si no lo son, hacia que lado se tendra la inclusin?

8. Demostrar la equivalencia de las inclusiones A B C y A B [ C:

9. Demostrar que la igualdad A (B C) = (A B) [ C es verdadera , si A C, y falsa si C A 6= ?:

10. Demostrar la inclusin ([Ak [ Bk ) ([ (Ak Bk )). Mostrar con un ejemplo que en el caso general no
hay una igualdad, k = 1; 2; : : :

11. Demostrar que A4B = (A [ B) (A \ B) :

12. Sea A un conjunto dado. Demostrar que el conjunto X es vaco, si y solamente, si A4X = A.
3.3. APLICACIONES, CORRESPONDENCIAS Y RELACIONES ENTRE CONJUNTOS 155

13. Demostrar las igualdades:


a) A4 (B4D) = (A4B) 4D; b) A \ (B4D) = (A \ B) 4 (A \ D) ;
c) A4A = ?:

14. Demostrar las inclusiones:


a) (A [ B) 4F (A4F ) [ (B4F ) ; b) A4B (A4C) [ (B4C) ;
c) (A [ B) 4 (C [ D) (A4C) [ (B4D) :
Mostrar con ejemplos que en el caso general no hay igualdades.

15. Demostrar las igualdades (X 0 es el complemento de X):


0
a) ((A0 [ B)0 [ (A [ B 0 ))0 = B A; b) (A B) = A0 [ B;
c) (A \ B) [ (A \ B 0 ) [ (A0 \ B) = A [ B:

16. Utilizando la ley de dualidad, simplicar la expresin ((X [ Y )0 \ (X 0 [ Y 0 ))0 :

17. Demostrar que para cualesquiera conjuntos E, F , G se cumplen las igualdades:


a) E (F [ G) = (E F ) [ (E G) ; b) (F [ G) E = (F E) [ (G E) ;
c) E (F \ G) = (E F ) \ (E G) ; d) (F \ G) E = (F E) \ (G E) :

18. Averiguar si son o no verdaderas las siguientes relaciones:


a) (A B) \ (C D) = (A \ C) (B \ D) ;
b) (A B) [ (C D) = (A [ C) (B [ D) :

19. Demostrar que (A B) C = (A C) (B D) :

20. Demostrar que (P Q) (A B) = ((P A) Q) [ (P (Q B)) :

21. Sean los conjuntos A y C no vacos. Demostrar que para que A B, C D, es necesario y suciente que
se tenga que A C B D. Es verdadera la armacin, si A y C son vacos?

22. Demostrar que si A P, B Q, entonces A B = (A Q) \ (B P):

Figura 3.33: Izq. George Dantzig (1914 -), famoso matemtico norteamericano, dedicado a la programacin lineal y
sus aplicaciones, creador del mtodo simplex de optimizacin lineal que junto con el matemtico ruso, premio nobel de
economa (1975), Leonid Kantorovich (Peterburg, 1912-) aplicaron estos mtodos y sus variantes a la optimizacin de
problemas econmico-matemticos. Der. El famoso bourbakista francs, Jean Dieudonn (Lille,1906 - Nancy, 1992),
especialista en topologa moderna, geometra algebraica y teoria de los invariantes. El acu la frase abajo Euclides!,
impulsando un nuevo tipo de enseanza moderna de la geometra.
Captulo 4

APLICACIONES, FUNCIONES
NUMRICAS Y SUS GRFICAS

En la Matemtica no encuentro ninguna imperfeccin, excepto quiz en el hecho de que los

hombres no comprenden de manera suciente el excelente uso de la Matemtica Pura.

Francis Bacon.

Antes preferira descubrir una causa que ganar el Reino de Persia.

Demcrito.

Experientia optima magistra.

Virgilio, Ovidio.

Unidad de competencia: Comprender los conceptos fundamentales de relacin y de funcin o aplicacin (relacin
funcional) como correspondencia entre conjuntos numricos, sus propiedades, su dominio e imagen y su representacin
grca, as como las transformaciones elementales utilizando procesos analiticos y grcos.

Indicadores de logro:

| Describir con sus propias palabras los conceptos de aplicacin, funcin y sus componentes.

| Establecer y comprender la diferencia entre funcin y relacin.

| Calcular el dominio e imagen de una relacin y funcin.

| Identicar y manejar las diferentes clases o tipos de funciones elementales.

| Determinar las principales caractersticas de las funciones y construir sus grcas.

(
3, si x 2Q p
Ser posible gracar las funciones f (x) = y g (x) = 2+ log (cos 2 x) ?
0, si x 2Q0

156
4.1. CONCEPTO DE FUNCIN 157

4.1. Concepto de Funcin

Sean X e Y conjuntos numricos (X; Y R). Se dice que est dada cierta funcin (relacin o aplicacin
funcional), denida sobre X, con valores en Y , si a fuerza de cierta ley f , a cada elemento x 2 X le corresponde
f
un elemento y 2 Y . Una funcin se representa con f : X ! Y , X ! Y , x ! f (x) o y = f (x).

| Denir cierta funcin f : X ! Y , signica indicar la terna (X; f; Y ), donde: X es el conjunto a aplicarse,
conjunto de salida, o el dominio de denicin de la funcin; Y es conjunto de llegada, o dominio de llegada
de la funcin o aplicacin; f es la ley o correspondencia mediante la cual, a cada elemento x 2 X se le hace
corresponder un elemento denido y 2 Y:
2x 1 p
As, y = 3x + 1, x ! 2
, f (x) = 1 x3 , son ejemplos de funciones porque para cada valor de
x +1 p p
x 2 X, existe un solo valor de y 2 Y ; f (x) = x, y = 4 3x2 + 1, no son ejemplos de funciones, pero si de
aplicaciones, pues para cada valor de x existen dos valores de y (uno positivo y otro negativo).

En este caso, el conjunto X suele denominarse dominio de denicin de la funcin, el smbolo x argumento
o variable independiente. El valor correspondiente y0 2 Y al valor jo x0 2 X del argumento x, se denomina
valor de la funcin en x0 , y se lo representa mediante f (x0 ). Cuando vara el argumento x, generalmente, los
valores de la funcin y = f (x), tambin varan, por eso y suele denominarse variable dependiente y x variable
independiente.

| El conjunto f (X) = fy 2 Y : 9x ((x 2 X) ^ (y = f (x)))g de todos los valores que la funcin toma sobre
el conjunto X, se denomina rango de la funcin (o dominio de valores de la funcin).

| Sea dada la funcin numrica f : R ! R, entonces los conjuntos

Dom f = fx 2 R : 9y con y = f (x)g = X; Im f = fy 2 R : 9x con f (x) = yg = Y;

se denominan dominio e imagen de la funcin f , respectivamente. As, por ejemplo, para y = 5x + 2, se tiene
2 p
Dom f = R e Im f = R; para x ! 2 , se tiene Dom f = R e Im f = [ 2; 0[; para f (x) = 1 x, se tiene
x +1
Dom f = fx 2 R : 1 x 0g = fx 2 R : x 1g =] 1; 1] e Im f = [0; +1[.

| A veces una funcin, en dependencia de la naturaleza de los conjuntos X e Y , suele llamarse aplicacin,
transformacin, morsmo, operador, funcional. El ms usado es el de aplicacin, simbolizado mediante:

f
f : X ! Y; X!Y

| Dos funciones f1 y f2 se consideran iguales, si poseen iguales dominios de denicin X, y para cualquier
x2 4
elemento x 2 X, se cumple que f1 (x) = f2 (x). As, por ejemplo, las funciones f1 (x) = y f2 (x) = x + 2
x 2
son iguales en el conjunto X = R f2g. Anlogamente, si X, Y R son conjuntos numricos, f : X ! Y y
g : X ! Y aplicaciones de X en Y , las aplicaciones S, R, P y Q de X en Y (S; R; P; Q : X ! Y ) denidas
mediante S(x) = (f + g) (x) = f (x) + g (x), R(x) = (f g) (x) = f (x) g (x), P (x) = (f g) (x) = f (x) g (x),
f f (x)
Q(x) = (x) = , 8x 2 X, se denominan suma, diferencia, producto y cociente (g (x) 6= 0, 8x 2 A)
g g (x)
f
de aplicaciones, respectivamente, y se simbolizan mediante f + g, f g, f g y , respectivamente. As,
g
3
por ejemplo, si f (x) = y g (x) = x son aplicaciones de X en Y , entonces: S(x) = (f + g) (x) =
x 1
2
3 3+x x 3 x2 x + 3 3 3x
x= , R(x) = (f g) (x) = +x = , P (x) = (f g) (x) = x= ,
x 1 x 1 x 1 x 1 x 1 x 1
f 3 3
Q(x) = (x) = =x = 2 , son las funciones suma (x 6= 1) , diferencia (x 6= 1) , producto (x 6= 1)
g x 1 x x
y cociente (x 6= 0; 1) obtenidas a partir de f y g.
158 CAPTULO 4. APLICACIONES, FUNCIONES NUMRICAS Y SUS GRFICAS

| Dada la funcin f : X ! Y , (X; Y R), se denomina grca de la funcin f al subconjunto de puntos


G R2 , denido mediante: G = f(x; y) : x 2 Dom f , y = f (x)g R2 :

4.1.1. Clasicacin simple de las aplicaciones

Cuando a la funcin f : X ! Y se la llama aplicacin, el valor f (x) 2 Y que toma para el elemento x 2 X,
generalmente se lo denomina como imagen del elemento x. La imagen total del conjunto A X segn la
aplicacin f : X ! Y viene denida por el conjunto

f (A) := fy 2 Y : 9x ((x 2 A) ^ (y = f (x)))g

de aquellos elementos de Y que son imgenes de los elementos del conjunto A:

El conjunto f 1 (B) := fx 2 X : f (x) 2 Bg de aquellos elementos de X, cuyas imgenes estn contenidas


en B, se denomina preimagen del conjunto B Y: El siguiente diagrama muestra las componentes de una
aplicacin.

Figura 4.1: Componentes de la aplicacin f : X ! Y:

Recuerda la clasicacin de aplicaciones f : X ! Y :


Sobreyectiva (o aplicacin de X sobre Y ), si f (X) = Y:
Inyectiva, si para cualesquiera elementos x1 , x2 2 X, se tiene que (f (x1 ) 6= f (x2 ))
) (x1 6= x2 ), es decir, elementos distintos poseen imgenes distintas.
Biyectiva, si la aplicacin f : X ! Y es al mismo tiempo inyectiva y sobreyectiva.

| Una aplicacin o funcin f : X ! Y puede no pertenecer a ninguna de las clases anteriores.

| Si una aplicacin f : X ! Y es biyectiva, es decir existe una correspondencia uno a uno entre los elementos
de X e Y , entonces aparece la nueva aplicacin f 1 : Y ! X, la misma que se dene de la siguiente manera:
1
f f
si x 7 ! y, entonces y 7 ! x, es decir, al elemento y 2 Y se le pone en correspondencia aquel elemento x 2 X,
imagen del cual, mediante f es y. Como f es sobreyectiva, el elemento x 2 X siempre existe, y como f tambin
es inyectiva, este x es nico. En este caso la aplicacin f 1 se denomina inversa de f o respecto de f .

De la construccin misma de la aplicacin f 1 : Y ! X, se desprende que f 1 es tambin biyectiva y que su


1
inversa f 1 : X ! Y coincide con f : X ! Y . De esta manera vemos que la propiedad de dos aplicaciones
de ser inversas es recproca: si f 1 es inversa para f , entonces f es la inversa para f 1 .
4.1. CONCEPTO DE FUNCIN 159

Ejemplos:

y+2 x 2 1
a) Si y = f (x) = 3x + 2 ) y = 3x + 2 ) x = )f + , adems f f 1 (x) =
(x) =
3 3 3
x 2 x 2 3x + 2 2
f + = 3 + +2 = x y f 1 (f (x)) = f 1 ( 3x + 2) = + = x. b) Si y = f (x) =
3 3 3 3 3 3
3x + 2
x+2 x+2 3y + 2 3x + 2 3x + 2 +2
)y= )x= 1
) f (x) = 1
, adems f f (x) = f = x 1 =x
x 3 x 3 y 1 x 1 x 1 3x + 2
3
x 1
x+2
x+2 3 +2
yf 1
(f (x)) = f 1
= x 3 = x.
x 3 x+2
1
x 3
Observaciones. Dos funciones f y f 1 , mutuamente inversas y tambin biyectivas, siempre cumplen las
relaciones f f 1 (x) = x y f 1 (f (x)) = x. Las grcas de las funciones y = f (x) y y = f 1 (x), mutuamente
inversas, son simtricas con respecto a la bisectriz del primer cuadrante, es decir, con respecto a la recta y = x.

y 10

0
-10 -5 0 5 10

-5

-10

x 2
Las funciones y = f (x) = 3x + 2 e y = f (x) = + son mutuamente inversas y biyectivas, y sus grcas
3 3
son simtricas con respecto a la recta y = x.

y 20

10

0
-20 -10 0 10 20

-10

-20

x+2 3x + 2
Las funciones y = f (x) = e y = f (x) = son mutuamente inversas y biyectivas (x 6= 1; 3), y sus
x 3 x 1
grcas son simtricas con respecto a la recta y = x.

| Se debe tener en cuenta que el smbolo f 1 (B) es la preimagen del conjunto B Y , y suele asociarselo
con el smbolo f 1 de funcin inversa, sin embargo, la preimagen de un conjunto est denida para cualquier
aplicacin f : X ! Y , inclusive en el caso de que f no sea biyectiva, y por ende no posea inversa.

| Si las aplicaciones o funciones f : X ! Y y g : Y ! Z estn denidas mediante esos conjuntos de salida


y de llegada (por ejemplo, X; Y; Z R), entonces se puede denir una nueva aplicacin denominada funcin
160 CAPTULO 4. APLICACIONES, FUNCIONES NUMRICAS Y SUS GRFICAS

compuesta (en general, la composicin de funciones no es conmutativa) de las anteriores: g f : X ! Z, y


cuyo valor sobre los elementos x 2 X, se denen con la frmula

(g f ) (x) = g (f (x)) ; (en ese orden!)

| El dominio de la funcin compuesta g f se calcula a partir de la frmula (g f ) (x) = g (f (x)) :

Dom (g f ) = fx 2 R : (x 2 Dom f ) ^ (f (x) 2 Dom g)g


p p
As, por ejemplo, si f (x) = x2 y g (x) = 1 x, entonces (g f ) (x) = g (f (x)) = g x2 = 1 x2 ,
Dom f = R =] 1; +1[, Dom g = fx 2 R : 1 x 0g = fx 2 R : x 1g =] 1; 1] y Dom (g f ) =
fx 2 R : (x 2 Dom f ) ^ (f (x) 2 Dom g)g = x 2 R : (x 2] 1; +1[) ^ x2 2] 1; 1] =

fx 2 R : ( 1 < x < +1) ^ (x 2 [0; 1])g = fx 2 R : x 2 [0; 1]g = [0; 1].

Observacin. Es errrado pensar que el dominio Dom (g f ) se calcula partiendo del resultado directo de
la composicin obtenida entre f y g.

| Para la composicin con ms de 2 funciones, en cadena, se aplica la propiedad asociativa de la composicin


de funciones. As, si se tiene f : X ! Y , g : Y ! Z, h : Z ! T , entonces h (g f ) = (h g) f :

(h (g f )) (x) = h ((g f ) (x)) = h (g (f (x))) = (h g) (f (x)) = ((h g) f ) (x) ;

que es lo que se quera demostrar.

Ejemplos:

x 1 p x2 1
Si f (x) = , g (x) = x2 , h (x) = x 1, entonces: a) (f g) (x) = f (g (x)) = 2 , b) (g f ) (x) =
x+2 p x +2 r
2
x 1 x 1 1 x 1
g (f (x)) = , c) (f h) (x) = f (h (x)) = p , d) (h f ) (x) = h (f (x)) = 1 =
r x+2 x 1+2 x+2
3 p 2 p
, e) (g h) (x) = g (h (x)) = x 1 = x 1, f ) (h g) (x) = h (g (x)) = x2 1, g) (f g h) (x) =
x+2 !
2
x 1 1 x 2 x 1
f ((g h) (x)) = f (x 1) = = , h) (h g f ) (x) = h ((g f ) (x)) = h =
x 1+2 x+1 x+2
s s p r
2
x 1 6x 3 6x 3 3 3
1= 2 = , i) (g h f ) (x) = g ((h f ) (x)) = g =
x+2 (x + 2) jx + 2j x+2 x+2

| Es evidente que, por lo general, la operacin de composicin de funciones no es conmutativa (es decir,
f g 6= g f ), pero la composicin de 3 funciones si es asociativa.

Ecuaciones denidas a travs de aplicaciones o funciones Sea f una aplicacin de A en B; a todo


elemento b de B le corresponde una proposicin del tipo: p(x) :f (x) = b, donde x representa la variable y b el
trmino independiente de la ecuacin: f (x) = b. Decir que x es una solucin de esta ecuacin equivale a decir
que b es imagen de x, o bien que x pertenece a f 1 (b) (imagen recproca de b). Es obvio, que el subconjunto
f 1 (b) de A, est constituido por todas las soluciones de la ecuacin.

Recuerda que segn sea el carcter de la aplicacin f , existir una, ninguna o varias soluciones
de la ecuacin f (x) = b. As, por ejemplo, si:

1. f es inyectiva ) existe a lo sumo una solucin (9 a lo sumo un x 2 A) (f (x) = b). La grca de una
funcin inyectiva puede ser cortada a lo sumo en uno de sus puntos por cualquier recta paralela al eje
horizontal Ox.
4.1. CONCEPTO DE FUNCIN 161

2. f es sobreyectiva o exhaustiva ) existe al menos una solucin (9x 2 A) (f (x) = b). La grca de una
funcin sobreyectiva puede ser cortada como mnimo en uno de sus puntos por cualquier recta paralela al
eje horizontal Ox.

3. f es biyectiva ) existe una y slo una solucin (9!x 2 A) (f (x) = b). En cualquier caso existir al menos
una solucin, si b 2 f (A), y ninguna si b 2
= f (A). La grca de una funcin biyectiva (que puede ser o bien
slo creciente o bien slo decreciente) puede ser cortada nicamente en uno de sus puntos por cualquier
recta paralela al eje horizontal Ox.

| Recurdese que la imagen de un conjunto A mediante la aplicacin f viene dada mediante

f (A) = fb 2 B : 9a 2 A; f (a) = bg:

Recuerda las caracterizaciones


de las aplicaciones
Aplicacin o funcin: Caracterizacin:
9 solucin de la ecuacin f (x) = b , b 2 f (A):
f es inyectiva , 8b 2 B; 9 a lo sumo un x 2 A : f (x) = b
f es exhaustiva , 8b 2 B; 9x 2 A : f (x) = b
f es biyectiva , 8b 2 B; 9!x 2 A : f (x) = b
f es biyectiva, f : A ! B ) 9f 1 : B ! A
Composicin: Si f : A ! B y g : B ! C ) g f : A ! C, (g f ) (x) = g (f (x))

Recuerda algunos tipos de funciones


Funcin: Caracterstica funcional. Ejemplos: Caracterstica grca:
f (x) = f ( x) : Su grca es simtrica con
Par
y = jxj ; y = 3x4 + 5 respecto al eje vertical Oy:
f (x) = f ( x) : Su grca es simtrica con
Impar
y = x jxj ; y = x3 respecto al origen O:
f (x) = f (x T ), T = periodo. Su grca se repite periodio-
Peridica y = sin x; T = 2 ; y = cos 2x 1; dicamente en cualquier inter-
T = ; y = tan x; T = : valo de longitud T:
Creciente: x1 < x2 ) f (x1 ) < f (x2 ) : Mientras mayor es el valor de x,
y = 3x; y = 3x2 1; x 2 [0; +1[ mayor es el valor de la ordenada y:
Montona
Decreciente: x1 < x2 ) f (x1 ) > f (x2 ) : Mientras menor es el valor de x,
y = x + 1; y = x2 + 3; x 2 [0; +1[ mayor es el valor de la ordenada y:
f (x) = mx + b: La grca es una recta de pendien-
Lineal y = x 1; m = tan = 1 ) = 450 ; te (inclinacin) m = tan y ordena-
b= 1 da al origen b:
f (x) = ax2 + bx + c: La grca es una parbola con vr-
Cuadrtica b b
y = x2 ; y = 3x2 + x + 2 tice V 2a ; f 2a :
f (x) = QPmn (x)
(x) con Pn y Qm polinomios. La grca es una curva compleja
Racional x 1 3x3 +x+2
y= 2x2 +5 ; y= x2 x+1
que puede tener asntotas.

Ejemplos:

1. Dada la aplicacin f : R ! R mediante f (x) = 3x2 + 9x 37: Calcular las antiimagenes de 7. Es f


inyectiva?
1
Solucin: f ( 7) = x 2 R : f (x) = 3x2 + 9x 37 = 7 .
162 CAPTULO 4. APLICACIONES, FUNCIONES NUMRICAS Y SUS GRFICAS

3x2 + 9x 30 = 0 ) x2 + 3x 10 = (x + 5) (x 2) = 0 ) x1 = 5, x2 = 2.
As pues, f 1 ( 7) = f 5; 2g, y, por tanto, la aplicacin no puede ser inyectiva, ya que f ( 5) = f (2) = 7,
siendo 5 6= 2:
(
2x; si x 0
2. Probar que f : Z ! N denida por f (x) = es una aplicacin biyectiva.
2x 1, si x < 0
Solucin: En primer lugar veamos que f es inyectiva: Dados x, y 2 Z, si f (x) = f (y) se pueden considerar
4 casos:

a) Si x 0, y 0; entonces 2x = 2y ) x = y:
1
b) Si x 0, y < 0; entonces 2x = 2y 1 ) x + y = . Este caso hay que descartarlo, ya que por
2
ser x e y enteros, su suma nunca puede ser un nmero fraccionario.
1
c) x < 0, y 0 ) 2x 1 = 2y ) x + y = . Tambin hay que descartarlo por la misma razn que
2
en b).
d ) x < 0, y < 0 ) 2x 1 = 2y 1 ) 2x = 2y ) x = y. Fijmonos que en los 2 casos posibles
a) y d), se llega a la conclusin x = y. Por tanto, f es inyectiva.
Probemos ahora que f es exhaustiva: Dado un elemento arbitrario y 2 N, hemos de ver que existe
y y
un x 2 Z tal que f (x) = y. Si el nmero y es par, entonces 2 Z y bastar con tomar x = . Si el
2 2
nmero y es impar, entonces su antiimagen ser un conjunto negativo x; tal que: f (x) = 2x 1 =
1 y
y)x= :
2
Observacin. Al ser y impar su antiimagen x no puede ser mayor o igual que cero, ya que en este
1 y
caso f (x) = 2x es par. Por otra parte, si y es impar, 1 y es par negativo, por tanto 2 Z:
2

Figura 4.2: Izq. Leonhard Euler (Basilea, 1707 - San Petersburg, 1783) matematico suizo, uno de los ms grandes
de todos los tiempos. Realiz numerosos descubrimientos en el campo del Clculo, teora de nmeros y en todas las
ramas matemticas de su tiempo. Posea unos dotes extraordinarios para todo tipo de clculo y una memoria envidiable,
siendo una computadora humana. Der. El famoso jurista, lsofo, militar y gran matemtico francs, Ren Descartes
(Cartesius) (La Haye, Turena, 1596 - Estocolmo, 1650). Fundador de la geometra analtica e investigador en varios
campos de la fsica. Realiz trabajos sobre la solucin y acotacin de races de ecuaciones algebraicas, describi las
curvas algebraicas mediante ecuaciones e introdujo notaciones modernas para el lgebra y geometra. Estudio la esencia
del pensamiento y mtodo cientcos. Muri de pulmona en Estocolmo bajo extraas circunstancias.

3. Anlisis de funciones elementales:

a) Estudiar el carcter de las funciones reales de variable real (f : R ! R), denidas por:
p p x 2
f1 (x) = x3 3
, f2 (x) = + x2 4, f3 (x) = x2 + 4, f4 (x) = , f5 (x) = cos x dando en
x 3
primer lugar su dominio de denicin.
Solucin: Dom f1 = R:
Veamos si f1 es inyectiva: Dados x, y 2 R si f1 (x) = f1 (y) ) x3 3
= y3 3
) x3 y3 = 0
4.1. CONCEPTO DE FUNCIN 163

) x3 y 3 = (x y) x2 + xy + y 2 = 0 ) x y = 0 ) x = y, y, por tanto, f1 es inyectiva.


Observacin. Si fuese el segundo factor x + xy + y 2 el nulo, se tendra x2 + xy + y 2 = 0
2
p p
y y 2 4y 2 y 3y 2
)x= = , que nicamente tiene sentido, sobre R; cuando 3y 2 = 0;
2 2
o sea cuando y = 0; en cuyo caso x = 0 = y (igualmente).
Esta aplicacin f1 es tambin exhaustiva, ya que dado y 2 R arbitrario, existe x 2 R tal que f (x) = y:
p
Dicho elemento se obtiene resolviendo la ecuacin f (x) = x3 3
= y ) x3 = 3 +y ) x = 3 y + 3
(que siempre existe por ser el ndice de la raz impar).
La funcin f1 es, pues, biyectiva.
p
Sea f2 (x) = + x2 4, su Dom f = x 2 R : x2 4 > 0 , es decir, 8x 2 R, x2 4 ) jxj 2.
p
(Ver gura).Estudiemos el carcter inyectivo de f2 : Dados x; y 2 R, si f2 (x) = f2 (y) ) + x 2 4=

Figura 4.3: Ejemplo 3 a). Dominio de f2 (x).

p
+ y2 4 ) x2 4 = y2 4 ) x2 = y 2 ) jxj = jyj, lo que nos indica que, q
por ejemplo, los elementos
2 p
3 y +3 entre s distintos, tienen la misma imagen, ya que f2 ( 3) = + ( 3) 4) = 9 4 =
p
32 4 = f2 (3) :
p
Esta funcin real tampoco es exhaustiva por tomar siempre valores positivos (+ : : :):
p
f3 (x) = x2 + 4, Dom f3 = R, ya que la raz siempre tiene sentido por ser x2 + 4 > 0, 8x 2 R:
La funcin f3 (x) tampoco es inyectiva, po la misma razn que no lo era f2 (x). Tampoco es exhaus-
tiva por tomar nicamente valores negativos (debido a ello, los nmeros reales positivos no tienen
antiimagen).
x 2
f4 (x) = , Dom f4 = fx 2 R : x 3 6= 0g, es decir, Dom f4 = R f3g.
x 3
x 2 y 2
Veamos que f4 es inyectiva: Dados x; y 2 R, si f4 (x) = f4 (y) ) = )
x 3 y 3
) xy 3x 2y + 6 = xy 2x 3y + 6 ) x = y ) x = y.
f4 (x) tambin es exhaustiva: Dado y 2 R arbitrario, 9x 2 R f3g tal que f (x) = y; en efecto
x 2
f (x) = =y
x 3
2 3y
) x 2 = xy 3y ) x (1 y) = 2 3y ) x = (1).
1 y
Hay que sealar que este elemento x existir siempre y cuando sea y 6= 1, por lo que podemos armar
que f4 (x) es una biyeccin de R f3g en R f1g :
2 3x
La funcin recpoca f4 1 (x) viene dada por (1), es decir: f4 1 (x) =
1 x
f5 (x) = cos x; Dom f = R: Esta funcin no es inyectiva, ya que, por ej.: los puntos y + 2 tienen
la misma imagen.
Tampoco es exhaustiva, ya que Im f = [ 1; 1], y los puntos que estn fuera de este intervalo no tienen
antiimagen.
b) Dar las imgenes de x = por f1 f5 y f5 f1 :
Solucin: Sabemos que f1 (x) = x3 3
y f5 (x) = cos x, por tanto (f1 f5 ) ( ) = f1 (f5 ( )) =
3 3 3
f1 (cos ) = ( 1) = 1
3 3
(f5 f1 ) ( ) = f5 (f1 ( )) = f5 = f5 (0) = cos 0 = 1.
164 CAPTULO 4. APLICACIONES, FUNCIONES NUMRICAS Y SUS GRFICAS

3
c) Obtener las antiimgenes del 1 por f5 f1 , y de por f1 f5 :
Solucin:
(f5 f1 ) (x) = f5 (f1 (x)) = f5 x3 3
= cos x3 3

3 3
(f1 f5 ) (x) = f1 (f5 (x)) = f1 (cos x) = (cos x) = cos3 x 3

1
(f5 f1 ) (1) = fx 2 R : (f5 f1 ) (x) = 1g = x 2 R : cos x3 3
=1 =
n p o
= x 2 R : x3 3
= 2k , k 2 Z = x 2 R : x = 3 ( 2 + 2k), k 2 Z .
1 3 3
(f1 f5 ) = x 2 R : cos3 x
= x 2 R : (f1 f5 ) (x) = 3
= 3
=
n o
3
= x 2 R : cos x = 0 = fx 2 R : cos x = 0g = x 2 R : x = + k , k 2 Z .
2
2x 4
4. Sea f : R ! R una funcin denida por f (x) = :
x 5
a) Dar su dominio y su recorrido (imagen de f ).
Solucin: Dom f = fx 2 R : x 5 6= 0g = R f5g, Im f = fy 2 R : 9x 2 R f5g ; f (x) = yg
2x 4
El que f (x) = y ) = y ) 2x 4 = xy 5y )
x 5
4 5y
) x (2 y) = 4 5y ) x = (1)
2 y
Este elemento x 2 R existir siempre y cuando sea y 6= 2; por lo que Im f = R f2g.
b) Estudiar su carcter.
2x 4 2y 4
Solucin: Veamos que f es inyectiva: Dados x, y 2 R, si f (x) = f (y) ) =
x 5 y 5
) 2xy 10x 4y + 20 = 2xy 4x 10y + 20 ) 6x =
6y ) x = y.
4 5y
Tambin es exhaustiva, dado y 2 R f2g arbitrario, existe un x = (1), tal que f (x) = y. En
2 y
efecto:
4 5y 8 10y 8 + 4y
2 4
2x 4 2 y (2 y) 6y
f (x) = = = = = y.
x 5 4 5y 4 5y 10 + 5y 6
5
2 y (2 y)
1
c) Calcular la expresin de f (x) :
2x 4 4 5y
Solucin: En el apartado a) hemos visto que si f (x) = = y, entonces x = , pues bien,
x 5 2 y
4 5x
est ltima ecuacin nos da la expresin de la funcin recproca, que ser: f 1 (x) =
2 x
Observemos que al efectuar las composiciones:
2x 4
4 5
4 5f (x) x 5
f 1 f (x) = f 1 (f (x)) = = =
2 f (x) 2x 4
2
x 5
4x 20 10x + 20
x 5 6x
= 2x 10 2x + 4 = 6 = x. En ambos casos se obtiene la aplicacin identidad (Id (x) = x) :
x 5

5. Dadas f , g aplicaciones de R en R, denidas por f (x) = x 5 y g (x) = x2 + 7. Hallar (g f ) ( 4) y


(f g) (3) :
Solucin:
2 2
(g f ) (x) = g (f (x)) = (f (x)) + 7 = (x 5) + 7 = x2 10x + 25 + 7 = x2 10x + 32.
2
Por tanto (g f ) ( 4) = ( 4) 10 ( 4) + 32 = 16 + 40 + 32 = 88.
2
Otra forma (g f ) ( 4) = g (f ( 4)) = g ( 4 5) = g ( 9) = ( 9) + 7 = 81 + 7 = 88.
(f g) (3) = f (g (3)) f 32 + 7 = f (16) = 16 5 = 11.
4.1. CONCEPTO DE FUNCIN 165

6. Si f , g son aplicaciones de R ! R tal que g (x) = x3 , (g f ) (x) = x3 3x2 + 3x 1. Hallar f (x) :


3
Solucin: (g f ) (x) = g (f (x)) = (f (x)) = x3 3x2 + 3x 1.
3 3 3
Ahora bien, x3 3x2 + 3x 1 = (x 1) , por tanto: (f (x)) = (x 1) ) f (x) = x 1:

7. Dadas las aplicaciones: f : R ! R y g : R ! R, denidas mediante las frmulas f (x) = x3 + 1, g (x) =


p
3
x 1. Hallar las expresiones correspondientes a las aplicaciones compuestas g f y g 1 f 1 :
p p3
Solucin: (g f ) (x) = g(f (x)) = g(x3 + 1) = 3 (x3 + 1) 1 = x3 = x:
p p
Calculemos f 1 : y = x3 + 1(= f (x)) ) x3 = y 1 ) x = 3 y 1 ) f 1 (x) = 3 x 1:
p
Anlogamente para g 1 : y = 3 x 1(= g(x)) ) y 3 = x 1 ) x = y 3 + 1 ) g 1 (x) = x3 + 1:
p p
Por tanto: (g 1 f 1 )(x) = g 1 (f 1 (x)) = g 1 ( 3 x 1) = ( 3 x 1)3 + 1 = x 1 + 1 = x.
1 1
Observemos que tanto g f como g f son las aplicaciones identidad (Id(x) = x).
1 1
Adems se verica que: f = g, y g = f:

8. Dadas las aplicaciones: f : R ! R, g : R ! Z, h : Z ! Z; x ! f (x) = x3 , x ! g(x) = E(x)1 ,


x ! h(x) = x + 1. Se pide:

a) Estudiar su caracter.
Solucin: f es inyectiva: Si f (x) = f (y) ) x3 = y 3 ) x3 y3 = 0 )
(x y)(x2 + xy + y 2 ) = 0 ) (x y) = 0 ) x = y:
p
f es exhaustiva: 8y 2 R, 9x 2 R tal que f (x) = x3 = y; el elemento x es justamente 3 y, que por
p p
ser de ndice impar existe para todo y 2 R: Observemos que f ( 3 y) = ( 3 y)3 = y. Por tanto f es
biyectiva.
g no es inyectiva: Puesto que 2;11 6= 2;99 y, sin embargo, E(2; 11) = 2 = E(2; 99).
g es exhaustiva: 8n 2 Z, 9x = n 2 R tal que g(x) = E(n) = n.
h(x) = x + 1 es inyectiva: si h(x1 ) = h(x2 ) ) x1 + 1 = x2 + 1 ) x1 = x2 .
h es exhaustiva: 8y 2 Z, 9x 2 Z tal que h(x) = x + 1 = y; basta para ello con tomar x = y 1.
(h(y 1) = (y 1) + 1 = y):
1 1
b) Obtener: [h (g f )(x)], h (g f ) (x)
Solucin:
aso ciativa
[h (g f )] (x) = [(h g) f ] (x) = (h g)(f (x)) = h(g(x3 )) = h(E(x3 )) = E(x3 ) + 1.
Por ejemplo: [h (g f )] (2;5) = E((2;5)3 ) + 1 = E(15;625) + 1 = 15 + 1 = 16:
1
Determinemos ahora h (g f 1 ) (x): Para ello empecemos calculando f 1
yh 1
:
p p
f 1 ? : y = x3 (= f (x)) ) x = 3 y ) f 1 (x) = 3 x
1 1
h ? : y = x + 1(= h(x)) ) x = y 1)h (x) = x 1
1 1 1 1
Por tanto: h (g f ) (x) = (h g) f (x) =
1 1
p 1
p p 1
= (h g)(f (x)) = h (g( 3 x)) = h (E( x)) = E( 3 x) 1:
3

1
p
Por ejemplo: h (g f 1 ) ( 8) = E( 3 8) 1 = 2 1 = 3:

9. Demostrar que si f : E ! F y A E; B E; entonces se cumple que

f (A [ B) = f (A) [ f (B): (1)

Solucin: Por denicin f (A [ B) = ff (x) : x 2 A [ Bg. Sea f (x) 2 f (A [ B), entonces x 2 (A [ B), es
decir, x 2 A _ x 2 B. Pero si x 2 A _ x 2 B, entonces f (x) 2 f (A) _ f (x) 2 f (B) y f (x) 2 (f (A) [ f (B)).
Con esto hemos demostrado la inclusin

f (A [ B) (f (A) [ f (B)): (2)


1 E(x) o [x] representa a la parte entera de x, por ej.: E(1;234) = 1, E(2;99) = 2:
166 CAPTULO 4. APLICACIONES, FUNCIONES NUMRICAS Y SUS GRFICAS

Sea f (x) 2 (f (A) [ f (B)), entonces f (x) 2 f (A) _ f (x) 2 f (B), de


donde x 2 A _ x 2 B, es decir x 2 (A [ B), y por eso f (x) 2 f (A [ B) y

(f (A) [ f (B)) f (A [ B). (3)

10. Demostrar que si f : E ! F y A F; B F , entonces se cumplen las relaciones:

1 1 1
a) f (A \ B) = f (A) \ f (B)
1
Solucin: Tenemos x 2 f (A \ B), entonces f (x) 2 (A \ B), es decir f (x) 2 A ^ f (x) 2 B. Pero
entonces x 2 f 1 (A) ^ x 2 f 1 (B), y por consiguiente x 2 f 1 (A) \ f 1 (B) . De esta manera
hemos demostrado la inclusin

1 1 1
f (A \ B) f (A) \ f (B) :

Para la demostracin de la inclusin inversa, tendremos:


x 2 f 1 (A) \ f 1
(B) , entonces x 2 f 1
(A) ^ x 2 f 1
(B), es decir f (x) 2 A ^ f (x) 2 B, o lo
que es lo mismo
1
f (x) 2 (A \ B), lo que nos da que x 2 f (A \ B). Por consiguiente

1 1 1
(f (A) \ f (B)) f (A \ B):

1 1 1
b) f (A n B) = f (A) n f (B)
1
Solucin: Sea x 2 f (A n B), entonces f (x) 2 (A n B), es decir f (x) 2 A ^ f (x) 2
= B. Pero entonces
x 2 f 1 (A) ^ x 2
=f 1 1 1
(B), lo que nos da x 2 f (A) f (B) . De esta manera tendremos

1 1 1
f (A n B) f (A) f (B) :

Si x 2 f 1 (A) f 1 (B) , entonces x 2 f 1 (A) ^ x 2 = f 1 (B). De donde f (x) 2 A ^ f (x) 2


= B, es
1
decir f (x) 2 (A n B). Pero entonces x 2 f (A n B). Lo que demuestra que

1 1 1
f (A) f (B) f (A n B):

La inclusin inversa ya fue demostrada anteriormente, por tanto se cumple b).


1 1 1
c) f (A [ B) = f (A) [ f (B)
1
Solucin: Si x 2 f (A [ B), entonces f (x) 2 (A [ B). De donde f (x) 2 A _ f (x) 2 B, y por tanto
1 1 1 1
x2f (A) _ x 2 f (B), es decir x 2 f (A) [ f (B) . De esta manera conclumos que

1 1 1
f (A [ B) f (A) [ f (B) :

Ahora, si suponemos que x 2 f 1 (A) [ f 1 (B) , entonces x 2 f 1 (A) _ x 2 f 1


(B) y f (x) 2
A _ f (x) 2 B, o f (x) 2 (A [ B), de donde x 2 f 1 (A [ B). Por consiguiente

1 1 1
(f (A) [ f (B)) f (A [ B):

11. Sea f : E ! F; P la familia de subconjuntos o partes de E, Q la familia de subconjuntos o partes de F .


Representamos mediante

f (P ) = ff (A) 2 Q : A 2 P g ;
1 1
f (Q) = f (B) 2 P : B 2 Q :

Demostrar que:
4.1. CONCEPTO DE FUNCIN 167

1
a) Si Q es un anillo, f (Q) tambin lo es.
Solucin: Como Q es un anillo, entonces de B1 2 Q; B2 2 Q se sigue que (B1 [ B2 ) Q y
(B1 B2 ) Q. Entonces, de acuerdo al anterior ejemplo, tenemos

1 1 1 1
f (B1 ) [ f (B2 ) = f (B1 [ B2 ) 2 f (Q) ;
1 1 1 1
f (B1 ) f (B2 ) = f (B1 B2 ) 2 f (Q)

1
es decir f (Q) es un anillo.
b) Si P es un anillo, f (P ) no necesariamente lo es.
Solucin: Sean E = fa; b; c; dg, F = fa0 ; b0 ; c0 ; d0 g, con f (a) = a0 , f (b) = f (c) = b0 , f (d) = d0 : La
familia
P = ffa; b; c; dg ; fa; bg ; fc; dg ; ?g

es un anillo, sin embargo

f (fa; bg) nf (fc; dg) = fa0 ; b0 g n fb0 ; d0 g = fa0 g 2


= f (P ) =
= ffa0 ; b0 ; d0 g ; fa0 ; b0 g ; fd0 ; c0 g ; ?g

es decir, f (P ) no es anillo.

12. Averiguar cales de las siguientes funciones f : [0; 1] ! [0; 3] son: inyectivas, sobreyectivas y biyectivas

a) x ! 3x
Solucin: Si y 2 [0; 3], entonces la ecuacin y = 3x posee no ms de una solucin x 2 [0; 1] : As, si
y 2 [0; 3], la solucin es x = log3 y, y para y 2 [0; 1[ no hay soluciones. Por tanto x ! 3x es inyectiva.
y 3

2.5

1.5

0.5

0
0 0.25 0.5 0.75 1

Funcin inyectiva (creciente) y = 3x


16 1
b) x ! 3 x
3 4
1 2 1p
Solucin: Sea y 2 [0; 3]. La ecuacin y = 3 16 3 x 4 posee la solucin x1 = 14 4 9 3y,
8 1 1 1 p
3 y 3, perteneciente a 0; 4 , y la solucin x 2 = 4 + 4 9 3y, 0 y 3, perteneciente a
1
4 ; 1 . De esta manera para 8y 2 [0; 3], existe 1 o 2 preimgenes, por lo tanto f es sobreyectiva.

y 3

2.5

1.5

0.5

0
0 0.25 0.5 0.75 1

16 1 2
Funcin sobreyectiva y = 3 3 x 4
168 CAPTULO 4. APLICACIONES, FUNCIONES NUMRICAS Y SUS GRFICAS
2
1
c) x ! 12 x
2
2 p p
Solucin: De la ecuacin y = 12 x 12 , y 2 [0; 3], encontramos x1 = 12 12 y3 , x2 = 21 + 12 y3 ,
teniendo que si 0 < y 3, entonces ambas races pertenecen a ]0; 1]. Por lo tanto la funcin analizada
es sobreyectiva.
y 3

2 .5

1 .5

0 .5

0
0 0 .2 5 0 .5 0 .7 5 1

1 2
Funcin sobreyectiva y = 12 x 2

d ) x ! 2 jx + 2j 3
Solucin: Sea y 2 [0; 3], entonces la ecuacin y = 2 jx + 2j 3 para y 2 [1; 3] posee una nica
y 1
solucin x = ; si y 2 [0; 1[, entonces la ecuacin no posee soluciones pertenecientes al segmento
2
[0; 1]. Por tanto x ! 2 jx + 2j 3 es una funcin inyectiva.
y 3

2 .5

1 .5

0 .5

0
0 0 .2 5 0 .5 0 .7 5 1

Funcin inyectiva (creciente) y = 2 jx + 2j 3

Practico lo que aprend:

1. Dadas las funciones f : X ! Y , g : X ! Y , h : X ! Y , (X; Y R) denidas mediante f (x) = x2 + 1,


1
g (x) = 1 + x, h (x) = . Hallar las funciones: a) 3f , b) 0;5g, c) f + g, d) f g, e) 2f 3g,
x 1
f g f g f h g f
f ) f g, g) , h) , i) f g, j) g f , k) f h, l) f 2g + h, m) f g h, n) , o) , p) ,
g f h g h f g
f f h h
q) f h g g h f , r) h h, s) , t) g 1 , u) h 1 , v) g 1 g, r) h h 1 , s) f h 1 g 1 h,
g g f f
t) g 1 h 1 , u) h 1 f g 1 , v) f g 1 h.
3
2. Hallar los dominios y en lo posible las imgenes de las siguientes funciones numricas: a) f (x) = ,
p x2
x+1 5 x 3 x 3x 1
b) g (x) = , c) h (x) = 4 , d) g (x) = , e) h (x) = p , f ) f (x) = p ,
x 2 x x2 x x+2 x 2 x
2 1 p p p p p
g) h (x) = p p , h) g (x) = x 3 2x, i) f (x) = 2 + x 3 2x, j) h (x) = 2x +
1 x 4 x
p p x+1 2x + 5 1 5
3 x 1 + 4x, k) h (x) = p , l) f (x) = p q , m) g (x) = p p ,
(x 3) 2 x 2
4
x + 2 3
x
x + 3 (x 1)
x 1 x+2 1
n) h (x) = p5
p3
, n) g (x) = p p
x 1 x+2 (x 5) 8 x 4 x 4 x + 3
4.2. CONSTRUCCIN DE GRFICAS DE FUNCIONES 169

3. Demostrar que si f : E ! F , A E, B E, entonces:

a) f (A \ B) (f (A) \ f (B)) ; b) (f (A) n f (B)) f (A n B) :

1 1
4. Sean f : E ! F , A F, B F . Demostrar que si A B, entonces f (A) f (B) :

5. Demostrar que si f : E ! F y A E, B F , entonces:

a) A f 1 (f (A)) ; b) f f 1 (B) = B;
c) f (A) \ B = f A \ f 1 (B) ; d) (f (A) B) , A \ f 1
(B) ;
e) (f (A) \ B = ?) , A \ f 1 (B) = ?

6. Cales de las funciones f : [ 1; 1] ! [0; 1]

a) x ! cos 2x ; b) x ! x2 + 1; c) x ! jxj ;
x+1 2x 1
d) x ! ; e) x ! ; f) x ! 2x 1
2 3

son inyectivas, sobreyectivas o biyectivas? Construir sus grcas.

7. Hallar el dominio reducido de biyectividad (es decir, aquellas partes donde f es biyectiva) para las fun-
ciones:

a) f (x) = x2 ; x 2 R; b) f (x) = 10x ; x 2 R;


x2
c) f (x) = x3 + x + 1; x 2 R; d) x ! 1
4

8. Hallar las expresiones explcitas de las funciones dadas paramtricamente, es decir, eliminar el parmetro
t y obtener la relacin y = f (x):

2 at 2 at2 2 at 2 at2
a) x = ; y = ; t 2 [0; +1[ b) x = ; y = ; t 2 [ 1; 0[ (a > 0) :
1 + t2 1 + t2 1 + t2 1 + t2

4.2. Construccin de Grcas de Funciones

A continuacin presentamos las principales transformaciones, operaciones y ejemplos realizables con la gr-
cas de las funciones. Dada la funcin y = f (x) y su grca, se trata de construir las grcas de las funciones
transformadas.

Grcas de varias funciones elementales Las grcas de las principales funciones elementales estn
descritas en el cuadro de principales funciones elementales (ver grca).

| Muchas de las grcas de funciones elementales se construyen fundamentalmente por tabulacin (con
uso de calculadora o computadora), es decir, conociendo la relacin y = f (x) hallamos la tabla de valores
x x1 x2 x3
y luego en el plano cartesiano unimos los puntos (x1 ; y1 ), (x2 ; y2 ), (x3 ; y3 ),
y = f (x) y1 y2 y3
(x4 ; y4 ), etc.
170 CAPTULO 4. APLICACIONES, FUNCIONES NUMRICAS Y SUS GRFICAS

Funciones: Recuerda la transformacin grca de y = f (x) :


f (x) + A, A 6= 0 Traslacin en A unidades a lo largo del eje Oy
f (x a), a 6= 0 Traslacin en a unidades a lo largo del eje Ox
kf (x), k > 0 k 6= 1 Dilatacin u homotecia a lo largo del eje Oy
f (kx), k > 0 k 6= 1 Contraccin a lo largo del eje Ox
f (x) Simetra con respecto al eje Ox
Eliminacin de la grca de la parte
jf (x)j
f (x) < 0 por reexin respecto a Ox
f ( x) Simetra con respecto al eje Oy
f (jxj) Simetra de valores x 0 con respecto al eje Oy
Si y = f (x) equivale a F (x; y) = 0
Existe simetra con respecto al origen O = O (0; 0)
y F (x; y) = F ( x; y)
Si f es funcin biyectiva, entonces la funcin inversa
f 1 = g se obtine permutando las variables x e y y luego
y = f (x) ) x = f (y) despejando y (y = g (x)) de la relacin obtenida. Si f no
) y = g (x), (g = f 1 ) es biyectiva, entonces x = f (y) (o y = g (x)) es una rela-
cin no funcional. En ambos casos las grcas de f y de
g = f 1 son simtricas con respecto al origen O = O (0; 0)

Principales funciones elementales.


4.2. CONSTRUCCIN DE GRFICAS DE FUNCIONES 171

4.2.1. Construccin de Rectas y Parbolas

Ecuacin de la lnea recta y su grca

La funcin del tipo f (x) = mx + b o y = mx + b se denomina funcin lineal y su representacin grca,


lnea recta. El coeciente m se denomina pendiente o inclinacin de la recta en un ngulo con respecto a
la horizontal, su valor se calcula mediante la relacin m = tan . El valor b se denomina ordenada al origen
O y nos da el corte o distancia existente entre el punto de interseccin de la recta con el eje Oy y el origen
de coordenadas O = (0; 0). Es evidente que aquellas rectas que poseen igual pendiente m son rectas paralelas,
casos particulares de rectas paralelas son las de ecuaciones x = a o y = b, las primeras son rectas paralelas al
eje Oy, y las segundas son paralelas al eje Ox. El mismo eje horizontal Ox tiene por ecuacin a y = 0, el eje
vertical tiene por ecuacin a x = 0.

| En Geometra Analtica se demuestra que dos rectas l1 y l2 son perpendiculares siempre y cuando entre
1
sus pendientes m1 y m2 se cumpla la relacin m1 = (es decir las pendientes son inversas y de signo
m2
contrario). Estas rectas forman entre s un ngulo recto, as, por ejemplo, las rectas x = 2 e y = 3, y las rectas
l1 : y = 2x 3 y l2 : y = 12 x + 4 son dos parejas de rectas mutuamente perpendiculares.

| Para construr una lnea recta es suciente conocer dos de sus puntos (distintos) P (x1 ; y1 ) y Q (x2 ; y2 ), los
mismos que pueden ser unidos por el segmento P Q. A la inversa, conocida la ecuacin de la recta y = mx + b,
tendremos que esta se satisface para los valores (x1 ; y1 ) y (x2 ; y2 ) siempre y cuando estos puntos pertenezcan a la
recta, es decir, se cumplan las relaciones y1 = mx1 +b e y2 = mx2 +b, esto puede ser representado tabularmente:
x y
x1 y1
.
x2 y2

| Dados 2 puntos distintos, P (x1 ; y1 ) y Q (x2 ; y2 ), la pendiente de la recta l que pasa por esos puntos viene
y2 y 1
dada por la relacin m = = tan , y la ecuacin de l viene dada mediante
x2 x1
y2 y1
l : y y1 = m (x x1 ) o l : y y1 = (x x1 )
x2 x1
La primera ecuacin suele denominarse ecuacin punto - pendiente, y la segunda ecuacin de la recta que pasa
por 2 puntos.

| Dada la funcin lineal f (x) = mx + b o y = mx + b, su funcin inversa se obtiene de esta ltima al


x b
permutar x con y, es decir, de x = my + b se tiene y = g (x) = y su grca es simtrica con respecto a la
m
x+2
bisectriz y = x. As, por ejemplo, para y = 3x 2, se tiene x = 3y 2 ) y = es la funcin inversa (ver
3
grca).

y
7 .5

2 .5

0
-7 .5 -5 -2 .5 0 2 .5 5 7 .5

x
-2 .5

-5

-7 .5

Para y = f (x) = 3x 2, se tiene x = 3y 2 ) y = x+2


3 es la funcin inversa de la dada, simtrica con
respecto a y = x.
172 CAPTULO 4. APLICACIONES, FUNCIONES NUMRICAS Y SUS GRFICAS

Figura 4.4: Izq. El famoso matemtico francs Laurent Schwartz (Pars, 1915 - ), medalla de oro y premio Fields (1950),
creador junto con el ruso Sergei L. Soboliev (Peterburg, 1908 - ) de la teora de distribuciones o funciones generalizadas.
Introdujo una axiomtica funcional para las distribuciones y varias aplicaciones para las ecuaciones diferenciales en
derivadas parciales. Conocido como autor del clebre curso de Anlisis Matemtico moderno de la Escuela Politcnica de
Paris y de otros textos de aplicaciones de las matemticas. Der. El hombre ms persistente para vencer al gran enigma
o conjetura de Fermat (desde 1994 teorema), el clebre matemtico ingls, Andrew Wiles (1953 -). Gran especialista en
geometra algebraica y curvas elpticas. El gran enigma planteado por Fermat (Beaumont de Lomagne, 1601 - Castres,
1665) que consiste en: sea N 3 n 3, no existe ninguna terna (x; y; z) de enteros no nulos tales que cumplan con la
ecuacin diofntica xn + y n = z n .

Ejemplos:

1. En las siguientes grcas presentamos conjuntos de rectas paralelas:

y 5

2.5

0
-2.5 -1.25 0 1.25 2.5

-2.5

-5

Conjunto de rectas paralelas al eje vertical Oy (x = 0). De izquierda a derecha tenemos: x = 2;5,
x = 1, x = 1;25, x = 2, x = 2;5. Se considera que para este tipo de rectas la pendiente es innita, pues
tan 900 ! 1.

0
-5 -2.5 0 2.5 5

-2

-4

Conjunto de rectas paralelas al eje horizontal Ox (y = 0). De arriba hacia abajo tenemos: y = 3, y = 2,
y = 1, y = 3, y = 5. Para todas estas rectas la pendiente es nula, es decir, m = 0.
4.2. CONSTRUCCIN DE GRFICAS DE FUNCIONES 173

y
15

10

0
-5 -2.5 0 2.5 5

x
-5

-10

-15

Ejemplos de rectas paralelas de pendiente m = 2 y valores distintos de b. De arriba abajo tenemos:


y = 2x + 6, y = 2x + 5, y = 2x + 2, y = 2x, y = 2x 1, y = 2x 5, y = 2x 7. Para la pendiente
tenemos tan = 2 ) 63:4320 :

2. En la siguiente grca presentamos un conjunto de rectas con la misma ordenada al origen:

20

0
-5 -2.5 0 2.5 5

-20

-40

Ejemplos de rectas no paralelas de igual ordenada al origen b = 5 y valores distintos para la pendiente
m. En sentido horario tenemos: y = 2x 5, y = 5x 5, y = 8x 5, y = x 5, y = 4x 5,
y = 0;2x 5, y = 6x 5.

3. Hallar la ecuacin de la recta y = mx + b que pasa por los puntos P ( 3; 1) y Q (5; 2) :

Solucin: P ( 3; 1) y Q (5; 2) pertenecen a la recta l : y = mx + b siempre y cuando se satisfagan las


relaciones:
(
P ( 3; 1) : 1 = 3m + b ) b = 1 + 3m
) 1 + 3m = 2 5m ) 8m = 1 ) m = 1=8 ) b = 11=8;
Q (5; 2) : 2 = 5m + b ) b = 2 5m

1 11 1
de donde tendremos que la recta buscada es y = x+ , su pendiente m = tan = , y entonces
8 8 8
= arctan (1=8) 7;12470 (ver grca).

y2 y1
Por otro lado, utilizando la frmula y y1 = (x x1 ), llegamos al mismo resultado: y 1 =
x2 x1
12 1 1 11 y2 y1
(x ( 3)) o y 1= (x + 3) que nos da y = x + , donde la pendiente m = =
5 ( 3) 8 8 8 x2 x1
2 1 1
= :
5 ( 3) 8
174 CAPTULO 4. APLICACIONES, FUNCIONES NUMRICAS Y SUS GRFICAS

y
7.5

2.5

0
-10 -5 0 5

-2.5

-5

1 11
Grca de la recta y = x + que pasa por los puntos P ( 3; 1) y Q (5; 2) y de las rectas
8 8
l1 : y = 2x + 10 y l2 : 3x + 2y + 1 que se cortan en el punto R (x0 ; y0 ) = R ( 3; 4) :

4. Dadas las ecuaciones de las rectas l1 : y = 2x + 10 y l2 : 3x + 2y + 1 = 0, hallar su punto de interseccin.


Solucin: El punto R (x0 ; y0 ) = l1 \ l2 de interseccin entre las rectas se lo obtiene resolviendo el sistema
(Ver grca anterior):
( 8
l1 : y = 2x + 10 < l1 : y = 2x + 10 3 1
) 3 1 ) 2x + 10 = x )
l2 : 3x + 2y + 1 = 0 : l2 : y = x 2 2
2 2
3 1
2x + x = 10 ) x = 3 ) y = 4 ) R (x0 ; y0 ) = R ( 3; 4)
2 2
5. Dada la recta l : y = 2x 1 y los puntos P (3; 2) y Q (1; 5). Hallar: a) la recta l1 paralela a l que pasa
por P ; b) la recta l2 perpendicular a l y que pasa por Q. Construr las grcas.
Solucin: a) La pendiente de la recta l : y = 2x 1 es m = 2 y la recta buscada de la misma pendiente
l1 : y = 2x + b pasa por P (3; 2), entonces se cumple la relacin 2 = 2 (3) + b ) b = 8. Por tanto, la
recta buscada es l1 : y = 2x + 8, que para gracarla basta tomar otro punto suyo distinto de P:
b) La pendiente de la recta l : y = 2x 1 es m = 2 y la recta perpendicular buscada ser de pendiente
1 1 1
m2 = = , es decir, l2 : y = x + b, que adems pasa por Q (1; 5), entonces se cumple la relacin
2 2 2
1 9 1 9
5 = (1) + b ) b = . Por tanto, la recta buscada es l2 : y = x + , que para gracarla basta tomar
2 2 2 2
otro punto suyo distinto de Q. Es evidente que la recta l2 es perpendicular simultneamente tanto a l
como a l1 , pues lkl1 y l?l2 , l1 ?l2 (ver grca).

y
10

7.5

2.5

0
-10 -5 0 5 10

x
-2.5

-5

La recta l : y = 2x 1 es paralela a la recta l1 : y = 2x + 8 y perpendicular a la recta l2 : y = 21 x + 92 .


La recta l1 pasa por el punto P (3; 2), y la recta l2 pasa por el punto Q (1; 5). La recta l2 es
perpendicular simultneamente tanto a l como a l1 , pues lkl1 y l?l2 , l1 ?l2 .

6. Dada la funcin lineal y = f (x) = 3x 2 y su grca. Construir las grcas de las funciones:

a) y = 3 (x + 4) 2 = 3x + 10 (traslacin horizontal)
Solucin:
4.2. CONSTRUCCIN DE GRFICAS DE FUNCIONES 175

20

10

0
-5 - 2 .5 0 2 .5 5

-1 0

Grcas de y = 3x 2 e y = 3 (x + 4) 2:

b) y = (3x 2) + 3 = 3x + 1 (traslacin vertical)


Solucin:
y
15

10

0
-5 - 2 .5 0 2 .5 5

x
-5

-1 0

-1 5

Grcas de y = 3x 2 e y = (3x 2) + 3:
c) y = (3x 2) (simetra con respecto a Ox)
Solucin:
y
15

10

0
-5 - 2 .5 0 2 .5 5

x
-5

-1 0

-1 5

Grcas de y = 3x 2 ey= (3x 2) :


d ) y = 3 ( x) 2= 3x 2 (simetra con respecto a Oy)
Solucin:
y

10

0
-5 - 2 .5 0 2 .5 5

x
-5

-1 0

-1 5

Grcas de y = 3x 2 e y = 3 ( x) 2:
e) y = 2 (3x 2) (dilatacin a lo largo de Oy)
Solucin:
176 CAPTULO 4. APLICACIONES, FUNCIONES NUMRICAS Y SUS GRFICAS

y 25

1 2 .5

0
-5 -2 .5 0 2 .5 5

-1 2 .5

-2 5

Grcas de y = 3x 2 e y = 2 (3x 2) :

f ) y = 3 (2x) 2 (contraccin a lo largo del eje Ox)


Solucin:
y
25

1 2 .5

0
-5 -2 .5 0 2 .5 5

-1 2 .5

-2 5

Grcas de y = 3x 2 e y = 3 (2x) 2:

g) y = j3x 2j (eliminacin de la grca de f (x) < 0)


Solucin:
y
15

10

0
-5 -2 .5 0 2 .5 5

x
-5

-1 0

-1 5

Grcas de y = 3x 2 e y = j3x 2j :

h) y = 3 jxj 2 (simetra de valores x 0 con respecto al eje Oy)


Solucin:
y

10

0
-5 -2 .5 0 2 .5 5

x
-5

-1 0

-1 5

Grcas de y = 3x 2 e y = 3 jxj 2:
4.2. CONSTRUCCIN DE GRFICAS DE FUNCIONES 177

Practico lo que aprend:

1. Hallar las ecuaciones de los 3 lados de los tringulos de vrtices A, B y C y gracarlos: a) A ( 8; 3),
B (2; 7), C (8; 1); b) A ( 6; 6), B (3; 8), C (6; 2); c) A ( 5; 4), B (1; 8), C (8; 0); d) A ( 8; 0), B (0; 7),
C (9; 0); e) A ( 6; 8), B (0; 6), C (7; 5).

2. Hallar las ecuaciones de las 3 alturas a los 3 lados de los tringulos de vrtices A, B y C, gracar y
comprobar que las alturas se cortan en un punto (ortocentro): a) A ( 4; 0), B (0; 8), C (4; 0); b) A ( 6; 4),
B (2; 6), C (6; 4); c) A ( 8; 0), B (0; 8), C (8; 0); d) A (0; 0), B (6; 6), C (6; 6); e) A ( 7; 0), B (1; 8),
C (1; 8).

3. Hallar las ecuaciones de las rectas paralelas a los 3 lados de los tringulos de vrtices A, B y C, que
pasan por los vrtices opuestos, gracarlas y comprobar que forman otro tringulo semejante al dado:
a) A ( 6; 2), B (0; 8), C (8; 0); b) A ( 6; 5), B (1; 6), C (5; 4); c) A ( 8; 8), B (1; 8), C (8; 4); d)
A ( 8; 0), B (0; 8), C (8; 0); e) A ( 6; 0), B (0; 6), C (0; 0).

Ecuacin de la parbola y su grca

Presentamos ahora algunas cuestiones pertenecientes a la grca de la funcin cuadrtica f (x) = ax2 +bx+c
(y = ax2 + bx + c), a 6= 0 o parbola. El coeciente a 6= 0 nos da la orientacin (vrtice hacia abajo cuando
a > 0, o vrtice hacia arriba cuando a < 0) y abertura (para 0 < a < 1 la abertura es mas grande que para
b b
a 1) de la parbola. El vrtice V de la parbola posee coordenadas V ;f (ver grca).
2a 2a

25

0
-10 -5 0 5 10

-25

-50

Presentamos 3 grcas de parbolas con valores de (a positivos) a = 0;25, c = 4, V (0; 4); a = 1, c = 16,
V (0; 16); y a = 4, c = 64, V (0; 64), respectivamente: y = 0;25x2 4, y = x2 16, y = 4x2 64.

20

0
-10 -5 0 5 10

-20

-40

Las 3 grcas de parbolas posen valores negativos para a: a = 1=4, b = 34 , c = 1, V ( 3=2; 25=16); a = 1,
b = 3, c = 4, V ( 3=2; 25=4); y a = 4, b = 12, c = 16, V ( 3=2; 25), respectivamente: y = 41 x2 34 x + 1,
y = x2 3x + 4, y = 4x2 12x + 16.
178 CAPTULO 4. APLICACIONES, FUNCIONES NUMRICAS Y SUS GRFICAS

| El punto de corte de la parbola con el eje vertical Oy se obtiene colocando el valor x = 0 en la relacin
y = ax2 + bx + c, es decir y = c.

| Los 2 puntos (si estos existen!) de corte de la parbola con el eje horizontal Ox se obtienen colocando el
valor y = 0 en la relacin y = ax2 +bx+c, es decir ax2 +bx+c = 0. Estos 2 puntos o valores x1 y x2 , mejor dicho
(x1 ; 0) y (x2 ; 0), se denominan races de la ecuacin cuadrtica ax2 + bx
p + c = 0 (esta ecuacinpser estudiada
b b2 4ac b + b2 4ac
con detalle ms tarde) y se calculan mediante las frmulas x1 = y x2 = .
2a 2a
| La expresin = b2 4ac se denomina discriminante de la funcin cuadrtica y = ax2 + bx + c (de la
ecuacin cuadrtica ax2 +bx+c = 0). Para la grca de la parbola se distinguen 3 casos: a) Si = b2 4ac > 0,
entonces x1 y x2 son valores reales distintos y la parbola corta al eje Ox en los puntos (x1 ; 0) y (x2 ; 0), teniendo
x1 + x2 x1 + x2
para el vrtice V ;f . b) Si = b2 4ac = 0, entonces x1 y x2 son valores reales
2 2
b
iguales y la parbola corta al eje Ox en un solo punto (x1 ; 0) = (x2 ; 0) = V ; 0 . c) Si = b2 4ac < 0,
2a
entonces x1 y x2 son valores imaginarios distintos y la parbola no corta al eje Ox, teniendo para el vrtice
b b
V ;f (ver grca).
2a 2a

y 50

37. 5

25

12. 5

0
-5 0 5 10

x
-12. 5

-25

Tres casos de grcas de parbolas (de izquierda a derecha): a) y = x2 x 20, = b2 4ac = 25 > 0,
x1 = 4, x2 = 5, V (1=2; 81=4); b) y = 3x2 + 18x + 31, = b2 4ac = 48 < 0, x1 y x2 son imaginarias
V ( 3; 4); c) y = 5x2 20x + 20, = b2 4ac = 0, x1 = x2 = 2, V (2; 0).

| Para obtener la ecuacin de una parbola es suciente conocer 3 de sus puntos (distintos y no alineados)
P (x1 ; y1 ), Q (x2 ; y2 ) y R (x3 ; y3 ). A la inversa, conocida la ecuacin de la parbola y = ax2 + bx + c, necesitamos
varios puntos (no alineados) para construir su grca: (x1 ; y1 ), (x2 ; y2 ), (x3 ; y3 ), etc., es decir, que cumplan las
x y
x1 y1
relaciones y1 = ax21 + bx1 + c, y2 = ax22 + bx2 + c, etc. Esto puede ser representado tabularmente: .
x2 y2

En calidad de ejemplo, busquemos la ecuacin de la parbola que pasa por P (x1 ; y1 ) = P ( 1; 0), Q (x2 ; y2 ) =
Q (5; 0) y R (x3 ; y3 ) = R (2; 8):
8 2
8
>
< P ( 1; 0) : 0 = a ( 1) + b ( 1) + c >
< a b+c=0
2 2
y = ax + bx + c ) Q (5; 0) : 0 = a (5) + b (5) + c ) 25a + 5b + c = 0
>
: 2 >
:
R (2; 8) : 8 = a (2) + b (2) + c 4a + 2b + c = 8
En este sistema de ecuaciones eliminamos c de las 2 primeras ecuaciones por suma y resta: a 25a b 5b +
c c == 0, entonces tendremos 24a 6b = 0 o 4a + b = 0. Ahora eliminamos c de la segunda y tercera:
25a 4a+5b 2b+c c = 0 8, entonces tendremos 21a+3b = 8. Finalmente, entre 4a+b = 0 y 21a+3b = 8
eliminamos b: 12a + 3b 21a 3b = 0 + 8, entonces 9a = 8 y a = 8=9, entonces b = 32=9 y c = 40=9. En
8 2 32 40
denitiva obtenemos la ecuacin de la parbola y = ax2 + bx + c = x + x+ (ver grca).
9 9 9
4.2. CONSTRUCCIN DE GRFICAS DE FUNCIONES 179

y 10

x
-5 -2.5 0 2.5 5 7.5 10
0

-10

-20

-30

8 2 32 40
Grca de la parbola y = x + x+ que pasa por los puntos P ( 1; 0), Q (5; 0) y R (2; 8), y cuyo
9 9 9
vrtice es V (2; 8).

| Para construir una parbola es conveniente saber que su grca es simtrica con respecto a la recta vertical
b
x= (ver grca).
2a

y
37.5

25

12.5

0
-7.5 -5 -2.5 0 2.5 5

x
-12.5

-25

Cada una de las siguientes parbolas es simtrica con respecta a la recta vertical x = 2ab : a) y = x2 + 4x 5 es
simtrica con respecto a la recta x = 2, x1 = 5, x2 = 1 y V ( 2; 9); b) y = x2 + 2x 3 es simtrica con
respecto a la recta x = 1, x1 = 3, x2 = 1 y V ( 1; 4); c) y = x2 + 2x es simtrica con respecto a la recta
x = 1, x1 = 0, x2 = 2 y V (1; 1).

| Para construir la relacin inversa (pero no funcin inversa ya que y = f (x) = ax2 +bx+c no es biyectiva!)
se permutan en y = f (x) = ax2 + bx + c las variables x e y, es decir, se obtiene x = g (y) = ay 2 + by + c, y
para esta relacin se aplican todas las caractersticas expuestas anteriormente, pero con la particularidad que se
obtendr una parbola horizontal o acostada. Estas 2 grcas son simtricas con respecto a la bisectriz y = x.
As, por ejemplo, para construir la relacin inversa (no funcin!) de la funcin y = f (x) = x2 5x, permutamos
p
x con y, obtenemos x = g (y) = y 2 5y y despejamos y para obtener dos funciones: y = 21 4x + 25 + 52 e
p
y = 25 21 4x + 25. Uniendo estas 2 funciones obtenemos la relacin inversa buscada, la misma que es simtrica
a la dada con respecto a la bisectriz y = x (ver grca).
180 CAPTULO 4. APLICACIONES, FUNCIONES NUMRICAS Y SUS GRFICAS

y 10

7.5

2.5

0
-5 -2.5 0 2.5 5 7.5 10

x
-2.5

-5

-7.5

Construccin de la relacin inversa correspondiente a la funcin y = x2 5x: permutamos x con y, obtenemos


p p
x = y 2 5y, despejamos y para obtener 2 funciones: y = 21 4x + 25 + 52 e y = 52 21 4x + 25. Al unir estas 2
funciones obtenemos la relacin inversa buscada, la misma que es simtrica a la dada con respecto a la
bisectriz y = x.

Ejemplos:

1. Dada la funcin cuadrtica y = f (x) = x2 y su grca. Construir las traslaciones:

a) y = x2 3
Solucin:
y 5

2. 5

0
-5 -2. 5 0 2. 5 5

-2. 5

-5

Grcas de y = x2 e y = x2 3:
2
b) y = (x 4)
Solucin:
y

3.75

2.5

1.25

0
-2.5 0 2.5 5 7.5

2
Grcas de y = x2 e y = (x 4) :
2
c) y = (x + 5)
Solucin:
4.2. CONSTRUCCIN DE GRFICAS DE FUNCIONES 181

3.75

2.5

1.25

0
-7.5 -5 -2.5 0 2.5

2
Grcas de y = x2 e y = (x + 5) :

2. Dada la funcin cuadrtica y = x2 . Construr las traslaciones dobles:

2
a) y = (x 3) + 5
Solucin:

10

7.5

2.5

0
-2.5 0 2.5 5

2
Grcas de y = x2 e y = (x 3) + 5:

2
b) y = (x + 2) 4
Solucin:

2. 5

0
-5 -2. 5 0 2. 5 5

-2. 5

-5

2 2
Grcas de y = x e y = (x + 2) 4:

3. Dada la funcin cuadrtica y = x2 . Construir las homotecias:

a) y = 2x2 ; y = 31 x2
Solucin:
182 CAPTULO 4. APLICACIONES, FUNCIONES NUMRICAS Y SUS GRFICAS

3.75

2.5

1.25

0
-5 -2.5 0 2.5 5

Grcas de y = x2 , y = 2x2 e y = 13 x2 :

Practico lo que aprend:

1. Grcar las parbolas, determinar sus caractersticas (cortes, vrtices, races) y construir las relaciones
inversas con sus grcas: a) f (x) = 0;25x2 4, b) f (x) = 4x2 + 1, c) f (x) = x (x 4), d) f (x) =
(x 3) (x + 6), e) f (x) = (x + 5) (x 1), f ) f (x) = 0;5x (x + 3) :

2. Dadas las funciones: a) f (x) = 4x 1, b) f (x) = 3x + 1, c) f (x) = x2 9, d) f (x) = x2 + 4x 5,


3 2
e) f (x) = , f ) f (x) = 2 jxj, g) f (x) = 2 (x + 3) , h) f (x) = 2=x, i) f (x) = x2 + 2x 15.
x 2
Construir para cada una de las funciones anteriores las siguientes grcas: 1) y = f (x), 2) y = 2f (x),
3) y = 3f (x), 4) y = f (x + 1), 5) y = f (x 2), 6) y = 2f (x + 1), 7) y = 3f ( x), 8) y = 2f (x) 3,
9) y = f (x) + 4, 10) y = f (2x), 11) y = f (3x 1), 12) y = 2f ( 2x + 5), 13) y = f (2x),

14) y = f (3x 2), 15) y = 3f ( 2x + 3), 16) y = f ( 3x), 17) y = f (3x 1), 18) y = 2f ( 2x + 3),

19) y = jf (x)j, 20) y = f (jxj), 21) y = jf (x) 1j, 22) y = 2 jf ( 2x + 3)j, 23) y = 2 jf (x)j + 4,

1
24) y = f (jxj 1), 25) y = 2 jf (3 jxj) 2j :
2

4.3. Prcticas con las Grcas de Algunas Funciones y Relaciones

A continuacin presentamos varios ejemplos de construccin de grcas matemticas de funciones y relaciones


mediante tabulacin, donde se pide encontrar: la grca de la curva por tabulacin (es decir, dando valores a x
obtener las y, o viceversa); sus cortes con los ejes Ox y Oy (es decir, los valores de x cuando y = 0, y los valores
de y cuando x = 0); sus simetras (simetra con respecto al eje Oy cuando F (x; y) = F ( x; y) o f (x) = f ( x);
simetra con respecto al eje Ox cuando F (x; y) = F (x; y) o g (y) = g ( y), simetra con respecto al origen
O cuando F (x; y) = F ( x; y)); asntotas (asntota horizontal y = b cuando los valores de x se acercan a
cierto valor nito o innito y la y tiende al valor b, asntota vertical x = a cuando los valores de y se acercan a
cierto valor nito o innito y la x tiende al valor a); puntos mximos y mnimos (es decir aquellos puntos donde
y = f (x) toma el valor mximo de entre todos los valores posibles de x, o aquellos puntos donde x = g (y) toma
el valor mximo de entre todos los valores posibles de y)2 , etc.

2 Los conceptos generales de asntotas, puntos mximos y mnimos tienen que ver con el clculo diferencial. Consultar los cursos
[1], [11] y [13] de la bibliografa.
4.3. PRCTICAS CON LAS GRFICAS DE ALGUNAS FUNCIONES Y RELACIONES 183

Recuerda sobre la funcin lineal y su grca:


Funcin lineal: Caractersticas:
y = f (x) = mx m es la pendiente de la recta, m = tan . La recta pasa por el origen O (0; 0)
y=b Rectas paralelas al eje horizontal. La recta es paralela al eje Ox
x=a Rectas paralelas al eje vertical. La recta es paralela al eje Oy
m es la pendiente de la recta. m = tan , es el ngulo de inclinacin
y = f (x) = mx + b de la recta con respecto a la horizontal Ox
b es la ordenada al origen (corte con el eje vertical Oy )
Las 2 rectas son paralelas , m1 = m2
y = m1 x + b 1
Las 2 rectas se cortan en el punto P (x0 ; y0 ) , m1 6= m2
y = m2 x + b 2
Las 2 rectas son perpendiculares , m1 = 1=m2
La funcin lineal f es biyectiva y por tanto posee funcin inversa g .
y = f (x) = mx + b ) La grca de g es otra recta y su grca es simtrica a la
x = my + b ) grca de la recta f con respecto al origen de coordenadas O (0; 0)
x b
y = g (x) = El producto de las pendientes de la recta y = f (x) = mx + b y de la recta
m
y = g (x) = xmb es igual a 1:

Recuerda sobre la funcin cuadrtica y su grca:


Funcin cuadrtica: Caractersticas:
y = ax2 , a > 0 Parbola con vrtice V (0; 0) hacia abajo. Si 0 < a < 1 es ms abierta.
y = ax2 , a < 0 Parbola con vrtice V (0; 0) hacia arriba. Si 1 < a < 0 es ms abierta
b b
y = ax2 + bx + c, a > 0 La parbola posee vrtice V 2a ; f 2a hacia abajo.
b b
y = ax2 + bx + c, a < 0 La parbola posee vrtice V 2a ; f 2a hacia arriba.
b b
La parbola general posee vrtice V 2a ; f 2a y su grca
b
es simtrica con respecto a la recta x = 2a :
Si b2 4acp > 0, la parbola corta al eje Ox en los puntos o races
p
2 b2 4ac b2 4ac
y = ax + bx + c x1 = b 2a y x2 = b+ 2a :
2
Si b 4ac = 0, la parbola corta al eje Ox en un nico punto (vr-
tice) x1 = 2ab = x2 :
Si b2 4ac < 0, la parbola no corta al eje Ox (races imaginarias)
La funcin cuadrtica f no es biyectiva y por tanto no posee funcin
inversa. Pero posee relacin inversa g que se obtiene al permutar x con y
y = f (x) = ax2 + bx + c ) en la relacin y = f (x). y = g (x) est compuesta de la unin de 2 funciones.
x = ay 2 + by + c ) La grca de g es una parbola acostada y su grca es simtrica a la
y = g (x) grca de f con respecto al origen de coordenadas O (0; 0)
La parbola y = g (x) posee vrtice V 0 simtrico al vrtice V con respecto
al origen de coordenadas O (0; 0).

Ejemplos:

1. y 2 = x2 x4 ) F (x; y) = y 2 x2 + x4 = 0

a) Asntotas no existen.
b) La curva es simtrica con respecto al origen O (0; 0), pues F (x; y) = F ( x; y)
c) No hay conversin total en producto, sin embargo, es factorable la parte derecha y 2 = x2 x4 =
x2 (x 1) (x + 1)
x: 1 0;7 0 0;7 1
d ) Variacin:
y: 0 0;5 0 0;5 0
e) Grca:
184 CAPTULO 4. APLICACIONES, FUNCIONES NUMRICAS Y SUS GRFICAS
y

0.5

0.25

0
-1 -0.5 0 0.5 1

-0.25

-0.5

2. y = x2=3

a) Sus intercepciones con Ox son: x2=3 = 0 ) x = 0. Con Oy son: y = 0:


q p
2=3 3 2 3
b) La curva es simtrica con respecto al eje Oy; pues y = ( x) = ( x) = x2
p3
c) Su extensin: y = x2=3 = x2 ) x 2] 1; +1[:

d ) Asntotas no existen.

e) No hay conversin en producto.

x: 1 8 0 8 +1
f ) Variacin:
y: +1 4 0 4 +1

g) Grca:
y

2 .5

1 .5

0 .5

0
-5 -2 .5 0 2 .5 5

1
3. y = x +
x

1
a) Sus intercepciones con Ox no existen: x + 6= 0. Con Oy no existen:
x
b) La curva es simtrica con respecto al origen O, pues F (x; y) = F ( x; y)
1
c) Su extensin: y = x + ) x 2] 1; 0[[]0; +1[, y 2] 1; +1[:
x
d ) Asntotas: es el eje vertical x = 0.

e) No hay conversin en producto.

x: 1 0 0 0+ +1
f ) Variacin:
y: 1 1 @ +1 +1

1
g) Grca y = x + :
x
4.3. PRCTICAS CON LAS GRFICAS DE ALGUNAS FUNCIONES Y RELACIONES 185
y
7.5

6.25

3.75

2.5

1.25
0
-5 -4 -3 -2 -1 0 1 2 3 4 5
-1 .2 5
x
-2.5

-3 .7 5

-5

-6 .2 5

-7.5

4. y 2 x2 + 8 = x2 ) F (x; y) = y 2 x2 + 8 x2 = 0

a) Sus intercepciones con Ox son: x2 x4 = 0 ) x = 0; 1. Con Oy son: y 2 = 0 ) y = 0:


2
b) Hay simetra con respecto a ambos ejes, pues no cambia para y y para x : ( y) x2 + 8 = x2 y
2 2
y 2 ( x) + 8 = ( x) . Hay simetra con respecto al origen O (0; 0), pues F (x; y) = F ( x; y) :
r
x2 x2
c) Su extensin: y = ) 0 ) x 2] 1; +1[; y 2] 1; +1[:
x2 + 8 x2 + 8
d ) Asntotas no existen.
e) No hay conversin en producto.
x: 1 4 0 4 +1
f ) Variacin:
y: 1 p2 0 p2 1
6 6
r
2 2 2 x2
g) Grca y x +8 =x )y = :
x2 + 8
y
0 .7 5

0 .5

0 .2 5

0
-5 -2 .5 0 2 .5 5

x
-0 .2 5

-0 .5

-0 .7 5

2 2
2 2 (x + 1) (y + 1)
5. x2 + 2y 2 + 2x + 4y 16 = 0 ) (x + 1) + 2 (y + 1) 19 = 0 ) p 2 + p19 2 = 1
19 ( )
2

p p
a) Sus intercepciones con Ox son: x2 + 2x 16 = 0 ) x = 1+ 17 ; x = 1 17 . Con Oy
son: 2y 2 + 4y 16 = 0 ) fy = 4g ; fy = 2g
b) La curva no es simtrica con respecto a los ejes coordenados:
2 2 p p p
19
p
19
c) Su extensin: (x + 1) + 2 (y + 1) = 19 ) x 2 [ 1 19; 1 + 19]; y 2 [ 1 2 ; 1+ 2 ]:

d ) Asntotas no existen.
e) No hay conversin en producto.
p p
x: 1 19 0 1+ 19
f ) Variacin:
y: 1 (y = 4) _ (y = 2) 1
2 2
g) Grca de x2 + 2y 2 + 2x + 4y 16 = 0 ) (x + 1) + 2 (y + 1) = 19 :
186 CAPTULO 4. APLICACIONES, FUNCIONES NUMRICAS Y SUS GRFICAS

2 y

1 .5

0 .5
0
-5 - 4 .5 - 4 - 3 .5 - 3 - 2 .5 - 2 - 1 .5 - 1 - 0 .5 0 0 .5 1 1 .5 2 2 .5 3
- 0 .5
x
-1

- 1 .5

-2

- 2 .5

-3

- 3 .5

-4

1 2 1 2
6. x2 y2 + x + y = 0 ) x + 2 y 2 = 0 ) (x + y) (x + 1 y) = 0

a) Sus intercepciones con Ox son: x = 0; x = 1. Con Oy son: y = 0; y = 1:


b) La curva no es simtrica con respecto a los ejes coordenados:
2 2
c) Su extensin: (x + 1) + 2 (y + 1) = 19 ) x 2] 1; +1[; y 2] 1; +1[:
d ) Asntotas no existen.
e) Si hay conversin en producto x2 y 2 + x + y = 0 ) (x + y) (x + 1 y) = 0 )
(
x+y =0)y = x
.
x+1 y =0)y =x+1

x: 1 0 +1
f ) Variacin de cada rama: x 2] 1; +1[, y 2] 1; +1[:
y: 1 fy = 0g ; fy = 1g +1
g) Grca de x2 y 2 + x + y = 0 ) (x + y) (x + 1 y) = 0 :
y 5

1
0
-5 -4 -3 -2 -1 0 1 2 3 4 5
-1 x

-2

-3

-4

-5

r
2 2 2 2 x2 3x 2 x2 3x 2
7. xy y x + 3x + 2 = 0 ) y = )y= .
x 1 x 1
n p o n p o p
3 17 3 17
a) Sus intercepciones con Ox son: x = 2 + 2 , x= 2 2 . Con Oy son: y = 2:
2 2
b) La curva es simtrica con respecto al eje Ox, pues la ecuacin x ( y) ( y) x2 + 3x + 2 = 0 no
cambia.
r
x2 3x 2 x2 3x 2
c) Su extensin: y = ) 0)
x 1 x 1
3 1
p p
2 2 17 x < 1 [ x 32 + 12 17 ; y 2] 1; +1[:
d ) Existe una asntota vertical x = 1.
e) No hay conversin en producto.
3 1
p 3 1
p
p p
x: 17 1 + 17 +1
f ) Variacin: x 2 [ 32 1
2 17; 1[[[ 32 + 12
17; +1[, y 2] 1; +1[: 2 2 2 2
y: 0 1 0 1
r
x2 3x 2
g) Grca de xy 2 y2 x2 + 3x + 2 = 0 ) y = :
x 1
4.3. PRCTICAS CON LAS GRFICAS DE ALGUNAS FUNCIONES Y RELACIONES 187
y 5

1
0
-0.5 0 0.5 1 1.5 2 2.5 3 3.5 4 4.5 5
-1 x

-2

-3

-4

-5

8. y 2 = x3

a) Sus intercepciones con Ox son: x = 0. Con Oy son: y = 0:


2
b) La curva es simtrica con respecto al eje Ox, pues la ecuacin ( y) = x3 no cambia.
p
c) Su extensin: y = x3 ) x3 0 ) x 2 [0; +1[; y 2] 1; +1[:

d ) No existen asntotas.

e) No hay conversin en producto.

x: 0 4 +1
f ) La variacin es x 2 [0; +1[, y 2] 1; +1[:
y: 0 8 1
p
g) Grca de y 2 = x3 ) y = x3 :
y 5

0
0 .2 5 0 .5 0 .7 5 1 1 .2 5 1 .5 1 .7 5 2 2 .2 5 2 .5 2 .7 5
-1 x

-2

-3

-4

-5

x
9. y =
x2 + 1

a) Sus intercepciones con Ox son: x = 0. Con Oy son: y = 0:

b) La curva no es simtrica con respecto a ningn eje, pero es simtrica con respecto al origen O, pues
( x)
no cambia para ( y) = 2 .
x +1
c) Su extensin: x 2] 1; +1[; y 2 [ 0;5; 0;5[

d ) No existen asntotas.

e) No hay conversin en producto.

x: 1 6 1 0 6 +1
f ) La variacin es x 2] 1; +1[, y 2 [ 0;5; 0;5[: 6 1 6
y: 0 37 2 0 37 0
x
g) Grca de y = :
x2 +1
188 CAPTULO 4. APLICACIONES, FUNCIONES NUMRICAS Y SUS GRFICAS
y

0.25

0
-7 .5 -5 -2 .5 0 2 .5 5 7 .5

-0 .2 5

10. y 2 + x2 y 1=0

a) Intercepciones con Ox no existen. Con Oy son: y = 1:


2
b) La curva es simtrica con respecto al eje Oy, pues no cambia para y 2 + ( x) y 1 = 0.
c) Su extensin: x 2] 1; +1[, y 2 [ 1; 1][]0; 1].
d ) Existe una asntota horizontal y = 0.
e) No hay conversin en producto.
f ) Variacin: x 2] 1; +1[, y 2 [ 1; 1][]0; 1]:
x: 1 1 0 1 +1
p p p p
5 1 1 5 5 1 1 5
y: ! 0; 1 2 2; 2 2 1 2 2; 2 2 ! 0; 1
g) Grca de y 2 + x2 y 1=0:
y
1.25

x
-5 -2.5 0 2.5 5
0

-1.25

-2.5

-3.75

-5
Captulo 5

NMEROS REALES

Has trado ante mi a un hombre que no sabe contar sus dedos?

Del Libro de los Muertos

Cunto es un Dios ms que el otro?

Texto Babilonio

En la mayor parte de las ciencias una generacin derriba lo que otra haba

construido, y lo que uno pareca haber demostrado rmemente otro lo deshace. Slo

en la matemtica cada generacin construye un nuevo piso sobre la vieja estructura.

Hermann Hankel

Unidad de competencia: Utilizar la exposicin axiomtica, operaciones, propiedades y aplicaciones del conjunto
de los nmeros reales como herramienta fundamental en el estudio de las matemticas.1 .

Indicadores de logro:

| Conocer e identicar los axiomas fundamentales y las propiedades bsicas del conjunto de nmeros reales R.

| Comprender la importancia del cuerpo de los nmeros reales R en el estudio de la matemtica y sus aplicaciones.

| Utilizar y aplicar las propiedades del cuerpo de los reales R en la demostracin de varios teoremas.

| Operar correctamente con los nmeros reales y sus subconjuntos.

| Utilizar ecientemente el mtodo de induccin completa en la demostracin de proposiciones.

En cierto congreso fsico celebrado en Odesa, se propuso el siguiente problema: expresar cualquier nmero entero y
positivo mediante 3 doses y signos matemticos

1 Un estudio ms completo de los cuerpos numricos se hallar en el apndice 1 al nal del libro y en [4], [11] y [13].

189
190 CAPTULO 5. NMEROS REALES

Figura 5.1: Izq. El famoso matemtico y genio griego, insigne mecnico e inventor de la antiguedad, Arqumedes (aprox.
287 - 212 A.C.). Desarroll mtodos para el clculo exacto de reas y volmenes de guras y cuerpos adelantndose a
los conceptos actuales del clculo innitesimal. Fue asesinado por un soldado romano en el asedio a su ciudad natal
Siracusa- Sicilia. Der. Apolonius de Prgamo (aprox. 260 - 170 A.C.). Matemtico griego, representante de la Escuela
de Alejandra. Su clebre trabajo (8 libros) sobre las cnicas, donde introdujo los trminos y propiedades de la parbola,
elipse, hiprbola, focos, etc. inuyeron profundamente en el desarrollo posterior de la geometra.

El lgebra comienza con la introduccin del conjunto de los nmeros naturales N = f1; 2; : : :g y los nmeros
enteros negativos f 1; 2; : : :g, o sea, con la formacin de los primeros sistemas numricos fundamentales, a
saber: el sistema de los nmeros enteros Z = f: : : 2; 1; 0; 1; 2; : : :g que n
consta de todos los nmeros oenteros,
a
positivos y negativos, incluyendo el cero, y del sistema ms amplio Q = x : x = ; a; b 2 Z; b 6= 0 de los
b
nmeros racionales, que consta de todos los nmeros enteros y fraccionarios (o quebrados), positivos y negativos.

Posteriormente,
n se efecta unaoampliacin del conjunto de los nmeros introduciendo los nmeros irracionales
a
Q0 = x : x 6= ; a; b 2 Z; b 6= 0 . El sistema, compuesto de todos los nmeros racionales e irracionales, se
b
llama sistema de nmeros reales R. En el curso de anlisis matemtico se da una construccin rigurosa del
sistema de nmeros reales R = Q [ Q0 . Finalmente la ampliacin algebraica de los nmeros reales R corresponde
a los nmeros complejos (a veces denominados imaginarios) C.

5.1. El Cuerpo de los Nmeros Reales

Pasamos a dar una denicin o exposicin (existen varias construcciones!) exacta y ms cmoda de los
nmeros reales R, tratando de reunir aquellas propiedades de los nmeros ya conocidas por el lector desde el
inicio de la educacin media. Esta denicin suele denominarse denicin axiomtica de los nmeros reales, y
est compuesta de los axiomas fundamentales y de varias propiedades independientes.

5.1.1. Axiomas y propiedades generales del conjunto de los nmeros reales

Denicin del conjunto de nmeros reales R El conjunto no vaco R = fa; b; : : : ; x; y; : : :g, se denomina
conjunto de los nmeros reales y sus elementos, nmeros reales, si se cumplen las siguientes condiciones o
axiomas:

I Axiomas de la suma:

Est denida la operacin (aplicacin) de suma + : R R ! R, que a cada par ordenado (x; y) de
elementos x e y de R le pone en correspondencia el elemento x + y 2 R, denominado suma de x e y.
Cumplindose adems las siguientes condiciones:
5.1. EL CUERPO DE LOS NMEROS REALES 191

1+ Existe un elemento neutro 0 (denominado cero, para el caso de la suma), tal que para cualquier x 2 R

x + 0 = 0 + x = x:

2+ Para cualquier elemento x 2 R existe el elemento x 2 R, denominado opuesto de x, y tal que

x + ( x) = ( x) + x = 0:

3+ La operacin (aplicacin) + es asociativa, es decir, para cualesquiera elementos x; y; z 2 R, se cumple

x + (y + z) = (x + y) + z:

4+ La operacin (aplicacin) + es conmutativa, es decir, para cualesquiera elementos x; y 2 R, se cumple

x + y = y + x:

Si sobre cierto conjunto G est denida una operacin (aplicacin) que satisface a los axiomas 1+ , 2+ , 3+ ,
entonces se dice que el conjunto G es un grupo. Si la operacin se denomina suma, entonces el grupo se
denonima aditivo. Si adems la operacin es conmutativa, es decir, se cumple la condicin 4+ , entonces el
grupo se llama conmutativo o abeliano 2 .

II Axiomas del producto:


Est denida la operacin (aplicacin) de producto : R R ! R, que a cada par ordenado (x; y) de
elementos x e y de R le pone en correspondencia el elemento x y 2 R, denominado producto de x e y,
y de tal manera que con relacin a la operacin del producto, el conjunto R n 0 es un grupo abeliano
multiplicativo, es decir se cumplen las siguientes condiciones:

1 Existe un elemento neutro 1 2 R n 0 (denominado uno, para el caso del producto), tal que para
cualquier x 2 R n 0, se tiene x 1 = 1 x = x
1
2 Para cualquier elemento x 2 R n 0 existe el elemento x 2 R n 0, denominado opuesto de x, y tal
que x x 1 = x 1 x = 1
3 La operacin (aplicacin) es asociativa, es decir, para cualesquiera elementos x; y; z 2 R n 0, se
cumple x (y z) = (x y) z:
4 La operacin (aplicacin) es conmutativa, es decir, para cualesquiera elementos x; y 2 R n 0, se
cumple x y = y x

I y II Enlace entre la suma y el producto:


El producto es distributivo con respecto a la suma, es decir, 8x; y; z 2 R, se tiene (x + y) z = x z + y z.
| Si sobre cierto conjunto K operan dos operaciones que satisface todos los anteriores axiomas, entonces
se dice que el conjunto K es un cuerpo.
Observacin: De aqu en adelante, por comodidad, generalmente, x y lo representaremos simplemente
con xy.

III Axiomas de orden:


Entre los elementos de R se tiene la relacin , es decir, para los elementos x; y de R se establece que se
cumpla o no que x y. Satisfacindose adems las siguientes condiciones:

0 8x 2 R (x x) :
1 (x y) ^ (y x) ) (x = y) :
2 (x y) ^ (y z) ) (x z) :
2 En honor de N. Abel (1802-1829), famoso matemtico noruego. Ver apndice.
192 CAPTULO 5. NMEROS REALES

3 8x 2 R; 8y 2 R (x y) _ (y x) :

La relacin en R se denomina relacin de desigualdad.


El conjunto entre cuyos elementos se tiene la relacin que satisface los axiomas 0 , 1 , 2 , se denomina
parcialmente ordenado, y si adems se cumple el axioma 3 , es decir, cualesquiera dos elementos del
conjunto son comparables, entonces el conjunto se denomina linealmente ordenado.
De esta manera vemos que el conjunto R est linealmente ordenado con la relacin de desigualdad entre
sus elementos.

I y III Enlace entre la suma y el orden en R:


Si x; y; z son elementos de R, entonces (x y) ) (x + z y + z)

II y III Enlace entre el producto y el orden en R:


Si x; y son elementos de R, entonces (0 x) ^ (0 y) ) (0 x y)

IV Axioma de completitud (o de continuidad):


Si X e Y son subconjuntos no vacos de R, que satisfacen la propiedad de que para cualesquiera elementos
x 2 X e y 2 Y se cumple x y, entonces existe el elemento c 2 R tal que x c y para cualesquiera
elementos x 2 X e y 2 Y .

V Axioma de completitud y de existencia del supremo


Denicin 1. Se dice que el conjunto X R est acotado superiormente (inferiormente), si existe el
nmero c 2 R tal que x c (respectivamente c x) para cualquier x 2 X.
El nmero c en este caso se denomina frontera superior (inferior) del conjunto X, o mayorante (minorante)
del conjunto X.
Denicion 2. Si un conjunto esta acotado superior e inferiormente, entonces se denomina acotado.
Denicin 3. El elemento a 2 X se denomina mximo o maximal (mnimo o minimal) del conjunto
X R, si x a (respectivamente a x ) para cualquier elemento x 2 X.
Introducimos los siguientes simbolismos:

(a = max X) : = (a 2 X ^ 8x 2 X (x a)) ;
(a = m n X) : = (a 2 X ^ 8x 2 X (a x)) :

De igual manera se utilizan los smbolos: maxx2X x, m nx2X x


Del axioma de orden 1 se sigue que si en un conjunto numrico hay un mximo (un mnimo), entonces
ste es nico. Sin embargo, no en cualquier conjunto, auque sea acotado existe un elemneto mximo
(mnimo). Por ejemplo, para X = fx 2 R : 0 x < 1g vemos que existe un mnimo (x = 0), y no existe
un mximo.
Denicin 4. El ms pequeo de los nmeros que acotan al conjunto X R por arriba, se denomina
supremo de X ( sup X): s = sup X := 8x 2 X ((x s) ^ (8s < s 9x 2 X (s0 < x0 ))).
0 0

Denicin 5. El ms grande de los nmeros que acotan al conjunto X R por debajo, se denomina
nmo de X ( nf X): i = nf X := 8x 2 X ((i x) ^ (8i < i 9x 2 X (x < i0 ))).
0 0 0

Las anteriores deniciones se simbolizan mediante: sup x, nf x y signican:


x2X x2X

sup X : m n fc 2 R : 8x 2 X (x c)g , nf X : max fc 2 R : 8x 2 X (c x)g :

Como habamos indicado antes no cualquier conjunto posee un mximo y un mnimo, por lo tanto es muy
necesario la introduccin de las anteriores deniciones y del siguiente
5.1. EL CUERPO DE LOS NMEROS REALES 193

Lema 1 (Principio del supremo). Cualquier subconjunto X R no vaco y acotado superiormente posee
un nico supremo.
Demostracin. Por cuanto la unicidad del elemento mnimo ya es conocida, necesitamos aclarar la exis-
tencia del supremo. Sea X R e Y = fy 2 R : 8x 2 X (x y)g el conjunto de las fronteras superiores
de X. Por las condiciones tenemos: X 6= ? e Y 6= ?, entonces por el axioma de completitud existe el
nmero c 2 R, tal que 8x 2 X 8y 2 Y (x c y). Por lo tanto el nmero c es un mayorante de X y
minorante de Y . Como mayorante de X, el nmero c es un elemento de Y , pero como minorante de Y ,
el nmero c es el elemento mnimo de Y . Por lo tanto, c = m n Y = sup X:
Anlogamente se demuestra la existencia y unicidad del nmo de un conjunto numrico acotado inferior-
mente, es decir, tiene lugar el siguiente
Lema 2. (X est acotado inferiormente) ) (9! nf X) :

Los principios de induccin matemtica y de Arqumides

El principio de induccin matemtica dice que: Si cierto subconjunto X N, es tal que 1 2 N, y conjunta-
mente con el nmero x 2 X, el nmero x + 1 tambin pertenece a X, entonces X = N:

(X N) ^ (1 2 X) ^ (8x 2 X : (x 2 X ) (x + 1) 2 X)) ) X = N

Denicin. El conjunto X R, se denomina inductivo, si junto con cada nmero x 2 X, se cumple que
tambin x + 1 2 X:

Ejemplos de conjuntos inductivos seran R, Z+ = f1; 2; 3; : : :g = N, R+ , y otros.

El principio de Arqumides dice que: Si jamos cierto nmero positivo h 2 R+ , entonces para cualquier
nmero real x 2 R, existe un nico nmero entero k 2 Z, tal que (k 1) h x < kh:

Consecuencias:

1
Para cualquier nmero positivo ", existe un natural n, tal que 0 < < ":
n
1
Si para el nmero x 2 R, se tiene que 0 x y para cualquier n 2 N, x < , entonces x = 0:
n
Para cualesquiera nmeros a, b 2 R, tales que a < b, existe un nmero racional r 2 Q, tal que a < r < b:

Para cualquier nmero x 2 R, existe un nico entero k 2 Z, tal que k 1 x < k:

El anterior nmero k, se representa con [x] y se denomina parte entera del nmero x. El valor fxg = x [x]
se denomina parte fraccionaria (decimal) del nmero x. Se tiene x = [x] + fxg, siendo fxg 0:

Algunas consecuencias de los axiomas

Todo conjunto X R, X 6= ?, que est acotado inferiormente admite nmo.


Demostracin: El conjunto X 0 = fx 2 R : x 2 Xg est acotado superiormente, luego existe s = sup X 0 ;
evidentemente, s es el nmo de X.

Si X R, es un conjunto de nmeros enteros que est acotado superiormente (resp., inferiormente),


entonces existe mx X (resp., mn X), es decir, sup X 2 X (resp., nf X 2 X).
Demostracin: Como X est acotado superiormente, existe s = sup X. Si s 2
= X, existe n 2 X, tal que
s 1=2 < n < s y, por tanto, para todo natural m > n ser s < s + 1=2 < n + 1 m, luego m 2= X
194 CAPTULO 5. NMEROS REALES

y, en consecuencia, n es el mximo de X, que no es posible pues n < s; esta contradiccin descarta la


posibilidad s 2
= X.

Propiedad arquimediana. Para cualesquiera nmeros reales x > 0 e y, existe un n 2 N, tal que
y < nx.
Demostracin: Si la propiedad fuese falsa, X = fnx : n 2 Ng estara acotado (pues y sera cota de X ) y
existira s = sup X. En este supuesto, por ser nx s para todo n 2 N, sera (n 1)x = nx x s x
para todo n 1 2 N, luego s0 = s x sera cota de X, lo cual no es posible ya que s0 < s y s es la menor
cota de X. Esta contradiccin prueba el teorema.

Parte entera de un nmero real. Para cada nmero real x, existe un nmero entero p tal que
p x < p + 1; p se llama parte entera de x y se denotar mediante E(x).
Demostracin: De acuerdo con la propiedad arquimediana (aplicada a 1 > 0 y a x), existe n 2 N tal que
x < n y, por tanto, X = fq 2 Z : x < qg no es vaco; como X est acotado inferiormente (admite a x
por cota inferior) y est constituido por nmeros enteros, existe m = m n X, de forma que x < m y
x m 1, pues m 1 2 Z no pertenece a X, luego p = m 1 es la parte entera de x.

A continuacin pasamos a analizar algunos subconjuntos de R y ciertas propiedades.

Nmeros racionales e irracionales

Proposicin (Q es denso en R). Dados x, y 2 R con x < y, la relacin x < r < y se verica para
innitos r 2 Q (signica que entre dos nmeros reales hay innitos racionales).
Demostracin: La propiedad arquimediana, aplicada a y x > 0 y a 1, arma que existe n 2 N tal que
1 < n (y x) esto es 1 + nx < ny. Si es p 2 Z la parte entera de nx, es decir p nx < p + 1, ser

nx < p + 1 nx + 1 < ny;

siendo pues nx < p + 1 < ny, para el racional r = (p + 1)=n se verica que x < r < y. Probada ya la
existencia de un r 2 Q tal que x < r < y, repitiendo el razonamiento para x; r 2 R y para r; y 2 R, se
llega a la existencia de s1 ; s2 2 Q tales que x < s1 < r < s2 < y; reiterando el proceso indenidamente,
se obtienen innitos racionales comprendidos entre x e y.

Observacin. La propiedad precedente, que se expresa diciendo que los nmeros racionales constituyen
un conjunto denso del sistema de los nmeros reales, puede enunciarse en los siguientes trminos: dado
x 2 R y para cualquiera que sea el nmero real " > 0, existen r, r0 2 Q tales que x " < r < x < r0 < x+"
(dado un nmero real x, existen nmeros racionales r y r0 tan prximos a l como se desee). Para
comprobar la validez de esta forma de enunciar que Q es denso en R, basta tomar en lugar de x e y los
x " y x o los x y x + ".

Ejemplo de nmero irracional. Segn se dijo anteriormente, los nmeros reales que no son racionales
se llaman irracionales; stos constituyen, pues, el conjunto R n Q = R Q. Obsrvese que cuanto se
ha hecho hasta ahora ha ido encaminado a introducir los nmeros irracionales, pues los racionales eran
ya conocidos. Ms adelante se ver que hay innitos nmeros irracionales, es ms, que la potencia del
conjunto R es superior a la de Q. Comenzamos comprobando con un ejemplo que existe algn nmero
irracional. El conjunto de nmeros reales X = r 2 Q : r3 < 2 no es vaco (pues 1 2 X) y est acotado
superiormente (pues 3=2 es una cota de X); por tanto, existe el nmero real x = sup X, que se llama
p p
raz cbica de 2 (x = 3 2). Pues bien, se va a probar que 3 2 es un nmero irracional, esto es, que dado
arbitrariamente r 2 Q, el supremo de X no puede ser r. Dado r 2 Q, con un razonamiento al absurdo, se
obtiene trivialmente que r3 6= 2 y, por tanto, ha de ser r3 < 2 o r3 > 2; en el primer supuesto se probar
5.1. EL CUERPO DE LOS NMEROS REALES 195

que r no es cota superior de X y en el segundo se llegar a que existen cotas superiores de X menores que
r, luego en cualquiera de los dos casos es r 6= sup X.
Si r3 < 2 ) d = 2 r3 > 0, con uno de los innitos racionales tales que 0 < < m nf1; d=9r2 ; d=9r; d=3g;
entonces, para r0 = r + se tiene
3 2 3 2 3
(r0 )3 = (r + ) = r3 + 3r2 + 3r + = (2 d) + 3r2 + 3r + <
2
< 2 d + 3r + 3r + < 2 d + (d=3) + (d=3) + (d=3) = 2;

luego r no es cota superior de X, pues r0 > r y r0 2 X. Si r3 > 2, esto es d = r3 2 > 0, sea uno de
los innitos racionales tales que 0 < < m nf1; d=9r2 ; d=9r; d=3g; entonces, para r0 = r es (r0 )3 > 2,
luego r0 < r0 es cota de X.

Nmeros racionales e irracionales. Como Q es un cuerpo3 , se puede asegurar que:

1. x 2 R y x son ambos racionales o ambos irracionales;


2. si r 2 Q y x 2 R, entonces r + x es racional o irracional segn que lo sea x;
3. x 2 R f0g y 1=x son ambos racionales o ambos irracionales;
4. si r 2 Q f0g y x 2 R, entonces rx es racional e irracional segn que lo sea x. Obsrvese que la
suma y el producto de nmeros irracionales puede ser racional o irracional.

La ordenacin de R es tal que no existen intervalos de R formados solamente por nmeros racionales o
solamente por irracionales, esto es: a) dados 2 irracionales distintos, entre ellos hay un racional; y b) dados
2 racionales distintos, entre ellos hay un irracional. La armacin a) es evidente, ya que los irracionales
p
son reales y Q es denso en R; para comprobar b) puede recurrirse a que, por ejemplo, 3 2 es irracional y
p p
1 < 3 2 < 2, ya que, dados r, s 2 Q con r < s, el nmero x = (s r)( 3 2 1) + r es irracional y tal que
r < x < s.

Proposicin. Todo nmero real es el extremo superior (resp., inferior) del conjunto de los nmeros
racionales menores (resp., mayores) que l; esto es, dado x 2 R, se tiene

x = supfr 2 Q : r < xg y x = nffr 2 Q : r > xg:

Demostracin: Slo se prueba la proposicin para el caso del supremo; para el del nmo se procedera
de modo anlogo. Desde luego, x es cota superior de X = fr 2 Q : r < xg y, por tanto, existe s = sup X
y es s x. Si fuese s < x, existira r 2 Q tal que s < r < xy, en consecuencia: a) por ser s < r, sera
r2= X; y b) como r < x, sera r 2 X. Esta contradiccin descarta la posibilidad s < x, luego es s = x.

Representacin de R en la recta

Una vez representados todos los nmeros racionales sobre una recta, existen puntos de la recta que no son
p p
imagen de ningn nmero racional ( 3 2, 5, etc.); se pretende establecer una correspondencia entre estos puntos
y los nmeros irracionales de modo que, si dos nmeros reales x y x0 tienen por imgenes a los puntos p y p0 ,
se verique que p est a la izquierda de p0 si, y slo si, es x < x0 . Esto se consigue si se conviene en que un
punto p cualquiera de la recta sea la imagen del nmero real x = sup X, donde X es el conjunto de los nmeros
racionales cuyas imgenes estn situadas a la izquierda de p; de este modo, todo punto de la recta sera imagen
de un nmero real (si no lo era de un racional, lo ser de un irracional).

Queda an pendiente el problema recproco: todo nmero real tendr imagen sobre la recta? Se supondr
que la recta tiene tal cantidad de puntos que es posible responder armativamente a la anterior pregunta; esto
es, se admite el llamado postulado de continuidad de la recta: todos los nmeros son representables sobre la recta.
3 Ver el apndice sobre estructuras algebraicas.
196 CAPTULO 5. NMEROS REALES

En consecuencia, la correspondencia que a cada nmero real le asocia su imagen en la recta ser una biyeccin
de R en la recta. Puesto que Q (resp., R n Q) es denso en R, no hay segmentos de la recta que estn formados
exclusivamente por puntos imgenes de racionales (resp., irracionales); los puntos imagen de los racionales estn
entremezclados con los que son imgenes de los irracionales.

Races, potencias y logaritmos

Potencias de base real y exponente entero.Dados x 2 R y n 2 Z, se llama potencia de base x


y exponente n al nmero real xn denido mediante: si n > 0, xn = x 0
| x{z x}; x = 1; si n < 0,
n veces
xn = 1=x n
. A partir de esta denicin resulta trivial comprobar que, si x, y 2 R y m, n 2 Z, es:

xn xm = xn+m , xn =xm = xn m
; xn =y n = (x=y)n ; (xn )m = xnm

Races de los nmeros reales.

a. Dados un nmero real x > 0 y un natural n, existe un nico nmero real y > 0 tal que y n = x; a y se le
p
llama raz n-sima de x y se pone y = n x. En efecto, para el nmero y = supfu 2 R : un < xg es y n = x;
n n
adems si para cierto y 0 > 0 fuese tambin (y 0 ) = x, sera y 0 = y pues (y 0 ) = y n .

b. Dados un nmero real x y un nmero natural n, conviene notar que: 1) Si es x > 0 y n par, adems de la
p p
raz n-sima y = n x > 0, el nmero real negativo y 0 = n x tambin satisface a la ecuacin (y 0 )n = x; a
y 0 se le llamar raz n-sima negativa de x > 0. 2) Si es x > 0 y n impar, no existe raz n-sima negativa
p
de x. 3) Si es x < 0 y n impar, el nmero real negativo y = n x recibe el nombre de raz n-sima
(negativa) de x, ya que y n = ( 1)n ( x) = x; en este caso, x no tiene ninguna otra raz n-sima. 4) Si es
x < 0 y n par, no existe ninguna raz n-sima de x, ya que, para todo y 2 R, y n no es negativo.

c. Para cualesquiera x, y 2 R, con x > 0 e y > 0, y si n; m 2 N, se verica (si x o y fuesen negativos o si se


considerasen races negativas, la generalizacin de las siguientes propiedades resulta evidente) :
p p p p p p p m p
n
xny = n
xy; n
x= n y = n x=y; n
x = n xm ;
q p p
m p p nh
n
x = m n
x; xmh = n xm para h 2 N:

Dado lo trivial de las demostraciones, slo se realizar la primera de ellas, quedando las dems como
ejercicios. Por la propia denicin de raz n-sima, resulta que:
p
n
p p p p n p n p n
x n y = n xy , n x n y = xy , n
x ( n y) = xy , xy = xy

Potencias de base real y exponente racional.

a. Dados un nmero real x > 0 y un nmero racional r = m=n, en donde n 2 N y m 2 Z, se llama potencia
p p m
de base x y exponente r al nmero real n xm = ( n x) , que se denota por xm=n . Esta denicin no
depende del representante m=n que se tome para denir r, ya que xm=n = xmh=nh para todo h 2 N

b. Para estas potencias se verican las siguientes propiedades, segn resulta trivial comprobar: para cua-
lesquiera nmeros reales x > 0 e y > 0 y si r, s 2 Q, es:

r
xr xs = xr+s ; xr =xs = xr s
; xr y r = (xy) ;
r s
xr =y r = (x=y) ; (xr ) = xrs :
5.1. EL CUERPO DE LOS NMEROS REALES 197

c. Respecto de la variacin de estas potencias, se puede asegurar que: 1) Si el exponente es positivo (resp.,
negativo), la potencia crece (resp., decrece) al crecer la base; 2) si la base es mayor (resp., menor) que
la unidad, la potencia crece (resp., decrece) al crecer el exponente; 3) si la base es mayor (resp., menor)
que la unidad, la potencia se puede hacer tan grande (resp., tan prxima a cero) como se desee, con tal
de tomar el exponente sucientemente grande; y 4) la potencia se puede hacer tan prxima a la unidad
como se quiera, con tal de tomar el exponente sucientemente prximo a cero.

Potencias de base y exponente reales.

a. Dados dos nmeros reales x > 0 e y, se llama potencia de base x y exponente y y se denota por xy al
nmero real (que existe y est bien denido):

si x > 1; xy = supfxr : r 2 Q; r < yg = nffxr : r 2 Q; r > yg;

si x < 1; xy = nffxr : r 2 Q; r < yg = supfxr : r 2 Q; r > yg:


De esta denicin se desprende trivialmente que, si r y s son nmeros racionales, es:

si x > 1; r < y < s ) xr < xy < xs ;


si x < 1; r < y < s ) xr > xy > xs :

b. La precedente denicin, para poder ser admitida, ha de comprender como caso particular a la de potencia
de exponente racional ; es decir, si y es racional, las deniciones anteriores han de proporcionar el mismo
nmero para xy. As ocurre, ya que (se considera el caso x > 1. Para x < 1 se razona igualmente) el
nmero xy es cota superior de fxr : r 2 Q; r < yg y cota inferior de fxr : r 2 Q; r > yg, luego se trata
del supremo del primer conjunto y del nmo del segundo, que es el xy que se deni.

c. Para cualesquiera nmeros reales x > 0, y > 0, u y v, se verica que:


u
xu xv = xu+v ; xu =xv = xu v
; xu y u = (xy) ;
u
xu =y u = (x=y) ; (xuv ) = xuv :

d. Respecto de la variacin de las potencias de base (positiva) y exponente reales, se puede asegurar que:
1) si es y > 0 (resp. , y < 0), la funcin x 7! xy es creciente (resp., decreciente); 2) si es x > 1 (resp.,
x < 1), la funcin y 7! xy es creciente (resp., decreciente); 3) dado x > 1 (resp., x < 1) xy puede hacerse
tan grande (resp., tan prximo a cero) como se desee con tal de tomar y sucientemente grande; 4) dado
x, xy se puede hacer tan prximo a 1 como se quiera con tal de tomar y sucientemente prximo a cero.

Logaritmos

a. Dados dos nmeros reales x > 0 e y > 0 y si es x 6= 1, existe un nico nmero u tal que xu = y, este
nmero se llama logaritmo en base x de y ( logx y ) y se denota por u = logx y. Para x > 1 existe logx y,
y si x < 1, como 1=x > 1, existe un u 2 R tal que (1=x)u = 1=y, luego xu = y, esto es existe logx y. La
unicidad del logaritmo es evidente, dado que u1 6= u2 ) xu1 6= xu2 .

b. Para cualesquiera nmeros reales positivos x; y; u y v, con u 6= 1 y v 6= 1, se verica que:


1) logu (xy) = logu x + logu y, logu (x=y) = logu x logu y;
2) logu xt = t logu x, para t 2 R;
3) logv x= logu x = logv u:
En efecto: 1) Llamando h = logu x, k = logu y, ser uh = x y uk = y; en consecuencia xy = uu+k , esto
es logu xy = h + k = logu x + logu y. 2) Llamando h = logu x, es decir x = uh , ser xt = uht , luego
logu xt = logu uht = ht = t logu x. 3) Llamando h = logu x, es decir uh = x, ser logv uh = logv x, luego
h logv u = logv x, esto es (logu x)(logv u) = logv x.
198 CAPTULO 5. NMEROS REALES

Valor absoluto y distancia

Denicin. Se llama valor absoluto de x 2 R al nmero real jxj = maxfx; xg, esto es:

si x > 0; jxj = x; si x = 0; jxj = 0 y, si x < 0; jxj = x:

Evidentemente, para todo x 2 R se tiene j xj = jxj. Tambin es fcil comprobar que, dados x, " 2 R con " > 0,
jxj < " equivale a " < x < ".

Propiedades del valor absoluto. Para cualesquiera x; y 2 R se tiene:

jxj 0; jxj = 0 , x = 0; jx + yj jxj + jyj ; jxyj = jxj jyj :

Demostracin: La comprobacin de las dos primeras es trivial; la segunda es equivalente a jxj jyj x + y
jxj + jyj, que es consecuencia de las jxj x jxj y jyj y jyj; la tercera se obtiene fcilmente analizando
por separado los casos que pueden presentarse segn los signos de x o y.

Denicin. Dados x; y 2 R, se llama distancia de x a y al valor d(x; y) = jx yj. Como consecuencia de


las propiedades del valor absoluto es evidente que, para cualesquiera x; y; z 2 R, se tiene:

d(x; y) 0; d(x; y) = 0 , x = y; d(x; y) = d(y; x); d(x; z) d(x; y) + d(y; z)

Estas 4 propiedades son los axiomas para denir la distancia entre elementos (puntos) de un conjunto.

Los smbolos +1 y 1

Denicin. Se llama ampliacin de R a R = R [ f+1; 1g, donde +1 y 1 son dos smbolos (no
nmeros) que se llaman ms innitoy menos innito, y se admite que es 1 < x < +1 para todo x 2 R;
los nmeros reales se dice que son los elementos nitos de R. Tambin se supone que +1 y 1 estn sujetos
a los siguientes convenios, en los que x denota un nmero real cualquiera:

x + (+1) = x ( 1) = +1; x + ( 1) = x (+1) = 1;

si x > 0; x (+1) = (+1) x = +1; si x < 0; x (+1) = (+1) x = 1;


si x > 0; x ( 1) = ( 1) x = 1; si x < 0; x ( 1) = ( 1) x = +1;
x= (+1) = x= ( 1) = 0;
(+1) + (+1) = (+1) ( 1) = (+1) (+1) = ( 1) ( 1) = +1;
( 1) + ( 1) = ( 1) (+1) = (+1) ( 1) = ( 1) (+1) = 1;

Observaciones. El recurso a los smbolos +1 y 1 simplica la notacin y la nomenclatura. Se introducen


por conveniencia ms que por necesidad. Las operaciones en las que intervienen +1 o 1 que no estn indicadas
de modo expreso en la relacin precedente carecen de signicado; as, por ejemplo, carecen de sentido expresiones
como: (+1) + ( 1), 0 (+1) o (+1)=(+1). Ntese nalmente que R no satisface a todos los axiomas de R.

Extremos de conjuntos no acotados. Si X R no est acotado superiormente (respectivamente, inferi-


ormente), se dir que +1 (respectivamente, 1) es su extremo superior (respectivamente, inferior). Segn esto,
todo conjunto de R tiene supremo e nmo (en R); si el conjunto est acotado superiormente (respectivamente,
inferiormente), su supremo (respectivamente, nmo) es nito y si no lo est es +1 (o 1).

Ejemplos:

1. Sean x; y; u y v nmeros reales cualesquiera (o elementos de un cuerpo ordenado). Prubese que:


5.1. EL CUERPO DE LOS NMEROS REALES 199

a) Si es x 6= 0, es x2 > 0 y en particular 1 > 0


Solucin: Si 0 < x, como (r < s; t > 0) ) rt < st, se concluye que (0 < x, x > 0) ) 0 x < x x,
luego 0 < x2 ; si 0 > x, ser 0 < x y por tanto ( x)2 > 0, es decir x2 > 0. Como 1 = 12 , ser 1 > 0.
b) Si es x > 0, es 1=x > 0
Solucin: Si fuese x > 0 y 1=x < 0, como (r < s; t > 0) ) rt < st, se concluira que (1=x < 0; x >
0) ) (1=x)x < 0 x, luego sera 1 < 0, que es falso.
c) Si es 0 < x < y, es 0 < 1=y < 1=x
Solucin: Segn la propiedad anterior es 0 < 1=x y 0 < 1=y. Como 1=x > 0, de x < y se inere
que x(1=x) < y(1=x), esto es 1 < y=x; de esta ltima desigualdad y como 1=y > 0, se inere que
1=y < (y=x)(1=y) = 1=x.
d ) Si es 0 < x < y y 0 < u < v, es xu < yv.
Solucin: De x < y y u > 0 se inere que xu < yu; de u < v e y > 0 se inere que uy < vy; por
tanto xu < yu < yv.

1. Se dice que (A; B) es una cortadura de nmeros racionales si A y B constituyen una particin propia de
Q (esto es, A [ B = Q, A \ B = ;, A 6= ; y B 6= ;) tal que (a 2 A; b 2 B) ) a < b. Prubese que, para
la cortadura (A; B) existen los nmeros reales = sup A y = nf B y que es = .
Solucin: Todo elemento de B es cota superior de A, luego A est acotado superiormente y existe
= sup A. Todo elemento de A es cota inferior de B, luego B est acotado inferiormente y existe
= nf B. Si fuese < , eligiendo 2 R de modo que < < (por ejemplo, = ( + )=2), se
tiene: como = sup A, existe a 2 A tal que < a ; como = nf B, existe b 2 B tal que b< ;
por tanto, sera b < a para ciertas a 2 A y b 2 B, que no es posible. Si fuese < , como Q es denso en
R existira c 2 Q tal que < c < ; por ser < c, c no podra pertenecer a A; por ser c < , c no podra
pertenecer a B; por tanto, habra un racional c que no pertenecera ni a A ni a B, lo cual no es posible.

2. Si X y X 0 son 2 conjuntos acotados superiormente de nmeros reales, prubese que:

a) Si X + X 0 = fy = x + x0 : x 2 X; x0 2 X 0 g, entonces sup(X + X 0 ) = sup X + sup X 0 :


Solucin: Sean s = sup X y s0 = sup X 0 ; para todo y = x + x0 2 X + X 0 , como x s y x0 s0 ,
ser y s + s0 , luego s + s0 es cota superior de X + X 0 . Adems s + s0 es la menor cota superior
de X + X 0 ya que, para cualquier k < s + s0 y llamando " = (s + s0 ) k > 0, ser s ("=2) <
s y s0 ("=2) < s0 , luego existen x 2 X y x0 2 X 0 tales que s ("=2) < x y s0 ("=2) < x0 y, por
tanto, k = (s "=2) + (s0 "=2) < x + x0 2 X + X 0 , es decir, k no es cota superior de X + X 0 .
b) Si los elementos de X y de X 0 son positivos y llamando X X 0 = fy = xx0 : x 2 X; x0 2 X 0 g,
entonces sup(X X 0 ) = sup X sup X 0 .
Solucin: Sean s = sup X y s0 = sup X 0 ; para todo y = xx0 2 X X 0 , como x s y x0 s0 , ser
0 0 0 0 0
y ss , luego ss es cota superior de X X . Adems ss es la menor cota supeior de X X ya que,
para cualquier k < ss0 y llamando = ss0 k > 0 y " = =(s + s0 ) > 0, ser s " < s y s0 " < s,
luego existen x 2 X y x0 2 X 0 tales que s " < x y s0 " < x0 y, por tanto,

k = ss0 = ss0 "(s + s0 ) < ss0 "(s + s0 ) + "2 =

(s ")(s0 ") < xx0 2 X X 0 .

(si s " < 0 y s0 " < 0, la ltima desigualdad podra no vericarse; en tal caso se sustituira "
por cualquiera de los innitos "0 tales que 0 < "0 < " y s "0 > 0) es decir, k no es cota superior de
X X 0.

3. Dados un nmero real x > 0 y un nmero natural n, prubese que existe el nmero real y = supfu 2 R :
un > 0; un < xg. Demustrese que y n = x.
200 CAPTULO 5. NMEROS REALES

Solucin: El conjunto C = fu 2 R : u > 0; un < xg es:


1) no vaco, ya que ]0; h[ C, para h = m nf1; xg, pues para todo u de este intervalo es un < u < x; y
2) acotado superiormente, ya que k = maxf1; xg es cota superior de C, pues para todo v > k es v n v x,
esto es v 2
= C. Por tanto, existe el nmero real y = sup C.
Hay que mostrar que y n no puede ser ni menor ni mayor que x. Para probar la falsedad de y n < x, bastara
hallar " > 0 tal que (y + ")n < x, ya que entonces y + " 2 C e y no sera cota de C; si es " < 1 se puede
poner :
n n n 2 n n 2 n n 1
(y + ") = y n + " y + y " + ::: + " <
1 3 n
n n n n n
< yn + " y 2
+ ::: + y 1
= y n + " ((y + 1) yn )
1 n

y entonces, tomando " de manera que 0 < " < m nf1; (x y n )=((y+1)n y n )g, ser (y+")n < y n +(x y n ) =
x, como se buscaba. Para probar la falsedad de y n > x, bastar hayar " > 0 tal que (y ")n > x, ya que
entonces y " sera cota de C e y no podra ser el supremo de C; un proceso anlogo al del caso anterior,
para " cumpliendo 0 < " < m nf1, (y n x)=((y + 1)n y n )g, conduce a (y ")n > x.

4. Sean los nmeros reales x > 0 e y > 0 y los racionales r y s. Basndose en la denicin de potencia de
base real y exponente racional, comprubese que:

a) (x < y, r > 0) ) xr < y r ; (x < y, r < 0) ) xr > y r .


Solucin: Para r > 0 y poniendo r = m=n (m 2 Z, n 2 N), si la propiedad fuese falsa, es decir si
n
xm=n y m=n , habra de ser (xm=n )n y m=n esto es xm y m , lo que implicara x y, que va
contra la hiptesis. Para r < 0, como x r < y r , ser xr = 1=x r > 1=y r = y r .
b) (r < s, x > 1) ) xr < xs ; (r < s, x < 1) ) xr > xs .
Solucin: Como xs > 0 y r s < 0, de la propiedad anterior se obtiene que x > 1 ) xr s
< 1r s
=
1 ) xr s xs < xs ) xr < xs ; para x < 1 se procede anlogamente.
c) Si es x > 1 y dado k 2 R, existe r0 2 Q tal que (r 2 Q, r r0 ) ) xr > k.
Solucin: Poniendo x = 1 + y con y > 0, es xn = 1 + n1 y + n2 y 2 + ::: + nn y n > ny para todo
n 2 N. Segn la propiedad arquimediana, existe n0 2 N tal que n0 y > k, luego xn0 > k; por tanto,
la propiedad se verica para r0 = n0 , ya que r n0 ) xr > xn0 > k.
d ) Si es x > 1 y dado " 2 R con " > 0, existe r0 2 Q con r0 > 0 tal que
(r 2 Q, 0 < r < r0 )) 1 < xr < 1 + ".
Solucin: Para conseguir 1 < xr , basta tomar r > 0. Como x > 1 y si es r > 0, resulta que
1=r
xr < 1 + " , x < (1 + ") ; ahora bien, segn la anterior propiedad, existe un racional 1=r0 tal que
la ltima relacin se verica para todo racional r tal que 1=r > 1=r0 , luego para r < r0 (con r > 0)
es xr < 1 + ".

5. Sean dados los nmeros reales x > 0, y > 0, u y v. Basndose en la denicin de potencia de base y
exponente reales y en las propiedades de las potencias de base real y exponente racional, prubese que:

a) ((x < y, u > 0)) xu < y u ; ((x < y, u < 0)) xu > y u
Solucin: Para u > 0, tomando r 2 Q de manera que u > r > 0 y como y=x > 1, se tiene: segn las
propiedades de las potencias de exponente racional, por ser r > 0 es (y=x)r > 1 y, segn la denicin
de potencia de exponente real, por ser u > r es (y=x)u > (y=x)r ; en consecuencia (y=x)u > 1, luego
y u > xu .Para u < 0,como x u < y u , ser xu = 1=x u > 1=y u = y u .
b) (u < v, x > 1)) xu < xv ; (u < v, x < 1) ) xu > xv
Solucin: Como xv > 0 y u v < 0, de la propiedad anterior se obtiene que x > 1 ) xu v
< 1u v
=
1 ) xu v xv < xv ) xu < xv ; para x < 1 se procede de anloga manera.
5.1. EL CUERPO DE LOS NMEROS REALES 201

c) Si es x > 1 y dado k 2 R, existe u0 2 R tal que (u 2 R, u 0)) xu > k; si es x < 1 y dado " 2 R
con " > 0, existe u0 2 R tal que ((u 2 R, u u0 )) xu < ".
Solucin: Para x > 1, segn las propiedades de las potencias de exponente racional, existe r 2 Q tal
que xr > k y, por tanto, si es u r se verica que xu xr > k, luego la propiedad se verica para
u0 = r. Para x < 1, como 1=x > 1, de lo que se acaba de obtener se inere que existe u0 2 R tal que
u u0 ) (1=x)u > 1=" ) xu < ".
d ) Dado " 2 R con " > 0, existe u0 2 R tal que (u 2 R, u0 < u < u0 )) jxu 1j < ".
Solucin: Si es x > 1, segn las propiedades de las potencias de exponente racional, existe r 2 Q con
r > 0 tal que xr < 1 + " y, por tanto, x r = 1=xr > 1=(1 + ") > 1 "; en consecuencia, se deduce
que r < u < r ) 1 " < x r < xu < xr < 1 + " ) jxu 1j < ", luego la propiedad se verica para
u0 = r. Para x < 1, basta aplicar el resultado anterior a 1=x > 1.

6. Dados dos nmeros reales x > 1 e y > 0, sean A = fv 2 R : xv < yg y A0 = fv 2 R : xv > yg. Recurriendo
a las propiedades de las potencias de base y exponente real, prubese que u = sup A y u0 = nf A0 son
iguales y que xu = y.
Solucin: Los conjuntos A y A0 , que son disjuntos y no vacos, son intervalos de la recta real: A es
intervalo indenido a la izquierda, pues (xv1 < y; v2 < v1 )) xv2 < y, y B es indenido a la derecha, pues
0
(xv1 < y; v2 < v1 )) xv2 > y; como (v 2 A; v 0 2 A0 )) xv < xv ) v < v 0 ,resulta que existen u = sup A
y u0 = nf A0 y es u u0 . Si fuese u < u0 , existiran dos nmeros reales h y k tales que u < h < k < u0 ,
luego h y k no perteneceran ni a A ni a A0 y, en consecuencia, xh = y y xk = y, lo que no es posible ya
que por ser h < k es xh < xk .
Para obtener que xu = y, se va a comprobar que xu no es ni mayor ni menor que y (slo se hace lo primero;
para lo segundo se procedera de forma anloga). Si fuese xu < y y, por tanto, y=xu = 1 + " con " > 0,
como existe un real > 0 tal que x 1 < ", sera x < y=xu , es decir xu+ < y; luego u + pertenecera
a A, que no es posible dado que u = sup A.

7. El conjunto T de nmeros reales es denso en R si, para cualesquiera x, y 2 R con x < y, existe t 2 T tal
que x < t < y. Prubese que :
p
a) T = t = r 3 : r 2 R es denso en R.
p p
Solucin: Dados x; y 2 R con x < y, como x= 3 < y= 3 y Q es denso en R, existe r 2 Q, tal que
p p p
x= 3 < r < y= 3 esto es x < r 3 < y, que prueba que T es denso en R
b) Si T es denso en R, para cualquier a 2 R es a = supft 2 T : t < ag.
Solucin: Como ft 2 T : t < ag est acotado superiormente por a, existe s = supft 2 T : t < ag y
es s a. Si fuese s < a, por ser T denso en R, existira t 2 T tal que s < t < a; como es s < t, t no
pertenecera a ft 2 T : t < ag; como es t < a, t pertenecera a ft 2 T : t < ag; esta contradiccin
descarta la posibilidad s < a, luego es s = a.
p
8. Dados un nmero racional r > 0 y un nmero natural n, prubese que n r es racional si, y slo si, el
numerador y el denominador de la forma irreducible de r son potencias n-simas de nmeros naturales.
p p
Si m 2 N, prubese que n m es racional si, y slo si, n m es natural.
p p
Solucin: Si r = hn =k n para ciertos h; k 2 N, ser n r = h=k 2 Q. En el supuesto de que s = n r es
racional, si s = h=k y r = p=q son las formas irreducibles de s y r, ser hn =k n = p=q; como hn =k n y p=q
son irreducibles, de hn =k n = p=q se inere que p = hn y q = k n ,es decir r = hn =k n .
p p
Si n m es natural, entonces es racional. Si n m es racional, segn se acaba de obtener es m = hn =k n para
p p
ciertos h; k 2 N primos entre s; como m es natural, ser k n = 1 y, por tanto, n m = n hn = h, que es
natural.

9. Hallar:
202 CAPTULO 5. NMEROS REALES
p p
a) Los valores de n 2 N para los que 2n + 6 + 2n 6 es racional.
p
Solucin: Obsrvese primeramente que ha de ser n 3, pues de lo contrario 2n 6 no sera real. Si
p p p p
r = 2n + 6 + 2n 6 es racional, tambin lo es r2 = 4n + n2 9 luego tambin lo ser n2 9,
lo cual slo ocurre si n2 9 es un cuadrado perfecto, esto es si n2 9 = m2 para cierto m 2 N (donde
es m < n); como n2 m2 n2 (n 1)2 = 2n 1, para que n2 m2 = 9, ha de ser 9 2n 1,
esto es n 5.Por tanto, los nicos valores de n que hay que considerar son n = 3; 4; 5; para n = 3,
p p p p
r = 12 no es racional; para n = 4, r = 14 + 2 no es raciona, pues de serlo lo sera r2 16 + 4 7;
p p
para n = 5, r = 16 + 4 = 6 es racional. La nica solucin del problema es pues n = 5.
p
b) Los valores de r 2 Q para los que r2 + 1 es racional.
p p
Solucin: Si r y r2 + 1 son racionales, tambin lo es s = r2 + 1 r; esta igualdad conduce a
p
(s + r)2 = r2 + 1, de la que se obtiene r = (1 s2 )=2s. Por tanto, para que r2 + 1 sea racional
p
es necesario que sea r = (1 s2 )=2s. Por tanto, para que r2 + 1 sea racional es necesario que sea
r = 1 s2 =2s para algn racional s 6= 0. Para cada uno de estos valores de r, ser:
r s
p (1 s2) (1 + s2 )
2
1 + s2
r2 + 1 = 2
+1= 2 = ;
4s (2s) 2s

que es racional; en consecuencia, las soluciones del problema son los elementos del conjunto

r= 1 s2 =2s : s 2 Q; s 6= 0 :

5.2. Mtodo de Induccin Completa

Para demostrar que una igualdad (o desigualdad), por ejemplo, que: 1 + 3 + 5 + : : : + (2n 1) = n2 , es cierta
para todo nmero natural (8n 2 N), existe el mtodo llamado de induccin completa, que consiste en:

1. Se prueba que la igualdad es cierta para n = 1 (o para varios valores de n). Esto es: 1 = 12 .

2. Se supone que es cierta para n = k, que es la hiptesis de induccin. En nuestro caso sera:

1 + 3 + 5 + ::: + (2k 1) = k 2 (1)


| {z }
hip tesis de induccin

3. Basados en la hiptesis, demostrar que la igualdad es tambin cierta para n = k + 1, es decir, que
?
1 + 3 + 5 + ::: + (2k 1) + (2(k + 1) 1) = (k + 1)2

En nuestro caso se procede as:


uso de la hip tesis de induccin
#
1 + 3 + ::: + (2k 1) + (2 (k + 1) 1) = k 2 + (2k + 1) = (k + 1)2 :
(1)

Esencia de este mtodo: Si del hecho de suponer cierta la igualdad para n = k, se deduce que tambin lo es
para n = k + 1, entonces el que sea cierta la igualdad para n = 1 (por haberla probado en particular) implica la
validez de la misma para n = 2, que a su vez implicar la validez para n = 3, y as sucesiva e indenidamente
para todo nmero natural.
P
Observaciones: Los smbolos de sumatoria ( ) que se lee sigma(letra griega mayscula correspondiente
Q
a nuestra S), y de producto ( ) que se lee pi (letra griega mayscula correspondiente a nuestra P):
P
n P
| En la expresin ai , el smbolo signica suma o sumatoria de los trminos ai , donde i es el ndice
i=m
inferior que va desde i = m (m es un natural, por ejemplo, desde i = 0, i = 1, i = 2 u otro valor) hasta i = n
5.2. MTODO DE INDUCCIN COMPLETA 203

(n es un natural m y sirve de tope o lmite superior de la sumatoria). Las sumatorias ms comunes son:
P
n P
n Pn
ai = am + am+1 + + an con m n, ai = a0 + a1 + + an o ai = a1 + a2 + + an :
i=m i=0 i=1

Q
n Q
| En la expresin ai , el smbolo signica producto de los trminos ai (se supone que ai 6= 0), donde
i=m
i es el ndice inferior que va desde i = m (m es un natural, por ejemplo, desde i = 0, i = 1, i = 2 u otro valor)
hasta i = n (n es un natural 1 y sirve de tope o lmite superior del producto).

Recuerda algunas de las propiedades de las sumatorias y productos:

P
n P
n
1. ai = a1 +a2 + +an es lo mismo que ak = a1 +a2 + +an , es decir una sumatoria no se altera al
i=1 k=1
Qn Q
n
cambiar el ndice de la suma. Anlogamente ai = a1 a2 an es lo mismo que ak = a1 a2 an :
i=1 k=1

P
n P
n
2. cai = ca1 + ca2 + + can = c (a1 + a2 + + an ) = c ai , es decir, en una sumatoria es posible
i=1 i=1
Q
n
sacar un factor que no depende del ndice i. Anlogamente cai = ca1 ca2 can = cn a1 a2 an :
i=1

P
n P
n P
n
3. (ai + bi ) = a1 + b1 + a2 + b2 + + an + bn = a1 + a2 + + an + b1 + b2 + + bn = ai + bi ,
i=1 i=1 i=1
Pn P
n P
n
(ai bi ) = a1 b1 +a2 b2 + +an bn = (a1 + a2 + + an ) (b1 + b2 + + bn ) = ai bi , es
i=1 i=1 i=1
decir una sumatoria es distributiva con respecto a la suma o diferencia de sus trminos. Para el producto
Q
n Q
n Qn
se tiene: ai bi = a1 b1 a2 b2 an bn = a1 a2 an b1 b2 bn = ai bi
i=1 i=1 i=1

4. Las propiedades 2. y 3. anteriores, conjuntamente se generalizan para la sumatoria mediante la relacin:


Pn
(cai dbi ) = ca1 db1 + ca2 db2 + + can dbn = ca1 + ca2 + + can db1 db2 dbn =
i=1
P
n P
n
= c (a1 + a2 + + an ) d (b1 + b2 + + bn ) = c ai d bi
i=1 i=1

Figura 5.2: Izq. El genio matemtico de las ecuaciones y funciones algebraicas, el noruego, Niels Abel (Finnoy,1802 -
Froland, 1829). En 1824 demostr la imposibilidad de resolver la ecuacin de 5to grado por mtodos algebraicos. Estudi
y desarroll la teora de las funciones elpticas, los grupos conmutativos o abelianos y algunas ecuaciones integrales.
Muri en extrema pobreza y de tuberculosis. Der. Bernhard Bolzano (Praga, 1781 - 1848), lsofo, tologo (sacerdote
pacista catlico) y matemtico checo. Sistematiz la teora de funciones y demostr varios teoremas sobre lmites y
continuidad de funciones. Precursor de la teora moderna de conjuntos y del innito con sus paradojas.

Ejemplos:

Consideremos algunos ejemplos de demostracin por induccin:


204 CAPTULO 5. NMEROS REALES

a1 (q n 1)
1. La suma de trminos de la progresin geomtrica (a1 + a1 q + a1 q 2 + + a1 q n ) es Sn = , q 6= 1
q 1
a1 (q 1)
Demostracin: Para n = 1 se tiene S1 = a1 = . Supongamos que la igualdad es vlida
q 1
para n = k, entonces

a1 q k 1 a1 q k 1 a1 q k+1 1
Sk = ) Sk+1 = Sk + ak+1 = + a1 q k = ;
q 1 q 1 q 1

es decir, la igualdad dada es vlida para n = k + 1. Por eso es vlida para cualquier n.

2. Demostrar por el mtodo de induccin completa, que para todo n 2 N son ciertas las igualdades:

n(n + 1)
a) 1 + 2 + ::: + n = :
2
1(1 + 1) 2
Solucin: Para n = 1: 1 = = = 1, es cierta. Supuesta cierta para n = k: 1+2+: : :+k =
2 2
k(k + 1)
, se trata de demostrar que tambin lo es para n = k + 1, es decir, que:
2

? (k + 1)((k + 1) + 1)
1 + 2 + : : : + k + (k + 1) =
2
En efecto:
hip tesis de induccin k(k + 1)
1 + 2 + ::: + k +(k + 1) = + (k + 1) =
| {z } 2

k(k + 1) + 2(k + 1) k 2 + k + 2k + 2 k 2 + 3k + 2 (k + 1)(k + 2)


= = = = :
2 2 2 2
Observemos que k 2 + 3k + 2 = (k + 1)(k + 2), por tanto, es verdadera la igualdad inicial.
n2 (n + 1)2
b) 13 + 23 + ::::: + n3 =
4
3 12 (1 + 1)2 4
Solucin: Para n = 1, 1 = = = 1;es cierta.
4 4
k 2 (k + 1)2
Supuesta cierta para n = k, 13 +23 +:::+k 3 = , probemos que tambin lo es para n = k +1,
4
2 2
? (k + 1) ((k + 1) + 1)
es decir, veamos que: 13 + 23 + ::: + k 3 + (k + 1)3 = .
4
hip tesis de induccin
En efecto : 13 + 23 + ::: + k 3 + (k + 1)3 =
| {z }
hip tesis de induccin

k 2 (k + 1)2 k 2 (k + 1)2 + 4(k + 1)3


= + (k + 1)3 = =
4 4
(k + 1)2 k 2 + 4 (k + 1) (k + 1)2 k 2 + 4k + 4 (k + 1)2 (k + 2)2
= = = :
4 4 4

c) 6 (1 + 7 + 72 + ::::: + 7n ) + 1 = 7n+1
Solucin: Para n = 1, 6(1 + 7) + 1 = 48 + 1 = 49 = 71+1 es cierto.
Supuesta esta ecuacin vlida para n = k, 6(1 + 7 + 72 + ::: + 7k ) + 1 = 7k+1 , probemos su validez
?
para n = k + 1. Es decir, veamos que: 6(1 + 7 + 72 + ::: + 7k + 7k+1 ) + 1 = 7(k+1)+1 = 7k+2 . En
efecto: 6(1 + 7 + 72 + ::: + 7k + 7k+1 ) + 1 =
| {z }
hip tesis de induccin

hip tesis de induccin


= (6(1 + 7 + 72 + ::: + 7k ) + 1) + 6;7k+1 ) + 1 = 7k+1 + 6 7k+1 = (1 + 6)k+1 = 7k+2 :

3. Demostrar por induccin:


5.2. MTODO DE INDUCCIN COMPLETA 205
n
X 1 n
a) = :
r=1
r(r + 1) n+1
1
X 1 1 1 1
Solucin: Para n = 1: = = = , es cierto.
r=1
r(r + 1) 1(1 + 1) 2 1+1
k
X 1 k
Supuesto cierto para n = k, = , vemos que tambin lo es para n = k + 1, es
r=1
r(r + 1) (k + 1)
decir, que:
k+1
X 1 ? k+1 k+1
= =
r=1
r(r + 1) (k + 1) + 1 k+2
En efecto,
k+1 k
!
X 1 X 1 1 hip tesis de induccin
= + =
r=1
r(r + 1) r=1
r(r + 1) (k + 1)((k + 1) + 1)
k 1 k (k + 2) + 1
= + = =
k + 1 (k + 1) (k + 2) (k + 1) (k + 2)
2
k 2 + 2k + 1 (k + 1) k+1
= = =
(k + 1) (k + 2) (k + 1) (k + 2) k+2
Nota:
k+1
X 1 1 1 1
= + + ::: + +
r=1
r(r + 1) 1(1 + 1) 2(2 + 1) k(k + 1)

1 1 1 1
+ = + + ::: + +
(k + 1)((k + 1) + 1) 2 2 3 k(k + 1)
k
X
1 1 1
+ = + :
(k + 1) (k + 2) r=1
r(r + 1) (k + 1) (k + 2)

b) 3 52n+1 + 23n+1 = 17 (17 signica mltiplo de 17).


Solucin: Para n = 1: 3 52+1 + 23+1 = 3 125 + 16 = 375 + 16 = 391 = 17 23 = 17 es cierto.
Supuesto cierto para n = k, 3 52k+1 + 23k+1 = 17, veamos que tambin lo es para n = k + 1:

3 52(k+1)+1 + 23(k+1)+1 = 3 52k+2+1 + 23k+3+1 = 52 3 52k+1 +

+23 23k+1 = 25 3 52k+1 + 8 23k+1 = (17 + 8) 3 52k+1 +

+8 23k+1 = 17 3 52k+1 + 8 3 52k+1 + 23k+1 =


hip tesis de induccin
= 17 + 17 = 17:

4. Demostrar que si n es un nmero entero positivo, 4n + 15n 1 se divide por 9.


Demostracin: Si n = 1, el nmero 4n + 15n 1 es igual a 18, es decir se divide por 9. Supongamos que
4k + 15k 1 se divide por 9 y sea n = k + 1, entonces

4k+1 + 15 (k + 1) 1 = 4 4k + 15k 1 45k + 18 = 4 4k + 15k 1 9 (5k 2) :

Pero, segn la hiptesis de induccin, 4k + 15k 1 se divide por 9, a causa de que el segundo miembro, y
junto con ste el primer miembro de la igualdad se divide por 9, lo que era necesario demostrar.

5. Demostrar que, si a y b son nmeros positivos y a < b, para cualquier n natural es vlida la desigualdad
an < bn .
Demostracin: Para n = 1, la armacin es evidente. Supongamos que ak < bk ; al multiplicar esta
desigualdad por un nmero positivo a, obtenemos ak+1 < abk . Pero b es nmero positivo debido a que
bk a < bk b, es decir, ak+1 < bk+1 , lo que era necesario demostrar.
206 CAPTULO 5. NMEROS REALES
n
n+1
6. Demostrar que para n > 1 es vlida la desigualdad n! < , (n! = 1 2 3 : : : n) :
2
k
k+1
Demostracin: Para n = 2 obtenemos una desigualdad correcta 2 < 9=4. Supongamos que k! < .
2
k
Entonces, segn la hiptesis de la induccin, (k + 1)! = k! (k + 1) < k+12 (k + 1). Si ahora demostramos
que
k k+1
k+1 k+2
(k + 1) < ; (2)
2 2
entonces el teorema ser demostrado, porque
k k+1
k+1 k+2
(k + 1)! < (k + 1) < ;
2 2

es decir, nuestra desigualdad se cumple para n = k + 1.


k+1
1
Evidentemente, la desigualdad (2) se puede escribir en la forma 2 < 1 + k+1 . No obstante, segn
la frmula del binomio de Newton
k+1
1 1
1+ = 1 + (k + 1) + : : : > 2;
k+1 k+1

por consiguiente, esta desigualdad (2) es vlida y la desigualdad inicial queda demostrada.

7. Demostrar el teorema: si el producto de varios nmeros (n 2) positivos es igual a 1, entonces su suma es


mayor o igual a n, es decir, si x1 x2 : : : xn = 1, x1 > 0, x2 > 0, . . . , xn > 0, entonces x1 + x2 + : : : + xn n.
Demostracin: Si n = 2 nos toca demostrar que si x1 x2 = 1, entonces x1 + x2 2. Como
p p 2 p
( x1 x2 ) 0 ) x1 + x2 2 x1 x2 0)
x1 + x2 p x1 + x2
x1 x2 = 1 ) 1 ) x1 + x2 2:
2 2
x1 + x2 p
De x1 x2 se desprende que la media aritmtica de 2 nmeros no es menor que la media
2
geomtrica. La igualdad se cumple cuando x1 = x2 = 1:
Por hiptesis tomemos x1 , x2 , : : :,xk , xk+1 tales que x1 x2 : : : xk xk+1 = 1. Si cada sumando es igual a 1,
entonces x1 + x2 + : : : + xk + xk+1 = k + 1, debido a que la desigualdad demostrada en este caso es vlida.
De no ser as, entonces entre los nmeros habr uno menor a 1 y otro mayor a 1. Admitamos que xk > 1
y xk+1 < 1. En este caso tendremos x1 : : : xk 1 (xk xk+1 ) = 1.
Este es el producto de k nmeros y por tanto es aplicable la hiptesis de induccin y por tanto podemos
armar que
x1 + : : : + xk 1 + xk xk+1 k

Pero entonces

x1 + : : : + xk 1 + xk + xk+1 k xk xk+1 + xk + xk+1 = k + 1 + (xk 1) (1 xk+1 ) > k + 1;

porque xk 1 > 0 y 1 xk+1 > 0, que es lo que queramos demostrar. Ntese que la igualdad se cumple
siempre y cuando xi = 1. Si xi 6= 1, entonces la desigualdad es rigurosa.
De este teorema resulta la desigualdad generalizada:
x1 + : : : + xn p
n
x1 : : : xn ; x1 > 0; : : : ; xn > 0;
n
que nos dice que la media aritmtica de n nmeros no es menor a la media geomtrica de esos nmeros
positivos.
5.2. MTODO DE INDUCCIN COMPLETA 207
p xi x1 : : : xn
Designando n x1 : : : xn = c y = yi , tendremos y1 : : : yn = = 1. Segn lo ya demostrado
c cn
x1 : : : xn x1 + : : : + xn
y1 + : : : + yn n) n) c, l.q.q.d.
c n

Aplicando la desigualdad ya demostrada a los nmeros 1, 2, 3,. . . , n, obtendremos:


n
p 1 + 2 + ::: + n p (n + 1) p n+1 n+1
n
) n! < 2
n n n
1 2 ::: n < ) n! < ) n! <
n n 2 2

8. Demostrar que para cualquier nmero entero positivo n tiene lugar la desigualdad jsin nxj n jsin xj :
Demostracin: Para n = 1 la desigualdad es evidentemente vlida. Suponiendo que jsin kxj k jsin xj,
demostremos que jsin (k + 1) xj (k + 1) jsin xj. En realidad, utilizando la desigualdad jcos kxj 1,
tenemos que

jsin (k + 1) xj = jsin kx cos x + sin x cos kxj jsin kxj jcos xj + jsin xj jcos kxj
jsin kxj + jsin xj k jsin xj + jsin xj = (k + 1) jsin xj , l.q.q.d.

Practico lo que aprend:

Demostrar por induccin:

n
X n
X n
X n
X
n (1 + 5n)
1. a) (2k 1) = n2 , b) 2k = n + n2 , c) (5k 2) = , d) 2k = 2 (2n 1)
2
k=1 k=1 k=1 k=1

n
X n n
X
k 1 ( 1) (1 6n) 1 2n3 3n2 2n + 3n4
2. a) ( 1) (3k 2) = , b) k3 k2 =
4 12
k=1 k=1

n
X n
X n
X n
(3 3n 1) n 2 + n2 k 1 ( 1) (1 14n) 1
3. a) 3k = , b) k2 k+1 = , c) ( 1) (7k 4) =
2 3 4
k=0 k=1 k=1

n
X n n
X n
k 2 (( 2) 1) k 4 ( 2) + 9 3n 1
4. a) ( 2) = , b) 3k + ( 2) =
3 6
k=1 k=0

n
X n
X 2
n (n + 1) (2n + 1) n (n + 1)
5. a) k 2 = 12 + 22 + : : : + n2 = , b) k 3 = 13 + 23 + : : : + n3 =
6 2
k=1 k=1

n+1 n
n
X sin x sin x n
X n
2 2 k 1 ( 1) 1 6n2 4n3 1
6. a) sin kx = x ; x 6= 2 k, b) ( 1) k3 =
sin 8
k=1 k=1
2
n
7. La frmula de De Moivre: (r (cos + i sin )) = rn (cos n + i sin n )
n 2n
2 (n + 1) (2n + 1) 2n + 1
8. a) n! > 2n 1
, si n > 2; b) 2n n! < nn , si n > 2; c) (n!) < ; d) (2n)! <
6 2
208 CAPTULO 5. NMEROS REALES

Figura 5.3: Izq. El gran analtico francs, Agustn Cauchy (Paris, 1789 - Sceaux, 1857), gran especialista
en anlisis clsico, hidrodinmica y creador de la teora de variable compleja. Trabajos en teora de grupos,
elasticidad y astronoma, etc. Desarroll la teora de lmites y de continuidad de funciones. Introdujo muchsimos
conceptos matemticos totalmente bien denidos e hizo contribuciones importantes a las ecuaciones diferenciales.
Era un monarquista consumado. Der. Bonaventura Francesco Cavalieri (Miln, 1598 - Bolonia, 1647), gran
matemtico, gemetra y astrnomo italiano. Discpulo de Galileo. Introdujo los indivisibles para el clculo de
las supercies de guras geomtricas y de volmenes, con lo que se anticip al clculo innitesimal. Estudi las
cnicas, la hidralica e introdujo el principio que lleva su nombre.

Figura 5.4: Izq. Isaac Newton (Woolsthorpe, Lincolnshire,1642 - Londres, 1727), el genio ingls irrepetible. Considerado
como el cientco ms grande de todos los tiempos, inventor (junto con Leibniz (Leipzig, 1646 - Hannover, 1716)) del
clculo diferencial (uxiones) e integral. Padre del Calculus y de la fsica clsica, de la teora de la gravitacin universal,
de las leyes fundamentales de la mecnica y de la ptica. Newton tambin fue director de la Casa de la Moneda inglesa.
Der. El clebre fsico, matemtico y astrnomo irlands William Rowan Hamilton (Dubln,1805 - Dunsink, 1865). Nio
prodigio, polglota y descubridor a los 18 de un error en la Mecnica Celeste de Laplace (Beaumont en Auge, 1749 - Pars,
1827). Gran especialista en ptica, mecnica, clculo variacional. Introdujo los cuaternios y el principio de Hamilton sobre
la accin mnima en mecnica.
Captulo 6

IGUALDADES Y DESIGUALDADES
NUMRICAS

La matemtica es el trabajo del espritu humano que est destinado tanto

a estudiar como a conocer, tanto a buscar la verdad como a encontrarla.

Evariste Galois

No hay ninguna rama de la matemtica, por abstracta que sea, que

no pueda aplicarse algn da a los fenmenos del mundo real.

N. Lobachevski

Unidad de competencia: Conocer los conceptos bsicos, principios operacionales, aplicaciones y procesos de
resolucin de igualdades y desigualdades numricas.

Indicadores de logro:

| Interpretar, analizar e integrar conceptos, principios y propiedades de las igualdades y desigualdades.

| Aprender a manipular y operar correctamente con las igualdades y desigualdades.

| Aplicar las propiedades de las desigualdades e igualdades en diferentes demostraciones de teoremas.

Un caballo come un fardo de forraje en un mes, una cabra en dos meses y una oveja en tres meses. En qu tiempo, el
caballo, la cabra y la oveja conjuntamente se comen ese mismo fardo?

Un caminante va de una ciudad a otra durante 10 das, un segundo caminante recorre el mismo camino en 15 das.
Despus de cuntos das se topan los caminantes, si salen al mismo tiempo al encuentro el uno con el otro?

Doce personas llevaban 12 panes: cada hombre lleva 2 panes, las mujeres llevan medio pan y los nios un cuarto de
pan. Cuntos hombres, mujeres y nios haba?

Dos amigos coman manzanas. El uno le dice al otro: dame 2 manzanas y entonces tendremos por igual. El otro
contesta: mejor dame t 2 manzanas y entonces tendr el doble que tu. Cuntas manzanas tena cada uno?

209
210 CAPTULO 6. IGUALDADES Y DESIGUALDADES NUMRICAS

Figura 6.1: Izq. El clebre matemtico ruso Nikolai Lobachevskii (Nizhni Novgorad, 1792 - Kazn, 1856), uno de los
creadores de la geometra no eucldea, aliengena o extraterrestre, la nueva geometra hiperblica, independiente del 5to
postulado de Euclides de las paralelas, y que arma que por un punto exterior a una recta pasa ms de una paralela a ella,
o que la suma de los ngulos de un tringulo es menor que 180 grados. Fue rector de Universidad de Kazn e hizo algunas
contribuciones al anlisis matemtico y a la solucin aproximada de ecuaciones. Der. El gran libro de la naturaleza
(el Universo) est escrito en lenguaje matemtico. Galileo Galilei (Pisa, 1564 - Arcetri, Florencia, 1642), famossimo
astrnomo, fsico experimentador (cinemtica, esttica, dinmica, hidralica, etc.) y matemtico italiano. Descubri
muchos planetas, satlites y cometas junto con el estudio de sus movimientos. Fue perseguido por la Inquisicin, que le
oblig a abjurar de las teoras de Nicols Coprnico (Torun, Polonia, 1473 - Frombork, 1543). En el proceso en su contra,
dijo: Eppur si muove y fue rehabilitado 350 aos despus por el papa polaco Juan Pablo II.

6.1. Conceptos Bsicos

Los conceptos de igualdad y de desigualdad son fundamentales en matemticas, con ayuda de estos es posible
comparar y relacionar las cantidades numricas reales. As, para describir que los nmeros son iguales entre s
utilizamos el smbolo =, y para indicar que los nmeros no son iguales, usamos los smbolos > (mayor), <
(menor), (mayor o igual), (mayor o igual). Los smbolos > y < se denominan smbolos de desigualdades
estrictas.

6.1.1. Deniciones y propiedades elementales de las desigualdades

Las expresiones con la forma P < Q (P Q); P > Q (P Q), donde P y Q pueden ser nmeros,
expresiones algebraicas con varias variables o valores de ciertas funciones, se llaman desigualdades. Los smbolos
< ( ), > ( ) se llaman signos de desigualdad y se leen respectivamente menor (menor o igual), mayor (mayor
o igual).

Las desigualdades que se escriben mediante los signos > y <, se llaman estrictas, y las que contienen los
signos y se llaman no estrictas. Una desigualdad no estricta equivale a la desigualdad estricta del mismo
signo ms la igualdad.

Se distinguen 2 tipos de desigualdades: las aritmticas (o numricas), cuya expresin contiene solo nmeros,
y las no aritmticas cuya expresin contiene, junto con los nmeros, valores de funciones o expresiones con una
o varias variables. As, por ejemplo:
p 1
7 > 2; 3 5; sin > ; e3 < 30; 2
9 y
2 3
3y 1
a < 1; sin2 x + tan x > 0; x2 + y 2 r2 ; 2x 3y > 2; 2;
y+4

son desigualdades aritmticas y no aritmticas, respectivamente. Se supone que las variables de los anteriores
6.1. CONCEPTOS BSICOS 211

ejemplos son reales: a, x, y, r 2 R:

Las expresiones que entran en las desigualdades pueden admitir distintos valores numricos, en dependencia
de los distintos valores que tomen sus argumentos. Para unos valores de los argumentos la desigualdad puede
convertirse en desigualdad estricta verdadera o falsa, para otros valores, no.

Entre las propiedades principales de las desigualdades tenemos:

(
Si a > b, entonces b < a
Antisimtrica:
Si a < b, entonces b > a
(
Si a > b y b > c, entonces a > c
Transitiva:
Si a < b, y b < c, entonces a < c

Operaciones aritmticas con las desigualdades

1) Dos desigualdades de un mismo signo o sentido pueden sumarse, y su resultado es una desigualdad del mismo
sentido:
Si a > b y c > d; entonces a + c > b + d
Si a < b y c < d; entonces a + c < b + d

2) Las siguientes reglas son vlidas cuando se multiplica (o se divide) una desigualdad por un mismo valor
positivo o negativo (en este caso cambian de sentido):
Si a>b y c > 0; entonces ac > bc
Si a<b y c > 0; entonces ac < bc
Si a>b y c < 0; entonces ac < bc
Si a<b y c < 0; entonces ac > bc
Todo lo anterior para las desigualdades estrictas (<, >) es vlido tambin para las no estrictas ( , ).

3) Como consecuencia de las reglas de adicin de las desigualdades y de multiplicacin de ambos miembros de
las desigualdades por un mismo valor, se obtiene la regla de sustraccin de las desigualdades de distinto
sentido:
De una desigualdad se puede, miembro a miembro (del 1 er miembro sustraer el 1 ro , y del 2 do miembro
sustraer el 2 do ) sustraer otra desigualdad que tiene el sentido contrario al de la 1 ra desigualdad :
Si a < b y c > d; entonces a c < b d
Si a > b y c < d; entonces a c > b d

4) Si al 1ro y 2do miembros de una desigualdad les agregamos un mismo valor, entonces se obtiene una de-
sigualdad del mismo sentido:
Si a > b; entonces a + c > b + c
Si a < b; entonces a + c < b + c

Denicin 1. La proposicin en la que dos nmeros o cantidades estn enlazados mediante el smbolo de
igualdad, se denomina igualdad numrica, y aquella en la que estn enlazados con el smbolo de desigualdad, se
denomina desigualdad numrica.

Ejemplos:

p p
1. Las expresiones 24 = 16, 5 = 5, 8 = 2 2, lg 32 = 5 lg 2, log3 81 = 4, sin 6 = sin 56 son igualdades.
p p
2. Mientras que las expresiones 7 > 3, 3 12 < 4, 3 < 5, lg 10 > lg 9;5, ln 0;3 < ln 0;7 son desigualdades.
212 CAPTULO 6. IGUALDADES Y DESIGUALDADES NUMRICAS

| Las igualdades o desigualdades numricas pueden ser verdaderas o falsas. La igualdad 7 = 7 es verdadera,
mientras que la igualdad 3 = 7 es falsa. Anlogamente, la desigualdad 13 < 15 es verdadera, mientras que 2 >
es falsa.

En adelante, si no indicamos lo contrario, consideraremos que las proposiciones a = b, a > b y a < b son
verdaderas, siendo a, b, c, : : : nmeros reales.

Las desigualdades a > b y c > d son desigualdades del mismo sentido, y a > b, c < d son de sentido contrario.
En la desigualdad a > b y en la igualdad a = b, a es el miembro izquierdo y b el derecho.

Las desigualdades, generalmente, son analizadas en el conjunto R de los nmeros reales, pues en cualquier
ampliacin de los mismos, los conceptos de mayor y menor no estn denidos. Por ejemplo, en el conjunto
de nmeros complejos C, para los nmeros complejos a = 3 2i y b = 8+0;5i, no estn denidas las relaciones
a > b o a < b, pero si a 6= b o a = b.

| En el conjunto R de los nmeros reales, se cumplen las leyes conmutativa (a + b = b + a, ab = ba),


asociativa (a + (b + c) = (a + b) + c, a (bc) = (ab) c) y distributiva (a (b + c) = ab + ac).

La teora de las desigualdades e igualdades se basa en 3 proposiciones:

Recuerda estas proposiciones sobre las desigualdades:

Proposicin 1. Para cualquier nmero a 2 R es verdadera una y solamente una de las siguientes
relaciones: a = 0, a > 0, a < 0.

Denicin 2. Si a > 0 es una desigualdad verdadera, entonces el nmero a se denomina positivo. Si a < 0
es una desigualdad verdadera, entonces el nmero a se denomina negativo.

Por tanto, el conjunto R puede descomponerse en 3 subconjuntos disjuntos: 1) los nmeros negativos, 2) el
nmero cero (neutro), y 3) los nmeros positivos.

Proposicin 2. Si a > 0 y b > 0 son desigualdades verdaderas, entonces a + b > 0 y ab > 0 tambin
son verdaderas.

Por induccin tenemos que si a1 > 0, a2 > 0,: : :, an > 0 son desigualdades verdaderas, entonces a1 + a2 +
::: + an > 0 y a1 a2 : : : an > 0 tambin son verdaderas.

Proposicin 3. Se cumplen las relaciones:

a. Las igualdades a b = 0 y a = b o bien ambas son verdaderas, o bien ambas son falsas.

b. Las desigualdades a b > 0 y a > b o bien ambas son verdaderas, o bien ambas son falsas.

c. Las desigualdades a > b y b < a o bien ambas son verdaderas, o bien ambas son falsas.

Denicin 3. Si a + b = 0 es una igualdad verdadera, entonces los nmeros a y b se denominan opuestos.


El opuesto de a se lo representa con a. El opuesto de a es a. Si a = 0, entonces a = 0, pues 0 + 0 = 0,
es decir ( a) = a, siendo el nmero 0 el opuesto de si mismo, pues resulta ser neutro.

6.1.2. Leyes de los signos

1. a b = a + ( b), es decir, la diferencia de dos nmeros es igual a la suma del minuendo con el opuesto
del sustraendo.
6.1. CONCEPTOS BSICOS 213

Figura 6.2: Izq. El famoso astrnomo, matemtico y telogo alemn Johann Kepler (Weil, Wurttenberg, 1571 - Ratis-
bona,1630). Hizo asombrosos clculos sobre las rbitas de los planetas para deducir de ah las muy conocidas Leyes de
Kepler, fundamento de la astronoma moderna. Se le considera como el fundador de la Mecnica Celeste y gran investi-
gador de la ptica. Der. El famoso matemtico francs de origen italiano Joseph Lagrange (Turn, 1736 - Pars, 1813),
sucesor de Leonhard Euler (Basilea, 1707 - San Petersburgo, 1783) como el ms grande matemtico de la Europa de ese
tiempo. Senador y conde del imperio de Napolen Bonaparte. Gran mecnico, especialista en lgebra (teora de ecua-
ciones, grupos), clculo clsico (varios teoremas y aplicaciones), clculo de variaciones y funciones analticas. Instaur el
sistema mtrico decimal. Explic el movimiento lunar y de otros satlites del sistema solar.

Efectivamente, como b + b = 0 (denicin de opuestos), entonces a = a + (( b) + b) (propiedad del cero),


o a = (a + ( b)) + b (propiedad asociativa). De donde a b = a + ( b) (denicin de diferencia de dos
nmeros).

2. (a + b) = a + ( b), es decir, el opuesto de la suma de 2 nmeros es igual a la suma de los opuestos.


En efecto, utilizando la primera ley, calculamos la diferencia ( a + ( b)) ( (a + b)) :

( a + ( b)) ( (a + b)) = ( a + ( b)) + ( ( (a + b))) =


= ( a + ( b)) + (a + b):

Utilizando las leyes asociativa y conmutativa de la suma y la denicin de opuestos, tendremos:

( a + ( b)) + (a + b) = ( a + a) + ( b + b) = 0:

Por lo tanto ( a + ( b)) ( (a + b)) = 0, de donde (proposicin 3) a + ( b) = (a + b):

3. (a b) = a + b, es decir, el opuesto de la diferencia de 2 nmeros es igual a la suma del opuesto del


minuendo con el sustraendo.
Utilizando las 2 leyes anteriores y la denicin de nmero opuesto, tenemos

(a b) = (a + ( b)) = a + b:

4. ab = a( b), es decir, el opuesto del producto de 2 nmeros es igual al producto de uno de los factores
por el opuesto del otro factor.
En efecto, calculamos la diferencia a( b) ( ab). Utilizando sucesivamente la ley de los signos 1, la ley
distributiva y la denicin de opuestos, tendremos:

a( b) ( ab) = a( b) + ab = a( b + b) = a 0 = 0:

As que a( b) ( ab) = 0, de donde ab = a( b):


214 CAPTULO 6. IGUALDADES Y DESIGUALDADES NUMRICAS

Principales consecuencias:

Teorema 1. El nmero opuesto a un nmero positivo es un nmero negativo, y el opuesto a un negativo


es un nmero positivo, es decir, si a > 0, entonces a < 0, y si a < 0 entonces a > 0:

Demostracin: Sea a > 0, entonces 0 + a > 0 (propiedad del cero) y 0 ( a) > 0 (ley de los signos 1), por
consiguiente 0 > a y a < 0 (proposicin 3).

Al contrario, si a < 0, entonces 0 > a y 0 a > 0 (proposicin 3), de donde 0 + ( a) > 0 y a > 0:

Teorema 2. El producto de un nmero positivo por un negativo es un nmero negativo.

Demostracin: Sean a > 0 y b < 0. Como b < 0, entonces su opuesto b > 0 (teorema 1). Por lo tanto
a( b) > 0 (proposicin 2) y (a( b)) < 0 (teorema 1). Como (a( b)) = ab (ley de los signos 4), entonces
ab < 0:

Anlogamente se demuestran los siguientes teoremas:

Teorema 3. El producto de 2 nmeros negativos es un nmero positivo.

Teorema 4. El cuadrado de todo nmero (excepto el 0) es un nmero positivo.

Teorema 5. Si los nmeros a1 ; a2 ; :::; an son todos negativos, entonces su suma a1 +a2 +:::+an es negativa.

Teorema 6. Un nmero positivo o nulo es mayor que un nmero negativo.

Teorema 7. Si a b < 0 entonces a < b y a la inversa.

Demostracin: Sea a b < 0, el opuesto es (a b) > 0 (teorema 1). Pero (a b) = a + b (ley de los
signos 3), y a + b = b a (ley de los signos 1), por lo tanto b a > 0, de donde b > a y a < b. Anlogamente
se demuestra la proposicin inversa.

| Conclumos en que si estn dados dos nmeros a y b, entonces es verdadera una, y solamente una de las
proposiciones a > b, a = b o a < b. Efectivamente, de acuerdo a la proposicin 1, para el nmero a b es
posible solamente una de las posibilidades: a b = 0, a b > 0 o a b < 0. De donde, aplicando la proposicin
3 y el teorema 7, obtendremos a = b, a > b o a < b.

Para las desigualdades numricas, se cumple la ley transitiva:

Teorema 8. Si a > b y b > c, entonces a > c.

Demostracin: Como a > b y b > c son verdaderas, entonces a b > 0 y b c > 0 tambin lo son (proposicin
3). Pero entonces, la desigualdad a b + b c > 0, tambin lo es (proposicin 2). De donde, a c > 0 y a > c
(proposicin 3).

| El teorema anterior es fcil generalizarlo para n desigualdades.

6.2. Suma y Diferencia de Desigualdades Numricas

Vamos a considerar que las operaciones algebraicas (suma, diferencia, etc.) y otras operaciones trascendentes
(logaritmizacin, potenciacin, etc.) aplicadas a las desigualdades, corresponden a las respectivas operaciones
efectuadas sobre las partes izquierdas y derechas de las desigualdades dadas. As, por ejemplo, sumar 2 de-
sigualdades signica sumar sus respectivas partes izquierdas y derechas, por separado. Tomar logaritmos a una
desigualdad, signica tomar logaritmos a la parte izquierda y a la parte derecha, por separado.
6.2. SUMA Y DIFERENCIA DE DESIGUALDADES NUMRICAS 215

Por comodidad, aplicaremos las diferentes operaciones a las desigualdades del tipo a > b, que equivale a
operar con la desigualdad b < a, donde a; b 2 R.

Teorema 9. Si a > b y c es un nmero cualquiera, entonces a + c > b + c.

Demostracin: De la desigualdad a > b se tiene que a b > 0, de donde a b+c c > 0 o (a+c) (b+c) > 0,
por lo tanto a + c > b + c (proposicin 3).

Conclumos que el sentido de una desigualdad no se altera si a cada uno de sus lados se le suma o se le resta
la misma cantidad.

Consecuencia. Si a > b + c, entonces a c > b, es decir, una cantidad se puede traspasar de un lado al
otro, cambindola de signo.

Figura 6.3: Izq. El famoso astrnomo y matemtico polaco Nicols Coprnico (Torun, 1473 - Frombork, 1543).
Hizo estudios de astronoma, matemticas, derecho y medicina en Polonia e Italia. Calcul las rbitas aparentes
del sol, luna y planetas. Con sus investigaciones, teoras, observaciones y experimentos destruy la cosmologa
de Ptolomeo, engao que estuvo en pie durante 1500 aos. Explic que la tierra y varios planetas giran sobre
su eje y alrededor del sol con lo que revolucion la astronoma. Der. El famoso pintor, torico del arte y
matemtico Albrecht Durer (Nuremberg, 1471 - 1528). Hizo estudios sobre el arte en Alemania, Holanda e Italia.
Escribi varios tratados sobre las proporciones, medidas y construcciones geomtricas, donde analiz cuestiones
de ornamentos geomtricos, de perspectiva, de ptica, formas arquitectnicas y de proyeccin ortogonal.

Ejemplo:

De la desigualdad 5 > 3;5, se tiene 5 4 > 3;5 4)1> 0;5

Teorema 10. Si a1 > b1 , a2 > b2 ,:::, an > bn , entonces a1 + a2 + : : : + an > b1 + b2 + ::: + bn .

Demostracin: De las desigualdades a1 > b1 , a2 > b2 ,:::, an > bn se tiene que a1 b1 > 0, a2 b2 >
0,:::, an bn > 0, de donde (a1 b1 ) + (a2 b2 ) + ::: + (an bn ) > 0, lo que nos da (a1 + a2 + : : : + an )
(b1 + b2 + ::: + bn ) > 0, es decir a1 + a2 + ::: + an > b1 + b2 + ::: + bn (proposiciones 2 y 3).

Conclumos que las desigualdades del mismo sentido pueden ser sumadas.

Ejemplos:

15 > 7 5> 3 5> 7 9> 7


+ 4> 3 , + 0> 3 , + 8>0 , + 5> 1
19 > 4 5> 6 3> 7 4> 8

Teorema 11. Si a > b y c < d, entonces a c>b d:

Demostracin: Si c < d, entonces d > c (proposicin 3). Sumando las desigualdades a > b y d > c (teorema
216 CAPTULO 6. IGUALDADES Y DESIGUALDADES NUMRICAS

10) obtenemos que a + d > b + c, de donde a c>b d (consecuencia del teorema 9).

Por tanto, 2 desigualdades de sentidos contrarios pueden restarse, quedando el sentido de la desigualdad que
hace el rol de minuendo.

Ejemplos:

10 > 7 12 < 18 5< 3 15 > 23


4 < 13 , 9>7 , 4>0 , 4<0
6> 6 3<9 9< 3 11 > 23

Practico lo que aprend:

Sumar y luego restar (cambiando o no el sentido de las desigualdades) cada una de las siguientes
parejas de desigualdades:

1. a) 2 < 5; 3> 5; b) 8 > 6; 3< 2; c) 10 < 17; 3 > 1

2. a) 12 < 2; 3> 5; b) 2 < 6; 4> 7; c) 10 > 4; 3<1

3. a) 5 > 6; 5< 2; b) 15 < 17; 3 > 1; c) 18 < 2; 3> 7

4. a) 3 < 6; 5> 7; b) 17 > 4; 3 < 8; c) 12 > 9; 5 > 2

5. a) 11 < 13; 12 < 3; b) 2 < 8; 5> 8; c) 3 < 4; 0 > 8

Figura 6.4: Izq. La clebre matemtica y autodidacta francesa Sophie Germain (Paris,1776 -1831), amante y estudiosa
de la teora de los nmeros y ecuaciones Diofnticas. Tuvo que hacerse pasar como el seor M. LeBlanc para acceder
a estudios matemticos universitarios y burlarse de las prohibiciones educativas contra las mujeres. Der. El prncipe
de los matemticos. Carl Friedrich Gauss (Brunswick, 1777 - Gotinga, 1855), nio prodigio y famossimo matemtico,
fsico y astrnomo, considerado como uno de los ms grandes matemticos de todos los tiempos. Gran especialista y
descubridor en campos como teora de nmeros, geometra clsica y no euclideana, anlisis de funciones y de series,
variable compleja, etc. Desarroll el mtodo estadstico de los mnimos cuadrados y la teora probabilstica de errores.
Dominaba casi todos los campos del saber matemtico y fsico de su tiempo. Fue profesor de Dedekind (Brunswick, 1831
- 1916), el creador del concepto de nmero real y complejo.

6.3. Producto de Desigualdades Numricas

Teorema 12. Si a > b, entonces ac > bc para c > 0, ac < bc para c < 0, y ac = bc para c = 0.
6.3. PRODUCTO DE DESIGUALDADES NUMRICAS 217

Demostracin: Como a > b, entonces a b > 0 (proposicin 3). Si c > 0, entonces (a b)c > 0 (proposicin
2), de donde ac bc > 0 y ac > bc (proposicin 3). Anlogamente, si a > b, entonces ac < bc para c < 0 y
entonces (a b)c < 0 (proposicin 2) y ac = bc para c = 0.
1
Es evidente que si c > 0, entonces > 0 y el Teorema 12 puede aplicarse al caso de divisin para c 6= 0 (o
c
1
producto por ).
c
Por tanto, cuando una desigualdad se multiplica (divide) por un nmero positivo, sta no cambia de sentido;
y cuando una desigualdad se multiplica (divide) por un nmero negativo, sta cambia al sentido contrario.

Ejemplos:

1. Multiplicar la desigualdad 3< 1;2 por 2:


Solucin: Por el teorema 12, tendremos 3 ( 2) > 1;2 ( 2) ) 6 > 2;4

2. Multiplicar 2a > b por b:


8
> 2
< Si b > 0 ) 2ab > b
Solucin: Si b < 0 ) 2ab < b2
>
:
Si b = 0 ) 2ab = 0 y b2 = 0 ) 2ab = b2

1 1 1 1
Consecuencia. Si a > b, entonces < para ab > 0, y > para ab < 0:
a b a b
a b 1 1 1 1
Demostracin: Dividimos a > b por ab. Si ab > 0, entonces > , de donde > , es decir <
ab ab b a a b
a b 1 1 1 1
(teorema 12, proposicin 3). Si ab < 0, entonces < y > , es decir > .
ab ab b a a b
Teorema 13. Si a > b, c > d, entonces para a > 0 y d > 0 tenemos que ac > bd, y para a < 0 y d < 0
tenemos que ac < bd:

Demostracin: Sean a > 0 y d > 0, considerando que c > d y a > b, obtenemos ac > ad y ad > bd (teorema
12), de donde ac > bd (teorema 8).

Por analoga se demuestra (teoremas 8, 12), tambin que ac < bd, si a < 0, d < 0, a > b, y c > d.

Ejemplos:

1. Multiplicar las desigualdades 2 > 3 y 5 > 4:


Solucin: Por el teorema 13: 2 (5) > 3 (4) ) 10 > 12:

2. Multiplicar las desigualdades 7 a2 + 3 > b y b > 3:


Solucin: De acuerdo al teorema 13, tenemos que como 7 a2 + 3 > 0 y b > 0, entonces 7b a2 + 3 > 3b:

Consecuencias:

1. Si a > c > 0 y b > d > 0, entonces ab > cd, es decir, desigualdades del mismo sentido y de tr-
minos positivos, pueden multiplicarse miembro a miembro, conservndose el sentido de la desigualdad.
Anlogamente, para las n desigualdades a1 > b1 > 0, a2 > b2 > 0, :::, an > bn > 0, se tiene que
a1 a2 :::an > b1 b2 :::bn :

2. Si a < b < 0 y c < d < 0, entonces ac > bd, es decir, desigualdades del mismo sentido y de trminos
negativos, pueden multiplicarse miembro a miembro, pero cambiando el sentido de la desigualdad.
218 CAPTULO 6. IGUALDADES Y DESIGUALDADES NUMRICAS

Figura 6.5: Izq. El clebre matemtico alemn Felix Klein (Dusseldorf, 1849 - Gotinga,1925), padre y constructor de
la geometra moderna. Aplic la teora de grupos a la geometra, deniendo el famoso Programa de Erlangen, como
conjunto de propiedades del espacio que son invariantes bajo cierto grupo de transformaciones. Especialista en funciones
algebraicas, grupos continuos, teora de ecuaciones, historia de las matemticas. Famoso pedagogo que foment e impuls
la enseanza de las matemticas a todo nivel. Der. David Hilbert (Konisberg, Prusia Oriental (ahora Kaliningrado,
Rusia), 1862 - Gotinga, 1943), genial matemtico alemn, padre de la formalizacin matemtica, creador de la matemtica
moderna del siglo XX y de los fundamentos de la geometra. Realiz avances signicativos casi en todas las ramas de
las matemticas: lgebra moderna, lgica y conjuntos (intento de formalizacin de la aritmtica), topologa, geometra
algebraica, ecuaciones integrales, anlisis funcional (espacios de Hilbert), ecuaciones de la fsica matemtica, etc. En el
congreso de matemticas (Pars, 1900) enunci sus 23 famosos problemas (todava no todos resueltos) que impulsaron el
desarrollo moderno de las matemticas, especialmente en el siglo XX.

a b
1. Si a > b > 0 y d > c > 0, entonces > , es decir, desigualdades de sentido contrario y de trminos
c d
positivos, pueden dividirse miembro a miembro, pero la desigualdad resultante conserva el sentido de la
primera desigualdad (o dividendo).

Ejemplos:

1
1. Multiplicando las desigualdades verdaderas 30 > 20, 5 > 3, 2 > , 12 > 4, tendremos 30 (5) (2) (12) >
2
1
20 (3) (4) ) 3600 > 120
2

2. Multiplicando las desigualdades verdaderas 5> 6 y 1> 3, tendremos 5 ( 1) < 6 ( 3) ) 6 <


18

Practico lo que aprend:

Multiplicar (cambiando o no su sentido) cada una de las siguientes parejas de desigualdades:

1. a) 12 > 5; 3 < 0; b) 10 > 7; 3 < 1; c) 12 < 15; 3< 1; d) 2 < 5; 3> 5:

2. a) 8 > 6; 3< 2; b) 10 < 17; 3 > 1; c) 12 < 2; 3> 5; d) 2 < 6; 4> 7:

3. a) 10 > 4; 3 < 1; b) 12 > 11; 5 > 1; c) 10 < 11; 13 < 3; d) 2 < 5; 5> 6:

4. a) 5 > 6; 5< 2; b) 15 < 17; 3 > 1; c) 18 < 2; 3> 7; d) 3 < 6; 5> 7:

5. a) 17 > 4; 3 < 8; b) 12 > 9; 5 > 2; c) 11 < 13; 12 < 3; d) 2 < 8; 5> 8:


6.3. PRODUCTO DE DESIGUALDADES NUMRICAS 219

6.3.1. Potenciacin y Radicacin de Desigualdades Numricas

Antes de exponer la potenciacin y radicacin con desigualdades, exponemos varios conceptos:


p
Denicin 4. La raz x del nmero real a 0, tal que n a = x, donde: 1) x 0, 2) xn = a, se denomina
raz aritmtica de a:
p
Tmese en cuenta, que por ejemplo, 4 = 2, pero la raz aritmtica es 2. As mismo, la raz cbica
p p
aritmtica de 8 es 2, pero todas las races cbicas de 8 son: x = 2, x = 1 3, x = 1 + 3.
n
Denicin 5. La potencia natural n 2 N del nmero a, viene denida como el producto a
| a {z: : : a} = a ,
n veces
a1 = a:

Denicin 6. La potencia racional r = p=q 2 Q del nmero a, viene denida como


p p
ar = aq = q
ap ; donde a 0; a0 = 1; donde a 6= 0;
r
p 1 1
a = a q = p = r ; donde a > 0
aq a

Propiedades de las races aritmticas


con ndices naturales:
Propiedad: Signicado:
p
n p p La raz n-sima de un producto es igual
ab = n
anb
al producto de sus races n-simas.
p k p
n La potencia k de una raz n-sima es igual
( n a) = ak
a la raz n-sima de ak .
mn
p p
n Un radical no altera al multiplicar el ndice
akm = ak
y la potencia por un mismo entero m.
r p
a n
a La raz n-sima de una divisin es igual
n
= pn
b b a la divisin de sus races n-simas.
p p
n k
p La raz n-sima de la raz k-sima se la
a = nk a
obtiene multiplicando sus ndices.
p La raz cuadrada del cuadrado perfecto
a2 = jaj
de a es igual al valor absoluto de a.

Propiedades de las potencias


racionales:
Propiedad: Signicado:
Para multiplicar potencias de igual base
ar1 ar2 = ar1 +r2
se suman los exponentes.
r2 Para elevar una potencia a otra potencia,
(ar1 ) = ar1 r2
se multiplican los exponentes.
r La potencia de un producto es igual al
(ab) = ar br
producto de potencias.
ar1 Para dividir potencias de una misma base
= ar1 r2
ar2 se restan los exponentes.
a r ar La potencia de una razn es igual a la ra-
=
b br zn de sus potencias.
a 0 La potencia nula de una razn o fraccin
= 1, a 6= 0, b 6= 0
b es igual a 1.
220 CAPTULO 6. IGUALDADES Y DESIGUALDADES NUMRICAS

Teorema 14. Si a > b > 0 y n 2 N, entonces an > bn .

Demostracin: El enunciado no es ms que la generalizacin de la consecuencia del teorema 13, en donde


se pone ai = a, bi = b, (i = 1; 2; :::; n).

Ejemplos:

1. 3 > 2, entonces con n = 5, tendremos que 35 > 25 :


4 4
3 7 3 7
2. < , entonces con n = 4, tendremos que < :
4 6 4 6

Teorema 15. Si a < b < 0 y n 2 N, entonces para n par an > bn , y para n impar an < bn :
n n
Demostracin: De a < b se tiene que a > b (teorema 12). Como a > 0 y b > 0, entonces ( a) > ( b)
n n
(teorema 14). Esta ltima desigualdad la podemos escribir en la forma ( 1) an > ( 1) bn . Si n es par
n n
( 1) = 1 y por tanto es verdad que an > bn . Si n es impar ( 1) = 1 es verdad que an > bn , y por
tanto an < bn .

Ejemplos:

1. Elevar al cuadrado y a la 5ta potencia la desigualdad 2> 3:


2 2 5 5
Solucin: Como 2<0y 3 < 0, tendremos que ( 2) < ( 3) y ( 2) > ( 3) :

2. Elevar al cuadrado la desigualdad a > b, suponiendo que ab > 0:


Solucin: Como ab > 0, entonces o a > 0 y b > 0, o a < 0 y b < 0 (proposicin 2, teoremas 2 y3). Para
a > 0 y b > 0 tenemos a2 > b2 . Para a < 0 y b < 0 tenemos a2 < b2 (teoremas 14, 15).

p p
Teorema 16. Si a > b > 0 y n 2 N, entonces n a > n b (para n par, solamente se toma la raz
aritmtica).
p p
Demostracin: Analicemos los 2 nmeros n a y n b. Solo una de las 3 siguientes expresiones es verdadera:
p p p p p p p p
n
a > n b, o n a = n b, o n a < n b. Si n a < n b es verdadera, entonces tambin lo es la desigualdad a < b
(teorema 14), lo que contradice la condicin del teorema. Anlogamente se demuestra la falsedad de la igualdad
p p p p
n
a = n b. Por lo tanto, es verdadera la desigualdad n a > n b.

Se concluye que, en una desigualdad con trminos positivos es posible extraer la raz n-sima (n 2 N) a
ambos lados, mantenindose el sentido de la desigualdad.

Teorema 17. Si a > b > 0 y r = p=q es un nmero racional positivo (r = p=q 2 Q+ ), entonces ar > br :

Demostracin: Tenemos que r = p=q con p; q 2 N. Utilizando sucesivamente los teoremas 14 y 16,
p p
tendremos que ap > bp , q ap > q bp , es decir, ap=q > bp=q .

En conclusin, una desigualdad con trminos positivos es posible elevar a una potencia racional r positiva
(r 2 Q+ ), mantenindose el sentido de la desigualdad.

Ejemplo:
2 2
De la desigualdad verdadera 6 > 5 se tiene que 6 3 > 5 3 , tambin es verdadera.
r r
Teorema 18. Si a > b > 0 y r = p=q es un nmero racional positivo (r = p=q 2 Q+ ), entonces a <b :

Demostracin: Aplicando sucesivamente la consecuencia del teorema 12 y el teorema 17, tenemos que
p p
1 1 1 q 1 q r r
a <b , luego a < b , es decir a <b .
6.3. PRODUCTO DE DESIGUALDADES NUMRICAS 221

Figura 6.6: Izq. El gran matemtico y gemetra griego, Euclides (cerca de 300 aos A.C.), cuyos estudios geomtricos se
han mantenido inclumes durante varios miles de aos. Sistematiz todos los conocimientos geomtricos de la antiguedad
y desarroll otros (demostr muchsimos teoremas) en sus Elementos (13 libros) que tuvieron y tienen una inuencia
enorme hasta nuestros das. Der. Leonhard Euler (Basilea, 1707 - San Petersburgo, 1783) el maestro de todos los
matemticos del siglo XVII. Estudi con Johannes Bernoulli y luego le sucedi a Daniel Bernoulli en la ctedra de San
Peterburgo. Aparte de hacer gigantescas contribuciones al Clculo, Algebra y Teora de Nmeros se le considera el padre
de la teora de grafos y de la topologa (estudio de los poliedros).

En conclusin, una desigualdad con trminos positivos es posible elevar a una potencia racional r negativa
(r 2 Q ), cambindose el sentido de la desigualdad.

Ejemplo:
5 5
De la desigualdad verdadera 8 > 5, se sigue que 8 3 <5 3 tambin es verdadera.

6.3.2. Ejemplos de algunas desigualdades numricas clsicas

Recuerda las siguientes desigualdades muy tiles y conocidas:

Ejemplos:

1. Recordemos el concepto de valor absoluto jaj del nmero real a y sus propiedades:
(
a; si a 0
jaj =
a; si a < 0

p
La denicin anterior de valor absoluto de a 2 R equivale a poner a2 = jaj. Se debe tener en cuenta,
adems que jaj a y a jaj por ser el nmero jaj siempre positivo o nulo, es decir, para a; b 2 R, se
cumplen las relaciones jaj a jaj, jbj b jbj
Sumando las 2 desigualdades anteriores, tendremos que

jaj jbj a+b jaj + jbj ) (jaj + jbj) a+b jaj + jbj ;

es decir, para a, b 2 R se cumple la desigualdad j a + b j j a j + j b j, es decir, el valor absoluto de la


suma de dos nmeros reales no supera la suma de valores absolutos de estos nmeros. Esta desigualdad se
denomina triangular. Mediante el mtodo de induccin matemtica se puede demostrar que para cualquier
Pn Pn
nmero nito de sumandos ai es vlida la desigualdad ms general ai jai j. La igualdad se
i=1 i=1
obtiene si todos los sumandos son nmeros del mismo signo.
222 CAPTULO 6. IGUALDADES Y DESIGUALDADES NUMRICAS

2. Para cualesquiera a, b 2 R se cumple la desigualdad ja bj jjaj jbjj, es decir, el valor absoluto de la


diferencia de 2 nmeros no es menor que el valor absoluto de la diferencia de los valores absolutos de estos
nmeros. Esto se desprende de la desigualdad triangular anterior:

jaj = j(a b) + bj ja bj + jbj ) jaj jbj ja bj ;

anlogamente, intercambiando a y b en jaj jbj ja bj, tendremos que

jbj jaj jb aj = j (a b)j = j 1j ja bj = ja bj ;

de donde ja bj < jaj jbj. Obtenemos nalmente el resultado inicial

(jaj jbj ja bj ^ jaj jbj > ja bj) ) jjaj jbjj ja bj

3. Para cualesquiera nmeros reales a y b se cumple la desigualdad a2 + b2 2 jabj.


La igualdad se obtiene cuando y slo cuando jaj = jbj. Pues
2
(a b) 0 ) a2 2ab + b2 0 ) a2 + b2 2ab ) a2 + b2 2 jabj
a b
4. Si a y b son nmeros reales del mismo signo (ab > 0), entonces + 2.
b a
La igualdad se obtiene cuando y solo cuando a = b. Anlogamente, se tiene
2 a2 b2 a b
(a b) 0 ) a2 + b2 2ab ) + 2) + 2
ab ab b a
a+b p
5. Desigualdad de Cauchy: si a, b 2 R+ , entonces ab, es decir, la media aritmtica de dos nmeros
2
no negativos no es menor que su media geomtrica. Esto sigue de las relaciones
2 a+b p
a2 + b2 2ab ) a2 + 2ab + b2 4ab ) (a + b) 4ab ) ab
2
La desigualdad anloga, demostrable por induccin, es vlida tambin para cualquier lista nita de nmeros
no negativos ai (i = 1; 2; 3; ::::; n) :
a1 + a2 + a3 + : : : + an p
n
a1 a2 a3 : : : an ;
n
es decir, la media aritmtica de n nmeros no negativos es mayor o igual a su media geomtrica. La
igualdad se obtiene, cuando todos los n nmeros son iguales.

6. Desigualdad de Cauchy-Buniakovski-Schwartz: para cualesquiera nmeros reales a1 , a2 , a3 ,:::, an , b1 , b2 ,. . . ,


bn se cumple la desigualdad

(a1 b1 + a2 b2 + : : : + an bn )2 (a21 + a22 + : : : + a2n )(b21 + b22 + : : : + b2n ):

La igualdad se cumple cuando y solo cuando los nmeros ai y bi son proporcionales, es decir, cuando
existen tales nmeros y , 2 + 2 6= 0, tales que para todo i = 1, 2, :::, n se cumple la igualdad
ai + bi = 0:

7. Relacin entre la medias aritmtica y cuadrtica de varios nmeros: el valor absoluto de la media aritmtica
de cualquier nmero nito de nmeros reales no supera a la media cuadrtica de estos nmeros es decir,
r
ja1 + a2 + : : : + an j a21 + a22 + : : : + a2n
;
n n
La igualdad se cumple slo cuando todos los n nmeros son iguales entre s.

8. Desigualdad de Holder (generalizacin de la desigualdad de Cauchy-Buniakovski): para cualesquiera nmeros


reales a1 , a2 , a3 ,: : :, an ; b1 , b2 ,: : :, bn con cualquier p > 1, se cumple la desigualdad
1 1
p p p q q q
ja1 b1 +a2 b2 + : : : + an bn j ( ja1 j + ja2 j + : : : + jan j ) p ( jb1 j + jb2 j + : : : + jbn j ) q ;

p 1 1
donde q = , o lo que es lo mismo + = 1:
p 1 p q
Captulo 7

ECUACIONES E INECUACIONES

Concibiendo el innito como lo he hecho, siento un verdadero placer y me abandono a l con agradeci-

miento... Y si vuelvo a bajar hacia lo nito, veo con una claridad y una belleza iguales los dos conceptos

(el ordinal y el cardinal) que de nuevo slo son uno y convergen en el concepto de nmero entero nito.

Georg Cantor

La idea que cada uno nos hacemos del nmero uno, sea cual fuere, debe ser cuidadosamente diferen-

ciada del nmero uno, como las ideas de la Luna deben ser diferenciadas de la propia Luna.

Gottlob Frege

Unidad de competencia: Resolver correctamente las ecuaciones y desigualdades numricas y algebraicas de


distintos tipos, introduciendo el concepto de conjunto de valores admisibles (c.v.a.).

Indicadores de logro:

| Identicar las propiedades de las ecuaciones e inecuaciones y sus sistemas y aplicarlas en su resolucin.

| Aprender a calcular el conjunto de valores admisibles (dominios) de una ecuacin, inecuacin o sistema.

| Aplicar las propiedades de las igualdades, ecuaciones e inecuaciones en diferentes demostraciones de teoremas.

| Resolver ejercicios y problemas sobre ecuaciones lineales y sistemas de ecuaciones lineales.

| Comprender el proceso de anlisis de una ecuacin e inecuacin y las caractersticas de sus races y soluciones.

| Traducir problemas expresados en lenguage comn a representaciones matemticas.

| Utilizar recursos analticos en el planteo y resolucin de problemas cotidianos.

| Utilizar convenientemente el mtodo de intervalos en la resolucin de inecuaciones racionales.

| Resolver algunas inecuaciones irracionales resducibles a inecuaciones lineales.

| Generar, ampliar y modicar procedimientos en la solucin de ecuaciones e inecuaciones con valor absoluto.

Cul es el equivalente de 100, si 5 6 = 33?

223
224 CAPTULO 7. ECUACIONES E INECUACIONES

Pasamos a estudiar las propiedades de los nmeros reales y sus aplicaciones.

7.1. Conceptos Generales

Supongamos que las expresiones f (x1 ; x2 ; :::xn ) y g(x1 ; x2 ; :::xn ) son relaciones o funciones reales de las
variables reales x1 , x2 , : : :, xn 2 R. Por ejemplo, f (x1 ) = 2x1 + 1, f (x1 ; x2 ) = x1 2x2 , g (x1 ; x2 ; x3 ) =
x1 x22 x3 2, etc.

Denicin. Los predicados 1 respecto de las variables x1 , x2 , : : :, xn , que poseen la forma f (x1 ; x2 ; : : : ; xn ) 7
g(x1 ; x2 ; : : : ; xn ), y se satisfacen para ciertos valores de las variables, se denominan inecuaciones. La igualdad
f (x1 ; x2 ; : : : ; xn ) = g(x1 ; x2 ; : : : ; xn ) que se satisface slo para ciertos valores de las variables, se denomina
ecuacin. Anlogamente se dene una desigualdad no estricta con los smbolos o .

Ejemplos:

1. 3x + 4y > x 2 es una inecuacin polinmica de 1er grado de 2 variables.

2. 3x 1=3 > x=4 + 2 es una inecuacin polinmica de 1er grado de 1 variable.


x
3. p 2x = 3, x 6= 1 es una ecuacin irracional de 1 variable.
x+1
4. x2 + 1 < 5x es una inecuacin polinmica de 2do grado de 1 variable.
p
5. 4 3 + 2x > x 1 es una inecuacin irracional de 1 variable.

6. x3 2=y 2x es una ecuacin polinmica de 3er grado de 2 variables.


3 x
7. x = 9, x 6= 1 es una ecuacin racional de 1 variable.
x+1
3 p
8. x = x + 1, x 6= 1 es una ecuacin irracional de 1 variable.
x 1
2x y
9. 1 es una inecuacin racional no estricta de 2 variables.
x + 3y
2x y 3z
10. = 4 es una ecuacin racional de 3 variables.
z 2x y

| Las expresiones o funciones colocadas a cada uno de los lados de una desigualdad o ecuacin se denominan
miembros o componentes. As, para 3x2 x 7 5x + a, 3x2 x es el miembro izquierdo y 5x + a es el derecho.

| Las inecuaciones pueden ser analizadas como desigualdades numricas con respecto a sus variables x1 ,
x2 ,: : :, xn 2 R, convirtindose sus miembros en constantes para ciertos valores reales jos de x1 , x2 , : : :, xn .
As, por ejemplo, la inecuacin 5x y > x2 + 1 se convierte en desigualdad numrica verdadera para x = 1 e
2
y = 1, pues 5 (1) ( 1) > (1) + 1; y se convierte en desigualdad numrica falsa para x = 3 e y = 1, pues
16 10.

| Recordamos que una funcin real f : A ! B es una correspondencia entre los elementos de un conjunto
A R y los elementos del conjunto B R, de tal manera que a todos y cada uno de los elementos x 2 A se
le pone en correspondencia un nico elemento y 2 B. Generalmente, para especicar una funcin, aparte de la
correspondencia, tambin se indica su dominio de denicin:

Dom f = fx 2 A : 9y 2 B con y = f (x)g :


1 Predicado es una expresin cuyo valor de verdad (verdadero o falso) depende de los valores que tomen las variables x1 ; x2 ; : : : ; xn :
7.1. CONCEPTOS GENERALES 225

Una funcin f : Am ! B de varias variables es una correspondencia entre los elementos de un conjunto
Am Rm y los elementos del conjunto B R, de tal manera que a todos y cada uno de los elementos
(x1 ; x2 ; : : : ; xm ) 2 Am se le pone en correspondencia un nico elemento z 2 B, respectivamente. Esta funcin
se la representa mediante z = f (x1 ; x2 ; : : : ; xm ). As, por ejemplo, z = 2x1 x22 es una funcin del tipo
p
z = f (x1 ; x2 ), denida para todo (x1 ; x2 ) 2 A2 = R2 . La funcin z = 3 x y t es del tipo z = f (x; y; t),
denida para aquellos (x; y; t) 2 A3 R3 que cumplen con las relaciones x y 0 y t 2 R. En general, el
dominio de z = f (x1 ; x2 ; : : : ; xm ) viene dado mediante

Dom f = f(x1 ; x2 ; : : : ; xm ) 2 A Rm : 9z 2 B R con z = f (x1 ; x2 ; : : : ; xm )g

Ejemplos:

1. y = f (x) = cos 3x, Dom f =] 1; +1[= R


p p
2. y = f (x) = 2 3x 6 5 x + 3, Dom f = fx 2 R : (3x 6 0) ^ (x + 3 0)g =
= fx 2 R : (x 2) ^ (x 3)g = fx 2 R : x 2g = [2; +1[
x
3. y = f (x) = , Dom f = fx 2 R : x 6= 2g = Rn f 2g
x+2
p
3 x 2
4. y = f (x) = 2
p , Dom f = fx 2 R : (x 0) ^ (5 2x > 0) ^ (x 6= 1)g =
1 x 5 2x
= fx 2 R : (x 0) ^ (x < 5=2) ^ (x 6= 1)g = fx 2 [0; 5=2[n f1gg

5. y = f (x) = xx , Dom f =]0; +1[= R+


p
5 x
6. y = p , Dom f = fx 2 R : (5 x 0) ^ (x 6 > 0)g = fx 2 R : (x 5) ^ (x > 6)g = ;
x 6
p 3 3
7. y = 2 6 3 2x, Dom f = fx 2 R : 3 2x 0g = x2R:x =] 1; ]
2 2
p
8. z = 3 x21 4x22 , Dom f = (x1 ; x2 ) 2 R2 : x21 4x22 0 = (x1 ; x2 ) 2 R2 : jx1 j 2 jx2 j

x2 + 3y
9. z = f (x; y) = , Dom f = (x; y) 2 R2 : x 2y 6= 0 = (x; y) 2 R2 : x 6= 2y
x 2y

10. z = 3 arc cos (x1 2x2 ), Dom f = (x1 ; x2 ) 2 R2 : jx1 2x2 j 1 =


= (x1 ; x2 ) 2 R2 : 1 x1 2x2 1 = (x1 ; x2 ) 2 R2 : 1 + 2x2 x1 1 + 2x2

Denicin 10. Se denomina dominio de denicin o conjunto de valores admisibles c:v:a: de una desigual-
dad o ecuacin al conjunto interseccin de los dominios de denicin de sus componentes o miembros izquierdo
y derecho.

Ejemplos:

1. El dominio de denicin de la ecuacin 5x = x2 + log 2x viene dado por la interseccin de los dominios
de denicin de sus miembros izquierdo y derecho. As, 5x est denido para todo x 2 R, y x2 + log 2x
est denido para x 2 R+ . Por lo tanto, la interseccin de estos dominios es el c:v:a: = fx 2 R+ g :
p
2. El dominio de denicin de la desigualdad 2x > 4 3 x viene dado por la interseccin de los dominios
de denicin de sus trminos o componentes. As, la componente 2x est denida para x 2 R, y la
p
componente 4 3 x est denida para x 3. Por lo tanto, el c:v:a: es fx 2 Rg \ fx 3g =] 1; 3]:
226 CAPTULO 7. ECUACIONES E INECUACIONES

Observacin: Si en la relacin del tipo f (x1 ; x2 ; : : : ; xn ) ? g(x1 ; x2 ; : : : ; xn ) en lugar de las variables


x1 , x2 , : : :, xn colocamos ciertos valores reales jos, pertenecientes al conjunto de valores admisibles c:v:a: o
dominio de denicin
((x1 ; x2 ; : : : ; xn ) 2 Dom f \ Dom g) ;

entonces sta se transforma en una proposicin (relacin) numrica, verdadera o falsa.

Ejemplos:

1. La desigualdad 2x y > x2 + xy + y 2 est denida para todos los valores reales x, y 2 R. Al colocar los
valores x = 1, y = 1, obtenemos la desigualdad verdadera 3 > 1. Al colocar x = 1, y = 1, obtenemos
la desigualdad falsa 1 > 3.

2. La ecuacin 2 log x = x2 98 est denida para x 2 R+ . Al colocar x = 10 obtenemos una igualdad


verdadera 2 log 10 = 100 98. Si colocamos el valor x = 2, obtenemos una igualdad numrica falsa
2 log 2 = 4 98:

Denicin 11. El conjunto de valores de las variables, tomado del dominio de denicin (c:v:a:) de la
desigualdad (ecuacin), tal que la convierte en una desigualdad (igualdad) numrica verdadera, se denomina
solucin de la desigualdad (ecuacin).

Ejemplos:

1. Los valores x = 1; y = 0 pertenecen al c:v:a: (dominio de denicin) de la desigualdad x2 + 5xy + y 2 <


2x + 3y y la convierten en la desigualdad verdadera 2 > 1, por lo tanto son solucin (no nica!) de la
desigualdad.

2. El valor x = 10 pertenece al c:v:a: de la ecuacin x2 98 = 2 log x, y la convierte en una igualdad numrica


verdadera, por tanto es solucin de la ecuacin. El valor x = 2, tambin pertenece al c:v:a: de la ecuacin,
pero sin embargo no la convierte en una igualdad numrica verdadera, por lo tanto x = 2 no es solucin
de la ecuacin.

Si cierto conjunto de valores de las variables son solucin de la desigualdad (ecuacin), entonces se dice
que este conjunto numrico satisface la desigualdad (ecuacin).

Las soluciones de una ecuacin, generalmente, se denominan races de la ecuacin.

Resolver una desigualdad (ecuacin) signica encontrar el conjunto de todas sus soluciones (races).

Denicin 12. Una desigualdad (ecuacin), se denomina identicamente verdadera, si para todos los valores
admisibles de las variables, esta se convierte en una desigualdad (igualdad) numrica verdadera. Si no existen
valores de las variables que conviertan a la desigualdad (ecuacin) en una desigualdad (igualdad) verdadera,
entonces esta se denomina identicamente falsa.

Ejemplos:

1. La desigualdad 2x2 + 3 > 0 es identicamente verdadera, la desigualdad 3x4 + 1 < 0 es identicamente falsa.

2. La desigualdad 1 sin x 1 es identicamente verdadera, la desigualdad cos x > 3 es identicamente


falsa.
1
3. La ecuacin x 2y = (3x 6y) es identicamente verdadera, la ecuacin 3x2 + y 2 = 4 es identicamente
3
falsa.
7.2. EQUIVALENCIA ENTRE DESIGUALDADES 227

Figura 7.1: Izq. El lsofo, gemetra (ensayo sobre las cnicas), fsico (Principio de Pascal, hidralica) y uno de los
padres del Clculo, de la teora de las probabilidades, combinatoria, teora de juegos y geometra proyectiva. Construy
la primera calculadora. Estudioso y a la vez detractor de algunas corrientes del cristianismo y tambin de la matemtica,
el francs Blaise Pascal (Clermont - Ferrand,1623 - Pars, 1662). Der. El elegido de los Dioses y padre del lgebra
moderna, el revolucionario francs, Evariste Galois (Bourg la Reine, 1811 - Paris, 1832). Abord el problema de la
resolucin general de las ecuaciones polinmicas y la insolubilidad de la quntica, con esto descubri la teora de grupos.
Fue muerto en un duelo, pero unos das antes de morir escribi su famosa carta a A. Chevalley, donde indica las condiciones
necesarias y sucientes para que una ecuacin algebraica de grado n 5 sea soluble por radicales.

2 6
4. La desigualdad jx + yj x2 y 4 es identicamente verdadera, las desigualdades (x 2) + (y + 3) < 0
y e x < 0 son identicamente falsas.
p
5. La ecuacin x2 4xy + 4y 2 = jx 2yj es identicamente verdadera, las ecuaciones 2x = 1 y log jxj =
x2 + 5 son identicamente falsas.

Denicin 13. Una ecuacin identicamente verdadera cuyos miembros o componentes poseen los mismos
conjuntos de valores admisibles c:v:a:, se denomina identidad. En este caso se dice que sus trminos o com-
ponentes son identicamente iguales. El cambio de una expresin por otra identicamente igual, se denomina
transformacin idntica.

Ejemplos:

1. La transformacin del tipo log x2 = 2 log x no es una transformacin idntica, pues el dominio de denicin
es distinto. Para log x2 el dominio es x 2 R= f0g, y para 2 log x, el dominio es x 2 R+ =]0; +1[.

2. La transformacin del tipo log x2 = 2 log jxj es una transformacin idntica, pues el dominio de denicin
de ambos trminos coincide, siendo x 2 R= f0g.
p
3. La transformacin del tipo x2 = jxj es una transformacin idntica, pues el dominio de denicin
coincide, siendo x 2 R.

7.2. Equivalencia entre Desigualdades

Al analizar las desigualdades aparecen dos problemas fundamentales, que consisten en:

1. Resolver una desigualdad, es decir, encontrar el conjunto de sus soluciones.

2. Demostrar la desigualdad, es decir, establecer si sta es identicamente verdadera o falsa. Este problema
est totalmente enlazado con el primero.
228 CAPTULO 7. ECUACIONES E INECUACIONES

| Para resolver los problemas anteriores se utilizan los conceptos de equivalencia entre desigualdades.

Denicin 14. Dos desigualdades se denominan equivalentes, si cualquier solucin de la primera es solucin
de la segunda y viceversa.

Para armar que las desigualdades

f1 (x1 ; x2 ; :::; xn ) 7 g1 (x1 ; x2 ; :::; xn ) y f2 (x1 ; x2 ; :::; xn ) 7 g2 (x1 ; x2 ; :::; xn )

son equivalentes, utilizaremos el simbolismo

f1 (x1 ; x2 ; :::; xn ) 7 g1 (x1 ; x2 ; :::; xn ) , f2 (x1 ; x2 ; :::; xn ) 7 g2 (x1 ; x2 ; :::; xn )

| De la denicin 14 se desprende que si el conjunto solucin de una de las desigualdades coincide con el
conjunto solucin de la segunda, entonces las desigualdades son equivalentes.

Ejemplos:

1. Las desigualdades 4x 2 > 50x y 2x 1 > 25x son equivalentes. Pues, si x = a es cualquier
solucin de la 1ra desigualdad, entonces 4a 2 > 50a es una desigualdad numrica verdadera. Ahora,
por el teorema 12, obtenemos la desigualdad numrica verdadera 2a 1 > 25a (hemos dividido la anterior
desigualdad por 2). Si comparamos sta ltima desigualdad con la 2da , observaremos que x = a convierte
a la 2da desigualdad en una desigualdad verdadera, es decir es su solucin. Anlogamente se demuestra la
proposicin inversa.

2. Las desigualdades 4x2 + 3 < 0, x2 + 5 < 0 y x2 x + 1 < 0 son equivalentes. Las 3 son identicamente
falsas y no poseen solucin, es decir el conjunto solucin es el conjunto vaco o ;.

| Formulamos los principales teoremas sobre la equivalencia entre desigualdades:

Teorema 19. Las desigualdades

f (x1 ; x2 ; :::; xn ) 7 g(x1 ; x2 ; :::; xn ) y (a)

f (x1 ; x2 ; :::; xn ) + h(x1 ; x2 ; :::; xn ) 7 g(x1 ; x2 ; :::; xn ) + h(x1 ; x2 ; :::; xn ) (b)

son equivalentes, si stas poseen el mismo conjunto de denicin.

Demostracin: Demostremos que cualquier solucin de la desigualdad (a) es solucin de la desigualdad (b).
Sea x1 = a1 , x2 = a2 , . . . , xn = an , cualquier solucin de (a). Entonces f (a1 ; a2 ; : : : ; an ) 7 g(a1 ; a2 ; : : : ; an ) es
una desigualdad numrica verdadera. Sumamos a cada uno de los trminos el valor h(a1 ; a2 ; : : : ; an ), por las
condiciones, ste valor existe, pues las desigualdades (a) y (b) poseen el mismo dominio de denicin. Por el
teorema 9, conclumos que la desigualdad numrica

f (a1 ; a2 ; : : : ; an ) + h (a1 ; a2 ; : : : ; an ) 7 g(a1 ; a2 ; : : : ; an ) + h(a1 ; a2 ; : : : ; an )

es verdadera. Por lo tanto cualquier solucin de la desigualdad (a) es solucin de (b).

A la inversa, sean x1 = b1 , x2 = b2 , . . . , xn = bn , cualquier solucin de (a), entonces

f (b1 ; b2 ; : : : ; bn )+h(b1 ; b2 ; : : : ; bn ) 7 g(b1 ; b2 ; : : : bn )+h(b1 ; b2 ; : : : ; bn ) es una desigualdad numrica verdadera.

Restando de los trminos de esta ltima desigualdad el valor h(b1 ; b2 ; : : : ; bn ) (teorema 9), obtenemos
f (b1 ; b2 ; : : : ; bn ) 7 g(b1 ; b2 ; : : : ; bn ) que es una desigualdad numrica verdadera.

Por tanto, cualquier solucin de la desigualdad (a), tambin lo es de la (b), y viceversa.


7.2. EQUIVALENCIA ENTRE DESIGUALDADES 229

Consecuencia. Cualquier trmino de una desigualdad puede traspasarse de un miembro al otro, pero con
signo contrario. En particular, las desigualdades

f (x1 ; x2 ; :::; xn ) 7 g(x1 ; x2 ; :::; xn ) y f (x1 ; x2 ; :::; xn ) g(x1 ; x2 ; :::; xn ) 7 0

son equivalentes.

Ejemplos:

1. Las desigualdades 5x + y + x2 > 3 + x2 y 5x + y > 3 son equivalentes, pues esta ltima se obtiene de la
primera, restando x2 de ambos miembros, o 5x + y + x2 x2 > 3 ) 5x + y > 3. Adems, los dominios
de denicin de ambas desigualdades coinciden, siendo ste el conjunto R.
p p
2. Las desigualdades x x 1 x y x 1 no son equivalentes en R. El dominio de denicin de la
1 desigualdad es R+ [f0g = [0; +1[, el dominio de la 2da desigualdad es todo R. Las 2 desigualdades
ra

son equivalentes en R+ [f0g = [0; +1[:


p p p p
3. La desigualdad 3x + 1 x 9 + 1 x es equivalente a la desigualdad 3x + 1 x 1 x 9
(consecuencia del teorema 19). Al simplicar trminos semejantes en la ltima desigualdad, obtendremos
p p
la desigualdad 3x 9, que equivale a x 3. Las desigualdades 3x + 1 x 9 + 1 x y 3x 9 no
son equivalentes, pues el dominio de denicin de la 1ra es x 2] 1; 1], el dominio de la 2da es todo R.
Como se observa, el dominio de denicin qued ampliado.
p p
4. La ecuacin 5x x = 10 x y la ecuacin x = 2 no son equivalentes en todo R. El dominio
de denicin de la 1ra ecuacin [0; +1[, y el dominio de la 2da ecuacin es todo R. Sin embargo, las 2
ecuaciones son equivalentes en R+ [f0g = [0; +1[:
Observacin. Al traspasar uno o varios trminos de un miembro a otro en una desigualdad (ecuacin),
la equivalencia se conserva, pero cuando se simplican los trminos semejantes, la equivalencia puede
alterarse, dando lugar a una posible ampliacin del dominio de denicin de la inecuacin (ecuacin).
De aqu la necesidad de siempre comprobar que las soluciones pertenezcan al dominio de denicin de la
desigualdad (ecuacin) original.
p p p p
5. La desigualdad x + x 4 > x 4 + 3 es equivalente a la desigualdad x + x 4 x 4 > 3
(consecuencia del teorema 19). La simplicacin de trminos nos conduce a la desigualdad x > 3. El
dominio o conjunto de valores admisibles, c:v:a: se ampli, y la equivalencia fue violada. En otros casos
la equivalencia puede ser no violada.
p p p
6. La desigualdad 2x 1 + 3 x2 + 1 3 x2 + 1 + 3 es equivalente a la desigualdad 2x + 3 x2 + 1
p
3 x2 + 1 4, siendo el c:v:a: = fx 2 Rg (consecuencia del teorema 19). La simplicacin de trminos
nos conduce a la desigualdad x 2. En este caso, el conjunto de valores admisibles, c:v:a: permanece sin
alteracin.

| Es importante entender que cuando se aplica la consecuencia del teorema 19 y adems se simplican
trminos semejantes, es necesario comprobar si el el c.v.a. sigue o no igual, y si ha cambiado, establecer si la
solucin est o no contenida el c.v.a.

Teorema 20. Las desigualdades

f (x1 ; x2 ; : : : ; xn ) 7 g(x1 ; x2 ; : : : ; xn ) (c) y


f (x1 ; x2 ; : : : ; xn )w(x1 ; x2 ; : : : ; xn ) 7 g(x1 ; x2 ; : : : ; xn )w(x1 ; x2 ; : : : ; xn ) (d)

son equivalentes si poseen el mismo c.v.a. y la desigualdad w(x1 ; x2 ; : : : ; xn ) > 0 en este c.v.a. es idnticamente
verdadera.
230 CAPTULO 7. ECUACIONES E INECUACIONES

Demostracin: Sean x1 = a1 , x2 = a2 , :::, xn = an una solucin cualquiera de (c), entonces f (a1 ; a2 ; : : : ; an ) 7


g(a1 ; a2 ; : : : ; an ) es una desigualdad numrica verdadera. Multiplicando esta desigualdad por el nmero pos-
itivo (segn la condicin del teorema) w(a1 ; a2 ; : : : ; an ), obtendremos una desigualdad numrica verdadera
(teorema 12)
f (a1 ; a2 ; : : : ; an )w(a1 ; a2 ; : : : ; an ) 7 g(a1 ; a2 ; : : : ; an )w(a1 ; a2 ; : : : ; an ):

Por tanto, los valores de las variables x1 = a1 , x2 = a2 , :::, xn = an son soluciones tambin de la desigualdad (d).
Anlogamente se demuestra la proposicin inversa de que cualquier solucin de la desigualdad (d) es solucin
de la desigualdad (c).

El siguiente teorema se demuestra en forma anloga al teorema (20).

Teorema 21. Las desigualdades

f (x1 ; x2 ; : : : ; xn ) > g(x1 ; x2 ; : : : ; xn ) (c) y


f (x1 ; x2 ; : : : ; xn ):w(x1 ; x2 ; : : : ; xn ) < g(x1 ; x2 ; : : : ; xn ):w(x1 ; x2 ; : : : ; xn ) (d)

son equivalentes si poseen el mismo c:v:a: y la desigualdad w(x1 ; x2 ; x3 ; ::::; xn ) < 0 en este c:v:a: es
idnticamente verdadera.

Las observaciones hechas con respecto al teorema 19, tienen lugar para los teoremas 20 y 21.

Ejemplos:

1. Las desigualdades (log x 2)x > 5x y log x 2 > 5 son equivalentes, pues su c.v.a. es para ambas
fx > 0g, y en este conjunto la desigualdad x > 0 es identicamente verdadera.
p p
2 p p 2 x x
2. Las desigualdades 1 x<x x y 1 p < x p son equivalentes (Teorema 20), siendo
x x x x
2
el c:v:a: fx > 0g pero la primera no es equivalente a la desigualdad 1 < x cuyo c:v:a: es fx 6= 0g. Ha
x
sucedido una ampliacin del c:v:a: de fx > 0g = R+ a fx 6= 0g = R= f0g. Por lo tanto puede haberse
2 2 p p
violado la equivalencia, as x = 1 es solucin de 1 < x, pero no de 1 x < x x:
x x

Teorema 22. Las desigualdades

f (x1 ; x2 ; : : : ; xn )
f (x1 ; x2 ; : : : ; xn ):g(x1 ; x2 ; : : : ; xn ) 7 0 y 70
g(x1 ; x2 ; : : : ; xn )

son equivalentes.

La demostracin se sigue de que si para ciertos valores de las variables la expresin f (x1 ; x2 ; : : : ; xn ) y
g(x1 ; x2 ; : : : ; xn ) son de distintos signos (caso <) (son de iguales signos (caso >)), entonces el producto f:g y
f
el cociente sern negativos (positivos) para estos valores de las variables.
g

Ejemplos:

x 3 5
1. La desigualdad > 0 equivale a la desigualdad (x 3) (2x + 5) > 0, x 6= . La solucin de
2x + 5 2
5
ambas es x < 2 [ fx > 3g.

3x + 1
2. La desigualdad < 0 equivale a la desigualdad (3x + 1) (x 4) < 0, x 6= 4. La solucin para ambas
x 4
1
es 3 <x<4
7.3. ECUACIN LINEAL O DE PRIMER GRADO 231

x (x + 12)
3. La desigualdad 0 equivale a la desigualdad x (x + 12) (x + 2) (x 3) 0, x 6= 2, 3.
(x + 2) (x 3)
La solucin para ambas es fx 12g [ f 2 < x 0g [ fx > 3g.
5x 3
4. La desigualdad 0 equivale a la desigualdad 5x x2 + 2 (2x 3) 0, x 6= . La
(x2 + 2) (2x 3) 2
solucin para ambas es 0 x < 23 .

7.3. Ecuacin Lineal o de Primer Grado

Una ecuacin lineal o de primer grado es aquella ecuacin f = 0, que despus de ser reducida y simplicada,
toma la forma ax + b = 0, a 6= 0.

La solucin de una ecuacin lineal se obtiene mediante las siguientes operaciones:


b
ax + b = 0; a 6= 0 ) ax = b; a 6= 0 ) x = ; a 6= 0
a
Partiendo de la solucin anterior y analizando todos los valores reales posibles que pueden tomar a y b, tendremos
el resumen: 8
> b
>
< a ; si a 6= 0
x= R; si a = 0 ^ b = 0
>
>
:
;; si a = 0 ^ b 6= 0
5
As, por ejemplo, para la ecuacin lineal 2x 5 = 0, con a = 2, tendremos la solucin nica x = . Para la
2
ecuacin lineal 0x + 0 = 0 con a = 0 y b = 0, se tiene que cualquier nmero real x 2 R es su solucin. Para
la ecuacin 0x + 3 = 0 con a = 0 y b = 3, vemos que no existen soluciones.

| El nombre de ecuacin lineal se debe a que la representacin geomtrica en el plano cartesiano de la


relacin y = ax + b, viene dada por una lnea recta de pendiente o inclinacin a (tan = a y de ordenada al
origen b) con respecto a la horizontal o eje Ox:

Recta y = ax + b, donde a es la pendiente (inclinacin de la recta, tan = a) y b es la ordenada al origen.

| Para resolver una ecuacin lineal o reducible a lineal, necesitamos simplicar los trminos semejantes,
hacer comn denominador y agrupar los trminos que contienen a la incgnita x, y los trminos independientes,
luego llegar a la forma ax + b = 0 que nos da la solucin buscada.

| Antes de reducir o simplicar una ecuacin, siempre es necesario buscar el conjunto de valores admisibles
de la ecuacin (c:v:a) y de sus trminos. Este conjunto est compuesto de todos aquellos valores reales que hacen
posible la existencia de la ecuacin original como de sus soluciones, es decir, son los valores admisibles en el
campo posible de variacin de las incgnitas o valores literales que intervienen en la ecuacin. As, por ejemplo,
3
para la ecuacin = 8, tendremos que c:v:a: = fx 2 R : x 6= 2g. Para la ecuacin (a 3) x 4 = 0,
x+2
tendremos que el c:v:a: = fa 2 R : a 6= 3g :
232 CAPTULO 7. ECUACIONES E INECUACIONES

Ejemplos:

Resolver las siguientes ecuaciones:

3x 2 x 1 x
1. =3
4 3 2
5 1 1 5
Solucin: c:v:a: = fx 2 Rg. Simplicando tendremos x = x+ )x= 36
12 2 2 2
1 x 2 1 x
2. 2x = 4 3
2 4 2
15 1 82
Solucin: c:v:a: = fx 2 Rg. Simplicando tendremos x = 2x + 10 ) x =
8 4 31
x 1 x 2 x 3
3. x + =1
2 3 4
Solucin: c:v:a: = fx 2 Rg. Simplicando tendremos
x 1 x 2 x 3 7 7 5
x + =1) x+ =1)x=
2 3 4 12 12 7

2x 1 2x + 3
4. =
x+4 x 5
Solucin: Para esta ecuacin tenemos que el conjunto de valores admisibles c:v:a: es x 6= 4, x = 6 5, es
decir, c:v:a: = fx 2 R : x 6= 4; x 6= 5g, y la solucin puede ser cualquier otro nmero, excepto 4 o 5.
Simplicando tendremos
2x 1 2x + 3
= ) (2x 1) (x 5) = (2x + 3) (x + 4) )
x+4 x 5
7
2x2 11x + 5 = 11x + 2x2 + 12 ) 22x = 7)x=
22

3
1
5. x =1
1
3+
2x
Solucin: Para esta ecuacin tenemos que el conjunto de valores admisibles c:v:a: es x 6= 0, es decir,
c:v:a: = fx 2 R : x 6= 0g, y la solucin puede ser cualquier otro nmero, excepto el 0.
Simplicando tendremos
3
1
x = 1 ) 2x 6 = 1 ) 2x 6 = 6x + 1 ) x =
7
1 6x + 1 4
3+
2x

3
2
1
1+
6. x =1
2
1+
1
1+
x
Solucin: Para esta ecuacin tenemos que el conjunto de valores admisibles c:v:a: es x 6= 0, es decir,
c:v:a: = fx 2 R : x 6= 0g, y la solucin puede ser cualquier otro nmero, excepto el 0.
Simplicando tendremos
3x
2
x+1 =1) 2 x =1)2 x = 3x + 1 ) x =
1
2x 3x + 1 4
+1
x+1
7.3. ECUACIN LINEAL O DE PRIMER GRADO 233

7. cx b (c x) = a (b x) b (a x) ; a; b; c 2 R
Solucin: Simplicando tendremos

cx b (c x) = a (b x) b (a x) ) bx bc + cx = bx ax )
8
> bc
>
< a + c ; si a + c 6= 0
bc + cx = ax ) (a + c) x = bc ) x = R; si (bc = 0) ^ (a + c = 0)
>
>
:
;; si (a + c = 0) ^ (bc 6= 0)
c:v:a: = fa; b; c 2 R : a 6= cg
2 5 2
8. + = 2
x 1 x+2 x +x 2
Solucin: Transformando a factores los denominadores tendremos
2 5 2
+ = ;
x 1 x+2 (x + 2) (x 1)
de donde tenemos que el conjunto de valores admisibles c:v:a: es x 6= 1, x 6= 2, es decir,

c:v:a: = fx 2 R : x 6= 1; x 6= 2g ;

y la solucin puede ser cualquier otro nmero, excepto el 1 y el 2.


Simplicando tendremos
2 (x + 2) + 5 (x 1) 2
= ) 2 (x + 2) + 5 (x 1) = 2
(x + 2) (x 1) (x + 2) (x 1)
3
2 (x + 2) + 5 (x 1) = 2 ) 7x 1=2)x=
7
9. a (b x) b (a x) + c (d x) = a b+c d; a; b; c; d 2 R
Solucin: Transformando el lado izquierdo de la ecuacin, tendremos cd (a b + c)x = a b+c d)
8
> a b + c d cd
>
< ; si a + b c 6= 0
a+b c
x= R; si ( a + b c = 0) ^ (a b + c d cd = 0)
>
>
:
;; si ( a + b c = 0) ^ (a b + c d cd 6= 0)

Se ve claramente que el c:v:a: = fa; b; c 2 R : a+b c 6= 0g

Practico lo que aprend:

Resolver las ecuaciones, indicar el c:v:a:, y comprobar la solucin dada:

5x 2 1 x 4 3x 3x 1 2x 4 3x 38
1. a) =3 , x = 22; b) =5 , x=
4 3 2 4 6 2 5
3 (x 5) 1 4x 1 3x 52
2. =1 , x=
4 12 2 5
3x 1 3x + 3 1 2x 1 x+3 2
3. a) = ,x= ; b) = ,x=
x+2 x 4 11 2x + 2 x 8 25
5 3
2 11 1
4. a) x = 1, x = ; b) x = 2, x = 2
1 2 1
1+ 1
2x 2x
8 2
> a b2
>
< ; si a 6= 0
x x 2a
5. + = 1; a; b 2 R, a 6= b, a 6= b; x = R; si (a = 0) ^ (b = 0)
a b a+b >
>
:
;; si (a = 0) ^ (b 6= 0)
234 CAPTULO 7. ECUACIONES E INECUACIONES
(
0 si a + b c 6= 0
6. a (b + 1 x) b (a + 1 x) + c (1 x) = a b + c; a; b; c 2 R; x =
R; si a+b c=0
2 2 5
7. = , x=1
x 3 x2 + 2x 15 x+5
3 2 1 1
8. + = 2 , x=
3x 2 2x + 1 6x x 2 6
5 7 2 29
9. = , x= .
2x2 + 14x + 24 x+4 2x + 6 12
2 3 1
10. = , no tiene solucin.
x2 + 10x + 24 x+4 x+6

7.4. Problemas de Planteo con Ecuaciones de Primer Grado

En la presente seccin presentamos la forma correcta de plantear un problema, su representacin matemtica


mediante ecuaciones algebraicas y su solucin.

Recuerda los conceptos


Descripcin: Ejemplos:
sobre las ecuaciones:
Igualdad con variables que
Ecuacin 3x 2 = 10;
se cumple slo para ciertos
algebraica y 2x = 3
valores de las mismas.
Conjunto de valores que con- x = 4 : 3 (4) 2 10;
Solucin de
vierten a la ecuacin o igual- x = 1; y = 5 : (5) 2 (1) 3;
una ecuacin
dad en una identidad ( ). x = 2; y = 7 : (7) 2 (2) 3
Resolucin de Proceso de busqueda de las 3x 2 = 10 ) 3x = 10 + 2
una ecuacin soluciones ) 3x = 12 ) x = 12=3 = 4
Incgnita Variable buscada x; y, etc.
Ecuacin del tipo a0 xn +
2x 3 = 0;
Ecuacin poli- a1 xn 1 + + an 1 x + an = 0,
4x2 3x + 1 = 0;
nmica x es la incgnita, a0 ; a1 , etc. p 3
2x5 3x x=0
son los coecientes dados.

Concepto: Descripcin: Ejemplos:


2x 3 1+x
Ecuacin reducible a la = 0;
2
5 x
Ecuacin ra- Pn (x) x 1 1 3x
forma = 0, don- + 1 = 0;
cional Qm (x) x 5 x+1
2
de P y Q son polinomios. x x 3
3 + 5x2
=0
2x
p x
Ecuacin en la que la incg- 2x 3 x + 1 = 0;
Ecuacin irra- 2
p
nita se halla en calidad de x 3 = 5 x = 0;
p
cional 3
1 5x p
cantidad subradical. + 2 = 4 x2 + 3
2
Expresin que no es ni verda- x es mayor que y, c d = 7,
Enunciado
dero, ni falso, o sea no es una z es mltiplo de 3t;
abierto
proposicin u + 2v w = 12

| Para resolver muchsimas ecuaciones algebraicas y problemas de planteo se debe tener muy en cuenta el
signicado y criterio matemticos de ciertas expresiones o palabras tales como preposiciones, verbos, enunciados
abiertos o verbales, etc.
7.4. PROBLEMAS DE PLANTEO CON ECUACIONES DE PRIMER GRADO 235

Recuerda los siguientes enun-


Formas simblicas:
ciados abiertos y verbales:
Un nmero disminuido en 7 x 7
Un nmero aumentado en 4 x+4
El duplo (doble) de un nmero 2x
El duplo de un nmero, disminuido en 3 2x 3
El duplo de un nmero disminuido en 3 2 (x 3)
El cubo de un nmero, aumentado en 5 x3 + 5
3
El cubo de un nmero aumentado en 5 (x + 5)
La suma de 2 nmeros impares conse- (2x + 1) +
cutivos es 24 (2x + 3) = 24
El costo de y libros es de $16 cada
16y
uno
El duplo de un nmero disminuido en
7 es igual al triple de este aumentado 2x 7 = 3x + 4
en 4
La edad de Pedro dentro de 3 aos y+3
La edad de Pedro hace 5 aos y 5

Formas verbales: Formas simblicas:


La suma de 2 nmeros consecutivos
z + (z + 1) = 39
es 39
La suma de 2 nmeros pares conse-
2z + (2z + 2) = 42
cutivos es 42
5 veces un nmero disminuido en 2 5 (z 2)
5 veces un nmero, disminuido en 2 5z 2
El triple de la suma de 3 nmeros 3 (x + y + z)
La cuarta parte de la diferencia de 1
(y z)
2 nmeros 4
Un quinto de la suma de los cua- 1 2
y + z2
drados de 2 nmeros 5
El duplo del cuadrado de un nmero
2x2 + 12
aumentado en 12
4 veces un nmero elevado al cubo,
4 x3 + 20
aumentado en 20

Recuerda el signicado matemtico de estas expresiones:

1. Las palabras de, del, de los se aplican con el criterio matemtico de multiplicacin:

a) El triple de un nmero ! 3 x
1 3
b) La mitad de los 3/5 de un nmero ! x
2 5
1
c) El duplo de la cuarta parte de un nmero ! 2 x
4
2. La expresin ... es a ... como ... es a ... seala una proporcin:
x 5
a) Un nmero es a otro como 5 es a 6 ! =
y 6
edad de Jose 4
b) La edad de Jos es a la edad de Luca como 4 es a 5 ! =
edad de Luc{a 5
236 CAPTULO 7. ECUACIONES E INECUACIONES

3x 7
c) El triple de un nmero es al doble de su cubo como 7 es a 6 ! =
2x3 6
3. La palabra por nos indica el cociente de 2 cantidades:
5
a) El 5 por ciento de un nmero es 120 ! x = 120
100
2
b) El 2 por 7 de 98 ! 98
7
10
c) El 10 por 30 de un nmero es 75 ! x = 75
30
4. Las palabras: es, que sea, obtiene, tiene , tendr, ser, se usan con el criterio de igualdad:
1
a) El doble de un nmero es la cuarta parte de 130 ! 2 x = 130
4
b) Qu nmero debemos restar a 15 para que nos de 8? ! 15 x = 8
c) Si al triple de un nmero le sumamos 18 se obtiene 36 ! 3 x + 18 = 36
1
d ) Edison tiene la mitad de dinero que tiene Sonia ! E = S
2
e) Si al duplo de la edad de Danilka se le suma 5 aos tendr 24 aos ! 2 x + 5 = 24

5. La palabra veces es sinnimo de producto:

a) 5 veces un nmero nos da 75 ! 5 x = 75


b) El duplo del cuadrado de un nmero da 162 ! 2 x2 = 162

6. Las palabras mayor, excede tienen el criterio de indicar que cierta cantidad tiene ms valor (es
mayor) que otra:

a) Un nmero excede a otro en 7 ! x y=7


Tambin podemos representar los nmeros mediante A = x y B = x 7
b) El triple de un nmero excede al doble de otro en 64 ! 3x 2y = 64
Tambin podemos representar los nmeros mediante A = 3x y B = 2x 64
c) El cubo de un nmero excede al cuadrado de otro en 47 ! x3 y 2 = 47
Tambin podemos representar los nmeros mediante A = x3 y B = x2 47
d ) Un nmero es mayor en 10 que otro ! x 10 = y
Tambin podemos representar los nmeros mediante A = x y B = x 10
e) Cierta cantidad es mayor que otra en 25 ! x 25 = y
Tambin podemos representar los nmeros mediante A = x y B = x 25

7. Las palabras menor y excedido son equivalentes e indican que cierta cantidad tiene menos
valor que otra:

a) Un nmero es menor en 7 que otro nmero ! x + 7 = y


Tambin podemos representar los nmeros mediante A = x + 7 y B = x
b) El cubo de un nmero es excedido por el cuadrado de otro en 25 ! x3 + y 2 = 25
Tambin podemos representar los nmeros mediante A = x3 25 y B = x2

8. Las palabras su, sus se usan con el criterio matemtico de indicar repeticin (producto) de
una misma cantidad:
1
a) Al cuadrado de nmero le disminuimos en su cuarta parte ! x2 x
4
7.4. PROBLEMAS DE PLANTEO CON ECUACIONES DE PRIMER GRADO 237

1
b) A la edad de Enrique la disminuimos en su propia mitad ! E E
2
2
c) Si a un nmero le aumentamos en sus dos terceras partes nos da 80 ! x + x = 80
3

Presentamos una tabla sobre la descripcin de enunciados verbales y abiertos y su interpretacin algebraica.

Ejemplos:

1. Representar en forma matemtica los enunciados:

a) Un nmero aumentado en 9 ! x + 9
b) Un nmero disminuido en 3 ! x 3
c) 12 disminuido un cierto nmero ! 12 x
d ) 5 veces la diferencia de un nmero con 2 ! 5(x 2)
e) 10 ms que el triple de un nmero ! 3x + 10
f ) 30 unidades ms que el doble de un nmero ! 2x + 30
g) El duplo de un nmero aumentado en 5 ! 2(x + 5)
h) El duplo de un nmero, aumentado en 5 ! 2x + 5

2. Problemas de planteo resueltos:

a) Un padre tiene 39 aos y su hijo 15. Dentro de cuantos aos la edad del padre ser el triple de la del
hijo.
Solucion: Sea el tiempo buscado = x. Dentro de x aos la edad del padre ser 39 + x; dentro de x
aos, la edad del hijo ser 15 + x. Por la condicin 39 + x = 3(15 + x) ) 6 = 2x ) x = 3 aos.
La solucin negativa 3, indica que la condicin pedida se realiz hace tres aos, en esa poca el
padre tenia 36 y su hijo 12.
Modicado el enunciado a la forma siguiente tendremos una solucin positiva. As: Un padre tiene
39 aos y su hijo 15.Que tiempo hace que la edad del primero era el triple de la edad del segundo?
Se tiene: 39 x = 3(15 x) esta es la ecuacion primitiva en la cual se ha sustituido x por x.
b) A qu hora entre las 4 y las 5 p.m. estn opuestas las agujas del reloj?
Solucion: Moviendo las agujas veremos que la hora aproximada para este suceso es aproximadamente
10 para las 5. Sabemos que el minutero al dar una vuelta completa (60 divisiones de 1 minuto), el
horario avanza en una hora, es decir, 5 divisiones de 1 minuto, o sea 1/12 de lo que recorre el minutero.
Si x es el nmero de divisiones de 1 minuto que recorre el minutero desde las 4, x=12 ser el nmero de
divisiones de 1 minuto que recorrer el horario, entonces tendremos x = 20+x=12+30 ) x = 600=11.
6
Por tanto, entre las 4 y 5 las manecillas estarn opuestas a las 4 y 600/11, es decir a las 4 y 54 11
minutos.
Compruebe que entre las 5 y las 6, antes que el minutero pase al horario, las agujas forman angulo
10
recto cuando se cumple la ecuacin x + 15 = 25 + x=12, es decir, a las 5 y 10 11 minutos.
Compruebe que entre las 5 y las 6, despus que el minutero pase al horario, las agujas forman
7
angulo recto cuando se cumple la ecuacin x = 25 + x=12 + 15, es decir, a las 5 y 43 11 minutos.
c) Dos veces la diferencia de un nmero con 3 es igual a 18. Hallar el nmero.
Solucion: Sea el nmero a hallar x. Del enunciado: dos veces la diferencia de un nmero con 3 es
igual a 18: 2(x 3) = 18 ) x 3 = 9 (simplicamos y extraemos la mitad) ) x = 12
El nmero pedido es 12 Rpta.
238 CAPTULO 7. ECUACIONES E INECUACIONES

d ) Qu nmero es menos que 60 en la misma medida que es mayor que 50?


Solucion: Sea x = el nmero pedido y la condicin x 60 = 50 x
110
Transponiendo x al primer miembro x + x = 50 + 60 ) 2x = 110 ) x = 2 = 55
El nmero pedido es 55 Rpta.
e) Si al triple de un nmero se le aumenta 5 resulta cuatro veces el nmero, menos 3. Cul es el nmero?
Solucion: Sea x = el nmero buscado y la condicin: Si al triple de un nmero se le aumenta 5
resulta cuatro veces el nmero, menos 3 ) 3x + 5 = 4x 3 ) 3x + 5 = 4x 3 ) x = 8:
El nmero pedido es 8 Rpta.
f ) Dos viajeros de los cuales uno camina a 18km/h, y el otro a 12km/h parten al mismo tiempo, el
primero de la ciudad P, el segundo de la ciudad Q dirigindose a una ciudad T. A que distancia de
la ciudad L se encontrarn sabiendo que de P a Q hay 120 km, y que de Q a L hay 280 km?
Solucion: Sea x la distancia que supondremos situada mas all de L, sobre el camino de T y sea R
el punto de encuentro:

Figura 7.2: Problema de los 2 viajeros

El viajero que parte de P recorrer : (120+280+x) kilometros. El que parte de Q recorrer: (280+x)
120 + 280 + x
kilometros. El tiempo empleado por el primer viajero ser: . Y el tiemplo empleado
18
280 + x
por el segundo: :
12
Como estos tiempos son iguales, puesto que los viajeros parten a la misma hora, se tendr la ecuacion:
120 + 280 + x 280 + x
= ) x = 40 kilometros.
18 12
Esta solucin negativa puede intepretarse como que los viajeros se encontrarn 40km ms ac de L
y no ms all.
Modicando el enunciado del problema como sigue: Dos viajeros, de los cuales uno va a 18 km/h y
el otro a 12 km/h, parten al mismo tiempo; el primero de P, el segundo de Q, dirigindose a T. A
qu distancia antes de L se encontrarn sabiendo que de P a Q hay 120 km, y de Q a L 280?
120 + 280 x 280 x
Solucion: La ecuacion ser: = ) x = 40:
18 12
En la ecuacin original al reemplazar x por x, se tendr : x = 40 km (solucin positiva).
g) Un tanque est provisto de tres llaves. La primera, abierta sola, lo llenara en 8 horas; la segunda
lo llenara en 12 horas; pero la tercera lo vaciara en 3 horas. En qu tiempo se llenar el tanque
abriendo las tres llaves simultaneamente?
Solucion: Sea t el tiempo buscado. La 1ra llave llenar en 1 hora 1=8 del tanque. La 2da llave
llenar en 1 hora 1=12 del tanque. La 3ra llave vaciara en 1 hora 1=3 del tanque.
1 1 1 1 1 1
En 1 hora se llenar + . Multiplicando este resultado por x se obtendr + x=1
8 12 3 8 12 3
tanque. De donde (3 + 2 8)x = 24 ) 3x = 24 ) x = 24=3 = 8 horas.
Esta solucin negativa indica que el problema es imposible. Existe una incompatibilidad en las condi-
ciones del enunciado, porque la suma de las dos fracciones 1=8 + 1=12 5=24 es menor que 1=3 u
8=24, as, el tanque no se llenar nunca, dando las dos primeras llaves menos lquido que el que el
lquido que deja salir la tercera.
Modicando el enunciado del problema a la forma:
7.4. PROBLEMAS DE PLANTEO CON ECUACIONES DE PRIMER GRADO 239

Un tanque est provisto de tres llaves, la primera lo llenara en 8 horas, la segunda lo llenara en 12
horas, pero la tercera lo vaciara en 3 horas.En qu tiempo quedar vaco el tanque, suponendolo
lleno?
1 1 1
Solucion: La ecuacion es entonces : + x = 1:
8 12 3
De donde se tiene x = 8 horas. Rpta.
h) Dividir 5000 en dos partes tales que la primera parte sea 200 ms que el triple de la segunda parte.
Solucion: Sean la segunda parte = x, la primera parte = 3x+200 y el enunciado, entonces concluimos
que la suma de las partes, debe ser igual a 5000 (el todo) ) x + 3x + 200 = 5000 ) 4x + 200 =
5000 ) 4x = 4800 ) x = 48004 = 1200
Luego las partes pedidas seran: segunda parte: x = 1200, primera parte 3x + 200 = 3(1200) + 200 =
3800
i ) El triple de un nmero, menos dos es igual al duplo del mismo aumentado en 3. Hallar el nmero.
Solucion: Sea: El nmero buscado = x y el enunciado: Si al triple de un nmero, menos dos es igual
al duplo del mismo aumentado en 3) 3x 2 = 2(x+3) ) 3x 2 = 2x+6 ) 3x 2x = 6+8 ) x = 8:
El nmero buscado es 8 Rpta.
j ) La suma de dos nmeros es 35 y su diferencia es 5. Hallar los numeros.
(
x + y = 35 (I)
Solucion: Sean los nmeros: x = nmero mayor, y = nmero menor y el enunciado:
x y=5 (II)
Sumando miembro a miembro (I) y (II): 2x = 40 ) x = 20. Reemplazamos el valor de x en (I)
) 20 + y = 35 ) y = 35 20 =) y = 15:
Los nmeros pedidos son: 20 y 15 Rpta.
k ) Una liebre perseguida por un galgo lleva ya adelantados 90 saltos y da 5 saltos mientras el perro
da 4, pero 7 saltos de la liebre equivalen a 5 del perro.Cuntos saltos tendr que dar el galgo para
alcanzarla?
Solucion: Sea x el nmero de saltos que da la liebre desde B hasta C:

Figura 7.3: Presecusin de la liebre por el perro galgo.

5
Desde A hasta C hay (90 + x) saltos de la liebre, pero slo (90 + x) saltos del perro. En este lapso la
7
5 4
liebre ha dado x saltos y el perro solo 54 x, de lo cual: (90 + x) = x ) 25(90 + x) = 7(4x) )
7 5
x = 750 saltos de la liebre. Entonces, los saltos del perro seran 54 750 = 600. Rpta.
l ) Despus de comprar 15 cajas de cerveza, del mismo precio, me sobran $38 y me faltan $47 para poder
comprar otra ms.De qu suma dispona?
Solucion: Sea el precio de cada caja de cerveza = x.
En el primer caso se tendra: el total de dinero = (# de cajas) (precio de cada caja) + (lo que sobra):
El total del dinero = 15x + |{z}
38 (I).
sobra
En el segundo caso se tendra : el total de dinero = 16x 47
|{z} (II)
falta
Como en ambos casos se trata de la misma cantidad, tendremos que (I)=(II):
15x + 38 = 16x 47 ) 38 + 47 = x ) x = 85:
240 CAPTULO 7. ECUACIONES E INECUACIONES

Para calcular la suma total de dinero podemos usar (I) o (II):


Usando (I) tenemos: suma total de dinero = 15(85) + 38 = 1275 + 38 = 1313 Rpta.
Usando (II) tendremos: suma total de dinero = 16(85) 47 = 1313 Rpta.
m) La suma de dos nmeros es 90 y su diferencia es igual a la tercera parte del mayor. Cules son estos?
(
x + y = 90 (I)
Solucion: Sean los nmeros x = nmero mayor, y = nmero menor y el enunciado: x .
x y= (II)
3
x x 6x x 5x
Sumando (I) y (II) ) 2x = 90 + ) 2x = 90 ) = 90 ) = 90 ) x = 54. Como
3 3 3 3
x + y = 90 ) 54 + y = 90 ) y = 36
Los nmeros pedidos son: 54 y 36 Rpta.
n) La mitad de la diferencia de un nmero con 2 es igual a la tercera parte de la suma del mismo numero
con 4. Hallar el numero.
Solucion: Sea: el nmero pedido = x y el enunciado: (La mitad de la diferencia de un nmero con
x 2 x+4
2) = (la tercera parte de la suma del mismo nmero con 4): = . Haciendo producto de
2 3
medios y extremos, tendremos 3(x 2) = 2(x + 4) ) 3x 6 = 2x + 8 ) 3x 2x = 6 + 8 ) x = 14:
El nmero buscado es 14 Rpta.
) Despus de haber perdido sucesivamente los 3/8 de su capital, 1/9 del resto y los 5/12 del nuevo
resto, una persona hereda $60 800 y de este modo la prdida se halla reducida la mitad de su fortuna
inicial. Cunto era esta fortuna?
Solucion: Sea: la fortuna inicial = x. Entonces:
La primera prdida: 83 x ! le queda: x 3x 5x
8 = 8
1 5x 5x 1 5x 8 5x
La segunda prdida: 9 8 ! le queda: 8 9 8 = 9 8 = 95 x
5 5x 5x 5 5x 7 5x 35
La tercera prdida: 12 9 ! le queda: 9 12 9 = 12 9 = 108 x
Considerando la herencia recibida ($60800), tendremos:
35x x x 35x 54x 35x 19x
108 + 60800 = 2 ) 60800 = 2 108 ) 60800 = 108 ) 60800 = 108
De donde: x = 345600. Rpta.
o) Tres nios se han repartido una bolsa de caramelos tomando el primero la mitad de los caramelos y
uno ms, el segundo la tercera parte de lo que qued, y el tercero el resto. Cuntos caramelos huba
en la bolsa?
Solucion: Sea el nmero de caramelos = x, entonces:
x
El 1ro toma: 2 + 1 ) queda x ( x2 + 1) = x x
2 1= x
2 1
1 x
do
El 2 toma: 3(2 1) queda ( x2 1) 1 x
3(2 1) = 2 x
3(2 1)
2 x
El 3ro toma: 3(2 1)
x
Luego, se debe cumplir que: 2 + 1 + 31 ( x2 1) + 23 ( x2 1) = x o x
2 + 1+ x
2 1 = x, de donde: x = x
El problema es indeterminado. Rpta.
p) Se compran cajones de naranjas a $100 cada uno; cada cajn contiene 20 kilos. Primero se vende la
mitad a $20 el kg, despus la cuarta parte a $15 el kg y por ltimo el resto se remata a $10 el kg,
ganando $11 250 en total. cuntos cajones de naranjas se han comprado?
Solucion: Si x es el nmero total de cajones comprados, entonces 20x es el nmero total de kg.
1ra venta : 10x(20) = 200x
2da venta : 5x(15) = 75x
ra
. Como se ha comprado los x cajones en 100x dlares, la ganancia
3 venta : 5x(10) = 50x
Total = 325x
ser: 325x 100x = 11250; de donde x = 50: Rpta.
7.4. PROBLEMAS DE PLANTEO CON ECUACIONES DE PRIMER GRADO 241

q) La tercera parte de la suma de dos numeros es 12 y la mitad de su diferencia es 8. Hallar los numeros.
Solucion: Sean los dos( nmeros a hallar: x = numero mayor, y = numero menor, entonces del
1
enunciado se obtiene: 3 (x + y) = 12 ) x + y = 36 (I)
1
2 (x y) = 8 ) x y = 16 (II)
52
Sumando miembro a miembro las ecuaciones (I) y (II), tendremos: 2x = 52 ) x = 2 = 26.
Reemplazando el valor de x en (I), tenemos 26 + y = 36 ) y = 36 26 ) y = 10
Los nmeros son: 26 y 10 Rpta.

3. Planteo de problemas con nmeros consecutivos:


| Para cualquier nmero x, podemos representar los nmeros enteros enteros consecutivos mediante: x,
(x+ 1), (x+2), (x+3), etc. (o x 2, x 1, x, x+ 1, x+ 2). As, por ejemplo, x ! 9, x+ 1 ! 10, x+ 2 ! 11
son consecutivos. En el caso de nmeros enteros pares consecutivos podemos representar mediante 2x,
(2x + 2), (2x + 4), etc. (o 2x 4, 2x 2, 2x, 2x + 2, 2x + 4). As, por ejemplo, 2x ! 8, 2x + 2 ! 10,
2x + 4 ! 12, etc. son pares consecutivos.
| Si x es nmero par, entonces los pares consecutivos son (x + 2), (x + 4), ... Y si x es entero, entonces
los impares consecutivos son (2x + 1), (2x + 3), (2x + 5), ...

a) Cules son los 2 nmeros enteros consecutivos cuya suma es 23?


Solucion: Sea el primer nmero entero = x, el numero consecutivo al primero x + 1. De la condicin
obtenemos:
22
x + (x + 1) = 23 ) 2x + 1 = 23 ) 2x = 22 ) x = =) x = 11.
2
Los nmeros son: primer nmero entero: x = 11, segundo nmero entero: x + 1 = 12 Rpta.
b) Hallar el mayor de 2 nmeros enteros consecutivos de modo que la cuarta parte del menor; exceda a
la quinta parte del mayor en 1.
Solucion: Sean los nmeros enteros: primer nmero entero = x, segundo nmero entero consecutivo
x (x + 1) 5x 4(x + 1)
x + 1. De la condicin obtenemos: =1) = 1 ) 5x 4x 4 = 20 )
4 5 20
x 4 = 20 ) x = 24:
Luego el mayor ser: x + 1 = 24 + 1 = 25 Rpta.
c) Cul es se nmero entero impar, tal que agregado a los cuatro nmeros impares que le siguen da
un total de 905?
Solucion: Podemos representar los cinco nmeros enteros impares consecutivos de la forma: 2x 3,
2x 1, 2x + 1, 2x + 3, 2x + 5, siendo 2x 3 el nmero buscado. Por la condicin:
900
(2x 3) + (2x 1) + (2x + 1) + (2x + 3) + (2x + 5) = 905 o 10x + 5 = 905 ) 10x = ) x = 90.
10
El nmero buscado es 2x 3 = 2(90) 3 = 177 Rpta.
d ) La suma al cuadrado menos la diferencia al cuadrado de dos nmeros impares consecutivos es 60.
Cul es el nmero menor?
Solucion: Sean los nmeros: El menor nmero entero = 2x + 1, su consecutivo = 2x + 3. De la
condicin: (Suma de los nmeros)2 (Diferencia de los nmeros)2 = 60, tenemos que (2x + 3 + 2x +
1)2 (2x + 3 2x 1)2 = 60 ) (4x + 4)2 (2)2 = 60 )
6x2 + 32x + 16 4 = 60 ) 16x2 + 32x 48 = 0:
2
Simplicando por 16: x + 2x 3 = 0 ) (x + 3) (x 1) = 0 ) x = 1 o x = 3:
Si x = 1 ) 2( 1) + 1 = 1 y si x = 3 ) 2( 3) + 1 = 5:
El menor es 5 Rpta.
e) Hallar el menor de tres nmeros enteros consecutivos si sabemos que los 3/4 del menor, sumado con
la tercera parte del nmero medio, equivale al mayor.
242 CAPTULO 7. ECUACIONES E INECUACIONES

Solucion: Consideremos los nmeros enteros consecutivos: x 1, x, x + 1 (en forma simtrica: menor,
3 3 9(x 1) + 4x
medio, mayor). Por la condicin: (x 1) + x = x + 1 ) = x + 1 ) 9(x 1) + 4x =
4 4 12
12(x + 1) ) 13x 9 = 12x + 12 ) x = 21:
El nmero menor ser: x 1 = 21 1 = 20 Rpta.
f ) El producto de tres nmeros enteros consecuticos es igual a 15 veces el segundo. Hallar su suma.
Solucion: Sean los nmeros enteros consecutivos: x 1, x, x + 1 (en forma simtrica: menor, medio,
mayor). Del enunciado: el producto de 3 numeros enteros consecutivos es igual a 15 veces el segundo,
tendremos (x 1)(x)(x + 1) = 15x. Simplicando x, (x 6= 0) obtenemos
(x 1)(x + 1) = 15 ) x2 1 = 15 ) x2 = 16 ) x = 4.
Si x = 4, entonces los nmeros son: 3, 4, 5. Si x = 4, los nmeros sern 5, 4, 3 Rptas.
g) Dos nmeros consecutivos son tales que la tercera parte del mayor excede en 15 a la quinta parte del
menor. Hallar el nmero mayor.
Solucion: Sean x 1, x los nmeros consecutivos. De la condicin: la tercera parte del mayor exede
en 15 a la quinta parte del menor, tenemos
1 1 x (x 1) 5x 3(x 1)
x 15 = (x 1) ) = 15 ) = 15 ) 5x 3x + 3 = 225 ) x = 111
3 5 3 5 15
Rpta.
h) Hallar 3 nmeros consecutivos tales que el cuadrado del nmero medio sea mayor en una unidad que
el producto de los otros dos.
Solucion: Utilizamos la forma simtrica para escribir los nmeros consecutivos: n 1, n, n + 1. De
la condicin tenemos que n2 = (n 1)(n + 1) + 1 ) n2 1 = (n 1)(n + 1) ) n2 1 = n2 1.
Cmo interpretar este resultado? Que se cumple para cualquier valor que se le asigne a n: Rpta.
i ) Hallar tres nmeros enteros consecutivos tales que el cuadrado del nmero medio sea mayor en una
unidad que el producto de los dos restantes.
Solucion: Consideremos los nmeros enteros consecutivos (menor, medio, mayor): x 1, x, x + 1.
Del enunciado tenemos: el cuadrado del nmero medio sea mayor en una unidad que el producto de
los dos restantes x2 1 = (x + 1)(x 1) ) x2 1 = x2 1:
Esta relacin se cumple para cualquier nmero entero x. Rpta.
j ) La suma de los cuadrados de 3 nmeros enteros consecutivos es 29. Hallar los nmeros.
Solucion: Sean x 1, x, x + 1 los nmeros enteros consecutivos. De la condicin tenemos: (x 1)2 +
x2 + (x + 1)2 = 29 ) x2 2x + 1 + x2 + x2 + 2x + 1 = 29 ) 3x2 = 27 ) x = 3:

Si x = 3, entonces los nmeros son: 3, 4 y 5. Si x = 3, entonces los nmeros son: -4, -3, -2. Rptas.

4. Problemas sin solucin o imposibles:


| Observaciones: Un problema es imposible cuando:

las soluciones son negativas debiendo ser esencialmente positivas,


m
una de las soluciones toma la forma , es decir, con divisin por 0,
0
las soluciones son fraccionarios, exigiendo la naturaleza que sean enteras,
los valores encontrados para las incgnitas no satisfacen las ecuaciones y condiciones.

a) Sabiendo que 7 metros de tela valen tanto como 3 metros de pao; se desea saber cual es el precio de
cada uno de estos gneros, sabiendo que 9 metros de pao cuestan $12 ms que 24 metros de tela.
Solucion:(Sea el precio del metro de tela = x, el precio del metro de pao = y. Por las condiciones
7x = 3y (I) 7x
tenemos . De (I) y = (III). Reemplazando: (III) en (II) tendremos:
9y 24x = 12 (II) 3
7x
9( ) 24x = 12 ) x = 4 dlares. x debe ser positivo, por ello el problema es imposible. Rpta.
3
7.4. PROBLEMAS DE PLANTEO CON ECUACIONES DE PRIMER GRADO 243

b) Doce ociales entre capitanes y tenientes, comen juntos y se hace un gasto total de 45 dlares.Cuntos
capitanes y tenientes son, si se sabe que los primeros han gastado 5 dlares cada uno y los segundos
3 dlares ms cada uno?
Solucion: Sean el nmero de capitanes = x, y el nmero de tenientes = y. Por las condiciones,
tenemos:
(
x + y = 12 (I)
. De (I) y = 12 x (III). Reeplazando (III) en (II) nos da 5x+3(12 x) = 45,
5x 3y = 45 (II)
de donde: x = 4;5 ) y = 7;5
La naturaleza del problema exige que los valores de x e y sean enteros, por ello el problema es
imposible. Rpta.
c) Hallar un nmero tal que la mitad ms seis, sea igual a dos veces su cuarta parte ms cuatro.
Solucion: Sea el nmero buscado = x. De acuerdo al enunciado:
x x x x
+ 6 = 2( + 4) (I) ) + 6 = + 8 ) x + 12 = x + 16 ) 12 = 16, imposible!
2 4 2 2
4
El problema no tiene solucin (x + 12 = x + 16 ) x (0) = 4 ) x = !). Rpta.
0
d ) Hallar dos nmeros cuya diferencia sea 15 y tales que tres veces el primero menos cuatro veces, el
segundo den 20 por diferencia y que el primero mas dos veces el segundo den 100 por suma.
8
>
< x y = 15 (I)
Solucion: De las condiciones del problema, tenemos las ecuaciones: 3x 4y = 20 (II) . El enunci-
>
:
x + 2y = 100 (III)
ado del problema se reduce a tres ecuaciones con dos incgnitas. Resolviendo las dos primeras, tendremos
x = 40, y = 25. Estos valores no satisfacen la ecuacin (III), por tanto, el problema no tiene solucin.

5. Problemas indeterminados:
Observaciones:
| Un problema es indeterminado cuando su enunciado puede conducir a menos ecuaciones (todas distintas)
que incgnitas o al aparecimiento de la forma indeterminada 00 . La indeterminacin puede tener lugar an
cuando se hayan encontrado tantas ecuaciones como incgnitas tiene el enunciado.

a) Cul es el nmero que restndole su mitad y 8, es igual a 4 veces su octava parte menos 2?
x x
Solucion: De acuerdo al enunciado se tiene: x 8 = 4( 2) (I) ) 8x 4x 64 = 4x 64
2 8
(II) ) 8x 8x = 64 64 ) 0 = 0 (x(0) = 0 ) x = 0=0):
El problema es indeterminado con una innidad de soluciones, pues el enunciado conduce a una
identidad. Rpta.
b) Hallar dos nmeros tales que dos veces el primero menos tres veces el segundo dan 5 por diferencia,
y que diez veces el primero menos 25 sea igual a 15 veces el segundo.
(
2x 3y = 5 (I)
Solucion: De acuerdo a las condiciones se tiene: . Despejando x de (I) nos da
10x 25 = 15y (II)
5 + 3y 10(5 + 3y)
x= (III). Reeplazando (III) en (II) tendremos que 25 = 15y ) 50 + 30y 50 =
2 2
30y ) 0 = 0 (0 (y) = 0 ) y = 0=0).
Aparece la indeterminacin porque las dos ecuaciones del problema no son distintas, es decir una esta
includa en la otra.

Practico lo que aprend:

1. Hallar 2 nmeros consecutivos de tal manera que la suma del duplo del menor y el triple del mayor sea
103. Rpta. 20 y 21
244 CAPTULO 7. ECUACIONES E INECUACIONES

2. La suma de 3 nmeros enteros consecutivos es n.Cul de estos nmeros nos representa al mayor? Rpta.
n=3 + 1

3. La diferencia entre el cuadrado del mayor y el cuadrado del menor de dos nmeros consecutivos es igual
a 5. Hallar los nmeros. Rpta. 2 y 3

4. Hallar tres nmeros impares consecutivos tales que la suma de los dos ltimos sea 29 ms que el primero.
Rpta. 23, 25, 27

5. Hallar tres nmeros naturales consecutivos tales que dos veces el menor sea 53 menos que tres veces el
mayor. Rpta 47, 48 y 49.

6. La suma de tres numeros pares consecutivos es 366. Hallar los nmeros. Rpta. 120, 122 y 124.

7. La suma de tres nmeros impares consecutivos es 237. Hallar los nmeros. Rpta. 39, 41 y 43.

8. Hallar tres nmeros consecutivos de modo que la diferencia entre el mayor y el menor sea igual a los 3/10
del intermedio. Rpta. 4, 5 y 6.

9. Sean dos nmeros consecutivos tales que la cuarta parte del mayor exceda en 5 a la sexta parte del menor.
Hallar el nmero menor. Rpta. 59.

10. El promedio de los cuadrados de dos nmeros consecutivos es 240.5, determinar el doble de la tercera
parte del nmero menor. Rpta. 10.

11. A un cierto nmero par, se le suma los dos nmeros pares que le preceden y los dos nmeros impares que
le siguen, obtenindose en total 108 unidades. El producto de los dgitos del nmero par es. Rpta. 4.

12. La suma de 6 nmeros pares consecutivos es 150, encontrar el ltimo trmino de estos. Rpta. 30.

13. A cierto nmero par se le suma los dos nmeros pares que le preceden y los dos nmeros impares que le
siguen, obtenindose en total 968 unidades. El producto de los dgitos del nmero par de referencia es.
Rpta. 36.

14. Hallar tres nmeros consecutivos tales que el cuadrado del nmero medio sea mayor en una unidad que el
producto de los dos restantes. Rpta. El problema tiene innito nmero de soluciones.

15. Hallar el menor de dos nmeros enteros consecutivos tales que la quinta parte del mayor exceda a la
sptima parte del menor en tres. Rpta. 49.

16. La suma de un nmero ms los tres cuartos del mismo es igual a 21 unidades ms la mitad de aquella
suma. Cul es la tercera parte de dicho nmero?. Rpta: 8

17. Si a una cantidad le quito el 40 % de dicha cantidad y a todo eso le agregara el 50 % de la cantidad que
me queda, esto ser igual que si a 180 le quitara la quinta parte. De qu cantidad se habla?. Rpta: 160

18. La suma de dos nmeros es 191, si el mayor se divide por el menor el cociente es 4 y el residuo es 16, la
diferencia de dichos nmeros es: Rpta: 121

19. Hallar la fraccin que es igual a 1, si se le suma 3 al numerador y se le resta 2 a su denominador, y es


igual a 1/2 si se le resta 1 a su numerador y 5 a su denominador. Rpta: 2/7

20. La suma, el producto y la diferencia de dos nmeros son entre s como: 9;28 y 25. Cul es el mayor de
estos nmeros? Rpta: 14

21. Juanito le dice a su mam: "De los 14000 que me diste gast 5860 soles ms de lo que no gast"Cunto
no gast? Rpta: 4070
7.5. INECUACIONES LINEALES 245

22. El producto de dos nmeros es 84. Si se aumenta 5 al multiplicando el producto es 119. Cules son los
nmeros?
Rpta: 7 y 12

23. Un granjero va al mercado llevando pollos, vende la mitad de los que llevaba ms 4 pollos, remata la
mitad ms 3 del resto y nalmente regala la mitad de los que le quedaban ms 2 pollos sobrndole 1 pollo.
Cuntos pollos llev al mercado? Rpta: 44

24. Despus de comprar 55 relojes del mismo precio, me soran $ 55 y me faltaran $62 para poder comprar 3
ms. Cunto dinero tena? Rpta: 2200

25. Se ha comprado por $ 60, cierto nmero de cuadernos, si se hubieran comprado 3 ms cada uno hubiera
costado $ 1 ms barato. Cuntos cuadernos eran? Rpta: 5

26. La suma de dos nmeros en 1043, su cociente es 27 y su resto es el mayor posible. El dividendo es?
Rpta: 1007

27. "Si a mis alumnos los hago sentar de 2 en 2 me faltan 3 carpetas; pero si los hago sentar de 4 en 4 me
faltan 14 alumnos para que en todas las carpetas haya el mismo nmero de ellos". Cuntos alumnos y
carpetas hay? Rpta: 26 y 10.

28. Si al denominador de una fraccin se le resta 5, el valor de la fraccin es 1; pero si al numerador se le


aumenta 28, el valor de la fraccin es 2. Cunto es el numerador? Rpta: 18

29. Entre Ren y Nadia tienen $ 20000. Si Ren le diera a Nadia $ 2000, ahora el dinero de Ronald y el de
Nina estaran en la relacin de 3 a 2 Cunto dinero tena Nadia? Rpta: $ 6000

30. Cul es la fraccin impropia que resulta duplicada, si se resta a sus dos trminos, la mitad de su numer-
ador? Rpta: 3/2.

7.5. Inecuaciones Lineales

Sea f = f (x1 ; x2 ; : : : ; xm ) una relacin o expresin de una o varias variables (argumentos). Resolver la
desigualdad
f <0 (1)
signica hallar un conjunto de valores del argumento (o argumentos) de la relacin f , para los cuales el valor de
la relacin f es negativa, es decir, para los que la desigualdad (1) se convierte en cierta desigualdad numrica
verdadera.

Anlogamente, resolver la desigualdad


f >0 (2)
signica hallar un conjunto de valores del argumento (argumentos) de la relacin f , para los cuales los valores
de esta funcin son positivos, es decir, para los que la desigualdad (2) se convierte en una desigualdad numrica
vlida.

| Un conjunto de valores de los argumentos de la relacin o funcin f , para los cuales la desigualdad se
convierte en una desigualdad numrica vlida, se llama conjunto de soluciones de la desigualdad o simplemente
solucin de la desigualdad. Dos desigualdades se consideran equivalentes, si los conjuntos de sus soluciones
coinciden.

| Resolver varias desigualdades signica hallar una o varias desigualdades equivalentes que nos den un
conjunto solucin para el cual todas y cada una de las desigualdades originales se cumplan. En otras palabras,
246 CAPTULO 7. ECUACIONES E INECUACIONES

Figura 7.4: Izq. El lsofo, matemtico (conceptos de lo innitamente grande y pequeo), gemetra (cuestiones sobre
perspectiva) y astrnomo griego Anaxgoras (Clazomene, 500 - Lmpsaco, 428 A.C.) de la escuela jnica, maestro en la
Escuela de Filosofa de Atenas de Pericles, Tucdides, Eurpedes, Demcrito, Empdocles y tal vez Scrates. Introdujo la
idea de un principio espiritual, ordenador del mundo, la divisibilidad indenida del espacio y del tiempo y la existencia de
un nmero innito de elementos, diferentes e irreductibles: Hay un poco de cada cosa en todas las dems, Lo semejante
busca a lo semejante. Explic la causa de los eclipses de sol y de luna. Der. El genial matemtico y sabio Muhamet
Al-Kwarismi (Kwarismi, 780 - Bagdad, 847), director de la Casa de la Sabidura de manuscritos de Bagdad. Trabajos en
aritmtica y lgebra (numeracin, nmeros primos, fracciones, numeracin decimal y cifras indes; ecuaciones de 1ro y
2do grados, regla del 3, problemas algebraico - geomtricos y clculo de sus propiedades). De el provienen los nombres
de algebra y algoritmo.

signica encontrar el conjunto interseccin de todas las soluciones de cada desigualdad. Para esto, se usan
transformaciones de desigualdades que conducen a las desigualdades equivalentes: la adicin a ambos miembros
de la desigualdad del mismo nmero y la multiplicacin (o divisin no nula) de ambos miembros de la desigualdad
por el mismo nmero (que conserva el sentido de la desigualdad si el nmero es positivo y cambia el sentido si
es negativo).

| La inecuacin f 7 0 de una sola variable o indeterminada que despus de ser reducida o simplicada,
toma la forma ax + b 7 0, a 6= 0, se denomina inecuacin lineal o de primer grado. El conjunto de soluciones de
la desigualdad de primer grado

ax + b > 0; a 6= 0 (3)

puede ser hallado de la manera siguiente: agregamos a ambos miembros de la desigualdad (3) el nmero b,
de esto resulta una desigualdad equivalente
ax > b; (4)

y dividimos a ambos miembros de la desigualdad por a. Si a > 0, entonces la desigualad (4) se convierte en
b
la desigualdad x > , la cual da un conjunto de soluciones de la desigualdad inicial (3). Este conjunto de
a
b
soluciones tambin puede escribirse en la forma x 2] ; +1[. Ahora, si a < 0, entonces la desigualdad (4)
a
b
es equivalente a la desigualdad x < , y el conjunto de nmeros reales que satisfacen a esta desigualdad, es
a
b
el conjunto de soluciones de la desigualdad inicial (3): x 2] 1; [.
a
(
ax + b > 0
| El conjunto de soluciones de un sistema de 2 desigualdades lineales (conjuncin) , se
cx + d > 0
halla como la interseccin de los 2 conjuntos de soluciones de cada una " de estas desigualdades. Y el conjunto
ax + b > 0
de soluciones de un sistema de 2 desigualdades lineales (disyuncin) , se halla como la unin
cx + d > 0
7.5. INECUACIONES LINEALES 247

Figura 7.5: Izq. Uno de los padres de la teora moderna de funciones, mile Borel (Saint-Arique,1871 - Pars, 1956),
gran cientco, popularizador de la ciencia, poltico y miembro de la resistencia francesa en tiempos de la ocupacin
nazi. Varios trabajos fundamentales sobre conjuntos (conjuntos y funciones de Borel), funciones analticas, teora de la
medida, teora de probabilidades, historia y losofa de las matemticas. No aceptaba el axioma de eleccin. Compuso
10 tomos de un total de 50 del Tratado de Teora de Funciones y 3 tomos del Tratado de Probabilidades. Der. El
famoso matemtico italiano Giuseppe Peano (Cuneo,1858 - Turn, 1932). Uno de los padres de la lgica simblica, que
utiliz para reformular las matemticas como trat de hacerlo. Especialista en teora de funciones. Deni totalmente los
nmeros naturales con los axiomas de Peano e introdujo una curva continua (fractal), sin derivada ni tangentes que ocupa
todo un cuadrado. Autor de su famoso Formulario Matemtico e inventor de la lengua articial llamada interlingua.

de los 2 conjuntos de soluciones de cada una de estas desigualdades.

Ejemplos:

3 x
1. Resolver la inecuacin lineal (x 3) + x 5
2 2
3 x
Solucin: (x 3) + x 5 ) 2x 92 x 5 ) 2x x 9
2 5)x 1=2. La solucin general
2 2
viene dada por la relacin x 1=2 o x 2 [ 1=2; +1[:
x x x 5
3
2. Resolver la inecuacin lineal < +1
24 3
x 3 x x 5 x 3 x x 5 11 7x
Solucin: < +1) <1) <1
4 2 3 4 2 3 12
) 11 7x < 12 ) 7x > 1)x> 1=7

1
La solucin general viene dada por la relacin x > 1=7 o x 2] 7 ; 1[:
8
> x 1 x 2
< 2 3 4
x 1 x 2
3. Resolver el sistema de desigualdades +
> 3
: x 1
2 2
x 2
2 4 < 3
Solucin: 8 8 8
> x x 2 1 > 1 1 1 > 1 1 1
< 2 4 3 < 4x + 2 3 < 4x 3 2
x x 2 1 1 1 1 1
) 1 6x
) 6x 1
>
:
3
x
2
x 2 1
2 >
: 1
2 >
:
2

2 3 < 4 6x + 23 < 1
4
1
6x <
1
4
2
3
8 8
> 1 1 > 2
< x 4 3 2 < x 2 [ 3 ; 1[
1
x 6 +1 ) x 2 [9; 1[ )?
>
:
2
1 2
>
: 5
x<6 4 3 x 2] 1; 2 [
Como no existe interseccin entre los 3 conjuntos resultantes, entonces el sistema de desigualdades no
tiene solucin (solucin ;. Ver Figura 7.6).
248 CAPTULO 7. ECUACIONES E INECUACIONES

Figura 7.6: Conjunto solucin (interseccin de los 3 rayos).

Figura 7.7: Representacin geomtrica de la solucin del ejemplo 4.

2 8
> x 1 1 x+1
< 3 2 4
6 x 2 1 3x+2
6 2 +
6 >
:
3 2
4. Resolver el sistema de desigualdades 6
6 "
3x
2
1
4 < x+3
3
6 5x 3
4 2 4 < x3 1
3x 1 1 x+1
2 3 < 3
| Observacin: de aqu, en adelante, mediante el smbolo f (llaves), representaremos al con-
junto interseccin (conjuncin) de sus componentes; y mediante el smbolo [ (corchetes),
representaremos a la unin (disyuncin) de sus componentes.
Solucin: 2 8 2 8
> x 1 1 x+1 > x 1 x+1 1
< 3 2 4 < 3 4 2
6 x 2 1 3x+2 6 x 2 3x+2 1
6 2 + 6
6 > 3 2 6 > 2 2 3
6 : 3x 1
< x+3 )6
: 3x x+3
< 1
6 " 2 4 3 6 " 2 3 4
6 5x 3 6
4 2 4 < x3 1 4
5x
2
x 1
3 < 4
3

3x 1 1 x+1 3x 1 x+1 1
2 3 < 3 2 3 < 3
2 8 2 8
> 1 7 1 >
< 12 x 12 2 < x 2 [13; +1[
6 1 6
6 x 2 6 x 2 [ 53 ; +1[
6 >
:
3 6 >
:
)6
6 "
7
6x 1 < 14 )6
6 " x 2] 1; 15 14 [
6 13 6
4 6 x + 13 < 34 4 x 2] 1; 26 5
[
7 5 1
6x 6 < 3 x 2] 1; 1[
"
;
) ) x 2] 1; 1[
x 2] 1; 1[

Como no existe interseccin entre los 3 primeros conjuntos resultantes, entonces tenemos que la solucin
parcial de las 3 primeras desigualdades es el conjunto vaco ? (ver Figura 7.7). La solucin parcial de las
2 ltimas desigualdades viene a ser la unin de sus soluciones, es decir el conjunto ] 1; 1[. La solucin
7.6. MTODO DE LOS INTERVALOS 249

general de todo el sistema de desigualdades viene a ser la unin de las soluciones parciales, es decir, el
conjunto ?[] 1; 1[=] 1; 1[.

Practico lo que aprend:

Resolver las siguientes inecuaciones con parmetros a y b:

1 2 x+a 1
1. a) (x a) < , b) ax + 3 > x b, c) ax + b < 2 bx, d) (a + 1) x + 4 < (1 2a) x + 3:
2 5 4 2
1 2 x a 1
2. a) ( x 3) < , b) ax 2 > bx a, c) (a 3) x + b < 2 (1 b) x, d) (a b) x + 4 <
3 5 4 2
(b 2a) x + 3:
Resolver las siguientes inecuacines lineales. Construir en el eje numrico R la solucin:
3 x 2 x 3 x 5 6x 3 x x 5
3. a) (4x 5) + x 5, b) (3x 5) + < , c) < :
2 4 5 10 2 10 4 2
1 2x 1 4 2x 3 x 4x 3 x 1 x
4. a) (x 5) + 3x 1, b) (9x 2) + < , c) < :
2 4 3 10 2 2 4 8
1 1 2x 2 3x 2 2 5x 5 x 1 2x 3 x
5. a) (2 4x) + , b) x + 0, c) :
4 3 9 3 10 4 12 8
6. Resolver los siguientes sistemas de desigualdades lineales:
8
( " > 1
2x 3 3 3 4 x
2x 2 3 4 x < 3x 1 > 3 (3x + 1)
3 3 1
a. a) , b) , c) 3 (3x 1) 2 x
3 < x2 1 1
3 (3x 2) 23 x
1 >
:
3
2x + 2 > 13 (3x 2)
2 8 2
2x 1 3 x2 > 1
< x 1 > 3 (x + 1) 2x 1 > 13 (x 1)
6 1 x 2 6 3
b. a) 4 4 2 (2x 1) < 3 , b) 3 (3x 1) 2 x , c) 4 4 (2x 3) 2 3x
3 x 2x
>
:
1 2 3 1 < 3 1 2x 1 > 21 (3x 2) x + 1 > 21 (3x + 2)

7. Resolver los siguientes sistemas de desigualdades lineales (combinaciones de conjunciones y


disyunciones) y gracar su solucin:
2 8 2 8 8 8
> x 1 1 > x 1 1 x+1 > > x 1 1
< 3 1 < > < 3 2
6
2
6
3 2 4 >
>
2
6 x 2
+ 1
4 6 x 2
+ 1 3x+2 >
> x 2
+ 1
0
6 > 2 3 6 > 2 3 2 < > 2 3
: 3x 1 6 : 3x 1 < x+3 : 3x 1
a) a) 6
6 " 2 4 < 3 , b) 6 " 2 4 3 , c) " 2 4 <1
6 6 >
>
5x 3 x 1 5x 3 x 1 > 5x 3
4 2 4 < 3 4 2 4 < 3 >
>
> 2 4 <0
3x 1 1
< x+1 3x 1 1
< x+1 : 3x 1 1
2 3 3 2 3 3 2 3 < 3
2 8 8 2 2 8
> x 1 x > x 1 x x+1 > x 1 x
< 8 1 > < 3 4
6
2 >
> 6
4 12 4
6
4
6 x 1 x >
> x 2 x 3x+2
6 x 2 x
6 > 4 + 3 4 < 4 2 +3 2 6 > 2 + 6 0
: 3x 1 6 : 3x 1 1 < x
b) a) 6
6 ( 2 4 < 3 , b) " 2
3x x
8 < x+3
3 , c) 6 " 2 4
6 >
> 6
4
5x 3
< x 1 >
> 5x 3x
< x 1
4
x 3
< 0
8 4 3 >
> 2 4 3 2 4
3x 1 1 x+1 : 3x 1 x x+1 x 1 1
2 3 < 3 6 3 < 3 2 3 < 3x

7.6. Mtodo de los Intervalos

El mtodo de los intervalos es la herramienta ms general para resolver inecuaciones racionales. Aqu lo
P (x)
utilizamos para resolver inecuaciones reducibles a lineales, es decir, reducibles a la forma ? 0, donde
T (x)
tanto P como T son polinomios (por ejemplo, factorables en productos lineales) de la variable real x. Este
mtodo consiste en determinar los intervalos de variacin de los signos de cada uno de los factores que intervienen
en la desigualdad, tanto en P como en T . Cada uno de estos intervalos de variacin de los signos es determinado
250 CAPTULO 7. ECUACIONES E INECUACIONES

Figura 7.8: Izq. El matemtico ruso, Victor Bunyakovsky (Bara, Padolskaya, 1804 - Peterburg, 1889). Alumno de
Augustin Cauchy (Pars, 1789 - Sceaux, 1857). Estudios y desarrollos en la teora de nmeros (mtodos analticos),
mximos y mnimos de funciones, clculo integral y teora de probabilidades. Escribi el famoso texto Fundamentos
Matemticos de la Teora de Probabilidades (1846). Promovi el desarrollo de las matemticas en toda Rusia. Der.
El mdico, matemtico y fsico italiano, Girolamo Cardano (Pava, 1501 - Roma, 1576). Se le debe la divulgacin de la
frmula de resolucin de una de las formas de la ecuacin de 3er grado, descubierta por Niccol Tartaglia (el tartamudo)
(Fontana) (Brescia, 1499 - Venecia, 1557) a quin prometi no publicarla. Adems public la resolucin de la ecuacin
de 4to grado, perteneciente a su alumno Lodovico Ferrari (Bolonia, 1522 - 1565). En mecnica invent la articulacin y
suspensin Cardn. Su obra matemtica fundamental es Ars Magna donde estn contenidos muchos conocimientos sobre
aritmtica y ecuaciones algebraicas hasta de 4to grado.

mediante el clculo de las races de P y T . As, por ejemplo, si P es una expresin de la variable x 2 R, y
3 2
puede ser descompuesta en la forma P (x) = 3 (x + 2) (2x 1) (x 5) , entonces P posee 6 races (incluyendo
su multiplicidad: x1 = 2, x2 = 1=2 (multiplicidad 3), x3 = 5 (multiplicidad 2)). De acuerdo a la posicin que
ocupan estas races en el eje real R, existiran 4 intervalos de variacin del signo de la expresin P (x), a saber:

El primer intervalo I, que va desde 1 hasta 2 (o] 1; 2] ):

El segundo intervalo II, que va desde 2 hasta 1=2 ( o ] 2; 1=2] ):

El tercer intervalo III, que va desde 1=2 hasta 5 ( o ]1=2; 5] ):

El cuarto intervalo IV, que va desde 5 hasta +1 ( o ]5; +1[ )

| El mtodo de los intervalos sirve para resolver cualquier desigualdad f 7 g, que despus de las debidas
Pn (x)
simplicaciones se transforma en una desigualdad racional equivalente y del tipo 7 0, donde P y T
Tm (x)
son polinomios con coecientes reales de grados n y m, respectivamente.

| Si suponemos que P (x) es un polinomio de n simo grado con coecientes reales, siendo c1 , c2 , : : :, ci todas
sus races reales de multiplicidades k1 , k2 , : : :, ki , respectivamente. El polinomio Pn (x) puede representarse en
la forma de producto (polinomio factorado linealmente):

Pn (x) = (x c1 )k1 (x c2 )k2 : : : (x ci )ki ;


7.6. MTODO DE LOS INTERVALOS 251

donde es evidente que k1 + k2 + + ki = n:

Para hallar todas las soluciones de la desigualdad Pn (x) 7 0 es suciente saber las races reales del polinomio
Pn (x), sus multiplicidades y el signo del polinomio Pn (x) en un punto arbitrario x0 :

Anlogamente se razona para la desigualdad Tm (x) 7 0, y con la descomposicin del polinomio Tm (x) en
la forma:
e e e
Tm (x) = (x c1 )k1 (x
e c2 )k2 : : : (x
e ci )ki ;
e

donde es evidente que e


k1 + e
k2 + +e
ki = m:

Para hallar todas las soluciones de la desigualdad Tm (x) 7 0 es suciente saber las races reales del
polinomio Tm (x), sus multiplicidades y el signo del polinomio Tm (x) en un punto arbitrario x0 :
Pn (x)
Ahora, para resolver la desigualdad 7 0 juntamos los anteriores resultados de variacin de los signos
Tm (x)
de Pn (x) y Tm (x) en una tabla, y utilizando la ley de los signos sacamos el signo de los productos del numerador
Pn (x)
y del denominador y luego del cociente . Ilustramos el mtodo de los intervalos mediante ejemplos.
Tm (x)

Ejemplos:

1. Resolver la desigualdad x2 4 0, x 2 R:
Solucin: x2 4 0 ) x2 4 ) jxj 4 o x 2] 1; 4] [ [4; +1[:

2. Resolver la desigualdad 4x2 + 9 > 0, x 2 R:


9 3 3 3
Solucin: 4x2 + 9 > 0 ) 4x2 9 < 0 ) x2 < ) jxj < o x 2] ; [:
4 2 2 2
3. Resolver la desigualdad x2 5x + 6 < 0, x 2 R:
Solucin: P2 (x) = x2 5x + 6 < 0 ) (x 3) (x 2) < 0 )
x 1 2 3 +1
x 3 0 +
x 2 0 + +
P2 (x) + +
Conjunto solucin: x 2]2; 3[ o 2 < x < 3:

4. Resolver la desigualdad x2 (x + 2)(x 1)3 > 0


Solucin: El polinomio Pn (x) = P6 (x) = x2 (x + 2)(x 1)3 posee las siguientes 6 races reales: x = 0
(de multiplicidad k1 = 2), x = 2 (de multiplicidad k2 = 1), x = 1 (de multiplicidad k3 = 3). Al pasar
a travs del punto x = 1 el polinomio cambia su signo y se convierte de negativo (para x < 1) a positivo
(para x > 1) puesto que x = 1 es la raz de multiplicidad tres (raz impar); al pasar por el punto x = 0
el polinomio no cambia de signo, puesto que x = 0 es una raz de multiplicidad 2 (raz par); al pasar a
travs del punto x = 2 el polinomio de nuevo cambia de signo y se convierte de negativo (para x < 2)
a positivo (para x > 2). Los intervalos de variacin del polinomio se detallan en la siguiente tabla:
x 1 2 0 1 +1
2
x + + 0 + +
x+2 0 + + +
3
(x 1) 0 +
P6 (x) + +
Conjunto solucin: x 2] 1; 2[[]1; +1[:
252 CAPTULO 7. ECUACIONES E INECUACIONES

P (x)
| Otro mtodo para resolver la desiguadad racional del tipo ? 0, donde P (x) y Q(x) son polinomios,
Q(x)
2
sera multiplicando a ambos miembros de la desigualdad anterior por el polinomio [Q(x)] , obtendremos
la desigualdad algebraica P (x) [Q(x)] ? 0 equivalente a la desigualdad original, excepto en los puntos
donde se hace cero el denominador Q(x).
2
(x + 4) (x 1)3
5. Resolver la desigualdad racional 0
(x 3) (x + 2)
2
P5 (x) (x + 4) (x 1)3
Solucin: Sea = . Multiplicando a ambos miembros de la desigualad por (x
Q2 (x) (x 3) (x + 2)
3)2 (x + 2)2 , obtenemos una desigualdad equivalente a la original:
2
(x + 4) (x 1)3 (x + 2)(x 3) 0; x 6= 2; 3

El conjunto de soluciones de la ltima desigualdad lo hallamos mediante el mtodo de intervalos:


x 1 4 2 1 3 +1
2
(x + 4) + 0 + + + +
(x 1)3 0 + +
x+2 0 + + +
x 3 0 +
P5 (x)
+ +
Q2 (x)
Conjunto solucin: x 2] 1; 2[[[1; 3[:
16 16 20
6. Resolver la desigualdad racional + + 3 2x x2
9 (x + 1) 3 (x + 1)2 9 (x 2)
Solucin: Pasando todos los trminos a la izquierda, simplicando y tomando comn denominador,
tendremos
16 16 20
x2 + 2x 3+ + + 0)
9 (x + 1) 3 (x + 1)2 9 (x 2)
9(x2 +2x 3)(x+1)2 (x 2)+16(x+1)(x 2)+48(x 2)+20(x+1)2
9(x+1)2 (x 2)
0
3
9x5 + 18x4 54x3 36x2 + 117x 54 9 (x + 3) (x + 2) (x 1)
) 2 0) 2 0
9 (x + 1) (x 2) 9 (x + 1) (x 2)
3
P5 (x) (x + 3) (x + 2) (x 1)
La solucin de esta desigualdad = 2 0, la hallamos mediante el mtodo
Q3 (x) (x + 1) (x 2)
de intervalos:
x 1 3 2 1 1 2 +1
(x 1)3 0 + +
x+2 0 + + + +
x+3 0 + + + + +
2
(x + 1) + + + 0 + + +
x 2 0 +
P5 (x)
+ + + +
Q3 (x)
Conjunto solucin: x 2] 1; 3] [ [ 2; 1[[] 1; 1][]2; +1[:

Practico lo que aprend:

Utilizando el mtodo de los intervalos, resolver las siguientes desigualdades:

3x 1 x+3 x2 3x x2 1 2
1. a) > 0, b) 2 0, c) > 0, d) 0, e) 0
4x2 1 9x 1 x2 + 7x + 12 2x (x 3) x (x2 1)
7.7. SOLUCIN DE ALGUNAS ECUACIONES E INECUACIONES IRRACIONALES 253

2x 1 2x x+1 x 1
2. a) < 0, b) 2 > 0, c) < 0, d) 2 3 <0
x3 (x2 4) x 9 (x 1) (x + 5) (x + 3) (x 5)

x 3 2x + 3 2x + 1 2x 1
3. a) 0, b) 0, c) < 0, d) 2 <0
(x 1) (x + 5) (x2 x) (x2 + 3x) x (x 6) x (x + 6)

3x 5 3x x+3 2x 1
4. a) > 0, b) 2 < 0, c) 0, d) >0
x (x2 7x) x 81 x (x 2) (x + 1) (x2 x) (2x + 5)

x+4 x+2 x x
5. a) 2 2 > 0, b) 2 0, c) 0, d) 0:
(x2 x) (2x + 5) (2x2 + x) (x3 + x2 ) x2 4 x2 8

7.7. Solucin de Algunas Ecuaciones e Inecuaciones Irracionales

Por desigualdad irracional se entiende a aquella desigualdad en la cual la incgnita (o sus expresiones) se
p p p
encuentran bajo el signo de uno o varios radicales. As, 3x 1
r x + 2 > 3, 4 2x r1 1 3
5x 1,
2x 1 p 2x 1
1 < x, son ejemplos de desigualdades irracionales; 2x 3 3x + 2 = 0, 3 = 2x,
p 3x + 2 p 3x + 2
4
12x 1 1 = 3 5x 1, son ejemplos de ecuaciones irracionales.

En este punto damos una breve explicacin mediante varios ejemplos de como resolver ecuaciones y desigual-
dades irracionales sencillas.

Muchas desigualdades y ecuaciones irracionales pueden ser transformadas a desigualdades y ecuaciones


racionales mediante la elevacin de ambos miembros de la desigualdad a cierta potencia adecuada. Para la
solucin de las desigualdades y ecuaciones irracionales, primero es necesario averiguar el conjunto de valores
admisibles c:v:a: que posee la desigualdad o ecuacin, es decir el conjunto de nmeros reales para los cuales
la desigualdad o ecuacin tienen sentido en el campo de los nmeros reales. Luego de esto es necesario tener
en cuenta que cuando elevamos a una potencia impar ambos miembros de la desigualdad (o ecuacin) siempre
se obtiene una desigualdad (o ecuacin) equivalente a la desigualdad (o ecuacin) inicial. Si ambos miembros
de la desigualdad los elevamos a una potencia par, obtendremos una desigualdad equivalente a la inicial que
tiene el mismo signo de la desigualdad solo en el caso en que ambos miembros de la desigualdad inicial no son
negativos.

Ejemplos:

p
1. Resolver la desigualdad 2x + 5 < 2x 1
Solucin: El conjunto de los valores admisibles de x se calcula a partir de las desigualdades (su interseccin)
(2x + 5 0) ^ (2x 1 0) que nos da que el c:v:a: = fx 5=2g \ fx 1=2g = fx 1=2g.
Ahora, la desigualdad original es equivalente a la interseccin de las soluciones de las siguientes desigual-
dades: 8
>
< 2x + 5 0
p
2x + 5 < 2x 1 ) 2x 1 > 0 )
>
: 2
2x + 5 < (2x 1)
8 8
>
< x 5=2 >
< x 5=2
) x > 1=2 ) x > 1=2 )x>2
>
: >
: 1
2x2 3x 2 > 0 x 2] 1; 2 [[]2; +1[

Por tanto la solucin es x > 2 o x 2]2; +1[:


p
2. Resolver la desigualdad x2 1>x 2
254 CAPTULO 7. ECUACIONES E INECUACIONES

Solucin:
2 8
>
< x2 1 > 0
6
p 6 x 2 0
6 >
:
2)6
2
x2 1 > x 6 ( x2 1 > (x 2) )
6
4 x2 1 0
x 2<0
2 8
>
< x 2] 1; 1[[]1; +1[
6
6 x 2
6 >
:
) 6
6 ( 4x 5 > 0
6
4 x 2] 1; 1] [ [1; +1[
x<2

2 8
>
< x 2] 1; 1[[]1; +1[
6 "
6 x 2
6 > x 2
6 : x > 5=4 )
6 ( x 2] 1; 1] [ [1; 2[
6
4 x 2] 1; 1] [ [1; +1[
x<2
"
x 1
)
x 1
"
x 1
La solucin general es o x 2] 1; 1] [ [1; +1[, o x 2 Rn] 1; 1[:
x 1
p p
3. Resolver la desigualdad 2x + 1 > 3 x
Solucin:
8
>
< 2x + 1 > 0
p p
2x + 1 > 3 x) 3 x 0 )
>
:
2x + 1 > 3 x
8
>
< x > 1=2
x 3 ) 2=3 < x 3
>
:
x > 2=3

La solucin general es 2=3 < x 3 o x 2]2=3; 3]:


p p
4. Resolver la desigualdad 22 x 10 x > 2
Solucin:
8
>
< 22 x>0
p p
22 x > 10 x + 2 ) 10 x 0 )
>
: p 2
22 x> 10 x+2
8 8
>
< x < 22 >
< x < 22
) x 10 ) x 10 ) 6 < x 10
>
: p >
:
10 x < 2 x>6

La solucin general es 6 < x 10 o x 2]6; 10]:


p p
5. Resolver la desigualdad 3 x 1 < x + 2 + 2
p p p p
Solucin: 3 x 1 < x + 2 + 2 ) x 1 < (x + 2) x + 2 + 6x + 12 + 12 x + 2 + 8
(
p x+2 0
) 5x 21 < (x + 14) x + 2 ) p
5x 21 < (x + 14) x + 2
7.7. SOLUCIN DE ALGUNAS ECUACIONES E INECUACIONES IRRACIONALES 255
(
x 2
) )x 2
x2R

La solucin general es x 2 o x 2 [ 2; +1[:


p p
3
6. Resolver la ecuacin x2 4=3 = x3 2x

Solucin: 8
> 2
p p < x 4=3 0
3 3
x2 4=3 = x3 2x ) x 2x 0 )
>
: 2 3 2
x 4=3 = x3 2x

8 " p
>
> x 2= 3
>
> p
8 p p >
> x 2= 3
>
> "
>
< x 2= 3 x + 2= 3 0 >
< p
p p x 2
) x x 2 x+ 2 0 ) p
>
: >
> 2 x 0
4 2 64 > "
3x 27 = 0 >
>
>
>
> x = 4=3
>
: x = 4=3
) x= 4=3

La solucin es x = 4=3. Slo este valor est contenido en cada uno de los 3 conjuntos disyuntivos.

Observacin. En los anteriores ejemplos, as como en los siguientes ejercicios, consideramos que las races
p p
son aritmticas, por ejemplo 4 = 2 (y no 4 = 2).

Practico lo que aprend:

Hallar el c:v:a: de las ecuaciones. Comprobar si x0 , x1 , x2 son o no soluciones:

p
1. x2 + 3 = 2; x0 = 1, x1 = 0, x2 = 1:
p
2. x (x + 1) = 0; x0 = 1, x1 = 1=2, x2 = 0:
p p
3. x x + 1 = 0; x0 = 1, x1 = 1=2, x2 = 0:
p p
4. x2 1= x 1; x0 = 1, x1 = 0, x2 = 1:
q p
2
5. (x + 1) x2 (x 2) = jxj jx + 1j x 2; x0 = 1, x1 = 0, x2 = 2:
p
6. x x 5 = 2x; x0 = 0, x1 = 1, x2 = 9:
p
7. x x + 5 = 2 jxj ; x0 = 9, x1 = 1, x2 = 0:

Por razonamiento, explicar por que las siguientes ecuaciones no poseen solucin:

p p p p p p p
1. 2 x= 1, x+2 x + 3 = 2, 1 x+ x 1 = 1, 2x + 5 + x + 2 = 0:
p p p
3
p p p p
3
p
2. 5 x x 6 = 2, x 4= 1 x, 100
x+ x + 5 = 2, x + 1=x = x 1:
p p p p p 2 p
3. 10 + 3 x2 1 + x4 5 x = 3, 5 x+ x 2 = (x 1) (x 6), (x + 2) (4 x) x 7 + 1 = 5.
256 CAPTULO 7. ECUACIONES E INECUACIONES

7.8. Inecuaciones con dos Incgnitas y su Interpretacin Geomtrica

Supongamos que tenemos cierta desigualdad con dos incgnitas F (x; y) 7 0. El conjunto de soluciones de
tal desigualdad es un conjunto M de pares ordenados de nmeros reales (x; y) 2 M R2 .

La unin del conjunto de soluciones de las desigualdades F (x; y) > 0, F (x; y) < 0 y de la ecuacin F (x; y) = 0
puede dar el conjunto de todos los puntos del plano R2 . As, por ejemplo, las soluciones de las desigualdades
x x x
> 0, < 0 y de la ecuacin = 0 dan un conjunto de puntos M R2 :
y y y
| Si la ecuacin (relacin implcita) F (x; y) = 0 tiene una nica solucin respecto a la variable y (x se
considera parmetro, o tambin al contrario, x es la variable e y es el parmetro), entonces esta solucin y = f (x)
(o x = g(y)) que cumple con la identidad F (x; f (x)) 0 (o F (g (x) ; y) 0) se considera una funcin dada
implicitamente mediante F (x; y) = 0.

| Ciertas conjuntos M R2 denidos por las desigualdades que tienen la forma y > f (x) se encuentran
arriba de la lnea y = f (x), y los conjuntos dados por las desigualdades y < f (x) se encuentran bajo la lnea
y = f (x). Los conjuntos M R2 se los representa sombreando las partes del plano representadas por las
desigualdades y > f (x), x > g(y) (o y < f (x), x < g(y)).

| Con ejemplos representaremos las partes del plano R2 dadas por las desigualdades
n tipo F (x; y) ? o0
2 2
y por su frontera de ecuacin F (x; y) = 0. Por ejemplo, los conjuntos M0 = (x h) + (y k) = r2 ,
n o n o
2 2 2 2
M1 = (x h) + (y k) < r2 y M2 = (x h) + (y k) > r2 representan a una circunferencia con
centro en el punto P (h; k) y radio r; al interior de la circunferencia (crculo sin frontera)
n y al exterior de lao
circunferencia (exterior del crculo sin frontera), respectivamente. El conjunto M = (x h) + (y k) ? r2
2 2

puede representar (segn se use o ) al crculo interior o exterior ms su circunferencia frontera.

| As mismo, el conjunto el M0 = y = ax2 + bx + c representa una parbola general, y en particular, el


conjunto M0 = fy = a (x x1 ) (x x2 )g representa la grca de una parbola que pasa por los puntos (races)
del eje real x = x1 y x = x2 y posee vrtice hacia abajo cuando a > 0, y hacia arriba cuando a < 0. El conjunto
M1 = fy < a (x x1 ) (x x2 )g representa los puntos interiores a la parbola, pero sin frontera. El conjunto
M2 = fy > a (x x1 ) (x x2 )g representa los puntos exteriores a la parbola, pero sin frontera. Finalmente
el conjunto M = fy ? a (x x1 ) (x x2 )g representa los puntos interiores o exteriores a la parbola ms su
frontera (segn se use o ).

| Igualmente, el conjunto M0 = fy = ax + bg representa la grca de una lnea recta de pendiente m =


tan = a y ordenada al origen b. Esta recta divide a todo el plano en 2 semiplanos colocados a cada uno de los
lados de la recta: los semiplanos M1 = fy < ax + bg y M2 = fy > ax + bg que no incluyen a la recta frontera.
El conjunto M = fy ? ax + bg representa a uno de los semiplanos ms su recta frontera (segn se use o ).

Ejemplos:

n o
2
1. El conjunto (x 2) + y 2 16 representa el exterior del crculo de radio r = 4 con centro en el punto
n o
2
P (2; 0) y el borde es la circunferencia (x 2) + y 2 = 16 (ver Figura 7.9).
Solucin:
(
y x2 4x
2. El sistema de desigualdades representa al sector comprendido entre la parbola y =
y x
x2 4x y la recta y = x ms sus fronteras.
Solucin: Ver Figura 7.10.

3. La desigualdad del tipo jxj + jyj < a, a > 0 (por ejemplo, jxj + jyj < 3) representa el interior de un rombo
7.8. INECUACIONES CON DOS INCGNITAS Y SU INTERPRETACIN GEOMTRICA 257

n o
2
Figura 7.9: Regin sombreada y frontera: M = (x 2) + y 2 16

Figura 7.10: Regin sombreada frontera: M = y x2 4x; y x

de vrtices en los puntos (a; 0), (0; a), ( a; 0) y (0; a), sin inclur la frontera jxj + jyj = a, e incluyendo
la frontera si la desigualdad es del tipo jxj + jyj a.
Solucin:La frontera del rombo viene dada mediante la relacin jxj + jyj = a, y representa un conjunto

Figura 7.11: Regin sombreada M = fjxj + jyj c = fjxj + jyj = ag :


ag y frontera M

de 4 rectas (ver Figura 7.11): x + y = a, si x 0, y 0; x + y = a, si x < 0, y 0; x y = a,


si x < 0, y < 0; x y = a, si x 0, y < 0. Estas rectas se obtienen tambin utilizando la denicin
de valor absoluto aplicado a cada cuadrante: I cuadrante (x 0; y 0); II cuadrante (x < 0; y 0); III
cuadrante (x < 0; y < 0); IV cuadrante (x 0; y < 0).

4. Hallar la solucin grca del sistema de desigualdades con 2 variables


8
>
> x+y <3
>
< 2x + 3y < 6
>
> 4x y < 4
>
:
y 3
258 CAPTULO 7. ECUACIONES E INECUACIONES

Solucin: Primeramente determinamos la frontera de la porcin M R2 , es decir las ecuaciones de las


rectas: x + y = 3, 2x + 3y = 6, 4x y = 4, y = 3 (ver Figura 7.12). Despus de trazar las rectas-
frontera, determinamos los semiplanos cuyos puntos satisfacen a cada una de las desigualdades dadas:
x+y < 3, 2x+3y < 6, 4x y < 4, y 3 (cada una de las desigualdades, geomtricamente, representa
un semiplano con ( , ) o sin (<, >) frontera). Para determinar el semiplano cuyos puntos satisfacen
a la desigualdad, tomamos un punto no perteneciente a la frontera (en nuestro caso no perteneciente a
la recta) y lo reeplazamos en la desigualdad. Si el punto tomado satisface a la desigualdad, entonces esa
porcin donde est el punto corresponde al semiplano buscado, caso contrario, es la otra porcin. La
solucin general corresponde a la interseccin de todos los respectivoas semiplanos:

Figura 7.12: Sector solucin sombreado M R2 (interseccin de los 4 semiplanos).

Practico lo que aprend:

Gracar los conjuntos M R2 que satisfacen a los sistemas de desigualdades:

8 8 x y 8 x y 8 x y
> >
> >
> >
>
< x+y 4 < 4 +9 <1 < +
6 10
2 < 7 8
1
1. a) x 0 , b) x>0 , c) x<0 , d) x 0
>
: >
> >
> >
>
y 0 : y<0 : y 0 : y 0
8 x y 8 x y 8 x y
8 >
> + <1 >
> + <1 >
> + <1
> >
> 3 y3 >
> 8x 6y >
> 8x 6y
< x y 12 < x < <
+ <1 + <1 + <1
2. a) x>1 , b) 6 6 , c) 6 6 , d) 6 6
>
: >
> x>0 >
> 6<x<8 >
> 6<x<8
y 1 >
> >
> >
>
: : :
y>0 y 0 y 0
8 8
( > >
y (x 4) (x + 6) < y (x 1) (x + 8) < 6<x+y 6
3. a) , b) y x+8 , c) 4<x y 5
y x+6 >
: >
:
6<x<0 5 x<4
( ( (
2 x + y 25 jx + 4j + jy 8j 12 jx + 3j + jy + 5j 18
4. a) , b) , c)
6 x 1 4<x y 4 jx 1j + jy + 2j 5

7.9. Conjuncin y Disyuncin de Desigualdades Numricas

Es muy comn el encontrarse con desigualdades del tipo a > b y c > d o del tipo a > b o c > d, es
decir con proposiciones unidas mediante los conectivos y (^ de conjuncin) u o (_ de disyuncin).

Denicin 7. Se denomina conjuncin de desigualdades a aquella proposicin compuesta de desigualdades


unidas mediante el conectivo ^ (y). Una conjuncin de desigualdades se considera verdadera, si es verdadera
7.9. CONJUNCIN Y DISYUNCIN DE DESIGUALDADES NUMRICAS 259

Figura 7.13: Izq. El conocido matemtico e historiador de la ciencia, el francs Josphep Louis Bertrand (Pars, 1882 -
Pars, 1900). Dado que fue un nio prodigio, a la edad de 11 aos y 5 meses se le permiti asistir a la escuela Politcnica
de Pars, donde naliz sus estudios en 1839. Sus principales trabajos versan sobre anlisis matemtico (criterios de
convergencia de series), teora de nmeros (postulado de Bertrand), geometra diferencial (curvas de curvatura doble),
teora de grupos y cculo de probabilidades (paradoja de Bertrand). Der. El genial lgico y matemtico britnico,
George Boole (Lincoln, 1815 - Cork, Irlanda, 1864). Fue un asombroso autodidacta, fundador de su propia escuela de
enseanza bsica. Combin el lgebra y el clculo y desarroll una lgebra especial de la lgica, el lgebra de Boole, como
rama de las matemticas. Introdujo la lgica simblica para representar procesos de razonamiento mediante smbolos
matemticos. Sin tener ttulo universitario fue profesor del Queens College de Cork. Su lgebra de Boole se aplica
a cualquier conjunto o retculo y posee valiossimas aplicaciones a la lgica, conmutacin telefnica, probabilidades y
computadores electrnicos. Aparte de sus estudios y textos sobre ecuaciones diferenciales y ecuaciones en diferencias,
public entre otras obras su famosa Investigacin sobre las leyes del pensamiento.

cada una de las desigualdades que intervienen en la conjuncin. Caso contrario, la conjuncin de desigualdades
es falsa.
8
< p1 7 q 1
>
| Una conjuncin de desigualdades ser representada con ayuda de la llave , donde p1 7 q1
>
:
pn 7 q n
es la primera desigualdad y pn 7 qn es la n-sima desigualdad.

Ejemplos:

8
>
< a>b
1. La proposicin a > b y c > d, y e < f es la conjuncin de desigualdades c>d
>
:
e<f
8
>
< 15 > 14
2. La conjuncin de desigualdades >1 es verdadera ya que cada una de las desigualdades que
>
:
6 < 12
intervienen es verdadera.
8
>
< 13 >p5 p
3. La conjuncin de desigualdades 2 < 2 es falsa, pues la desigualdad 2 < 2 es falsa.
>
:
2 3

Denicin 8. Se denomina disyuncin de desigualdades a aquella proposicin compuesta de desigualdades


unidas mediante el conectivo _ (o). Una disyuncin de desigualdades se considera verdadera, si es verdadera
por lo menos una de las desigualdades que intervienen en la disyuncin. Caso contrario, la disyuncin de
desigualdades es falsa.
260 CAPTULO 7. ECUACIONES E INECUACIONES
2
p1 7 q 1
6
Una disyuncin de desigualdades la representaremos con un corchete 4 ; donde p1 7 q1 es la
pn 7 q n
primera desigualdad y pn 7 qn es la n-sima desigualdad.

Ejemplos:

"
a>b
1. La proposicin a > b o c < des una disyuncin de desigualdades y se la representa mediante :
c<d
2
12 > 14
6
2. La disyuncin de desigualdades 4 3 < 7 es verdadera, pues contiene a la desigualdad verdadera 3 < 7,
8 5
es decir, por lo menos una de las desigualdades es verdadera.
"
10 > 5
3. La disyuncin de desigualdades es falsa, pues sus 2 desigualdades son falsas.
3<0
"
0 < 12
4. La disyuncin de desigualdades p es verdadera, pues sus desigualdades son ambas verdaderas.
3> 2

| Las desigualdades que intervienen en una conjuncin o disyuncin de desigualdades las denominaremos
componentes de la conjuncin o disyuncin. No se descartan los casos en que tengamos combinaciones de
conjunciones y disyunciones compuestas, stas se denominan construcciones de desigualdades.

Ejemplos:

2 (
a1 > b1
6
1. La construccin 4 a2 > b2 es una disyuncin, compuesta de una 1ra componente que es la conjuncin
a3 > b3
(
a1 > b1
y de una 2da componente que es la desigualdad a3 > b3 :
a2 > b2
8 "
>
> a1 > b1
>
>
< a2 > b2
2. La construccin ( es una conjuncin, compuesta de un 1er componente que es la disyuncin
>
> a3 > b 3
>
>
: a4 > b4
" (
a1 > b1 a3 > b3
y de una 2da componente que es la conjuncin :
a2 > b2 a4 > b4

Los valores de verdad (verdadero o falso) que pueda tomar una construccin depender de los valores de
verdad que puedan tomar las distintas componentes y sus combinaciones.

Ejemplos:

8
>
< " 13 > 5
1. La construccin 4<1 es verdadera como conjuncin de la desigualdad verdadera 13 > 5 y la
>
:
17 > 9
"
4<1
disyuncin verdadera :
17 > 9
7.9. CONJUNCIN Y DISYUNCIN DE DESIGUALDADES NUMRICAS 261
2
13 > 14
6 "
2. La construccin 4 2>3 es verdadera como disyuncin de la desigualdad falsa 13 > 14 y la disyun-
1<6
"
2>3
cin verdadera :
1<6
8 "
>
> 4> 1
>
> "
< 3<3 4> 1
3. La construccin " es falsa como conjuncin de la disyuncin verdadera y la
>
> 1 > 3 3<3
>
>
: 6< 1
"
1>3
disyuncin falsa :
6< 1

| Lo anterior se generaliza al caso en que tengamos conjunciones y/o disyunciones con desigualdades e
igualdades o sus combinaciones.

Ejemplos:

" (
a=b a=b
1. Las construcciones y son una disyuncin y conjuncin de igualdades, respectivamente.
c=d c=d
2 8
a>b >
< a>b
6
2. As mismo, las construcciones 4 c = d y c = d son una disyuncin y conjuncin de igualdades y
>
:
e=f e<f
desigualdades, respectivamente.

7.9.1. Simplicaciones y Observaciones


(
a>b
| Es evidente que la conjuncin equivale a la desigualdad doble b < a < c, y la conjuncin
a<c
(
a>b
equivale a la desigualdad doble b < a 6 c:
a6c
" "
a>b a<b
| Anlogamente, la disyuncin equivale a la desigualdad a > b, y la disyuncin equivale
a=b a=b
a la desigualdad a 6 b:

| Las desigualdadesdel tipo a < b y c < d se denominan desigualdades estrictas, y las desigualdadesdel
tipo a > b y c 6 d se denominan
" desigualdades no estrictas. Es evidente que la expresin de desigualdad a 6= b
a>b
equivale a la disyuncin :
a<b

Las conjunciones y disyunciones ya analizadas para valores numricos, tambin son aplicables a desigualdades
con una o varias variables. Analizando la desigualdad (x + 3)(x + 2) > 0 vemos que esta se cumple en los 2
casos:
(x + 3 > 0 y x + 2 > 0) o (x + 3 < 0 y x + 2 < 0)
Por lo tanto, para encontrar la solucin de una desigualdad es necesario analizar sus formas proposicionales
(predicados) unidos mediante varios conectivos como y(conjuncin ^) u o(disyuncin _). La desigualdad
anterior tomara la forma equivalente

f[(x + 3 > 0) ^ (x + 2 > 0)] _ [(x + 3 < 0) ^ (x + 2 < 0)]g


262 CAPTULO 7. ECUACIONES E INECUACIONES

Ejemplos:

1. La forma proposicional del tipo desigualdad


( x + 5 > 0 y desigualdad x 1 < 0 es una conjuncin de
x+5>0
desigualdades, representada mediante .
x 1<0
| El dominio de denicin de una conjuncin de desigualdades viene a ser la interseccin de los dominios
de denicin de sus componentes.
(
x 1>3
2. El dominio de denicin de la conjuncin p viene a ser fx > 9g, pues ambas desigualdades
x>3
se satisfacen al mismo tiempo para x > 9:
8
>
< 2x 1 > 3
3. El dominio de denicin de la conjuncin 4x 12 viene a ser f2 < x 3g, pues para estos valores
>
:
3x + 1 < 13
reales se satisfacen al mismo tiempo todas las desigualdades.
8 (
>
< 2x 8
4. La conjuncin de desigualdades x 6<0 representa una doble conjuncin. Su dominio de deni-
>
:
x 2 0
cin es f4 x < 6g, valores que se obtienen intersecando los 3 dominios de denicin de las 3 desigualdades.
(
x 1>3
5. El dominio de denicin de la conjuncin viene a ser el conjunto vaco ;, pues no existe
4x 12
interseccin entre los 2 dominios de denicin de las 2 desigualdades.

Llegamos a la siguiente

Denicin 11. Se denomina solucin de una conjuncin de desigualdades al conjunto de valores de


las variables que es solucin y de la primera componente de la conjuncin, y de la segunda componente de la
conjuncin, y de la tercera, etc. Es decir, si ese conjunto es solucin al mismo tiempo de todas las componentes
de la conjuncin.

Para comprobar si un conjunto de valores es solucin de una conjuncin de desigualdades, es suciente


reemplazar estos valores en todas y cada una de sus componentes y establecer la veracidad o falsedad de la
conjuncin obtenida de desigualdades numricas.

Los valores obtenidos de las variables deben pertenecer a cada uno de los dominios de denicin de cada
una de las componentes.

Resolver una conjuncin de desigualdades signica hallar el conjunto de todas sus soluciones.

Ejemplos:

(
2x > 3y
1. Sea la conjuncin de desigualdades . Los valores x = 1, y = 1 pertenecen al dominio
log x > 1 + y
de denicin tanto de la primera fx
( 2 R; y 2 Rg, como de la segunda desigualdad fx 2 R+ ; y 2 Rg.
2>3
Colocando estos valores tendremos , que representa una conjuncin falsa de desigualdades
log 1 > 2
numricas, pues log 1 = 0.
Si colocamos los valores x = 10 e y = 20, tambin pertenecientes
( a los dominios de denicin,
20 > 60
obtendremos la conjuncin verdadera de desigualdades numricas .
log 10 > 19
7.9. CONJUNCIN Y DISYUNCIN DE DESIGUALDADES NUMRICAS 263

Por lo tanto, la pareja x = 1; y = 1 no es solucin de la conjuncin dada, y la pareja x = 10, y = 20 si


es una de las soluciones.
(
x>y 5
2. La conjuncin de desigualdades posee un conjunto demasiado amplio (innito) de soluciones
y x2
(ver Figura 7.14), por ejemplo, todas las parejas del tipo (x; y), donde x = 0 e y 2 [0; 5[ tambin
son soluciones. Se comprueba facilmente que las parejas (0; 0), (0; 1), (0; 1;1), (0; 3), (0; 4;25), etc. son
algunas de las soluciones de la conjuncin dada.

(
x>y 5
Figura 7.14: Conjunto solucin de la conjuncin (interseccin) de desigualdades.
y x2

Denicin 11. Se denomina solucin de una disyuncin de desigualdades al conjunto de valores de


las variables que es solucin o de la primera componente de la conjuncin, o de la segunda componente de la
conjuncin, o de la tercera, etc. Es decir, si ese conjunto es solucin por lo menos de una las componentes de
la disyuncin.

Para comprobar si un conjunto de valores es solucin de una disyuncin de desigualdades, es suciente


reemplazar estos valores en todas y cada una de sus componentes y establecer la veracidad o falsedad de la
disyuncin obtenida de desigualdades numricas.

Los valores obtenidos de las variables deben pertenecer a cada uno de los dominios de denicin de cada
una de las componentes.

Resolver una disyuncin de desigualdades signica hallar el conjunto de todas sus soluciones.

Ejemplos:

"
2x > 1 + 5y
1. Sea la disyuncin de desigualdades . Los valores x = 1; y = 1 pertenecen al dominio de
3x > 4y
denicin tanto de la primera
" fx 2 R; y 2 Rg, como de la segunda desigualdad fx 2 R; y 2 Rg. Colocando
2>6
estos valores tendremos , que representa una disyuncin falsa de desigualdades numricas, pues
3>4
las 2 digualdades son falsas. Si colocamos los valores x = 5 e y = 2, tambin pertenecientes" a los
10 > 9
dominios de denicin, obtendremos la disyuncin verdadera de desigualdades numricas ,
15 > 8
pues las 2 desigualdades son verdaderas. Por lo tanto, la pareja x = 1; y = 1 no es solucin de la
disyuncin dada, y la pareja x = 5, y = 2 si es una " de las soluciones. Anlogamente, para la pareja
2>2
x = 1; y = 2=5 tendremos la disyuncin verdadera , pues la primera desigualdad es falsa y la
3 > 8=5
segunda verdadera.
264 CAPTULO 7. ECUACIONES E INECUACIONES
"
x>y 5
2. La disyuncin de desigualdades posee un conjunto demasiado amplio (innito) de soluciones
y 2
(ver Figura 7.15), por ejemplo, todas las parejas del tipo (x; y), donde x = 0 e y 2 [0; 5[ tambin son
soluciones. Se comprueba facilmente que las parejas (0; 0), (0; 1), (3; 1), (0; 3), etc. son algunas de las
soluciones de la disyuncin dada. Tmese en cuenta que soluciones, por ejemplo, podran ser todas las
parejas (x; y), con y, tales que y 2; o todas las parejas (x; y) tales que x > y 5; o todas las parejas
(x; y) tales que satisfacen las desigualdades x > y 5 y y 2. La solucin general la representamos
mediante el siguiente grco, en donde la parte sombreada representa al innito nmero de puntos de la
solucin.

"
x>y 5
Figura 7.15: Conjunto solucin de una disyuncin (unin) de desigualdades
y 2

Practico lo que aprend:

Hallar los dominios de las siguientes conjunciones y disyunciones de desigualdades. Gracarlos


en R:

8 8 8 8
> 2x 1 > 7 > 2x < 12 >
> 2x + 1 > 7 > 4x 3 > 9
>
< x 1 >
< x 3 >
< >
<
x 1 x 1
1. a) 2 , b) 1 , c) > 2 , d) 1
> x3 5
>
:
>
>
: x 5 >
>
> 3x >
>
: x 5
x<2 x< 2 : 2x < 1 2x < 1
3 2 4 4
2 2 2 2
x 1>8 2x 2 < 12 2x 1 7 5x 6 > 9
6 2x 1 6 x+3 6 x 2 6 x 1
6
2. a) 6
4 2 , b) 6
4 1 , c) 6 > 2 , d) 6
4 1
x4 x 5 x 4 3x 3 x3
x< 3 + < 2 x< 1 x<1
2 2 3 5 3
8 8 8 8
> 3 2x > 2 > 3x 12 >
> 2x 3 > 7 > 4x + 3 9
>
< x 1 >
< x+3 >
< >
<
x 3 3x 1
3. a) 1 , b) 1 , c) > 2 , d) 1
5 3
>
>
: x x< 1
>
>
: x2 >
>
> 3x >
>
: x2
x<2 : 3x < 1 x< 1
3 4 5 4

7.10. Construccin de Desigualdades

Cuando se trata de resolver un sinnmero de desigualdades, intervienen las intersecciones (conjuncin) o


uniones (disyuncin) de las soluciones parciales, dando lugar a las as denominadas construcciones de desigual-
dades, para las que si no se tiene cuidado, le llevan confusiones y conclusiones equivocadas. As por ejemplo,
al resolver la desigualdad x2 + 3x + 2 > 0, tendremos:
7.10. CONSTRUCCIN DE DESIGUALDADES 265

(x + 1)(x + 2) > 0 , f(x + 1 > 0) ^ (x + 2 > 0)g _ f(x + 1 < 0) ^ (x + 2 < 0)g ;
donde el smbolo ^ (conjuncin) signica y, el smbolo _ (disyuncin) signica o. Sin embargo, la resolu-
cin anterior, con ayuda de las operaciones de conjuncin y de disyuncin, puede ser reducida a la siguiente
construccin equivalente de desigualdades:

2 (
x+1>0 "
6
6
6 ( x+2>0 )
x+2>0
;
6 x+1<0 x+1<0
4
x+2<0
donde el corchete [::: signica una disyuncin (unin, o) de desigualdades, y las llaves f::: una conjuncin
(interseccin, y) de desigualdades.

Al utilizar conjunciones y disyunciones de desigualdades es posible construr desigualdades compuestas que


pueden ser resueltas por pasos, yendo desde el interior al exterior de la construccin.

Ejemplos:

1. Observar la siguiente construccin de desigualdades:


2 ( 2 (
5>3 V "
6 6
6 6
6 ( 31 < 4 )6 (
F
)
F
) V;
6 3<8 6 V V
4 4
2 0 V

donde mediante V y F asignamos los valores de verdad de las conjunciones y disyunciones numricas.
Recordamos que:
| Una conjuncin de valores es verdadera, si y solamente si, son verdaderas todas sus componentes al
mismo tiempo.
| Una disyuncin de valores es verdadera, si y solamente si, es verdadera por lo menos una de sus
componentes.

2. Analizar la siguiente construccin de desigualdades, si x = 1 e y = 3


2 (
x+y >3
6
6
6 ( 3x < 4
6 x+y <8
4
x 0

Solucin: Para x = 1 e y = 3, tenemos


2 ( 2 (
x+y >3 4>3 "
6 6
6 6
6 ( 3x < 4 )6 (
3<4
)
V
) V;
6 x+y <8 6 4<8 V
4 4
x 0 1 0

por lo tanto, los valores x = 1 e y = 3 son una de las soluciones de la construccin de desigualdades de
partida.

Si representamos mediante A1 , A2 , . . . , An distintas construcciones de desigualdades (Ai = Ai (x1 ; x2 ; : : :)),


entonces tenemos las siguientes deniciones:
266 CAPTULO 7. ECUACIONES E INECUACIONES

Denicin 12. El conjunto de valores 8 de las variables x1 = a1 , x2 = a2 , . . . , xn = an , se denomina


>
> A1
>
< A
2
solucin de la construccin de desigualdades , si este conjunto es solucin de la componente A1 , y de la
>
>
>
:
An
componente A2 , ..., y de la componente An . Es decir, de todas al mismo tiempo.

Denicin 13. El conjunto de valores de las variables x1 = a1 , x2 = a2 , . . . , xn = an , se denomina


2
A1
6 A
6 2
solucin de la construccin de desigualdades 6 , si este conjunto es solucin de la componente A1 , o de la
4
An
componente A2 , ..., o de la componente An . Es decir, de por lo menos de una de las construcciones Ai .

| Resolver una construccin de desigualdades signica encontrar el conjunto de todas sus soluciones.

Ejemplo:
2 (
5x + 1 > 2
6
6 x y<2
El valor x = a ser solucin de la construccin de desigualdades 6 6
( , si x = a es solucin
4 2x + y > 3
3y > 1
" (
5x + 1 > 2 2x + y > 3
disyuncin de desigualdades , o solucin de la conjuncin de desigualdades . A
x y<2 3y > 1
"
5x + 1 > 2
su tiempo x = a ser solucin de la disyuncin de desigualdades , si x = a es solucin de las
x y<2
(
2x + y > 3
desigualdades (3x + 1 > 2) o (x y < 2), y solucin de la conjuncin si x = a es solucin de las
3y > 1
desigualdades (2x + y > 3) y (3y > 1).
8
>
> A1
>
< A
2
| El dominio de denicin de la conjuncin es la interseccin de los dominios de denicin de las
>
> A::::
>
:
An
componentes A1 ; A2 ; : : : ; An .
2
A1
6 A
6 2
| El dominio de denicin de la disyuncin 6 es la unin de los dominios de denicin de las
4 A::::
An
componentes A1 ; A2 ; : : : ; An .

Ejemplos:

(
5x + 10 < 0
1. Hallar el dominio de denicin de la construccin de desigualdades
3x 6 > 0:

( (
5x + 10 < 0 5x < 10
Solucin: Dom = Dom =R
3x 6 > 0 3x > 6
7.10. CONSTRUCCIN DE DESIGUALDADES 267
8 (
>
> log x + 1 > 2
>
>
< x>1
2. Hallar el dominio de denicin de las desigualdades "
>
> 2x > 1
>
>
: x<3
(
log x + 1 > 2
Solucin: Dom =] 1; 0[= R es el dominio de denicin de la conjuncin y
x>1
"
2x > 1
Dom = R de la disyuncin. Por tanto, el dominio de denicin de toda la construccin, que
x<3
es una conjuncin, son todos los nmeros reales positivos R+ .
2 8
< 1 >1
6
6 : x
6
3. Hallar el dominio de denicin de la construcin de desigualdades 6 " x > 2
6 log x > 5
4
p
x<3
8
< 1 >1
Solucin: Dom x =] 1; 0[[]0; +1[= R n f0g es el dominio de denicin de la conjuncin y
: x> 2
"
log x > 5
Dom p = [0; +1[= R+ [ f0g de la disyuncin. Por tanto, el dominio de denicin de toda la
x<3
construccin, que es una disyuncin, son los nmeros R n f0g [ R+ [ f0g, es decir, todos los reales R.

| No se debe confundir el dominio de denicin de una construccin de desigualdades con el conjunto de


valores admisibles de cada una de sus componentes, ni tampoco con el conjunto solucin de la construccin.

Denicin 14. Dos construcciones de desigualdades son equivalentes, si cualquier solucin de una de ellas,
tambin es solucin de la otra, y viceversa.

7.10.1. Propiedades de las Construcciones de Desigualdades e Igualdades

Recuerda las propiedades fundamentales de las construcciones de desigualdades:

1. Si una conjuncin posee una componente Ai idnticamente falsa (F ), entonces la conjuncin es identica-
mente falsa (solucin ?).

Ejemplo:
(
x>3
) ?; pues la componente x2 < 0 es idnticamente falsa (F ) :
x2 < 0

2. Si una conjuncin posee una o varias componentes Ai idnticamente verdaderas, entonces la conjuncin
puede ser reducida a las restantes, siempre y cuando el dominio de denicin de stas sea igual al dominio
de denicin de la conjuncin original.

Ejemplos:

8
>
< x > 15
2.1 2
x +2>0 ) fx > 15 , pues las componentes x2 + 1 > 0 y 3x < 0 son idnticamente
>
:
3x < 0
verdaderas.
268 CAPTULO 7. ECUACIONES E INECUACIONES
8
< x+1>0 1
2.2 1 ; fx + 1 > 0 , pues a pesar de que la componente 2 > 0 es idnticamente verdadera,
: > 0 x
x2
el dominio de la desigualdad
8 x + 1 > 0 es ms amplio que el de la conjuncin original. La forma
(
< x+1>0 x+1>0
correcta sera poner: 1 ) :
: >0 x 6= 0:
x2

3. Si una disyuncin posee una o varias componentes Ai idnticamente falsas (F ) (solucin ?), entonces la
disyuncin se reduce a las restantes componentes.

Ejemplo:
2
5x > 3
6
4 x2 < 0 ) [5x > 3 , pues las componentes x2 < 0 y sin x > 7 son idnticamente falsas (F ) :
sin x > 7
2
A1
6
4. Si una disyuncin 4 : : : contiene componentes idnticamente verdaderos, entonces es equivalente a stas
An
componentes, siempre y cuando su dominio de denicin coincida con el dominio de denicin de toda la
disyuncin.

Ejemplos:

"
x2 + 1 > 0
4.1 ) x2 + 1 > 0
7x 3 0
2 2 2
1 1 1
4 2
> 0 4 2
> 0 4 >0 1
4.2 x ) x . Sin embargo x2 no es equivalente a 2
> 0.
x+4 0 x 6= 0 x+4 0 x

5. Propiedades conmutativas para la conjuncin y disyuncin:


( ( " "
f1 (x) 7 0 f2 (x) 7 0 f1 (x) 7 0 f2 (x) 7 0
a) ) ; b) )
f2 (x) 7 0 f1 (x) 7 0: f2 (x) 7 0 f1 (x) 7 0

6. Propiedades asociativas para la conjuncin y disyuncin:

8 8 ( 8
>
< (f1 (x) 7 0 >
< f1 (x) 7 0 < f1 (x) 7 0
>
a) Para la conjuncin f2 (x) 7 0 ) f2 (x) 7 0 ) f2 (x) 7 0
>
: >
: >
:
f3 (x) 7 0 f3 (x) 7 0 f3 (x) 7 0
2 2 " 2
f1 (x) 7 0 f1 (x) 7 0 f1 (x) 7 0
6 " 6 6
b) Para la disyuncin 4 f2 (x) 7 0 )4 f2 (x) 7 0 ) 4 f2 (x) 7 0
f3 (x) 7 0 f3 (x) 7 0 f3 (x) 7 0

7. Propiedades distributiva entre la conjuncin y la disyuncin:

2 (
8 f1 (x) 7 0
> f1 (x) 7 0
< " 6
6 f (x) 7 0
a) f2 (x) 7 0 )6 ( 2 es la propiedad distributiva entre la conjuncin y disyuncin.
> 6 f1 (x) 7 0
: 4
f3 (x) 7 0
f3 (x) 7 0
7.10. CONSTRUCCIN DE DESIGUALDADES 269
2 (
f1 (x) 7 0
6
6 ( f3 (x) 7 0
6
8 "
> f 1(x) 7 0 6 f1 (x) 7 0
>
> 6
>
< 6
f 2(x) 7 0 6 f (x) 7 0
b) " ) 6 ( 4 es la propiedad distributiva entre la conjuncin y varias
> f 3(x) 7 0 6 f2 (x) 7 0
>
> 6
>
: 6
f 4(x) 7 0 6 ( f3 (x) 7 0
6
6 f2 (x) 7 0
4
f4 (x) 7 0:
disyunciones.
8 "
2 >
>
> f1 (x) 7 0
f(1 (x) 7 0 >
<
6 f (x) 7 0
c) 4 f2 (x) 7 0 ) " 2 es la propiedad distributiva entre la disyuncin y la conjun-
>
> f1 (x) 7 0
f3 (x) 7 0 >
>
: f3 (x) 7 0:
cin.

| Todas las propiedades anteriores son muy similares y equivalentes a las propiedades entre los conjuntos,
as, por ejemplo, la ltima propiedad puede ser escrita en la forma:
2 2
f1 (x) 7 0
( A
6 6 (
4 f2 (x) 7 0 ) 4 B ) A [ (B \ C) ) (A [ B) \ (A [ C) ;
| {z }
f3 (x) 7 0 C Propiedad distributiva

por lo tanto, cualquier propiedad de construccin entre desigualdades puede ser demostrada, simplicada o
transformada utilizando las propiedades y operaciones entre conjuntos.

Ejemplos:

(
3x = 5
1. El predicado (3x = 5) ^ (x + y = 7) equivale a la construccin
x+y =7
"
2x + y = 1
2. El predicado (2x + y = 1) _ (x y = 6) equivale a la construccin
x y=6
2 (
x=2+y (
6 x=2+y
3. La construccin 4 x 2 y es una disyuncin de la conjuncin y de la ecuacin
2
x 5 y;
x +1=5
x2 + 1 = 5.
8 2
>
> x
(+ 3 > 2y
>
< 6
4 3x + 2 = 2y
4. La construccin mixta de conjunciones y disyunciones es una construccin a 3
>
> x 8
>
:
x + y = 10
niveles (existen 2 llaves y 1 corchete!) de igualdades y desigualdades.

Practico lo que aprend:

Reducir y expresar las siguientes construcciones de conjunciones y disyunciones en forma


equivalente mediante llaves y corchetes:

1. a) (x 2 > 3x) ^ (x=2 x 5), b) (x=3 1 > x) _ (3x=2 x< 2), c) (x=3 > 3x) ^ (x=2 2 5x)

2. a) (x=4 1 > 2x)_(x=3 2x 1), b) (x=2 5 > x=3)_(3x x=2 2), c) (x=2 > x)^(x=2 1 3x)
270 CAPTULO 7. ECUACIONES E INECUACIONES

3. ((x > 3) ^ (x=3 2 0)) _ ((x=3 1 > x) _ (3x=2 x< 2))

4. ((2x > 1) _ (x=2 2 0)) ^ (( x=3 1> x) _ (x=2 x=3 < 4))

x 1 x 1 x x x
5. > 3 _ 2 0 _ 1>x ^ < 2
2 2 3 4 2

2 x 3x x x x 3 x 5x x
6. < 2+ _ 2 ^ 1 _ 4
3 2 2 3 3 2 3 2

2x 1 x x 1 x 1 3x x
7. > 2 x _ 1 _ > ^ < 4
4 2 3 2 5 2

2 x 3x x x 2x 3 x 5x 1
8. <1 _ 5 ^ 1 _ x :
3 2 2 3 3 2 3 2

7.10.2. Construccin de Ecuaciones y su Equivalencia

A continuacin exponemos, sin demostracin, los principales criterios de equivalencia entre ecuaciones:

1. Las ecuaciones f (x1 ; : : : ; xn ) = g(x1 ; : : : ; xn ) y f (x1 ; : : : ; xn )+h(x1 ; : : : ; xn ) = g(x1 ; : : : ; xn )+h(x1 ; : : : ; xn )


son equivalentes siempre y cuando posean el mismo dominio de denicin.
Ejemplos:

1.1 Las ecuaciones x2 = x + 2, x2 + x 2 = 0 son equivalentes.


p p p p
1.2 x2 + x 1 = x 1 y x2 + x 1 x 1 = 0 son equivalentes, pero no equivalen a x2 = 0.

2. Las ecuaciones f (x1 ; : : : ; xn ) = g(x1 ; : : : ; xn ) y f (x1 ; : : : ; xn )h(x1 ; : : : ; xn ) = g(x1 ; : : : ; xn )h(x1 ; : : : ; xn )


son equivalentes, si poseen el mismo dominio de denicin y la desigualdad h(x1 ; : : : ; xn ) 6= 0 es idntica-
mente verdadera en ese dominio.
Ejemplo:

2.1 La ecuacin 3 log2 x = 8 log x no es equivalente a la ecuacin 3 log x = 8, pues los dominios de
denicin de stas 2 ecuaciones son idnticos (R+ =]0; +1[), pero para x = 1, log x = 0, y por tanto
la desigualdad log x 6= 0 no es idnticamente verdadera en R+ =]0; +1[:
(
f (x1 ; x2 ; :::; xn ) f (x1 ; x2 ; :::; xn ) = 0
3. La ecuacin = 0 y la conjuncin son equivalentes.
g(x1 ; x2 ; :::; xn ) g(x1 ; x2 ; :::; xn ) 6= 0

Ejemplos:
(
5x + 1 5x + 1 = 0
3.1 Se cumple la equivalencia =0)
x x 6= 0
(
7x2 x2 = 0
3.2 Se cumple la equivalencia =0)
x x=6 0
(
2
x (2x 3) (x + 1) x (2x 3) = 0
3.3 Se cumple la equivalencia =0)
x+1 x 6= 1
(
x 3 x 3=0
3.4 Se cumple la equivalencia 2 =0)
x (x + 2) (x 6= 0) ^ (x 6= 2)
7.11. INECUACIONES ALGEBRAICAS CON UNA INCGNITA 271

4. La ecuacin del tipo producto f1 (x1 ; : : : ; xn )f2 (x1 ; : : : ; xn ) fk (x1 ; : : : ; xn ) = 0 equivale a la disyuncin
2
f1 (x1 ; : : : ; xn ) = 0
6 f (x ; : : : ; x ) = 0
6 2 1 n
de ecuaciones 6
4
fk (x1 ; : : : ; xn ) = 0

Ejemplos:
2
x+1=0
6
4.1 Se tiene la equivalencia (x + 1)(x 3)(x 4) = 0 ) 4 x 3 = 0
x 4=0
2
x+1=0
6
4.2 No se tiene la equivalencia (x + 1)(x 3) log x = 0 ; 4 x 3 = 0 , pues el dominio de denicin
log x = 0
de la ecuacin es R+ =]0; +1[, y el dominio de denicin de la disyuncin es todo R. Si tomamos,
por ejemplo, el valor x = 1 veremos que este es solucin de la disyuncin, pero
8 no es solucin de la
>
> x>0
< 2 x+1=0
>
ecuacin. Sin embargo, se cumple la equivalencia: (x + 1)(x 3) log x = 0 )
> 6
> 4 x 3=0
>
:
log x = 0:

Practico lo que aprend:

"
x+y >5
1. Mostrar si los valores x = 2, y = 15 son o no soluciones de la disyuncin p
2x + 3y < x
(
x + lg (1 x) < 3
2. Mostrar si el valor x = 10 es o no solucin de la conjuncin
2x + 4 > 5:
8 (
>
> x+3>5
>
>
< 2x 1 < 3
3. Establecer si el valor x = 5 es o no una solucin de la construccin "
>
> x 4<2
>
>
: x 5<3
8 "
>
> 3x + 5 > 2
>
>
< x<1
4. Mostrar si x = 5 es o no solucin de la construccin (
>
> 3 (x 1) > 2
>
>
: x+1> 6
2 (
x 14 > 2
6
5. Establecer si el valor x = 5;5 es o no solucin de la construccin 4 x + lg x < 3 , si el dominio de
3x 4 > 5
denicin de las 3 componentes son los naturales N:

7.11. Inecuaciones Algebraicas con una Incgnita

| Analizamos la desigualdad de primer grado ax > b, y de acuerdo a los valores que toman los coecientes
(parmetros) a, b, tendremos las equivalencias:
272 CAPTULO 7. ECUACIONES E INECUACIONES

2 (
a>0
6
2 ( 6 x > b=a
6 (
a>0 6 a<0
6 6
6 ax > b 6
6 ( 6 x < b=a
6 6 8
6 a<0 6 > a=0
ax > b ) 6 )6 <
6 ax >b 6
6 ( 6 b<0
6 6 >
:
4 a=0 6 1<x<1
6 8
ax > b 6 > a=0
6 <
6
4 b>0
>
:
F

Ejemplos:

1. Resolver 5x 7 < 3 (x + 1)
Solucin: 5x 7 < 3 (x + 1) ) 5x 7 < 3 (x + 1) ) 2x < 10 ) x < 5 ) x 2] 1; 5[

2. Resolver (a + 1)x + 3 < 5a + 1:


Solucin: (a + 1)x + 3 < 5a + 1 ) (a + 1)x < 5a 2)
2 ( 2 ( 2 (
a+1>0 a> 1 a> 1
6 6 5a 2 6 5a 2
6 (a + 1)x < 5a 2 6 x< 6
6 ( 6 ( a+1 6 ( x < a+1
6 a+1<0 6 a < 1 6 a< 1
6 6 6
)6 )6 5a 2
)6 5a 2
6 6 6
6 ( (a + 1)x < 5a 2 6 ( x > a+1 6 ( x > a+1
6 a+1=0 6 a= 1 6 a= 1
4 4 4
(a + 1)x < 5a 2 0x < 7 F (?)

7.11.1. Solucin de Conjunciones de desigualdades

Ahora analizamos una conjuncin de desigualdades de primer grado:


8
>
> x > a1
>
< x>a
2
| Si a1 > a2 > ::: > an , entonces se cumple la equivalencia ) x > an para la conjuncin de
>
>
>
:
x > an
desigualdades.
8
>
< x> 2
Ejemplo: Se cumple la equivalencia x>5 ) x > 5, pues 5 > 3 > 2:
>
:
x>3
8
>
> x < b1
>
< x<b
2
| Si b1 > b2 > : : : > bn ; entonces se cumple la equivalencia ) x < bn para la comjuncin de
>
>
>
:
x < bn
desigualdades.
8
>
< x< 2
Ejemplo: Se cumple la equivalencia x < 3 ) x < 3, pues 2 > 2 > 3:
>
:
x<2
7.11. INECUACIONES ALGEBRAICAS CON UNA INCGNITA 273
8
>
> x > a1
>
>
>
>
>
> (
< x>a x>a
n
| Si a = max fa1 ; a2 ; :::; an g y b = m n fb1 ; b2 ; :::; bk g, entonces ) , que es una
>
> x < b1 x<b
>
>
>
>
>
>
:
x < bk
(
x>a
equivalencia, donde es evidente que la conjuncin de desigualdades posee solucin, si y solamente, si
x<b
la desigualdad numrica a < b es verdadera. Para a b la conjuncin dada es falsa (?).

Ejemplos:

8
>
> x 3>5
>
< 2x 3>1
1. Resolver la conjuncin de desigualdades
>
> x 1<2
>
:
x<4
8 8
>
> x 3>5 >
> x>8 (
>
< 2x >
<
3>1 x>2 x>8
Solucin: ) ) ) F (?)
>
> x 1<2 >
> x < 3 x < 3
>
: >
:
x<4 x<4

1 1
2. Hallar el dominio de denicin de la funcin y = lg (3x 1) + p +
2 3x x
8 8
>
< 3x 1>0 >
< x > 1=3
Solucin: Tenemos la conjuncin 3x 2>0 ) x > 2=3 ) x > 2=3 ) x 2 [2=3; +1[
>
: >
:
x 6= 0 x 6= 0
( (
x=a x=a
| Observacin: Se cumple la siguiente equivalencia )
x=b a=b
(
x + 5a 1=0
3. Resolver la conjuncin de ecuaciones
x = 3a (x + 2)

Solucin: Tenemos las equivalencias


( ( (
x + 5a 1=0 x = 1 5a x = 1 5a
) )
x = 3a (x + 2) x = 23 a 1 1 5a = 3a=2 1

( (
x = 1 5a x = 7=13
) )
a = 4=13 a = 4=13

7.11.2. Solucin de Disyunciones de Desigualdades


2
x > a1
6 x>a
6 2
| Si a1 > a2 > ::: > an ; entonces se cumple la equivalencia 6 ) x > an para la disyuncin de
4
x > an
desigualdades.
274 CAPTULO 7. ECUACIONES E INECUACIONES
2
x < b1
6 x<b
6 2
| Si b1 > b2 > ::: > bk , entonces 6 ) x < b1 es una equivalencia para la disyuncin de desigual-
4
x < bk
dades.
2
x > a1
6
6 ::::::: "
6
6 x > an x>a
| Si a = m n fa1 ; a2 ; :::; an g y b = max fb1 ; b2 ; :::; bk g, entonces 6
6 x<b ) , que es una
6 1 x<b
6
4 :::::::
x < bk
equivalencia.

Ejemplos:

"
x > 3x + 1
1. Resolver la disyuncin de desigualdades
x < 2x
" " "
x > 3x + 1 2x > 1 x < 12 1
Solucin: ) ) ) x 2] 1; 2 [[]0; +1[
x < 2x x<0 x>0
"
(3a + 1) x 2 < x + 3
2. Resolver la disyuncin de desigualdades con parmetro
2 + 3ax > 14
" "
(3a + 1) x 2 < x + 3 ax < 53
Solucin: ) )
2 + 3ax > 14 ax > 4
2 2 (
a>0
6 6 5
6 6 2 8
6 6 ( x < 3a > a 2 8
6 6 a=0 < ">0 > a
6
6
6
6
6
6 x< 5 < ">0
6 6 5 6 > 3a 6 5
6 6 ( 0x < 3 6 : x> 4 6 >
6 :
x < 3a
6 6 a<0 6 8 a 6 x > a4
6 4 6 6 (
6 6 > a
"=0
6 5
x > 3a 6 < 6
6 a=0
6 2 ( )6 1<x<1 )6
6 a>0 6 > 6 1<x<1
6 6 : 6 8
6 6 6 F 6 >
6 6 4 6 8
6 6 ( x> a 6 > a
6 < a
"<0
6 6 a=0 6 < "<0 6
4 5
x > 3a
6 6 6 5 >
6 6 4 > x > 3a :
6 6 x < a4
6 6 ( 0x > 4 :
x < a4
6 6 a<0
4 4
x < a4

Practico lo que aprend:

Resolver las construcciones de conjunciones y disyunciones. Ayudarse de grcas en el eje


numrico R:

1. a) (x 2 > 3x) ^ (x=2 x 5), b) (x=3 1 > x) _ (3x=2 x< 2), c) (x=3 > 3x) ^ (x=2 2 5x)

2. a) (x=4 1 > 2x)_(x=3 2x 1), b) (x=2 5 > x=3)_(3x x=2 2), c) (x=2 > x)^(x=2 1 3x)

3. a) ((x 2 > 0) ^ (x=2 x 0)) _ (x=2 5 > x=3), b) ((x=3 > 2) ^ (3x 4 < 0)) _ (2x > 1)

4. a) (x 3> 2x) ^ (x=4 2x 1), b) (x=3 2 > 0) _ (3x=4 2x < 0), c) (x=3 > 3x) ^ (x=2 2 0)
7.11. INECUACIONES ALGEBRAICAS CON UNA INCGNITA 275

5. a) (x=2 3> 3x)_(x=3 x 1), b) (x=2 2 > x=4)_(3x x=3 1), c) (x=2 > 2x)^(x=2 1 x)

6. a) ((x 4 > 0) ^ (x=3 2x 0)) _ (x=3 5 > x=2), b) ((x=4 > 2) ^ (3x 1 < 0)) _ (x > 1)

Figura 7.16: Izq. El matemtico y mecnico ruso Nikolai G. Chebotaryov (Kamentse Podolskom, 1894 - Kazn, 1947).
Estudios e investigaciones sobre algebra (funciones y nmeros algebraicos, grupos de Galois), teora de nmeros (resolvi
el problema de Frobenius como generalizacin del teorema de Dirichlet sobre los nmeros primos en las progresiones
aritmticas), clculo variacional y geometra (demostracin de la hiptesis de T. Klauzen sobre la cuadratura de lnulas).
Se le considera como el fundador de la escuela algebraica moderna rusa. Der. El matemtico norteamericano Paul Cohen
(Log Brench, 1934 - ). Investigaciones y trabajos sobre la fundamentacin de la matemtica, teora de conjuntos y lgica
matemtica. Mostr la imposibilidad de demostrar la hiptesis del continuo (no existe un conjunto cuyo cardinal sea
estrictamente superior a card(N) y estrictamente inferior a card(R)) partiendo de la axiomtica de los conjuntos de
Zermelo-Frenkel. Por sus trabajos recibi el premio y medalla Fields (1966).

7.11.3. Solucin de Inecuaciones Racionales

Toda desigualdad racional puede ser reducida a la forma equivalente


(
f (x) f (x)g(x) 7 0
70) ;
g(x) g(x) 6= 0

donde la desigualdad f (x)g(x) 7 0 puede ser resuelta por el ya analizado mtodo de los intervalos.

Ejemplos:

3x + 4
1. Resolver la desigualdad racional x >
x+3
Solucin: Tenemos las equivalencias
3x + 4 3x + 4 x2 4
x> )x >0) >0
x+3 x+3 x+3
"
3<x< 2
) x2 4 (x + 3) > 0 )
2<x<1

1
2. Resolver la desigualdad racional 1:
x
Solucin: Tenemos las siguientes equivalencias
(
1 1 1 x x(1 x) 0
1) 1 0) 0)
x x x x 6= 0
276 CAPTULO 7. ECUACIONES E INECUACIONES
8 "
> x 0 "
< x<0
) x 1 )
>
: x 1
x 6= 0

Practico lo que aprend:

Utilizando la metodologa anterior, resolver las siguientes desigualdades racionales:

x 4 3x 1 x+4 x+4 x 2 x+6


1. a) x> , b) x, c) > 1, d) > 2, e) 1 > , f) 0 >
x+3 x 2 2x + 3 3x 1 5x 1 x 2
4 1 x+4 x+4 2 x
2. a) x+1> , b) > x, c) > 1, d) 2, e) 1 > , f) 0 >
x+3 x 2 2x x 1 5x 1 x 2
x 3x x+4 x+4 x x 4
3. a) 1> , b) < x + 1, c) 1, d) > 2, e) 1 , f ) 2x >
x+3 x 2 2x + 3 x 4 5x 1 x 2
4 1 x x+4 x 3 2 x+6
4. a) x + 1 > , b) > , c) > 1, d) 2, e) x > , f) 0 >
x x 2 3 x x 5x 1 x 2

7.12. Introduccin a las Inecuaciones Irracionales

7.12.1. Solucin de algunas Inecuaciones Irracionales (con un solo radical de ndice


par)

Vamos a analizar algunas desigualdades irracionales simples en el campo de los nmeros reales R. La
desigualdad g(x) > 0; se denomina irracional, si g (x) es una funcin algebraica irracional. Por ejemplo, la
p
funcin y = (x2 + 1)2 no es irracional ya que su ley de correspondencia equivale a la funcin y = x2 + 1:

Para resolver una desigualdad irracional, se deben tener en cuenta las siguientes condiciones:

La expresin subradical bajo un radical de ndice par debe ser no negativa.


p
2k
Los valores de las expresiones con radical par deben ser no negativas, es decir, f 0:

Transformaciones Equivalentes Para resolver las desigualdades o ecuaciones irracionales se utilizan las
siguientes transformaciones equivalentes, aplicables al caso en que se tenga un solo radical de ndice par:
8 8
> > 2n 2n
< f (x) > g(x) < f (x) > g (x)
1. f (x) > 0 ) f (x) > 0
>
: >
:
g(x) 0 g(x) 0
8
>
< f (x) 0
p
2k
2. f (x) < g(x) ) g(x) > 0
>
:
f (x) < g 2k (x)
8
>
< f (x) 0
p
2k
3. f (x) = g(x) ) g(x) 0
>
:
f (x) = g 2k (x)
2 8
>
< g(x) 0
6
6 f (x) > 0
p 6 >
:
4. 2k f (x) > g(x) ) 6
6 ( (x) > g (x)
f 2k
6
4 g (x) < 0
f (x) 0
7.12. INTRODUCCIN A LAS INECUACIONES IRRACIONALES 277

Ejemplos:

p
1. Resolver la desigualdad 2x + 5 < 2x 1:
8 8
> > 5
p < 2x + 5 0 < x 2
Solucin: 2x + 5 < 2x 1 ) 2x 1 0 ) x > 12
>
: >
:
2x + 5 < (2x 1)2 (x 2)(x + 21 ) > 0
2 (
8 x > 21
> x > 1
6 "
< " 2 6 x > 2 x>2
) x>2 )6 6
( ) )x>2
>
: 4 x> 2 1 F (?)
x < 12
x < 12
p
2. Resolver la desigualdad x2 1>x 2:
Solucin: Tenemos las equivalencias
2 8 2 8
>
< x2 1 > 0 >
< (x 1)(x + 1) > 0
6 6
p 6 x 2 0 6 x 2
6 >: 6 >
:
x2 1 > x 2 ) 6 6 ( x2 1 > (x 2)2 )6
6 ( 4x 5 > 0
6 6
4 x2 1 0 4 (x 1)(x + 1) 0
x 2<0 x<2
2
x 2 "
p 6 x 1
Resumiendo tendremos x2 1>x 2)4 x 1 )
x 1
1 x<2
p
3. Resolver la desigualdad x2 + x + 1 > 1 x x2 :
p
Solucin: Representando x2 + x + 1 con la letra t, tendremos
( x2 + x = t2 1, donde t 0. Tendremos
p t 0
la siguiente equivalencia x2 + x + 1 > 1 x x2 )
t > 2 t2 :
8
( >
t 0
0 < t"
) ) t>1 ) t > 1:
(t 1)(t + 2) > 0 >
:
t< 2
"
p 2 2 x< 1
Por lo tanto x2 + x + 1 > 1 x x )x +x+1>1)
x>0

Practico lo que aprend:

Explicar si las siguientes desigualdades son o no equivalentes:

1 1
1. a) (x + 2) > (2x 4) y x + 2 > 2x 4, b) 3 (2x 7) < 3 (x + 5) y 2x 7 > x + 5
3 3
1 1 1 1
2. a) x + 1 + <4+ y x + 1 < 4, b) x + 2 < x+5 y x+2< x+5
x 6 x 6 x 1 x 1
p
3. a) x2 > 1 y x 2 1 > 0, (x 1) > 0 y x (x 1) > 0, b) x 1 > 0 y x 1 > 0, c) x2 > x4 y x2 < 1
x 3
4. a) x x2 + 1 > 2 x2 + 1 y x 2 > 0, b) x2 < x4 y x2 > 1, c) < 0 y (x 3) (x + 2) < 0
x+2
x+4 2 2
5. a) 0 y (x + 4) (x 1) 0, b) (3x 1) < (x + 3) y 3x 1 < x + 3, c) jx + 1j > jxj y
x 1
2
(x + 1) > x2
278 CAPTULO 7. ECUACIONES E INECUACIONES
q p
2 2
6. a) (2x + 1) > 1 y j2x + 1j > 1, b) j1 xj 1 y 1 x 1, c) (x 1) < x2 y x 1 < 2x
q q
2 2 1
7. a) (x 3) > (2x + 1) y jx 3j > j2x + 1j, b) x5 > 1 y x > 1, c) 1 y 1 x, d) x2 > 1 y
x
x>1
1 1 x2 p
8. a) > 2 y x2 + x + 1 > x2 + 1, b) 0 y x 0, c) 3x 2 1 y 3x 2 1.
x2
+1 x +x+1 x
p p p
9. a) x + 2 3 y x + 2 9, b) 2x 3 2 y 2x 3 4, c) x2 + 2 < 2 y x2 + 2 < 4:
p
p p p p x (x 1) x 1
10. a) x2 (x 1) < 1 y jxj x 1 < 1, b) x2 + x > x + x y x2 > x, c) >0 y > 0:
x 4 x 4
Explicar si las siguientes desigualdades son o no equivalentes en los conjuntos dados:
1 1 1 1
11. a) x2 x y x2 x, X = fx : x 6= 0g : b) x2 x y x2 x, X = fx : x > 0g :
x x x x
p p
12. a) x + 1 > x y x + 1 > x2 , X = fx : x 0g : b) x + 1 > x y x + 1 > x2 , X = fx : x 1g :
p p
13. a) x2 > 1 y x > 1, X = fx : x 0g : b) x 1< 2 x y x 1<2 x, X = fx : x 2 [1; 2]g :
1 1
14. a) > 1 y 1 > x + 2, X = fx : x > 2g : b) > 1 y 1 > x + 2, X = fx : x < 2g :
x+2 x+2
p p p p p p p
15. a) x x 1 x2 x + x < 2x + 3 y x < 2x + 3, X = fx : x 1g : b) 2 x > 21 x y x > 1 x,
X = fx : x 2 Rg :
p p p p
16. a) x (x + 2) > x (2x 5) y x + 2 > 2x 5, X = fx : x > 0g : b) x (x + 2) > x (2x 5) y
x + 2 > 2x 5, X = fx : x 0g :
Resolver grcamente en R2 los siguientes sistemas de desigualdades:
( ( ( (
3x + 4y > 12 2x y 4 x+y <5 x y 3
17. a) , b) , c) , d)
x y>5 x + 2y 2 3x y > 6 2x + 3y 6
8 "
8 >
> x>1
( > ( >
>
1<x<2 < y(> 3x + 2 2x y 10 < x< 1
18. a) , b) y < x+1 , c) , d) (
2<y<3 >
: x 0 >
> y>1
x> 2 >
>
: x+y < 1
8 (
2 >
> x+y >1
x 2
+y <4 >
>2 " (
( < x+y < 1 2x y < 5 x + 2y < 17
6 "
19. a) 4 x>y , b) , c) , d)
>
> x y>1 3x + y > 9 x 3y > 9
x< y >
>
: x 2y < 1

7.13. Ecuaciones e Inecuaciones con Valor Absoluto

En esta seccin presentamos ecuaciones e inecuaciones con valor absoluto, es decir, casos en los que las
variables o incgnitas estn bajo el signo de uno o varios valores absolutos j:j. Estudiaremos la metodologa
para resolver este tipo de ecuaciones e inecuaciones y transformaremos las mismas a sistemas de ecuaciones e
inecuaciones equivalentes, pero sin valor absoluto. Se analizarn ejemplos y ejercicios de diversa dicultad.

Antes de pasar a los mtodos de"solucin de ecuaciones y desigualdades con valores absolutos, recordamos
a; si a 0
la denicin de valor absoluto jaj = :
a; si a < 0
7.13. ECUACIONES E INECUACIONES CON VALOR ABSOLUTO 279

Utilizando las operaciones de conjuncin y de disyuncin tendremos la equivalencia


2 (
" a 0
6
y = a, si a 0 6 y=a
y = jaj ) o 6
6
(
y = a, si a 0 4 a 0
y= a

Anlogamente, para el caso de una expresin o funcin tendremos la equivalencia


2 (
f (x1 ; x2 ; : : : ; xn ) 0
6
6 y = f (x1 ; x2 ; : : : ; xn )
y = jf (x1 ; x2 ; : : : ; xn )j ) 6
6
(
4 f (x1 ; x2 ; : : : ; xn ) 0
y = f (x1 ; x2 ; : : : ; xn )

Figura 7.17: Izq. El famoso fsico y matemtico britnico Paul Dirac (Bristol, 1902 - Tallahassee, Florida, 1984). De-
sarroll una formulacin matemtica muy elegante de la Mecnica Cuntica que la modic para incorporar la Mecnica
Ondulatoria de Schrodinger (Viena, 1887 - 1961) y la teora de la Relatividad de Einstein (Ulm, 1879 - Princeton,
1955). Hizo algunas predicciones sobre la existencia de nuevas micropartculas. Arm que toda partcula debe tener
su antipartcula y predijo la existencia (?) del monopolo magntico. Junto con Enrico Fermi (Roma, 1901 - Chicago,
1954) desarroll la estadstica Fermi-Dirac que predice el comportamiento de un sistema de partculas. En 1933, Dirac
y Schrodinger recibieron el Nobel de Fsica por sus trabajos sobre mecnica cuntica. Der. El matemtico italiano,
Ulises Dini (Pisa, 1845 - 1918). Discpulo de los matemticos franceces Joseph L. Bertrand (Pars, 1882 - 1900) y Charles
Hermite (Dieuze, 1822 - Pars, 1901). Trabajos sobre geometra diferencial, anlisis matemtico y funciones devariable
real, teora de funciones analticas y cilndricas. Son conocidos sus teoremas sobre convergencia uniforme de series y su
criterio de Dini sobre convergencia de series de Fourier.

Ejemplo:

2 (
x 1
6
6 y =x+1
y = jx + 1j ) 6
6
(
4 x 1
y = (x + 1)

7.13.1. Solucin de Ecuaciones y Inecuaciones con Valor Absoluto

Pasamos a analizar las desigualdades que contienen valores absolutos ya sea para las variables o para sus
miembros.
280 CAPTULO 7. ECUACIONES E INECUACIONES

1. Para la desigualdad jf (x; y)j < g(x; y) tenemos la equivalencia


(
f (x; y) < g(x; y)
jf (x; y)j < g(x; y) )
f (x; y) > g(x; y)

2. Para la desigualdad jf (x; y)j > g(x; y) tenemos la equivalencia


(
f (x; y) > g(x; y)
jf (x; y)j > g(x; y) )
f (x; y) < g(x; y)

3. La ecuacin del tipo jf (x; y)j = g(x; y) no posee soluciones cuando g(x; y) < 0 (solucin ?). Para otros
valores de g(x; y) tenemos la equivalencia

8
>
< g(x;
" y) 0:
jf (x; y)j = g(x; y) ) f (x; y) = g(x; y)
>
:
f (x; y) = g(x; y)

Figura 7.18: Izq. El famoso lsofo y matemtico griego, Zenn de Elea (Elea, c. 495 - c. 430 a. C.), nacido en la Magna
Grecia (Italia meridional). Discpulo de Parmnides (Elea, c. 515 a. C. - 475 a. C.). Utiliz la lgica para demostrar que el
ser es nico y no mltiple, y que el movimiento es imposible, mediante sus 4 paradojas famosas: a) Paradoja de Aquiles
y la tortuga, b) Paradoja de la dicotoma, c) Paradoja de la echa, d) Paradoja del estadio. Platn (Atenas o Egina, c.
427 - c. 347 a. C.) y Aristteles (Estagira, 384 - Calcis, Eubea, 322 a. C.) intentaron sin xito total resolver las paradojas
de Zenn hasta que matemticos posteriores denieron el lmite, continuidad, las series (su convergencia) y el innito.
Der. El famoso mecnico, gran especialista en aerodinmica y matemtico ruso, Nicolai Zhukovskii (Orejov, Vladimirov,
1847 - Mosc, 1921). Trabajos, investigaciones y aplicaciones sobre ecuaciones diferenciales en derivadas parciales y sus
soluciones aproximadas, teora de funciones de variable compleja y perles de alas, teora de rbitas planetarias y de
cometas, hidrodinmica. Se le considera como el padre de la aviacin sovitica.

Ejemplos:

1. Resolver j2x 3j < 1:


( (
2x 3<1 x<2
Solucin: j2x 3j < 1 ) ) ) x 2 ]1; 2[ :
2x 3> 1 x>1

2. Resolver j3x + yj < 1


( (
3x + y < 1 y<
3x + 1
Solucin: j3x + yj < 1 ) )
3x + y > 1 y>
3x 1
( (
3x 1 < 3x + 1 1<1
) )
3x 1 < y < 3x + 1 3x 1 < y < 3x + 1
7.13. ECUACIONES E INECUACIONES CON VALOR ABSOLUTO 281

) 3x 1<y< 3x + 1
Construir la grca correspondiente!

3. Resolver j2x yj x<3


( (
2x y <x+3 y>x 3
Solucin: j2x yj x<3) )
2x y> x 3 y < 3x + 3
( (
x 3 < y < 3(x + 1) x 3 < y < 3(x + 1)
) )
x 3 < 3(x + 1) x> 3

4. Resolver j2x + 3j > 3


" "
2x + 3 > 3 x>0
Solucin: j2x + 3j > 3 ) )
2x + 3 < 3 x< 3

5. Resolver la desigualdad con 2 variables j2x + yj + y > 2


" "
2x + y > 2 y y> x+1
Solucin: j2x + yj + y > 2 ) )
2x + y < y 2 x< 1:
Construir la grca correspondiente!

6. Resolver jx j2x 3jj > 2


" "
x j2x 3j > 2 j2x 3j < x 2
Solucin: jx j2x 3jj > 2 ) )
x j2x 3j < 2 j2x 3j > x + 2
2 ( 2 (
2x 3<x 2 x<1 2
6 6 x>5
6 2x 3>2 x 6 x > 53
)6
6
" )6
6
" )4 1
4 2x 3>x+2 4 x>5 x<
3
2x 3< x 2 x < 1=3

7. Resolver la ecuacin jx 1j = 2x 1
Solucin: 8 8
>
< 2x
" 1 0 >
< x
" 1=2
2
jx 1j = 2x 1) x 1 = 2x 1 ) x=0 )x=
>
: >
: 3
x 1 = 1 2x x = 2=3

8. Resolver la ecuacin o relacin de 2 variables jxj + y = 2


2 ( 2 (
8 y 2 x 0
>
< 2
" y 0 6 6
6 x=2 y 6 y=2 x
Solucin: jxj + y = 2 ) x=2 y ) 6 6
( ) 66
( . La grca de
>
: 4 y 2 4 x 0
x=y 2
x=y 2 y =x+2
la relacin nos presenta 4 tringulos o semiplanos abiertos. La grca de jxj + y = 2 corresponde al
tringulo inferior abierto:

2.5

1.25

0
-2.5 -1.25 0 1.25 2.5

-1.25

-2.5

Relacin jxj + y = 2
282 CAPTULO 7. ECUACIONES E INECUACIONES

Practico lo que aprend:

1. Resolver las ecuaciones y inecuaciones con valor absoluto:

a) a) j3x 2j = 2x+3, b) 2 j7x 2j = 8 3 j2x 4j, c) 2 j3 2xj = 5 3x, d) 2 j2x 4j 7 = 3 j2x + 4j


b) a) j4x 1j = 2x 3, b) j2x 4j = 3 j2x + 4j, c) j2x 3j = 4x + 2, d) j7x 2j + 2x j2x 4j =
3 j2x + 4j 1, e) jx 2j x jx + 1j = 3 jx 1j 1, f ) jxj x jx + 1j = x + jx 1j + 1

2. Resolver las ecuaciones e inecuaciones con valor absoluto e interpretarlas como distancias
en R:

a) a) jx 3j = 5, b) jx + 2j = 1, c) jx 2j = 4, d) jx 1j = 1, e) j2 3xj = 5, f ) j3x 6j = 9,
g) j8 3xj = 4, h) j4x 1j = 10, i) j3x=2 1j = 1, j) j2 3x=4j = 2, k) j 2 3xj = 2, l)
j3 x=4j = 3:
b) a) j7x + 3j 2, b) jx 1j > x, c) j2 3xj 5x, d) j3 2xj 8x, j7x + 5j = 2, e) j1 xj > 2x,
f ) j1 4xj = x, g) j3 2xj = x, h) j3 x=2j x, i) j4 x=3j 4 x, j) jx=3 1j x, k)
j2 x=5j > 2x:

7.13.2. Desigualdades ms Generales con dos Variables y Valor Absoluto

A continuacin pasamos a ver algunos ejemplos y mtodos de resolucin de desigualdades con 2 variables y
valor absoluto basados en construcciones con disyunciones y conjunciones:

Ejemplos:

1. Resolver 3 jx + yj + 2 jx yj < 2
Solucin: 2 8
>
< x+y 0
6 x y 0
6 >
6 :
6 3(x + y) + 2(x y) < 2
6 8
6 > x+y 0
6 <
6
6 x y<0
6 >
6 : 3(x + y) 2(x y) < 2
3 jx + yj + 2 jx yj < 2 ) 6 8
6 >
6 < x+y <0
6
6 x y 0
6 >
6 : 3(x + y) + 2(x y) < 2
6 8
6 >
6 < x+y <0
6
4 x y<0
>
:
3(x + y) 2(x y) < 2
2 (
x y x
6
6 y< 5x + 2
6 8
6 >
6 < y x
6
6 y >x
6 >
:
6
)6 8 y < x=5 + 2=5
6 >
6 < y < x
6
6 y x
6 >
:
6
6
6 (y > x=5 2=5
4 x<y< x
y > 5x 2:
7.13. ECUACIONES E INECUACIONES CON VALOR ABSOLUTO 283

Trazando todas las lneas rectas frontera y hallando sus puntos de interseccin, tendremos los siguientes
sistemas de ecuaciones y los valores de los vrtices o intersecciones:
( ( ( (
y = x=5 + 2=5 y = x=5 + 2=5 y = x=5 2=5 y = x=5 2=5
; ; ;
y = 5x 2; y = 5x + 2; y = 5x + 2; y = 5x 2:

1 1 1 1 1 1 1 1
A ; ; B ; ; C ; ; D ; :
2 2 3 3 2 2 3 3
La grca de este conjunto viene representada por el interior (sin frontera) del rombo oblicuo ubicado en
el centro del siguiente grco (Figura 7.19):

Figura 7.19: Grca y relacin 3 jx + yj + 2 jx yj < 2

2. La solucin de la desigualdad anterior 3 jx + yj + 2 jx yj < 2, tambin puede ser presentada en la forma


2 (
1=2 < x 1=3
6
6 5x 2 < y < x=5 + 2=5
6 (
6 1=3 x 1=3
6
6
6
6 ( x=5 2=5 < y < x=5 + 2=5
6 1=3 < x < 1=2
4
x=5 2=5 < y < 5x + 2:
p
3. Describir y gracar las relaciones x2 + y 2 1, jxj + jyj 1, max fjxj ; jyjg 1 (Figura 7.20)

p
Figura 7.20: Relaciones mtricas: x2 + y 2 1, jxj + jyj 1, max fjxj ; jyjg 1:

n p o
4. Si se desea relacionar los conjuntos A = f(x; y) : jxj + jyj r; r 0g, B = (x; y) : x2 + y 2 r; r 0
y C = f(x; y) : max fjxj ; jyjg r; r 0g, representados en las 3 grcas anteriores para r = 1, pero con
un radio r 0, tendremos que si (x; y) 2 A, entonces jxj + jyj r, y
p p
x2 + y 2 x2 + 2 jxj jyj + y 2 = jxj + jyj r;

es decir, (x; y) 2 B, lo que a su vez implica que


p
max fjxj ; jyjg x2 + y 2 r;

que signica que (x; y) 2 C. Por lo tanto, llegamos a la relacin A B D.


284 CAPTULO 7. ECUACIONES E INECUACIONES

5. Resolver la desigualdad jy 2j + jxj < 2

Solucin: Tenemos las siguientes equivalencias

2 8 2 8
>
< y 2 >
< y 2
6 6
6 > x 0 6 > x 0
6 : 6 :
6 y 2+x<2 6 y < x+4
6 8 6 8
6 > y 2 6 > y 2
6 < 6 <
6 6
6 x<0 6 x<0
6 > 6 >
6 : y 2 x<2 6 : y <x+4
jy 2j + jxj < 2 ) 6 8
6 > y<2 )6
6
8
6 < 6 >
< y<2
6 6
6 x 0 6 x 0
6 > 6 >
6 : y+2+x<2 6 :
6 8 6 8 y>x
6 > 6 >
6 < y<2 6 < y<2
6 6
4 x<0 4 x<0
>
: >
:
y+2 x<2 y> x

2 8
>
< 2 y < x+4
6 2< x+4
6 > 2 (
6 : 2 y < x+4
6 x 0
6 8 6
6 > 6
6 < 2 y <x+4 6 ( 0 x<2 2 (
6 6 2 y <x+4 x<y <x+4
6 2<x+4 6
6 > 6 6
6 : x<0 6 2<x<0 6 2<x<0
)6 8
6 > )6
6
( )6
6
(
6 < x<y<2 6 x<y<2 4 x<y < x+4
6 6
6 x<2 6
6 > 6 ( 0 x<2 0 x < 2:
6 : x 0 6 x<y<2
6 8 4
6 >
6 < x<y<2 2<x<0
6
4 x<2
>
:
x<0

x+y
6. Resolver la desigualdad >2
x y
2
x+y
x+y >2
6 x y
Solucin: Tenemos las siguientes equivalencias >2)4 x+y
x y < 2
x y
" " "
x+y 3y x
x y >2 x y >0 (3y x)(x y) > 0
) x+y ) 3x y )
x y < 2: x y <0 (3x y)(x y) < 0

2 2 ( 2 ( 2 (
y > x=3 x=3 < y < x x=3 < y < x
6 6 6 6
6 6 y<x 6 x=3 < x 6
6 6 ( 6 ( 6 ( x>0
6 6 y < x=3 6 x < y < x=3 6 x < y < x=3
6 4 6 6
6 6 6
6 y>x 6 x=3 > x 6 x<0
)6
6
2 ( )6
6
( )6
6
(
6 y < 3x 6 x < y < 3x 6 x < y < 3x
6 6 6 6
6 6 6 6
6 6 ( y>x 6 ( x < 3x 6 ( x>0
6 6 y > 3x 6 3x < y < x 6 3x < y < x
4 4 4 4
y<x 3x < x x<0
7.13. ECUACIONES E INECUACIONES CON VALOR ABSOLUTO 285
2 8
>
< x
">0
6 x
6 > 3 <y <x
6 :
6 x < y < 3x
)6
6 8
6 > x
"<0
6 <
4 x < y < x3
>
:
3x < y < x:

Otro mtodo de solucin sera mediante las siguientes transformaciones equivalentes


(
x+y jx + yj jx + yj > 2jx yj
>2) >2)
x y jx yj x y 6= 0:
2 8
>
< x+y 0
6
6 > x y>0
6 :
6 x + y 2x + 2y > 0
6 8
6 > x+y 0
6 <
6
( 6 x y<0
6 > :
jx + yj 2jx yj > 0 6
) )6 8 x + y + 2x 2y > 0
x y 6= 0 6 > x+y <0
6 <
6
6 x y>0
6 >
6 : x y 2x + 2y > 0
6 8
6 >
6 < x+y <0
6
4 x y<0
>
:
x y + 2x 2y > 0
2 8
>
< y x
6
6 > y<x
6 :
6 y > x=3
6 8 2 8
6 > y >
< x
6 < x ">0
6 6 x=3 < y < x
6 y>x 6 >
6 >: 6 :
6 6 x < y < 3x
)6 8 y < 3x )6
6 > y< x 6 8
6 < 6 > x<0
6 6 < "
6 y<x 4 3x < y < x
6 > >
6 : y > 3x :
x < y < x=3
6 8
6 >
6 < y< x
6
4 y>x
>
:
y < x=3
Y nalmente un tercer mtodo sera mediante las siguientes transformaciones equivalentes:
2
x+y (x + y) 3x2 10xy + 3y 2
>2) 2 >4) 2 <0)
x y (x y) (x y)
2 8 2 8
>
< x 6= y >
< " x>0
6 6 x=3 <y<x
( 6 x > 0 6 >
6 >: 6 :
x y 6= 0 6 6 x < y < 3x
)6 8 x=3 < y < 3x )6 8
6 > 6
(x 3y) (3x y) < 0 6 < x 6= y 6 > x<0
6 6 < "
4 x<0 4 3x < y < x
>
: >
:
3x < y < x=3 x < y < x=3

7. Ejercicio. Resolver y gracar la desigualdad jx 1j + jy + 1j > jx + 1j + jy 1j


| Observacin: Anlogamente se resuelven las desigualdades del tipo

A1 jf1 (x; y) j + A2 jf2 (x; y) j + + Ak jfk (x; y) j = g (x; y) :


286 CAPTULO 7. ECUACIONES E INECUACIONES

Figura 7.21: Izq. El clebre abogado y matemtico ingls Arthur Cayley (Richmond, 1821- Cambridge, 1895). Estudios
en geometra algebraica y teora de invariantes, ecuaciones diferenciales, funciones elpticas, mecnica celeste y de mecan-
ismos. Autor del clculo matricial con muchsimas propiedades de las matrices y de la denicin y varias propiedades de
los grupos nitos. Trabajos fundamentales sobre geometra (interpretacin de la geometra de Lobachevsky). Der. El
famoso matemtico ruso, fundador de la escuela rusa de matemticas, Pafnuty Chebyshev (Okatovo, 1821 - Peterburg,
1894). Estudioso del anlisis matemtico, teora de la aproximacin de funciones mediante polinomios (polinomios de
Chebyshev), teora de los nmeros (distribucin de los nmeros primos), teora de probabilidades (ley de los grandes
nmeros), clculo variacional, teora de supercies y mecnica (teora de mecanismos).

8. Resolver la ecuacin jx + yj + jx yj = 2
Solucin: Tenemos las equivalencias
2 8 2 8
>
< x+y 0 >
< y x
6 x y 0 6 y x
6 > 6 >
6 : 6 :
6 x+y+x y =2 6 x=1
6 8 6 8
6 > x+y 0 6 > y x
6 < 6 <
6 6
6 x y<0 6 y x
6 > 6 >
6 : x+y x+y =2 6 : y=1
jx + yj + jx yj = 2 ) 6 8
6 > )6 8
6 >
6 < x+y <0 6 < y< x
6 6
6 x y 0 6 y x
6 > 6 >
6 : x y+x y =2 6 : y= 1
6 8 6 8
6 > 6 >
6 < x+y <0 6 < y< x
6 6
4 x y<0 4 y>x
>
: >
:
x y x y=2 x= 1
2 (
1 y 1
6 2 8
6 x=1 > 1 y 1
6 ( <
6 1 x 1 6
6 6 x=1
6 6 >
6 y=1 6 : x= 1
)6
6
( )6 8
6 >
6 1 x 1 6 < 1 x 1
6 6
6 y= 1 4 y=1
6 ( >
:
6 1<y<1 y= 1
4
x= 1

Practico lo que aprend:

Resolver e interpretar gracamente las siguientes relaciones con valor absoluto:

x 2
1. a) j3x + 1j > 2, b) j2 xj > 5, c) jx 3j 1, d) < 1, e) j2x + yj > 1,
2x + 3
7.13. ECUACIONES E INECUACIONES CON VALOR ABSOLUTO 287

1
f ) jx 3yj , g) j3x + yj 2, h) jx jyjj < 3, i) jx 2j + y < 3
2
Resolver y gracar en el plano R2 las siguientes relaciones, ecuaciones y desigualdades con
valor absoluto:

2. a) jx 3j = j5 yj, b) jy + 2j = j1 xj, c) jx 2j = 3 jyj, d) j2 3yj = x j5 xj, e) jy 1j = y+jx 1j,


f ) jy + 3j = jxj, f ) jy + 1j = x jx + 1j, g) x + jy 1j = y + jx 1j
( ( ( (
jxj > y jxj jyj < 4 jxj + jyj < 2 jx + yj > 2
3. a) , b) , c) , d)
jx yj < 1 jx 1j + jy 1j > 1 jx 3j jy 1j < 0 jx yj < 3
( ( ( (
jyj < x jxj + jyj > 2 jxj + jyj < 4 jx yj < 4
4. a) , b) , c) , d)
jx + yj > 1 jx + 1j jy 1j < 3 jxj jy 3j > 0 jx + yj > 2
8 8 8
> > > (
< jy 1j + jxj < 9 < jxj + jyj < 4 < jx yj 2 jxj < 2 y
5. a) jx + 2j jyj < 4 , b) jx + 1j + jy 2j > 3 , c) j2x + yj 3 , d)
>
: >
: >
: jyj > 2 x
jx 1j jy 1j < 5 jx 3j jy 2j < 0 jyj 3

Figura 7.22: Izq. El genio matemtico suizo, Johann Bernoulli (Basilea, 1667 - Basilea, 1748), hermano de Jakob
(Basilea, 1655 - Basilea, 1705) y padre de Daniel (Groningen, Holanda, 1700 - Basilea, 1782). Mdico, mecnico y gran
matemtico, especialista en anlisis clsico (clculo diferencial, autor de la regla de LHpital para calcular lmites),
mecnica (relojera), padre del clculo variacional y geometra (isocrona, construccin de curvas). Sola tener muchsimas
controversias con su hermano Jakob por resolver problemas matemticos, e inclusive expuls de su casa a su hijo Daniel
por haber compartido con l el premio de la Academia de Ciencias de Pars. Fue profesor de los genios L. Euler (1707
- 1783) y G. LHpital (1661 - 1704). Sucedi a su hermano Jakob como Profesor de matemticas en la Universidad de
Basilea. Enseo matemticas en Groningen, Holanda. Der. El Teologo, mecnico y famoso matemtico suizo, Jakob
Bernoulli (Basilea, 1655 - 1705). Profesor de matemticas en la Universidad de Basilea. Continuador de los estudios e
investigaciones de Newton (1642 - 1727) y Leibniz (1646 - 1716) en clculo innitesimal (introdujo el trmino integral,
analiz las series numricas, espiral logartmica, nmeros de Bernoulli, ecuacin diferencial lineal y de Bernoulli). Padre
del clculo variacional (problemas isoperimtricos, catenaria, teora de puentes). En probabilidades, descubri el Teorema
de Bernoulli o Ley de los grandes nmeros (la probabilidad de un suceso es el lmite de la frecuencia relativa de dicho suceso
cuando el nmero de pruebas tiende a innito), desarroll el anlisis combinatorio. Escribi el famoso Ars conjectandi
(Arte de las conjeturas).
Captulo 8

MATRICES, DETERMINANTES Y
SISTEMAS LINEALES

El cero indio signic el vaco o la ausencia, pero tambin el

espacio, el rmamento, la bveda celeste, la atmsfera y el ter,

as como la nada, la cantidad desdeable, el elemento insignicante.

Georges Ifrah

Para Tales ... la cuestin primaria no era qu sabemos, sino cmo lo sabemos.

Aristteles

Unidad de competencia: Utilizar los conceptos bsicos, operaciones y transformaciones del lgebra matricial
para reconocer sus aplicaciones y utilizarlas en la resolucin de problemas.

Indicadores de logro:

| Conocer y manejar los conceptos fundamentales sobre las matrices, sus operaciones y sus aplicaciones.

| Resolver ejercicios y problemas con matrices y sus operaciones.

| Conocer y manejar los conceptos fundamentales sobre los determinantes y sus aplicaciones.

| Aplicar las matrices a la solucin de sistemas lineales.

| Resolver ejercicios y problemas sobre sistemas de ecuaciones lineales con distintos mtodos.

| Encontrar, interpretar y vericar el conjunto solucin de un sistema de ecuaciones lineales.

| Descomponer fracciones racionales en fracciones parciales utlizando sistemas de ecuaciones lineales.

Cuatro hermanos tienen $ 45. Si el dinero del 1ro es aumentado en $ 2, el del 2do reducido en $ 2, se duplica el del 3ro y
el del 4to se reduce a la mitad, y todos los hermanos tendrn la misma cantidad. Cunto dinero tena cada uno?

Dispongo de $ 1 para comprar 40 caramelos: de 1, 4 y 12 centavos. Cuntos caramelos de cada tipo puedo comprar?

288
8.1. ELEMENTOS DE LA TEORA DE MATRICES 289

8.1. Elementos de la Teora de Matrices

En la presente seccin pasamos a describir el lgebra elemental de matrices y sus operaciones fundamentales.

Se denomina matriz a una caja numrica compuesta de m las y n columnas. Los valores de m y n se
denominan orden de la matriz. Una matriz puede ser cuadrada, cuando m = n o rectangular cuando m 6= n.
Las matrices, generalmente, son representadas con letras latinas maysculas A, B, C, etc.; o en la forma
A = (aij ), B = (bij ), donde aij representa al elemento ubicado en la la i y columna j :
0 1
a11 a12 a1n ! 1ra la
B a C
a2n C ! 2da la
B 21 a22
A=B C
@ A
am1 am2 amn ! mesima la
# # #
1ra 2da n esima
| {z }
c o lu m n a s

Una matriz A, en forma simplicada, puede ser escrita en la forma A = Amn o A = (aij )mn , donde se indica
que posee m las y n columnas, o que las las i recorren los valores i = 1; 2; : : : ; m (i = 1; m) y las columnas
j recorren los valores j = 1; 2; : : : n (j = 1; n). As, por ejemplo, a34 representa al elemento de la matriz A
ubicado en la interseccin de la 3ra la con la 4ta columna, as mismo, b52 representa al elemento de la matriz
B ubicado en la interseccin de la 5ta la con la 2da columna.

Ejemplos:

!
2 5
1. La matriz A2 2 = es numrica, cuadrada de dimensiones 2 2; a12 = 5, a21 = 0. La matriz
0 7
!
a 5 b
B2 3 = es literal, rectangular de dimensiones 2 3; b13 = b, b22 = 0.
1 0 4
!
1 x 5
2. La matriz Y2 2 (x) = es funcional, cuadrada de dimensiones 2 2; y11 = 1, y21 = x2 . La
x2 1
matriz C2 2 = 2 5 0 7 es numrica, rectangular de dimensiones 1 4; c13 = 0, a14 = 7.

3. La matriz X1 n (x) = a11 a12 a1n es literal, rectangular de dimensiones 1 n y denominada


0 1
y11
B C
B y21 C
vector horizontal. La matriz Ym 1 =B
B ..
C es literal, rectangular de dimensiones m 1 y se denomina
C
@ . A
ym1
vector vertical.

8.1.1. Tipos de matrices

Matriz cuadrada (A = Amm ) es aquella cuyo nmero de ! las y nmero de columnas coincide o es igual
0 1
a m. As, por ejemplo, A1 1 = ( 3), B2 2 = , etc.
2 3
!
0
Matriz rectangular (A = Amn ) es aquella que cumple con m 6= n. Las matrices A2 1 = y
c
!
0 1 2
B2 3 = son matrices rectangulares, etc.
2 3 0
290 CAPTULO 8. MATRICES, DETERMINANTES Y SISTEMAS LINEALES

Matriz triangular (A = Amm ) es aquella matriz cuadrada que cumple con la condicin aij = 0 para
i < j (matriz triangular inferior), es decir, todos los elementos ubicados por encima de la diagonal
principal son nulos; o con la condicin aij = 0 para i > j (matriz triangular superior), es decir,
todos los elementos
0 1 ubicados
0 por debajo1de la diagonal principal son nulos. As, por ejemplo, A3 3 =
2 0 0 7 2 1
B C B C
@ 4 ,
1 0 A 3 3 B = @ 0 4 9 A son matrices triangular inferior y superior, respectivamente,
7 8 1 0 0 1
etc.

Matriz diagonal (A = Amm ) es la que cumple con la condicin aij = 0 para i 6= j, 0 es decir, todos1los
2 0 0
B C
elementos ubicados fuera de la diagonal principal son nulos. As, por ejemplo, A3 3 = @ 0 1 0 A es
0 0 1
una matriz diagonal.

Matriz identidad (A = Amm ) es la que cumple con la condicin aij = 1 para i = j y todos los elementos!
1 0
ubicados fuera de la diagonal principal son nulos (aij = 0 para i 6= j). As, por ejemplo, I2 2 = ,
0 1
0 1
1 0 0
B C
I3 3 = @ 0 1 0 A, etc. son matrices identidades.
0 0 1

Matriz nula (O!= Omn ) es la que cumple


! con la condicin aij = 0 para todo i y j. As, por ejemplo,
0 0 0 0 0
O2 2 = , O2 3 = , etc. son matrices nulas.
0 0 0 0 0

Matriz transpuesta de Amn es aquella matriz ATnm que se obtiene ! de Amn al convertir sus las !
en
1 3 1 4
columnas y sus columnas en las. As, por ejemplo, si A2 2 = , entonces AT2 2 = ;
4 0 3 0
0 1
! 2 4
2 9 7 B C
as mismo, si B2 3 = , entonces B3T 2 = @ 9 1 A. Si una matriz Amm coincide con
4 1 0
7 0
T T
su transpuesta
0 Amm1, es decir, si Amm = Amm , entonces Amm se denomina simtrica, por ejemplo
2 4 3
B C
B = @ 4 8 7 A = BT :
3 7 1

1 1 1
Matriz inversa de Amm es la matriz Amm que cumple con la relacin Amm Amm = Amm Amm = Imm ,
es decir, el producto es igual a la matriz identidad del mismo orden. Si Amm no posee inversa entonces
se denomina singular o degenerada. Como se ver! ms tarde Amm admite inversa siempre y !cuando
1 1 2 1
det A 6= 0. As, por ejemplo, si A2 2 = , det A = 1, entonces A2 12 = , pues
1 2 1 1
! ! !
1 1 2 1 1 0
A2 2 A2 12 = = A2 12 A2 2 = = I2 2 :
1 2 1 1 0 1

| Para calcular la matriz inversa A 1 de A = Amm existen entre otros mtodos el de Gauss y el mtodo
de igualacin. Analizamos este ltimo mtodo que es conveniente para matrices pequeas 2 2 y 3 3.
8.1. ELEMENTOS DE LA TEORA DE MATRICES 291
!
3 1
Supongamos que se desea invertir A = , entonces por denicin de inversa, tendremos:
4 2
! ! !
1 3 1 x y 1 0
AA = = )
4 2 z t 0 1
| {z } | {z }
inversa buscada m atriz identidad
8 8
! ! >
> 3x + z = 1 >
> 3x + z = 1
>
< 4x + 2z = 0 >
< 2x + z = 0
3x + z t + 3y 1 0
= , ) )
4x + 2z 2t + 4y 0 1 >
> t + 3y = 0 >
> t + 3y = 0
>
: >
:
2t + 4y = 1 t+y =1
8
>
> x=1 ! !
>
< z= 2
1 x y 1 1=2
) A = =
>
> y = 1=2 z t 2 3=2
>
:
t = 3=2
! !
1
1 1=2 1 2 1
Hemos obtenido A = = . Ahora, en calidad de comprobacin sabemos
2 3=2 2 4 3
que debe cumplirse que A 1 A = I :
! ! !
1 1 2 1 3 1 1 2 0
A A= = =I
2 4 3 4 2 2 0 2

Anlogamente se calcula la inversa para una matriz A = A3 3.

8.1.2. Operaciones con matrices

Igualdad entre matrices: dos matrices Amn y Bmn del mismo orden, se denominan iguales si y solamente
si, sus respectivos elementos aij y bij son iguales, es decir, aij = bij . As, por ejemplo,
8 8
! ! >
>a=b >
> a= b
>
< >
<
a 3 b 3 3=3 3=3
A2 2 = = B2 2 = , , ;
x y x2 5y >
> x = x2 >
> (x = 0) _ (x = 1)
>
: >
:
y = 5y y=0

es decir, A y B son iguales, siempre y cuando se cumplan las condiciones sealadas.

Suma y diferencia de matrices: dos matrices Amn y Bmn del mismo orden pueden ser sumadas o
restadas, y su matriz resultante se obtiene sumando o restando los respectivos elementos aij y bij , es
decir:
Amn Bmn = (aij ) (bij ) = (aij bij )
! !
1 3 4 3
As, por ejemplo, si A2 2 = y B2 2 = , entonces
5 0 2 1
! ! ! !
1 3 4 3 1+4 3 3 3 0
A2 2 + B2 2 = + = = ;
5 0 2 1 5+2 0+1 7 1
! ! ! !
1 3 4 3 1 4 3+3 5 6
A2 2 B2 2 = = = :
5 0 2 1 5 2 0 1 3 1
292 CAPTULO 8. MATRICES, DETERMINANTES Y SISTEMAS LINEALES
! !
1 3 4 2 4 3
As mismo, si A2 3 = y B2 3 = , entonces
5 0 2 5 2 1
! ! ! !
1 3 4 2 4 3 1+2 3+4 4 3 1 7 1
A2 3 + B2 3 = + = = ;
5 0 2 5 2 1 5 5 0+2 2+1 0 2 3
! ! ! !
1 3 4 2 4 3 1 2 3 4 4+3 3 1 7
A2 3 B2 3 = = = :
5 0 2 5 2 1 5+5 0 2 2 1 10 2 1

Recuerda las observaciones:


La suma de matrices es conmutativa: A + B = B + A:
La suma de matrices es asociativa: A + (B + C) = (A + B) + C:
La matriz A es la opuesta de A y A+A=O

Producto de un escalar (nmero) por una matriz: si k 2 R (C) y A = Amn , entonces el produc-
to kA = kAmn = k (aij ) = (kaij ), es decir, para multiplicar un escalar por una matriz, se multiplica
el escalar k por todos y cada
! uno de los elementos de la matriz A.! As, por ejemplo, si k =!1=6
3 1 6 1 3 1 6 3=6 1=6 1
y A2 3 = , entonces kA2 3 = = =
10 12 1 6 10 12 1 10=6 2 1=6
!
1=2 1=6 1
:
5=3 2 1=6

Producto de matrices: si A = Amn y B = Bnk entonces es posible el producto de estas dos matrices
en ese orden (el producto de matrices no es conmutativo), teniendo: Amn Bnk = Cmk , donde el nmero
n de columnas de la matriz A (multiplicando) coincide con el nmero n de columnas de la matriz B
(multiplicador). Los elementos cij de la matriz producto Cmk se obtienen como sumas algebraicas de los
productos de los respectivos elementos de las las de A por los respectivos elementos de las columnas de
B, as, por ejemplo:

c11 = a11 b11 + a12 b21 + + a1;m 1 bk 1;1 + a1;m bk;1 ;


c12 = a11 b12 + a12 b22 + + a1;m 1 bk 1;2 + a1;m bk;2 , etc.
! !
1 3 4 3
As, por ejemplo, si A2 2 = y B2 2 = , entonces
5 0 2 1
! ! ! !
1 3 4 3 ( 1) 4 + (3) 2 1 ( 3) + (3) 1 2 6
A2 2 B2 2 = C2 2 = = = ;
5 0 2 1 (5) 4 + 0 (2) 5 ( 3) + 0 (1) 20 15
! !
19 12 1 2
que no es igual a B2 2 A2 2 = (no hay conmutatividad ); as mismo, si A2 2 =
3 6 5 3
!
2 3 3
y B2 3 = , entonces
1 0 1
! ! !
1 2 2 3 3 1 ( 2) + (2) 1 1 (3) + (2) 0 1 (3) +2 ( 1)
A2 2 B2 3 = =
5 3 1 0 1 5 ( 2) + ( 3) 1 5 (3) + ( 3) 0 5 (3) + ( 3) ( 1)
!
0 3 1
= = C2 3
13 15 18

Recuerda las observaciones:


En general, el producto de matrices no es conmutativo: AB 6= BA:
El producto de matrices es asociativo: (AB) C = A (BC) :
8.1. ELEMENTOS DE LA TEORA DE MATRICES 293

El producto matricial es distributivo con respecto a la suma o diferencia de matrices: A (B C) =


AB BC:
A diferencia de lo que sucede con los escalares (de R o C), donde, si a b = 0 ) (a = 0) _ (b = 0), con
las matrices puede
! suceder que A!6= O y B 6= O y sin embargo AB = O. Esto sucede, por ejemplo, con
0 2 1 0
A= yB= , siendo AB = O.
0 0 0 0

Potencia natural de una matriz: si A = Amm , entonces Ak se dene con la igualdad Ak = A


| A{z A} =
! k veces
1 1
Akmm . As, por ejemplo, si A2 2 = , entonces
2 0
! ! ! !2 !
3 1 1 1 1 1 1 1 1 1 1
A = =
2 0 2 0 2 0 2 0 2 0
! ! !
3 1 1 1 5 3
= = :
2 2 2 0 6 2
0 1 0 1
2 0 0 24 0 0
B C 4 B 4 C k
As mismo, si D3 3 = @ 0 1 0 A, entonces D = @ 0 1 0 A, y siempre Imm = I por
4
0 0 1 0 0 ( 1)
ser la matriz identidad I el elemento neutro para el producto de matrices. As mismo, para una matriz
k
diagonal Dmm = akii .

Recuerda las relaciones entre matrices transpuestas, inversas y potencias. Si las matrices A y
B poseen las dimensiones adecuadas, entonces se cumple que:
T T 1
a) (A B) = AT BT b) (AB) = B T AT c) (AB) =B 1
A 1
1 1 T T k k T 1 k 1
d) AT = A e) A = A f) A = Ak

Practico lo que aprend:

! !
2a b z 1 a+b 3
1. Igualdad entre matrices: Dadas las matrices A = , B = , C =
x3 22y x 8
!
a b t2 6
. Con que condiciones se cumplen las igualdades: a) A = B, b) B = C:
x2 4x + 6 43z
! !
1 3 0 3
2. Suma, diferencia, productos con matrices: Sean A = , B = , C =
2 5 1 2
! ! !
4 1 1 3 4 0 2 3
,D= ,E= . Calcular:
5 0 0 2 1 1 4 0

a) A + B b) B 2A c) 5A 4B + 3C d) 2E 3D
T T
e) BA f) BA AB g) (BA) AT B T h) (CBA)
i) A2 B 2 j) A3 B 3 k) ABD BCE l) DT B E T A
T T T T
m) DT E T n) 3E T DT 2DT E T o) (AD) (BE) p) (2AD) + (3BE)
T T
q) AB r) 3AD 2BE s) DE T EDT t) (AE) (BD)

3. Mostrar que:

a) A2 I 2 = (A I) (A + I) b) A3 I 3 = (A I) A2 + A + I
T
c) A3 + B 3 = (A + B) A2 AB + B 2 d) (ABC) = C T B T AT
T 2 T 1
e) A2 = AT f) B 1
AT = A BT
294 CAPTULO 8. MATRICES, DETERMINANTES Y SISTEMAS LINEALES

Figura 8.1: Izq. El famoso matemtico sovitico, fundador de la escuela de topologa sovitica, Pavel Sergeyevich
Aleksandrov (Bogorodsk, Noginsk, 1896 - Mosc, 1992). Trabajos fundamentales sobre espacios mtricos y topologa
algebraica. Creador, junto con Pavel S. Urysohn (Odesa, 1898 - Batz, 1925) de la nocin de espacio compacto y de
espacios metrizables. Desarroll la teora de la dimensin y su teora homolgica y las propiedades topolgicas de las
guras. Sintetiz la topologa combinatoria con la conjuntista. Trabajos sobre teora de funciones de variable real y teora
de conjuntos (potencia de los conjuntos de Borel). Generaliz el teorema de James W. Alexander (1888 - 1971). Der.
El conocido matemtico y gometra sovitico Aleksandr Danilovich Aleksandrov (Bolyn, Ryazanskoi, 1912 - ). Fundador
de la escuela sovitica de geometra en conjunto. Trabajos fundamentales sobre geometra, ecuaciones en derivadas
parciales, topologa, clculo variacional, historia y losofa de la matemtica. Construy e investig la geometra interior
de supercies convexas con ayuda de aproximaciones de poliedros convexos. Introdujo nuevos mtodos para el estudio
de las propiedades mtricas de guras y variedades mtricas. Gran popularizador de la geometra.

0 1 0 1 0 1
1 0 0 0 0 1 1 0 1
B C B C B C
4. Sean A = @ 0 2 0 A, B = @ 0 2 0 A , C = @ 0 1 0 A,
0 1 1 2 0 0 0 0 1

0 1 0 1 0 1
1 0 0 1 0 0 1 0 1
B C B C B C
D=@ 0 2 0 A, E = @ 0 1 0 A, F = @ 0 1 0 A. Calcular:
1 0 1 0 0 2 1 0 1

a) A + B b) 3B 2A + 4F c) 5A 4B + 3C d) 2E 3D + 2F
T T
e) BAF f ) BA AB F g) (BA) AT B T h) (CBA)
i) A2 B 2 j) A3 B 3 k) ABD BCE l) D B ET A
T
T T T T
m) DT E T n) 3E T DT 2DT E T o) (AD) (BE) p) (2AD) + (3BE)
T T
q) F AB r) 3AD 2BE s) DE T EDT t) (AE) (BD)

Y mostrar que:

a) A2 I 2 = (A I) (A + I) b) A3 I 3 = (A I) A2 + A + I
T
c) A3 + B 3 = (A + B) A2 AB + B 2 d) (ABC) = C T B T AT
T T 2
e) C T B T AT = ABC f) A2 = AT

! !
1 3 3 2
5. Operaciones con matrices inversas: Dadas las matrices A = , B = , C =
2 5 1 1
8.2. SISTEMAS DE ECUACIONES LINEALES 295

Figura 8.2: Izq. El matemtico sovitico, ucraniano Evgenii B. Dynkin (), gran especialista en procesos aleatorios y
procesos de Markov (Riazan, 1856 - Petrogrado, 1922). Der. Los matemticos franceces y esposos, Marie_Louise Dubreil-
Jacotin (1905 - 1972), Paul Dubreil (1904 - 1994), especialistas en lgebra moderna, (autores de las conocidas Lecciones
de lgebra moderna) junto a la matemtica alemana Emmy Noether (Erlangen, 1882 - Bryn Mawr, Pennsylvania, 1935).

0 1
! 1 0 0
3 2 B C
,D=@ 0 2 0 A. Calcular:
1 2
1 0 1
1 1 1 1 1
a) A b) B c) D + D d) BA B A
1 T 1 1 1 1
e) CA f ) 3B (AB) g) (ABC) h) (2CBA)
1 1 1 T T 1 T 1 1 T 1 T 1 T
i) A B C j) A B C k) A B C l) A2 BT C
1 T 1 T 1
m) CT B 1
BT C n) D2 DT o) DT D 1
3D p) DT D 1
3D
1 T 1 1 1
q) A B r) AT B T C T s) B T A2 AT C 1
t) D 1
DT 3D

8.2. Sistemas de Ecuaciones Lineales

Pasamos a utilizar y a aplicar las operaciones con matrices a la resolucin de sistemas de ecuaciones lineales,
es decir, a los sistemas compuestos de m ecuaciones con n incgnitas de grado uno.

8.2.1. Sistemas de ecuaciones lineales m n. Mtodo de reduccin de Gauss.

El sistema de la forma: 8
>
> a11 x1 + a12 x2 + + a1n xn = b1
>
< a x +a x +
21 1 22 2 + a2n xn = b2
(1)
>
> ::::::::::::::::::::::::::::::::::::::::::::::::::::::::
>
:
am1 x1 + am2 x2 + + amn xn = bm
compuesto de m ecuaciones con n incgnitas xj , siendo aij 2 R los coecientes del sistema; bi 2 R los coecientes
independientes, con i = 1,2,: : :,m; j = 1,2,: : :,n, se denomina sistema de ecuaciones lineales m n. El valor de
i nos da el nmero de la ecuacin (i = 1,2,: : :,m ecuaciones), y el valor de j nos da el nmero de la incgnita
( j = 1,2,: : :,n incgnitas). Por ejemplo, a23 x3 nos indica que tenemos el coeciente correspondiente a la 2da
ecuacin y a la 3ra incgnita x3 .

| El conjunto de n valores jos x01 , x02 , , x0n , se denomina solucin del sistema lineal (1), si al susti-
tuir cada incgnita xj por su respectivo valor x0j en todas y cada una de las m ecuaciones, obtenemos m
identidades.
( Si el sistema lineal (1) no posee solucin alguna, entonces se denomina incompatible, por ejem-
3x 2y = 1
plo: ) ;. Si el sistema lineal (1) tiene solucin, se llama compatible. Un sistema com-
6x + 4y = 3
296 CAPTULO 8. MATRICES, DETERMINANTES Y SISTEMAS LINEALES
(
x 2y = 0
patible es determinado, si posee una solucin nica, por ejemplo: ) [x = 2; y = 1]. Un
2x + 5y = 1
sistema
( compatible es indeterminado,
( si posee ms de una solucin (innito
( nmero de soluciones),
( por ejem-
3x y = 1 x=t x=0 x=1
plo: ) , donde t 2 R. Si t = 0 ) ; si t = 1 ) ; si
6x 2y = 2 y = 3t 1 y= 1 y= 4
(
x= 2
t= 2) ; etc.
y= 7

| Uno de los mtodos mas convenientes (por su posibilidad de automatizarlo) para resolver el sistema lineal
(1) es el denominado mtodo de eliminacin de Gauss. Este mtodo es la generalizacin del ya conocido mtodo
elemental de suma y resta o reduccin.

Es evidente que el sistema lineal (1) no se altera si: a) intercambiamos 2 o ms ecuaciones, b) multiplicamos
una o ms ecuaciones por una constante distinta del cero, c) sumamos a una o varias de las ecuaciones dadas
otras cualesquiera multiplicadas por una constante, d) eliminamos del sistema las ecuaciones con coecientes
proporcionales a una de estas, etc.

El mtodo de Gauss consiste en transformar primeramente el sistema lineal (1) en una tabla numrica
compuesta de todos los coecientes aij y bi , donde cada una de las las numricas as obtenidas juega el mismo
rol que las ecuaciones iniciales del sistema. Luego, utilizando las propiedades anteriores, que las llamaremos
transformaciones elementales por las (suma y resta entre las, etc.) se trata de llegar a una forma diagonal,
representada en el siguiente esquema:
0 1 0 1
a11 a12 a1n b1 0 0
B a a2n b2 C B C
B 21 a22 C transform aciones elem entales B 0 0 C
B C ! B C
@ A @ A
am1 am2 amn bm 0 0
| {z } | {z }
Sistem a lineal original Sistem a lineal nal

Suponiendo que a11 6= 0 (si este elemento es nulo, entonces intercambiamos la 1ra la con la 2da , o con la 3ra , o
con otra), podemos con ayuda de este elemento pivote a11 6= 0 y la suma y resta de las otras las de la matriz o
tabla ampliada, eliminar los primeros coecientes de cada una de las las siguientes, obteniendo de esta manera
una nueva matriz o tabla de la forma (ver los ejemplos):
0 1
a11 a12 a1n b1
B 0 e
a22 a2n eb2 C
e
B C
B C
@ A
0 e
am2 e
amn bm e

Consideramos ahora al elemento pivote e a22 6= 0 (si este elemento es nulo, entonces intercambiamos la 2da la
ra ta
con la 3 , o con la 4 , o con otra), y con ayuda de transformaciones elementales llegamos a la forma
0 1
b
a11 0 a1n bb1
b
B 0 e
a22 a2n eb2 C
e
B C
B C;
@ A
0 0 b b
amn bm
de donde ya podemos encontrar los valores de las incgnitas (por ejemplo, de la anterior tabla tenemos que
bbm
amn xn = bbm ) xn =
b , y las otras incgnitas se encuentran por sustitucin de abajo hacia arriba), o seguir
b
amn
sucesivamente con el proceso de eliminacin hasta llegar a la forma:
0 1
0 0
B 0 0 C
B C
B C
@ A
0 0
8.2. SISTEMAS DE ECUACIONES LINEALES 297

De esta ltima tabla podemos despejar las variables fundamentales, stas estn colocadas en la diagonal prin-
cipal, el resto de variables pasan al otro lado de la igualdad.

En los siguientes ejemplos representaremos mediante f1 , f2 , f3 , etc. las las de la matriz ampliada de Gauss,
y a la derecha de cada la colocamos las operaciones entre las las de la matriz ampliada.

Observacion: Al resolver un sistema de ecuaciones lineales, no necesariamente debemos llegar a la forma


diagonal, es suciente llegar a la forma triangular superior, hallar la solucin de la ltima incgnita y luego
sucesivamente encontrar las otras soluciones (yendo de abajo hacia arriba).

Ejemplos:

8
>
< 2x y + 3z = 2
1. Utilizando el mtodo de Gauss, resolver el sistema 3 3: x 2y + 2z = 3
>
:
3x + y z = 5
Solucin: Armando la matriz ampliada, tenemos
0 1 0 1
2 1 3 2 f1 2 1 3 2 f1
B C B C
@ 1 2 2 3 A f2 ! @ 0 3 1 4 A 2f2 f1 !
3 1 1 5 f3 0 5 11 16 2f3 3f1
0 1 0 1
6 0 8 2 3f1 f2 6 0 0 6 f1 + 8f3
B C B C
@ 0 3 1 4 A f2 !@ 0 3 0 3 A f2 + f3 !
0 0 28 28 3f3 + 5f2 0 0 1 1 f3
0 1 8 8
1 0 0 1 f1 >
< 1x + 0y + 0z = 1 >
< x=1
B C
@ 0 1 0 1 A f2 ! 0x 1y + 0z = 1 ! y=1
>
: >
:
0 0 1 1 f3 0x + 0y 1z = 1 z= 1

Observacin: en las operaciones entre las fi es siempre necesario escribir la la pivote al nal de la
operacin.
8
>
< 2y + 3z = 3
2. Utilizando el mtodo de Gauss, resolver el sistema lineal 3 3: 5x 2y + 2z = 7
>
:
3x + y z = 2
Solucin: Armando la matriz ampliada y permutando la primera y segunda las (para que el pivote no
sea nulo), tenemos
0 1 0 1
0 2 3 3 f1 5 2 2 7 f1
B C B C
@ 5 2 2 7 A f2 ! @ 0 2 3 3 A f2 !
3 1 1 2 f3 3 1 1 2 f3
0 1 0 1
5 2 2 7 f1 5 0 1 4 f1 f2
B C B C
@ 0 2 3 3 A f2 !@ 0 2 3 3 A f2 !
0 11 11 11 5f3 3f1 0 0 55 55 2f3 11f2
0 1 0 1
5 0 0 5 f1 f3 1 0 0 1 f1
B C B C
@ 0 2 0 0 A f2 + 3f3 ! @ 0 1 0 0 A f2 !
0 0 1 1 f3 0 0 1 1 f3
8 8
>
< 1x + 0y + 0z = 1 >
< x=1
1y + 0z = 0 ! y=0
>
: >
:
0x + 0y 1z = 1 z=1
298 CAPTULO 8. MATRICES, DETERMINANTES Y SISTEMAS LINEALES
8
>
> 3x 4y + 2z + u = 2
>
< x + y + 3z 2u = 4
3. Utilizando el mtodo de Gauss, resolver el sistema 4 4:
>
> 2x + 3y 4z 3u = 3
>
:
4x 5y + z 2u = 1
Solucin: Armando la matriz ampliada, tenemos
0 1 0 1
3 4 2 1 2 f1 3 4 2 1 2 f1
B 1 1 3 2 4 C f2 B C 3f
B C B 0 7 7 7 14 C 2 f1
B C !B C !
@ 2 3 4 3 3 A f3 @ 0 17 16 11 5 A 3f3 2f1
4 5 1 2 1 f4 0 1 5 10 5 3f4 4f1

0 1 0 1
3 0 6 3 6 f1 + 4f2 1 0 2 1 2 f1
B 0 1 1 1 2 C f2 B 0 1 1 1 2 C f
B C B C 2
B C !B C !
@ 0 0 33 6 39 A f3 17f2 @ 0 0 11 2 13 A f3
0 0 6 9 3 f4 f2 0 0 2 3 1 f4
0 1 0 1
11 0 0 7 4 11f1 + 2f3 11 0 0 7 4 f1
B 0 1 0 9 9 C 11f2 + f3 B 0 1 0 9 9 C f
B C B C 2
B C !B C !
@ 0 0 11 2 13 A f3 @ 0 0 11 2 13 A f3
0 0 0 37 37 11f4 2f3 0 0 0 1 1 f4
0 1 8 8
11 0 0 0 11 f1 7f4 >
> 11x = 11 >
> x=1
B C >
< >
<
B 0 1 0 0 0 C f2 9f4 y=0 y=0
B C ! !
@ 0 0 11 0 11 A f3 + 2f4 >
> 11z = 11 >
> z= 1
>
: >
:
0 0 0 1 1 f4 u= 1 u=1
8
>
> 2x y+z u= 1
>
< 5x + 3y + 2u = 6
4. Utilizando el mtodo de Gauss, resolver el sistema 4 4:
>
> 3y 4z 3u = 2
>
:
x + 2y 3z 2u = 0
Solucin: Armando la matriz ampliada, tenemos
0 1 0 1
2 1 1 1 1 f1 2 1 1 1 1 f1
B 5 3 0 2 6 C f2 B C 2f + 5f
B C B 0 1 5 1 7 C 2 1
B C !B C !
@ 0 3 4 3 2 A f3 @ 0 3 4 3 2 A f3
1 2 3 2 0 f4 0 5 7 3 1 2f4 f1

0 1 0 1
2 0 6 2 6 f1 + f2 1 0 3 1 3 f1
B 0 1 5 1 7 C f2 B C f
B C B 0 1 5 1 7 C 2
B C !B C !
@ 0 0 19 0 19 A f3 3f2 @ 0 0 1 0 1 A f3
0 0 32 2 34 f4 5f2 0 0 16 1 17 f4
0 1 0 1
1 0 0 1 0 f1 + 3f3 1 0 0 0 1 f1 + f4
B 0 1 0 1 2 C f2 + 5f3 B 0 1 0 0 1 C f +f
B C B C 2 4
B C !B C !
@ 0 0 1 0 1 A f3 @ 0 0 1 0 1 A f3
0 0 0 1 1 f4 16f3 0 0 0 1 1 f4
8 8
>
> 1x + 0y + 0z + 0u = 1 >
> x= 1
>
< 0x + 1y + 0z + 0u = 1 >
< y=1
!
> 0x + 0y 1z + 0u = 1
> > z=1
>
>
: >
:
0x + 0y + 0z + 1u = 1 u= 1
8.2. SISTEMAS DE ECUACIONES LINEALES 299
8
>
> 3x 2y + z 3t + u + 2v = 2
>
< 2x + 2y 2z + t u 3v = 1
5. Resolver el sistema lineal1 4 6:
>
> 3x y z 5t + 2u v = 3
>
:
4x 2y z 3t + 3u + v = 2
Solucin: Partiendo de la tabla de Gauss, tendremos
0 1 0 1
3 2 1 3 1 2 2 3 2 1 3 1 2 2
B 2 2 2 1 1 3 1 C B 0 3 6 6 3 9 15 C
B C B C
B C! !B C!
@ 3 1 1 5 2 1 3 A @ 0 0 3 7 3 1 8 A
4 2 1 3 3 1 2 0 0 0 57 3 21 75
0 1
57 0 0 0 14 3 46
B 0 57 0 0 77 69 89 C
B C
!B C
@ 0 0 57 0 64 30 23 A
0 0 0 57 3 21 75
De donde obtenemos la solucin general:
8 14 1
>
> x = 57 u 19 v + 46
57
>
< y = 77 u + 23 v 89
57 19 57 ; u; v 2 R
> 64 10 23
>
> z = 57 u 19 v + 57
: 1 7
t = 19 u 19 v + 25
19

o en forma vectorial
0 1 0 1 0 1 0 1
x 14 3 46
B y C 1 B 77 C 1 B 69 C 1 B 89 C
B C B C B C B C
B C= B Cu + B Cv + B C ; u; v 2 R
@ z A 57 @ 64 A 57 @ 30 A 57 @ 23 A
t 3 21 75
solucin del sistem a hom ogneo solucin particular

8.2.2. Sistemas de ecuaciones lineales n n. Mtodo de los determinantes de


Cramer.

El presente mtodo desarrollado por el matemtico suizo Gabriel Cramer (1704-1752), sirve fundamental-
mente para resolver sistemas de ecuaciones lineales compatibles (con solucin nica) n n:

Cuando resolvemos un sistema lineal 2 2, tenemos el siguiente resultado:


8
( > b a b2 a12
a11 x + a12 y = b1 < x = 1 22
) a a
11 22 a12 a21
a21 x + a22 y = b2 > b1 a21 b2 a11 ; a12 a21 a11 a22 6= 0;
: y=
a12 a21 a11 a22
el mismo que puede ser representado en la forma tabular:

b1 a12 a11 b1
b2 a22 a21 b2
x= ; y=
a11 a12 a11 a12
a21 a22 a21 a22

Por lo tanto un sistema lineal 2 2, puede ser resuelto con ayuda de aquellas tablas cuadradas de 2 las y 2
columnas, denominadas determinantes de 2 2:
1 La teora completa sobre los sistemas lineales m n se expone en los cursos de algebra lineal. Ver [6].
300 CAPTULO 8. MATRICES, DETERMINANTES Y SISTEMAS LINEALES

Analizamos las matrices cuadradas


0 1
! a11 a12 a13
a11 a12 B C
A1 1 = a11 ; A2 2 = ; A3 3 = @ a21 a22 a23 A , etc.
a21 a22
a31 a32 a33

y sus respectivos determinantes

a11 a12 a13


a11 a12
det A1 1 = a11 ; det A2 2 = ; det A3 3 = a21 a22 a23 , etc.
a21 a22
a31 a32 a33

Introducimos la siguiente denicin2 :

Denicin.

det A1 1 = ja11 j := a11

a11 a12
det A2 2 = := a11 a22 a12 a21 , es decir el producto de los trminos de la diagonal principal
a21 a22
menos el producto de los trminos de la diagonal secundaria.

a11 a12 a13


det A3 3 = a21 a22 a23 := a11 a22 a33 + a21 a32 a13 + a31 a12 a23
a31 a32 a33
a13 a22 a31 a11 a23 a32 a12 a21 a33 , es decir (regla de Sarrus) los productos de las diagonales que van de
arriba hacia abajo y de izquierda a derecha menos los productos de las diagonales que van de abajo hacia
arriba y de izquierda a derecha, de acuerdo al esquema

&+ a11 a12 a13


a21 a22 a23
a31 a32 a33
a11 a12 a13
% a21 a22 a23

n
X i+j
det An n = ( 1) aij Mij , i = 1; 2; : : : ; n (yendo por la la i), o
j=1
n
X i+j
det An n = ( 1) aij Mij , j = 1; 2; : : : ; n (yendo por la columna j),
i=1
donde el smbolo Mij representa al menor (determinante de orden (n 1) (n 1)) que resulta de eliminar
del determinante original n n la i-sima la y la j-sima columna.

Recuerda las propiedades de los determinantes3 :

1. Un determinante no vara al trasponer la matriz cuadrada, es decir si hacemos el intercambio de las las
por las columnas: aij ! aji .

2. Si un determinante posee una la o columna repleta de ceros, entonces el determinante es nulo.

3. Si en un determinante se permutan 2 las (columnas), entonces el valor del determinante cambia de signo.
2 La teora completa de los determinantes n n se la estudia en los cursos de lgebra Lineal. Ver [1], [6], [11].
3 Las demostraciones de las propiedades de los determinantes pueden consultarse en [6].
8.2. SISTEMAS DE ECUACIONES LINEALES 301

4. Si en un determinante existen 2 las (columnas) proporcionales (por ejemplo iguales), entonces el valor
del determinante es cero.

5. Si en un determinante una de sus las (columnas) se multiplica por un escalar k, entonces el valor del
determinante queda multiplicado por k:

6. Si en un determinante una la (columna) se la representa como una suma, entonces el determinante es


igual a la suma de 2 determinantes, el primero de los cuales tiene por la (columna) al primer sumando,
y el segundo determinante tiene por la (columna) al segundo sumando, siendo los restantes elementos
iguales.

7. Si una de las las (columnas) del determinante es la combinacin lineal (por ejemplo una suma algebraica
de las restantes), entonces el determinante es igual a cero.

8. Si a los elementos de una la (columna) de un determinante se le suma los elementos correspondientes de


otra la (columna) multiplicados por un mismo nmero, entonces el valor del determinante no cambia.

Ejemplos:

(
5x + 9y = 4
1. Por la Regla de Cramer, resolver el sistema 2 2:
8x + 3y = 5
Solucin:
4 9
5 3 ( 4 3) (5 9) 57
x= = = ) x = 1;
5 9 (5 3) (8 9) 57
8 3

5 4
(
8 5 (5 5) (8 4) 57 x=1
y= = = )y= 1)
5 9 (5 3) (8 9) 57 y= 1
8 3

(
3x + 7y = 15
2. Por la Regla de Cramer, resolver el sistema 2 2:
11x + 8y = 2
Solucin:
15 7
2 8 (15 8) (2 7) 106
x= = = )x= 2
3 7 (3 8) (11 7) 53
11 8

3 15
(
11 2 (3 2) (11 15) 159 x= 2
y= = = )y=3)
53 53 53 y=3

8
>
< x + y + z = 12
3. Por la Regla de Cramer, resolver el sistema 3 3: 3x 5y + 2z = 1
>
:
2x + 3y + 4z = 26
302 CAPTULO 8. MATRICES, DETERMINANTES Y SISTEMAS LINEALES

Solucin: Utilizando la regla de Sarrus, tenemos:

12 1 1
1 5 2
26 3 4 12 ( 5) 4 + 2 (26) (3) +26 (5) 3 (2) 12 + 4 129
x = = = )x=3
1 1 1 5 (4) +2 ( 2) +3 (3) 2 (5) 3 (2) 4 (3) 43
3 5 2
2 3 4

1 12 1
3 1 2
2 26 4 4 12 (2) 2 + 3 (26) 2 26 (2) 4 (3) 12 172
y= = = )y=4
43 43 43
1 1 12
3 5 1
2 3 26 5 (26) +2 + 12 (3) 3 2 (5) 12 + 3 26 (3) 215
z= = = )z=5
43 43 43
8
>
< x=3
) y=4
>
:
z=5
8
>
< 2x + 3y + z = 14
4. Por la Regla de Cramer, resolver el sistema 3 3: 3x y + 2z = 5
>
:
x + 2y z = 7
Solucin: Los valores de los determinantes los encontramos por la regla de los menores (por ejemplo,
utilizando los elementos de la primera la):

14 3 1
5 1 2 1 2 5 2 5 1
14 3 +
7 2 1 2 1 7 1 7 2
x = = =
2 3 1 1 2 3 2 3 1
2 3 +
3 1 2 2 1 1 1 1 2
1 2 1
14 (1 4) 3 ( 5 14) + (10 + 7) 32
= = )x=2
2 (1 4) 3 ( 3 2) + (6 + 1) 16

2 14 1
3 5 2 5 2 3 2 3 5
2 14 +
1 7 1 7 1 1 1 1 7
y = = =
16 16
2 ( 5 14) 14 ( 3 2) + (21 5) 48
= = )y=3
16 16

2 3 14
3 1 5 1 5 3 5 3 1
2 3 + 14
1 2 7 2 7 1 7 1 2
z = = =
16 16
2 ( 7 10) 3 (21 5) + 14 (6 + 1) 16
= = )z=1
16 16
8.2. SISTEMAS DE ECUACIONES LINEALES 303
8
>
< x=2
) y=3
>
:
z=1

8
>
< 4x + 7y z = 4
5. Por la Regla de Cramer, resolver el sistema 3 3: 3x + 5y 2z = 9
>
:
2x 13y 3z = 2

Solucin: Los valores de los determinantes los encontramos por la regla de los menores (por ejemplo,
utilizando los elementos de la segunda columna):

4 7 1
9 5 2 9 2 4 1 4 1
7 +5 + 13
2 13 3 2 3 2 3 9 2
x = = =
4 7 1 3 2 4 1 4 1
7 +5 + 13
3 5 2 2 3 2 3 3 2
2 13 3
7 (27 + 4) + 5 (12 + 2) + 13 (8 9) 160
= = )x=2
7 ( 9 + 4) + 5 ( 12 + 2) + 13 ( 8 + 3) 80

4 4 1
3 9 2 3 2 4 1 4 1
4 9 2
2 2 3 2 3 2 3 3 2
y = = =
80 80
4 ( 9 + 4) 9 ( 12 + 2) 2 ( 8 + 3) 80
= = )y= 1
80 80

4 7 4
3 5 9 3 9 4 4 4 4
7 +5 + 13
2 13 2 2 2 2 2 3 9
z = = =
80 80
7 (6 + 18) + 5 (8 + 8) + 13 ( 36 + 12) 400
= = )z=5
80 80

8
>
< x=2
) y= 1
>
:
z=5

8
>
> 2x y + z u = 1
>
< 2x y 3u = 2
6. Resolver por Cramer el sistema 4 4:
>
> 3x z + u = 3
>
:
2x + 2y 2z + 5u = 6

Solucin: Los valores de los determinantes los encontramos por la regla de los menores (por ejemplo,
utilizando los elementos de la primera columna) para los determinantes de 4to orden, por Sarrus para los
304 CAPTULO 8. MATRICES, DETERMINANTES Y SISTEMAS LINEALES

determinantes de 3er orden:

1 1 1 1
2 1 0 3 1 0 3 1 1 1 1 1 1 1 1 1
3 0 1 1 0 1 1 2 0 1 1 3 1 0 3 +6 1 0 3
6 2 2 5 2 2 5 2 2 5 2 2 5 0 1 1
x = =
2 1 1 1 1 0 3 1 1 1 1 1 1 1 1 1
2 1 0 3 2 0 1 1 2 0 1 1 +3 1 0 3 2 1 0 3
3 0 1 1 2 2 5 2 2 5 2 2 5 0 1 1
2 2 2 5
(5 6 2) 2 (5 + 2 2 2) 3 ( 6 2 + 5 + 6) + 6 ( 1 + 3 + 1) 0
= = )x=0
2 (5 6 2) 2 (5 + 2 2 2) + 3 ( 6 2 + 6 + 5) 2 ( 1 + 3 + 1) 9

Anlogamente tenemos:

2 1 1 1
2 2 0 3 2 0 3 1 1 1 1 1 1 1 1 1
3 3 1 1 2 3 1 1 2 3 1 1 +3 2 0 3 2 2 0 3
2 6 2 5 6 2 5 6 2 5 6 2 5 3 1 1
y = 9 = 9
2 ( 6) 2 (6) + 3 (6) 2 (6) 18
= = )y=2
9 9

2 1 1 1
2 1 2 3 1 2 3 1 1 1 1 1 1 1 1 1
3 0 3 1 2 0 3 1 2 0 3 1 +3 1 2 3 2 1 2 3
2 2 6 5 2 6 5 2 6 5 2 6 5 0 3 1
z = 9 = 9
2 ( 5) 2 (5) + 3 (5) 2 (5) 15 5
= = )z=
9 9 3

2 1 1 1
2 1 0 2 1 0 2 1 1 1 1 1 1 1 1 1
3 0 1 3 2 0 1 3 2 0 1 3 +3 1 0 2 2 1 0 2
2 2 2 6 2 2 6 2 2 6 2 2 6 0 1 3
u = 9 = 9
2 (4) 2 ( 8) + 3 ( 8) 2 ( 6) 12 4
= = )u=
9 9 3

8
>
> x=0
>
>
>
< y=2
) 5
>
> z=
>
> 3
>
: u= 4
3

8
>
> 5x 3y z u = 14
>
< 4x 7y + 3z + 5u = 4
7. Resolver por Cramer el sistema 4 4:
> 2x 5y z + 7u = 22
>
>
:
6x 2y 9z = 37

Solucin: Anlogamente, calculando los determinantes de 4to orden por menores (escogiendo cualquier
8.2. SISTEMAS DE ECUACIONES LINEALES 305

la o columna con la mayor cantidad de ceros) y luego los determinantes de 3er orden por Sarrus, tenemos:
14 3 1 1 5 14 1 1
4 7 3 5 4 4 3 5
22 5 1 7 2 22 1 7
37 2 9 0 6 37 9 0
x= ; y=
5 3 1 1 1600
4 7 3 5
2 5 1 7
6 2 9 0
3200 1600
x= ) x = 2; y = )y=1
1600 1600
5 3 14 1 5 3 1 14
4 7 4 5 4 7 3 4
2 5 22 7 2 5 1 22
6 2 37 0 6 2 9 37
z= ; u=
1600 1600
4800 6400
z= ) z = 3; u = )u=4
1600 1600
8
>
> x= 2
>
< y=1
)
>
> z= 3
>
:
u=4

Practico lo que aprend:

Ecuaciones y sistemas de ecuaciones reducibles a primer grado:

3 2 3 1 1 3 5 x 1 x 6 x 5 x 2
1. a) = , b) = x , c) = , d) + = + ,
2 3
4 3x 1 2x 3 3) x (2x x x 2 x 7 x 6 x 3
2 2
2x 3
ax b bx + a a2 + b2 2 3 1 2 3
e) + = 2 2
, f) = , g) =
a+b a b a b 1 2x 1 3x x + 2 x (x + 2) x
2. Qu relacin debe existir entre a y b para que las 2 ecuaciones siguientes tengan la misma raz o solucin?
3 4a x 1 3 x
Comprobar el resultado. 2x = , (3x 2b) = b:
2 3 3 3 2 3
3. Hallar los valores de a, b, c para que las 3 ecuaciones siguientes tengan idntica solucin. Comprobar el
x 1 x 3 x 2x
resultado. 2x a = 3b , 4b (2x b) = + c, 1 a = c:
2 2 3 2 3 3
4. Hallar los valores de a, b, c para que las 3 ecuaciones siguientes tengan idntica solucin. Comprobar el
resultado. 3x a = 13 (3x + b), 31 (3x b) = 2c x, 2x + 2a = 13 (3x 2c) :

5. Qu valores deben tomar a, b y c para que la raz de la 1ra ecuacin sea el doble de la raz de la
1 2
2da y el triple de la 3ra ecuacin? Comprobar el resultado. x a = (x + b), (3x b) = 2c x,
3 3
1
2x a = 2 (3x 2c) :

6. Mediante introduccin de nuevas variables o sustitucin, resolver los sistemas de ecuaciones:


8 8 8
> x 3 > 9 > 5 4
< = < 4x + = 21 < + =5
a) y 4 b) y c) 2x + y 2x 3y
> x 1 1 > 18 > 15 2
: = : 3x + = 17 : + =5
8 y + 2 2 y 2x + y 2x 3y
5 4 8 8
>
< + =9 < x=y =z < x l =y m=z n
d) 2x 1 3y + 2 e) p q r f) p q r
> 15 2 : ax + by + cz + d = 0 :
: = 13 ax + by + cz + d = 0
2x 1 3y + 2
306 CAPTULO 8. MATRICES, DETERMINANTES Y SISTEMAS LINEALES
(
3x + ky = 5 + k
7. Hallar los valores del parmetro k para que el sistema de ecuaciones posea solucin
2x + 5y = 8
( (
(1 + 2k) x + 5y = 7 (2k 1) x + ky = 6
nica. El sistema no tenga solucin. Y el sistema posea
(2 + k) x + 4y = 8 7;5x + 4y = 3
innito nmero de soluciones.

Resolver mediante reduccin o sustitucin los sistemas de ecuaciones lineales:


8 8
( > 2x + y 3z = 0 ( >
3x 2y = 1 < 2x 3y = 5 < 3x + y 3z = 1
8. a) , b) 2x y + 4z = 5 , c) , d) 2x 2y + z = 1
2x + y = 3 >
: 3x + 2y = 12 >
:
3x + 2y z = 4 3x + 2y 2z = 3
8 8
( > 2x + y 3z = 0 ( >
3x 2y = 2 < 2x + 3y = 5 < 3x y 3z = 1
9. a) , b) 3x 2y + 4z = 0 , c) , d) 2x + 2y 4z = 0
x + 3y = 4 >
: 3x 4y = 1 >
:
x + 2y 3z = 0 3x + 2y 3z = 2
8 8
8 > x y + 3z t = 2 > 2x y + 3z t = 3
( > 2x y + 3z = 4 >
> >
>
2x 3y = 8 < < 2x + y 2z + 2t = 3 < 4x + y 2z + t = 4
10. a) , b) 3x + y 2z = 2 , c) , d)
3x + 4y = 11 >
: >
> x + 2y z + t = 1 >
> x+y z t=0
x + 2y z = 0 >
: >
:
x + 2y z t = 1 x + 2y z = 2
Analizar y resolver los siguientes sistemas lineales:
8 8
< x=y =z < x l =y m=z n
11. Resolver los sistemas de ecuaciones: a) p q r , b) p q r
: ax + by + cz + d = 0 :
ax + by + cz + d = 0
(
3x + ky = 5 + k
12. Hallar los valores del parmetro k para que el sistema de ecuaciones posea solucin
2x + 5y = 8
(
(1 + 2k) x + 5y = 7
nica, interpretar el sistema geomtricamente. Para que valor de k, el sistema no
(2 + k) x + 4y = 8
(
(2k 1) x + ky = 6
tiene solucin?, interpretar el sistema geomtricamente. Para que valor de k, el sistema
7;5x + 4y = 3
posee innito nmero de soluciones?, interpretar el sistema geomtricamente.

Calcular los determinantes:

1 2 5 2 2 2 0 2
13. Calcular los siguientes determinantes: a) , b) , c) , d) ,
3 4 3 4 3 4 0 4

2 1 1 2 1 2 1 1 x 1 a b c 1
e) , f) , g) , h) , i) , j) , k) ,
3 5 8 3 2 4 3 3 y 3 b a c 1

x 1 x a 1 x 2 1+x 2x 1 3 + x
l) , m) , n) , o)
0 3 x+a 1 1 x x+3 3 x 2x + 1

x 1 2 5 2 2x 4
14. Resolver las siguientes ecuaciones: a) = 8, b) = 0, c) = 0,
3 4 3 4 x 8 4x
0 2 2x 3 2 5 2 2 (x 1) 4
d) =x 1, e) = 8, f ) = 0, g) =0
0 x 3 4 3 4 + 5x 8 4 (x 1)

1 2 1 1 0 1
15. Utilizando la regla de Sarrus, calcular los siguientes determinantes: a) 0 1 1 , b) 0 1 1 ,
2 0 1 2 0 1
8.2. SISTEMAS DE ECUACIONES LINEALES 307

0 2 1 0 0 1 2 2 1 1 2 1 5 0 1 0 2 1
c) 0 1 1 , d) 0 1 1 , e) 0 1 0 , f) 0 1 1 , g) 0 1 1 , h) 0 4 1 ,
2 0 1 2 0 1 2 0 1 2 0 0 2 0 1 2 0 1
0 0 1 8 2 1 2 3 1 1 4 1 2 0 1 1 2 1
i) 0 1 1 , j) 0 1 0 , k) 0 1 0 , l) 0 1 1 , m) 0 1 1 , n) 0 4 1
12 0 1 2 0 1 2 0 1 2 0 0 2 0 1 2 0 0

16. Utilizando la regla del clculo por menores, tomar una la o columna (por ejemplo, aquella que contenga
3 2 1 4 0 1 12 2 1
ms ceros), y calcular los determinantes: a) 0 1 1 , b) 0 1 1 , c) 0 1 1 ,
2 0 1 2 0 1 5 0 1
0 0 1 6 2 1 8 2 1 5 0 1 0 2 1 7 0 1
d) 0 5 1 , e) 0 1 0 , f) 0 1 1 , g) 0 1 1 , h) 0 1 1 , i) 0 1 1
2 0 1 2 0 1 2 0 0 2 0 1 1 0 1 2 0 1

17. Utilizando la regla del clculo por menores, tomar una la o columna (por ejemplo, aquella que contenga
ms ceros), calcular los determinantes:
2 1 1 0 3 1 1 0 6 1 1 0 4 1 1 0
1 0 0 4 1 0 0 4 1 0 0 4 1 0 0 4
a) , b) , c) , d) ,
0 5 3 0 0 5 3 0 0 5 3 0 0 5 3 0
0 4 5 1 0 4 5 1 0 0 5 1 0 4 5 1
2 1 1 6 3 1 1 4 3 1 1 0
1 0 0 4 1 2 0 4 1 0 0 4
e) , f) , g)
8 5 3 0 0 5 3 0 0 5 3 0
0 4 5 1 0 4 5 1 0 1 5 1
Utilizando el mtodo de Cramer, resolver los sistemas:
8 8 8 8
>
< 2x y + z = 2 >
< 3x y + 2z = 4 >
< x + 3y + z = 5 >
< x 3y + z = 1
18. a) x 2y + 3z = 2 , b) x 2y + 3z = 2 , c) 2x + 2y + 3z = 7 , d) 2x 2y + 3z = 3 ,
>
: >
: >
: >
:
x y + 4z = 4 2x y + 4z = 5 3x y + 4z = 6 3x y + 4z = 6
8 8 8
>
< 2x y + 2z = 4 >
< 3x + 3y + z = 5 >
< 2x + 3y + z = 6
e) x y + 3z = 4 , f ) x + 2y + 3z = 7 , g) x + 2y 3z = 0 :
>
: >
: >
:
3x y + 4z = 7 2x y + 4z = 6 2x + y + 4z = 7
8 8 8
>
> x 3y + z t = 0 >
> 2x y + 2z = 2 >
> 3x + 3y + z t = 2
>
< x 2y + 3z 2t = 1 >
< x y + 3z t = 2 >
< x + 2y + 3z = 3
19. a) , b) , c)
>
> x y + 4z = 1 >
> 3x y + 4z = 4 >
> 2x y + 4z = 4
>
: >
: >
:
2x y + 4z + 3t = 5 3x y + 4z + 3t = 7 2x y + 4z + t = 3
Utilizando el mtodo de Gauss, resolver los siguientes sistemas de ecuaciones lineales.
Adems, si los sistemas son cuadrados n n, resolverlos por el mtodo de Cramer:
8
( > ( (
3x 2y = 1 < 2x + y 3z = 0 x+y =8 2x 3y = 5
20. a) , b) 2x y + 4z = 5 , c) , d)
2x + y = 3 >
: 3x 2y = 1 3x + 2y = 12
3x + 2y z = 4
8
( > ( (
2x + 3y = 5 < 3x y 3z = 1 3x 2y = 1 2x 3y = 8
21. a) , b) 2x + 2y 4z = 0 , c) , d)
3x 4y = 1 >
: 2x + y = 3 3x + 4y = 11
3x + 2y 3z = 2
8 8
> ( ( >
< 2x y + 3z = 4 3x 2y = 1 2x 3y = 1 < 2x y + 3z = 4
22. a) 3x + y 2z = 2 , b) , c) , d) 3x + y 2z = 2
>
: 2x + 3y = 5 3x + 4y = 84 >
:
x + 2y z = 0 x + 2y z = 2
308 CAPTULO 8. MATRICES, DETERMINANTES Y SISTEMAS LINEALES

Figura 8.3: Izq. Andrei N. Kolmogorov (Kataev) (Tambov, 1903 - Mosc, 1987) historiador de la ciencia y uno de
los ms grandes matemticos soviticos del siglo XX. Gran especialista en balstica, series trigonomtricas, lgica y
teora de conjuntos, teora de funciones y anlisis funcional y especialmente teora de las probabilidades y procesos
estocsticos, donde introdujo la axiomtica de Kolmogorov para denir la probabilidad como medida. Der. Un joven
talento matemtico, ganador de la Medalla Fields en 1970, el equivalente al premio Nobel en matemticas, el sovitico
Sergei Novikov (Gorki, 1938 - ), especialista en geometra, topologa algebraica y lgica. Sus padres, Petr Novikov (1901
- 1975) y Liudmila Keldysh, y su to Mstislav Keldysh (1911 - 1978), fueron tambin famosos matemticos soviticos.

8 8 8 8
>
> 2x y + 3z = 4 >
> 2x y + 3z = 4 >
> 2x 2y + 3z = 3 >
> x y + 3z = 3
>
> >
> >
> >
>
>
> >
> >
> >
>
< x 2y + 3z = 2 < 3x 2y 3z = 2 < 2x y + 5z = 6 < 2x y + 3z = 4
23. a) 3x + y z = 3 , b) 5x + y 6z = 0 , c) 3x y 2z = 0 , d) 3x + 2y z = 4
>
> >
> >
> >
>
>
> 2x + 3y + 2z = 7 >
> x + 3y 3z = 1 >
> 4x y 3z = 0 >
> 2x + 3y + 2z = 7
>
> >
> >
> >
>
: x + 2y + 3z = 6 : x 2y + 3z = 2 : 2x + 2y 3z = 1 : x + 2y + 3z = 6
8 8 8 8
>
> x 2y + 3z = 0 >
> x y + 3z = 3 >
> 2x 2y z = 1 >
> x 2y + 3z = 0
>
> >
> >
> >
>
>
> >
> >
> >
>
< 2x 3y + 5z = 4 < 2x y + 3z = 4 < 3x 2y z = 0 < 2x 3y + 5z = 4
24. a) 4x y 2z = 1 , b) 3x + 2y z = 4 , c) 5x + y 6z = 0 , d) 4x y 2z = 1
>
> >
> >
> >
>
> 2x y 3z = 2
> >
> 2x + 3y + 2z = 1 >
> 2x + 3y 3z = 1 >
> 2x y 3z = 2
>
> >
> >
> >
>
: 2x + 2y 3z = 1 : x + 2y + 3z = 0 : x 2y + 3z = 0 : 2x + 2y 3z = 1
8 8 8
>
> x y + 3z + 2u = 5 >
> 2x z t = 0 >
> x 2y + 2z = 1
>
< 2x y + 3z + 2t = 6 >
< 3x 2y + 2u = 3 >
< 2x 3y + 5t = 4
25. a) , b) , c)
> 3x + 2y z + 3u = 7
> >
> y 8z + 2t = 0 >
> 4x y 2z + t = 2
>
: >
: >
:
3x 2t = 1 3z 2t + u = 2 2y 3z + 2u = 1
8 8 8
>
< x y + 3z t + 2u = 4 >
< 2x 2y z t = 2 >
< x 2y + 2z + u = 0
26. a) 2x y + 3z + 2t u = 5 , b) 3x 2y z + 2u = 2 , c) 2x 3y + 5z = 4
>
: >
: >
:
3x + 2y z + 3u = 7 5x + y 8z + 2t = 0 4x y 2z + t = 2
8 8 8
>
> 2y z + u = 2 >
> 2x y z + 6t = 6 >
> 2x y 2z + t = 0
>
> >
> >
>
>
> >
> >
>
< x z t= 1 < x 2y + 2z = 1 < x 2y + 2z = 1
27. a) 2x 2y + 2u = 2 , b) x 3y + t = 1 , c) x 3y + 5t = 3
>
> >
> >
>
>
> y 2z + 2t = 1 >
> x y 2z + t = 1 >
> x y 2z + t = 1
>
> >
> >
>
: z 2t + u = 0 : 2y z + 2u = 1 : y 3z + 2u = 0

8.3. Aplicacin: Descomposicin en Fracciones Parciales

Analizamos la clase de las funciones racionales, muy tiles en el clculo. Como veremos, algunas funciones
racionales pueden descomponerse en fracciones ms simples o fracciones parciales. Para realizar esta descom-
8.3. APLICACIN: DESCOMPOSICIN EN FRACCIONES PARCIALES 309

posicin es necesario factorar el denominador de la fraccin y luego, con ayuda del mtodo de los coecientes
indeterminados e igualacin, resolver un sistema de ecuaciones lineales que nos da los coecientes buscados de
tal descomposicin en fracciones parciales.

Toda funcin racional puede ser representada en la forma de fraccin racional, es decir, como la razn de
dos polinomios:
Qm (x) B0 xm + B1 xm 1 + ::: + Bm
=
fn (x) A0 xn + A1 xn 1 + ::: + An
Sin limitar la generalidad del razonamiento, supondremos que estos polinomios no tienen races comunes, es
decir, no admiten simplicacin, o como suele decirse, son primos entre si.

| Si el grado del numerador es inferior al del denominador, la fraccin se llama propia, caso contrario, la
fraccin se denomina impropia.

| Si la fraccin es impropia, al dividir el numerador por el denominador (segn la regla de divisin de


polinomios) se puede representar la fraccin dada como la suma de un polinomio y de una fraccin propia:
Qm (x) Fk (x)
= Mm n (x) + ;
fn (x) fn (x)
Fk (X)
donde, M (x) es un polinomio de grado m n, y es una fraccin propia, siendo el grado k de F menor
fn (x)
que el grado n de f .

Ejemplos:

x4 3
1. Sea una fraccin racional impropia.
x2 + 2x + 1
Al dividir el numerador por el denominador (segn la regla de divisin de 2 polinomios), obtenemos:
x4 3 4x 6
= x2 2x + 3
x2 + 2x + 1 x2 + 2x + 1
Como se ver, hemos obtenido un polinomio de menor grado (parte entera de grado m n = 2) y una
nueva fraccin propia o funcin racional (parte fraccionaria, k = 1).
2x7 3x 1
2. Sea 2 una fraccin racional impropia.
2x2
(x + 1) (x 2)
Al dividir el numerador por el denominador (segn la regla de divisin de dos polinomios), obtenemos:
2x7 3x 1 46x4 24x3 72x2 3x 1
2 = x2 + 3x + 9 + 2
2x2 (x + 1) (x 2) 2x2 (x + 1) (x 2)
Hemos obtenido un polinomio de menor grado (parte entera de grado m n = 2) y una nueva fraccin
propia o funcin racional (parte fraccionaria, k = 4).

Denicin. Las Fracciones Racionales propias del tipo:

A
I. ;
x a
A
II. (k 2 N; k 2)
(x a)k
Ax + B
III. (las races del denominador son complejas, es decir, p2 4q < 0)
x2 + px + q
Ax + B
IV. k
(k 2 N; k 2, y las races del denominador son complejas, p2 4q < 0),
(x2 + px + q)

se denominan fracciones simples del tipo I, II, III, IV, respectivamente.


310 CAPTULO 8. MATRICES, DETERMINANTES Y SISTEMAS LINEALES

8.3.1. Fracciones racionales y su descomposicin en fracciones parciales

Pasamos a demostrar que toda fraccin racional propia puede ser descompuesta en una suma de fracciones
simples, por ejemplo, del tipo I, II, III, IV.
F (x)
Sea una fraccin racional propia, donde los coecientes de los polinomios que la integran son nmeros
f (x)
reales y la fraccin dada es irreducible, es decir, el numerador F y el denominador f no tienen races comunes.

Teorema 1. Sea x = a una raz multiple de orden k del denominador, es decir, f (x) = (x a)k f1 (x) 6= 0.
F (x)
Entonces la fraccin propia dada se puede descomponer en una suma de dos fracciones propias:
f (x)
F (x) A F1 (x)
= + ; (1)
f (x) (x a)k (x a)k 1 f1 (x)
donde A es una constante, A 6= 0 y F1 (x) es un polinomio de grado inferior al grado del denominador
(x a)k 1 f1 (x):

Demostracin. Escribimos la identidad


F (x) A F (x) Af1 (x)
= k
+ ; (2)
f (x) (x a) (x a)k 1 f1 (x)
la misma que se verica para cualquier A. Denamos la constante A de modo que el polinomio F (x) Af1 (x)
sea divisible por (x a): En virtud del teorema de Bezout, es necesario y suciente que se verique la igualdad
F (a) Af1 (a) = 0. Como f1 (a) 6= 0 y F (a) 6= 0; podemos denir A de una manera unvoca mediante la
F (a)
igualdad A = . Para este A se cumple que F (x) Af1 (x) = (x a)F1 (x), donde F1 (x) es un polinomio
f1 (a)
de grado inferior al del polinomio (x a)k 1 f1 (x). Simplicando la fraccin en la frmula (2) por (x a),
obtenemos la igualdad (1).
F1 (x)
Corolario. Tambin a la fraccin racional propia que entra en la igualdad (1), se puede
(x a)k 1 f1 (x)
aplicar un razonamiento anlogo. As, si el denominador tiene una raz mltiple x = a de orden k, podemos
escribir:
F (x) A A1 Ak 1 Fk (x)
= k
+ k 1
+ ::: + + ;
f (x) (x a) (x a) x a f1 (x)
Fk (x)
donde es otra fraccin propia irreducible a la cual se puede aplicar el teorema anterior, esto si f1 (x)
f1 (x)
tiene otras races reales.

Ahora, analizamos el caso en el que el denominador posee races complejas. Hay que recordar que las races
p
complejas de un polinomio con coecientes reales estn conjugadas en pares, as, por ejemplo, si x1 = 1 3i es
3 2
___ p p
una raz del polinomio P3 (x) = x + x 2x + 12, entonces, el conjugado de x1 , x1 = x2 = 1 3i = 1 + 3i
tambin es una raz de P3 (x).

Hay que tomar en cuenta que todo polinomio Pn (x) de grado impar (n = 2k 1, k 2 N) y de coecientes
reales, siempre posee por lo menos una raz real.

En la descomposicin de un polinomio en factores reales, a cada par de races complejas conjugadas cor-
responde una expresin de segundo grado x2 + px + q. Si las races conjugadas son de orden m la expresin
correspondiente ser (x2 + px + q)m .

Teorema 2. Si f (x) = (x2 + px + q)m g1 (x); donde el polinomio g1 (x) no es divisible por x2 + px + q , la
F (x)
fraccin racional propia puede ser representada mediante la suma de dos fracciones propias
f (x)
F (x) Mx + N G1 (x)
= 2 + 2 ; (3)
f (x) (x + px + q)m (x + px + q)m 1 g (x)
1
8.3. APLICACIN: DESCOMPOSICIN EN FRACCIONES PARCIALES 311

donde G1 (x) es un polinomio de grado inferior al del polinomio (x2 + px + q)m 1


g1 (x)

Demostracin. En la identidad
F (x) F (x) Mx + N F (x) (M x + N )g1 (x)
= 2 = 2 + (4)
f (x) (x + px + q)m g1 (x) (x + px + q)m (x2 + px + q)m g1 (x)
que se verica para todo M y N , hallamos M y N de modo que el polinomio F (x) (M x + N )g1 (x) se divida
por x2 + px + q. Para esto es necesario y suciente que la ecuacin

F (x) (M x + N )g1 (x) = 0

tenga las mismas races i que el polinomio x2 + px + q. Por lo tanto, remplazando

F( i )
F( i ) [M ( i ) + N ] g1 ( i )=0 M( i )+N =
g1 ( i )
F( i )
Pero es cierto nmero complejo determinado que tendra la forma K + iL, donde K y L 2 R. As
g1 ( i )
L K L
M( i )+N = K +iL, de donde M +N = K y M = L M = , N = . Si estos son los valores
de los coecientes M y N , entonces el polinomio F (x) (M x + N )g1 (x) tiene por raz a i , y por tanto,
tambin a la raz conjugada i . En este caso, el polinomio es divisible sin resto por las diferencias x ( +i )
yx ( i ), y tambin por su producto, es decir, por x2 + px + q:

Representando el cociente de esta divisin por G1 (x), tendremos:

F (x) (M x + N )g1 (x) = (x2 + px + q)G1 (x)


Simplicando por x2 + px + q la ltima fraccin de la igualdad (4), obtendremos la igualdad (3), quedando claro
que G1 (x); es un polinomio de grado inferior al del denominador, lo que se deseaba demostrar.
F (x)
Si ahora aplicamos los resultados del teorema 1 y 2 a la fraccin propia , podemos descomponer y
f (x)
sealar sucesivamente todas las fracciones simples, correspondientes a todas las races del denominador f (x).

De todo lo anterior tenemos el siguiente resultado:

Si F (x) = (x a) (x b) : : : (x2 + px + q) : : : (x2 + lx + s) ;


F (x)
entonces la fraccin puede descomponerse de la manera siguiente :
f (x)
8
> F (x) A A1 A 1
>
> = + + + +
>
> f (x) (x a) (x a) 1 x a
>
>
>
> B B1 B 1
>
> + + + + +
>
> (x b) (x b) 1 x b
>
< ::::::::::::::::::::::::::::::::::::::::::::::::::::::::::::::::::::
Mx + N M1 x + N 1 M 1x + N 1 (5)
>
>
>
> + 2 + 2 1
+ + +
>
> (x + px + q) (x + px + q) x2 + px + q
>
>
>
> :::::::::::::::::::::::::::::::::::::::::::::::::::::::::::::::::::
>
>
>
> Px + Q P1 x + Q1 P 1x + Q 1
: + 2 + 2 + +
(x + lx + s) (x + lx + s) 1 x2 + lx + s

Podemos determinar los coecientes A, A1 ,...,B, B1 , etc. mediante un sistema de ecuaciones lineales. Como
la igualdad (5) es una identidad, entonces, al reducir estas fracciones a un comn denominador, obtendremos
en los numeradores del primero y del segundo miembros polinomios idnticos. Igualando los coecientes de los
trminos segn las mismas potencias de x, obtendremos un sistema de ecuaciones lineales cuya solucin sera
los coecientes buscados A, A1 ,...,B, B1 , etc.
312 CAPTULO 8. MATRICES, DETERMINANTES Y SISTEMAS LINEALES

Tambin podemos determinar A, A1 ,...,B, B1 , etc. por el mtodo de los coecientes indeterminados, con-
siderando que los polinomios obtenidos en ambos miembros de la igualdad, despus de la reduccin al comn
denominador, deben ser identicamente iguales, y por tanto, los valores que toman estos polinomios son iguales
para cada valor particular de x. Al tomar x ciertos valores particulares convenientes, obtenemos las ecuaciones
necesarias para la determinacin de los coecientes.

| Conclumos que toda fraccin racional propia puede ser representada como una suma de fracciones simples.

Ejemplos:

x2 + 3
1. Descomponer la fraccin en fracciones parciales simples.
(x + 1)3 (x 2)
Solucin: Por la frmula (5) tendremos:

x2 + 3 A A1 A2 B
= + + +
(x 1)3 (x + 2) (x 1)3 (x 1)2 (x 1) (x + 2)

Reduciendo a un comn denominador e igualando los numeradores tendremos:

x2 + 3 = A(x + 2) + A1 (x 1)(x + 2) + A2 (x 1)2 (x + 2) + B(x 1)3 (*)

o bien

x2 + 3 = Ax + 2A + A1 x2 + A1 x 2A1 + A2 x3 3A2 x + 2A2 + Bx3 3Bx2 + 3Bx B

x2 + 3 = (A2 + B)x3 + (A1 3B)x2 + (A + A1 3A2 + 3B)x + (2A 2A1 + 2A2 B)

igualando los coecientes de x3 , x2 , x1 , x0 , obtenemos el sistema de ecuaciones lineales para determinar


los coecientes: 8
> 0 = A2 + B
>
>
< 1=A
1 3B
>
> 0 = A + A1 3A2 + 3B
>
:
3 = 2A 2A1 + 2A2 B

La solucin del sistema lineal anterior es: A = 34 ; A1 = 29 ; A2 = 7


27 ; B= 7
27 .

Algunos coecientes, tambin pueden ser determinados a partir de las ecuaciones que se obtienen de la
igualdad (*), que es una identidad con respecto a x Cuando a la variable x le asignamos ciertos valores
particulares:
Para x = 1, de (*) tenemos que 4 = 3A, es decir A = 43 . Para x = 2, de (*) tenemos que 7 = 27B;
7
B = 27 .
Si adjuntamos a estas 2 ecuaciones otras 2 ya obtenidas mediante igualacin de los coecientes de las
mismas potencias de x, obtendremos 4 ecuaciones para determinar los otros 2 coecientes desconocidos.
Finalmente, tendremos la descomposicin:

x2 + 3 4 2 7 7
= 3 + 2 +
(x + 1)3 (x 2) 3 (x + 1) 9 (x + 1) 27 (x + 1) 27 (x 2)

x4 + 1
2. Descomponer en fracciones parciales.
(x2 5x + 6)(x2 + 2)
Solucin: Primero notamos que no es una fraccin propia, pues

x4 + 1 5x3 8x2 + 10x 11


=1+ 2
(x2 2
5x + 6)(x + 2) (x 5x + 6) (x2 + 2)
8.3. APLICACIN: DESCOMPOSICIN EN FRACCIONES PARCIALES 313

Ahora descomponiendo el denominador en factores, tendremos que

5x3 8x2 + 10x 11 5x3 8x2 + 10x 11


1+ = 1 + ;
(x2 5x + 6) (x2 + 2) (x 3)(x 2) (x2 + 2)
de donde se ve claramente que la descomposicin en fracciones parciales tendr la forma

5x3 8x2 + 10x 11 A B Cx + D


2
= + + 2
(x 3)(x 2) (x + 2) (x 2) (x 3) (x + 2)
Haciendo comn denominador e igualando los numeradores, tendremos la identidad

5x3 8x2 + 10x 11 = A (x 3) (x2 + 2) + B (x 2) (x2 + 2) + (Cx + D) (x 2) (x 3) ;

es decir que

5x3 8x2 + 10x 11 = Ax3 + 2Ax 3Ax2 6A + Bx3 + 2Bx 2Bx2


4B + Dx2 5Dx + 6D + Cx3 5Cx2 + 6Cx

Agrupando tendremos

5x3 8x2 + 10x 11 = (C + B + A) x3 + (D 3A 5C 2B) x2 +


+ (6C + 2A + 2B 5D) x + 6D 6A 4B

Igualando coecientes tendremos el sistema lineal


8
>
> 5=A+B+C
>
< 8 = 3A 2B 5C + D
>
> 10 = 2A + 2B + 6C 5D
>
:
11 = 6A 4B + 6D

25 10 17 82
Cuya solucin es: C = ,D= ,A= ,B= :
66 33 6 11
La descomposicin denitiva toma la forma

x4 + 1 5x3 8x2 + 10x 11


= 1 + =
(x2 5x + 6)(x2 + 2) (x2 5x + 6) (x2 + 2)
17 82 5 4 + 5x
=1 + +
6 (x 2) 11 (x 3) 66 (x2 + 2)

4x6 10x5 + 6x4 13x3 + 18x2 6x + 5


3. Descomponer en fracciones parciales.
2
(x3 5x2 ) (x2
+ 1)
Solucin: Por descomposicin en factores del denominador nos da

4x6 10x5 + 6x4 13x3 + 18x2 6x + 5 A B C Dx + E Fx + G


2 = + 2+ + 2 + 2
x2 (x 5) (x2 + 1) x x (x 5) (x + 1) (x + 1)2

Ahora igualando los numeradores, tendremos que

4x6 10x5 + 6x4 13x3 + 18x2 6x + 5 = Ax (x 5) (x2 + 1)2 + B (x 5) (x2 + 1)2 +

+ (Dx + E) x2 (x 5) (x2 + 1) + (F x + G) x2 (x 5)
Ax (x 5) (x2 + 1)2 + B (x 5) (x2 + 1)2 + (Dx + E) x2 (x 5) (x2 + 1) + (F x + G) x2 (x 5)
Agrupando trminos semejantes, tendremos

4x6 10x5 + 6x4 13x3 + 18x2 6x + 5 = (D + A) x6 + (B + E 5D 5A) x5 +

+ (2A 5B + F 5E + D) x4 + (2B + E 5D + G 5F 10A) x3 +


314 CAPTULO 8. MATRICES, DETERMINANTES Y SISTEMAS LINEALES

+ (A 5G 10B 5E) x2 + ( 5A + B) x 5B

Igualando los coecientes de las respectivas potencias, tendremos el sistema lineal 7 7


8
>
> 4=A+D
>
>
>
> 10 = 5A + B 5D + E
>
>
>
>
< 6 = 2A 5B + D 5E + F
13 = 10A + 2B 5D + E 5F + G
>
>
>
> 18 = A 10B 5E 5G
>
>
>
> 6 = 5A + B
>
>
:
5 = 5B

Cuya solucin es: A = 1, B = 1, C = 2, D = 1, E = 1, F = 1, G = 2.


En denitiva, tendremos la descomposicin:

4x6 10x5 + 6x4 13x3 + 18x2 6x + 5 1 1 2 x+1 x+2


2 = + +
x2 (x 5) (x2 + 1) x x2 x 5 (x2 + 1) (x2 + 1)2

Descomposicin de fracciones racionales

Q(x)
Supongamos que se desea calcular la descomposicin correspondiente de la fraccin racional , donde
f (x)
Q (x) y f (x) son polinomios sin races comunes. Si la fraccin dada es impropia , la representamos como suma
F (x) F (x)
de un polinomio M (x) y una fraccin racional propia . Ahora, la fraccin puede representarse
f (x) f (x)
como una suma de fracciones parciales simples (segn la seccin anterior).

Caso I Las races del denominador son reales y diferentes, es decir f (x) = (x a)(x b) : : : (x d)
F (x)
En este caso la fraccin se descompone en fracciones simples del tipo I, y
f (x)

F (x) A B D
= + + +
f (x) x a x b x d

Ejemplos:

2x2 5x 1
1. Descomponer
x3 x
2x2 5x 1 A B C
Solucin: Tenemos que = + + , de donde
x3 x x x 1 x+1
2x2 5x 1 = A (x 1) (x + 1) + Bx (x + 1) + Cx (x 1)

Poniendo en la anterior identidad los valores x = 0, x = 1 y x = 1, tendremos:


8
>
< 1= A
4 = 2B
>
:
6 = 2C

2x2 5x 1 1 2 3
En denitiva 3
= + :
x x x x 1 x+1
5x 1
2. Descomponer
x2 + x 30
5x 1 A B
Solucin: Tenemos que = + , de donde 5x 1 = A (x 5) + B (x + 6).
x2 + x 30 x+6 x 5
8.3. APLICACIN: DESCOMPOSICIN EN FRACCIONES PARCIALES 315

Poniendo en la anterior identidad los valores x = 5, x = 6, tendremos:


( (
24 = 11B 24=11 = B
)
31 = 11A 31=11 = A

De donde
5x 1 31 24
= + :
x2 + x 30 11 (x + 6) 11 (x 5)

Caso II Las races del denominador son reales, pero algunas races son mltiples, es decir f (x) = (x
a) (x b)n : : : (x d)k .
m

F (x)
En este caso la fraccin se descompone en fracciones simples del tipo I y II.
f (x)

Ejemplos:

z+1
1. Descomponer 2
z2 (z 1)
z+1 A B C D
Solucin: Tenemos que 2 = + 2+ + , de donde
z2 (z 1) z z z 1 (z 1)2

2 2
z + 1 = Az (z 1) + B (z 1) + Cz 2 (z 1) + Dz 2

Poniendo en la anterior identidad los valores z = 0, z = 1, z = 1 y z = 2, tendremos el sistema lineal:


8
>
> 1=B
>
< 2=D
>
> 0 = 4A + 4B 2C + D
>
:
3 = 2A + B + 4C + 4D

Cuya solucin es: A = 3, B = 1, C = 3, D = 2. De aqu tendremos


z+1 3 1 3 2
2 = + 2 + 2:
z2 (z 1) z z z 1 (z 1)

x2 + 2
2. Descomponer
(x + 1)3 (x 2)
x2 + 2 A B C D
Solucin: Se tendra = + + + , de donde A = 1,
(x + 1)3 (x 2) (x + 1)3 (x + 1)2 (x + 1) (x 2)
B = 1=3, C = 2=9, D = 2=9. De aqu tenemos

x2 + 2 1 1 1 2 1 2 1
= + + :
(x + 1)3 (x 2) (x + 1)3 3 (x + 1)2 9x+1 9x 2

Caso III El denominador tiene races complejas simples, es decir, diferentes:

f (x) = (x2 + px + q)(x2 + lx + s) : : : (x a)m : : : (x d)k

F (x)
Para este caso la fraccin se descompone en fracciones simples del tipo I, II y III.
f (x)

Ejemplos:

x
1. Descomponer
(x2 + 1)(x 1)
316 CAPTULO 8. MATRICES, DETERMINANTES Y SISTEMAS LINEALES

x Ax + B C
Solucin: Tendremos que = 2 + , y por consiguiente, x = (Ax + B)(x 1) +
(x2 + 1)(x 1) x +1 x 1
2
C(x +1). Poniendo en la ltima igualdad, sucesivamente x = 0, x = 1 y x = 1, tendremos: 0 = B +C,
C = 1=2 y 2 = 2A 2B, respectivamente. Resolviendo este sistema obtenemos: C = 1=2, B = 1=2 y
A = 1=2.
Obtenemos nalmente
x 1 x 1 1 1
= +
(x2 + 1)(x 1) 2 x2 + 1 2 x 1
x+1
2. Descomponer
(x2
+ x + 1)(x 1)
(x + 1) Ax + B C
Solucin: Ponemos 2 = + , de donde, se debe cumplir que
(x + x + 1)(x 1) x2 + x + 1 x 1
8
>
< 0=A+C
x + 1 = (Ax + B) (x 1) + C x2 + x + 1 , 1= A+B+C
>
:
1= B+C

que se resuelve para los valores A = 2=3, B = 1=3, C = 2=3, es decir


x+1 1 2x + 1 2
= + :
(x2 + x + 1)(x 1) 3 x2 + x + 1 3 (x 1)

Caso IV El denominador contiene varias races complejas mltiples:

f (x) = (x2 + px + q)m (x2 + lx + s)n : : : (x a)l : : : (x d)k

F (x)
Para este caso las fraccines simples del tipo IV entran tambin en la descomposicin de la fraccin :
f (x)

Ejemplo:

x4 + 4x3 + 11x2 + 12x + 8


Descomponer
(x2 + 2x + 3)2 (x + 1)
Solucin: Descomponiendo en fracciones simples

x4 + 4x3 + 11x2 + 12x + 8 Ax + B Cx + D E


2 2
= 2 2
+ 2 + ;
(x + 2x + 3) (x + 1) (x + 2x + 3) x + 2x + 3 x + 1
tenemos por igualacin de numeradores

x4 + 4x3 + 11x2 + 12x + 8 = (Ax + B)(x + 1)+

+(Cx + D)(x2 + 2x + 3)(x + 1) + E(x2 + 2x + 3)2

Combinando los dos mtodos dados para determinar los coecientes, hallaremos que: A = 1, B = 1, C = 0,
D = 0, E = 1 De donde tendremos:

x4 + 4x3 + 11x2 + 12x + 8 x 1 1


= 2 + :
(x2 + 2x + 3)2 (x + 1) (x + 2x + 3)2 x+1

Ejemplos:

52x5 9x4 41x3 6x2 1


1. Descomponer f (x) = 2
x2 (2x + 1) (x2 x + 1)
8.3. APLICACIN: DESCOMPOSICIN EN FRACCIONES PARCIALES 317

Solucin: Descomponiendo en fracciones parciales, que queda en manos del lector, tendremos

52x5 9x4 41x3 6x2 1 A B C D Ex + F


2 = 2
+ + 2 + + 2 :
x2 (2x + 1) (x2 x + 1) x x (2x + 1) (2x + 1) x x+1

Resolviendo el respectivo sistema lineal, obtendremos

1 3 1 2 11x 13
f (x) = + + + 2 :
x2 x (2x + 1)2 (2x + 1) x x+1

1
2. Descomponer f (x) = 2 2
x (x2 + 2x + 1) (x3 1)
Solucin: Descomponiendo en fracciones parciales

1 1
2 2 = 2 2 2 =
x (x2 + 2x + 1) (x3 1) x (x + 1) (x 1) (x2 + x + 1)

A B C D E Fx + G Hx + I
= + 2 + + 2 + + 2 +
x (x + 1) x + 1 (x 1) (x 1) x + x + 1 (x + x + 1)2
2

Resolviendo el respectivo sistema lineal, que queda en manos del lector, obtendremos

1 1 1 1 1 x 5 1
f (x) = 2 + +1
x 4 (x + 1) x + 1 36 (x 1)2 9 (x 1) 9 x2 + x + 1 3 (x2 + x + 1)
2

2x8 5x7 x6 + 10x5 20x4 + 17x3 10x2 + 4x 1


3. Descomponer f (x) = 2 2
x3 (x 1) (2x2 x + 1)
Descomponemos en fracciones parciales la fraccin

2x8 5x7 x6 +10x5 20x4 +17x3 10x2 +4x 1 A B C D E F x+G Hx+I


x3 (x 1)2 (2x2 x+1)2
= x3 + x2 + x + x 1 + (x 1)2
+ 2x2 x+1 + (2x2 x+1)2

Resolviendo el respectivo sistema lineal, que queda en manos del lector, obtendremos

1 1 1 x 1 2
f (x) = 3
+ 2 + 2
x x 1 (x 1) 2x2 x + 1 (2x2 x + 1)

Practico lo que aprend:

Descomponer la parte izquierda en fracciones parciales y comprobar las igualdades:

3x 1 1 5 x+3 5 4
1. a) = + , b) =
4x2 1 4 (2x 1) 4 (2x + 1) 9x2 1 3 (3x 1) 3 (3x + 1)

x2 3x 18 28 x2 + x 1 1 11 1
2. a) = + 1, b) = + +
x2 + 7x + 12 x+3 x+4 2x (x 3) 6x 6 (x 3) 2
2 1 2 1 x 3 3 1 1 5
3. a) = + , b) 2 2 = 2 +
x (x2 1) x 1 x x+1 x (x 4) 4x 4x 16 (x 2) 16 (x + 2)
1 1 x+2 x 1 1 x
4. a) 2
= , b) = +
(x 2) (x + 1) 5 (x 2) 5 (x2 + 1) (x 1) (x2 x + 1) x 1 x2 x+1
x+1 2 1 x 2
5. 2 = 2 +
(x 1) (x2 x + 1) (x 1) x 1 x2 x+1

1 1 1 1 x x 1
6. 3 2 = 2 + 3 2
(x 1) (x2 + 1) 2 (x 1) 2 (x 1) 4 (x 1) 2 (x2 + 1) 4 (x2 + 1)
Descomponer las fracciones en fracciones parciales con coecientes reales:
318 CAPTULO 8. MATRICES, DETERMINANTES Y SISTEMAS LINEALES

7. Descomponer en fracciones parciales:

2 4x4 + 3x + 4 3x2 + 6x 13
a) b) c)
9x2 4 x3 + 3x2 + 2x x2 + 2x 3
1 1 x x2 3x5 + 2
d) e) f)
x4 1 x + 4x2 + 5x + 2
3 x + 3x3 + 2x2
4
1 x3 4x2 + 3x 1 x3 + 3x2 + 4x
g) 2
h) i)
3x 5x 2 x5 3x4 + 3x3 x2 x4 + 2x3 2x 1
1 3 1 + x x2
j) 2 k) l)
(x3 4x) x3 + 2x x2 2 x3 + 2x2 + x + 2

2x 1 2x x+1 x 3 2x + 3
8. , , , , :
x (x2 4) x2 9 (x 1) (x + 5) (x 1) (x + 5) (x2 x) (x2 + 3x)
x+1 3x 1 x+1 x+2
9. 2, 2 2, 2 2, 2 :
x2 (x 1) x2 (x 2) (x + 1) (x2 x) (2x + 5) (2x2 + x) (x3 + x2 )
x 1 x x 1 x+2 x 2
10. 2, , , , ,
x (x2 + 1) x3 + 1 x3 8 x (x2 + 1)2 x6 1 (x2 x + 2) (x3 + x2 )

Figura 8.4: Izq. El famossimo matemtico y fsico terico ingls, especialista en los agujeros negros y teora general de
la relatividad, sucesor de Newton (Woolsthorpe, 1642 - Londres, 1727) en Cambridge, Stephen Hawking (Oxford, 1942
-). Padece de la rara enfermedad de esclerosis lateral amiotrca. Se comunica con el resto del mundo va computadora,
mediante seales y pulsaciones. Es autor de la famosa Breve historia del tiempo y otras famosas obras sobre la teora
general de la relatividad donde analiza la estructura espacio - tiempo, la gravedad, los pasos de gusano, las dimensiones,
etc. Der. Ernst Zermelo (Berln, 1871 - Friburgo, 1953). Tuvo un rol muy importante en la axiomatizacin y el desarrollo
de la teora de conjuntos. Es muy famoso el Teorema de Zermelo: Para todo conjunto E , existe una relacin de orden
que hace de E un conjunto bien ordenado. Este teorema es equivalente al axioma de eleccin: Para toda aplicacin f de
un conjunto A en el de las partes de un conjunto B tal que, para todo elemento a 2 A, f (a) es no vaco, existe una
aplicacin g de A en B tal que para todo elemento a 2 A, g (a) pertenece a f (a) :
Captulo 9

TRIGONOMETRA I

La matemtica es la ciencia que obtiene conclusiones necesarias.

Benjamin Peirce

La edad de oro de la matemtica no fue la poca de Eucldes, sino la nuestra.

C.J. Keyser

Unidad de competencia: Identicar y aplicar principios, deniciones, propiedades y resultados referidos al


estudio trigonomtrico del tri ngulo en situaciones pr cticas.

Indicadores de logro:

| Reconocer el concepto, generacin y mtodos de medicin de un ngulo y arcos de circunferencia.

| Conocer y comprender los conceptos fundamentales de crculo trigonomtrico y funciones trigonomtricas.

| Utilizar estrategias metacognitivas para hallar las funciones trigonomtricas de ngulos notables.

| Usar estrategias, datos y mtodos trigonomtricos para obtener los lados y/o ngulos en un tringulo rectngulo.

| Aprender los mtodos de escritura, identicacin y construccin de ngulos bajo varias condiciones.

| Resolver variados tipos de problemas y aplicaciones trigonomtricas.

Figura 9.1: Componentes o parmetros de un tringulo: lados (BC = a, AC = b, AB = c), ngulos (]A = ,
]B = , ]C = ).

319
320 CAPTULO 9. TRIGONOMETRA I

9.1. Introduccin y Funciones Trigonomtricas

| Dado cierto tringulo plano, los objetivos fundamentales de la trigonometra consisten en hallar los
parmetros (lados, ngulos, vrtices, orientacin, relaciones entre ngulos, relaciones entre lados, etc.) descono-
cidos del tringulo, conociendo algunos de ellos (Figura 9.1). Por ejemplo, conociendo los lados de un tringulo,
podemos hallar sus ngulos; conociendo el rea y 2 ngulos, encontramos sus lados, etc. Este tipo de clculos
surgieron de las necesidades planteadas por la astronoma (astrologa), la navegacin, la agrimensura, etc. No
es de asombrarse de que la trigonometra, por siglos, haya sido considerada como una parte de la astronoma y
de la geometra, que en sus inicios, como lo dice su nombre, se dedicaba a hacer mediciones terrestres.

| Los antiguos griegos ya haban desarrollado mtodos para encontrar los parmetros desconocidos de
un tringulo, sus astrnomos desarrollaron tablas para relacionar los ngulos con las longitudes de arco de
una circunferencia. Posteriormente fueron los sabios indes y rabes los que introdujeron los conceptos de
seno, coseno, tangente, etc, y sus respectivas tablas aproximadas de los valores de las razones entre los lados
del tringulo. En la Edad Media, los ngulos fueron ya medidos en radianes, introducindose las funciones
trigonomtricas, directas e inversas, de argumentos numricos. Despus de esto, la trigonometra, fue poco a
poco, adquiriendo su aspecto contemporneo.

Presentamos una visin de la trigonometra plana elemental, basada en nociones geomtricas intuitivas.
Antes de pasar a las deniciones de las funciones trigonomtricas, estudiaremos los ngulos y su medicin.

Figura 9.2: Izq. El gran apasionado de las matemticas, el francs Charles Hermite (Dieuze,1822 - Pars, 1901).
Uno de los profesores de Poincar. Estudioso de la resolucin de la ecuacin de quinto grado y gran especialista en
teora de nmeros (nmeros transcendentes), inventor de las frmulas, matrices y polinomios hermticos. Der. La
famosa matemtica y lsofa Hypatia de Alexandra (370 - 415), lapidada por las turbas por su rechazo a convertirse al
cristianismo.

9.1.1. Concepto de ngulo y su medicin

Geomtricamente, un ngulo viene a ser la gura formada por 2 rayos o semirectas de un mismo plano, que
parten de un mismo punto. Aqu no se hace distincin entre sus 2 lados, pues los ngulos AOB y BOA se
consideran iguales. Adems, geomtricamente no se consideran ngulos negativos.

Pasamos a generalizar el concepto de ngulo, de tal manera que este pueda ser: positivo, negativo o nulo.
Trazamos un eje Ox (que podra ser horizontal) desde cierto punto (origen) O, tomamos cierto valor r > 0
(radio) y trazamos, con centro en O, una circunferencia C0 que corte al eje Ox en el punto A. El segmento
!
dirigido que une el centro O con un punto arbitrario M 2 C0 , lo denominaremos radio vector OM . El ngulo
!
M OA = viene generado por la rotacin del radio vector OM en el sentido antihorario. OA es el lado o
posicin inicial, OM es el lado o posicin nal (Figura 9.3).
9.1. INTRODUCCIN Y FUNCIONES TRIGONOMTRICAS 321

Denicin. El ngulo se considera positivo ( > 0) si este est generado por la rotacin del radio vector
!
OM en el sentido antihorario. se considera negativo ( < 0) si este est formado por la rotacin del radio
!
vector OM en el sentido horario.

!
Figura 9.3: Angulo generado por el radio vector OM :

!
| Sin embargo, el extremo M del radio vector OM no solamente corresponde al giro que genera al ngulo
!
, sino a un innito nmero de ngulos generados al girar (en sentido positivo o negativo) el radio vector OM
un nmero entero de revoluciones completas, pues el extremo M invariablemente caer en el mismo lugar. Estos
ngulos vienen dados por las frmulas = + 360 k, k 2 Z en grados, o = + 2 k, k 2 Z en radianes. El
!
entero k nos indica el nmero de revoluciones completas que puede girar el vector OM junto con su extremo
M , para volver a caer en M , tanto en el sentido positivo (k > 0), como en el sentido negativo (k < 0). Si k = 0
no existe ninguna rotacin, excepto el giro original = . El valor de k es cualquier entero 0, 1, 2, : : :

Ejemplo:
!
Si el radio vector OM gira en sentido positivo un ngulo = 75 con respecto al lado inicial OA, entonces
!
al radio vector OM le corresponden un innito nmero de ngulos. Si k 0, entonces tendremos: = = 75 ,
para k = 0; = 435 , para k = 1; = 795 , para k = 2; etc. Si k < 0, tendremos: = 285 , para k = 1;
= 645 , para k = 2; etc. Resumiendo:

= + 360 k ) = 75 + 360 k; k2Z

Medicin de ngulos

| En el anterior punto se ha visto que una circunferencia se la puede dividir en 360 partes iguales, denom-
inndose cada parte o ngulo generado, grado sexagesimal o grado angular (1 ). Este sistema de medicin
de ngulos se remonta a los Babilonios. En este sistema la circunferencia comprende 4 cuadrantes iguales de
90 grados cada uno. La media circunferencia corresponde a 180 .

| Aparte de la medida angular o sexagesimal anterior, tenemos la medicin centecimal, que divide a una
circunferencia en 400 partes iguales. Cada parte o ngulo se denomina grado centecimal (1 c). En este caso,
cada cuadrante corresponde a 100 c y la media circunferencia corresponde a un ngulo de 200 c.

Ejemplos:

1. Transformar los grados sexagesimales: 15 , 45 , 60 , 90 , 120 , 150 , 10 , 60 , 420 , 750 a centecimales.


Solucin: Sea x el valor en grados sexagesimales, y el valor en grados centecimales, tenemos la proporcin

x 360
= ;
y 400 c
322 CAPTULO 9. TRIGONOMETRA I

de donde, despejando el valor buscado de y, tendremos:

400 c 10 c
y= x= x
360 9

Los resultados respectivos los ponemos en forma de tabla:


x 15 45 60 90 120 150 -10 -60 420 750
y c 16.66 50 66.66 100 133.33 166.66 -11.11 -66.66 466.66 833.36

2. Transformar los grados centecimales: 25 c, 100 c, 150 c; 360 c, 480 c, 880 c a grados sexagesimales.
Solucin: Llamando x al valor en grados sexagesimales, y al valor en grados centecimales, tenemos la
proporcin
x 360
= ;
y 400 c
360 9
de donde, despejando el valor buscado de x, tendremos: x = 400 c y = 10 c y. Los resultados
respectivos los ponemos en forma de tabla:
y c 25 100 150 360 480 880
x 22;5 90 135 324 432 792

| La forma ms natural y de mayor aplicacin para medir los ngulos es la que utiliza los radianes. Se
denomina radin al ngulo central cuyo respectivo arco tiene una longitud igual al radio.

OA N
Figura 9.4: Descripcin del ngulo en radianes: OA1 = N1 :

Supongamos que es el ngulo central (Figura 9.4) al que corresponde los arcos de circunferencia AC0 B y
AC1 B1 , de radios OA = r0 y OA1 = r1 . Designando a la longitud del arco AC0 B mediante l0 , y a la longitud
l0 l1
del arco A1 C1 B1 mediante l1 (ver gura), mostremos que se cumple la relacin: = . Efectivamente, para
r0 r1
calcular la longitud de un arco l de circunferencia, sabemos que el valor de toda la circunferencia es C = 2r
(es decir, C = diametro ), que adems corresponde a un ngulo de 360 . Por lo tanto tenemos la relacin:

2r l r
= )l=
360 180
r0 r1
De aqu (Fig. 9.4), tendremos que l0 = y l1 = respectivamente, y como es el mismo para ambos
180 180
casos, entonces despejando , tendremos

l0 180 l1 180 l0 l1
= = ) = =
r0 r1 r0 r1
Por tanto, para un ngulo central , la razn de la longitud del arco a la longitud del radio es constante. El
l0
valor correspondiente a cierto ngulo central se denomina medida en radianes del ngulo . La unidad
r0
l0
de medida en radianes, corresponde al caso en el que = 1, es decir cuando = 1. Que se denomina 1 radin.
r0
9.1. INTRODUCCIN Y FUNCIONES TRIGONOMTRICAS 323

Convendremos en escribir el valor en radianes de un ngulo , mejor sin ayuda de la palabra rad. As, por
ejemplo = 2;5 signica que el ngulo mide 2;5 radianes.

Ejemplos:

1. Hallar el valor de 1 radin en grados.


Solucin: Conociendo que la circunferencia contiene al radio 2 veces (pues C = 2 r), entonces tenemos
que 360 = 2 rad.) 180 = rad., de donde, en 1 rad. se tendrn
180 180
1 rad. = ' ' 57;296 ' 57 170 44;800
3;1416

| Recuerda: Si es el valor de un ngulo medido en grados, y a es su valor medido en radianes, entonces


las relaciones para transformar grados en radianes y viceversa, son:
a 180a
= )a= o =
180 180

3 3
2. Hallar los valores en grados de 1;5 rad., rad., rad.
5 2
3
180a 180 1;5 180
Solucin: Utilizando la relacin = , tenemos que: = = 85;944 , = 5 = 108 ,
3
180
= 2 = 270 , respectivamente.

3. Transformar a radianes los ngulos de 15 , 75 , 540


15 75 5
Solucin: Utilizando la relacin a = , tendremos que: a = = (o 0;2618 rad.), a = =
180 180 12 180 12
540
(o 1;309 rad.), a = = 3 (o 9;4248 rad.), respectivamente.
180
4. Presentamos algunas equivalencias entre varios valores de ngulos en grados y radianes:
30 45 60 90 120 135 150 180 270 360
2 3 5 3
a (rad:) 6 4 3 2 3 4 6 2 2
| La longitud de un arco de circunferencia, correspondiente a un ngulo central dado, sale de la relacin
2r l 2r l
ya conocida = ) = ) l = r, donde , debe estar dado en radianes.
360 2

Ejemplo:

Calcular la longitud l de un arco de circunferencia, correspondiente a un ngulo central de 45;5 , si se


conoce que el radio r de la circunferencia es de 50 cm.
45;5
Solucin: Primero transformamos = 45;5 a radianes. Tenemos a = = ' 0;79413 rad. Mediante
180 180
l = r tendremos l = 0;79413 50 ' 39;707 cm.

9.1.2. Denicin de las funciones trigonomtricas

| Las funciones trigonomtricas de un ngulo agudo , (0 < < 90 ), vienen denidas como razones entre
los lados de un tringulo rectngulo, es decir, entre catetos e hipotenusa (Figura 9.5, izquierda). Las funciones
trigonomtricas son 6: 3 directas (seno, coseno y tangente) y sus 3 respectivas inversas (cosecante, secante
y cotangente). Las funciones trigonomtricas, seno, tangente y secante tienen sus respectivas cofunciones,
coseno, cotangente y cosecante, y viceversa:
324 CAPTULO 9. TRIGONOMETRA I

Recuerda las funciones trigo-


nomtricas en el tringulo
cateto opuesto a hipotenusa c
sin = = csc = =
hipotenusa c cateto opuesto a
cateto adyacente b hipotenusa c
cos = = sec = =
hipotenusa c cateto adyacente b
cateto opuesto a cateto adyacente b
tan = = cot = =
cateto adyacente b cateto opuesto a

Las anteriores deniciones, basadas en las relaciones entre los lados de un tringulo rectngulo, carecen de
sentido para valores de no pertenecientes al intervalo ]0; 2 [. En un tringulo rectngulo, carecen de sentido,
por ejemplo, las funciones de ngulos, tales como 0 , 120 , 150 , 30 , 95 , etc., pues no es posible construr
un tringulo rectngulo con uno de esos ngulos.

p
Figura 9.5: Izquierda: tringulo rectngulo con catetos a y b, hipotenusa c = a2 + b2 y ngulo agudo .
Derecha: denicin del ngulo en el crculo trigonomtrico.

| Para denir las funciones trigonomtricas de un ngulo cualquiera, utilizamos el crculo trigonomtrico.
Trazamos 2 ejes mutuamente perpendiculares Ox y Oy. Desde su interseccin O, con un radio r = 1 (r puede
ser arbitrario), trazamos una circunferencia que corte al eje Ox en el punto A (Figura 9.5, derecha).
!
Sea M un punto de la circunferencia C0 , correspondiente al ngulo generado y al radio vector ! r = OM =
fx; yg. El lado inicial del ngulo ser siempre OA y su lado nal ser OM , de esta manera, el ngulo estar
generado por el vector ! r , al girar este en el sentido positivo, y partiendo desde el eje Ox. Mediante r = j!
r j,
! !
representaremos al mdulo o longitud del radio vector r , es decir, dist (0; M ) = r = j r j > 0. Ms tarde,
consideraremos r = j! r j = 1.

| Considerando lo anterior, pasamos a denir las funciones trigonomtricas de un ngulo cualquiera :

Recuerda las funciones tri-


gonomtricas en el crculo
sin = y=r csc = r=y
cos = x=r sec = r=x
tan = y=x cot = x=y

(
x = r cos
| De las 2 primeras frmulas se deduce que:
y = r sin

| Para la circunferencia unitaria (radio vector !


r unitario), es decir, para r = 1, las funciones trigonomtricas
sin = y csc = 1=y
vienen denidas en la forma: cos = x sec = 1=x
tan = y=x cot = x=y
9.1. INTRODUCCIN Y FUNCIONES TRIGONOMTRICAS 325

9.1.3. Signos de las funciones trigonomtricas

Como los ejes coordenados Ox y Oy dividen al crculo trigonomtrico unitario en 4 cuadrantes, entonces
las funciones trigonomtricas cambian de signo, en dependencia de la posicin del extremo M del radio vector
!r = fx; yg, r = 1. De acuerdo a las deniciones anteriores de las funciones trigonomtricas, es fcil ver que se
tienen los siguientes signos (Figura 9.6):

FuncinnCuadrante I II III IV
sin ; csc + +
cos ; sec + +
tan ; cot + +

Figura 9.6: Crculo trigonomtrico, ngulo y cuadrantes con sus respectivos signos I (+; +), II ( ; +),
III ( ; ), IV (+; ). El primer signo corresponde a las abscisas x y el segundo corresponde al signo de las
ordenadas y:

9.1.4. Variacin de las funciones trigonomtricas para 2 [0; 2 [

Pasamos a analizar la variacin de las funciones seno, coseno, tangente, etc., cuando 2 [0; 2 [, es decir
cuando el ngulo vara de 0 a 360 . Para esto, analizamos el crculo trigonomtrico unitario y las variaciones
de los valores de x e y en las ya conocidas deniciones:

sin =y cos =x tan = y=x


csc = 1=y sec = 1=x cot = x=y

| Recuerda las variaciones de las funciones trigonomtricas por cuadrantes:

FuncinnCuadrante 2 [0; 2 ] 2] 2 ; ] 2] ; 32 ] 2] 32 ; 2 [
sin 0%1 1&0 0& 1 1%0
cos 1&0 0& 1 1%0 0%1
tan 0 % +1 1%0 0 % +1 1%0
cot +1 & 0 0& 1 +1 & 0 0& 1
sec 1 % +1 1% 1 1& 1 +1 & 1
csc +1 & 1 1 % +1 1% 1 1& 1

El smbolo % nos indica que la funcin crece, mientras que & nos indica que decrece, en tal o cual cuadrante.
Pueden presentarse algunos casos en los que para ciertos valores del ngulo , el valor de la funcin no est
326 CAPTULO 9. TRIGONOMETRA I

denido o determinado, en estos casos hemos puesto los smbolos +1, para indicar que crece en forma ilimitada,
pero positiva; y 1 para indicar que crece en forma ilimitada, pero negativa. As, por ejemplo, diremos que
tan , cuando es igual a 2 (o 90 ), no existe, pero que tan crece ilimitadamente y en forma positiva, cuando
2 I se acerca al valor de 2, simblicamente escribimos l m tan ! +1.
!2

El cuadro de variacin de las funciones trigonomtricas procede del anlisis de las siguientes grcas:

| Variacin del sin y cos : varan de acuerdo a como varan y y x, respectivamente (Figura 9.7).

Figura 9.7: Izquierda: variacin del coseno, cos = x. Derecha: variacin del seno, sin =y .

y
| Variacin de la tan : Por denicin se tiene que tan =
, por tanto, buscamos en el crculo
x
y
trigonomtrico un segmento cuya magnitud algebraica sea igual a , y analizamos su variacin.
x

Figura 9.8: Izquierda: variacin de la tangente, tan = y=x en los cuadrantes I y III. Derecha: variacin de
la tangente, tan = y=x en los cuadrantes III y IV .

Analizando la (Figura 9.8), en el extremo del radio OA, sobre el eje Ox, trazamos la lnea tangente AT ,
!
con A como origen. El sentido positivo lo elegimos hacia arriba. Si el vector OM forma un ngulo 2 I con el
eje Ox, entonces continuamos el segmento OM hasta toparnos con la lnea tangente AT en el punto N . Como
2 I, entonces ON lo consideramos positivo. De la semejanza entre los tringulos OM1 M y OAN tendremos
la proporcin
M1 M AN y AN
= ) = ) tan = AN:
OM1 OA x 1

Si 2 II, entonces analizando la (Figuras 9.8) tendremos que AN es negativo y las coordenadas del vector
! y
OM tienen otros signos, a saber y < 0, por lo tanto tan = y AN tienen signos iguales. De la semejanza
x
entre los tringulos rectngulos OM M1 y OAN , que poseen el mismo ngulo agudo, tendremos la proporcin

jM1 M j AN M1 M
= ) = AN ) tan = AN
jOM1 j OA OM1
9.1. INTRODUCCIN Y FUNCIONES TRIGONOMTRICAS 327

Donde hemos usado el valor absoluto porque estamos trabajando con segmentos.

Para los casos en que 2 III o 2 IV , el anlisis es anlogo y tambin se demustra que tan = AN .
y
Mirando las (Figuras 9.8) llegamos a las conclusiones del cuadro para la variacin de la tan = = AN .
x
x
| Variacin de la cot : Por denicin se tiene que cot =
, por tanto, buscamos en el crculo
y
x
trigonomtrico un segmento cuya magnitud algebraica sea igual a , y analizamos su variacin.
y

Figura 9.9: Izquierda: variacin de la cotangente, cot = x=y en los cuadrantes I y III. Derecha: variacin
de la cotangente, cot = x=y en los cuadrantes II y IV .

Anlogamente al caso anterior, analizando la (Figuras 9.9), en el extremo del radio OB, sobre el eje Oy,
trazamos la lnea tangente BL, con B como origen. El sentido positivo lo elegimos hacia la derecha. Si el vector
!
OM forma un ngulo 2 I con el eje Ox, entonces continuamos el segmento OM hasta toparnos con la lnea
tangente BL en el punto P . Como 2 I, entonces OP lo consideramos positivo. De la semejanza entre los
tringulos rectngulos OM1 M y OAN tendremos la proporcin
OM1 BP x BP
= ) = ) cot = BP:
M1 M OB y 1

Para los casos en que 2 II, 2 III o 2 IV , el anlisis es anlogo y tambin se demuestra que cot = BP
x
(Figura 9.9). Mirando las (Figuras 9.9) se llega a las conclusiones del cuadro de variacin de cot = = BP .
y
La variacin del resto de funciones no las analizaremos, pero tendremos en cuenta que
1 1 1 1
csc = = ; csc = =
y sin x cos

9.1.5. Construccin de un ngulo conociendo el valor dado de cierta funcin


trigonomtrica

Presentamos ejemplos de construccin del ngulo 2 [0; 2 [ conociendo el valor de una de las funciones
trigonomtricas.

Ejemplos:

3
1. Sea sin =
4
3
Solucin: Dividimos el radio OB = 1 en 4 partes iguales y a la distancia de del origen O, trazamos
4
una perpendicular a OB, hasta que esta corte a la circunferencia en los puntos M y M1 . Obtendremos los
ngulos \AOM = 1 2 I y \AOM1 = 2 2 II, que son los buscados (Figura 9.10).
328 CAPTULO 9. TRIGONOMETRA I

Figura 9.10: Construccin de un ngulo por el valor dado de una funcin trigonomtrica: 1. sin > 0 (cuadrantes
I y II), 2. sin < 0 (cuadrantes III y IV), 3. cos > 0 (cuadrantes I y IV), 4. cos < 0 (cuadrantes II y III).

2
2. Sea sin =
3
2
Solucin: Dividimos el radio OB = 1 en 3 partes iguales y a la distancia de del origen O, pero con
3
y = 2, trazamos una perpendicular a OB, hasta que esta corte a la circunferencia en los puntos M y
M1 . Obtendremos los ngulos \AOM = 1 2 III y \AOM1 = 2 2 IV , que son los buscados (Figura
9.10).
3
3. Sea cos =
5
3
Solucin: Dividimos el radio horizontal OA = 1 en 5 partes iguales. A la distancia del origen O,
5
trazamos una perpendicular a OA hasta que corte a la circunferencia en los puntos M y M1 . Obtendremos
los ngulos buscados \AOM = 1 2 I y \AOM1 = 2 2 IV (Figura 9.10).

4. Sea cos = 0;6


6 3 3
Solucin: Dividimos el radio horizontal OA = 1 en 5 partes iguales (0;6 = = ). A la distancia del
10 5 5
origen O y con x = 3, trazamos una perpendicular a OA hasta que esta corte a la circunferencia en los
puntos M y M1 . Los ngulos buscados son \AOM = 1 2 II y \AOM1 = 2 2 III (Figura 9.10).

5. Sea tan = 1;4


7
Solucin: Como 1;4 = , entonces trazando el eje de tangentes AT , siendo el punto de tangencia el
5
A, tomamos en sentido positivo el segmento AN , igual a 1;4 radios. Ahora unimos el punto N con el
centro O, continuando OM en el otro sentido obtendremos M1 . Obtenemos as los 2 ngulos buscados:
1 = \AOM 2 I y 2 = \AOM1 2 III (Figura 9.11).

5
6. Sea tan =
4
Solucin: Trazamos el eje de tangentes AT , siendo el punto de tangencia el A, tomamos en sentido
negativo el segmento AN , igual a 1;25 radios. Ahora unimos el punto N con el centro O, continuando
OM en el otro sentido obtendremos M1 . Obtenemos as los dos ngulos buscados: 1 = \AOM 2 II y
2 = \AOM1 2 IV (Figura 9.11).

| Si se desea eliminar la dualidad del ngulo buscado , entonces necesitamos poner sobre cierta restriccin.
As, por ejemplo, en el sexto ejemplo, se podra exigir que < < , y obtener solamente el valor de 1 .
2
9.2. FUNCIONES TRIGONOMTRICAS DE CIERTOS NGULOS NOTABLES. GRFICAS 329

Figura 9.11: Construccin de un ngulo por el valor dado de una funcin trigonomtrica: Izquierda: tan >0
(cuadrantes I y III), Derecha: tan < 0 (cuadrantes II y IV).

| Cuando se trate de construr el ngulo , partiendo de valores dados de la secante (sec ) o cosecante
1 1
(csc ), podemos utilizar las relaciones inversas (sec = ) y (csc = ) y costrur mediante el coseno
cos sin
8 5
y el seno. As, por ejemplo, si sec = , entonces cos = .
5 8

9.2. Funciones Trigonomtricas de Ciertos ngulos Notables. Gr-


cas

Entre los ngulos notables, encontramos = 45 , = 30 y = 60 . Para encontrar los valores de sus
respectivas funciones trigonomtricas (que correspondonderan al I cuadrante, pues 0 < < 2 ), construmos
los tringulos representados en la (Figura 9.12), y a partir de los valores de sus catetos e hipotenusas, y de las
deniciones de cada funcin, hallamos los valores de todas sus funciones trigonomtricas.

| En la (Figura 9.12) tenemos un tringulo rectngulo issceles, con catetos iguales a = 1 y b = 1, y con
ngulos agudos, evidentemente tambin iguales, = = 45 . Por el Teorema de Pitgoras tendremos que la
p p
hipotenusa c = a2 + b2 = 2. De esta manera, las funciones trigonomtricas para = 45 seran
p p
p p 2 2
sin = 2; cos = 2; tan = 1; cot = 1; sec = ; csc =
4 4 4 4 4 2 4 2

Figura 9.12: Izquierda: tringulo issceles para la denicin de las funciones trigonomtricas de 45 . Derecha:
tringulo equiltero para la denicin de las funciones trigonomtricas de 30 y 60 .

| En la g. 9.12 tenemos un tringulo equiltero de lado a = b = c = 2, y evidentemente con 3 ngulos iguales


= = = 60 . Si bajamos una perpendicular o altura (que viene a ser tambin mediana, mediatriz y bisectriz)
de el vrtice superior a la base, entonces obtenemos dos tringulos rectngulos iguales, donde = = 60 y
= 2 = 30 . Tomando el tringulo rectngulo derecho, tenemos que el cateto 2c = 1, la hipotenusa b = 2, por
330 CAPTULO 9. TRIGONOMETRA I
q p
c 2
lo que segn el Teorema de Pitgoras, el cateto (altura) h = b2 2 = 3, y las funciones trigonomtricas
para = 30 y para = 60 seran

p p p p
sin =6 = 1=2 cos =6 = 3=2 tan =6 = 3=3 cot =6 = 3 sec =6 = 2 3=3 csc =6 = 2
p p p p
sin =3 = 3=2 cos =3 = 1=2 tan =3 = 3 cot =3 = 3=3 sec =3 = 2 csc =3 = 2 3=3

| Cuando trabajamos con el crculo trigonomtrico, debemos tener presente que las coordenadas fx; yg del
extremo del radio vector ! r satisfacen la relacin x2 + y 2 = r2 , y si r = j!
r j = 1, entonces se cumple la relacin
x2 + y 2 = 1, en el crculo trigonomtrico unitario.

A continuacin presentamos la siguiente tabla de valores de las funciones trigonomtricas para ciertos ,
siendo por comodidad r = 1. Se recomienda memorizar la tabla:

0 30 45 60 90 180 270 360


Funcinnngulo
0 =6 =4 =3 =2 3 =2 2
p p
sin =y 0 1=2 2=2 3=2 1 0 1 0
p p
cos =x 1 3=2 2=2 1=2 0 1 0 1
p p
tan = y=x 0 3=3 1 3 @ 0 @ 0
p p
cot = x=y @ 3 1 3=3 0 @ 0 @
p p
sec = 1=x 1 2 3=3 2 2 @ 1 @ 1
p p
csc = 1=y @ 2 2 2 3=3 1 @ 1 @

9.2.1. Dependencia entre las funciones trigonomtricas de cierto ngulo


!
Sea el ngulo generado al girar el radio vector !
r = fx; yg = OM , siendo j!
r j = r, entonces
y x
sin = , cos = y x2 + y 2 = r2 ;
r r
y2 x2
de donde sin2 = , cos2 = .
r2 r2
| Sumando estas dos ltimas relaciones, tendremos:

y2 x2 y 2 + x2 r2
sin2 + cos2 = + = = =1
r2 r2 r2 r2
Por tanto
8 : sin2 + cos2 =1 (1)

Esta es la primera forma trigonomtrica del teorema de Pitgoras.


y y=r sin x x=r cos
| Sabemos adems que tan = = = y que cot = = = . Por tanto
x x=r cos y y=r sin

sin cos
tan = ; cot = (2)
cos sin
| Si dividimos la identidad pitagrica sin2 + cos2 = 1 por cos2 , obtendremos
2 2
sin2 cos2 1 sin cos2 1
+ = ) + = ;
cos2 cos2 cos2 cos cos2 cos

es decir
tan2 + 1 = sec2 ; (3)
9.2. FUNCIONES TRIGONOMTRICAS DE CIERTOS NGULOS NOTABLES. GRFICAS 331

que es una segunda forma trigonomtrica del teorema de Pitgoras.

| Si dividimos la identidad pitagrica sin2 + cos2 = 1 por sin2 , obtendremos


2
sin2 cos2 1 sin2 cos 2 1
+ = ) + =
sin2 sin2 sin2 sin2 sin sin
que nos da
1 + cot2 = csc2 ; (4)
que es una tercera forma trigonomtrica del teorema de Pitgoras.

| Es evidente que utilizando una combinacin de las anteriores identidades trigonomtricas, podremos
encontrar los valores de todas las funciones trigonomtricas para cierto valor de . Analicemos los siguientes
ejemplos y una tabla de relaciones:

Ejemplos:

1. Sea sin = 0;8, hallar los valores del resto de funciones.


Solucin: Esta cuestin debe resolverse utilizando las anteriores identidades:
4 1 1 5
sin = 0;8 = ) csc = = 4 =
5 sin 5
4
De la relacin pitagrica sin2 + cos2 = 1 se deduce que
s r
p 4
2
9 3
2
cos = 1 sin ) cos = 1 = =
5 25 5
El doble signo es necesario escribirlo porque no sabemos en que cuadrante termina el ngulo .
sin
De la identidad tan = tendremos que
cos
sin 5=4 25
tan = = =
cos 3=5 12
El signo depende del cuadrante.
cos 1
De la identidad cot = , o mejor de cot = tendremos
sin tan
1 1 12
cot = = =
tan 25=12 25
| Utilizando restricciones de cuadrante para el ngulo es posible eliminar la dualidad o doble signo .

2. Sea cot = 2;4, siendo 3 =2 < < 2 . Hallar los valores del resto de funciones.
12 5
Solucin: Como cot = 2;4 = ) tan = que tambin es negativa, pues 2 IV .
5 12
De la identidad pitagrica 1 + cot2 = csc2 se deduce que
s r
p 12
2
169 13
2
csc = 1 + cot ) csc = 1+ = = ;
5 25 5
siendo negativa la csc por ser 2 IV .
1 1 1 5
Como sin = tendremos sin = = = que tambin es negativa por ser 2 IV .
csc csc 13=5 13
De la identidad pitagrica tan2 + 1 = sec2 se deduce que
s r
p 5
2
169 13
2
sec = 1 + tan ) sec = + 1 + =+ = ;
12 144 12
siendo positiva la sec por ser 2 IV .
1 1 1 12
De la relacin cos = tendremos cos = = = que es positiva por ser 2 IV .
sec sec 13=12 13
332 CAPTULO 9. TRIGONOMETRA I

| Una forma muy simple de calcular el resto de valores de las funciones trigonomtricas, conociendo una de
ellas, es utilizando directamente las identidades pitagricas en un tringulo rectngulo. Los signos se colocan de
acuerdo al cuadrante o de acuerdo a la restriccin que haya sobre el ngulo . Analicemos el siguiente ejemplo:

Conociendo el valor de la tan , hallar los valores del resto de funciones trigonomtricas.

Sabemos que
cateto opuesto a tan p
tan = = ) c2 = a2 + b2 ) c = tan2 + 1;
cateto adyacente b 1
de donde, gracando un tringulo rectngulo correspondiente a los anteriores valores para los catetos a = tan
p
y b = 1 e hipotenusa c = tan2 + 1 (Figura 9.13), tendremos:

Figura 9.13: Tringulo rectngulo para determinar todas las funciones trigonomtricas en funcin de tan .

tan 1
sin = p ; cos = p ; tan = tan ;
2
tan +1 tan2 +1
p
tan2 +1 p 1
csc = sec = tan2 +1 cot =
tan tan

| Anlogamente podemos calcular los valores de todas las funciones trigonomtricas por el valor dado de
una de ellas, estos valores vienen dados en la siguiente tabla:

Recuerda cada funcin


trigonomtrica en
funcin de las otras:
Funcin sin cos tan cot
p
sin sin 1 cos2 p tan p 1
p 1+tan2 1+cot2
cos 1 sin2 cos p 1 p cot
1+tan2 1+cot2
p
tan psin 1 cos2
tan 1
2 cos cot
p1 sin
cot 1 sin2 p cos 1
1 cos2 tan cot
sin
p p
sec p 1 1
1 + tan 2 1+cot2
1 sin2 cos cot
p p
csc 1 p 1 1+tan2
1 + cot2
sin 1 cos2 tan

Los valores para el resto de funciones lo dejamos en calidad de ejercicio.

9.2.2. Identidades y ejemplos varios

| Cuando una igualdad, en la que su miembro izquierdo y derecho son aparentemente distintos, toma valores
iguales para todos los valores admisibles de la variable o ngulo, entonces sta se denomina identidad. Si en
9.2. FUNCIONES TRIGONOMTRICAS DE CIERTOS NGULOS NOTABLES. GRFICAS 333

la identidad intervienen funciones o expresiones trigonomtricas, entonces estamos tratando con identidades
trigonomtricas. As, por ejemplo, la igualdad cos2 + sin2 = 1 es una identidad trigonomtrica, pues como
sin
ya habamos visto, es vlida para 8 2 R. Lo mismo podemos decir de la igualdad tan = , que se cumple,
cos
1
o tiene sentido para cualquier , exepto para 6= (2k + 1), k 2 Z. Por el contrario, la igualdad cos 2x = no
2 2
representa una identidad, pues no se cumple para cualquier x, sin embargo se cumple solamente para aquellas
x= + k, k 2 Z, en este caso la igualdad anterior representa una ecuacin trigonomtrica.
3
Ejemplos:

3 sin x 2 cos x 9
1. Calcular el valor de la expresin , si cot x =
3 sin x + 2 cos x 2
Solucin: Dividiendo numerador y denominador de la fraccin dada por sin x (sin x 6= 0), tendremos:

3 sin x 2 cos x 9
3 2
3 sin x 2 cos x sin x sin x 3 2 cot x 2
= = = = 2
3 sin x + 2 cos x 3 sin x 2 cos x 3 + 2 cot x 9
+ 3+2
sin x sin x 2

2. Dado el valor sec cos = b. Hallar el valor de sec2 + cos2 .


Solucin: Elevando al cuadrado la expresin original tendremos:
2
(sec cos ) = sec2 | {zcos } + cos
2sec 2
= b2
1

) sec2 2 + cos2 = b2 ) sec2 + cos2 = b2 + 2


cos x + cot x
3. Mostrar que la fraccin no puede ser negativa.
sin x + tan x
Solucin: Transformando la fraccin, tendremos:
cos x
cos x + 2
sin x = (sin x cos + cos x) cos x = (sin x + 1) cos x 0
sin x (sin x cos x + sin x) sin x (cos x + 1) sin2 x
sin x +
cos x
pues, ninguna de las fracciones anteriores es negativa, ni tampoco su producto. Adems 1 sin x 1y
1 cos x 1.

4. Hallar el valor del ngulo 0 < X < , si 2 sin X = cos2 X.


2
Solucin: Utilizando la identidad sin2 X + cos2 X = 1, la igualdad anterior toma la forma de ecuacin
cuadrtica:
p
2 2 2 4+4 p
2 sin X = 1 sin X ) sin| {zX} + 2| sin{z X} |{z}
1 = 0 ) sin X = = 1 2
2
ax2 bx c
p
El primer valor sin X = 1 2 ' 2;4142 no es vlido, pues para 8X 2 R se tiene que 1 sin X 1.
p p
El segundo valor sin X = 1+ 2 ' 0;4142 si es vlido y corresponde a un ngulo X = arcsin 1+ 2 '
0;42708 180
0;42708 rad., o ' 24;47 .

3 sin cot + cos2


5. Hallar el valor de la expresin 2 si se conoce que tan = 3=4 y < <
(sin cos ) + 2 sec csc2
3 =2.
Solucin: En este caso necesitamos poner todas las funciones trigonomtricas de la fraccin dada en
funcin de la tan , luego reemplazar el valor tan = 12 , pero respetando el signo de cada funcin para
334 CAPTULO 9. TRIGONOMETRA I

2 III:
2

2 3 ptan 1
tan + p 1
3 sin cot + cos 1+tan2 1+tan2
2 = 2 p
(sin cos ) 2 csc2 1+tan2
ptan p 1
2 tan
1+tan2 1+tan2

De donde
! !2
3
1 q 1
3 q 4
3 +
1+( 34 )
2
1+( 34 )
2 3 4 4 2
4
3 5 3 + 5 17
!2 q != 1 2
=
3 1+( 34 )
2 2 5 23
5 3
q 4 q 1 2 3
2 2
1+( 3
4 ) 1+( 34 ) 4

| En trigonometra es muy frecuente trabajar con identidades, es decir con igualdades que se cumplen
para cualquier valor admisible de la variable. Para cerciorarnos de que cierta igualdad dada es una identidad, o
mejor dicho para demostrar la identidad, por lo general, se transforma un solo miembro (rara vez ambos!) de la
igualdad, por lo general el ms grande, luego se lo reduce hasta convertirlo en el otro miembro. Generalmente,
al miembro sujeto a transformacin es bueno expresarlo en funcin solo de senos y cosenos, realizar las debidas
simplicaciones, y utilizando las identidades elementales bsicas, llegar al otro miembro. No hay que olvidar
que los smbolos de igualdad (=) y/o de identidad ( ) deben ser utilizados con mucho cuidado. Podemos
poner una cadena de igualdades o transformaciones utilizando el smbolo de igualdad (=), solamente si estamos
trabajando en un solo miembro de la igualdad dada, y solamente despus de efectivamente mostrar que una
parte se transforma en la otra, podemos denitivamente colocar el smbolo de igualdad (=) o identidad ( )
entre ambos miembros. En este caso, queda demostrada la identidad.

Ejemplos:

cot y sin y cos y 1


1. Demostrar la identidad cot2 y: 2 = 2
(cos y + sin y) 1
Solucin: Reducimos el miembro izquierdo al derecho, ponindolo primeramente en funcin de senos y
cosenos: cos y cos y
cot y sin y cos y sin y sin y cos y sin y sin y cos y
2 = 2 2 = =
(cos y + sin y) 1 cos y + 2 cos y sin y + sin y 1 1 + 2 cos y sin y 1

cos y sin2 y cos y cos y 1 sin2 y 1 sin2 y cos2 y 1


= 2 = 2 = 2 = = cot2 y
2 cos y sin y 2 cos y sin y 2 sin y 2 sin2 y 2
El primer miembro de la igualdad dada se transform exactamente en el segundo, entonces la identidad
queda demostrada.

2. Mostrar que tan + cot = sec csc :


Solucin: Trabajando en el primer miembro, tendremos:

sin cos sin2 + cos2 1 1


tan + cot = + = = = sec csc
cos sin sin cos sin cos

cos x sin x
3. Mostrar que + = sin x + cos x:
1 tan x 1 cot x
Solucin: Trabajando en el miembro izquierdo, tendremos:

cos x sin x cos x sin x cos2 x sin2 x


+ = sin x
+ cos x = + =
1 tan x 1 cot x 1 cos x 1 sin x cos x sin x sin x cos x

cos2 x sin2 x cos2 x sin2 x (cos x sin x) (cos x + sin x)


= = = cos x + sin x
cos x sin x cos x sin x cos x sin x cos x sin x
9.2. FUNCIONES TRIGONOMTRICAS DE CIERTOS NGULOS NOTABLES. GRFICAS 335

sin y 1 + cos y
4. Mostrar que + = 2 csc y:
1 + cos y sin y
Solucin: Realizando las debidas simplicaciones en el miembro izquierdo, tendremos:
2
sin y 1 + cos y sin2 y + (1 + cos y) sin2 y + 1 + 2 cos y + cos2 y
+ = = =
1 + cos y sin y (1 + cos y) (sin y) (1 + cos y) (sin y)

2 + 2 cos y 2 (1 + cos y) 2
= = = 2 csc y
(1 + cos y) (sin y) (1 + cos y) (sin y) sin y

csc2 2
5. Demostrar la identidad csc2 + 1 = cot :
1 tan
Solucin: Transformamos el primer miembro y lo ponemos solo en funcin de la cot :

csc2 2 csc2 1 1 cot2 1


csc2 +1= 1 csc2 1 = cot 1 cot2 =
1 tan 1 cot cot

cot (cot 1) (cot + 1)


cot2 = cot (cot + 1) cot2 = cot :
cot 1

sin2 1 1
6. Demostrar la identidad 3
= 3
+ tan
cos cos sec tan
Solucin: Trabajando solo en el segundo miembro, tendremos:

1 1 1 sin 1
cos3 + tan sec tan = cos3 + cos 1 sin
=
cos cos

1 sin 1 1 sin cos 1 sin +sin cos2 (1 sin ) cos4


cos3 + cos 1 sin = cos3 + cos 1 sin = cos3 (1 sin ) =
cos

1 sin +sin cos2 sin2 cos2 cos4 1 sin +sin cos2 cos2 (sin2 +cos2 )
cos3 (1 sin ) = cos3 (1 sin ) =

1 sin +sin cos2 cos2 1 sin cos2 (1 sin ) (1 sin )(1 cos2 ) 1 cos2 sin2
cos3 (1 sin ) = cos3 (1 sin ) = cos3 (1 sin ) = cos3 = cos3

7. Demostrar la identidad tan2 t sin2 t = sin2 t tan2 t


Solucin: Probemos transformar el miembro derecho y convertirlo al izquierdo:

sin2 t 1 cos2 t 1
sin2 t tan2 t = sin2 t 2
= sin2 t = 1 sin2 t =
cos t cos2 t cos2 t

sin2 t
sin2 t = tan2 t sin2 t
cos2 t

9.2.3. Funciones trigonomtricas de argumento negativo y su reduccin

Analicemos la (Figura 9.14) y las funciones trigonomtricas de los ngulos positivo y negativo ( ):
!
Supongamos que el vector unitario fx; yg = OM , forma con el eje horizontal Ox un ngulo , y el vector
!0 !
unitario fx; yg = OM , simtrico con respecto al eje Ox al vector OM 0 , un ngulo ( ). Por ser simtricos,
tendremos:
sin = OM1 ; sin ( ) = OM2

Las proyecciones OM1 y OM2 sobre el eje Oy, son iguales entre si en valor absoluto (jOM1 j = jOM2 j), pero de
signos contrarios, por lo tanto OM2 = OM1 lo que nos da sin ( ) = sin .
! !
Anlogamente, por ser simtricos los vectores OM y OM 0 con respecto al eje Ox, sus proyecciones sobre Ox
tambin son iguales, por tanto cos ( ) = cos .
336 CAPTULO 9. TRIGONOMETRA I

Figura 9.14: Crculo trigonomtrico para la denicin de funciones trigonomtricas de argumento negativo y su
paso a funciones de argumento positivo.

De aqu se deduce facilmente que:

sin ( ) sin cos ( ) cos


tan ( )= = = tan ; cot ( )= = = cot ;
cos ( ) cos sin ( ) sin

1 1 1 1
sec ( )= = = sec ; csc ( )= = = csc
cos ( ) cos sin ( ) sin
Resumiendo, tenemos:

| Las funciones trigonomtricas de argumento negativo satisfacen las relaciones:

Recuerda las relaciones trigono-


mtricas con ngulos negativos
sin ( ) = sin csc ( ) = csc
cos ( ) = cos sec ( ) = sec
tan ( ) = tan cot ( ) = cot

Ejemplos:

1. La expresin C = sin (120 ) cos (150 ) tan (225 ) es negativa, pues

signo (C) = (+) ( ) (+) = ( ) . Por tanto, C < 0:

sin ( 210 ) cos (120 ) tan2 ( 45 )


2. Determinar el signo de la expresin A =
csc ( 480 ) sec ( 780 )
Solucin: Utilizando el cuadro anterior de reduccin de las funciones trigonomtricas de argumento
negativo, y los respectivos signos de cada funcin trigonomtrica en cada cuadrante, tendremos
2
sin ( 210 ) cos (120 ) tan2 ( 45 ) sin 210 cos 120 ( tan 45 )
= =
csc ( 480 ) sec ( 780 ) csc (360 + 120 ) sec (2 360 + 60 )

sin 210 cos 120 tan2 45 sin 210 cos2 120 tan2 45
=
csc 120 sec 60 sec 60
Por tanto
2 2
( ) ( ) (+)
signo (A) = = (+) , es decir, A > 0:
(+)
9.2. FUNCIONES TRIGONOMTRICAS DE CIERTOS NGULOS NOTABLES. GRFICAS 337

tan2 (7 =4) cos ( 8 =3) csc ( 5 =6)


3. Determinar el signo de B =
sin3 ( 3 =4) sec2 ( 7 )
Solucin: Por analoga con el ejemplo anterior, tendremos
7 8 5
tan2 4 cos 3 csc 6 tan2 (315 ) cos ( 480 ) csc ( 150 )
3 = =
sin 3
4 sec2 ( 7 ) sin3 ( 135 ) sec2 ( 1260 )

2 2
(tan (315 )) cos (480 ) ( csc (150 )) (tan (315 )) cos (360 + 120 ) ( csc (150 ))
3 2 = 3 2 =
( sin (135 )) (sec (1260 )) ( sin (135 )) (sec (3 360 + 180 ))
2
(tan (315 )) cos (120 ) ( csc (150 ))
3 2
( sin (135 )) (sec (180 ))
Por tanto
2
( ) ( ) (( ) (+)) (+) ( ) ( )
signo (B) = 3 2 = = ( ) , es decir, B < 0:
(( ) (+)) ( ) ( ) (+)

9.2.4. Frmulas de reduccin. Funciones trigonomtricas y sus grcas

| Se denominan frmulas de reduccin a aquellas reglas que nos permiten calcular el valor de cierta funcin
trigonomtrica del ngulo (siendo > 90 o < 0 ) mediante las correspondientes funciones trigonomtricas
de un ngulo agudo . Las frmulas de reduccin dependern del cuadrante (ver Figuras a continuacin) en el
que termine el ngulo y del eje horizontal Ox o vertical Oy con respecto al cual se lo mida. Tendremos los
siguientes casos:

Recuerda las siguientes frmulas de reduccin:

Aqu se pretende dar las frmulas para las cuales los valores de las funciones trigonomtricas de cualquier
ngulo , 0 360 , se puedan expresar por los correspondientes valores de un ngulo agudo. estas frmulas
se denominan frmulas de reduccin.

Arriba: frmulas de reduccin para ngulos que terminan en el I y II cuadrantes (con respecto al eje vertical
y). Abajo: frmulas de reduccin para ngulos que terminan en el I y II cuadrantes (con respecto al eje
horizontal x).
338 CAPTULO 9. TRIGONOMETRA I

1. < < , es decir 2 II. En este caso, el ngulo puede ser representado de 2 formas =
2
( (
90 + 1 o 90 + 60 , siendo 1 = 60 o
. As, por ejemplo, si = 150 , entonces = 150 =
180 2 180 30 , siendo 2 = 30
Para obtener las frmulas de reduccin, analicemos los anteriores crculos trigonomtricos.
! !
Analizando las guras, vamos a suponer que OM = fx; yg y O1 N = fx1 ; y1 g, entonces

sin (90 + ) = OM1 = y, cos = O1 N1 = x1

Como los tringulos rectngulos OM1 M y O1 N1 N son iguales, entonces

jOM1 j = jO1 N1 j ) jyj = jx1 j ) y = x1 ) sin (90 + ) = cos ;

siendo y > 0 y x1 > 0.


De los 2 primeros crculos unitarios superiores, tambin se deduce que

jOM2 j = jO1 N2 j ) jxj = jy1 j ) x = y1 ) cos (90 + ) = sin ;

sin (90 + ) cos


pues x < 0 e y1 > 0, adems tan (90 + ) = = = cot , cot (90 + ) =
cos (90 + ) sin
cos (90 + ) sin
= = tan .
sin (90 + ) cos
!
De los 2 segundos crculos unitarios inferiores, en los que el vector OM forma con el eje Ox el ngulo
180 , el vector O1 N , el ngulo , y tal que: sin (180 ) = OM1 , sin = O1 N1 . Se tiene adems que
las coordenadas OM1 y O1 N1 son iguales en magnitud y signo: OM1 = O1 N1 , o bien sin (180 ) = sin .
Luego cos (180 ) = OM2 , cos = O1 N2 .
Los valores de las coordenadas OM2 y O1 N2 son de igual valor absoluto, pero de signos contrarios:
OM2 = O1 N2 , es decir cos (180 ) = cos .
sin (180 )
A continuacin hallamos los valores de la tangente y cotangente: tan (180 ) = =
cos (180 )
sin
= tan , cot (180 )= cot .
cos
Ejemplos:
p
a) cos 2 =3 = cos ( =3) = cos =3 = 1=2, b) tan 5 =6 = tan ( =2 + =3) = cot =3 = 3=3,
p
c) sin 150 = sin (180 30 ) = sin 30 = 1=2, d) cot 120 = cot (180 60 ) = cot 60 = 1= 3, e)
cos 110 = cos (90 + 20 ) = sin 20 ' 0;342

2. ( < < 3 =2, es decir 2 III. En este caso, el ngulo puede ( ser representado de 2 formas =
180 + 1 o 180 + 30 , siendo 1 = 30 o
. As, por ejemplo, si = 210 , entonces = 210 =
270 2 270 60 , siendo 2 = 60
Para obtener las frmulas de reduccin, analicemos los siguientes crculos trigonomtricos. En las guras
se han representado en los 2 crculos trigonomtricos unitarios superiores los ngulos 180 + y :
sin (180 + ) = OM1 , sin = O1 N1 . Las coordenadas OM1 y O1 N1 son de igual valor absoluto, pero de
signos contrarios, es decir, OM1 = O1 N1 , luego sin (180 + ) = sin .
En forma semejante, se establece que: cos (180 + ) = cos , tan (180 + ) = tan , cot (180 + ) =
cot .
En los 2 crculos trigonomtricos unitarios inferiores de la gura se muestran los ngulos 270 y ,
donde se ve claramente que sin (270 ) = OM1 , cos = O1 N2 , siendo OM1 = O1 N2 , y de donde
sin (270 ) = cos .
De forma anloga obtenemos: cos (270 )= sin , tan (270 ) = cot , cot (270 ) = tan .
9.2. FUNCIONES TRIGONOMTRICAS DE CIERTOS NGULOS NOTABLES. GRFICAS 339

Arriba: frmulas de reduccin para ngulos que terminan en el I y III cuadrantes (con respecto al eje
horizontal x). Abajo: frmulas de reduccin para ngulos que terminan en el I y III cuadrantes (con
respecto al eje vertical y).

Ejemplos:
p
a) cos 4 =3 = cos ( + =3) = cos =3 = 1=2, b) tan 7 =6 = tan ( + =6) = tan =6 = 3=3,
p
c) sin 210 = sin (270 60 ) = cos 60 = 1=2, d) cot 240 = cot (270 30 ) = tan 30 = 3=3,
e) cos 200 = cos (180 + 20 ) = cos 20 ' 0;939 69, f ) sin 4 =3 = sin (3 =2 =6) = cos =6 =
p
3=2.

3. 3( =2 < < 2 , es decir 2 IV . En este caso, el ngulo puede (ser representado de 2 formas =
270 + 1 o 270 + 30 , siendo 1 = 30 o
. As, por ejemplo, si = 300 , entonces = 300 =
360 2 360 60 , siendo 2 = 60

Las frmulas de reduccin se obtienen en forma anloga mediante los respectivos crculos trigonomtricos:

sin (360 )= sin , cos (360 ) = cos , tan (360 )= tan , cot (360 )= cot ;

sin (270 + ) = cos , cos (270 + ) = sin , tan (270 + ) = cot , cot (270 + ) = tan .

Ejemplos:
p p
a) cos 11 =6 = cos (2 =6) = cos =6 = 3=2, b) tan 7 =6 = tan (3 =2 =3) = cot =6 = 3,
p p
c) sin 300 = sin (270 + 30 ) = cos 30 = 3=2, d) cot 330 = cot (360 30 ) = cot 30 = 3,
e) cos 290 = cos (270 + 20 ) = sin 20 ' 0;342 02, f ) sin 5 =3 = sin (3 =2 + =6) = cos =6 =
p
3=2.

Observaciones. Las frmulas de reduccin expuestas en los 3 puntos anteriores son generales o universales,
es decir, el ngulo puede ser como no ser agudo. As, por ejemplo, si > 360 o < 0 , entonces = +360 n,
donde 0 360 y n 2 Z, y sin ( + 90 ) = sin ( + 360 n + 90 ) = sin (360 n + ( + 90 )) = sin ( + 90 ),
cos = cos ( + 360 n) = cos , por tanto sin (90 + ) = cos . As mismo, la frmula sin (90 + ) = cos
es vlida para cualquier ngulo 2 R.
340 CAPTULO 9. TRIGONOMETRA I

Recuerda las reglas para las frmulas de reduccin: Ejemplos:


| Si el argumento (ngulo) de una funcin trigonomtrica f
cos 150 = cos (180 30 ) =
reducida tiene la forma f (180 ), f (180 + ), f (360 ), p
cos 30 = 3=2;
f (360 + ), o f ( ), f ( + ), f (2 ), la denominacin
tan 5 =4 = tan ( + =4) =
de la funcin reducida f no cambia, pero el signo correspon-
tan =4 = 1:
de a f en el cuadrante dado.
| Si el argumento (ngulo) de una funcin trigonomtrica f
reducida tiene la forma f (90 ), f (90 + ), f (270 ), sin 300 = sin (270 + 30 ) =
p
f (270 + ), o f ( =2 ), f ( =2 + ), f (3 =2 ), f (3 =2 + ) cos 30 = 3=2;
la denominacin de la funcin reducida f cambia a la cofun- cot 5 =3 = cot (3 =2 + =6) =
p
cin (seno a coseno y viceversa, etc.), pero el signo corres- tan =6 = 3=3:
ponde a f en el cuadrante dado.

Periodicidad y curvas de las funciones trigonomtricas

! !
Si se tiene el crculo unidad (r = 1 = OM ) y el vector OM forma con el eje horizontal Ox un ngulo
, entonces a este argumento o ngulo le podemos sumar un nmero entero n de vueltas (cada vuelta o
revolucin es de 360 o 2 radianes) y volvemos a caer en el mismo punto de partida M , y consecuentemente,
tambin volvemos a tener los mismos valores de las funciones trigonomtricas:

Recuerda las relaciones:


sin = sin ( + 360 n) = sin ( + 2 n) csc = csc ( + 360 n) = csc ( + 2 n)
cos = cos ( + 360 n) = cos ( + 2 n) sec = sec ( + 360 n) = sec ( + 2 n)
tan = tan ( + 360 n) = tan ( + 2 n) cot = cot ( + 360 n) = cot ( + 2 n)

Lo anterior nos motiva a introducir la siguiente

Denicin. La funcin f se denomina peridica si existe un nmero T 6= 0, tal que f (x) = f (x + T ) =


f (x T ). El periodo fundamental es el menor nmero T 0 > 0 que cumple con f (x) = f (x + T 0 ) = f (x T 0 ).

Ejemplos:

1. Funciones peridicas importantes son el sin, cos, sec, csc cuyo periodo es T = 2 (360 ), y tan y cot
cuyo periodo es T = (180 ). La grca y los valores de una funcin peridica siempre se repiten a cada
intervalo de longitud x = T .

2. Mostremos que el periodo fundamental de la funcin sin es T = 2 .


Solucin: Supongamos que existe un real L tal que sin x = sin (x + L) para cualquier x 2 R. Colocando
x = 0 y x = =2 tendremos que: sin L = 0, sin ( =2 + L) = 1 ) sin L = 0, cos L = 1. El menor ngulo
positivo L que cumple con las 2 ltimas relaciones es L = 2 ) T = 2 .

3. sin (x + 2 n) = sin x, cos (x + 2 n) = cos x, tan (x + n) = tan x, etc., donde n 2 Z.


p
4. cos ( 330 ) = cos ( 330 + 360 ) = cos 30 = 3=2.
p
5. cot ( 17 =3) = cot ( 17 =3 + 6 ) = cot ( =3) = 3=3.
p
6. sin 1200 = sin (2 =3 + 6 ) = sin (2 =3) = sin =3 = 3=2.

7. En las 2 grcas contiguas (Figura 9.15, 9.16) se puede ver como se construye la funcin y = sin x (en
forma anloga se construye la grca de y = cos x) a partir del crculo trigonomtrico y tambin del hecho
9.2. FUNCIONES TRIGONOMTRICAS DE CIERTOS NGULOS NOTABLES. GRFICAS 341

Figura 9.15: Construccin de la curva y = sin x, T = 2 a partir del crculo trigonomtrico.

Figura 9.16: Grcas de las funciones trigonomtricas seno y coseno, y = sin x y y = cos x, T = 2 :

de que las curvas de las funciones sin y cos son las mismas, pero desplazadas en x = =2 unidades la
una de la otra, pues cos x = sin (x + =2), T = 2 , estn denidas para cualquier x 2 R, sus valores
cumplen con las relaciones 1 sin x 1, 1 cos x 1. Con respecto al seno, tenemos: la curva es
simtrica con respecto al origen O, pues sin x = sin ( x), es decir es funcin impar; en los intervalos
] =2 + 2 n; =2 + 2 n[ la curva crece de 1 a 1, y en los intervalos ] =2 + 2 n; 3 =2 + 2 n[ la curva
decrece de 1 a 1; sus puntos mximos o picos son para x = 2 + 2 n, n 2 Z; sus mnimos son para
x = 2 + 2 n, n 2 Z y sus ceros son para x = n, n 2 Z. Con respecto al coseno, tenemos: la curva
es simtrica con respecto al eje Oy, pues cos x = sin ( x), es decir es funcin par; en los intervalos
] + 2 n; 0 + 2 n[ la curva crece de 1 a 1, y en los intervalos ]0 + 2 n; + 2 n[ la curva decrece de
1 a 1; sus puntos mximos o picos son para x = 0 + 2 n, n 2 Z; sus mnimos son para x = (2n + 1) ,
n 2 Z y sus ceros son para x = 2 (2n + 1), n 2 Z.

8. En la siguiente (Figura 9.17 y siguiente) se puede ver como se construye la funcin y = tan x (en forma
anloga se construye la grca de y = cot x) a partir del crculo trigonomtrico. La funcin y = tan x con
periodo T = est denida para x 2 R, excepto x = 2 (2n + 1), n 2 Z, sus valores cumplen con la relacin
1 < tan x < +1. La curva y = tan x es simtrica con respecto al origen O, pues tan x = tan ( x), es
decir es funcin impar; en los intervalos ] =2 + n; =2 + n[ la curva crece de 1 a +1; la tangente
no posee puntos mximos ni mnimos y sus ceros son para x = n, n 2 Z. Con respecto al cotangente,
tenemos: la curva o grca de y = cot x puede ser construida desplazando la grca de la funcin y = tan x
hacia la izquierda a lo largo del eje Ox en =2 de acuerdo con la relacin cot x = tan (x + =2).
342 CAPTULO 9. TRIGONOMETRA I

Figura 9.17: Construccin de la curva y = tan x, T = a partir del crculo trigonomtrico.

y 50

25

0
-5 -2.5 0 2.5 5

-25

-50

Grca de la funcin trigonomtrica tangente y = tan x, T = :

y 50

25

0
-5 -2.5 0 2.5 5

-25

-50

Grca de la funcin trigonomtrica cotangente y = cot x, T = :

9. A continuacin presentamos las grcas de las funciones y = sec x y y = csc x, T = 2 :

y 50

25

0
-5 -2 .5 0 2 .5 5

-2 5

Grca de la funcin trigonomtrica secante y = sec x, T = 2 :


9.2. FUNCIONES TRIGONOMTRICAS DE CIERTOS NGULOS NOTABLES. GRFICAS 343

y 50

25

0
-5 -2 .5 0 2 .5 5

-2 5

-5 0

Grca de la funcin trigonomtrica cosecante y = csc x, T = 2 :

10. Dada la funcin peridica y = f (x) = sin x, T = 2 y su grca, construir las siguientes traslaciones:

a) y = sin x + 4; y = sin x 3

Solucin:

y 5

2. 5

0
-5 -2. 5 0 2. 5 5

-2. 5

-5

Grcas de y = sin x; y = f (x) = sin x + 4; y = f (x) = sin x 3

b) y = sin (x 5)

Solucin:

y 1

0.5

0
-5 -2.5 0 2.5 5

-0.5

-1

Grcas de y = sin x, y = sin (x 5) :

c) y = sin (x =2)

Solucin:
344 CAPTULO 9. TRIGONOMETRA I

y 1

0.5

0
-5 -2.5 0 2.5 5

-0.5

-1

Grcas de y = sin x e y = sin (x =2) = cos x:

11. Dada la funcin peridica y = cos x. Construir las homotecias:

1
a) y = 3 cos x; y = 2 cos x
Solucin:

2. 5

1.25

0
-5 -2. 5 0 2. 5 5

x
-1. 25

-2. 5

1
Grcas de y = cos x, y = 3 cos x e y = 2 cos x:

12. Dada la funcin peridica y = cos x. Construir y = 3 cos (x + 4) 2

Solucin:

y 2

x
-5 -2.5 0 2.5 5
0

-2

-4

-6

Grcas de y = cos x e y = 3 cos (x + 4) 2

x
13. Dada la funcin peridica y = cos x. Construir y = cos 2x y y = cos
3
Solucin:
9.2. FUNCIONES TRIGONOMTRICAS DE CIERTOS NGULOS NOTABLES. GRFICAS 345

y
1

0.5

0
-15 -10 -5 0 5 10 15

-0.5

-1

Grcas de y = cos x, y = cos x3 e y = cos 2x:

14. Dada la funcin peridica y = sin x. Construir la homotecia y simetra y = 3 sin ( x)


Solucin:

2.5

1.25

0
-5 -2.5 0 2.5 5

-1.25

-2.5

Grcas de y = sin x e y = 3 sin ( x)

15. Dada la funcin peridica y = sin x. Construir (la homotecia, contraccin y eliminacin de la parte y < 0)
y = j4 sin 2xj
Solucin:

y
3.75

2.5

1.25

0
-5 -2.5 0 2.5 5

Grca de y = sin x e y = j4 sin 2xj :

16. Dada la funcin peridica y = sin x. Construir y = 2 sin jxj :


Solucin:
346 CAPTULO 9. TRIGONOMETRA I

y 2

0
-10 -5 0 5 10

-1

-2

Grcas de y = sin x e y = 2 sin jxj :

17. Dada la funcin peridica y = sin x. Construir:

a) y = 4 jsin (x 2)j + 1
Solucin:

3.75

2.5

1.25

0
-5 -2.5 0 2.5 5

Grcas de y = sin x e y = 4 jsin (x 2)j + 1:

b) y = j4 sin (x 2) + 1j
Solucin:

3.75

2.5

1.25

0
-10 -5 0 5 10

Grcas de y = sin x e y = j4 sin (x 2) + 1j :

c) y = 4 sin jx 2j + 1
Solucin:
9.2. FUNCIONES TRIGONOMTRICAS DE CIERTOS NGULOS NOTABLES. GRFICAS 347

0
-15 -10 -5 0 5 10 15

-2

Grcas de y = sin x e y = 4 sin jx 2j + 1:

Practico lo que aprend:

Dadas las funciones y = f (x): a) y = 2 cos x, b) y = 3 sin x, c) y = 2 sin 3x, d) y = 3 cos 2x,

e) y = 0;25 tan 2x. Construir para cada una de las funciones anteriores las siguientes grcas: 1) y = f (x),
2) y = 2f (x), 3) y = 3f (x), 4) y = f (x + 1), 5) y = f (x 2), 6) y = 2f (x + 1), 7) y = 3f ( x),

8) y = 2f (x) 3, 9) y = f (x) + 4, 10) y = f (2x), 11) y = f (3x 1), 12) y = 2f ( 2x + 5), 13) y =
f (2x), 14) y = f (3x 2), 15) y = 3f ( 2x + 3), 16) y = f ( 3x), 17) y = f (3x 1), 18) y = 2f ( 2x + 3),
19) y = jf (x)j, 20) y = f (jxj), 21) y = jf (x) 1j, 22) y = 2 jf ( 2x + 3)j, 23) y = 2 jf (x)j + 4,
1 1
24) y = f (jxj 1), 25) y = 3 jf ( jxj) + 1j, 26) y = f ( jxj + 1), 27) y = jf ( jxj) + 2j :
2 2
Ejemplos:

1. Dada la aplicacin f : R ! [ 1; 1] con f (x) = sin x. Hallar:

a) f (0). Solucin: f (0) = sin 0 = 0:


1
b) f ( ). Solucin: f ( ) = sin = :
6 6 6 2
p
2
c) f ( ). Solucin: f ( ) = sin = :
4 4 4 p2
3
d ) f ( ). Solucin: f ( ) = sin = :
3h i 3 3 2
e) f ; . Solucin: f ) = 1, f = 1. El argumento vara de a , y los valores
2 2 2 2 h i n 2 2o
de la funcin van de 1 a +1. Por consiguiente f ; = sin x : x = [ 1; 1].
i h 2 2 n 2 i 2 ho
f) f ; . Solucin: Anlogamente tendremos f ] ; [ = sin x : x 2 ; =] 1; 1[:
2 2 2 2 2 2
h i h i n h io 1
g) f 0; . Solucin: f 0; = sin x : x 2 0; = 0; :
6 6 6 2
h) f ([0; 2 ]). Solucin: f ([0; 2 ]) = fsin x : x 2 [0; 2 ]g = [ 1; 1] :
i ) f 1 (0). Solucin: Por cuanto sin x = 0 para x = k ; k 2 Z, entonces, f 1 (0) = fx : sin x = 0g =
k .
1 1 n n
j ) f 1 ( ). Solucin: Si sin x = ) x = ( 1) arcsin 12 + n = ( 1) + n ; n 2 Z. Por tanto
2 2 6
1 n
f 1 ( ) = ( 1) + n ; n 2 Z.
2 6 (
p p )
2 p 2 n
k. f 1 ( ). Solucin: As mismo f 1 ( 22 ) = x : sin x = = ( 1) + n ; n 2 Z:
2 2 4
348 CAPTULO 9. TRIGONOMETRA I
p ( p )
3 p 3
1 1 3 n
k) f ( ). Solucin: As mismo f ( 2 ) = x : sin x = = ( 1) 3 + n ; n 2 Z:
2 2
m. f 1 ([ 1; 1]). Solucin: Por denicin f 1 ([ 1; 1]) = fx : f (x) = sin x 2 [ 1; 1]g. Mostremos
que f 1 ([ 1; 1]) = R: Sea x 2 f 1 ([ 1; 1]) y = sin x, entonces f (x) = , 2 [ 1; 1], y
n
por tanto x = (( 1) arcsin + n ), x 2 R , y por consiguiente f 1 ([ 1; 1]) R. Si x 2 R,
entonces sin x 2 [ 1; 1] y x 2 f 1 ([ 1; 1]), es decir R f 1 ([ 1; 1]), lo que en denitiva nos da
f 1 ([ 1; 1]) = R.
n o
1
n. f (] 1; 1[). Solucin: De la igualdad sin x = 1 tenemos que el conjunto A = x : x = +n ; n2Z
2
1
de valores de x, el mismo que no pertenece a f (] 1; 1[). Por lo tanto de acuerdo al anterior ejemplo
tendremos que f 1 (] 1; 1[) = R A.
o. f 1
0; 12 . Solucin: Tenemos f 1
0; 21 = x : sin x 2 0; 12 .
1 1 1 n
Sea x 2 f 0; 2 y = sin x, entonces 2 0; 2 y x = ( 1) arcsin x + n ; n 2 Z.
Sea n = 2k un valor jo, entonces x = arcsin + 2k , teniendo que vara de 0 a 1=2 y x de 2 k a
1 1
2k + , es decir x 2 2k ; 2k + :
6 6
Sea n = 2k + 1 un valor jo, entonces x = arcsin x + (2k + 1) , y si vara de 0 a 1=2, entonces
la variable x va de (2k + 1) hasta 2k + 56 , es decir x 2 2k + 56 ; (2k + 1) .
De esta manera

1 1 1 5
f 0; [ 2k ; 2k + [ [ 2k + ; (2k + 1) :
2 k2Z 6 k2Z 6

Es correcta tambin la inclusin inversa por cuanto para

1 1
x 2 2k ; 2k + ox2 2k + ; (2k + 1)
6 6

los valores del sin x 2 0; 21 . Por eso tendremos

1 1 1 5
f 0; = [ 2k ; 2k + [ [ 2k + ; (2k + 1)
2 k2Z 6 k2Z 6

2. Averiguar cales de las siguientes funciones f : [0; 1] ! [0; 3] son: inyectivas, sobreyectivas y biyectivas
x
a) x ! 3 sin
2
x
Solucin: Como para cualquier y 2 [0; 3], la ecuacin y = 3 sin posee solucin nica x =
2
2 y x
arcsin , perteneciente al segmento [0; 1], entonces x ! 3 sin es biyectiva.
3 2
y 3

2.5

1.5

0.5

0
0 0.25 0.5 0.75 1

x
x
Funcin biyectiva (creciente) y = 3 sin :
2
x
b) x ! tan
4
9.2. FUNCIONES TRIGONOMTRICAS DE CIERTOS NGULOS NOTABLES. GRFICAS 349

x 4
Solucin: Sea y 2 [0; 1], entonces la ecuacin y = tan posee solucin nica x = arctan y,
4
perteneciente al segmento [0; 1], si y 2 [0; 1]. Si y 2 [0; 3], entonces la ecuacin no posee soluciones
pertenecientes a [0; 1]. Por consiguiente, la ecuacin (1), para cualquier y 2 [0; 3], posee no ms de
x
una solucin x 2 [0; 1], y por tanto la funcin x ! tan es inyectiva.
4
y 3

2. 5

1. 5

0. 5

0
0 0. 25 0. 5 0. 75 1

x
Funcin inyectiva (creciente) y = tan 4 :

3 5
3. Dada la funcin f (x) = tan x, <x< , hallar su inversa.
2 2
3 5
Solucin: Mostremos que la funcin dada es la biyeccin f : ; ! R. Para esto representamos
2 2
con x = 2 + , < < . Entonces 8y 2 R la ecuacin y = tan x toma la forma y = tan , 2
i h 2 2
; . De aqu = arctan y, y utilizando el hecho de que x = 2 + , encontramos que x = 2 +arctan y,
2 2
3 5
teniendo adems que si y 2 R, entonces x 2 ; , es decir, se cumple la biyectividad. Y por cuanto
2 2
3 5 3 5
para cada y 2 R le corresponde un nico x 2 ; , entonces la funcin inversa f 1 : R ! ;
2 2 2 2
dene la correspondencia
3 5
y ! 2 + arctan x, x 2 ; :
2 2

4. Escribir las expresiones explcitas de las siguientes funciones dadas paramtricamente:

a) x = a cos t, y = a sin t, 0 t 2
Solucin: Por cuanto la funcin x ! a cos t, t 2 [0; ] es una biyeccin [0; ] ! [ a; a], entonces
8x 2 [ a; a] de la igualdad x = a cos t encontramos el nico valor t = arc cos xa , perteneciente al
segmento [0; ]. Colocando este valor en la segunda igualdad, tendremos:
r r
x x x2 p
y = a sin arc cos 2
= a 1 cos arc cos =a 1 , es decir, y = a2 x2 ; x 2 [ a; a] :
a a a2
b) x = a cos t, y = a sin t, t 2 (a > 0)
Solucin: Representemos mediante + = t. Entonces, si 2 [0; ], tendremos que t 2 [ ; 2 ], y
segn esto la primera igualdad se transforma a la forma x = a cos :
La funcin ! a cos es una biyeccin [0; ] ! [ a; a], por eso 8x 2 [ a; a] encontramos que
x x x
= arc cos = arc cos yt=2 arc cos . Colocando el valor encontrado de t en la
a a p a
segunda igualdad, tendremos y = a2 x2 ; x 2 [ a; a] :

3 5
5. Escribir la expresin explcita de la funcin f : ; ! [4 ; 5 ], dada implcitamente mediante la
2 2
relacin
3 5
sin x cos y = 0; x 2 ; ; y 2 [4 ; 5 ] (1)
2 2
350 CAPTULO 9. TRIGONOMETRA I

3 5
Solucin: Para cualquier valor jo x 2 ; tenemos sin x = q, q 2 [ 1; 1]. Por eso (1) es equivalente
2 2
a la ecuacin cos y = q, la misma que sobre el segmento [4 ; 5 ] posee una nica solucin. Con esto queda
3 5
demostrado la existencia de la funcin f : ; ! [4 ; 5 ] :
2 2
Para la escritura anloga de la funcin f , transformamos (1) a la forma sin x sin 2 y = 0. De donde

x 2 +y x+ 2 y
2 sin cos = 0:
2 2
Igualando a cero cada uno de los factores, encontramos dos valores para y :

y=x + 2n ; n 2 Z (2); y= x+ + 2n ; n 2 Z (3)


2 2
3 5
En el caso (2), de la condicin x 2 ; se sigue que y 2 [(2n + 1) ; (2n + 2) ] y no pertenece a
2 2
[4 ; 5 ] 8n 2 Z, es decir, y = x 2 + 2n no es un valor de la funcin f para ningn n 2 Z. En el caso
3 5
(3), de la condicin x 2 ; se sigue que y 2 [(2n 2) ; (2n 1) ] [4 ; 5 ] para n = 3. Para
2 2
13 3 5
este valor de n, de (3) se sigue que y = x+ , x2 ; :
2 2 2

Practico lo que aprend:

1. Hallar los valores de 0;1, 0;25, 1;5, 2, 5, 10, 25 y 50 grados en radianes.

2. Completar la siguiente tabla de valores:


Valor en grados 10 60 105 135 165 240 300 315 405 750
Valor en radianes
Valor en radianes =15 =18 =9 =4 2 =3 4 =3 11 =6 13 =6 3 4;5
Valor en grados

3. En las siguientes desigualdades, reemplazar el valor de por su valor aproximado 3;1416 : 0 < 1 < =2,
=2 < 2 < , < 3 < 3 =2, 3 =2 < 4 < 2 .

4. Indicar en que cuadrante se hallan los ngulos de 1, 2, 3, 4, 5, 6, 7, 8, 9 y 10 radianes.


1
5. Mostrar que el rea de un sector circular puede ser hallado con la frmula S = 2 r2 , en donde es la
medida en radianes del ngulo central y r el valor del radio.

6. Mostrar que el rea del segmento cuyo arco de circunferencia es inferior a la semicircunferencia viene dada
por la frmula S = 12 r2 ( sin ), donde es la medida en radianes del arco y r el valor del radio.

7. Hallar el signo de las siguientes expresiones:


a) sin 10 sin 11
10 cos 5 cos 65 , b) (sin 20 sin 15 ) sin 154 , c) (cos 20 cos 15 ) cos 154 ,
d) (cos 87 cos 117 ) sin 107 , e) (sin 2;5 cos 2;7) cos 3;6
sin 2x+cos 3x
8. Dada f (x) = sin 3x+cos 2x . Hallar: a) f (0), b) f ( 12 ), c) f ( 6 ), d) f ( 2 ), e) f ( )
sin 4 sin 52 +cos 3
9. Hallar los valores de: a) cos 8 , b) (sin 2 )cos + (cos 2 )sin 1;5
a4 cos 2 4a3 b sin 32 +6a2 b2 cos 0 4ab3 cos +b4 sin
10. Simplicar la expresin: P (a; b) = a3 sin 52 +3a2 b sin 92 +3ab2 cos 6 b3 cos 15
2

sin x cos x+sin 7x cos 12x


11. Mostrar que la funcin f (x) = 1 2 cos 2x es impar.

12. Hallar todas las soluciones de las desigualdades: a) sin 2x < 0, b) cos 3x > 0, c) tan x < 0
9.2. FUNCIONES TRIGONOMTRICAS DE CIERTOS NGULOS NOTABLES. GRFICAS 351
p 2
p
13. Hallar los dominios de existencia de las funciones: a) f (x) = sin x, b) f (x) = sin x , c) f (x) = cos x,
3 p 2 2
d) f (x) = , e) f (x) = tan x, b) f (x) = , c) f (x) =
cos x tan x sin x tan x
14. Hallar los recorridos de las funciones: a) f (x) = 1 + sin x, b) f (x) = 3 cos 2x:

15. Hallar el valor mximo y mnimo de la funcin f (x) = sin2 x + sin x + 1

16. Indicar el perodo positivo de las funciones: a) f (x) = sin 2x + sin x2 , b) f (x) = cos 2x cos x3 .
p
17. Determinar el signo de las siguientes expresiones: a) tan 8 sin 98 cot 8 , b) (tan 18 tan 15 ) cos 2 ,
11
c) (tan 12 cot 12 ) tan(sin 2 ), d) (cot 2;6 cot 2;5)(tan 2;6 tan 2;5).
tan 2x+cot 3x
18. Dada: f (x) = 1+tan2 2x . Hallar: a) f ( 12 ), b) f ( 6 ), c) f ( 2 ), d) f (0), e) f ( ):
tan 8 cot 72 +sin 3
19. Calcular: a) 1+tan 54 +cot 4
, b) (tan 3 )tan =4
+ (tan 6 )sin :

a4 cot =4+4a3 b tan 5 =4+6a2 b2 cot 9 =4+4ab3 sin =2+b4 cos 10


20. Simplicar la expresin: P (a; b) = a3 tan =4 3a2 b tan( =4)+3a2 b cot 5 =4 b3 cot 3 =4

tan x cot 2x+tan 7x cot 5x


21. Mostrar que la funcin f (x) = 1+2 tan2 x es par.

22. Hallar los conjuntos soluciones de las desigualdades: a) tan 2x < 0, b) cot 3x > 0:
p p 2
23. Hallar el dominio de denicin de las funciones: a) f (x) = tan x, b) f (x) = cot x, c) f (x) = tan x ,
d) f (x) = cot32 x .

24. Hallar los recorridos de las siguientes funciones: a) f (x) = 1 + tan2 x, b) f (x) = 2 cot2 x:

25. Hallar los valores mximo y mnimo de la funcin f (x) = 1 + tan 4 + sin x:

26. Hallar el perodo positivo mnimo de las funciones: a) f (x) = tan 2x + tan x=2, b) f (x) = cot 2x cot x=3:

27. Transformar los siguientes valores a las mismas funciones de un ngulo del I cuadrante: a) sin 310o ,
b) cos( 108 ), c) tan( 740 ), d) cot 8;4 , e) cos 10:

28. Transformar los siguientes valores a las cofunciones de un ngulo del I cuadrante: a) sin 257 , b) cot( 19
6 ):

29. Transformar los valores a funciones de un ngulo del I cuadrante: a) sin 495 , b) cos( 35
8 ), c) tan 5;1 .

30. Simplicar los valores: a) sin(3 =2+ ) cos( 3 ) cot(5 =2+ ), b) sin( 17 ) cos( + ) tan(3 =2 ):
p p
31. Se sabe que 6 1 = cos +26 k + i sin +26 k , donde k = 0, 1, 2, 3, 4, 5 e i = 1. Hallar todos los valores
p
de 6 1, represetndolos con z1 , z2 , z3 , z4 , z5 , z6 .

32. Sea la funcin f : R ! [ 1; 1] dada mediante f (x) = cos 3x. Hallar:


a) f (0) b) f c) f d) f
h i i6 h h4 i 3
e) f ; f) f ; g) f 0; h) f ([0; 2 ])
2 2 2 2 p6 ! p !
1 1 1 1 2 1 3
i) f (0) j) f k) f l) f
2 2 2
" p #! " p p #!
1 1 3 1 2 2 1
m) f ([ 1; 0]) n) f 0; o) f ; p) f ( 1)
2 2 2
h i
33. Para f : 0;
! R dada mediante: a) f (x) = tan x, b) f (x) = cot x .
2
h i h i h i 1 p 1 p
Hallar: f 0; , f 0; , f ; , f 1 (]0; 1]), f 1 p ; 3 , f 1 p ; 1; 3 :
6 4 6 3 3 3
x
34. La funcin f : [ 1; 1] ! [0; 1] dada mediante x ! cos es inyectiva, sobreyectiva o biyectiva. Gracar.
2
352 CAPTULO 9. TRIGONOMETRA I

35. Hallar el dominio reducido de biyectividad para las funciones:


1
a) f (x) = sin x, x 2 R b) f (x) = cos x, x 2 R c) f (x) = sin , x > 0
x
36. Hallar las funciones inversas de:
5 3 3
a) f (x) = sin x, x 2 ; b) f (x) = sin x, x 2 ; c) f (x) = cos x, x 2 [2 ; 3 ]
2 2 2 2
3
d) f (x) = cos x, x 2 [ 7 ; 6 ] e) f (x) = tan x, x 2 ; f) f (x) = cot x, x 2 ] ; 0[
2 2
3 5 3
37. Dada la funcin implcita f : [ ; 2 ] ! ; mediante cos x + sin y = 0, x 2 [ ; 2 ], y 2 2 ; 52 .
2 2
Hallar su expresin explcita.
3 3
38. Dada la funcin implcita f : [ ; 2 ] ! ; mediante cos x + sin y = 0, x 2 [ ; 2 ], y 2 2; 2 .
2 2
Hallar su expresin explcita.

Figura 9.18: Izq. El genial matemtico, fsico, astrnomo y lsofo Henri Jules Poincar (Nancy, 1854 - Pars, 1912).
Introdujo, analiz y aplic a las ecuaciones diferenciales y a la geometra las funciones automorfas. Se le considera como el
padre de la topologa moderna y sistemas dinmicos. Especialista en mecnica celeste (problema de los 3 cuerpos, estudio
de los anillos de Saturno, etc.). Hizo valiosos estudios sobre ecuaciones diferenciales especiales, de la fsica-matemtica,
y en otros campos ms. Practicamente dominaba la matemtica de su tiempo y la desarroll en varidsimos campos
y aplicaciones. Se adelant a Albert Einstein (Ulm, 1879 - Princeton, 1955) en la deduccin de muchos resultados de
la teora especial de la relatividad. Son famosas sus obras sobre la losofa de la ciencia. Der. El gran matemtico
y autodidacta hind que domaba y trataba de dominar a sus amigos los nmeros, Srinivasa Ramanujan (Erode, 1887
- Kumbakonam, 1920). Genial especialista en teora de los nmeros (fracciones continuas, distribucin de los nmeros
primos, series numricas divergentes, series de Riemann, integrales elpticas, funciones hipergeomtricas, etc.). Estudi
muchsimo al nmero , desarroll mtodos (series) potentsimos para calcularlo con miles de decimales. Fue protegido del
famoso matemtico ingls H. Hardy (Cranleigh, 1877 - Cambridge, 1947) en Cambridge, donde enferm con tuberculosis
levndolo de regreso a la India. Posea una intuicin matemtica poderossima que sale de cualquier explicacin posible.
Apndice A

RESUMEN DE ALGUNAS
FRMULAS

El espritu divino se manifest de modo sublime en esta maravilla del

anlisis, este prodigio de un mundo ideal, este intermediario entre el ser


p
y el no-ser, al que llamamos raz imaginaria de la unidad negativa ( 1).

Gottfried W. Leibniz

Rerum omnium magister usus.

Cesar

A.1. lgebra

Proporciones:

a c
= ,a d=b c
b d

Propiedades:

a c d c a b d b a b c d a c a c
a) = , b) = , c) = , d) = , e) = , f) = =
b d b a c d c a a c b d b d

Operaciones con exponentes:

n n
(a b c) = an bn cn ; an bn cn = (a b c)
a n an an a n
= n
; n
= ; am an = am+n
b b b b
n
am n
:a =a m n
; 1 : an = a0 : an = a n ; (am ) = amn

Operaciones con radicales:

353
354 APNDICE A. RESUMEN DE ALGUNAS FRMULAS
p
m p p p p p p p
a b c = m a m b m c; m a m b m c = m a b c
r p p r
a m
a m
a a n p p n
m
= mp ; p = m ; a m = m an ; m an = a m
m
pb b p b p b p p p
m p
an = m anp ; m an = mp anp ; mp anp = m an

Observacin: las anteriores races se consideran aritmticas.

Progresiones:
(a1 + an ) n [2a1 + d (n 1)] n
Aritmticas: an = a1 + d (n 1) ; Sn = ; Sn =
2 2
Geomtricas:

un q u1 u1 un q
un = u1 q n 1
; Sn = (q > 1) ; Sn =
(q < 1) ;
q 1 1 q
n 1 n
un q u1 (1 q )
Sn = (q > 1) ; Sn = (q < 1) ;
q 1 1 q
u1
S1 = l m Sn = ; (jqj < 1)
n!1 1 q

Logaritmos:

loga N = x , ax = N; aloga N = N
loga a = 1; loga 1 = 0; loga (N M ) = loga N + loga M
N p 1
loga = loga N loga M; loga (N m ) = m loga N; loga m N = loga N
M m
1 logb N
logb a = ; loga N = ; loga N = loga b logb N
loga b logb a

Combinatoria:

Variaciones: Anm = m (m 1) (m 2) : : : (m n + 1)

Permutaciones: Pn = 1 2 3 : : : n = n!

n Anm n (n 1) (n 2) : : : (n m + 1) n! n n m
Combinaciones: Cm = = = ; Cm = Cm
Pm 1 2 3:::m m! (n m)!
Algunas identidades. Binomio de Newton:

a2 b2 = (a b) (a + b) , a3 b3 = (a b) a2 + ab + b2 ,
2
a3 + b3 = (a + b) a2 ab + b2 , a4 b4 = (a b) (a + b) a2 + b
an bn = (a b) an 1 + an 2 b + an 3 b2 + + bn 1 ; n 2 N; par o impar
n n n 1 n 2 n 3 2
a + b = (a + b) a a b+a b bn 1 ; n 2 N; impar

Binomio de Newton:

n Pn n!
(x + a) = k=0 xn k ak =
k! (n k)!
= xn + Cn1 axn 1 + Cn2 a2 xn 2 + : : : + Cnn 2 an 2 x2 + Cnn 1 an 1 x + an =
Pn
= k=0 Cnk xn k ak ; Tk+1 = Ckn ak xn k
k
n Pn n! ( 1) Pn k
(x a) = k=0 xn k ak = k=0 Cnk ( 1) xn k ak :
k! (n k)!
1 2 m 2 m 1
1 + Cm + Cm + : : : + Cm + Cm + 1 = 2m ; 1 Cm 1
+ Cm2
Cm3
+ ::: 1=0
p
2 b b2 4ac
ax + bx + c = 0; a 6= 0 ) x1;2 = ;
Ecuacin cuadrtica: 2a
b c
a (x x1 ) (x x2 ) = 0 ) x1 + x2 = ; x1 x2 =
a a
A.2. GEOMETRA ANALTICA 355

A.2. Geometra Analtica

Distancia entre 2 puntos y problemas de la recta:

q
2 2
P1 (x1 ; y1 ) ; P2 (x2 ; y2 ) ) dist (P1 ; P2 ) = (x1 x2 ) + (y1 y2 )
dist (P1 ; P2 ) 0; dist (P1 ; P2 ) = dist (P2 ; P1 ) ;
dist (P1 ; P3 ) dist (P1 ; P2 ) + dist (P2 ; P3 ) :
y 2 y1 y2 y 1
P1 (x1 ; y1 ) ; P2 (x2 ; y2 ) ) m = ; y y1 = (x x1 ) ;
x2 x1 x2 x1
x y
y y1 = m (x x1 ) ; + =1
a b
l1 : y = m1 x + b1 ; l2 : y = m2 x + b2 ) l1 kl2 , m1 = m2 ; l1 ?l2 , m1 m2 = 1;
m1 m2
= ] (l1 ; l2 ) ) tan =
1 + m1 m2
A1 B2 A2 B1
l1 : A1 x + B1 y + C1 = 0; l2 : A2 x + B2 y + C2 = 0 ) = ] (l1 ; l2 ) ) tan =
A1 A2 + B1 B2
Ax0 + By0 + C
P (x0 ; y0 ) ; l : Ax + By + C = 0 ) dist (P; l) = p
A2 + B 2

2 2
La circunferencia: x2 + y 2 = R2 , (x h) + (y k) = R2
2 2
x2 y2 (x h) (y k)
La elipse: + = 1, + =1
a2 b2 a2 b2
2 2
x2 y2 (x h) (y k)
La hiprbola: = 1, = 1, xy = 1
a2 b2 a2 b2
2
La parbola: y = ax2 + bx + c, y 2 = 4px, (y k) = 4p (x h)

A.3. Trigonometra

Denicin de las funciones trigonomtricas:

En el crculo trigonomtrico unitario (p (xp ; yp ) 2 x2p + yp2 = R2 , R = 1), las funciones trigonomtricas del
yp
ngulo se denen con las relaciones: cos = xp , sin = yp , tan =
xp
Identidades bsicas y Pitagricas:

sin x 1 1 1 cos x
tan x = ; sec x = ; csc x = , tan x cot x = 1 o cot x = =
cos x cos x sin x tan x sin x
sin2 x + cos2 x = 1; sec2 x tan2 x = 1 o sec2 x = 1 + tan2 x; csc2 x cot2 x = 1
1 1
sin2 x = ; cos2 x =
1 + cot2 x 1 + tan2 x

Funciones de los ngulos negativos:

sin ( x) = sin x; cos ( x) = cos x


tan ( x) = tan x; cot ( x) = cot x
sec ( x) = sec x; csc ( x) = csc x

Frmulas de transformacin:
sin ( x) = sin x; cos ( x) = cos x; sin (2 x) = sin x; cos (2 x) = cos x
3 3
sin 2 x = cos x; cos 2 x = sin x; sin 2 x = cos x; cos 2 x = sin x
356 APNDICE A. RESUMEN DE ALGUNAS FRMULAS

Funciones de la suma y diferencia de ngulos:

sin(x y) = sin x cos y cos x sin y; cos(x y) = cos x cos y sin x sin y
tan x tan y cot x cot y 1
tan(x y) = ; cot (x y) =
1 tan x tan y cot x cot y

Transformacin de sumas y diferencias en productos y viceversa:

sin x + sin y = 2 sin 12 (x + y) cos 12 (x y); sin x sin y = 2 cos 12 (x + y) sin 21 (x y)


cos x + cos y = 2 cos 12 (x + y) cos 12 (x y); cos x cos y = 2 sin 21 (x + y) sin 12 (x y)
sin (x y) x x
tan x tan y = ; 1 + cos x = 2 cos2 ; 1 cos x = 2 sin2
cos x cos y 2 2
sin x sin y = 12 fcos(x y) cos(x + y)g ; cos x cos y = 12 fcos(x y) + cos(x + y)g
sin x cos y = 21 fsin(x y) + sin(x + y)g

Ecuaciones trigonomtricas elementales:

k
sin x = a; jaj 1 ) x = ( 1) arcsin a + k ; k 2 Z
cos x = a; jaj 1 ) x = arc cos a + 2k ; k 2 Z
tan x = a ) x = arctan a + k ; k 2 Z:
sin x = sin a ) x = a 2k o x = ( a) 2k ; k 2 N
cos x = cos a ) x = a 2k o x = a 2k ; k 2 N
tan x = tan a ) x = a 2k y x 6= 2 k ; k 2 N

Enlace entre los valores de las funciones trigonomtricas inversas:

k
Arc sin x = k + ( 1) arcsin x; k 2 Z
Arc cos x = 2k arc cos x; k 2 Z
Arc tan x = k + arctan x; k 2 Z

Observacin: arcsin x, arc cos x, arctan x son los valores principales de las respectivas funciones inversas.

Leyes de los senos, cosenos y tangentes:

a b c
= = ; A+B+C =
sin A sin B sin C
2 2 2 2 2 2
a =b +c 2bc cos A; b =a +c 2ac cos B; c2 = a2 + b2 2ab cos C
tan 21 (A + B) a+b 1
tan 2 (B + C) b+c tan 21 (A + C) a+c
1 = ; 1 = ; 1 =
tan 2 (A B) a b tan 2 (B C) b c tan 2 (A C) a c

Funciones trigonomtricas de ngulos mitad:


r r
x 1 cos x x 1 + cos x
sin = ; cos =
r2 2 2 2
1 cos x sin x 1 cos x
tan x2 = = = = csc x cot x
1 + cos x 1 + cos x sin x

Funciones trigonomtricas de ngulos dobles:

2 cos2 x = 1 + cos 2x; 2 sin2 x = 1 cos 2x


2 tan x
sin 2x = 2 sin x cos x; tan 2x =
1 tan2 x
2 2
cos 2x = cos x sin x = 1 2 sin x = 2 cos2 x
2
1
A.3. TRIGONOMETRA 357

Funciones trigonomtricas de ngulos mltiplos:

sin 3x = 3 sin x 4 sin3 x; sin 4x = 4 sin x cos x 8 sin3 x cos x


cos 3x = 4 cos3 x 3 cos x; cos 4x = 8 cos4 x 8 cos2 x + 1
3 tan x tan3 x 4 tan x 4 tan3 x
tan 3x = 3 ; tan 4x =
1 3 tan x 1 6 tan2 x + tan4 x
3 5
sin 5x = 5 sin x 20 sin x + 16 sin x; cos 5x = 16 cos5 x 20 cos3 x + 5 cos x
5 3
tan x 10 tan x + 5 tan x tan (n 1) x + tan x
tan 5x = ; tan nx =
1 10 tan2 x + 5 tan4 x 1 tan (n 1) x tan x

Potencias de algunas Funciones trigonomtricas:

1 cos 2x
sin2 x = sin4 x = 3
8
1
2 cos 2x + 1
8 cos 4x
2
1 + cos 2x 3 1 1
cos2 x = cos4 x = 8 + 2 cos 2x + 8 cos 4x
2
sin3 x = 34 sin x 14 sin 3x sin5 x = 5
8 sin x
5 1
16 sin 3x + 16 sin 5x
cos3 x = 34 cos x + 14 cos 3x cos5 x = 5 5 1
8 cos x + 16 cos 3x + 16 cos 5x

Expresiones de sin x, cos x y tan x en funcin de tan x2 :

2 tan x2 1 tan2 x
2 2 tan x2
sin x = ; cos x = ; tan x =
1 + tan2 x2 1 + tan2 x
2 1 tan2 x2

Figura A.1: Izq. El famoso matemtico polaco Stefan Banach (Cracovia, 1892 - Lvov, 1945), gran matemtico del siglo
XX, uno de los padres del Anlisis Funcional, creador de las lgebras de Banach. Autodidacta, discpulo del matemtico
polaco Hugo Steinhaus (Jaslo, 1887 - Jaslo, 1972). Ceador de la famosa escuela matemtica del Lvov (maestro de S.
Mazur, W. Orlicz, I. Schauder). Autor de su famosa monografa Teora de las Operaciones Lineales, con la que nace el
anlisis funcional moderno. Fue cruelmente maltratado por los nazis en tiempos de la ocupacin nazi de Polonia. Der.
Nina Bari (Mosc, 1901 - 1961). Especialista en teora de funciones de variable real y en especial en teora de series
trigonomtricas, donde estudi su unicidad y ortogonalidad de sistemas. Fue discpula (junto con P. S. Aleksandrov,
M. A. Lavrentiev, A. N. Kolmogorov, P. S. Urisohn) del famoso matemtico ruso y fundador de la escuela moscovita
moderna, mtrica de teora de las funciones, Nikolai Luzin (Tomsk, 1883 - Mosc, 1950).
Apndice B

ESTRUCTURAS. SMBOLOS.
CRONOLOGA. GLOSARIO

Considerando la Matemtica desde el comienzo del mundo hasta la

poca de Newton, lo que l ha hecho es, con mucho, la mitad mejor.

G. Leibniz

In Scientia Naturali principia observationibus conrmari debent.

Carl von Linneo

B.1. Estructuras Algebraicas

Ley de composicin interna

Denicin.- Se denomina ley de composicin interna sobre un conjunto E, a una operacin en E, o a toda
aplicacin: E E ! E; (x; y) ! x y

Una operacin en E puede satisfacer propiedades diferentes:

Asociativa: 8x 2 E, 8y 2 E, 8z 2 E, x (y z) = (x y) z
Conmutativa: 8x 2 E, 8y 2 E, x y = y x
Elemento neutro: La ley admite un elemento neutro si 9e 2 E tal que: 8x 2 E; x e = e x = x
Se dice que x 2 E posee simtrico si 9x0 2 E tal que: x x0 = x0 x = e.
_
Elemento simtrico: 0
Se dice que x es el simtrico de x.

Si se dispone de una segunda operacin T en E, se dice que es distributiva con respecto a T , si:

8x 2 E; 8y 2 E; 8z 2 E; x (yT z) = (x y) T (x z) ; y

(xT y) z = (x z) T (y z)

358
B.1. ESTRUCTURAS ALGEBRAICAS 359

Grupo

Denicin.- Se dice que (G; ) es un grupo si es una operacin sobre G 6= ; que verica que:

es asociativa,
admite un elemento neutro. Todo elemento de G posee un simtrico.
Si adems la operacin es commutativa, entonces G es un grupo conmutativo o abeliano.

Si la operacin es la suma o la multiplicacin (de nmeros, de funciones, de matrices, etc.) en lugar de


escribir a la pareja (G; +) o (G; ) uno habla de grupo aditivo G o de un grupo multiplicativo G.

Por ejemplo el grupo aditivo de los reales (R; +), el grupo multiplicativo de los complejos no nulos (C ; ).

Propiedades.- Sea (G; ) un grupo, entonces:

1) el elemento neutro es nico,


2) el simtrico de todo elemento x es nico. Se lo denota mediante x 1 ;
3) 8x 2 E; 8y 2 E (x y) 1 = y 1 x 1 ;
4) todo elemento es regular a la derecha y a la izquierda: x y = x z ) y = z, y x = z x ) y = z
1 1
5) el paso al inverso es idempotente: 8x 2 E x =x

Ejemplos:

(Z; +), (Q; +), (R; +), (C; +), (Q ; ), (R ; ), (C ; ) son grupos conmutativos.

(N; +) no es un grupo.

(V; +), donde V representa el conjunto de los vectores del plano y (=, ), donde = representa el conjunto de
traslaciones, son los grupos conmutativos.

(D; ), donde D es el conjunto de desplazamientos es un grupo no conmutativo. (H; 0), donde H es conjunto
de homotecias no es un grupo.

Denicin.- Sea (G; ) un grupo, y H una parte no vaca de G. Se dice que (H; ) es un subgrupo de (G; )
si H es un grupo. La misma denicin para un subgrupo abeliano.

Propiedades.- Sea (G; ) un grupo con H G, entonces se tienen las equivalencias:

1. (H; ) es un subgrupo de (G; )


8
>
< H 6= ?
2. 8 (x; y) 2 H 2 : x y 2 H
>
:
8x 2 H : x 1 2 H
(
H 6= ?
3.
8 (x; y) 2 H 2 : x y 1
2H

Un grupo posee por lo menos 2 subgrupos triviales: (G; ) y (feg; ). Los otros, si estos existen, se dicen
propios.

Proposicin-. La interseccin de 2 subgrupos es un subgrupo.


360 APNDICE B. ESTRUCTURAS. SMBOLOS. CRONOLOGA. GLOSARIO

Anillos. Cuerpos

Denicin: Se dice que (A; T; ) (o (R; T; )) es un anillo si:

1) (A; T ) es un grupo conmutativo,


2) es asociativa, posee un elemento neutro,
3) la ley es distributiva con respecto a la ley de T .

S adems la ley es conmutativa, se tiene un anillo conmutativo.

Proposicin.- Si (A; T; ) es un anillo, entonces:

1. 8x 2 A : x e = e x = e (e el elemento neutro para T );

2. 8(x; y) 2 A2 : x y 0 = x0 y = (x y)0 (mediante 0


sobre el elemento representamos al simtrico del
elemento para la ley de T ).

Ejemplos:

(Z; +; ), (Q; +; ), (R; +; ), (C; +; ) son anillos conmutativos. En los anillos, las 2 propiedades anteriores
se traducen como:

1. 8x 2 A : x 0=0 x=0

2. 8(x; y) 2 A2 : x ( y) = ( x) y= (x y)

Si Mn (R) es el conjunto de las matrices n n con coecientes reales, (Mn (R); +; ) es un anillo no
conmutativo.

Denicin.- Sea (A; T; ) un anillo. Se dice que x es un divisor de cero si x 6= e y si existe un y 2 A


(y 6= e) vericando: x y = e

Los anillos (Z; +; ), (Q; +; ), (R; +; ), (C; +; ) no poseen divisores de cero. Este no es el caso para
Mn (R): tomamos por ejemplo n = 2; es fcil de vericar que:

! ! !
2 0 0 0 0 0
=
0 0 0 3 0 0
| {z }| {z } | {z }
x y e

Un anillo que no posee divisores de cero se denomina ntegro.

Denicin.- Sea = una parte no vaca de A. Se dice que (=; T; ) es un ideal bilateral de (A; T; ), si:

(=; T ) es un subgrupo de (A; T )


8x 2 =; 8y 2 A : x y 2 = e y x 2 =

Ejemplos:

(pZ; +; ) es un ideal de (Z; +; ).

(Q; +; ) no es un ideal de (R; +; ).

Denicin.- Se dice que (K; T; ) es un cuerpo, si:


B.1. ESTRUCTURAS ALGEBRAICAS 361

1) (K; T; ) es un anillo,
2) todo elemento distinto de e es inversible para la ley .

Si adems, la ley es conmutativa, entonces se habla de un cuerpo conmutativo.

Proposicin.- (K; T; ) es un cuerpo si y solamente, si:

1) (K; T; ) es un grupo conmutativo.


2) (K feg; ) es un grupo.
3) es distributiva con respecto a T .

Ejemplos:

(Q; +; ), (R; +; ), (C; +; ) son cuerpos conmutativos.

(Z; +; ), (Mn (R); +; ) no son cuerpos.

Proposicin.- Si (K; T; ) es un cuerpo; adems de las propiedades de los anillos, se tiene

8x 2 K 8y 2 K : x y = e , (x = e o y = e)

Un cuerpo K es un anillo sin divisores de cero (es ntegro).

Recapitulacin de las estructuras algebraicas

Sea E un conjunto no vacio, entonces:

T ley de composicin interna 8 (x; y) 2 E 2 : xT y 2 E


T asociativa 8 (x; y; z) 2 E 3 : (xT y) T z = xT (yT z)
(E; T ) grupo
T admite un elemento neutro e to- 9e 2 E; 8x 2 E : xT e = eT x = x
do elemento de E posee simtrico 8x 2 E; 9x0 2 E : xT x0 = x0 T x = e
(E; T ) grupo
conmutativo T conmutativa 8 (x; y) 2 E 2 : xT y = yT x
o abeliano
ley de composicin interna
8 (x; y) 2 E 2 : x y 2 E
asociativa
8 (x; y; z) 2 E 3 : (
(x y) z = x (y z)
(E; T; ) distributiva con respecto a T
x (yT z) = (x y) T (x z)
anillo 8 (x; y; z) 2 E 3 :
(xT y) z = (x z) T (y z)
9" 2 E; 8x 2 E : x " = " x = x
admite un elemento neutro "
(E; T; ) todo elemento de E feg 8x 2 E feg 9x 1 2 E
cuerpo posee simtrico para x x 1=x 1 x="
(E; T; ) ani-
llo conmuta-
es conmutativa 8 (x; y) 2 E 2 : x y = y x
tivo o cuerpo
conmutativo
362 APNDICE B. ESTRUCTURAS. SMBOLOS. CRONOLOGA. GLOSARIO

B.2. Algunos Smbolos Matemticos

Smbolos: Su signicado:
2 pertenece a un conjunto
2
= no pertenece a un conjunto
fa; bg conjunto fomado por los elementos a y b
? conjunto vaco
j: tal que
A B A contenido en B
A B A contiene a B

Smbolos: Su signicado:
?
= igualdad que hay que probar
6 = distinto
A A0 Ac complemento de A
E A o E=A E menos A
P (E) conjunto de las partes o subconjuntos de E
[ unin
\ interseccin
[ Ai unin de los conjuntos Ai , subndice i 2 I
i2I

Smbolos: Su signicado:
n
\ Ai interseccin de n conjuntos Ai , i = 1; : : : ; n
i=1
4 diferencia simtrica entre conjuntos
(x; y) par ordenado de elementos x e y
A B producto cartesiano de A por B
(x; y; z) terna ordenada de los elementos x, y, z
A B C producto cartesiano de A por B por C
N conjunto de nmeros naturales (0 2 = N)
Z conjunto de nmeros enteros
Q conjunto de nmeros racionales

Smbolos: Su signicado:
R conjunto (cuerpo) de los nmeros reales
C conjunto (cuerpo) de los nmeros complejos
R A B relacin entre elementos de A y B
f
f : A ! B, A ! B aplicacin de A en B
f (x), x ! f (x) imagen por f de un elemento arbitrario x
BA conjunto de las aplicaciones de A en B
f (X) imagen por f del subconjunto X
f 1 aplicacin recproca de f


B.2. ALGUNOS SMBOLOS MATEMTICOS 363

Smbolos: Su signicado:
f 1 (y) antiimagen o preimagen por f del elemento y
f 1 (Y ) antiimagen o preimagen por f del subconjunto Y
g f , g(f (x)) aplicacin compuesta de f con g
f jA f restringida al conjunto A
_ o (disyuncin)
^ y (conjuncin)
P (negacin) no P
P )Q P implica Q (si P , entonces Q)

Smbolos: Su signicado:
?
) implicacin que se ha de probar
P (Q Q implica a P (si Q, entonces P )
, si y slo si (lgicamente equivalentes)
p (x) funcin proposicin
fX 2 E j p (x)g los elementos de E tales que cumplen p (x)
8 para todo (cualquieraq que sea)
9 existe por lo menos uno
9! existe uno y slo uno

Smbolos: Su signicado:
xRy, x y x relacionado con y
e
x clase de equivalencia de representante x
A=R conjunto de las clases (mdulo R)
x<y x menor que y
x y x menor o igual que y
x>y x mayor que y
x y x mayor o igual que y
ajb a divide a b

Smbolos: Su signicado:
[a; b] intervalo cerrado de extremos a y b : a x b
]a; b[ intervalo abierto de extremos a y b : a < x < b
[a; b[, [a; b) intervalo mixtos o semiabiertos de extremos a y b
sup X supremo de X
nf X nmo de X
m n fx1 ; : : : ; xn g menor de los elementos x1 ; : : : ; xn
Pn
ai sumatoria de los ai , i = 1; : : : ; n
i=1
n
ai producto de los ai , i = 1; : : : ; n
i=1


364 APNDICE B. ESTRUCTURAS. SMBOLOS. CRONOLOGA. GLOSARIO

Smbolos: Su signicado:
n! n factorial
Im f imagen de f
E (x), [x] parte entera de x
z complejo conjugado de z
(A, + , ) conjunto A con dos operaciones internas
jxj valor absoluto o mdulo de x

B.3. Principales Jalones en la Historia de la Matemtica


30000 A. C. Presencia de muescas numricas.

8000 A. C. Aparicin de los calculi en Mesopotamia y en otros lugares de Oriente Medio.

3300 A. C. Primeras cifras en Sumer y Elam. Primera numeracin escrita.

2700 A. C. Cifras sumerias cuneiformes.

2000 A. C. Aparicin de la base decimal.

1800 A. C. Numeracin babilonia culta. Primera numeracin de posicin.

1300 A. C. Aparicin de las cifras chinas.

s. VI A. C. Descubrimiento de los valores irracionales. Pitgoras.

s. IV A. C. Primera crisis del concepto de innito. Aristteles.

300 A. C. Numeracin alfabtica griega.

s. III A. C. Aparicin del primer cero de la historia en la numeracin culta babilonia. Se formula por
primera vez la idea de lmite. Arqumedes.

s. II A. C. Numeracin de posicin china sin cero. Aparicin de las nueve cifras brahaminas que se
convertirn en cifras indues. Primeros siglos d. C. Los nmeros negativos.

ss. IV/V Numeracin de posicin ind. Numeracin decimal, con cero.

ss. V/IX Numeracin de posicin maya con un cero.

Fines s. VIII Llegada del clculo ind a Bagdad.

Inicio s. IX Al-Juwarizmi sobre el clculo ind.

s. X Cifra ghobar en el Magreb y en la pennsula Ibrica. Estas cifras, cuya grafa es distinta de las
cifras hindis que se usaban en el Oriente Medio rabe, son las antepasadas de las cifras que hoy se usan
en Occidente.

ss. XII/XIII Presencia del cero de la numeracin india en Occidente.

ss. XII/XV poca en la que las cifras rabes se estabilizan grcamente en Europa occidental para
dar origen a la forma denitiva que tienen en la actualidad.

s. XIII Primera utilizacin de una sucesin. Fibonacci.

ss. XV/XVI Con el uso de la imprenta, las cifras indorabes adquieren su grafa denitiva. El algoritmo
o clculo escrito utilizando la numeracin de posicin con un cero, se impone en Occidente.
B.4. GLOSARIO DE TRMINOS MATEMTICOS ELEMENTALES 365

s. XVI Primera utilizacin sistemtica de las funciones continuas. Bombelli. Cardan y Bombelli formulan
por primera vez los nmeros complejos.

Fines s. XVI Invento de la notacin literal (algebraica). Vite.

1635 Los valores innitesimales. Cavalieri.

1638 Se formula por primera vez el agregado innito. Galileo.

1639 Invencin de la geometra analtica. Descartes.

1654 Se formula por primera vez el principio de la induccin matemtica. Pascal.

Hacia 1677 Invento del clculo innitesimal. Newton. Leibniz. Primera utilizacin sistemtica de las
series innitas. Newton. Leibniz.

1797 Descubrimiento de una interpretacin geomtrica de los nmeros complejos. Gauss.

1820 Se formula por primera vez la potencia de un agregado. Bolzano.

1825 Descubrimiento de los nmeros algebraicos, que no pueden expresarse por radicales. Abel.

1843 Invencin de las cuaternos. Hamilton.

1844 Descubrimiento de los nmeros trascendentes. Liouville. Primera teora de las magnitudes extensi-
bles. Grassmann.

1867 Por primera vez se formula explcitamente el principio de permanencia de las leyes formales. Hankel.

1872 Primera teora cientca de los valores irracionales. Dedekind.

1883 Segunda teora cientca de los valores irracionales. Cantor. Invencin de lo transnito. Cantor.

1897 Descubrimiento de las antinomias de la teora de los agregados y conjuntos. Burali-Forti.

1900 Presentacin en el I Congreso de Matemticos en Pars por parte de David Hilbert de sus 23
problemas.

1996 Demostracin de la conjetura de Fermat. Andrew Wiles.

B.4. Glosario de Trminos Matemticos Elementales

A
Acutngulo: Tringulo que tiene sus tres ngulos agudos.

Aleatorio: Relativo al azar, producto del azar. Que no corresponde a lo determinstico.

Altura de un tringulo: Segmento bajado del vrtice al lado opuesto en forma perpendicular.

ngulo: Abertura formada por dos semirectas con un mismo origen denominado vrtice.

ngulos adyacentes: Son los que tienen un lado comn y sus otros lados pertecen a otras semirectas.

ngulo agudo: ngulo que mide menos de 90o ( =2 radianes).

ngulos complementarios: Son dos ngulos que suman 90o ( =2 radianes).

ngulos consecutivos: Son los que tienen un lado comn.


366 APNDICE B. ESTRUCTURAS. SMBOLOS. CRONOLOGA. GLOSARIO

ngulo central: ngulo formado por dos radios de una misma circunferencia.

ngulo diedro: Cada una de las regiones determinadas por dos semiplanos que se cortan. Los semiplanos
se llaman caras del ngulo diedro.

ngulo extendido (llano): Mide 180o ( radianes).

ngulo inscrito: ngulo formado por 2 cuerdas con un extremo comn que yace en la circunferencia.

ngulo obtuso: Mide ms de 90o ( =2 radianes) y menos de 180o ( radianes).

ngulo poliedro: Figura determinada por tres o ms semirrectas de origen comn, no coplanares, y tales
que el plano determinado por dos de ellas consecutivas deje a las restantes en un mismo semiespacio.

ngulo recto: Aquel que mide 90o ( =2 radianes).

ngulo semiinscrito: ngulo formado por una cuerda y una tangente trazada por un extremo de la
cuerda a la circunferencia.

ngulos suplementarios: 2 ngulos que suman 180o ( radianes).

ngulo triedro: Figura determinada por la interseccin de tres diedros cuyas aristas concurren a un
punto comn llamado vrtice.

Apotema: El apotema de un polgono regular, es el segmento perpendicular a un lado trazado desde el


centro.

Arco: Parte de una circunferencia o de otra curva.

Arco coseno: Relacin inversa de la funcin coseno. Si cos x = y entonces arc cos y = x:

Arco seno: Relacin inversa de la funcin seno. Si sin x = y entonces arcsin y = x:

Arco tangente: Relacin inversa de la funcin tangente.Si tan x = y entonces arctan y = x:

Asntota: Una curva tiene como asntota una recta, si la distancia de un punto P de la curva a la recta
tiende a cero cuando el punto P se aleja indenidamente del origen de coordenadas recorriendo la curva. Tambin
se puede decir que una asntota es una tangente a la curva en el innito.

Asociativa (propiedad): Una operacin interna en E, se dice que cumple la propiedad asociativa si
dados 3 elementos cualesquiera del conjunto E, se cumple que a (b c) = (a b) c, escribindose entonces
directamente en la forma: a b c.

Axioma: Proposicin considerada como muy evidente y verdadera y que no puede demostrarse.

Axioma de continuidad: Axioma de la recta real que arma la existencia de una biyeccin entre los
puntos de la recta y el continuo de los nmeros reales.

Axioma de paralelismo: Si dos rectas son cortadas por una transversal y la suma de los ngulos
interiores, situados a un lado de esa transversal es menor de dos rectos, las dos rectas se cortan a ese mismo
lado de la transversal.

Axiomas de A. N. Kolmogorov: Conjunto de axiomas que caracterizan la nocin o concepto de


probabilidad y que constituyen el modelo matemtico de los fenmenos aleatorios.

Axiomas de G. Peano: Axiomas de la aritmtica con los que se denen los nmeros naturales y la
induccin.
B.4. GLOSARIO DE TRMINOS MATEMTICOS ELEMENTALES 367

B
n P
n
Binomio (de Newton): Expresin algebraica de 2 trminos. Ejemplo: 5a b3 . (a + b) = Ckn an k k
b
k=0
es el binomio de Newton.

Bisectriz: Bisectriz de un ngulo es el lugar geomtrico de los puntos que equidistan de los lados de un
ngulo.

C
Catetos: Lados que forman el ngulo recto de un tringulo rectngulo.

Cero de una funcin: Todo punto x0 para el cual f (x0 ) = 0.

Cclico (Polgono): Polgono que se puede inscribir en una circunferencia.

Cifra signicativa: Todas las cifras excepto el cero.

Cilindro: Producto cartesiano de una circunferencia por una recta. Cuerpo geomtrico que se obtiene por
la rotacin de una curva generatriz en torno a un eje.

Circulo: Regin interior de una circunferencia, x2 + y 2 r2

Circunferencia: 1. Lugar geomtrico de todos los puntos que estn en un mismo plano y que equidistan
2 2
de un punto llamado centro. 2. Linea curva, plana de ecuacin (x h) + (y k) = r2 , C = C (h; k), r > 0:

Circunferencia goniomtrica (Crculo unidad): Circunferencia de radio 1 y centro C (0; 0), que se
utiliza para denir las funciones trigonomtricas: x2 + y 2 = 1.
n
X
n
Coecientes binomiales: Coecientes Ckn que aparecen en el desarrollo del binomio (a + b) = Ckn an bn k
,
k=0
n!
donde Ckn = k!(n k)! y n! = n (n 1) (n 2) 2 1, 0! = 1.

Combinacin lineal: El vector ! !; x


y es la combinacin lineal de los n vectores dados (x1
!; : : : ; x!), si:
2 n
Xn
!
y = !
k xk , donde k son escalares.
k=1

Combinatoria: Parte de la matemtica que analiza las diferentes formas de contar y agrupar los elementos
o subconjuntos (posibilidades) de un universo y que satisfacen ciertas propiedades.

Complejos iguales: Dos nmeros complejos son iguales si y slo si sus partes reales son iguales y sus
partes imaginarias tambin.

Composicin de Funciones: Dadas 2 funciones y 3 conjuntos con f : A ! B, g : B ! C. Se llama


composicin g f (en ese orden!) de las funciones f y g a la nueva funcin g f : A ! C, denida mediante
(g f )(x) = g[f (x)].

Conjuntos Equipotentes: Conjuntos entre los cuales se puede establecer una correspondencia 1 a 1
(biyeccin) entre sus elementos.

Conjunto Finito: Conjunto que tiene un nmero limitado de elementos (n elementos) y que no es
equipotente con ninguna de sus partes propias.

Conjunto numerable: Aquel cuyos elementos pueden ser colocados en una correspondencia 1 a 1 con
los elementos del conjunto de los naturales N.

Conjunto Innito: Conjunto de un nmero ilimitado de elementos y que es equipotente con alguna de
368 APNDICE B. ESTRUCTURAS. SMBOLOS. CRONOLOGA. GLOSARIO

sus partes propias.

Conjunto por Comprensin: Es aquel para el que se enuncia la propiedad comn de sus elementos.
Ejemplo: las vocales.

Conjunto por Extensin: Es aquel para el que se sealan todos sus elementos. Ejemplo: V = fa; e; i; o; ug :

Conjugado: En general, la conjugada de una expresin de la forma a b es a b. En particular, el


conjugado del nmero complejo a bi = z es el complejo a bi = z, donde hay que considerar siempre los
respectivos signos.

Conmutativa: Propiedad con la que no cambia el resultado de una operacin al alterar el orden de los
elementos que se operan. La operacin denida en un conjunto E, se dice que es conmutativa, si dados 2
elementos cualesquiera a y b de E se cumple que a b = b a.

Cono: Cuerpo slido engendrado por la rotacin de un tringulo rectngulo alrededor de uno de sus catetos.
El otro cateto forma la base circular del cono, mientras que la hipotenusa (generatriz) forma la supercie cnica.
z 2 = x2 + y 2

Cono Oblicuo: Cono, cuyo eje cae en forma oblicua a la base.

Cono Recto: Cono, cuyo eje cae perpendicularmente a la base.

Cono Truncado: Porcin de cono comprendida entre la base y un plano paralelo a la misma.

Consecuente: La segunda parte B (clusula entonces) de una proposicin condicional: A ) B.

Constante: Cantidad cuyo valor se mantiene inalterable. Ejemplos: ' 3;14159, e ' 2;718 3.

Constante de proporcionalidad: Si las variables x e y estn relacionadas, por ejemplo, por la relacin
y = kx, se dice que k es la constante de proporcionalidad entre ellas.

Coordenadas: Nmeros que permiten situar un punto en el plano (2 nmeros), o en el espacio (3 nmeros),
o en otro espacio (n nmeros). Normalmente se trabaja con las llamadas coordenadas cartesianas en las que los
ejes a partir de los cuales se sitan los puntos, son perpendiculares y la unidad sobre ellos mide lo mismo.

Coplanarios: Puntos o vectores situados en un mismo plano.

Corona Circular: Figura plana (anillo) comprendida entre 2 circunferencias concntricas.

Corolario: Teorema demostrado como una consecuencia inmediata de otro teorema.

Cuadriltero: Polgono de cuatro lados.

Cuarta Proporcional: Es cualquiera de los 4 trminos de una proporcin discreta a=b = c=d.

Cuerda: Segmento que une 2 puntos cualesquiera de la circunferencia.

Cuerpo polidrico: Cuerpo limitado por caras planas.

Cuerpo redondo: Cuerpo limitado, a lo menos, por una cara curva.

Cua Esfrica: Porcin de volumen de una esfera, comprendida entre un huso esfrico y el dimetro de
la esfera que pasa por los extremos del huso.

D
Deca: Prejo griego que signica 10.
B.4. GLOSARIO DE TRMINOS MATEMTICOS ELEMENTALES 369

Decgono Regular: Poligono de 10 lados iguales y ngulos tambin de igual medida.

Decgramo: Medida de masa equivalente a diez gramos.

Declitro: Medidad de capacidad equivalente a diez litros.

Decmetro: Medida de longitud equivalente a diez metros.

Deci: Prejo que signica dcima parte.

Dcima: Cada una de las diez partes iguales en que se divide una unidad o un todo.

Decmetro: Medida de longitud equivalente a la dcima parte del metro.

Dcuplo: Que contiene un nmero 10 veces.

Deduccin: Conclusin basada en un conjunto de proposiciones verdaderas.

Denicin: Proposicin que clarica o explica el signicado de un trmino o una frase.

Delta: Cuarta letra del alfabeto griego: , .

Demostracin: Proceso por el cual, mediante una serie de razonamientos lgicos, se llega a establecer la
verdad de una proposicin o teorema a partir de ciertas hiptesis.

Dependencia lineal: Un vector depende linealmente de otros si se puede poner como combinacin lineal
de ellos. As, el vector w depende linealmente de los vectores u y v si se pueden encontrar 2 nmeros, a y b
a2 + b2 6= 0 , tales que w = au + bv:

Descomposicin factorial: Factorar un nmero entero o una expresin algebraica, consiste en poner
ese nmero (expresin) como producto de factores primos o expresiones de menor grado (en Z, Q, R, C,
p
entre polinomios). Son descomposiciones factoriales: 12 = 22 3, a2 4b4 = a 2b2 a + 2b2 , x2 3 =
p4
p4 2
x 3 x + 3 , b + 1 = (b + i) (b i)

Desigualdad: Una desigualdad es una expresin o relacin que indica que un nmero, cantidad o expresin
puede ser mayor, menor, mayor o igual, o menor o igual (>, <, , ) que otra.

Diagonal: Segmento rectilneo que une 2 vrtices no consecutivos un polgono o de otra gura geomtrica.

Diagrama: Figura grca que explica un fenmeno estadstico, fsico, qumico, matemtico, etc.

Dimetro: Mxima cuerda que pasa por el centro y divide a la circunferencia en 2 semicircunferencias,
d = 2r.

Diedro (ngulo): Cada una de las regiones determinadas por dos planos que se cortan.

Dimensin de una matriz: Orden de una matriz o nmero de las y columnas que tiene la matriz. As,
A3 2 es una matriz de dimensin 3 2 (tiene 3 las y 2 columnas).

Diplo: Prejo griego que signica doble.

Disco (bola): Unin de la circunferencia con el crculo: x2 + y 2 r2 .

Discriminante: En la ecuacin de 2do grado ax2 + bx + c = 0, a 6= 0, la expresin = b2 4ac, se llama


discriminante, siendo los coecientes a, b, c reales. Si > 0, la ecuacin tiene 2 soluciones reales. Si = 0, la
ecuacin solucin real doble. Si < 0, la ecuacin tiene 2 races complejas conjugadas.

Disjuntos: Conjuntos cuya interseccin es vaca, A \ B = ;.


370 APNDICE B. ESTRUCTURAS. SMBOLOS. CRONOLOGA. GLOSARIO

Distancia: Es el espacio o intervalo de lugar o tiempo entre dos cosas o sucesos. Longitud del segmento de
recta comprendido entre 2 puntos del espacio. Es la relacin d : E E ! R entre los elementos x, y, z, : : : de un
conjunto E, que cumple con las propiedades: 1) d (x; y) 0, 2) d (x; y) = d (y; x), 3) d (x; y) d (x; z) + d (z; y) :

Distributiva (propiedad): Sean 2 operaciones, y T denidas en cierto conjunto E. La operacin es


distributiva respecto de la operacin T , si se verica que a (bT c) = (a b) T (a c), 8a; b; c 2 E. As, en R:
a (b + c) = a b + a c

Dodecaedro: Poliedro regular de 12 caras.

Dodecgono: Polgono regular de 12 lados.

E
Ecuacin: Es toda igualdad vlida slo para algn(nos) valor(es) de la(s) variable(s) o indeterminadas.
Ejemplos: 6x = 18; x y = 7, x3 + px + q = 0, sin (3x) = 1=2, 23 4x = 8, log (2x 7) = 1

Ecuacin bicuadrada: Ecuacin de 4to grado de la forma ax4 + bx2 + c = 0, a 6= 0.

Ecuacin cuadrtica: Ecuacin de 2do grado expresada mediante ax2 + bx + c = 0, donde a 6= 0.

Ecuacin cbica: Ecuacin de 3er grado del tipo ax3 + bx2 + cx + d = 0, donde a 6= 0.

Ecuacin curtica: Ecuacin de 4to grado del tipo ax4 + bx3 + cx2 + dx + e = 0, a 6= 0.

Ecuacin diferencial: Ecuacin que contiene derivadas y cuya funcin incgnita se halla bajo el signo
de derivada o diferencial. Por ejemplo, F (x; y; y 0 ) = 0.

Ecuacin exponencial: Se reere a la ecuacin en la cual la incgnita aparece en algn exponente.


1 2x
3 = 1=27

Ecuacin cuadrtica incompleta: Ecuacin cuadrtica de la forma ax2 + c = 0 o ax2 + bx = 0.

Ecuacin literal: Ecuacin cuyas cantidades conocidas estn representadas por letras o parmetros.

Ecuacin logartmica: Ecuacin en la cual aparecen expresiones logartmicas con la incgnita. log (1 2x) =
10, log2x 8 = 4.

Ecuacin numrica: Ecuacin cuyas cantidades conocidas estn representadas por nmeros. 3x2 log x =
2

Ecuacin trigonomtrica: La ecuacin trigonomtrica es aquella cuyas incgnitas son el argumento


principal de las funciones trigonomtricas. sin 2x = 1, sin x 3 cos 2x = 1

Ecuaciones compatibles: Ecuaciones que tienen al menos una solucin comn.

Ecuaciones equivalentes: Ecuaciones que tienen las mismas soluciones en cierto dominio D.

Ecuaciones independientes: Ecuaciones que no poseen las mismas soluciones.

Ecuaciones simultneas: Ecuaciones para las cuales se verican valores o soluciones iguales de las
incgnitas.

F
Fraccin decimal: Fraccin que tiene por denominador una potencia positiva de 10.

Fraccin impropia: Fraccin cuyo numerador es mayor que el denominador.


B.4. GLOSARIO DE TRMINOS MATEMTICOS ELEMENTALES 371

Fraccin irreductible: Fraccin que no se puede simplicar ms por ser numerador y denominador
mutuamente primos.

Fracciones equivalentes: Aquellas que tienen el mismo valor a=b = c=d , ad = bc. 3=5 = 6=10

Fraccin peridica: Fraccin tal que al hallar el cociente del numerador para el denominador, se obtiene
un nmero decimal en el que un nmero nito de cifras se repiten indenidamente. A las cifras que se van
repitiendo se le llama periodo. Si el periodo empieza inmediatamente a la derecha de la coma decimal, se llama
peridica pura. Si entre la coma decimal y el periodo existen otras cifras, se llama peridica mixta. 1=3 = 0;333,
T = 3; 5;012727, T = 27.

Fraccin ordinaria: Fraccin cuyos trminos son nmeros enteros. a=b; a; b 2 Z, b 6= 0.

Fraccin propia: Aquella cuyo numerador es menor que el denominador.

Factor: Cada uno de los trminos de un producto.

Factorial: Producto obtenido al multiplicar un nmero entero positivo dado, por todos los enteros positivos
inferiores a ese nmero. Se simboliza por n!. 0! = 1, 5! = 5 4 3 2 1 = 120

Finito: Que tiene n, trmino o lmite.

Funcin: Es una relacin, ley o correspondencia f entre 2 conjuntos X e Y , representada con (f; X; Y ),
donde cada elemento del conjunto llamado de salida ( X ), est relacionado con uno, y slo un, elemento de otro
conjunto llamado rango ( Y ); es decir, es una expresion matemtica del tipo f : X ! Y o y = f (x), donde
x 2 X, y 2 Y y para cada valor que se le asigna a la x, le corresponde un solo valor en la y. De lo contrario es
una relacin no funcional.

Funcin Lineal: Funcin del tipo f (x) = ax o f (x) = ax + b, cuya representacin grca es una recta
de pendiente m = tan = a y ordenada al origen igual a b. Se dene una funcin lineal con dos variables como
una expresin del tipo f (x; y) = ax + by.

Funcin Contnua: Una funcin numrica f : D ! R, D R es continua en el punto x0 2 D, si y slo si:


1 ) Existe l m f (x) = L cuando x tiende a x0 . 2o ) Existe f (x0 ) tal que f (x0 ) = L, es decir, l m f (x) = f (x0 ).
o
x!x0
f es continua en el conjunto D si es continua en todos y cada uno de los puntos x 2 D. Gracamente, f es
continua en D si puede ser trazada sin interrupciones o sin levantar la punta del lpiz del papel, es decir, para
un pequeo incremento x ! 0 de.la variable independiente x se obtiene un pequeo incremento y ! 0 de.la
variable dependiente y.

G
Geometra: Rama de las matemticas que estudia las relaciones cuantitativas entre los distintos conjuntos
u objetos de un espacio, las propiedades de las guras y las relaciones entre los puntos, lineas, imgenes, ngulos,
supercies y cuerpos en general. Estudia adems los grupos de transformaciones posibles entre los elementos o
puntos de un espacio.

Geometra plana: Trata de las guras cuyos puntos y lineas estn situados en un plano.

Geometra del espacio: Trata de las guras cuyos elementos no estn todos en el mismo plano.

Grado de un trmino algebraico: Es la suma de los exponentes de la parte literal de un trmino


algebraico deg 5a2 b = 2 + 1

Grado sexagesimal: Medida que procede de dividir una circunferencia en 360 partes (grados). El grado
sexagecimal est dividido en 60 partes iguales llamadas minutos y cada minuto est dividido en 60 partes
372 APNDICE B. ESTRUCTURAS. SMBOLOS. CRONOLOGA. GLOSARIO

llamadas segundos.

H
Hectrea: Medida de supercie que equivale a 10000 metros cuadrados.

Hectgramo: Medida de peso equivalente a 100 gramos.

Hectlitro: Medida de capacidad equivalente a 100 litros.

Hectmetro: Medida de longitud equivalente a 100 metros.

Hemisferio: Cada una de las dos partes de una esfera, limitadas por un crculo mximo.

Heptaedro: Poliedro de siete caras.

Heptgono: Polgono de siete lados.

Heptgono regular: Polgono de siete lados iguales.

Hexa: Prejo que signica seis.

Hexaedro: Poliedro de 6 caras regulares, ms conocido como cubo.

Hexgono: Polgono de seis lados.

Hexgono regular: Polgono de seis lados iguales. Sus ngulos interiores son iguales y miden 120 cada
uno.

Hexagrama: Figura plana compuesta de dos tringulos equilteros que se cortan entre s, de modo que
cada lado de uno es paralelo a un lado del otro y forman un hexgono.

Hiprbola: Lugar geomtrico de los puntos del plano cuya diferencia de distancia a dos puntos jos,
llamados focos, es constante.

Hipotenusa: El mayor de los lados de un tringulo rectngulo y que s opuesto al ngulo recto.

Hiptesis: Enunciado o proposicin (suposicin) que se toma como base de un razonamiento matemtico.

Homogneo: Compuesto o formado por elementos de igual naturaleza.

Homlogos: Elementos homlogos son los que tienen la misma posicin en guras de igual forma.

I
Icosaedro: Poliedro de veinte caras.

Icosaedro regular: Poliedro de veinte caras iguales que son tringulos equilteros.

Identidad: Igualdad que se cumple para cualquier valor de la(s) variable(s) o indeterminadas que contiene.
Ejemplo: a2 b2 = (a b) (a + b), x + y = y + x.

Igualacin: Mtodo para resolver sistemas de ecuaciones.

Incentro: Punto en que se cortan las bisectrices interiores de un tringulo. Este punto es el centro de la
circunferencia inscrita al tringulo

Incgnita: Cantidad desconocida que es preciso determinar en una ecuacin.

Inecuacin: Desigualdad en la cual existe una o varias incgnitas. 2x2 x + 1 < 0, x 3y 0:


B.4. GLOSARIO DE TRMINOS MATEMTICOS ELEMENTALES 373

K
Kilo: Prejo que signica mil.

Kilgramo: Unidad de masa que equivale a mil gramos.

Kilolitro: Medida de capacidad equivalente a mil litros.

Kilmetro: Medida de longitud que equivale a mil metros.

Kilmetro cuadrado: Unidad de supercie equivalente a la de un cuadrado de lado 1 kilmetro.

L
Lneas (rectas) paralelas: Rectas con iguales pendientes: y = mx + b1 , y = mx + b2 . Lneas que por
traslacin guardan entre ellas una distancia constante y que no se juntan por mucho que se prolonguen.

Lineas (rectas) perpendiculares: Rectas cuyas pendientes se relacionan mediante la relacin m1 m2 =


1. Lneas ortogonales cuyas tangentes al cortarse forman un ngulo de 90 .

Lnea quebrada: Lnea formada por varios segmentos de recta colocados uno a continuacin de otro.

Lneas secantes: Lneas que se cortan en un punto. Linea recta que corta a la circunferencia en 2 puntos.

Logaritmo: El logaritmo de un nmero A, respecto de otro llamado base b, es el exponente C al que hay
que elevar la base b para obtener dicho nmero: logb A = C , bC = A, donde A > 0; b > 0, b 6= 1 es la base:

M
Magnitud escalar: Magnitud que viene determinada por un nmero real. Por ejemplo, 5 litros, 18 c.
P
xi
Media aritmtica: Cociente entre la suma de los trminos de una sucesin y el nmero de ellos. x =
n
Media armnica: Inversa de la media aritmtica de los inversos de los trminos de una sucesin. xH =
n
P 1
xi

Media geomtrica: Cada uno de los medios de una proporcin continua y que es igual a la raz cuadrada
p
del producto de los extremos. Valor xG = n x1 x2 xn con xi > 0.

Mediana (de un trapecio): Segmento que une los puntos medios de los lados no paralelos del trapecio.

Mediana (de un tringulo): Segmentos que unen los puntos medios de los lados de un tringulo con
sus vrtices opuestos.

Mediatriz: Recta perpendicular levantada en el punto medio de un segmento.

Mega: Prejo que signica un milln.

Megmetro: Medida de longitud que equivale a 1.000 kilmetros.

Mensurable: Que se puede medir.

Metra: Sujo que signica medir o medida.

Micra: Medida de longitud equivalente a la millonsima parte de un metro.

Micro: Prejo que signica la millonsima parte de la unidad principal.

Miembro de una ecuacin: Cada una de las 2 partes que separa el signo igual en la ecuacin. El
374 APNDICE B. ESTRUCTURAS. SMBOLOS. CRONOLOGA. GLOSARIO

1er miembro es lo que est a la izquierda del signo igual y el 2do a lo que est a su derecha. En la ecuacin
2x + 4 = 3x 2, el 1er miembro est constituido por 2x + 4 y el 2do es 3x 2:

Mili: Prejo que indica milsima parte.

Milgramo: Milsima parte de un gramo.

Milmetro: Milsima parte del metro.

Milla: Unidad de longitud equivalente a 1.609,347 metros.

Muestreo: Proceso estadstico que estudia las relaciones existentes entre una poblacin (universo) y sus
subpoblaciones (muestras) representativas, extradas aleatoriamente de la misma.

N
Nmeros complejos (forma algebraica): Un nmero complejo es un nmero que puede ser escrito en la
forma a + bi donde a y b son nmeros reales e i es la unidad imaginaria que cumple con la relacin i2 = 1.
Los nmeros complejos C forman el cuerpo de los complejos con sus respectivos axiomas, propiedades y cuya
representacin geomtrica corresponde a los puntos del plano R2 .

Nmeros enteros: Los nmeros enteros son los elementos del conjunto Z = f: : : 3; 2; 1; 0; 1; 2; 3; :::g.
Est compuesto de los enteros negativos Z = f 1; 2; 3; :::g, los enteros positivos Z+ = f1; 2; 3; :::g = N y
del 0.

Nmeros irracionales: Un numero irracional es aquel cuya representacin decimal innita posee cifras
p
que no forman un modelo repetitivo (no hay periodo) no representable como fraccin. Por ejemplo: 2 y son
nmeros irracionales. El conjunto de irracionales se representa con Q0 = fx : x 6= a=b con a; b 2 Z, b 6= 0g

Nmeros naturales: Los nmeros naturales son todos aquellos enteros positivos. Se les representa por
la letra N. Es decir: N = f1; 2; 3; :::g, N0 = f0; 1; 2; 3; :::g.

Nmeros racionales: Si m y n son enteros. Y m, n son diferentes de cero entonces m=n es un nmero
racional. El conjunto de racionales se representa con Q = fx : x = a=b con a; b 2 Z, b 6= 0g

Nmeros reales: El conjunto de los nmeros reales es la unin de los racionales y los irracionales,
R = Q [ Q0 y que estn en correspondencia 1 a 1 con todos los puntos de la recta continua. Los nmeros
reales forman un conjunto que cumple con todos los axiomas de cuerpo. Ejemplos de nmeros no reales son
p
los nmeros imaginarios puros (complejos del tipo z = ib, b 6= 0, i = 1), o aquellos que resultan de intentar
hallar la raz de ndice par o el logaritmo de un nmero real negativo.
p
Nmero imaginario: Nmero del tipo tipo z = ib, b 6= 0, i = 1 que resulta, por ejemplo, de extraer
la raz de ndice par de un nmero real negativo.

Nmero perfecto: Nmero entero y positivo igual a la suma de sus divisores positivos, excluido l mismo.
6 = 3 + 2 + 1:

O
Oblicungulo: Tringulo que no tiene ningn ngulo recto.

Obtusngulo: Tringulo que tiene un ngulo obtuso.

Octgono: Polgono de ocho lados.

Octante: Cada una de las ocho partes iguales en que se puede dividir el espacio R3 mediante un sistema
rectangular de coordenadas Oxyz.
B.4. GLOSARIO DE TRMINOS MATEMTICOS ELEMENTALES 375

Operacin binaria: Sea dado el conjunto A 6= ;. En A est denida una operacin binaria , si 8a; b 2 A,
se tiene que a b = c 2 A. Ejemplo: A = Q, = +, 8a; b 2 Q se tiene a b = a + b = c 2 Q.

Orden de una matriz: Indica el nmero de las y de columnas que tiene la matriz. Se expresa de la
forma m n donde m es el nmero de las y n el de columnas.

As una matriz de orden 3 2 es una matriz A3 2 de 3 las y 2 columnas. Si m = n la matriz de dice


cuadrada y es de orden m.

Ordenada: Segunda componente del par ordenado (x; y) que determinan un punto del plano R2 en un
sistema de coordenadas cartesianas o rectangulares.

Origen: Punto de interseccin de los ejes de un sistema de coordenadas cartesianas rectangular u oblicuo.

Ortocentro: Punto del tringulo donde se cortan sus alturas. Este punto es el centro de la circunferencia
circunscrita al tringulo.

Ortoedro: Paraleleppedo cuyas bases son rectngulos y sus aristas laterales perpendiculares a las bsicas.

Ortogonal: Lo que est en ngulo recto.

valo: Curva cerrada con dos ejes de simetra perpendiculares entre s y compuesta de varios arcos de
circunferencia tangentes entre s.

P
Parbola: Lugar geomtrico de todos los puntos del plano que equidistan, a la vez, de un punto dado y
2
de una recta dada. El punto dado es el foco y la recta dada, la directriz de la parbola. y k = 4p (x h)

Paradoja: Razonamiento que parece demostrar que es cierto algo que evidentemente es falso o parece
falso.

Paraleleppedo: Prisma cuyas bases son paralelgramos.

Paralelgramos: Cuadrilteros cuyos lados opuestos son paralelos. En ellos, los lados opuestos tienen la
misma longitud, los ngulos opuestos son iguales y las diagonales se cortan en su punto medio.

Pentadecgono: Polgono de 15 lados.

Pentadecgono regular: Polgono de 15 lados iguales. Cada ngulo interior mide 156 .

Pentgono: Polgono de 5 lados.

Pentgono regular: Polgono de 5 lados iguales. Cada ngulo interior mide 108 .

Permetro: Longitud de un polgono cerrado o longitud de una curva .

Permetro de un polgono: Corresponde a la suma de las longitudes de sus lados.

Perodo: Cifra o cifras que se repite(n) en una fraccin decimal peridica: 2;01212. Nmero real T tal que
para una funcin se cumple f (x) = f (x + T ) : sin x = sin (x + 2 ), T = 2 .

Perpendicular: Rectas que se cortan formando ngulos rectos.

Pi o : Nmero irracional que corresponde a la razn entre la longitud de la circunferencia y su dimetro.


' 3;1416

Pirmide: Cuerpo geomtrico que tiene como base un polgono cualquiera y como caras laterales tringulos
376 APNDICE B. ESTRUCTURAS. SMBOLOS. CRONOLOGA. GLOSARIO

con un vrtice comn.

Pirmide truncada: Porcin de pirmide comprendida entre la base y un plano paralelo a ella.

Planimetra: Parte de la matemtica que se ocupa del clculo de reas mediante planmetros.

Planmetro: Instrumento utilizado para medir reas de guras planas.

Planos coaxiales: Planos que tienen en comn una recta.

Planos paralelos: Planos A1 x + B1 y + C1 z = D1 y A2 x + B2 y + C2 z = D2 de R3 que no tienen ningn


punto en comn o que A1 =A2 = B1 =B2 = C1 =C2 6= D1 =D2 :

Planos perpendicularess: Planos A1 x + B1 y + C1 z = D1 y A2 x + B2 y + C2 z = D2 de R3 que cumplen


con A1 A2 + B1 B2 + C1 C2 = 0:

Planos secantes: Planos que se intersectan formando una recta.

Polidgitos: Nmeros constitudos por ms de una cifra.

Poliedro: Slido limitado por polgonos llamados caras.

Poliedro regular: Poliedro cuyas caras son polgonos regulares.

Polgono: Figura plana limitada por una linea poligonal cerrada.

Polgono circunscrito: Un polgono est circunscrito a una circunferencia cuando sus lados son tangentes
a la misma.

Polgono convexo: Polgono cuyos ngulos interiores son todos menores o iguales a 180 o tal que 2
puntos interiores cualesquiera pueden ser unidos mediante un segmento de recta totalmente contenido en el
polgono.

Polgono equiangular: Polgono que tiene todos sus ngulos interiores iguales.

Polgono equilateral: Polgono que tiene todos sus lados iguales.

Polgono inscrito: Un polgono est inscrito en una circunferencia cuando todos sus vrtices son puntos
de la circunferencia.

Polgono circunscrito: Todos los lados del poligono son tangentes a una circunferencia.

Polgono regular: Es el polgono que tiene de igual medida sus lados y congruentes sus ngulos.

Polgonos semejantes: Dos polgonos son semejantes si tienen ngulos iguales y sus lados correspondi-
entes proporcionales.

Polinmico: Forma desarrollada muy semejante a un polinomio.

Polinomio: Expresin algebraica de la forma Pn = a0 xn + a1 xn 1 + :::an 2 x2 + an 1 x + an , donde ai son


los coecientes numricos, n 2 N es el grado del polinomio, y x es la variable o indeterminada.

Porcentaje: Es una razn cuyo consecuente es 100. Ejemplo: 13 % = 13/100.

Postulado: Principio que se admite sin demostracin.

Potencia: Producto de un nmero, llamado base, por s mismo, n veces: an = a a a

Producto Cartesiano: Dados 2 conjuntos no vacos A y B, su producto cartesiano A B es el conjunto


B.4. GLOSARIO DE TRMINOS MATEMTICOS ELEMENTALES 377

de todas las parejas (x; y) llamadas pares ordenados que se forman tomando como 1er componente un elemento
de A y como 2do componente un elemento de B, en ese orden. El producto cartesiano se lee A por B o A cruz
B. Se generaliza a A B C, etc.

Progresin aritmtica: Sucesin de nmeros tal que la diferencia entre cada trmino y su precedente es
una diferencia constante; a esta diferencia d se la denomina diferencia de la progresin, tal como: 2; 5; 8; 11; 14;...;
d = 3:

Progresin geomtrica: Sucesin de nmeros tal que cada trmino se obtiene multiplicando su precedente
por un valor constante r, denominado razn de la progresin. Por ejemplo: 3; 6; 12; 24; 48;...; r = 2:

Proporcin: Es la igualdad de 2 razones a=b = c=d. Ejemplo: como 3 : 5 = 0; 6 y 6 : 10 = 0; 6 entonces


ambas razones son de igual valor con lo que se forma la proporcin 3 : 5 = 6 : 10. En una proporcin el producto
de los extremos es igual al producto de los medios: a=b = c=d , ad = bc.

Proporcin armnica: Conjunto de tres nmeros en los que el mayor forma con el menor, la misma
razn que la existente entre la diferencia del mayor y el del medio, y el medio y el menor. Por ejemplo: 3; 4 y 6.

Proposicin bicondicional: Proposicin en que una proposicin condicional y su conversa son ambas
verdaderas y se combinan en una sola proposicin. Tambin se denomina proposicin si y slo si: A , B:

Proposicin Condicional: Proposicin que puede expresarse como una proposicin Si-entonces: A ) B.
Por ejemplo, "Si un polgono es un hexgono, entonces tiene exactamente seis lados". La 1ra parte de la
condicional se denomina antecedente, la 2da parte se denomina consecuente.

Proporcin continua: Es la proporcin cuyos medios son iguales.

Proporcionalidad directa: Dos cantidades son directamente proporcionales si al multiplicar una, vara
tambin la otra en el mismo factor: y = x. Ejemplo, un dulce vale $2, entonces 9 dulces valen 9 2 = $18.

Proporcin discreta: Es la proporcin cuyos medios son distintos.

Proporcionalidad inversa: Dos cantidades son inversamente proporcionales si al multiplicar una, la


otra disminuye en el mismo factor: y = =x. Ejemplo: 4 trabajadores se demoran 20 das en hacer una obra, 8
trabajadores demoran en hacer la misma obra 10 das.

Proporciones iteradas: Son igualdades de dos o ms razones. Ejemplo, a : b : c = 2 : 3 : 5.

Proposicin Si-entonces: Proposicin que consta de 2 clusulas, una que empieza con la palabra si y
otra que comienza con la palabra entonces y plantea una conclusin; una proposicin condicional: A ) B:.

R
Racionalizar: Operacin que consiste en eliminar los radicales del denominador sin cambiar el valor
original de la fraccin.

Radin: Unidad de medida de ngulos que equivale a un ngulo central que subtiende un arco de longitud
igual al radio de esta circunferencia.

Radicacin: Operacin inversa a la potenciacin que consiste en encontrar la base de una potencia, dados
p
el resultado de ella y su exponente: k a = b , bk = a.

Radical: Simbolo que indica la operacin de extraer la raz k-sima de un nmero.

Radio: Segmento trazado de un punto de una circunferencia o esfera al centro. La longitud del segmento
tambin se denomina radio.
378 APNDICE B. ESTRUCTURAS. SMBOLOS. CRONOLOGA. GLOSARIO

Radio (de una circunferencia): Segmento que une el centro con un punto cualquiera de la circunferencia.

Radio (de una esfera): Segmento que une el centro de la esfera con un punto cualquiera de la supercie
esfrica.

Radio (de un polgono): Se llama radio de un polgono regular al radio de la circunferencia circunscrita
al polgono.
!
Radio vector: Segmento orientado que va del foco u origen a otro punto del plano o del espacio OM .

Raz (de una ecuacin): Solucin o valor que al sustituirlo en una ecuacin, sta se convierte en identidad
3
numrica. x = 1 es raz de la ecuacin 3x3 2x + 1 = 0 , pues 3 ( 1) 2 (x) 1 + 1 0.
p p
Raz cuadrada: Expresin radical de ndice dos. a= 2
a
p
Raz Cbica: Expresin radical de ndice tres 3
a.

Rango: En estadstica, es la diferencia entre el mayor y el menor de los datos ordenados. En funciones es
el conjunto de llegada o imagen total: f : A ! B, Im f = f (A) B. En matrices es el nmero mximo de las
o columnas linealmente independientes.

Razn: Comparacin o relacin entre 2 cantidades por cuociente. Ejemplo, si un nio tiene 5 aos y otro
3 aos, decimos que sus edades estn, respectivamente, en la razn 5 : 3.

Recproco: Corresponde al valor inverso de un nmero, de manera tal que al efectuar el producto entre
ambos, resulta 1.

Recta: Es la representacin grca de la funcin lineal. Funcin de la forma y = ax + b de R en R que


representa una linea recta.

Rectas convergentes: Rectas que tienen un punto en comn, que se cortan.

Rectas paralelas: Rectas, en un mismo plano, que no tienen puntos en comn por tener pendientes
iguales.

Rectngulo (tringulo): Tringulo que tiene un ngulo recto.

Rectngulo (cuadriltero): Paralelgramo con lados opuestos iguales y sus cuatro ngulos congruentes.

Rectngulo (trapecio): Trapecio que tiene un lado perpendicular a las bases.

Reduccin (mtodo de Gauss): Nombre dado a uno de los mtodos para resolver sistemas de ecuaciones
lineales.

Reexiva: Propiedad de las relaciones binarias que indica que todo elemento est relacionado consigo
mismo.

Regin: Parte del espacio.

Revolucin: Rotacin alrededor de un eje de cualquier gura.

Rombo: Paralelgramo de cuatro lados y dos pares de ngulos congruentes.

Romboide: Paralelgramo que tiene dos lados opuestos iguales y dos pares de ngulos opuestos congru-
entes.

Rotacin: Giro alrededor de un eje.


B.4. GLOSARIO DE TRMINOS MATEMTICOS ELEMENTALES 379

S
Secante: Recta que intercepta a la circunferencia en dos puntos no coincidentes. Toda secante determina
una cuerda. Se llama secante de dos o ms rectas a otra recta que las intersecta.

Seccin: Figura que resulta de la interseccin de una supercie con un slido.

Seccin cnica: Seccin que se origina al cortar con un plano un cono circular recto. Son secciones cnicas
la circunferencia, la elipse, la parbola y la hiprbola.

Sector circular: Regin limitada por 2 radios y el arco subtendido por ellos.

Sector esfrico: Porcin de volumen de esfera que est engendrada por un sector circular que gira
alrededor de un dimetro de la esfera. Est formada por un casquete y su cono.

Segmento: Porcin de recta limitada por dos puntos.

Segmento circular: Regin limitada por una cuerda y el arco determinado por ella.

Segundo: Unidad de tiempo que equivale a la 60 ava parte de un minuto.

Semejantes (guras): Figuras cuyos ngulos homlogos son congruentes y sus segmentos homlogos
proporcionales.

Semejantes (trminos): Trminos que tienen el mismo factor literal. Por ejemplo 5ab y 7ab.
1
X 1
X
Serie: Suma innita de una sucesin ordenada de trminos, nmeros o funciones : ak , fk (x)
k=0 k=0

Serie Aritmtica: Serie cuyos trminos forman una progresin aritmtica.


n
X n
X
Serie convergente: Serie cuya suma parcial tiene un lmite denido: l m ak = S, l m fk (x) =
n!1 n!1
k=0 k=0
S (x)

Serie divergente: Serie que no tiene un lmite denido: @S o S ! 1

Serie geomtrica: Serie cuyos trminos forman una progresin geomtrica.

Sexagesimal: Que tiene por base el nmero 60.

Sexagsimo: Cada una de las 60 partes iguales en que se puede dividir un todo.

geomtrico de los puntos que equidistan de los extremos de un trazo.

Simetra axial: Es la simetra con respecto a un eje o recta.

Simetra especular: Es la simetra respecto a un plano.

T
Tangente: Recta que intersecta a la circunferencia u otra curva en un solo punto, llamado punto de
sin x
tangencia. Funcin trigonomtrica denida como tan x = , x 6= k, k 2 Z:
cos x
Teorema: Es una proposicin que puede ser demostrada. La demostracin consta de un conjunto de
razonamientos que conducen a la evidencia de la verdad de la proposicin. En el enunciado de todo teorema se
distinguen dos partes: la hiptesis, que es lo que se supone, y la tesis, que es lo que se quiere demostrar.

Trmino independiente: En un polinomio o ecuacin, el nmero que no aparece multiplicando a las


380 APNDICE B. ESTRUCTURAS. SMBOLOS. CRONOLOGA. GLOSARIO

letras o incgnitas. Por ejemplo, en 7x3 + 2x 1, el trmino independiente es 1.

Trmino algebraico: Expresin que contiene nmeros y variables o indeterminadas (letras). Ejemplo:
3xy.

Trminos Semejantes: Son los que tienen la parte literal en forma idntica. Ejemplos: 5xy 3 , 7xy 3 .

Tercera proporcional: Corresponde al cuarto trmino de una proporcin continua.

Transversal de gravedad: Segmentos que unen el vrtice con el punto medio del lado opuesto en un
tringulo.

Trapecios: Cuadrilteros con un par de lados paralelos.

Trapezoides: Cuadrilteros sin lados paralelos.

Tringulos semejantes: Dos tringulos son semejantes si tienen sus ngulos iguales o sus lados propor-
cionales.

Trinomio: Expresin algebraica de tres trminos o monomios. Ejemplos: 3x + 2y 5z, 2x2 + 5x 1.

V
Valor Absoluto: Siendo x un nmero real cualquiera, su valor absoluto jxj es el nmero real que verica
las siguientes condiciones: jxj = x, si x 0 y jxj = x, si x < 0. De esta forma: j6j = 6 por ser 6 > 0, mientras
que j 4j = ( 4) = 4 por ser 4 < 0.

Valor Relativo: Valor que depende de la posicin que dicha cifra ocupa en el nmero.
Bibliografa

[1] Galindo, Edwin; Gortaire J., Danilo. Matemticas Superiores. Teora y Ejercicios. Prociencia Editores,
Impresora Oset Internacional. Quito, 2003.

[2] Potapov, M.; Alexandrov, V.; Pasichenko, P. lgebra y anlisis de funciones elementales. Editorial Mir.
Mosc, 1986.

[3] Lidski V. y otros. Problemas de Matemticas Elementales. Editorial Mir. Mosc, 1972.

[4] Zorich V. A. Anlisis matemtico T. I. Editorial Nauka. Mosc, 1981 (en ruso).

[5] Guetmnova, A.; Panov, M.; Petrov, V. Lgica: en forma simple sobre lo complejo. Diccionario. Editorial
Progreso. Mosc, 1991.

[6] Kurosh G. lgebra Superior. Editorial Mir. Mosc, 1976.

[7] Potapov, M.; Rozov, M. Temas selectos de matemticas elementales. Editorial Mir. Mosc, 1974.

[8] Hall, H. S., Knight, S. R. lgebra Superior. UTEHA. Mxico, 1964.

[9] Alfonseca Manuel. Diccionario Espasa. 1000 Grandes Cientcos. Editorial Espasa Calpe. Madrid, 1996.

[10] Boyer Carl. Historia de la Matemtica. Alianza Editorial. Madrid, 1986.

[11] Apostol Tom. Calculus Vol. 1. Editorial Revert. Barcelona, 1982.

[12] Chambadal Lucien. Diccionario de Matemticas. Ediciones Grijalbo. Barcelona, 1984.

[13] de Burgos Juan. Clculo Innitesimal. Teora y problemas. Editorial Alhambra. Madrid, 1984.

[14] Lipschutz Seymur. Teora de Conjuntos y Temas Anes. Ed. McGraw-Hill. Bogot, 1982.

[15] Baillif J. C. Los rompecabezas lgicos de Baillif. Editorial Revert. Barcelona, 1985.

381

You might also like